You are on page 1of 744

VCE mATHEmATiCS

UNiTS 3 & 4

mATHS QUEST 12
mathematical
methods CAS

Ti-NSPiRE 2.0 EDiTiON

VCE mATHEmATiCS
UNiTS 3 & 4

mATHS QUEST 12
mathematical
methods CAS
Ti-NSPiRE 2.0 EDiTiON

BRiAN HODGSON NiCOLAOS KARANiKOLAS


BEVERLY LANGSFORD-WiLLiNG mARK DUNCAN TRACY HERFT
LiBBY KEmPTON JENNiFER NOLAN GEOFF PHiLLiPS

CONTRiBUTiNG AUTHORS
RUTH BAKOGiANiS | ANDREW mENTLiKOWSKi | mARK BARNES | KYLiE BOUCHER
JENNY WATSON | CAROLiNE DENNEY | SONJA STAmBULiC | ELENA iAmPOLSKY
ROSS ALLEN | ROBERT CAHN | RODNEY EBBAGE

First published and revised 2010 by


John Wiley & Sons Australia, Ltd
42 McDougall Street, Milton, Qld 4064
Typeset in 10/12pt Times LT
John Wiley & Sons Australia, Ltd 2010
The moral rights of the authors have been asserted.
National Library of Australia
Cataloguing-in-Publication data
Title:

Maths quest 12 mathematical methods CAS:


TI-Nspire 2.0 edition / Brian Hodgson ... [et al.].
ISBN:
978 1 7424 6465 7 (student pbk.)
978 1 7424 6466 4 (student ebook)
978 1 7424 6468 8 (teacher pbk.)
978 1 7424 6469 5 (teacher ebook)
Notes:
Includes index.
Target Audience: For secondary school age.
Subjects:
Mathematics Textbooks.
Other Authors/
Contributors:
Hodgson, Brian.
Dewey Number: 510

Reproduction and communication for educational purposes


The Australian Copyright Act 1968 (the Act) allows a maximum
of one chapter or 10% of the pages of this work, whichever is the
greater, to be reproduced and/or communicated by any educational
institution for its educational purposes provided that the educational
institution (or the body that administers it) has given a remuneration
notice to Copyright Agency Limited (CAL).
Reproduction and communication for other purposes
Except as permitted under the Act (for example, a fair dealing
for the purposes of study, research, criticism or review), no part
of this book may be reproduced, stored in a retrieval system,
communicated or transmitted in any form or by any means without
prior written permission. All inquiries should be made to the
publisher.
Screenshots from TI-Nspire CAS calculator reproduced with
permission of Texas Instruments
Illustrated by Aptara and the Wiley Art Studio
Typeset in India by Aptara
Printed in Singapore by
Craft Print International Ltd
10 9 8 7 6 5 4 3 2 1

Contents
Introduction viii
About eBookPLUS x
Acknowledgements xi
CHAPTER 1

Graphs and polynomials 1


1a The binomial theorem 1
1B
1C
1D
1E
1F
1G

Exercise 1A 6
Polynomials 6
Exercise 1B 10
Division of polynomials 11
Exercise 1C 15
Linear graphs 15
Exercise 1D 20
Quadratic graphs 22
Exercise 1E 28
Cubic graphs 29
Exercise 1F 34
Quartic graphs 38
Exercise 1G 45

Summary 47
Chapter review 50
eBookPLUS activities 56

CHAPTER 2

Functions and transformations 57


2a Transformations and the parabola 57

Exercise 2A 63
2B The cubic function in power form 65

Exercise 2B 69
2c The power function (the hyperbola) 71
2D
2E
2F
2G
2h
2I

Exercise 2C 76
The power function (the truncus) 78
Exercise 2D 84
The square root function in power form 86
Exercise 2E 91
The absolute value function 92
Exercise 2F 95
Transformations with matrices 96
Exercise 2G 101
Sum, difference and product functions 103
Exercise 2H 106
Composite functions and functional
equations 108
Exercise 2I 110

2J Modelling 111

Exercise 2J 117
Summary 120
Chapter review 123
eBookPLUS activities 129

Exam Practice 1
Based on Chapters 12 130

CHAPTER 3

Exponential and logarithmic


equations 132
3a The index laws 132

Exercise 3A 136
3B Logarithm laws 138

Exercise 3B 141
3C Exponential equations 142

Exercise 3C 146
3D Logarithmic equations using any base 147

Exercise 3D 149
3E Exponential equations (base e) 151

Exercise 3E 153
3F Equations with natural (base e)

logarithms 154
Exercise 3F 155
3G Inverses 156
Exercise 3G 158
3H Literal equations 159
Exercise 3H 162
3I Exponential and logarithmic modelling 162
Exercise 3I 164
Summary 167
Chapter review 168
eBookPLUS activities 171

CHAPTER 4

Exponential and logarithmic graphs 172


4a Graphs of exponential functions

with any base 172


Exercise 4A 181
4B Logarithmic graphs to any base 182
Exercise 4B 190
4c Graphs of exponential functions
with base e 191
Exercise 4C 196

4d Logarithmic graphs to base e 197


4E

4F
4G

4H

Exercise 4D 201
Finding equations for graphs of exponential
and logarithmic functions 202
Exercise 4E 205
Addition of ordinates 206
Exercise 4F 211
Exponential and logarithmic functions
with absolute values 212
Exercise 4G 215
Exponential and logarithmic modelling
using graphs 216
Exercise 4H 217

Summary 220
Chapter review 222
eBookPLUS activities 228

6i Trigonometric functions with

an increasing trend 311


Exercise 6I 313

Summary 314
Chapter review 317
eBookPLUS activities 321

Exam Practice 2
Based on Chapters 16 322

CHAPTER 7

Differentiation 324
7a Review gradient and rates of change 324

Exercise 7A 328
7B Limits and differentiation from

CHAPTER 5

Inverse functions 229


5a Relations and their inverses 229

Exercise 5A 233
5b Functions and their inverses 234
Exercise 5B 239
5c Inverse functions 240
Exercise 5C 243
5d Restricting functions 245
Exercise 5D 249
Summary 252
Chapter review 253
eBookPLUS activities 257

CHAPTER 6

Circular (trigonometric) functions 258


6A Revision of radians and the unit circle 258
6B
6c
6D
6e
6f

6g
6h

vi

Exercise 6A 263
Symmetry and exact values 264
Exercise 6B 271
Trigonometric equations 273
Exercise 6C 281
Trigonometric graphs 282
Exercise 6D 288
Graphs of the tangent function 291
Exercise 6E 295
Finding equations of trigonometric
graphs 295
Exercise 6F 297
Trigonometric modelling 299
Exercise 6G 301
Further graphs 303
Exercise 6H 310

Contents

7C
7d
7E
7F
7g

7h
7i
7J

first principles 332


Exercise 7B 336
The derivative of xn 338
Exercise 7C 340
The chain rule 341
Exercise 7D 343
The derivative of ex 344
Exercise 7E 347
The derivative of loge (x) 347
Exercise 7F 349
The derivatives of sin (x), cos (x)
and tan (x) 351
Exercise 7G 354
The product rule 355
Exercise 7H 356
The quotient rule 357
Exercise 7I 359
Mixed problems on differentiation 360
Exercise 7J 364

Summary 366
Chapter review 368
eBookPLUS activities 372

CHAPTER 8

Applications of differentiation 373


8a Equations of tangents and normals 373

Exercise 8A 375
8b Sketching curves 376
Exercise 8B 384
8c Maximum and minimum problems
when the function is known 386
Exercise 8C 389
8d Maximum and minimum problems
when the function is unknown 390
Exercise 8D 394

8e Rates of change 396

10D Measures of variability of discrete random

Exercise 8E 398
8f Related rates 401
Exercise 8F 404
8G Linear approximation 404
Exercise 8G 406

Summary 529
Chapter review 531
eBookPLUS activities 536

Summary 408
Chapter review 410
eBookPLUS activities 415

CHAPTER 9

Integration 416
9a Antidifferentiation 416

Exercise 9A 423
9b Integration of ex, sin (x) and cos (x) 426

Exercise 9B 428
9c Integration by recognition 430

Exercise 9C 433
9d Approximating areas enclosed by

9e

9f
9g
9H
9I
9J

functions 435
Exercise 9D 438
The fundamental theorem of integral
calculus 441
Exercise 9E 446
Signed areas 448
Exercise 9F 452
Further areas 455
Exercise 9G 458
Areas between two curves 460
Exercise 9H 464
Average value of a function 466
Exercise 9I 468
Further applications of integration 468
Exercise 9J 471

Summary 474
Chapter review 476
eBookPLUS activities 481

Exam Practice 3
Based on Chapters 19 482

CHAPTER 10

Discrete random variables 484


10a Probability revision 484

Exercise 10A 498


10b Discrete random variables 501
Exercise 10B 507
10c Measures of centre of discrete random
distributions 510
Exercise 10C 516

distributions 518
Exercise 10D 526

CHAPTER 11

The binomial distribution 537


11a The binomial distribution 537

Exercise 11A 548


11b Problems involving the binomial distribution

for multiple probabilities 552


Exercise 11B 557
11c Markov chains and transition matrices 559
Exercise 11C 571
11D Expected value, variance and standard
deviation of the binomial distribution 573
Exercise 11D 577
Summary 581
Chapter review 583
eBookPLUS activities 588

CHAPTER 12

Continuous distributions 589


12a Continuous random variables 589

Exercise 12A 594


12b Using a probability density function to

12C

12D
12e
12f
12G

find probabilities of continuous random


variables 595
Exercise 12B 602
Measures of central tendency and
spread 605
Exercise 12C 611
Applications to problem solving 613
Exercise 12D 616
The normal distribution 619
Exercise 12E 623
The standard normal distribution 626
Exercise 12F 633
The inverse cumulative normal
distribution 636
Exercise 12G 640

Summary 643
Chapter review 646
eBookPLUS activities 651

Exam Practice 4
Based on Chapters 112 652

Answers 654
Index 730

Contents

vii

Introduction
Maths Quest 12 Mathematical Methods CAS is specifically designed for the VCE Mathematical
Methods CAS course and based on the award-winning Maths Quest series. This resource
contains:
a student textbook with accompanying eBookPLUS
a teacher edition with accompanying eGuidePLUS
a solutions manual.

Student textbook
Full colour is used throughout to produce clearer graphs and headings, to provide bright,
stimulating photos and to make navigation through the text easier.
Clear, concise theory sections contain worked examples and highlighted important text and
remember boxes.
Icons appear for the eBookPLUS to indicate that interactivities and eLessons are available
online to help with the teaching and learning of particular concepts.
Worked examples in a Think/Write format provide clear explanation of key steps and suggest
presentation of solutions. Many worked examples have eBookPLUS icons to indicate that a
Tutorial is available to elucidate the concepts being explained. Worked examples also contain
CAS calculator instructions and screens to exemplify judicious use of the calculator.
Exercises contain many carefully graded skills and application problems, including multiplechoice questions. Cross references to relevant worked examples appear with the first matching
question throughout the exercises. Exercises also contain questions from past VCE examination
papers along with relevant exam tips.
A selection of questions is tagged as technology-free to indicate to students that they should
avoid using their calculators or other technologies to assist them in finding a solution.
Exam practice sections contain examination-style questions, including time and mark allocations
for each question. Fully worked solutions are available on the eBookPLUS for students.
Each chapter concludes with a summary and chapter review exercise containing examinationstyle questions (multiple choice, short answer and extended response), which help consolidate
students learning of new concepts. Also included are questions from past VCE exams along
with relevant exam tips.
Technology is fully integrated (in line with VCE recommendations).

Student website eBookPLUS


The accompanying eBookPLUS contains the entire student textbook in HTML plus additional
exercises. Students may use the eBookPLUS on laptops, school or home computers, and cut and
paste material for revision or the creation of notes for exams.
Career profiles and History of mathematics place mathematical concepts in context.
Investigations, often suggesting the use of technology, provide further discovery learning
opportunities.

viii

Introduction

WorkSHEET icons link to editable Word documents that may be completed on screen or printed
and completed by hand.
SkillSHEET icons link to printable pages designed to help students revise required concepts, and
contain additional examples and problems.
Interactivity icons link to dynamic animations which help students to understand difficult
concepts.
eLesson icons link to videos or animations designed to elucidate concepts in ways that are more
than what the teacher can achieve in the classroom.
Tutorial icons link to one-way engagement activities which explain the worked examples in
detail for students to view at home or in the classroom.
Test yourself tests are also available and answers are provided for students to receive instant
feedback.

Teacher website eGuidePLUS


The accompanying eGuidePLUS contains everything in the eBookPLUS and more. Two tests
per chapter, fully worked solutions to WorkSHEETs, the work program and other curriculum
advice in editable Word format are provided. Maths Quest is a rich collection of teaching and
learning resources within one package.
Maths Quest 12 Mathematical Methods CAS provides ample material, such as exercises,
analysis questions, investigations, worksheets and technology files, from which teachers may set
school assessed coursework (SAC).

Introduction

ix

Next generation teaching and learning


About eBookPLUS
This book features eBookPLUS: an electronic
version of the entire textbook and supporting
multimedia resources. It is available for you
online at the JacarandaPLUS website
(www.jacplus.com.au).
LOGIN
Once you have created your
account, you can use the same
email address and password
in the future to register any
eBookPLUS titles you own.

Using the JacarandaPLUS website


To access your eBookPLUS resources, simply
log on to www.jacplus.com.au. There are three
easy steps for using the JacarandaPLUS system.
Step 1. Create a user account
The first time you use the JacarandaPLUS
system, you will need to create a user account.
Go to the JacarandaPLUS home page
(www.jacplus.com.au) and follow the
instructions on screen. An activation email
will be sent to your nominated email address.
Click on the link in this email and your
activation will be complete. You can now use
your nominated email address and password
to log in to the JacarandaPLUS system.
Step 2. Enter your registration code
Once you have activated your account and
logged in, enter your unique registration code
for this book, which is printed on the inside
front cover of your textbook. The title of your
textbook will appear in your bookshelf. Click
on the link to open your eBookPLUS.
Step 3. View or download eBookPLUS resources
Your eBook and supporting resources are
provided in a chapter-by-chapter format.
Simply select the desired chapter from
the drop-down list. The student eBook tab
contains the entire chapters content in
easy-to-use HTML. The student resources tab
contains supporting multimedia resources
for each chapter.

About eBookPLUS

Using eBookPLUS references


eBook plus

eBookPLUS logos are used throughout


the printed books to inform you that a
multimedia resource is available for the
content you are studying.

Searchlight IDs (e.g. int-0001) give you instant


access to multimedia resources. Once you are
logged in, simply enter the searchlight ID for
that resource and it will open immediately.

Minimum requirements
Internet Explorer 7, Mozilla Firefox 1.5
or Safari 1.3
Adobe Flash Player 9
Javascript must be enabled (most browsers
are enabled by default).

Troubleshooting
Go to the JacarandaPLUS help page at
www.jacplus.com.au
Contact John Wiley & Sons Australia, Ltd.
Email: support@jacplus.com.au
Phone: 1800 JAC PLUS (1800 522 7587)

Acknowledgements
The authors and publisher would like to thank the following copyright holders, organisations
and individuals for their assistance and for permission to reproduce copyright material in this
book.

Images:
Banana Stock: page 494 Corbis Corporation: pages 317, 585 Digital Vision:
pages 53, 219, 577 Image 100: page 127 ImageState: page 387 John Wiley & Sons
Australia: page 500/Taken by Sharon Way, page 518/Taken by Kari-Ann Tapp Photodisc:
pages 154, 156, 218, 222, 273, 299, 302, 472, 559, 561, 572, 578, 580, 649 Photolibrary/
Claver Carroll: page 479 PureStock: page 26 Stockbyte: pages 549, 550 Photograph
courtesy and copyright Transurban Limited, page 527

Text:
VCAA: All Victorian Curriculum and Assessment Authority material has been used with
permission. For more information go to www.vcaa.vic.edu.au.
Every effort has been made to trace the ownership of copyright material. Information that will
enable the publisher to rectify any error or omission in subsequent editions will be welcome. In
such cases, please contact the Permissions Section of John Wiley & Sons Australia, Ltd.

Acknowledgements

xi

1A
1B
1C
1D
1E
1F
1G

The binomial theorem


Polynomials
Division of polynomials
Linear graphs
Quadratic graphs
Cubic graphs
Quartic graphs

Graphs and
polynomials
AreAS oF STudy

Review of algebra of polynomials, solution of


polynomial equations with real coefficients
of degree n having up to and including n real
solutions

Graphs of polynomial functions including


axis intercepts, stationary points and points of
inflection, domain (including maximal domain),
range and symmetry

eBook plus

1A

Digital doc

The binomial theorem

10 Quick Questions

In Maths Quest 11 Mathematical Methods CAS we learned the following binomial expansions:
(x + a)2 = x2 + 2xa + a2
(x + a)3 = x3 + 3x2a + 3xa2 + a3
These are called binomial expansions because the expressions in the brackets contain two
terms, bi meaning 2.
By continuing to multiply successively by a further (x + a), the following expansions would
be obtained:
(x + a)4 = (x3 + 3x2a + 3xa2 + a3)(x + a)
= x4 + 4x3a + 6x2a2 + 4xa3 + a3
(x + a)5 = (x4 + 4x3a + 6x2a2 + 4xa3 + a3)(x + a)
= x5 + 5x4a + 10x3a2 + 10x2a3 + 5xa4 + a5
The coefficients associated with each term can be arranged in a triangular shape as shown:
(x + a)0
(x +

a)1

(x + a)2

(x + a)3

(x + a)4
(x + a)5

1
1

2
3

4
5

1
1
3
6

10

1
4

10

1
5

Chapter 1

Graphs and polynomials

Notes
1. The first and last numbers of each row are 1.
2. Each other number is the sum of the two numbers immediately above it.
This triangle is known as Pascals triangle. Each number can also be obtained using
combinations, as follows.
Row
0
0

0
1
0

1
2
0

2
3
0

3
4
0

1
1
2
1

3
1
4
1

2
2
3
2

4
2

3
3
4
3

4
4

n n
n!
Note: r = Cr = n r ! r !

( )
n
Remember that nCr is another way of writing .
r
For example, the expansion of (x + a)6 can be written using combinations and
then evaluated:
6
6
6
6
6
6
6
( x + a)6 = x 6 a 0 + x 5 a1 + x 4 a 2 + x 3 a3 + x 2 a 4 + x1a 5 + x 0 a6
5
2
4
1
3
0
6
= x 6 + 6 x 5 a + 15 x 4 a 2 + 20 x 3 a3 + 15 x 2 a 4 + 6 xa 5 + a6
Now the binomial theorem can be formally stated.
n
n
n
n
( ax + b) n = ( ax ) n b0 + ( ax ) n 1 b1 + . . . +
( ax )1 bn 1 + ( ax )0 bn
n
1
n 1
0
Notes
1. The indices always sum to n, that is, the powers of (ax) and b sum to n.
2. The power of ax decreases from left to right while the power of b increases.
3. The number of terms in the expansion is always n + 1.
n
(ax)n r br.
4. The (r + 1)th term is
r
The binomial theorem can also be stated using summation notation:
( ax + b) n =

r=0

r ( ax)n r br

, where

means the sum of terms from r = 0 to r = n.

r= 0

Maths Quest 12 Mathematical Methods CAS for the TI-Nspire

Worked exAMple 1

Use the binomial theorem to expand ( 2 x 3)4.


ThINk

WrITe

Method 1: Technology-free
1

Complete the binomial theorem expansion


where ax is the 1st term, b is the 2nd term and
n is the index, using the appropriate row of
Pascals triangle to assist.

4
4
(2 x 3)4 = (2 x )4 ( 3)0 + (2 x )3 ( 3)1
1
0
4
4
+ (2 x )2 ( 3)2 + (2 x )1 ( 3)3
2

3
4
+ (2 x )0 ( 3)4
4

Evaluate the combinations and the powers.

Simplify.

= 1(16x4) + 4(8x3)(3) + 6(4x2)(9)


+ 4(2x)(27) + 1(81)
= 16x4 96x3 + 216x2 216x + 81

Method 2: Technology-enabled
1
On a Calculator page, press:
MENu b
3:Algebra3
2:Expand2
Complete the entry line as:
expand((2x 3)4)
Then press ENTER .

(2x 3)4 = 16x4 96x3 + 216x2 216x + 81

Write the result.

Worked exAMple 2
5

Expand the binomial expression


+ x .
2
x

ThINk
1

Complete the binomial expansion where


2
ax = 2 , b = x and n = 5, using row 5 of Pascals
x
triangle to assist.

Evaluate the powers.

Simplify.

WrITe
5

2
2
2
2 2

x 2 + x = x 2 + 5 x 2 x + 10 x 2 x
2
2 3
2
+ 10 2 x + 5 2 x 4 + x 5
x
x
32
16
8
+ 5 8 x + 10 6 x 2
x
x
x10
4
2
+ 10 4 x 3 + 5 2 x 4 + x 5
x
x
32 80 80 40
= 10 + 7 + 4 +
+ 10 x 2 + x 5
x
x
x
x
=

Chapter 1

Graphs and polynomials

Worked exAMple 3

State the coefficient of i

eBook plus

x2

and

ii

x4

in

( 3 2 x )8

, without the use of technology.

Tutorial

int-0516

ThINk
i

ii

WrITe

Find the appropriate term by using the


binomial theorem.

Evaluate the term.

State the coefficient.

Find which term gives a power of x4.

i x0, x1, x2

The powers of the 1st term decrease and the


powers of the 2nd term increase 0, 1, 2, . . .
use this to find which term gives a power
of x2.

Worked example 3

The third term gives a power of x2.

Third term =

8 6 2
3 ( 2x)
2

= 28 729 4x2
= 81 648x2
The coefficient of x2 is 81 648.
i i x0, x1, x2, x3, x4

The fifth term gives a power of x4.


8 4 4
3 ( 2x)
Fifth term =
4

Evaluate the term.

= 70 81 16x4
= 90 720x4
3

State the coefficient.

The coefficient of the fifth term is 90 720.

Worked exAMple 4

Find the fourth term in the expansion of ( x 2 y)5.


ThINk
1

Find the fourth term by using the binomial


theorem.

Evaluate the term.

WrITe

5
Fourth term = x2(2y)3
3
= 10 x2 8y3
= 80x2y3

Worked exAMple 5
5

Find and evaluate the term that is independent of x in the expansion of x 3 + .

x2
ThINk
1

Find how the powers of x are generated


in the expansion from left to right.

WrITe

1
Powers of x are (x3)5 = x15, (x3)4 2 = x10,
x
2
3
1
1

(x3)3 = x5, (x3)2 = x0 . . .


x2
x2
that is, x15, x10, x5, x0.

Maths Quest 12 Mathematical Methods CAS for the TI-Nspire

Find the required term.

The fourth term is independent of x.

Evaluate.

Fourth term =

5 3 2 1
3 ( x ) x 2

x 6

= 10 x 6
= 10
4

State the solution.

The term that is independent of x is the fourth


term, 10.

Worked Example 6

Find the coefficient of y4 in the expansion of ( y + 3) 3 ( 2 y)5.


Think

Write

y4 terms will result when multiplying from the


first and second brackets respectively: terms
1 and 2, terms 2 and 3, terms 3 and 4 and terms
4 and 5.

Write down the sum of these 4 products, using


Pascals triangle to assist.

y4 terms = y3[5(2)4(y)] + 3y2(3)[10(2)3(y)2]


+ 3y(3)2[10(2)2(y)3] + 33[5(2)(y)4]
= 80y4 + 720y4 1080y4 + 270y4
= 170y4

Evaluate.

State the solution.

The coefficient of y4 is 170.

REMEMBER

1. Pascals triangle:
1
1
1
1
1

1
2

3
4

1
5
2. Binomial theorem:

1
3

6
10

1
4

10

1
5

n
n
n
n
(ax + b) n = (ax ) n b 0 + (ax ) n 1 b + .. . +
(ax )b n 1 + (ax )0 b n

1
n
0
n 1
Notes
1. The powers of (ax) and b sum to n.
2. There are n + 1 terms in the expansion.
n
n r br .
3. The (r + 1)th term is
r (ax )

Chapter 1 Graphs and polynomials

exerCISe

1A
eBook plus

The binomial theorem


1 We 1

a (x + 3)2
d (2x + 3)4

Digital doc

SkillSHEET 1.1
Binomial
expansions

use the binomial theorem to expand each of the following.


b (x + 4)5
e (7 x)4

c (x 1)8
f (2 3x)5

2 We2 Expand each of the following binomial expansions.


3

a x + 1

b 3x 2

c x2 + 3

d 3 2 x
x 2

State the coefficient of i x2 ii x3 and iii x4 in each of the following.

3 We3

c 2 + 3x
x

b (2x + 1)5

a (x 7)3

d x2 3

e 7x + 3

x 2

4 MC The coefficient of x3 in 3 x 2 is:

x
135

B 45
C 75
A
5 MC Which of the following does not have an
B 3x 2 1

A (x + 6)8

D 45
x5

E 135

term when expanded?

C 6x + 5

D (8 3x)5

E 2x 1

x 2

e
f
2

6 MC If x 3 + 2 = ax15 + bx10 + cx 5 + d + 5 + 10 , then a + b + c + d + e + f equals:

x
x
x
A 15
B 31
C 63
D 243
E 127
7 MC Which one of the following expressions is not equal to (2x 3)4?

( 2 x 3)6
( 3 2 x )2

A (3 2x)

B (2x 3)(2x

D 16x4 24x3 + 36x2 54x + 81

E 16x4 96x3 + 216x2 216x + 81

3)3

8 We4 Find the fourth term in the expansion (x + 3y)6.


9
10
11
12
13

Find the third term in the expansion of 3

x
, assuming ascending powers of x.
4
6
2

We5
Find and evaluate the term that is independent of x in the expansion of 3 x + 2 .

x
5
2 4
Find and evaluate the term independent of x in the expansion of x 3 .

x
4
3

Find and evaluate the term that is independent of x in the expansion of x 2 + 2 .

x
We6 Find the coefficient of p4 in the expansion of (p + 3)5 (2p 5).

14 In the expansion of (2a 1)n, the coefficient of the second term is 192. Find the value of n.

1B

polynomials
A polynomial in x is an expression that consists of terms which have non-negative integer
powers of x only.
P(x) is a polynomial in x if:
P(x) = an xn + an 1 xn 1 + . . . + a2 x2 + a1 x + a0

Maths Quest 12 Mathematical Methods CAS for the TI-Nspire

where n is the degree (or highest power) of the polynomial and is a non-negative integer. The
values of an, an 1, ..., a2, a1 and a0 are called the coefficients of their respective power of
x terms.

Worked Example 7

Which of the following expressions are not polynomials?

a x6 4 x4 + 2 x3 + 7 x
c 7 3 xy + 4 x 2 x 3 +

e 3 x2

b x 2 + x3 x2 + 6 x 5
d 8 + 2 x 3 x2 + 9 x3 x4

2
x 2

Think
1

Write

a and d are polynomials because they are

expressions with non-negative integer powers


of x only.
2

b is not a polynomial as it has a power of 92 , which

is not an integer.
3

c is not a polynomial as it has a power of


1
2

( ), which is not an integer, and it also

b, c and e are not polynomials.

has one term, 3xy, which is not a power


of x only.
4

2
= 2 x 2 and so
x2
has a power that is not a positive integer.
e is not a polynomial because

Polynomials can be added and subtracted by collecting like terms.

Worked Example 8

Given that P( x ) = 6 2 x + 3 x 2 + x 4 , Q( x ) = x 5 2 x 4 + x 2 5 x 2 and R( x ) = x 2 4, find:


a P ( x ) + Q( x )
b P ( x ) R( x ).
Think

Write/display

Method 1: Technology-free
a

a P(x) + Q(x) = 6 2x + 3x2 + x4 + x5 2x4

Add the polynomials.

Collect like terms.

Subtract the polynomials.

Remove brackets.

= 6 2x + 3x2 + x4 x2 + 4

Collect like terms.

= x4 + 2x2 2x + 10

+ x2 5x 2

= x5 x4 + 4x2 7x + 4
b P(x) R(x) = 6 2x + 3x2 + x4 (x2 4)

Chapter 1 Graphs and polynomials

Method 2: Technology-enabled
a

On a Calculator page, define the


polynomials P(x), Q(x) and R(x).
To do this, press:
Menu b
1:Actions 1
1:Define 1
Complete the entry lines as:
Define p(x) = 6 - 2x + 3x2 + x4
Define q(x) = x5 - 2x4 + x2 - 5x - 2
Define r(x) = x2 - 4
Press ENTER after each entry.

To calculate P(x) + Q(x), complete


the entry line as:
p(x) + q(x)
Press ENTER

To calculate P(x) - R(x), complete the


entry line as:
p(x) - r(x)
Press ENTER .

Evaluating polynomials
A value for a polynomial, P(x), can be found for a particular value of x by simply substituting
the given value of x into the polynomial expression and evaluating. That is, polynomial functions
are evaluated in the same way as any function.

Maths Quest 12 Mathematical Methods CAS for the TI-Nspire

Worked Example 9

For the polynomial P( x ) = 2 x 4 x 3 + 5 x 2 6 x + 4, find:


a its degree
b P(1)
c P(2).
Think

Write

The degree of the polynomial is the highest


power of x.
b 1 Substitute the given value of x into
the polynomial expression.
2

a The degree of P(x) is 4.


b P(1) = 2(1)4 (1)3 + 5(1)2 6(1) + 4

=21+56+4=4

Evaluate.

Substitute the given value of x into


the polynomial expression.

Evaluate.

P(2)

= 2(2)4 (2)3 + 5(2)2 6(2) + 4


= 32 + 8 + 20 + 12 + 4 = 76

Worked Example 10

If p( x ) = ax 5 + x 4 3 x 3 + bx 5, p( 1) = 5 and p(2) = 65 , find the values of a and b.


Think

Write/display

Method 1: Technology-free
1

Substitute a given value of x into the


polynomial and equate it to the given answer.

Simplify the equation.

Make b the subject of the equation and call


this equation [1].

Substitute a given value of x into the


polynomial and equate it to the given answer.

Simplify the equation.

P(1) = a(1)5 + (1)4 3(1)3 + b(1) 5


= 5
a

+ 1 + 3 b 5 = 5
a + 4 b = 0
b = 4 a

P(2) = a(2)5 + (2)4 3(2)3 + b(2) 5


= 65
32a + 16 24 + 2b 5 = 65
32a + 2b 13 = 65
32a + 2b = 52

Substitute [1] into [2].

Substituting b = 4 a:

32a + 2(4 a) = 52

Solve this equation for a.

Substitute the value of a into equation [1].

Substituting a = 2 into equation [1]:



b = 4 2

Find the value of b.

10

State the solution.

[1]

[2]

32a + 8 2a = 52
30a = 60
a = 2

=6
Therefore, a =

and b = 6.

Chapter 1 Graphs and polynomials

Method 2: Technology-enabled
1
On a Calculator page, define the polynomial
P(x) by completing the entry line as:
Definep(x) = a x5 + x4 3x3 + b x 5
Then press ENTER .
To calculate the values of a and b, complete
the entry line as:
solve (p(1) = 5 and p(2) = 65,a)
Then press ENTER .

Given p(x) = ax5 + x4 3x3 + bx 5 and solving


p(1) = 5 and p(2) = 65 gives a = 2 and b = 6.

Write the answer.

reMeMBer

1. If P(x) = a nx n + an 1xn 1 + . . . + a2x2 + a1x + a0 and n is a non-negative integer then


P(x) is a polynomial of degree n and an, an 1, . . . a2, a1 are called coefficients and R.
2. A polynomial P(x) is evaluated in the same way as any function.
exerCISe

1B

polynomials
1 We 7

Which of the following are not polynomial expressions?

i x3 2x

ii x4 + 3x2 2x + x

iii x7 + 3x6 2xy + 5x

iv 3x8 2x5 + x2 7

v 4x6 x3 + 2x 3

vi 2 x 5 + x 4 x 3 + x 2 + 3 x

2
x

2 We8
Given that P(x) = 8 3x + 2x2 + x4, Q(x) = x5 3x4 4x2 1
and R(x) = 8x3 + 7x2 4x then find:
a P(x) + Q(x)
b Q(x) R(x)
c 3P(x) 2R(x)
d 2P(x) Q(x) + 3R(x).
eBook plus

3 We9 For each of the following polynomials, find: i its degree ii P(0) iii P(2) and iv P(1).

Digital doc

Spreadsheet 043
Evaluating
polynomials

eBook plus
Digital doc

SkillSHEET 1.2
Simultaneous
equations

a P(x) = x6 + 2x5 x3 + x2

b P(x) = 3x7 2x6 + x5 8

c P(x) = 5x6 + 3x4 2x3 6x2 + 3

d P(x) = 7 + 2x 5x2 + 2x3 3x4

4 MC If P(x) = x8 3x6 + 2x4 x2 + 3, then P(2) is equal to:


A 479
B 95
C 31
D 481
5 We 10
6
7
8
9

If P(x) =

2x7

ax5

3x3

+ bx 5, P(1) = 4 and P(2) = 163, find a and b.

Find a and b, given that f (x) =


+ bx3 3x2 4x + 7, f (1) = 2 and f (2) = 5.
For Q(x) = x5 + 2x4 + ax3 6x + b, Q(2) = 45 and Q(0) = 7. Find a and b.
Find a and b if P(x) = ax6 + bx4 + x3 6, 3P(1) = 24 and 3P(2) = 102.
ax4

MC

If P(x) = ax4 x3 + 3x2 5 and P(1) = 1, then a is equal to:

a
A 1

B 0

C 2

b If f (x) = xn 2x3 + x2 5x and f (2) = 10, then n is equal to:


A 4
B 6
C 7
D 5
10

E 103

Maths Quest 12 Mathematical Methods CAS for the TI-Nspire

1C

division of polynomials

eBook plus

Interactivity
When sketching cubic or higher order graphs, it is necessary to
int-0246
factorise the polynomials in order to find the x-intercepts. As will be
Division of polynomials
shown later in this section, division of polynomials can be used to
factorise an expression.
When one polynomial, P(x), is divided by another, D(x), the result can be expressed as:
P( x)
R( x )
= Q( x ) +
D( x )
D( x )
where Q(x) is called the quotient,
R(x) is called the remainder, and
D(x) is called the divisor.

Worked exAMple 11

eBook plus

Find the quotient, Q(x), and the remainder, R(x), when x 4 3 x 3 + 2 x 2 8


is divided by the linear expression x + 2.

Tutorial

int-0517
Worked example 11

ThINk

WrITe

Method 1: Technology-free
1

Set out the long division with each polynomial in


descending powers of x. If one of the powers of x
is missing, include it with 0 as the coefficient.

Dividex into x4 and write the result above.

5
6

7
8

Multiply the result


underneath.

x3

by x + 2 and write the result

Subtract and bring down the remaining terms


to complete the expression.

x + 2 x 4 3x3 + 2 x 2 + 0 x 8
x3
x + 2 x 3x 3 + 2 x 2 + 0 x 8

x3
3x 3 + 2 x 2 + 0 x 8
x+2
4 +2 3
x
x

The polynomial x3 5x2 + 12x 24, at the top, is


the quotient.
The result of the final subtraction, 40, is the
remainder.

x4

x3
x+2
3x3 + 2 x 2 + 0 x 8
(x 4 + 2 x 3 )
5x3 + 2 x 2 + 0 x 8

Dividex into 5x3 and write the result above.


Continue this process to complete the long
division.

x+2

x4

x 3 5 x 2 + 12 x 24
x4

3x3 + 2 x 2 + 0 x 8

(x 4 + 2x3 )

5x3 + 2 x 2 + 0 x 8

( 5 x 3 10 x 2 )
12 x 2 + 0 x 8
(12 x 2 + 24 x )
24 x 8

( 24 x 48)
40
The quotient, Q(x), is x3 5x2 + 12x 24.
The remainder, R(x), is 40.

Chapter 1

Graphs and polynomials

11

Method 2: Technology-enabled
1

On a Calculator page, press:


Menu b
2:Number 2
8:Fraction Tools 8
1:Proper Fraction 1
Complete the entry line as:
x 4 3x3 + 2 x 2 8
propFrac (
)
x+2
Then press ENTER .

Write the answer.

Dividing x4 - 3x3 + 2x2 - 8 by x + 2 gives a quotient,


Q(x), of x 3 5 x 2 + 12 x 24 and a remainder, R(x),
of 40.

Note: P(2) = (2)4 3(2)3 + 2(2)2 8


= 16 + 24 + 8 8
= 40
The remainder when P(x) is divided by (x + 2) is P(2).
This leads to the remainder theorem, which states:
When P(x) is divided by ( x a), the remainder is P(a)
or

when P(x) is divided by ( ax + b), the remainder is P b .
a
Furthermore, if the remainder is zero, then (x a) is a factor of P(x).
This leads to the factor theorem, which states:
If P( a) = 0, then ( x a) is a factor of P(x)
or
b
if ( ax + b) is a factor of P(x), then P = 0.
a

Note: If (x a) is a factor of P(x) and a is an integer, then a must be a factor of the term
independent of x. For example, if (x 2) is a factor of P(x), then the term independent of x
must be divisible by 2. Therefore, (x 2) could be a factor of x3 2x2 x + 2, but (x + 3) could
not be a factor.

Worked Example 12

Determine whether or not D( x ) = ( x 3) is a factor of P ( x ) = 2 x 3 4 x 2 3 x 8.


Think

Write

Method 1: Technology-free
1

12

Evaluate P(3).

P (3) = 2(3)3 4(3)2 3(3) 8


= 54 36 9 8
=1

Maths Quest 12 Mathematical Methods CAS for the TI-Nspire

If P(3) = 0 then (x 3) is a factor of P(x), but


if P(x) 0, (x 3) is not a factor of P(x).

P (3) 0 so (x 3) is not a factor of P(x).

Method 2: Technology-enabled
1

On a Calculator page, define P(x) by


completing the entry lines as:
Define p( x ) = 2 x 3 4 x 2 3 x 8
p(3)
Press ENTER after each entry.

Since p(3) = 1, (x - 3) is not a factor of P(x).

p(3) 0 so ( x 3) is not a factor of P(x).

Worked Example 13
a Factorise P ( x ) = 2 x 3 x 2 13 x 6. b Solve 2 x 3 x 2 13 x 6 = 0.
Think

Write

Method 1: Technology-free
a

Use the factor theorem to find a value for


a where P(a) = 0 and a is afactor of the
numerical term. Try a = 1, 1, 2, 2, 3, 3,
6, 6 until a factor is found.

Divide P(x) by the divisor (x + 2) using long


division.

Express P(x) as a product of linear and


quadratic factors.

Factorise the quadratic, if possible.

P(1) = 2(1)3 (1)2 13(1) 6


= 18
0
P(1) = 2(1)3 (1)2 13(1) 6
=4
0
P(2) = 2(2)3 (2)2 13(2) 6
= 20
0
P(2) = 2(2)3 (2)2 13(2) 6
=0
So (x + 2) is a factor.

2 x 2 5x 3

x + 2 2 x 3 x 2 13 x 6
(2 x 3 + 4 x 2 )

5 x 2 13 x 6
( 5 x 2 10 x )
3x 6
( 3 x 6)
0
P(x) = (x + 2)(2x2 5x 3)
= (x + 2)(2x + 1)(x 3)

Chapter 1 Graphs and polynomials

13

Rewrite the equation in factorised


form, using the answer to part a .

b 2x3 x2 13x 6 = 0

(x + 2)(2x + 1)(x 3) = 0

x = 2, 1 or 3
2

Use the Null Factor Law to state the


solutions.
Method 2: Technology-enabled
a

On a Calculator page, define the


polynomial P(x) by completing the
entry line as:
Define p( x ) = 2 x 3 x 2 13 x 6
Then press ENTER .
To factorise P(x), complete the entry
line as:
factor (p(x))
Then press ENTER .

Write the answer.

To solve P(x) = 0, complete the entry


line as:
solve ( p( x ) = 0, x )
Then press ENTER .

Write the answer.

Factorising p(x) = 2x3 - x2 - 13x - 6 gives


P ( x ) = ( x 3)( x + 2)(2 x + 1)
b

Solving 2x3 - x2 - 13x - 6 = 0 gives

1
x = 2, x = , or x = 3
2

REMEMBER

P( x)
R( x )
= Q( x ) +
D( x )
D( x )
where Q(x) is called the quotient,
R(x) is called the remainder,
D(x) is called the divisor.
2. Remainder theorem: If P(x) is divided by (x a), then the remainder is P(a).
3. Factor theorem: If P(a) = 0, then (x a) is a factor of P(x).
b
If (ax + b) is a factor of P(x), then P = 0.
a
4. If (x a) is a factor of P(x), then a must be a factor of the term independent of x.
1.

14

Maths Quest 12 Mathematical Methods CAS for the TI-Nspire

exerCISe

1C

division of polynomials
1 We 11
Find the quotient, Q(x), and the remainder, R(x), when each of the following
polynomials are divided by the given linear expression.
a x3 2x2 + 5x 2, x 4
b x5 3x3 + 4x + 3, x + 3
c 6x4 x3 + 2x2 4x, x 3
d 3x4 6x3 + 12x, 3x + 1
2 a For each corresponding polynomial in question 1, evaluate:
i P(4)
ii P(3)
iv P(

iii P(3)

1
)
3

b Compare these values to R(x) in question 1 and comment on the result.


3 We 12
a
b
c
d
eBook plus
Digital doc

Spreadsheet 096
Finding factors
of polynomials

In each of the following determine whether or not D(x) is a factor of P(x).

P(x) = x3 + 9x2 + 26x 30, D(x) = x 3


P(x) = x4 x3 5x2 2x 8, D(x) = x + 2
P(x) = 4 9x + 6x2 13x3 12x4 + 3x5, D(x) = 4 x
P(x) = 4x6 + 2x5 8x4 4x3 + 6x2 9x 6, D(x) = 2x + 1

4 MC Examine the equation f (x) = x4 4x3 x2 + 16x 12.


a Which one of the following is a factor of f (x)?
A x+1
B x
C x+2
D x+3
b When factorised, f (x) is equal to:
A (x + 1)(x 3)(x + 4)
C (x + 2)(x 4)(x + 3)(x + 1)
E x(x 1)(x + 2)(x + 3)

E x4

B (x + 2)(x 2)(x 3)(x 1)


D (x 1)(x + 1)(x 3)(x 4)

5 We 13a Factorise the following polynomials.


b P(x) = 3x3 13x2 32x + 12
a P(x) = x3 + 4x2 3x 18
c P(x) = x4 + 2x3 7x2 8x + 12

d P(x) = 4x4 + 12x3 24x2 32x

6 We13b Solve each of the following equations.


b 2x4 + 10x3 4x2 48x = 0
a 3x3 + 3x2 18x = 0
c 2x4 + x3 14x2 4x + 24 = 0
7 If (x 2) is a factor of

x3

ax2

8 If (x 1) is a factor of

x3

x2

eBook plus

d x4 2x2 + 1 = 0

Digital doc

WorkSHEET 1.1

6x 4, then find a.

ax + 3, then find a.

9 Find the value of a if (x + 3) is a factor of 2x4 + ax3 3x + 18.


10 Find the value of a and b if (x + 1) and (x 2) are factors of ax3 4x2 + bx 12.
11 If (2x 3) and (x + 2) are factors of 2x3 + ax2 + bx + 30, find the values of a and b.

1d

linear graphs
Linear graphs are polynomials of degree 1. Graphs of linear functions are straight lines and may
be sketched by finding the intercepts.

revision of properties of straight line graphs

1. The gradient of a straight line joining two points is:


y y1
m= 2
x2 x1

B (x2, y2)
A (x1, y1)
0

Chapter 1

Graphs and polynomials

15

2. The general equation of a straight line is:


y = mx + c
where m is the gradient
and c is the value of the y-intercept.
3. The equation of a straight line passing through the
point (x1, y1) and having a gradient of m is:
y y1 = m(x x1)

Gradient = m

A (x1, y1)
x

4. The intercept form of the equation of a straight line is:

y
(0, b)

x y
+ = 1 or bx + ay = ab
a b

(a, 0)
0

5. Parallel lines have the same gradient.


6. The product of the gradients of two lines that are perpendicular equals 1.
1
That is, m1 m2 = 1 or m1 =
m2
Worked Example 14

Sketch the graph of the linear function 3 x 2 y = 6 by indicating the intercepts.


Think

Write/Draw

Substitute y = 0 into the equation.

Solve the equation for x to find the x-intercept.

Substitute x = 0 into the equation.

Therefore, the x-intercept is 2.

Solve the equation for y to find the y-intercept.

When x = 0, 3 0 2y = 6
y = 3

Draw a set of axes.


Indicate the x-intercept and y-intercept and rule a
line through these points.

Therefore, the y-intercept is 3.

When y = 0, 3x 2 0 = 6
x=2

y
0

3x 2y = 6
x
2

Worked Example 15

Find the equation, in the form ax + by + c = 0, of each straight line described below.
a The line with a gradient of 2 and passing through ( 3, 2 )
b The line passing through (0, 8) and ( 2, 2 )
c The line that passes through (3, 4) and is parallel to the line with equation y 2 x 5 = 0
d The line that passes through (1, 3) and is perpendicular to the line with equation y + 2 x 3 = 0
16

Maths Quest 12 Mathematical Methods CAS for the TI-Nspire

ThINk
a

WrITe

Write the rule for the pointgradient


form of the equation of a straight
line, y y1 = m(x x1).

Substitute the value of the gradient,


m, and the coordinates of the point
(x1, y1), into the equation.

Expand the brackets.

Express the equation in the form


required.

Write the rule for the gradient, m, of


a straight line, given 2 points.

Substitute the values of (x1, y1) and


(x2, y2) into the rule and evaluate the
gradient.

Substitute the values of m and (x1, y1)


into the rule for the pointgradient
form of the equation of a straight line.
(Coordinates of either point given may
be used.)

Expand the brackets.

Express the equation in the form


required.

State the gradient of the given line,


which is the same as the gradient of the
parallel line.

y y1 = m(x x1)

y (2) = 2(x 3)

y + 2 = 2x 6
y 2x + 8 = 0
or 2x y 8 = 0
y y1
b
m= 2
x2 x1
28
0
6
=
2
=3
=

y y1 = m(x x1)
y 8 = 3(x 0)

y 8 = 3x
3x y + 8 = 0
c y 2x 5 = 0 becomes y = 2x + 5.

The gradient of the parallel lines is 2.

Write the rule for the pointgradient


form of the equation of a straight line.

y y1 = m(x x1)

Substitute the values of m and the


coordinates (x1, y1) = (3, 4).

y 4 = 2(x 3)

Simplify and write in the required form.

Find the gradient of the given line.

y 4 = 2x 6
2x y 2 = 0
d y = 2x + 3

The gradient of the line is 2.

Find the gradient of the perpendicular


line.

Write the rule for the pointgradient


form of the equation of a straight line.

y y1 = m(x x1)

Substitute the values of m and the


coordinates (x1, y1) = (1, 3).

y 3 = 12 (x 1)

Simplify and write in the required form.

The gradient of the perpendicular line is 12 .

2y 6 = (x 1)
x 2y + 5 = 0

Chapter 1

Graphs and polynomials

17

The domain and range of functions


The domain of a function, y = f (x), is the set of values of x for which the function is defined (that
is, all x-values that can be substituted into f (x) and an answer found).
The range of f (x) is the set of values of y for which the function is defined.
If the rule and the domain of a function are given, then the function is completely defined.
For example, y = 4x, x 0
f (x) = 4x, x 0
or
f : (, 0] R, f (x) = 4x

Interval notation
Restricted domains or ranges can be represented by interval notation in three forms.
1. The closed interval.
2. The open interval.
3. The half-open interval.
a

[a, b] = {x : a x b}

(a, b) = {x : a < x < b}

[a, b) = {x : a x < b}

If the domain or range is unrestricted, it can be denoted as R or (, ).


R+ = (0, )
R+ {0} = [0, )

R = ( , 0)
R {0} = (, 0]
Worked exAMple 16

Sketch the graph of each of the following functions, stating the domain and range
of each.
a 4 x 2 y = 8, x [ 3, 3]
b f ( x ) = 1 2 x, x ( , 1)
ThINk
a

WrITe/drAW/dISplAy

Substitute the smallest value of x


into the equation.

a When x = 3,
12

2y = 8
2y

= 20
y = 10

Solve the equation for y, to find an


end point of the straight line.

State the coordinates of the end


point.

(3, 10) is a closed end of the line.

Substitute the largest value of x into


the equation.

When x = 3,
12 2y = 8

Solve the equation for y, to find the


other end point of the line.

2y

State the coordinates of the 2nd end


point.

Plot the two points on a set of axes


with closed circles (since both
points are included).

= 4
y=2

(3, 2) is the other closed end of the line.


y
2
3

0
4

(3, 10)

18

Maths Quest 12 Mathematical Methods CAS for the TI-Nspire

10

(3, 2)
2 3

4x 2y = 8,
x [3, 3]

Draw a straight line between the two


points.

Find the intercepts and mark them


on the graph.

When x = 0, y = 4
When y = 0, x = 2
The x-intercept is 2 and the y-intercept is 4.

10

State the domain, which is given


with the rule.

The domain is [3, 3].

11

State the range from the graph.

The range is [10, 2].

Method 1: Technology-free
b

There is no smallest value of x, so


substitute the largest value of x into
the equation and find y.

b When x = 1,

y = f (1)
=3

State the coordinates of the upper


end point.

(1, 3) is an open end of the line.

Substitute another value of x within


the domain into the equation (that is,
a value of x < 1, since x (, 1))
and find y.

When x = 2,
y = f (2)
=5

State the coordinates of the point.

(2, 5) is another point on the line.

Plot the 2 points on a set of axes and


mark the point (1, 3) with an open
circle.

Rule a straight line from (1, 3)


to (2, 5) and beyond. An arrow
should be placed on the other end to
indicate that the line continues.

f(x) = 1 2x,
x (, 1)
(2, 5)
(1, 3)

y
5
3

2 1 0

Note that there are no intercepts.

State the domain, which is given


with the rule.

The domain is (, 1).

State the range by examining the graph.

The range is (3, ).

Method 2: Technology-enabled
 n a Graphs page, complete the function entry
O
line as:
f 1( x ) = 1 2 x | < x < 1
Then press ENTER .

Chapter 1 Graphs and polynomials

19

reMeMBer

Linear graphs
1. Linear equations are polynomials of degree 1.
y y1
2. Gradient, m = 2
x2 x1
3. General equation is
ax + by + c = 0 or y = mx + c
where m = gradient and c = y-intercept.
4. Equation if a point and the gradient are known:
y y1 = m(x x1)
5. Equation if the intercepts are known:
x y
+ =1
a b
6. Parallel lines have the same gradient.
7. If m1 and m2 are the gradients of perpendicular lines, then:
m1 m2 = 1
1
m1 =
or
m2

exerCISe

1d

linear graphs
1 We 14
Sketch the graph of each of the following linear functions
by indicating the intercepts.
a 2x + 3y = 12
b 2y 5x 10 = 0
c 2x y = 1
2 We 15a Find the equation, in the form ax + by + c = 0, of each straight line
described below.
a The line with a gradient of 3 and passing through (2, 1).
b The line with a gradient of 2 and passing through (4, 3).

eBook plus
Digital doc

SkillSHEET 1.3
Gradient

eBook plus
Digital doc

SkillSHEET 1.4
Using gradient
to find the
value of a
parameter

20

3 We 15b Find the equation, in the form ax + by + c = 0, of each straight line


described below.
a The line passing through (3, 4) and (1, 10).
b The line passing through (7, 5) and (2, 0).
4 MC Which one of the following points does not lie on the straight line with equation
2y 3x 6 = 0?
A (2, 6)
B (2, 0)
C (0, 3)
D (1, 2)
E (4, 9)
5 We 15c Consider the points A(2, 5) and B(1, b).
a Find b if:
i the gradient of the straight line AB is 2
ii the equation of the straight line AB is y x = 7.
b Find the general equation of the straight line which passes through (4, 5) and is parallel to
the line with equation y 3x + 4 = 0.
c We 15d Find the equation in the form ax + by + c = 0 that passes through (2, 4) and is
perpendicular to the line with equation 2y x + 1 = 0.

Maths Quest 12 Mathematical Methods CAS for the TI-Nspire

6 Match each of the following graphs with the appropriate rule below.
b
c
a
y
y
y
(2, 4)

0
1

i x + 2y + 4 = 0
iv 3y + 2x = 6
eBook plus

ii y = 3

iii y 2x 2 = 0

v y 2x = 0

vi x = 2

7 State the range for each function graphed below.


a

(4, 3)

Digital doc

SkillSHEET 1.5

Interval
notation

Digital doc

(3, 3)

(5, 2)
eBook plus

y
4

(4, 2)

(6, 5)

(5, 6)

SkillSHEET 1.6
Domain and
range for
linear graphs

(5, 2)

8 We 16 Sketch the graph of each of the following functions, stating i the domain and i i the
range of each.
a 4y + 3x = 24, x [12, 12]
b 2x 5y = 10, x < 5
c 4x 3y 6 = 0, x [2, 5)
9 Find the equation of the straight line which passes through the point (2, 5) and is:
a parallel to the line with equation y = 3 2x
b perpendicular to the line with equation y = 3x 7.
Write equations in the form ax + by + c = 0.
10 Find the equation of the straight line which passes through the point (3, 1) and is:
a parallel to the line with equation 4x 2y = 13
b perpendicular to the line with equation 4x 2y = 13.
11 MC If the straight lines 3x y = 2 and ax + 2y = 3 are parallel then a = :
A 6
B 2
C 2
D 3
E 6
12 MC If the straight lines 5x + y 3 = 0 and bx y 2 = 0 are perpendicular, then
b is equal to:
A 5

1
5

C 5

1
5

E 3

Chapter 1

Graphs and polynomials

21

1e

Quadratic graphs
Quadratic functions are polynomials of degree 2. Graphs of quadratic functions are parabolas
and may be sketched by finding the turning point and intercepts.

revision of quadratic functions


1. The general form of the quadratic function is y = ax2 + bx + c, x R.
2. The graph of a quadratic function is called a parabola and:
(a) for a > 0, the graph has a minimum value
(b) for a < 0, the graph has a maximum value
(c) the y-intercept is c

b
(d) the equation of the axis of symmetry and the x-value of the turning point is x =
2a
(e) the x-intercepts are found by solving the equation ax2 + bx + c = 0.
2
3. The equation ax + bx + c = 0 can be solved by either:
(a) factorising
or
b b 2 4 ac
(b) using the quadratic formula, x =
.
2a
4. The turning point can be found by completing the square (see page 23). The turning point is
located on the axis of symmetry, which is halfway between the x-intercepts.

The discriminant
The value of (b2 4ac), which is the value inside the square root sign in the quadratic formula,
determines the number of solutions to a quadratic equation or the number of x-intercepts on a
quadratic graph.
This value is called the discriminant.
1. If b2 4ac > 0, there are two solutions to the equation and there are two x-intercepts on the
graph.
2. If b2 4ac > 0 and is a perfect square, the solutions are rational; otherwise they are
irrational.
3. If b2 4ac = 0, the two solutions are equal and there is one x-intercept on the graph; that is,
the graph has a turning point on the x-axis.
4. If b2 4ac < 0, there are no real solutions and there are no x-intercepts on the graph.

Worked exAMple 17

eBook plus

Use the discriminant to determine the number of x-intercepts for


the quadratic function f ( x ) = 2 x 2 + 3 x 10.
ThINk

22

WrITe

Tutorial

int-0518
Worked example 17

b = 3, c = 10

Find the values of the quadratic coefficients a,


b and c using the general quadratic function,
y = ax2 + bx + c.

a = 2,

Evaluate the discriminant.

b2 4ac = 32 4(2)(10)
= 9 + 80
= 89

If the discriminant is greater than 0, there are


two x-intercepts. If it is not a perfect square, the
solutions are irrational.

b2 4ac > 0
So there are two x-intercepts, which are both
irrational.

Maths Quest 12 Mathematical Methods CAS for the TI-Nspire

Worked Example 18

Sketch the graph of the function f ( x ) = 12 5 x 2 x 2, showing all intercepts. Give exact answers.
Think

Write/draw

f(0) = 12 5(0) 2(0)2


= 12
The y-intercept is 12.
f(x) = 12 5x 2x2 = 0
(4 + x)(3 2x) = 0

Evaluate f (0) to find the y-intercept (or state the


value of c).
State the y-intercept.
Set f(x) = 0 to find the x-intercepts.
Factorise the quadratic (or use the quadratic
formula).
Solve the equation using the Null Factor Law.

State the x-intercepts.

The x-intercepts are 4 and 2 .

Draw a set of axes and mark the intercepts or the


coordinates of the points where the graph crosses
the axes.

1
2
3
4

4 + x = 0 or 3 2x = 0
x = 4 orx = 32
3

y
12 (0, 12)
f(x) = 12 5x x2

Sketch a parabola through the intercepts.


(4, 0)
4

( 32 , 0)
x
0 1 2

The x-coordinate of the turning point of a quadratic function is exactly halfway between
4 + 3

2
=
the two x-intercepts, so for worked example 18, x =
(or 1 14 ). Substitute x = 5
4
2
4
into the original equation to find the y-coordinate of the turning point.

b
The x-coordinate of the turning point can also be found by using the formula x =
, where
2a
ax2 + bx + c = 0.

Finding turning points by completing the square


Consider the general quadratic equation:
y = ax2 + bx + c
By completing the square, this equation may be manipulated into the form
y = a(x b)2 + c
where the turning point is (b, c).
This way of writing the function is known as the power form or turning point form. The
transformations associated with this form will be discussed more fully in chapter 2.
Worked Example 19

For the function y = 2( x + 3)2 4, find:


a the coordinates of the turning pointb the domain and range.
Think

Write

Write the general formula.

y = a(x b)2 + c

Write the function.

y = 2(x + 3)2 4

a 1

Identify the values of a, b and c.

a a = 2, b = 3, c = 4

Chapter 1 Graphs and polynomials

23

b 1
2

State the coordinates of the turning


point (b, c).
Write the domain of the parabola.
Write the range y c (as a < 0).

The turning point is (3, 4).


b The domain is R.

The range is y 4.

Worked Example 20

The function graphed at right is of the form y = x 2 + bx + c.


Find:
a the ruleb the domainc the range.
Write the answers to b and c in interval notation.

y
(5, 10)

(1, 6)
Think
a

Write
a y = a(x b)2 + c

Write the general rule for a quadratic


in turning point form.

Find the values of b and c using the


given turning point.

Since the turning point is (1, 6):


b = 1, c = 6

State the value of a (given).

a=1

Substitute these values in the rule.

So y = 1(x + 1)2 6

Expand the brackets.

Simplify.

Use the graph to find the domain.


Look at all the values that x can take.

State the domain in interval notation.

Use the graph to find the range. Look


at all the values that y can take.

State the range in interval notation.

= x2 + 2x + 1 6
= x2 + 2x 5
The rule is y = x2 + 2x 5.
b x 5

Domain = [5, )
c y 6

Range = [6, )

Worked Example 21
1

Sketch the graph of y = 2 ( x 1)2 + 2, clearly showing the coordinates of the turning point and the
intercepts with the axes. State its range.
Think

24

Write/Draw

Write the general equation of the parabola.

y = a(x b)2 + c

Identify the values of the variables.

a = 12 , b = 1, c = 2

Maths Quest 12 Mathematical Methods CAS for the TI-Nspire

Write a brief statement on the transformation of the


basic parabola.

The graph of y = x2 is dilated in the y direction


by the factor of 12 (that is, it is wider than the
basic curve); it is translated 1 unit to the right
and 2 units up.

State the shape of the parabola (that is, positive or


negative).

a > 0; the parabola is positive.

State the coordinates of the turning point (b, c).

The turning point is (1, 2).

As both a and c are positive, only the y-intercept


needs to be determined. Find the y-intercept by
making x = 0.

y-intercept: x = 0
y = 12 (0 1)2 + 2
= 12 (1)2 + 2
= 12 + 2
= 2 12

Sketch the graph:


Drawasetofaxesandlabelthem.Plottheturning
point and the y-intercept. Sketch the graph of the
positive parabola, so that it passes through the
points previously marked.

2 2
2
0

Since y 2, that is the range.

x
1
y = 12 (x 1)2 + 2

The range is y [2, ).

Worked exAMple 22

Sketch the graph of y = 3 + 8 x 2 x 2, showing the turning point and all intercepts, rounding answers
to 2 decimal places where appropriate.
ThINk

WrITe/drAW

Method 1: Technology-free
1

Find y when x = 0.

When x = 0, y = 3

State the y-intercept.

The y-intercept is 3.

Let the quadratic equal zero.

When y = 0,
3 + 8x 2x2 = 0

Solve for x using the quadratic formula.


x=
=
=
=

82 4( 2)(3)

2( 2)
88

4
8 2

22

4
4

=2

22

22
2

or

Chapter 1

2+

22
2

Graphs and polynomials

25

State the x-intercepts, rounding to 2 decimal


places.

Use the formula for the x-value of the turning

b
point, x =
.
2a

The x-intercepts are 0.35 and 4.35.

x=

2( 2)
x=2

To calculate the y-coordinate of the turning point,


substitute x = 2 into the function.

y = -2(2)2 + 8(2) + 3
y = 11

State the turning point.

The turning point is (2, 11).

Draw a set of axes and mark the coordinates of


the turning point and the points where the graph
crosses the axes.
Sketch a parabola through these points.

10

y
12

(2, 11)

9
f(x) = 3 + 8x

2x2

3 (0, 3)
(0.35, 0)
(4.35, 0)
x
0
1
4 5

Method 2: Technology-enabled
On a Graphs page, complete the function entry
line as:
f(x) = 3 + 8x - 2x2
Then press ENTER .
To label the coordinates of the intercepts and
turning point, press:
Menu b
6:Analyze Graph 6
Select the appropriate action.

Note: Function notation includes the rule, the domain and the co-domain. For example,
f(x): [2, 1] R, where f (x) = x2 3, is a parabola with rule f (x) = x2 3 and domain [2, 1].
The range is a subset of the co-domain, R.

Worked Example 23

The weight of a person t months after a


gymnasium program is started is given by the
2
function: W ( t ) = t 3 t + 80, where t [0, 8]
2
and W is in kilograms. Find:
a the minimum weight of the person
b the maximum weight of the person.

26

Maths Quest 12 Mathematical Methods CAS for the TI-Nspire

Think
1

Write

Complete the square to find the turning point.

t2
3t + 80
2
1
= 2 [t 2 6t + 160]

W=

= 2 [t 2 6t + 9 + 160 9]
1

= 2 [(t 3)2 + 151]


1

= 2 (t 3)2 + 75.5
2
3
4
5
6
7
8
9

State the minimum turning point.


Find the end point value for W when t = 0.
State its coordinates.
Find the end point value of W when t = 8.
State its coordinates.
On a set of axes, mark the end points and turning point.
Sketch a parabola between the end points.
Locate the maximum and minimum values of W on the graph.

The turning point is (3, 75.5).


When t = 0,
W = 80
One end point is (0, 80).
When t = 8,
W = 88
The other end point is (8, 88).
W (kg)
Maximum (8, 88)

90
80 (0, 80)
70

Minimum (3, 75.5)


0 1 2 3 4 5 6 7 8 t (months)

a
b

State the minimum weight from the graph.


State the maximum weight from the graph.

a
b

The minimum weight is 75.5 kg.


The maximum weight is 88 kg.

REMEMBER

Quadratic graphs
1. Quadratic equations are polynomials of degree 2.
2. The general equation is y = ax2 + bx + c.
3. The quadratic formula is given by the equation
b b 2 4 ac
2a
4. The discriminant is b2 4ac and if:
(a) b2 4ac > 0, there are two x-intercepts. If b2 4ac is a perfect square, the
intercepts are rational.
(b) b2 4ac = 0, there is one x-intercept, which is a turning point.
(c) b2 4ac < 0, there are no x-intercepts.
5. The turning point form of the quadratic graph or parabola is:
y = a(x b)2 + c and the turning point is (b, c).
6. The equation of the axis of symmetry of a parabola and the x-value of the turning
b
point is given by the expression x =
.
2a
7. The axis of symmetry is halfway between the x-intercepts.
x=

Chapter 1 Graphs and polynomials

27

exerCISe

1e
eBook plus
Digital doc

Spreadsheet 103
Discriminant

Quadratic graphs
1 We17
use the discriminant to determine the number of x-intercepts for each of the
following quadratic functions.
a f (x) = x2 3x + 4
b f (x) = x2 + 5x 8
c f (x) = 3x2 5x + 9
2
2
d f (x) = 2x + 7x 11
e f (x) = 1 6x x
f f (x) = 3 + 6x + 3x2
2 We18
Sketch the graphs of each of the following functions,
showing all intercepts. Give exact answers.
a f (x) = x2 6x + 8
c f (x) = 10 + 3x x2

eBook plus

b f (x) = x2 5x + 4
d f (x) = 6x2 x 12

Digital doc

Spreadsheet 107
Quadratic graphs

3 Find the turning point for each of the functions in question 2.


Give exact answers.

eBook plus

4 We19 For each of the following functions find:


i the coordinates of the turning point
ii the domain
iii the range.
2
b y = (x 6)2
a y=2x
c y = (x + 2)2
d y = 2(x + 3)2 6

Digital doc

SkillSHEET 1.7
Domain
and range for
quadratic graphs

5 We20
Each of the functions graphed below is of the form y = x2 + bx + c. For each
function, give: i the rule
ii the domain
iii the range.
Write the answers to b and c in interval notation.
a

(1, 6)
x

0
(1, 2)

(1, 9)

(2, 3)

(4, 16)

6 We21
Sketch the graphs of the following, clearly showing the coordinates of the turning
point and the intercepts with the axes.
a y = 2x2 + 3
b y = (2x 5)(2x 3)
c y = (2x 3)2 8
Consider the function with the rule y = x2 2x 3.
7 MC
a It has x-intercepts:
A (1, 0) and (3, 0)
B (1, 0) and (3, 0)
C (1, 0) and (3, 0)
1, 0)

E (0, 1) and (0, 3)


D (2, 0) and (
b It has a turning point with coordinates:
A (1, 0)

B (2, 3)
(x

+
8 MC The function f (x) =
A (3, )
B (, 3]

C (1, 4)
3)2

D (1, 4)

+ 4 has a range given by:


C [4, )
D (, 4]

9 MC The range of the function y = (x 4)2, x [0, 6] is:


A [0, 16]
B [4, 16]
C [0, 4]
D (4, 12]

E [0, 16)

10 We22 Sketch the graph of each of the following functions, showing


the turning point and all intercepts. Round answers to 2 decimal
places where appropriate.
a f (x) = (x 2)2 4
b f (x) = (x + 4)2 + 9
2
c y = x + 4x + 3
d y = 2x2 4x 6

28

Maths Quest 12 Mathematical Methods CAS for the TI-Nspire

eBook plus
Digital doc

Spreadsheet 041
Function grapher

E (1, 0)
E R

eBook plus
Digital doc

Spreadsheet 108
Quadratic graphs
turning
point form

11 Sketch the graph of each of the functions below and state i the domain and ii the range of each
function.
b y = x2 + x 1, x R+
a y = x2 2x + 2, x [2, 2]
c f (x) = x2 3x 2, x [10, 6]
d f (x) = 5 + 6x 3x2, x [5, 3)
12 We23 The volume of water in a tank, V m3, over a 10 month period is given by the function
V(t ) = 2t 2 16 t + 40, where t is in months and t [0, 10].
Find:
Maximum height
a the minimum volume of water in the tank
b the maximum volume of water in the tank.
Tower

13 A ball thrown upwards from a tower attains a


height above the ground given by the function
h(t) = 12t 3t2 + 36, where t is the time in
seconds and h is in metres.
Find:
a the maximum height above the ground that
the ball reaches
b the time taken for the ball to reach the
ground
c the domain and range of the function.

eBook plus
Digital doc

WorkSHEET 1.2

1F

Ball

h(t) = 12t 3t2 + 36

Ground

14 A section of a roller-coaster at an amusement park follows the


path of a parabola. The function h(t) = t2 12t + 48, t [0, 11],
models the height above the ground of the front of one of the
carriages, where t is the time in seconds and h is the height in
metres.
a Find the lowest point of this section of the ride.
b Find the time taken for the carriage to reach the lowest point.
c Find the highest point above the ground.
d Find the domain and the range of the function.
e Sketch the function.

Cubic graphs
Cubic functions are polynomials of degree 3. In this section, we will look at how graphs of cubic
functions may be sketched by finding intercepts and recognising basic shapes.

Forms of cubic functions


Cubic functions may take several forms. The three main forms are described below.

General form
The general form of a cubic function is
y = ax3 + bx2 + cx + d
If a is positive (that is, a > 0), the function is called a positive cubic. Several positive cubics
appear below.
y

x
x

Chapter 1

Graphs and polynomials

29

If a is negative (that is, a < 0), the function is called a negative cubic. Several negative cubics
appear below.
y

x
x

Basic form
Some (but certainly not all) cubic functions are transformations of the form y = x3, which has a
point of inflection at the origin. These may be expressed in the power form
y = a(x b)3 + c
where (b, c) is the point of inflection.
For example, y = 2(x 3)3 + 5 is the graph of y = x3 translated +3 from the y-axis, +5 in the
y direction and dilated by a factor of 2 from the x-axis.
This form, called basic form or power form, works in the same way as a quadratic equation
expressed in turning point form or power form:
y = a(x b)2 + c
where (b, c) is the turning point and a is the dilation factor.
The power form and its transformations will be discussed in more detail
in chapter 2.
y

y = x3

y = a(x b)3 + c

(b, c)
x

Factor form
Cubic functions of the type
y = a(x b)(x c)(x d)
are said to be in factor form, where b, c and d are the x-intercepts. Often a cubic function in
general form may be factorised to express it in factor form.
y

y = a(x b)(x c)(x d)


where a > 0

y = (x + 2)(x 1)(x 3)
y

2
b

30

c d

Maths Quest 12 Mathematical Methods CAS for the TI-Nspire

3 x

repeated factors
y

A twice only repeated factor in a factorised cubic function


indicates a turning point that just touches the x-axis.

y = (x a)2 (x b)

Worked exAMple 24

eBook plus

For each of the following graphs, find the rule and express
it in factorised form. Assume that a = 1 or a = 1 .
a

f(x)

Tutorial

int-0519

f(x)

x
2 0

ThINk
a

Worked example 24

WrITe
a The graph is a positive cubic, so a = 1.

Find a by deciding whether the graph is


a positive or negative cubic.

use the x-intercepts 4, 0 and 3 to find


the factors.

The factors are (x + 4), x and (x 3).

Express f(x) as a product of a and its


factors.

f(x) = 1(x + 4) x(x 3)

Simplify.

f(x) = x(x + 4)(x 3)

Find a by deciding whether the graph is


a positive or negative cubic.

use the x-intercept 2, which is also a


turning point, to find the repeated factor.

(x + 2)2 is a factor.

use the other x-intercept, 3, to find the


other factor.

(x 3) is also a factor.

Express f(x) as a product of a and its


factors.

f(x) = 1(x + 2)2(x 3)

Simplify.

f(x) = (3 x)(x + 2)2

b The graph is a negative cubic, so a = 1.

Worked exAMple 25

Sketch the graph of y = x 3 x 2 10 x 8, showing all intercepts.


ThINk
1

Find y when x = 0.

WrITe/drAW

When

x = 0, y = 8

Chapter 1

Graphs and polynomials

31

State the y-intercept.

The y-intercept is 8.

Let P(x) = y.

Let P(x) = x3 x2 10x 8

Use the factor theorem to find a factor of the


cubic P(x) = x3 x2 10x 8.

P(1) = 13 12 10(1) 8
= 18
0
P(1) = (1)3 (1)2 10(1) 8
=0
so (x + 1) is a factor.

Use long division, or otherwise, to find the


quadratic factor.

By long division:
x2 2x 8

x + 1 x 3 x 2 10 x 8
(x 3 + x 2 )
2 2 10 8
x
x
( 2 x 2 2 x )

8x 8

( 8 x 8)
0
y = (x + 1)(x2 2x 8)
= (x + 1)(x 4)(x + 2)

Factorise the quadratic, if possible.

Express the cubic in factorised form and let it


equal 0 to find the x-intercepts.
Solve for x using the Null Factor Law.
Alternatively, use a CAS calculator to solve
for x.

If (x + 1)(x 4)(x + 2) = 0

State the x-intercepts.


Sketch the graph of the cubic.

The x-intercepts are 2, 1, and 4.

9
10

x = 1, 4 or 2

y
2 1 0

y = x3 x2 10x 8
4

8
Exam tip When sketching graphs, ensure that they are smooth, that relevant turning points and intercepts
are labelled, and that they are drawn within the correct domain (end points should be shown using or ) using
[Authors advice]
an appropriate scale.

32

Maths Quest 12 Mathematical Methods CAS for the TI-Nspire

Restricting the domain of cubic functions


1. If the domain is R then the range is also R.
2. To find the range if the domain is restricted, it is necessary to look at the end points and
turning points, then find the highest and lowest y-values.
y
(6, 8)
For example:
The range can not be stated for the diagram at right because
the y-coordinate of the local minimum is not known.
Recall that cubic functions that do not have any turning
points can have only one x-intercept.

0
(4, 3)

Coordinate of local
minimum required

Worked Example 26

Sketch the graph of y = x 3 5 x, where x ( 2, 1], using the unrestricted function as a guide. State
the domain and range, without the use of technology.
Think

WRITE/DRAW

Decide whether it is a positive or negative cubic


by looking at the coefficient of x3.

Negative cubic

Find the x-intercept/s.

When y = 0,
x3 5x = 0
x(x2 + 5) = 0
x = 0 (x2 + 5 0)
The x-intercept is 0.

Find the y-intercept.

When x = 0,
y = (0)3 5(0)
=0
The y-intercept is 0.

Find y when x has the value of the lower end point


of the domain.

When x = 2, y = (2)3 5(2)


= 18

State the coordinates of this end point and decide


whether it is open or closed.

The open end point is (2, 18).

Find y when x has the value of the upper end point.

When x = 1, y = (1)3 5(1)



=6

State the coordinates of this end point and decide


whether it is open or closed.

The closed end point is (1, 6).

Mark these points on a set of axes.

Sketch the part of the cubic between the end


points.

10

Verify this graph using a graphics calculator.

(2, 18)

(1, 6)
0

11

State the domain, which is given with the rule.

The domain is (2, 1].

12

From the graph, state the range. Note that the


intercept is not included in the domain.

The range is [6, 18).

Chapter 1 Graphs and polynomials

33

reMeMBer

Cubic graphs
1. The general equation is y = ax3 + bx2 + cx + d.
2. Basic shapes of cubic graphs:
(a) If a > 0:
Positive cubic

Power form

y = a(x b)3 + c

(b, c)
x

Factor form

Repeated factor

y
y = a(x b)(x c)(x d)
where a > 0

c d

y = (x a)2 (x b)

(b) If a < 0, the reflections through the x-axis of the types of graph in the above figures
are obtained.
exerCISe

1F
eBook plus

Cubic graphs
1 We24
For each of the following graphs, find the rule and express it in factorised form.
Assume that a = 1 or a = 1.
b
a
y
y

Digital doc

Spreadsheet 013
Cubic graphs
factor form

Match each of the following graphs to the most appropriate rule below.
a

34

0 1

2 0

Maths Quest 12 Mathematical Methods CAS for the TI-Nspire

y
0

ii
iv
vi
viii

y = (x + 3)(1 x)(x 4)
y = (x + 2)2(5 x)
y = (x + 4)(x + 2)(x 1)
y = (x + 2)2(x 5)

y = (x 3)3
y = (x + 4)(x + 2)(1 x)
y = (x + 3)(x 1)(x 4)
y = (3 x)3

3 We25 Sketch the graph of each of the following, showing all intercepts.
a y = x3 + x2 4x 4
b y = 2x3 8x2 + 2x + 12
3
c y = 24 + 26x 2x
d y = 18 21x + 8x2 x3
Verify your answers by using a calculator.
4 MC
a Fully factorised, x3 + 6x2 + 12x + 8 is equal to:
A (x + 3)3
B (x + 2)3
3
D (x 3)
E (x + 2)(x 2)2
b The graph of y = x3 + 6x2 + 12x + 8 is:
B
A
y
y

4
0

i
iii
v
vii

eBook plus
Digital doc

Spreadsheet 014
Cubic graphs

C (x 2)3

5 MC The function graphed in the figure could have the following rule:
A y = (x 2)3 + 2
y
B y = (x + 2)3 + 2
C y = (2 x)3 + 2
10
D y = (x + 2)3 2
3
E y = (x 2)
(2, 2)

Chapter 1

Graphs and polynomials

35

eBook plus
Digital doc

6 MC The graph of f (x) = 5(x + 1)3 3 is best represented by:


A

Spreadsheet 015
Cubic graphs
y = a( x b ) 3 + c
form

(1, 3)

(1, 3)

x
(1, 3)

y
(1, 3)

(1, 3)
x

The graph of f (x) = 2(x 1)2 (x + 3) is best represented by:

7 MC
A

(0, 6)

(1, 0) 0

(0, 6)

(3, 0)

(3, 0)

(1, 0)

(1, 0)
x

(3, 0) 0

y
(3, 0)
(1, 0)

(0, 6)

36

(1, 0)

(0, 6)

(0, 6)

(3, 0) 0

Maths Quest 12 Mathematical Methods CAS for the TI-Nspire

8 MC

The graph shown is best represented by the equation:

A y = (x
+b
B y = (x a)3 + b
C y = (a x)3 + b
D y = (x + a)3 + b
E y = (x + a)3 + b

a)3

(a, b)
(0, c)
x

9 MC
A y =
B y =
C y =
d y =
E y =

If a < 0 and b, c > 0 then the graph shown is best represented by the equation:
y
b
( x + a ) 2 ( x c)
2
a c
b
( x + a) 2 (c x )
b
a2c
b
( x a ) 2 ( x + c)
a2c
b
a
c
0
( x + a)2 ( x b)(c x )
a2c
b
( x a ) 2 ( x c)
a2c

10 WE26 Sketch the graph of each of the following restricted functions, using the unrestricted
function as a guide. State i the domain and ii the range in each case.
a f(x) = x3 + x2 10x + 8, x [2, )
b f(x) = 3x3 5x2 4x + 4, x [2, 1]
c f(x) = 3x3 + 4x2 + 27x 36, x (0, 1]
d f(x) = 3x x3, x [1, 2)
e f(x) = x3 + 2x, x [2, 1) (0, 3]
f f(x) = 2x3 x, x (1, 1) [2, 3)
11 The function f(x) = x3 + ax2 + bx 64 has x-intercepts (2, 0) and (4, 0). Find the values of a
and b.
12 The functions y = x3 2x2 + ax + 10 and y = 6 + (a + b)x 4x2 x3 both have (1, 0) as an
x-intercept. Find the values of a and b.
13 The cross-section of a glass vessel that is 6 cm high can be modelled by the cubic function f(x)
and its reflection through the y-axis, g(x), as shown at right.
y
f(x) = a(x + b)3 + c
a Find the values of a, b and c, and hence state the rule
g(x)
(4, 6)
of f(x).
b Find the rule for g(x) and state its domain and range.
(3, 3)
c What is the width of the vessel when the height is
3.375 cm?
0

(2, 0)

14 The distance of a group of hikers, d km, from their starting point t hours after setting off on a
hike can be modelled by the function with the rule:
d(t) = at2 (b t)
The hikers are 3 km from the start after 2 hours and return to the starting point after 5hours.
a Find the values of a and b.
b Hence, give the rule for d(t ) stating its domain and range.

Chapter 1 Graphs and polynomials

37

c Sketch the graph of d(t ).


d Find to the nearest 100 metres the maximum distance of the hikers from their starting
point and the time, to the nearest minute, that it occurs.

1G

Quartic graphs
Quartic functions are polynomials of degree 4. The general form of a quartic is:
y = ax4 + bx3 + cx2 + dx + e
When sketching the graphs of quartic functions, all axes intercepts can be found by
factorisation and a sign diagram used to check the shape. If a sign diagram is not sufficient and
the basic shape is not recognised, then a graphics calculator could be used to establish the shape
of the graph.

Basic shapes of quartic graphs


Positive quartics (a > 0)
1. y = ax4

2. y = ax4 + cx2, c 0
y

3. y = ax2(x b)(x c)

4. y = a(x b)2(x c)2


y

c x
0

The repeated factor x2


shows there is a turning
point at the origin. The
factors (x b) (x c)
show x-intercepts at
x = b and x = c.
5. y = a(x b)(x c)3

6. y = a(x b)(x c)(x d )(x e)


y

x
b

The cubed factor (x c)3


shows the graph as a point
of inflection at x = c.
38

The repeated factors


(x b)2 and (x c)2
show the graph touches
the x-axis at x = b and
x = c.

Maths Quest 12 Mathematical Methods CAS for the TI-Nspire

ex

The factors show intercepts at


x = b, c, d and e.

Negative quartics (a < 0)


If a < 0, that is, each of the above rules is multiplied by 1, then the
graphs are reflected through the x-axis.
For example, the graph of y = x4 (at right) is a reflection,
through the x-axis, of the graph of y = x4.
Similarly, the graph of y = x4 + x2 = (x4 x2) is a reflection
through the x-axis of the graph of y = x4 x2.
Note: The above graphs can be translated horizontally or
vertically but this is considered in chapter 2.
To find the x-intercepts of a quartic function, let y = 0 and solve
the equation for x.
Repeated factors touch the x-axis as they do for cubic and
quadratic functions.

y = x 4
y
x

y = x 4 + x2
Worked exAMple 27

Sketch the graph of y =

eBook plus

x4

x3

7 x2

+ 5 x + 10, showing all intercepts.

Tutorial

int-0520

ThINk

Method 1: Technology-free
1 Find the y-intercept.

Let y = P(x).
Find two linear factors of the quartic expression, if
possible, using the factor theorem.

Find the product of the two linear factors.

use long division to divide the quartic by the


quadratic factor x2 x 2 (or use another
method).

Worked example 27

WrITe/drAW

When x = 0, y = 10
The y-intercept is 10.
Let P(x) = x4 x3 7x2 + 5x + 10
P(1) = (1)4 (1)3 7(1)2 + 5(1) + 10
=8
0
P(1) = (1)4 (1)3 7(1)2 + 5(1) + 10
=0
(x + 1) is a factor.
P(2) = (2)4 (2)3 7(2)2 + 5(2) + 10
=0
(x 2) is a factor.
(x + 1)(x 2) = x2 x 2
x2

x2

x4

x3

(x x
4

x2

x2

+ 5 x + 10

2x2 )

0 5 x 2 + 5 x + 10
( 5 x 2 + 5 x + 10)
0
6

Express the quartic in factorised form.

y = (x + 1)(x 2)(x2 5)

Factorise the quadratic factor, x2 5, using


difference of perfect squares.

y = (x + 1)(x 2)(x + 5)(x 5)

To find the x-intercepts, set y equal to zero.

Let y = (x + 1)(x 2)(x + 5)(x 5) = 0

Solve for x using the Null Factor Law.

10

State the x-intercepts.

x = 1, 2, 5
The x-intercepts are 1, 2, 5 and 5.

Chapter 1

Graphs and polynomials

39

11

Sketch the graph of the quartic.

y
(0, 10)
(1, 0)
( 5, 0)
3 2 1 0

(2, 0)
1

( 5, 0)
x
2 3

Method 2: Technology-enabled
1

On a Calculator page, define the polynomial


P(x) by completing the entry line as:
Define p( x ) = x 4 x 3 7 x 2 + 5 x + 10
Then press ENTER .
To find the y-intercept, complete the entry
line as: p(0)
Then press ENTER .
To find the x-intercepts, complete the entry
line as: solve ( p( x ) = 0, x )
Then press ENTER .

To sketch the graph of P(x), open a


Graphs page.
Complete the function entry line as:
f 1( x ) = p( x )
Then press ENTER .

Worked exAMple 28

eBook plus

Sketch the graphs of each of the following equations, showing the coordinates
of all intercepts. Use a CAS calculator to find the coordinates of
the turning points, rounding to 2 decimal places as appropriate.
a y = x 2 ( x 1)( x + 2 )
b y = ( x + 3)2 ( x 1)2
ThINk
a

40

WrITe/drAW

State the function.

Find the y-intercept.

a y = x2(x 1)(x + 2)

When x = 0, y = 0
The y-intercept is 0.

Maths Quest 12 Mathematical Methods CAS for the TI-Nspire

Tutorial

int-0521
Worked example 28

Find the x-intercepts.

When y = 0,
0 = x2(x 1)(x + 2)
x = 2, 0, 1

State the x-intercepts, noting where


the graph touches and where it cuts
the x-axis.

The graph touches the x-axis at x = 0.


The other x-intercepts are 2 and 1.

State the coordinates of the turning


points.

The minimum turning points are (1.44, 2.83) and


(0.69, 0.40).
The maximum turning point is (0, 0).

Sketch the graph of the quartic, using


a CAS calculator to assist.

(2, 0)

(0, 0)
0

(1, 0)
(0.69, 0.40)

(1.44, 2.83)
b

b y = (x + 3)2(x 1)2

State the function.

Find the y-intercept.

When x = 0,
y = (3)2(1)2
= 9
The y-intercept is 9.

Find the x-intercepts.

When y = 0,
0 = (x + 3)2(x 1)2
x = 3, 1

State the points where the graph


touches the x-axis from the repeated
factors.

The graph touches the x-axis at x = 3 and x = 1.

State the coordinates of the turning


points.

The maximum turning points are (3, 0) and (1, 0),


and the minimum turning point is (1, 16).

Sketch the graph of the quartic, using


a graphics calculator to assist.

y
(3, 0)

(1, 0)
0

(0, 9)

(1, 16)

Chapter 1 Graphs and polynomials

41

Worked Example 29

Determine the equation of the graph shown.


y
3
3

1 0

1 2

Think

Write/display

Method 1: Technology-free
1

State the x-intercepts.

The x-intercepts are 3, 1, 1, 2.

Write the equation using factor form with a


dilation factor of a.

y = a(x + 3)(x + 1)(x 1)(x 2)

State the y-intercept.

The y-intercept is 3.

Substitute the coordinates of the point


where the graph crosses the y-axis into the
equation.

(0, 3) 3 = a(0 + 3)(0 + 1)(0 1)(0 2)

Solve the equation to find a.

3=a6
a=

Write the equation.

1
2

y = 12 ( x 1)( x 2)( x + 3)( x + 1)

Method 2: Technology-enabled

42

On a Calculator page, complete the entry line


as:
a ( x + 3) ( x + 1) ( x 1) ( x 2) y
Then press ENTER .
To calculate the value of a, complete the
entry line as:
solve(y = 3, a) | x = 0
Then press ENTER .
To find the equation of y, complete the entry
line as:
1
y|a =
2
Then press ENTER .

Write the equation.

y=

( x 2)( x 1)( x + 1)( x + 3)


2

Maths Quest 12 Mathematical Methods CAS for the TI-Nspire

Worked Example 30

Sketch the graph of y = x 4 2 x 2 , x( 1, 1] , using the unrestricted function as a guide. State the
domain and the range in each case.
Think

Write/Draw

State the function.

y = x4 2x2, x (1, 1]

Find the y-intercept.

When x = 0,
y = (0)4 2(0)2
=0

State the y-intercept.

The y-intercept is 0.

Find the x-intercepts.

When y = 0
x4 2x4 = 0

Factorise the quartic expression.

Solve for x.

x = 0 is the only solution


(as x2 + 2 0).

State the x-intercepts.

The only x-intercept is 0.

Find y when x is one end point of the domain.

When x = 1,
y = (1)4 2(1)2
= 3

State the coordinates and whether it is an open or


closed point.

(1, 3) is an open end point.

10

Find y when x is the other end point of the domain.

When x = 1,
y = (1)4 2(1)2
= 3

11

State the coordinates and whether it is an open or


closed point.

(1, 3) is a closed end point.

12

Sketch the graph of the quartic, using knowledge


of basic shapes or a CAS calculator to assist, over
the domain.

x2(x2

+ 2) = 0

y
(0, 0)

0
(1, 3)

(1, 3)
y = x 4 2x2

13

State the domain, which is given with


the rule.

The domain is (1, 1].

14

From the graph, state the range.

The range is [3, 0].

Chapter 1 Graphs and polynomials

43

REMEMBER

Quartic graphs
1. General equation is y = ax4 + bx3 + cx2 + dx + e.
2. Basic shapes of quartic graphs:
(a) If a > 0:
y = ax4

y = ax4 + cx2, c 0

y = ax2(x b)(x c)

y = a(x b)2(x c)2

y = a(x b)(x c)3

y = a(x b)(x c)(x d)(x e)

c 0

ex

(b) If a < 0, then the reflection through the x-axis of the types of graph in the figures
above is obtained.

44

Maths Quest 12 Mathematical Methods CAS for the TI-Nspire

exerCISe

1G
eBook plus
Digital doc

SkillSHEET 1.8
Solving quartic
equations

Quartic graphs
1 We27 Sketch the graph of each of the following, showing all intercepts.
a y = (x 2)(x + 3)(x 4)(x + 1)
b y = 2x4 + 6x3 16x2 24x + 32
c y = x4 4x2 + 4
d y = 30x 37x2 + 15x3 2x4
e y = 6x4 + 11x3 37x2 36x + 36

eBook plus
Digital doc

Spreadsheet 105
Quartic graphs
factor form

2 We28 Sketch the graph of each of the following equations, showing the coordinates of all
intercepts. use a calculator to find the coordinates of the turning points, rounding to 2 decimal
places as appropriate.
a y = x2 (x 2)(x 3)
b y = (x + 1)2 (x 1)2
c y = (x 1)2(x + 1)(x + 3)
d y = (x + 2)3 (1 x)
3 MC Consider the function f (x) = x4 8x2 + 16.
a When factorised, f (x) is equal to:
A (x + 2)(x 2)(x 1)(x + 4)
C (x + 3)(x 2)(x 1)(x + 1)
E (x 2)2 (x + 2)2
b The graph of f (x) is best represented by:
B
A
y
2

y
16

y
16

16

B (x 1)(x 4)(x + 4)
D (x 2)3 (x + 2)

y
16

y
4

c If the domain of f (x) is restricted to [2, 2], then the range is:

A [0, 16]
D R+

B [0, 10]
E [0, )

C [2, 12]

d If the range of f (x) is restricted to (0, 25) then the maximal domain is:

A [2, 3)

B (2, 3)

C (3, 2)

D (3, 3)

e If the domain of f (x) is restricted to (1, 0), then the range is:

A (0, 16)

B (0, 4)

C (1, 9)

D (9, 16)

f If the domain of f (x) is restricted to [0, ), then the range is:

A R

B R+

C [0, )

D [0, 16)

E (3, 4)

E [9, )
E [2, )

Chapter 1

Graphs and polynomials

45

4 We29 Determinetheequationofeachofthefollowinggraphs.
b
a
y
y
8

2 1 0

1 0

5 We30 Sketch the graph of each of the following restricted functions, using the unrestricted
function as a guide. State i the domain and ii the range in each case.
a y = (2 x)(x2 4)(x + 3), x [2, 3]
b y = 9x4 30x3 + 13x2 + 20x + 4, x (2,1]
c y = (x 2)2(x + 1)2, x (, 2]

d y = 4x2 x4, x [3, 2]

6 The function f (x) = x4 + ax3 4x2 + bx + 6 has x-intercepts (2, 0) and (3, 0). Find the values of
a and b.
7 The function f (x) = x4 + ax3 + bx2 x + 6 has x-intercepts (1, 0) and (3, 0). Find the values of
a and b.
8 The functions y = (a 2b)x4 3x 2 and y = x4 x3 + (a + 5b)x2 5x + 7 both have an
x-intercept of 1. Find the value of a and b.
eBook plus
Digital doc

Investigation
Quartics and
beyond

46

Maths Quest 12 Mathematical Methods CAS for the TI-Nspire

Summary
Pascals triangle

1
1
1
1
1
1

2
3

4
5

1
1
3
6

10

1
4

10

1
5

Binomial theorem

n
n
n
n
(ax + b) n = (ax ) n b 0 + (ax ) n 1 b + .. . +
(ax )b n 1 + (ax )0 b n

1
n
0
n 1
Notes
1. Indices add to n.
2. There are n + 1 terms in the expansion.
n
3. The (r + 1)th term is (ax ) n r br .
r
Polynomials

If P(x) = an xn + an 1 xn 1 + ... + a2 x2 + a1 x + a0 and n is a non-negative integer then P(x) is a polynomial


of degree n and an, an 1, ... , a2, a1 are called coefficients and R.
Remainder theorem:
If P(x) is divided by (x a), then the remainder is P(a). If P(x) is divided by (ax + b) then the remainder is
b
P .
a
Factor theorem:
1. If P(a) = 0, then (x a) is a factor of P(x) or if (ax + b) is a factor of P(x), then
b
P = 0
a
2. If (x a) is a factor of P(x) then a must be a factor of the term independent of x.
Linear graphs

Linear equations are polynomials of degree 1.


General equation is
ax + by + c = 0
or
y = mx + c
wherem = gradient
c = y-intercept
y y1
m= 2
The gradient
x2 x1
Equation if a point and the gradient is known:

y y1 = m(x x1)

Chapter 1 Graphs and polynomials

47

Parallel lines have the same gradient.


If m1 and m2 are the gradients of perpendicular lines, then:

m1 m2 = 1
1
or m1 =
m2
Quadratic graphs

Quadratic equations are polynomials of degree 2.


General equation is
y = ax2 + bx + c
x=

Quadratic formula is

b 2 4 ac
2a

Discriminant = b2 4ac and


1. if b2 4ac > 0, there are 2 x-intercepts (and if b2 4ac is a perfect square, the intercepts are rational)
2. if b2 4ac = 0, there is 1 x-intercept
3. if b2 4ac < 0, there are no x-intercepts.
The power form or turning point form of the quadratic is:

y = a(x b)2 + c
and the turning point is (b, c).
b
The equation of the axis of symmetry and the x-value of the turning point of a parabola is
.
2a
The axis of symmetry is halfway between the x-intercepts.
Cubic graphs

Cubic equations are polynomials of degree 3.


General equation is y = ax3 + bx2 + cx + d
Basic shapes of cubic graphs:
1. If a > 0:
Basic form
Positive cubic
y

y = a(x b)3 + c

(b, c)

Factor form
y

Repeated factor
y

y = a(x b)(x c)(x d)


where a > 0

c d

y = (x a)2 (x b)

2. If a < 0, then the reflections through the x-axis of the types of graph in the above figures are obtained.
Quartic graphs

Quartic equations are polynomials of degree 4.


General equation is y = ax4 + bx3 + cx2 + dx + e
48

Maths Quest 12 Mathematical Methods CAS for the TI-Nspire

Basic shapes of quartic graphs:


1. If a > 0:
y

b
x

y=

y = a(x - b)2(x - c)2

ax4
y

b
x

y=

ax4

cx2,

y = a(x - b)(x - c)3

c0

y = ax2(x - b)(x - c)

e x

y = a(x - b)(x - c)(x - d)(x - e)

2. If a < 0, then reflection through the x-axis of the types of graph above is obtained.
Note: It is possible to translate the cubic and quartic graphs shown in the cubic graphs and quartic graphs
sections above.
Functions

A function is fully defined if the rule and domain are given.


The domain of a function is the set of values of x for which the function is defined.
The range of a function is the set of values of y for which the function is fully defined.
Restricted domains can be represented by interval notation:
[a, b] = {x: a x b} (a, b) = {x: a < x < b} [a, b) = {x: a x < b}

Chapter 1 Graphs and polynomials

49

chapter review
Short answer

1 Expand each of the following:


x 2
b
2 x

a (2y 3x)5

2 If a factor of P(x) = 7 + ax + 5x2 + 15x3 + bx4 is


(x2 1), find the values of a and b.
3 Factorise each of the following expressions:
a x3 12x2 + 17x + 90
b 2x4 + 7x3 31x2 + 36
4 Find the equation of each of the straight lines
described below.
a The line which passes through the
points (5, 6) and (1, 1).
b The line which is perpendicular to the line with
equation 2x y + 10 = 0 and passes through the
point (3, 3).
5 Sketch the graph of y = 8 2x x2, by labelling the
turning point and all intercepts. State its domain
and range.
6 Sketch the graph of y = 3x2 + 8x 3, x [3, 0).
State the range of this function.
7 aIf (x + 3) is a factor of f(x) = x3 + bx2 + ax 18
and g(x) = ax2 + bx 75, then find the values of
a and b.
b Sketch the graph of f(x) by labelling all
intercepts.
8 Sketch the graph of f(x) = x4 7x3 + 12x2 + 4x 16.
Multiple choice

1 When expanded, (1 2x)5 is equal to:


A 1 + 2x 4x2 8x3 + 16x4 + 32x5
B 1 2x + 4x2 8x3 + 16x4 32x5
C 5 10x + 20x2 40x3 + 80x4 160x5
D 1 + 2x 4x2 + 8x3 16x4 + 32x5
E 1 10x + 40x2 80x3 + 80x4 32x5
2 The coefficient of x5 in the expansion of
8
1

is:
4
x

x 2
A 4096
D 16 384

50

B 131 072
E 16 384

C 4096

3 Assuming descending powers of x, the fifth term of


10
1

the expansion of 3 x + is:

x
81
2
A 153 090x
B 243x4
C
x2
81
E 5
D 729x2
x
4 Which of the following expressions is not a
polynomial?
A x3 + 3x 1
B x4 5x3 + 3x2 6x
C x 21 x11 + x 3
3

D x 4 + 5 x 3 2 x 2 + 5 x 3
E x6 x5 + 2x4 x3 + 4x 2
5 The value of P(3) in the polynomial,
P(x) = x5 4x3 3x2 + 10x + 1, is:
B 139
C 191
A 31
D 6
E 1
6 The degree of the polynomial
(5 6x + x3 + 7x6) (x2 3x4 + 2) when expanded is:
A 24
B 8
C 10
D 16
E 21
7 The remainder when x5 + 2x4 + 4x3 5x + 3 is
divided by (x + 3) is:
B 51
C 171
A 271
D 3
E 108
8 For which one of the following polynomial
expressions is (x 2) not a factor?
A x3 + 3x2 4x 12
B x4 2x3 6x2 8x + 2
C x4 + 2x3 7x2 8x + 12
D x3 + x2 10x + 8
E 2x3 + 3x2 9x 10
9 Which one of the following is a
factor of 2x4 4x3 10x2 + 12x?
A (x 2)
B (x + 3)
C (x + 1)
D (x 4)
E (x 3)
10 The rule for the graph shown is:
A 2x + y + 4 = 0
y
B x 2y 4 = 0
2
C 2y x 4 = 0
D x + 2y 4 = 0
0
E 4x + 2y = 0

Maths Quest 12 Mathematical Methods CAS for the TI-Nspire

Questions 11 and 12 refer to the graph below, which


has a gradient of 2.
11 The value of b must be:
A 5
B 3
C 1
D 1
E 4

y
(2, b)
0

1 0 2
2

(3, 5)

12 The y-intercept is:


A (0, 3)
B (0, 2)
1
D (0, 1)
C (0, 2 )
1
E ( 2 , 0)

D 2
3
E 76
Questions 14 and 15 refer to the function with the
rule: y = 2x2 + 8x - 10 where x (6, 2).

15 The range of this function is:


A (18, 14)
B (10, 14)

D [ 18, 14]
E (14, 10)

(2, 14)

x
x

(2, 14)

0 12
10

(2, 14)

E
65

C [18, 14)

16 The graph of y = 3x3 could be:


B
A
y
y

10

(6, 14)

10

C
65

(2, 10)

0 12

65

14 Which one of the following graphs could represent


this function?
(6, 14)

x
(2, 3.6)

(6, 6)

13 If 3x2 + 4x 5 = 0, then the value of the


discriminant is:
A 76
B 44
C 44

(6, 22)

(6, 14)

17 Which of the following intercepts does the graph of


2 1 0
10

56

f(x) = 6 + 11x + 3x2 2x3 have?


A ( 12 , 0), (2, 0), (3, 0) and (0, 6)
B (2, 0), (2, 0), (3, 0) and (0, 6)
C ( 12 , 0), (2, 0), (3, 0) and (0, 6)

D (2, 0), (1, 0), (3, 0) and (0, 6)


E ( 12 , 0), (3, 0), (2, 0) and (0, 6)

Chapter 1 Graphs and polynomials

51

18 The rule for the graph shown below could be:


A f(x) = (x 1)2(x + 3)
y
f(x)
B f(x) = (x + 1)(x 3)2
C f(x) = (x + 1)2(3 x)
x
D f(x) = (x2 1)(x + 3)
3
1 0
E f(x) = (x 3)(x + 1)2

D
19 The rule for the graph shown below could be:
A f(x) = x(x + 2)3
y
f(x)
B f(x) = x(x 2)2
C f(x) = x2(x 2)2
D f(x) = x(x 2)3
E f(x) = x(2 x)2
0

0 1

21 The graph with equation y = x2 is translated 3 units


down and 2 units to the right. The resulting graph
has the equation:
A y = (x - 3)2 + 2
B y = (x - 2)2 + 3
C y = (x - 2)2 - 3
D y = (x + 2)2 - 3
E y = (x + 2)2 + 3
[ VCAA 2006]

Extended response

1 An empty parfait glass has been left on a table with the rim just touching
a wall. Ants are marching in a line down into the parfait glass and then up
the other side, following the path of a parabola. They begin their journey
where the glass touches the wall, 18 cm above the table.
a The stem of the glass is 4cm long and the diameter of the top of the
glass is 5cm. Find the rule for the quadratic function that describes
the shape of the glass.
b State the domain and range of the function.
c If there is fruit juice in the bottom of the glass to a depth of 1 cm,
find the coordinates of the point where the ants first touch the juice.
Round answers to the nearest whole number.
d Using function notation, write the rule for the surface of the crosssection of the juice in the glass.

52

20 The graph of y = (x + 3)2(x 1)(x 3) is best


represented by:
A

Maths Quest 12 Mathematical Methods CAS for the TI-Nspire

2 A rogue satellite has its distance from Earth, d thousand kilometres, modelled by a cubic function of time,
t days after launch. After 1 day it reaches a maximum distance from Earth of 4000 kilometres, then after
2 days it is 2000 kilometres away. It effectively returns
to Earth after 3 days, then moves further and further
away.
a What is the satellites initial distance from Earth?
b Sketch the graph of d versus t for the first 6 days
of travel.
c Express d as a function of t.
d The moon is approximately 240000 kilometres
from Earth. Which is closer to Earth after 8 days,
the satellite or the moon? By how far?
e The satellite is programmed to self-destruct.
This happens when it is 490000 kilometres
from Earth. What is the life span of
the satellite?
f State the domain and range of d(t).
3 A bridge spans a narrow canal as shown in the diagram at right.
a Find the equation of a parabola that models the shape of the archway.
b Show that a barge 3 m wide and carrying cargo with a total height of 1.7 m
(with a rectangular cross-section) cannot fit under the arch.
c How much cargo (in height, correct to 1 decimal place) must be removed
for the barge to fit under the bridge?

y
3
2
1
Barge

x
4 In the town of Newtonia there is an annual 10m race (for the Polynomial Cup)
2 1 0 1 2
for mini robots that have been programmed with mathematical formulas by
Surface of canal
Professor Liebnitz. There is a lot of betting on the race as the professor keeps
the formulas secret and is known to favour surprise winners.
The three contestants were programmed as follows, where x is the distance from the start line in metres
and t is the time in minutes:

Liney
x = 2.4 + 0.75t

Quadder
x = 0.2t(t - 5.1)

Cubric
x = 0.2t(t - 5.1)(t - 9.1)
Using a CAS calculator, describe the motions of the three contestants, specifically:
a the direction they travelled in and how fast they were moving
b where and when they changed direction
c where and when they passed or met each other
d who won the race and by how much.
Sketch the graphs of their movements on the same set of axes, labelling all relevant points. You will need an
extra graph to get a close-up of the finish.

5 The diagram at right shows a main road passing through O, A, C and E. The road crosses a river at point O
and 3 kilometres further along the road at point C. Between O and C, the furthest the river is from the road
is 8.54 kilometres, at a point D, 2.25 kilometres east of a northsouth line
through O. Point A is 1 kilometre east of point O. If point O is taken as the
N
origin and the road as the x-axis, then the path of the river can be modelled
River
W
E
by a quartic function, as shown in blue.
S
a Give the coordinates of C and D.
O A
C Main road
b Find the rule for the quartic function, f(x).
E
c How far is the river from the main road along the track AB?
d A canoeing race, of at least 17 kilometres in length, along the river is
B
being organised. It is suggested that the race could start at O and finish
D
at C. Is this course satisfactory? Why?

Chapter 1 Graphs and polynomials

53

6 Willie Wonkie, of Willie Wonkies Construction Company, makes a sketch of the symmetrical W for a large
neon sign as shown below. The x- and y-axes represent the supporting crosspieces. The width of the W along
the x-axis is 6 metres and the point on the vertical support is 21 metres above the horizontal support. The W
4
can be modelled by a quartic function, with all x-intercepts exactly evenly spaced.
y

a
b
c
d

Find the rule for the letter W.


If the top of the W is 8 metres wide, find the coordinates of the highest points of the letter.
State the domain of the function.
Use a graphics calculator to find the coordinates of the lowest points of the W, giving values correct to
3 decimal places. Hence find the range of the function.
e In order to test the strength of his design, Willie Wonkie moves the horizontal crosspiece so that it just
touches the lowest points of the W. Find the new rule that describes the W now.
f State the domain and range of the new function.
Note: The following questions use differentiation of polynomials.
7 A plane cruising at 10000 m is coming in to land at an airport at sea level, as can be seen in the diagram below.
y
Planes

10 km

flight path
50 km

10 000 m
x

Airport

If the plane descends smoothly and makes no changes in direction, show that a possible model would be
y = ax2(x - b).
a Find the equation if the plane begins its descent when 50km horizontally from the airport.
b What is the altitude of the plane when it is 2km horizontally from the airport?
c How accurate do you think this model is?

54

Maths Quest 12 Mathematical Methods CAS for the TI-Nspire

8 The diagram below shows a symmetrical skateboard ramp with horizontal platforms at A and B, and vertical
supportingstrutsatCandD.
y
A

B
E

G
2m

4m

x
3m

D
2m
4m

a Write an equation for a quartic function that models the ramp, assuming a smooth connection at
A and B.
b Show that the right half of the ramp can be modelled by a cubic equation y = a(x b) (x 4)2 and find its
equation by evaluating a and b.
c The right-hand side can also be modelled by two smoothly connected parabolas.
i IfthestrutDFis1mlong,findtheequationofthelowerparabolapassingthroughF.
ii Find the equation of the upper parabola if it meets the lower one at F, and show that the connection is
not smooth (that is, their gradients are not equal at the point where they meet).
iii Show that the two parabolas meet smoothly at (3, 0.75) provided the lower parabola passes
through F.
eBook plus
d Which model is the closest to the actual ramp if the strut is really 1.6 m long?
Digital doc

Test Yourself
Chapter 1

Chapter 1

Graphs and polynomials

55

eBook plus

ACTIvITIeS

Chapter opener
Digital doc

10 Quick Questions: Warm up with ten quick


questions on graphs and polynomials. (page 1)
1A

The binomial theorem

Tutorial

We 3 int-0516: Watch a worked example on


binomial expansion. (page 4)
Digital doc

SkillSHEET 1.1: Practise binomial expansions using


Pascals triangle. (page 6)
1B

Polynomials

Digital docs

Spreadsheet 043: Investigate evaluating


polynomials. (page 10)
SkillSHEET 1.2: Practise solving simultaneous
equations. (page 10)
1C

Division of polynomials

Interactivity

Divisionofpolynomialsint-0246: Consolidate your


understanding of the division of polynomials and
rational functions. (page 11)
Tutorial

We 11 int-0517: Watch a worked example on the


division of polynomials. (page 11)
Digital docs

Spreadsheet 096: Investigate finding factors of


polynomials. (page 15)
WorkSHEET 1.1: Binomial expansion, division of
polynomials and solving and factorising polynomial
equations (page 15)
1D

Linear graphs

Digital docs

SkillSHEET 1.3: Practise calculating the gradient of


parallel and perpendicular lines. (page 20)
SkillSHEET 1.4: Practise using the gradient to find
the value of a parameter. (page 20)
SkillSHEET 1.5: Practise using interval notation.
(page 21)
SkillSHEET 1.6: Practise finding the domain and
range for linear graphs. (page 21)
1E

Quadratic graphs

Tutorial

We 17 int-0518: Watch a worked example on using


the discriminant. (page 22)
Digital docs

Spreadsheet 103: Investigate the value of the


disciminant. (page 28)
Spreadsheet 107: Investigate quadratic graphs.
(page 28)
56

SkillSHEET 1.7: Practise recognising domain and


range for quadratic graphs. (page 28)
Spreadsheet 041: Investigate graphs of functions.
(page 28)
Spreadsheet 108: Investigate quadratic graphs in
turning point form. (page 28)
WorkSHEET 1.2: Calculate gradients, axial
intercepts and values of the discriminant, sketch
graphs of polynomials, and determine equations for
graphs. (page 29)
1F

Cubic graphs

Tutorial

We 24 int-0519: Watch a worked example on


determining the rule of a cubic. (page 31)
Digital docs

Spreadsheet 013: Investigate cubic graphs in factor


form. (page 34)
Spreadsheet 014: Investigate cubic graphs. (page 35)
Spreadsheet 015: Investigate cubic graphs of the
form y = a(x b)3 + c. (page 36)
1G

Quartic graphs

Tutorials

We 27 int-0520: Watch a worked example on


sketching the graph of a quartic. (page 39)
We 28 int-0521: Watch a worked example on
finding the turning points of a quartic using a CAS
calculator. (page 40)
Digital docs

SkillSHEET 1.8: Practise solving quartic equations.


(page 45)
Spreadsheet 105: Investigate quartic graphs in factor
form. (page 45)
Investigation: Quartics and beyond (page 46)
Chapter review
Digital doc

Test Yourself Chapter 1: Take the end-of-chapter test


to test your progress. (page 55)
To access eBookPLUS activities, log on to
www.jacplus.com.au

Maths Quest 12 Mathematical Methods CAS for the TI-Nspire

2
Functions and
transformations
AREAS OF STUDY

The behaviour of functions of a single real


variable, including key features of their graphs
such as axis intercepts, stationary points
and points of inflection, domain (including
maximal domain) and range, asymptotic
behaviour, symmetry and the algebra of
functions, including composition of functions.
The behaviour of these functions is linked to
applications in practical situations.
Identification of key features of graphs of:
power functions y = xn for n N and n = 1,
2, 1 and transformations of these to the
2
form y = a(x + b)n + c where a, b and c R
the modulus function y = | x |
Transformations from y = f (x) to
y = Af (n(x + b)) + c (where A, n, b and
c R and f is one of the functions specified
above) and the relation between the original
function and the graph of the transformed

2A
2B
2C
2D
2E
2F
2G
2H
2I

Transformations and the parabola


The cubic function in power form
The power function (the hyperbola)
The power function (the truncus)
The square root function in power form
The absolute value function
Transformations with matrices
Sum, difference and product functions
Composite functions and functional
equations
2J Modelling

function (including families of transformed


functions for a single transformation parameter)
Graphs of sum, difference, product and
composite functions of f and g, where f and g
are functions of the types above
Recognition of the general form of possible
models for data presented in graphical or
tabular form, using polynomial and power
functions
Applications of simple combinations of the
above functions, and interpretation of features
of the graphs of these functions in modelling
practical situations
The relationship of f (x y), f (xy) and f to values
of f (x) and f (y) for different functions f
Composition of functions, where f composition g
is defined by f (g(x)), given rang domf ; for
4
example x 2 + 5 , x 3 + 2
x 1
eBook plus

2A

Transformations and
the parabola

Digital doc

10 Quick Questions

Transformations
In this chapter we consider the basic graphs of the quadratic and cubic functions, the hyperbola
and truncus, square root and absolute value functions.
The following transformations of the above graphs are discussed: dilation, reflection and
translation.

Dilation
A dilation is the stretching or compressing of a graph.
Let the basic graph be y = f (x).

Chapter 2

Functions and transformations

57

Dilation away from the x-axis: y = af (x


(x)
1. Stretches or compresses the graph f (x) by a factor of a from the x-axis.
2. Each y-value of the basic graph is multiplied by a factor of a, that is (x, y) (x, ay).
3. When | a | > 1, the graph of f (x) is stretched and becomes narrower.
4. When 0 < | a | < 1, the graph of f (x) is compressed and becomes wider.
Dilation away from the y-axis: y = f (nx)
1. Stretches or compresses the graph f (x) by a factor of 1 from the y-axis.
n
x
2. Each x-value of the basic graph is multiplied by a factor of 1 , that is (x, y) ( , y).
n
n
3. When |n| > 1, the graph of f (x) is compressed from the y-axis and becomes narrower.
4. When 0 < |n| < 1, the graph of f (x) is stretched from the y-axis and becomes wider.
Note: For the graphs we will be looking at in this chapter a horizontal dilation can be expressed
as a vertical dilation. For example, (2
(2x + 1)3 can be written as 23 ( x + 12 )3 = 8(
8( x + 12 )3. So in this
1
case a horizontal dilation from the y-axis by a factor of 2 is the same as a vertical dilation from
the x-axis by a factor of 8. This can simplify the process of describing transformations for these
particular graphs.
The concept of dilation is illustrated in the following diagram:
y

Original graph dilated


from the x-axis

Original graph dilated


from the y-axis

Original graph

Ref lection
Reflection provides a mirror image of a graph.
Reflection can take place in one or both axes.
Let the basic graph again be y = f (x).
Ref lection in the x-axis: y = f (x
(x)
1. The mirror image of the original graph appears across the x-axis (the mirror line).
2. Each y-value is the negative of the original, the x-value is unchanged, that is (x, y) (x, y).
Reflection in the y-axis: y = f (x)
1. The mirror image of the original graph appears across the y-axis (the mirror line).
2. Each x-value is the negative of the original, the y-value is unchanged, that is (x, y) (x, y).
Reflection in both axes: y = f (x)
1. The basic graph is reflected in the x-axis and then the y-axis (or vice versa).
2. Both the x- and y-values are the negatives of the original, that is (x, y) (x, y).
The concept of reflection is shown in the diagram below. The red star is the original graph.
y

Reflection in the y-axis

Reflection in the x-axis

58

Maths Quest 12 Mathematical Methods CAS for the TI-Nspire

Reflection in both axes

Translation: y = f (x b) + c
A translation slides the graph. Translation can be horizontal (to the right or left along the x-axis),
or vertical (up or down along the y-axis).
Consider our basic graph y = f (x).
1. If y = f (x b) the basic graph is translated b units parallel to the x-axis:
(a) in the positive direction (i.e. to the right) when b > 0
(b) in the negative direction (i.e. to the left) when b < 0.
Each x-value has b added to it, that is (x, y) (x + b, y).
2. If y = f (x) + c, the basic graph is translated c units parallel to the y-axis:
(a) in the positive direction (i.e. up) when c > 0,
(b) in the negative direction (i.e. down) when c < 0.
Each y-value has c added to it, that is (x, y) (x, y + c).
3. If y = f (x b) + c the basic graph is translated both horizontally and vertically.
y

x
x
Original graph

x
Vertical translation up
y

Vertical translation down

Horizontal translation
to the left

Horizontal translation
to the right

Naturally, the graph can be subject to a combination of two or more transformations.


eBoo
k plus
eBook
Digital doc

Spreadsheet 132
Transformations

Combination of transformations
When describing transformations that have been applied to a basic graph f (x), it is best to put
the graph into the format y = af (x b) + c. The order of transformations is important as dilations
and reflections are applied before translations, so ensure that you describe the transformations in
this order (remember D-R-T).
In this chapter we shall consider graphs, derived from basic curves, using single
transformations dilations, reflections or translations as well as combinations of those.
Modelling of data will also be considered.

The quadratic function in power form


The graph of y = x2 is a parabola with the turning point at the origin.
The domain of the function is R and the range is R+ {0}.
Throughout this section we refer to the graph of y = x2 as the basic
parabola. Let us now consider the effect of various transformations on
the graph of this basic parabola.

Chapter 2

Functions and transformations

59

Quadratic functions are also power functions. Power functions are functions of the form
f (x) = xn, n R. The value of the power, n, determines the type of function. When n = 1,
f (x) = x, and the function is linear. When n = 2, f (x) = x2 and the
a=2
function is quadratic. Other power functions will be discussed later.
y
a=1 1
a = 2
Under a sequence of transformations of f (x) = xn, n R, the
general form of a power function, is f (x) = a(x b)n + c (where a, b,
c and n R). All linear and quadratic polynomials are also linear and
x
quadratic power functions, because all linear and quadratic functions
0
are transformations of f (x) = x and f (x) = x2, respectively.
When a quadratic function is written in turning point form it
is written in power form. For example, the quadratic function
y = ax2
y = x2 + 4x + 6 can also be represented as the power function
y = (x + 2)2 + 2.

Dilation
In power form, a is the dilation factor. It dilates the graph in the y direction. The larger | a | is,
the thinner the graph of the parabola. If | a | is a proper fraction, that is, 0 < | a | < 1, the graph is
wider than the basic parabola.

Reflection
If a is negative, the graph of the basic parabola is reflected in
the x-axis, that is, the graph is flipped upside down.
If x is replaced with x, the graph of the basic parabola is
reflected in the y-axis, that is, the graph is flipped sideways.
Due to its symmetry, this effect cannot be seen on the basic
parabola, but it is more obvious with a parabola that has
already been translated. For example, the graphs of y = (x 3)2
and y = (x 3)2 are reflections of each other across the y-axis.

y = (x 3)2 y

y = (x 3)2
(0, 9)

(3, 0)0 (3, 0)

Translation
Horizontal translation
If b > 0, the graph of the basic parabola is translated horizontally to
the right, and if b < 0, the graph of the basic parabola is translated
horizontally to the left. For example, a graph with the equation
y = (x 2)2 is a basic parabola that has been translated 2 units to the
right, and a graph with the equation y = (x + 3)2 is a basic parabola that
has been translated 3 units to the left.
If the coefficient of x is not 1, the equation must be rewritten in the
form y = a(x b)2 + c in order to be able to work out the value of b.
For example, y = (4x + 3)2 is translated 43 of a unit to the left, since

y
b=

0
3
2
y = (x b)

b=2

y = (4 x + 3)2
3
= [4( x + 4 )]2
3

= 16( x + 4 )2
Vertical translation
If c > 0, the graph is translated vertically upward, and if c < 0, the
graph is translated vertically downward. For example, the graph with
equation y = x2 + 2 is a basic parabola that has been translated 2 units
up, and the graph with equation y = x2 1 is a basic parabola that has
been translated 1 unit down.

60

Maths Quest 12 Mathematical Methods CAS for the TI-Nspire

c=2
c = 1

2
x
0
1 y = x2 + c

eBoo
k plus
eBook
Digital doc

Spreadsheet 108
The quadratic
function
in power form

Combination of transformations

y y = a(x b)2 + c

The graph of y = a(x


+ c shows the combination of the
transformations shown above.
The turning point of the graph is (b, c).
The domain of the parabola is R and the range is [c, ) if a > 0 or
[, c) if a < 0.
b)2

(b, c)
0

WORKED EXAMPLE 1

State the changes required to transform the graph of y = x2 into the graph of y = 2(x
2( + 3)2 4.
THINK

WRITE

Write the general formula for the parabola.

y = a(x b)2 + c

Identify the value of a.

a=2

State the effect of a on the graph.

The graph of y = x2 is dilated by the factor


of 2 from the x-axis.

Identify the value of b.

b = 3

State the effect of b on the graph.

The graph is translated 3 units to the left.

Identify the value of c.

c = 4

State the effect of c on the graph.

The graph is translated 4 units down.

We can use transformations to find the equation of the function from its graph by first examining
the new position of the turning point.
WORKED EXAMPLE 2

Use transformations to find the equation of this function.

(4, 2)

0
THINK

WRITE

Write the general formula of the parabola.

y = a(x b)2 + c

From the graph state the horizontal translation and


hence the value of b.

Translated 4 units to the right, so b = 4.

From the graph, state the vertical translation and


hence the value of c.

Translated 2 units up, so c = 2.

Substitute the values of b and c into the general formula. y = a(x 4)2 + 2

The graph of the parabola passes through the origin.


Substitute x = 0 and y = 0 into the formula.

Using (0, 0):


0 = a(0 4)2 + 2

Solve for a, which is the dilation factor.

0 = 16a + 2
16a = 2
2
a = 16
=

1
8

Chapter 2

Functions and transformations

61

Substitute the value of a into y = a(x 4)2 + 2 and


write your answer.

The equation of the parabola shown is:


y = 81 ( x 4)2 + 2

WORKED EXAMPLE 3

Given the equation y = kx2, determine the effect on the graph y = x2, when k = {2, 3, 4}. Sketch the graphs.
THINK

WRITE/DISPLAY

On a Graphs page, complete the


function entry line as:
f 1(x) = x2
Then press ENTER .

Complete the function entry lines as:


f 2(x) = 2x2
f 3(x) = 3x2
f 4(x) = 4x2
Press ENTER after each entry.

Answer the question by describing the


changes in words.

As the value of k increases the graph becomes thinner


and stretches away from the x-axis.

REMEMBER

1. The graph of y = x2 is called a basic parabola.


2. The graph of y = a(x b)2 + c is the basic parabola, dilated
by the factor of a from the x-axis, translated b units horizontally
(to the right if b > 0 or to the left if b < 0) and c units vertically
(up if c > 0 or down if c < 0).
3. If a is negative, the graph is reflected in the x-axis.
4. If x is replaced with x, the graph is reflected in the y-axis.
5. The turning point of the parabola is (b, c).
6. The domain of the parabola is R.
7. The range is y c if a > 0 or y c if a < 0.

62

Maths Quest 12 Mathematical Methods CAS for the TI-Nspire

y y = a(x b)2 + c

(b, c)
0

EXERCISE

2A

Transformations and the parabola


1 WE 1
State the changes required to transform the graph of y = x2 into the graph of each
of the following.
a y = 2x
b y = 13 x 2
2
c
e
g
i
k

y = 3x2
1
y = 1 2 x2
y = (x + 3)2
y = (x + 2)2 1
y = 1 2(3 + x)2

y = x2 6

d
f
h
j
l

y = (x 2)2
y = 2(3 x)2
y = (x 0.5)2 + 2
1

y = 3(2 x 3)2 4

1
2 MC The equation of a parabola is given by
(2 x )2 + 3 . Increasing m will result in the
m
graph being:
A translated further to the left
B translated further up
C thinner
D wider
E reflected in the y-axis

3 Match the graphs of the parabolas with the following equations.


a y = x2 + 2
b y = 2(xx 2)2
c

y = 2 (x + 2)2

e y = (2 + x)2 + 2

iii

ii
y = x2

y = 2 (2 x ) 2

iv

0 2
2
2
v

4 WE2 Use transformations to find the equation of each function.


a

(2, 2)

y
0

(1, 2)
2)

(1, 3)

2 x

y
4

(2, 4)

5 MC The equation of the graph shown opposite is best given by:


A y = (x c)2 + d
B y = c (x b)2

2
C y = (x + c) + b
D y = (c x)2 + d
2
E y = d (x c)

y
d
b
a0

6 Find the equation of the image of y = x2 under each of the following


transformations:
a dilation by the factor of 12 from the x-axis
b a reflection in the x-axis
c a translation by 2 units to the right and 1 unit down

Chapter 2

Functions and transformations

63

d a dilation by the factor of 3 from the x-axis, followed by translation of


2 units down
e a reflection in the x-axis, followed by translation of 3 units to the left.
7 Find the equations of these graphs.
a

y
1

0
1

y
3

y
2

x
1 0

0 2

y
8

9
6
0

2 2

0 x

8 WE3 Find the equation of y = x2 under the following sequential transformations


(in order):
a dilation by a factor of 2 from the x-axis
b reflection in the x-axis
c translation of 1 parallel to the x-axis
d translation of 3 parallel to the y-axis.
9 Find the image of the point ((x, y) under each of the following transformations:
a reflection in the y-axis
b reflection in the x-axis
c dilation by a factor of 3 from the x-axis
d dilation by a factor of 2 from the y-axis
e dilation by a factor of 13 from the y-axis
f translation of 2 units horizontally in the positive direction
g translation of 1 unit parallel to the y-axis.
10

The parabola has a turning point at (z,


( 8); it intersects the y-axis at y = 10 and one of the
x-intercepts is x = 5. Find:
a the value of z
b the equation of the parabola.

11 For the parabola whose range is y 3, whose x-coordinate of the turning point is 4 and whose
1
y-intercept is y = 2 3 , find:
a the y-coordinate of the turning point
b the equation of the parabola
c the coordinates of the x-intercepts.
64

Maths Quest 12 Mathematical Methods CAS for the TI-Nspire

12 The design shown in the diagram at right can be obtained by taking


the red portion of the parabola and transforming it to form each of
the other 9 fragments. (One or more transformations may be used to
form each fragment.) If the highlighted fragment is given by f (x
(x),
2 x 2, define the other 9 fragments in terms of f (x
(x) and specify
their domains.

2B

3
2 0

The cubic function in power form


The graph of the function y = x3 is shown at right: Both the domain and
y
range of the function are R. The function is constantly increasing and
has a stationary point of inflection (where the gradient is 0) at the origin
(0, 0).
x
0
Throughout this section we shall refer to the shape of the graph of
3
y = x as a positive cubic, or a basic cubic curve.
Cubic functions are also power functions. Power functions are
functions of the form f (x) = xn, n R. The value of the power, n,
determines the type of function. When n = 1, f (x) = x and the function
is linear. When n = 2, f (x) = x2 and the function is quadratic. When n = 3, f (x) = x3 and the
function is cubic. When n = 4, f (x) = x4 and the function is quartic. Other power functions will
be discussed later.
Under a sequence of transformations of f (x) = xn, n R, the general form of a power
function is f (x) = a(x b)n + c (where a, b, c and n R). All linear and quadratic polynomials
are also linear and quadratic power functions, but this is not the case for cubic functions
(or quartic functions). For example, a cubic power function in the form f (x) = a(x b)3 + c has
exactly one x-intercept and one stationary point of inflection. A cubic polynomial in the form
f (x) = ax3 + bx2 + cx + d can have one, two or three x-intercepts and is therefore not a power
function.
All cubic power functions are also cubic polynomials, but not all cubic polynomials are cubic
power functions. For example, the cubic function y = 2(x 3)3 + 1 is a polynomial and a power
function. It is the graph of y = x3 under a sequence of transformations.

Dilation

a=2
a=1
a = 12

The value a is the dilation factor; it dilates the graph from the x-axis.
The larger a is, the thinner the graph.
0

y = ax3
eBoo
k plus
eBook
Digital doc

Spreadsheet 015
Cubic function
y = a( x b ) 3 + c

y = (x 1)3 y

Reflection
If a is negative, the graph of the basic cubic is reflected in the
x-axis, that is, the graph is flipped upside down.
If x is replaced with x, the graph of the basic cubic is
reflected in the y-axis, that is, the graph is flipped sideways.
For example, the graphs y = (x 1)3 and y = (x 1)3 are
reflections of each other across the y-axis.

Chapter 2

(1, 0) 0

y = (x 1)3

(1, 0)

(0, 1)

Functions and transformations

65

Translation
Horizontal translation
If b > 0, the graph of the basic cubic is translated horizontally to
the right, and if b < 0, the graph of the basic cubic is translated
horizontally to the left. For example, the graph with equation
y = (x 2)3 is a basic cubic translated 2 units to the right, and the
graph of y = (x + 3)3 is a basic cubic, translated 3 units to the left, that
is, parallel to the x-axis in the negative direction.
If the coefficient of x is not 1, the equation must be rewritten in
the form y = a(x b)3 + c in order to be able to work out the value of
b. For example, the graph of y = (2x
(2 5)3 is translated 25 units to the
3
right, since y = (2x
(2 5)

b = 3

b=2

y = (x b)3

= [2( x 25 )]3
= 8( x 25 )3
Vertical translation
The value of c translates the graph vertically or along the y-axis. If
c > 0, the graph is translated vertically up, and if c < 0, the graph is
translated vertically down.
The coordinates of the stationary point of inflection are (b, c).
For example, if y = x3 is translated 1 unit up, the equation of the
resulting graph is y = x3 + 1 and the point of inflection is (0, 1); if
it is translated 2 units down, the equation of the resulting graph is
y = x3 2 and the point of inflection is (0, 2).

Combination of transformations
The graph of y = a(x b)3 + c shows the combination of the
transformations described above.
Finally, the domain and range of y = a(x b)3 + c are R (all
real numbers).

y
1

c=1
c = 2
x

0
22

y = x3 + c

y = a(x b)3 + c

(b, c)
0

WORKED EXAMPLE 4

State the changes necessary to transform the graph of y = x3 into the graph of y = 2(x
2( + 1)3 4.
THINK

66

WRITE

Write the general equation of the cubic function.

y = a(x b)3 + c

Identify the value of a.

a=2

State the effect of a on the graph.

The graph is dilated by the factor of 2 in


the y direction.

Identify the value of b.

b = 1

State the effect of b on the graph.

The graph is translated 1 unit to the left.

Identify the value of c.

c = 4

State the effect of c on the graph.

The graph is translated 4 units down.

Maths Quest 12 Mathematical Methods CAS for the TI-Nspire

WORKED EXAMPLE 5

For each of the following graphs:


i state the coordinates of the stationary point of inflection
ii find the x- and y-intercepts
iii sketch the graph
iv state the transformations that the graph of y = x3 has undergone to form each new equation.
a y = (x
( + 3)3 1
b y = (4 x)3 + 6
THINK
a Write the equation.

WRITE/DRAW
a y = (x + 3)3 1
i b = 3, c = 1

i Since the rule is of the form y = a(x b)3 + c,

Stationary point of inflection: (3, 1)

identify the values of b and c and hence write


the coordinates of the stationary point of
inflection (b, c).

ii Find the y-intercept by letting x = 0.

i i y-intercept: x = 0,

y = (0 + 3)3 1
= 27 1
= 28
x-intercept: y = 0
(x + 3)3 1 = 0
(x + 3)3 = 1
x + 3 = 1
x = 4

Find the x-intercept by letting y = 0.

iii To sketch the graph on a set of labelled axes,

iii

mark the stationary point of inflection and the


x- and y-intercepts, then sketch the positive
cubic passing through the points marked.

(4, 0)
0
(3, 1)

iv State the kind of reflection and the vertical and

Write the equation.


i Since the rule is of the form y = a(x b)3 + c,

identify the values of b and c and hence write the


coordinates of the stationary point of inflection
(b, c).

ii Find the y-intercept by letting x = 0.

Find the x-intercept by letting y = 0.


Note: Do not round off until the very last step;
for graphing purposes, round off your final
answer to 1 decimal place.

i v The graph is reflected in the x-axis.

horizontal translations.

There is a horizontal translation


of 3 units to the left and a vertical
translation of 1 unit down.
b

y = (4 x)3 + 6
i b = 4, c = 6

Stationary point of inflection: (4, 6)

i i y-intercept: x = 0,

y = (4 0)3 + 6
= 64 + 6 = 70
x-intercept: y = 0
(4 x)3 + 6 = 0
(4 x)3 = 6
4x=

6
3

x=4+ 6
5.8

Chapter 2

Functions and transformations

67

iii To sketch the graph on a set of labelled

iii

axes, mark the stationary point of inflection and


the x- and y-intercepts, then sketch the positive
cubic passing through the points marked.

(4, 6)

0
iv State the kind of reflection and the vertical

(5.8, 0)

i v The graph is reflected in the y-axis.

and horizontal translations.

There is a horizontal translation of


4 units to the right and a vertical
translation of 6 units up.

To find the equation of the curve from a given graph, we need to establish exactly what
transformations were applied to the basic cubic curve. This is best done by observing the shape
of the graph and the position of the stationary point of inflection.
WORKED EXAMPLE 6

Find the equation of the curve, if it is of the form y = a(x


( b)3 + c.
(x

y
5
3

THINK

Write the general equation of the cubic function.

y = a(x b)3 + c

Write the coordinates of the stationary point of


inflection (b, c) and hence state the values of b
and c.
Substitute the values of b and c into the general
formula.

The stationary point of inflection is (1, 3).


So b = 1, c = 3.

The graph passes through the point (0, 5)


(y-intercept). Substitute the coordinates of this
point into the equation.

Using (0, 5):


5 = a(0 1)3 + 3

On a Calculator page, press:


MENU b
3:Algebra 3
1:Solve 1
Complete the entry line as:
solve(5 = a (0 1)3 + 3,a)
Then press ENTER .

Write the solution for the equation.

a = 2

Substitute the value of a into y = a(x 1)3 + 3.

y = 2(x 1)3 + 3

68

WRITE/DISPLAY

y = a(x 1)3 + 3

Maths Quest 12 Mathematical Methods CAS for the TI-Nspire

REMEMBER

1. The graph of y = x3 is a basic cubic curve.


2. The graph of y = a(x b)3 + c where a > 0 is the basic cubic, dilated by the factor of a
from the x-axis, translated b units along the x-axis (to the right if b > 0, or to the left if
b < 0) and c units along the y-axis (up if c > 0, or down if c < 0).
y = a(x b)3 + c

(b, c)

3. If a < 0, the graph is reflected in the x-axis.


4. The stationary point of inflection is at (b, c).
5. Both the domain and range are R.
EXERCISE

2B

The cubic function in power form


1 WE4
State the changes necessary to transform the graph of y = x3 into the graph of each
of the following.
b y=

a y = 7x3
c
e
g
i

y = x3 + 4
y = (x 1)3
y = 4(2 x)3
y = 3(x + 3)3 2

k y = 1 (2 x + 5)3
4
2

2
3

x3

d
f
h
j

y = 6 x3
y = (x + 3)3
y = 6(7 x)3
1
y = 6 2 ( x 1)3

y = 3 2(4 + 2 x )3

Which of these transformations were applied to the graph of y = x3 to obtain each of the
graphs below?
i reflection in the x-axis
i i translation to the left
i i i translation to the right
i v translation up
v translation down
a

y
x

0
x

Chapter 2

Functions and transformations

69

3 WE5 For each of the following graphs:


i find the stationary point of inflection
ii find the x- and y-intercepts
iii state the transformation(s) that
the graph y = x3 has undergone to
produce the given graph
iv sketch the graph.

EXAM TIP
Be careful when sketching
graphs use appropriate scales on the axes, clearly
draw the graph and use the correct domain. Show at
least one scale point on each axis and two coordinate
points on each curve (including intercepts).
[Authors advice]

eBoo
k plus
eBook

a y = 3 x3
4

b y = 1 2x
2 3

c y = 2 x3 6
3

d y = 2(x 4)3

f y = 4(1 x)3

e y=

( x 2)3

g y = (x 1)3 + 2
i y = 2(x + 1)3 6

Digital doc

Spreadsheet 236
Function grapher

h y = 3 (x + 2)3

Questions 4 to 6 refer to the function y = 2(mx 4)3 3.


4 MC The coordinates of the stationary point of inflection are:

A ( 4 , 3)
m

4 3
C ( , )
m m

B (4m, 3)

4
E ( , 3)
m

4
D ( , 3m3 )
m

5 MC The graph of y = x3 is dilated in the y direction by the factor of:


A 2

C 2m3

B 2m

2
m

2
m3

6 MC If m > 1, increasing m will cause the graph to become:


A wider
B thinner and translated not as far to the right
C shifted further to the left
D shifted further to the right
E shifted further down
7 Find the equation of the graph resulting from each of the following transformations of the
graph of y = x3:
a a dilation by the factor of 12 from the x-axis
b a reflection in the x-axis and a translation by 5 units to the left
c a translation by 3 units to the right and 1 unit down
d a dilation in the y direction by the factor of 2, followed by the vertical translation
of 3 units
e a reflection in the x-axis, then a translation of 1 unit to the left and 1 unit down.
8 Find the equation of the graph resulting from the following sequential transformations of the
graph of y = x3:
a dilation by a factor of 2 from the x-axis
b reflection in the y-axis
c translation of 2 in the positive direction parallel to the x-axis
d translation of 1 in the negative direction parallel to the y-axis.
9 WE6 Find the equations of these curves, if they are of the form y = a(x
( b)3 + c.
(x
a

(1, 2)
1

70

Maths Quest 12 Mathematical Methods CAS for the TI-Nspire

x
0

y
12
1 1 0
2

10 MC The graph of y = 2(x


2( + 3)3 + 1 has been reflected in the x-axis, shifted 3 units to the right
and 1 unit up. The equation of the resulting graph is:
A y = 2(3 x)3 + 2
B y = 2(x + 2)3 + 2
C y = 2(2 x)3
D y = 2(x 3)3 + 1
E y = 2x
2 3
11 The graph of a cubic function of the form y = a(x
( b)3 + c has a stationary point of inflection
(x
at (1, 4) and cuts the y-axis at y = 2. Find the equation of the function.
12

The graph of y = a(b x)3 + c has a stationary point of inflection at (2, 1) and passes
through the point (1, 12 ).
a Find the equation of the curve.
b State the shape of the curve (that is, whether it is positive or negative cubic).

13 The graph of y = a(x


( b)3 + c cuts the x-axis at x = 4 and the y-axis at y = 28. If it is known
(x
that the dilation factor is equal to 1:
a find the position of the stationary point of inflection
b sketch the graph.

2C

The power function (the hyperbola)


The graph shown at right is called a hyperbola and is given by the
y
equation y = 1 .
x
Power functions are functions of the form f (x) = xn, n R. The
y=0
value of the power, n, determines the type of function. We saw earlier
x
0
that when n = 1, f (x) = x and the function is linear. When n = 2,
f (x) = x2 and the function is quadratic. When n = 3, f (x) = x3 and the
function is cubic. When n = 4, f (x) = x4 and the function is quartic.
x=0
The power function that produces the graph of a hyperbola has a
1
value of n = 1. Thus, the function f ( x ) = can also be expressed as
x
the power function f (x) = x1.
The graph exhibits asymptotic behaviour. That is, as x becomes very large, the graph
approaches the x-axis, but never touches it, and as x becomes very small (approaches 0), the
graph approaches the y-axis, but never touches it. So the line x = 0 (the y-axis) is a vertical
asymptote and the line y = 0 (the x-axis) is the horizontal asymptote. Both the domain and the
range of the function are all real numbers, except 0; that is, R\{0}.
The graph of y = 1 can be subject to a number of transformations.
x
a

Consider y =
+ c or y = a(x b) 1 + c
xb

Dilation

The value a is a dilation factor. It dilates the graph from the


x-axis.

y=0
0

Reflection
If a is negative, the graph of the basic hyperbola is reflected in the
x-axis. If x is replaced with x, the graph of the basic hyperbola is
reflected in the y-axis.

Chapter 2

x=0

a=2
a=1
a = 12
x
y = ax

Functions and transformations

71

1
1
and y =
x3
x3
are reflections of each other across the y-axis.
For example, the graphs of y =

x = 3
y=

Translation

x=3
y = x 1 3

x 3

Horizontal translation
x
y=0
The value b translates the graph b units horizontally,
that is, parallel to the x-axis. If b > 0, the graph is
translated to the right, and if b < 0, the graph is
(0, 13 )
translated to the left. For example, the graph with
1
equation y =
is a basic hyperbola translated
x3
3 units to the right. This graph has a vertical asymptote of x = 3 and domain R\{3} (and a
horizontal asymptote y = 0). If a basic hyperbola is translated 3 units to the left, it becomes
1
y=
, with a vertical asymptote of x = 3 and domain R\{3}. Hence, the equation of the
x+3
vertical asymptote is x = b and the domain is R\{b}. The horizontal asymptote and the range
remain the same, x = 0 and R \ {0}, respectively.
Vertical translation
The value c translates the graph c units vertically, that is, parallel to the y-axis. If c > 0, the graph
is translated upward, and if c < 0, the graph is translated c units downward. The graph
1
with equation y = + 3 is a basic hyperbola translated 3 units up. This graph has a horizontal
x
asymptote of y = 3 and a range of R\{3} (and a vertical asymptote x = 0). If a basic hyperbola is
1
translated 3 units down, it becomes y = 3, with a horizontal asymptote of y = 3 and a range
x
of R\{3} (and a vertical asymptote x = 0). Hence the equation of the horizontal asymptote is
y = c and the range is R\{c}.
Always draw the asymptote as a dotted line and label it with its equation (for example, y = 3)
at the end of the asymptote. Ensure that the graph continues to approach the asymptote getting
closer but not touching or crossing the asymptote or bouncing away from the asymptote.

Combination of transformations
The graph of y =

a
+ c shows the combination of these transformations.
xb
y

eBoo
k plus
eBook
Digital doc

Spreadsheet 051
The hyperbola

y=c

y = x
c
b+

c
0

x=b

Finally, if the coefficient of x is a number other than 1, to obtain the value of h the equation
should be rearranged first. For example,
4
4
y=
=
3 x + 6 3( x + 2)
Therefore, b = 2 (not 6 as it may seem at first); that is, the graph is translated 2 units to
the left.

72

Maths Quest 12 Mathematical Methods CAS for the TI-Nspire

WORKED EXAMPLE 7

1
State the changes that should be made to the graph of y = in order to obtain the graph of
x

y = 4 1.
x+2
THINK

WRITE

a
+c
xb

Write the general equation of the hyperbola.

y=

Identify the value of a.

a = 4

1
State the changes to y = , caused by a.
x

1
The graph of y = x is dilated by the
factor of 4 from the x-axis and reflected in
the x-axis.

Identify the value of b.

b = 2

State the effect of b on the graph.

The graph is translated 2 units to the left.

Identify the value of c.

c = 1

State the changes to the graph, caused by c.

The graph is translated 1 unit down.

WORKED EXAMPLE 8

For the graph of y =

2
+ 2, state:
x3

a the equations of the asymptotes


c the range.

b the domain

THINK
a

WRITE

Write the general equation of the hyperbola.

Identify the values of b and c and hence


write the equations of the asymptotes:
Horizontal asymptote: y = c
Vertical asymptote: x = b

a y=

a
+c
xb

b = 3, c = 2
Horizontal asymptote: y = 2
Vertical asymptote: x = 3

State the domain of the hyperbola: R\{b}.

b Domain: R\{3}

State the range of the hyperbola: R\{c}.

c Range: R\{2}

Sketching the graph of the hyperbola by hand can be easily done by following these steps:
1. Find the position of the asymptotes.
2. Find the values of the intercepts with the axes.
3. Decide whether the hyperbola is positive or negative.
4. On the set of axes draw the asymptotes (using dotted lines) and mark the intercepts
with the axes.
5. Treating the asymptotes as the new set of axes, sketch either the positive or negative
hyperbola, making sure it passes through the intercepts that have been previously marked.

Chapter 2

Functions and transformations

73

WORKED EXAMPLE 9

eBoo
k plus
eBook

2 4
, clearly showing the intercepts
x+2
with the axes and the position of the asymptotes.
Sketch the graph of y =
THINK
1

a
+c
Compare the given equation with y =
x

b
and state the values of a, b and c.
Write a short statement about the effects
1
of a, b and c on the graph of y = .
x

Write the equations of the asymptotes.


The horizontal asymptote is at y = c.
The vertical asymptote is at x = b.
Find the value of the y-intercept by
letting x = 0.

int-0522
Worked example 9

WRITE/DRAW

Tutorial

Find the value of the x-intercept by


making y = 0.

b = 2,

a = 2,

c = 4

1
is dilated by the factor of
x
2 from the x-axis, translated 2 units to the left
and 4 units down.
Asymptotes: x = 2; y = 4
The graph of y =

y-intercept: x = 0
2
y=
4
0+2
= 1 4
= 3
Point (0, 3)
x-intercept: y = 0
2
0=
4
x+2
2
=4
x+2
2 = 4( x + 2)
= 4x + 8
4x = 2 8
= 6
x=
=
Point

6
4
3
2

3
2

,0

To sketch the graph:

a Draw the set of axes and label them.


b Use dotted lines to draw the asymptotes. The

( 32 , 0)
0

asymptotes are x = 2 and y = 4.

(0, 3)

c Mark the intercepts with the axes. The

intercepts are y = 3 and x =

3
.
2

d Treating the asymptotes as your new set of

x
y = 4

x = 2

axes, sketch the graph of the hyperbola (as a


is positive, the graph is not reflected); make
sure the upper branch passes through the x- and
y-intercepts previously marked.
The next example shows how to find the equation of the hyperbola from its graph.

74

Maths Quest 12 Mathematical Methods CAS for the TI-Nspire

WORKED EXAMPLE 10

Find the equation of the graph shown.

y
6
3
0 2

THINK

WRITE

a
+c
xb

Write the general equation of the hyperbola.

y=

From the graph, identify the values of b and c


Remember that the equation of the horizontal
asymptote is y = c and of the vertical asymptote is
x = b.

b = 2,

Substitute the values of b and c into the formula.

y=

Substitute the coordinates of any of the 2 known


points of intersection with the axes into the formula
(say, x-intercept).

Solve for a.

Substitute (4, 0):


a
0=
+3
42
a
0= +3
2
a
= 3
2
a = 6

Substitute the value of a into y =

Transpose (optional).

a
+ 3.
x2

y=

c=3

a
+3
x2

6
+3
x2

y = 3

6
x2

REMEMBER

1
is called a hyperbola.
x
a
2. The graph of y =
+ c is the graph of the basic
xb
hyperbola, dilated by the factor of a from the
x-axis, translated b units horizontally (to the right if
b > 0, or to the left if b < 0) and c units vertically (up if
c > 0, or down if c < 0). If a < 0, the graph is reflected
in the x-axis. The equations of the asymptotes are:
x = b and y = c. The domain of the function is R\{b} and its
range is R\{c}.
1. The graph of y =

Chapter 2

y= a

xb

+ c
y=c
x

0
x=b

Functions and transformations

75

EXERCISE

2C

The power function (the hyperbola)


1

1
State the changes that should be made to the graph of y = in order to obtain the
x
graph of each of the following.
3
1
2
a y=
b y=
c y=
x
x6
x
2
1
2
d y=
e y= +7
f y= 5
x+4
x
x
4
1
2
g y=
3
+6
h y=
i y=
4
4+x
x3
x 1
WE7

1
Which of the following transformations were applied to the graph of y = to obtain each
x
of the graphs shown below?
i translation to the right
i i translation to the left
iii translation up
i v translation down
v reflection in the x-axis
a

WE8 For each of the following, state:

i the equations of the asymptotes

a y=

76

i i the domain

i i i the range.

b y=

1
x+6

c y=

d y=

2
3 x

e y=

3
+4
x

g y=

4
2
x+6

h y=

5
+1
2 x

Maths Quest 12 Mathematical Methods CAS for the TI-Nspire

3
x2

1
5
x
1
y=
m
n+ x
y=

4 For each of the following graphs, state:


i the equations of the asymptotes
i i the domain
i i i the range.
a

y
2

y
1
0

2
0

n
0

a
m

eBoo
k plus
eBook

4
1
2
3
5 On the same set of axes sketch the graphs of y = , y = , y = and y =
.
x
x
3x
3x

Digital doc

Spreadsheet 236
Function
grapher

WE9 Sketch each of the following, clearly showing the position of the asymptotes and the
intercepts with the axes. Check your answers, using a CAS calculator.
a y= 1
b y = 1 1
c y= 3 3
x+3
x+2
x 1 4
2

6
d y=
e y=
f y = 3 +6
3
1 x
x+5
x2
1
2
4
g y = 1
h y= +
i y= 1 +4
2 x
5 1+ x
2x + 3
2
x
+
3
j y=
k y=
l y = 4x + 3
1
x2
x 1
3 4x
MC The equation of the graph shown is likely to be:

1
x4
1
C y = 3
4x
A y = 3+

E
8

y = 3

1
+4
x3
1
D y=
3
4x

B y=

3
0

1
x4

MC Which of the following is a true statement for the graph of y =

A
B
C
D
E

The domain is R\{1}.


The range is R\{3}.
The equation of the horizontal asymptote is y = 3.
The equation of the vertical asymptote is x = 2.
None of the above.

Chapter 2

2
3?
x +1

Functions and transformations

77

9 WE10 Find the equation for each of the following hyperbolas, if they are of the form
a
y=
+ c.
xb
a

eBoo
k plus
eBook
Digital docs

WorkSHEET 2.1
History of
mathematics
The history of
some major
curves

2D

10

y
2
112
0

34

y
5

3 4

11
1

1
If a function is given by f ( x ) = , sketch each of the following, labelling the asymptotes
x
and the intercepts with the axes.
a f (x + 2)
b f (x) 1
c f (x) 2

d f (1 x) + 2
e f (x 1) 1
f 1 f (x 2)

11 Sketch the graph of yx 3x + 1 = 0, and


state its domain and range. (Hint: First
transpose the equation to make y the
subject.)

EXAM TIP
When you are asked to give the
domain and range in a question, be sure to
demonstrate clearly which answer is which. For
example, for y = x2, dom f : R, ran f : [0, ].
[Authors advice]

The power function (the truncus)


The graph shown at right is known as a truncus. The equation of
the graph is given by:

y = 12
x
Power functions are functions of the form f (x) = xn, n R.
y=0
x
0
The value of the power, n, determines the type of function. We
saw earlier that when n = 1, f (x) = x and the function is linear.
x=0
When n = 2, f (x) = x2 and the function is quadratic. When n = 3,
f (x) = x3 and the function is cubic. When n = 4, f (x) = x4 and
the function is quartic. When n = 1, f (x) = x1 and the power function produces the graph of a
hyperbola.
The power function that produces the graph of a truncus has a value of n = 2. Thus, the
function f ( x ) = 12 can also be expressed as the power function f (x) = x2.
x
The function is undefined for x = 0. Hence, the equation of the vertical asymptote is x = 0 and
the domain of the function is R\{0}.

78

Maths Quest 12 Mathematical Methods CAS for the TI-Nspire

We can also observe that the graph approaches the x-axis very closely, but never touches it. So
y = 0 is the horizontal asymptote.
Since the whole graph of the truncus is above the x-axis, its range is R+ (that is, all positive
real numbers).
1
Similar to the graphs of the functions, discussed in the previous sections, the graph of y = 2
x
can undergo various transformations.
a
+ c , or y = a (x b)2 + c.
Consider the general formula y =
( x b) 2

Dilation

The value a is the dilation factor. It dilates the graph from


the x-axis. The dilation factor does not affect the domain,
range or asymptotes.

y=0

a=3
a=2
x

x=0
y

Reflection
If a is negative, the graph of a basic truncus is
reflected in the x-axis. The range becomes R (that
is, all negative real numbers).

a
y = 2
x

y = 12
x

y=0

y = 12

If x is replaced with x, the graph of the basic


truncus is reflected in the y-axis. The effect of this
reflection cannot be seen in the basic graph, but
it becomes more obvious if the graph has been
translated horizontally first. For example,
1
1
the graphs of y =
and y =
( x 3)2
( x 3)2
are reflections across the y-axis. The vertical
asymptote changes from x = 3 to x = 3 and the
domain changes from R \{3} to R \{3}.

x=0
y
1
( x 3)2

1
y=
( x 3)2

y=

y=0

x = 3

x=3

Translation
Horizontal translation
The value b translates the graph b units horizontally. If b > 0,
the graph is translated to the right, and if b < 0, the graph is
translated left. For example, the graph of the equation
1
results from translating a basic truncus 3 units
y=
( x 3)2
to the right. The vertical asymptote is x = 3

y
b = 2
y=0

b=3
0

1
y =
2
(x b)

Chapter 2

Functions and transformations

79

and the domain is R \{3}. If a basic truncus is translated 2 units to the left, it becomes
1
y=
, where the vertical asymptote is x = 2 and the domain is R \{2}. Hence, the
( x + 2)2
equation of the vertical asymptote is x = b and the domain is R \{b}. The range is still R+ and the
equation of the horizontal asymptote is y = 0.
Vertical translation
y
1
The value c translates the graph c units vertically. If c > 0 the graph
y =
+c
x2
is translated upward, and if c < 0, the graph is translated c units
1
c=1
downward. For example, the graph with equation y = 2 + 1 results
x
y=1
1
when a basic truncus is translated 1 unit upward. The horizontal
0
c = 1 x
asymptote is y = 1 and the range is (1, ). If a basic truncus is
1
1
y = 1
translated 1 unit down, it becomes y = 2 1, with y = 1 as
x=0
x
the horizontal asymptote and (1, ) as the range. Hence the
equation of the horizontal asymptote is y = c and the range is (c, ).
Note: If a is positive (see graph below), the whole graph of the truncus is above the line y = c
(the horizontal asymptote) and hence its range is y > c, (c, ).
If a is negative, the whole graph is below its horizontal asymptote and therefore the range is
y < 0, or (, c).
y

y
y=3
y=3

3
0

The graph of y =

x
x

a
+ c shows the combination of these
( x b) 2

y=

a
+ c
(x b)2

transformations.
y=c

c
0

x=b
WORKED EXAMPLE 11

State the transformations required to change the graph of y =


THINK

80

1
1
1.
into the graph of y =
2
( x 2 )2
x

WRITE

a
+c
( x b) 2

Write the general formula for the truncus.

y=

Identify the value of a.

a = 1

Maths Quest 12 Mathematical Methods CAS for the TI-Nspire

State the effect of a on the graph.

The graph of y =

Identify the value of b.

b=2

State the effect of b on the graph.

The graph is translated 2 units to the right.

Identify the value of c.

c = 1

State the effect of c on the graph.

The graph is translated 1 unit down.

x2

is reflected in the x-axis.

WORKED EXAMPLE 12

2
4 , state:
( 3 + x) 2
a the equations of the asymptotes
For the function y =

b the domain

THINK
a

b
c

c the range.
WRITE

a y=

a
+c
( x b) 2

Write the general formula for the truncus.

Write the general equations of the asymptotes.

Vertical asymptote: x = b
Horizontal asymptote: y = c

Identify the values of b and c.

b = 3, c = 4

State the equations of the asymptotes by


substituting the values of b and c into
corresponding formulas.

Asymptotes: x = 3 and y = 4

Write the domain of the truncus, which is


R\{b}.
1

Check whether a is positive or negative.

Write the range (which for a > 0 is y > c).

b Domain: R\{3}
c a>0

Range: y > 4

a
+ c, then compare the given
( x b) 2
equation with the general formula to see what changes should be made to the basic curve (the
1
graph of y = ) to transform it to the one you want. This should give you an idea of how the
x2
graph will look.
To sketch the graph of a truncus, first put it in the form y =

The following algorithm can then be used:


1. Find the position of the asymptotes.
2. Find the intercepts with the axes.
3. On the set of axes, draw the asymptotes
(using dotted lines), label with the
equation and mark the x- and
y-intercepts.
a>0
4. Treating the asymptotes as the new set of
axes, sketch the basic truncus curve.
5. Make sure the curve passes through the points marked on the axes.

Chapter 2

a<0

Functions and transformations

81

WORKED EXAMPLE 13

1
, clearly showing the position
( x + 1)2
of the asymptotes and the intercepts with the axes (correct to 1 decimal place).
Sketch the graph of y = 2

THINK

WRITE/DRAW

a
+c
( x b) 2

Write the general formula for the truncus.

y=

Identify the values of a, b and c.

a = 1,

Write a short statement about the transformations


1
the graph of y = 2 should undergo in order to be
x
changed into the one in question.

The graph of y =

Write the equations of the asymptotes


(y = c and x = b).

Asymptotes: x = 1 and y = 2

Find the x-intercept (round off to 1 decimal place).

x-intercept: y = 0

b = 1,

1
is reflected in the
x2
x-axis, translated 1 unit to the left and
2 units up.

0 = 2
1
= 2
( x + 1)2
( x + 1)2 = 12
x =
1
2

x =
.

Find the y-intercept.

To sketch the graph: draw the set of axes and label


them; use dotted lines to draw asymptotes; mark the
x- and y-intercepts; treating the asymptotes as the new
set of axes, draw the basic truncus curve upside down
(since a is negative); make sure it intersects the axes
in the right places.

1
( x + 1)2

1
2
1
2

x +1 =

c=2

1 or x =

0.3

1
2

1.7

y-intercept: x = 0
1
y = 2
(0 + 1)2
= 2 1
=1
y

y=2

(1.3,
1.3, 0)
(0, 1)
0

(1.7, 0)

x = 1

In the above example we have considered sketching the graph from the given equation.
Sometimes the opposite task is required; that is, the equation of the function should be
established from its graph.

82

Maths Quest 12 Mathematical Methods CAS for the TI-Nspire

WORKED EXAMPLE 14

Find the equation of the curve shown in this diagram.

x=2

12

THINK

WRITE

y=

a
+c
( x b) 2

Write the general equation of the truncus.

Compared to y =

b = 2 and c = 0

Substitute the values of b and c into the formula.

y=

Substitute the coordinates of the y-intercept into the


formula.

Using (0, 2 ) :

1
, the graph is shifted 2 units to
x2
the right. (There is no shift along the y-axis.)

a
+0
( x 2)2
a
=
( x 2)2

1
a
=
2 (0 2)2
a
=
4
1
a = 4 ( 2)

Solve for a.

a = 2

Substitute for a in the equation.

y=

( x 2)2

REMEMBER

1. The graph of y =
2.

3.
4.
5.
6.

x2

is called a truncus.

a
+ c is the basic truncus curve,
The graph of y =
( x b) 2
dilated by a factor of a from the x-axis and translated
b units along the x-axis (to the right if b > 0 or to the
left if b < 0) and c units along the y-axis (up if c > 0
or down if c < 0). If a is negative, the graph is reflected
in the x-axis.
The vertical asymptote is x = b.
The horizontal asymptote is y = c.
The domain is R\{b}.
The range is y > c if a > 0, or y < c if a < 0.

Chapter 2

y=

a
(x b)2 + c

c
0

Functions and transformations

83

EXERCISE

2D

The power function (the truncus)


1
1 WE11
State the transformations required to change the graph of y = 2 into the graph of
x
each of the following:

1
2
a y=
b y= 3
c y=
2
2
( x + 2)2
x
x
d y=

2
( x 3)2

e y=

(4 +

x )2

4
+1
( x 3)2
1
2 MC To obtain the graph shown, the graph of y = 2 was:
x
A reflected in the x-axis and translated 2 units down
B translated 2 units to the left
C reflected in the x-axis and translated 2 units to the left
D reflected in the x-axis and translated 2 units to the right
E reflected in the x-axis and translated 2 units up
g y = 3 1
x2

h y=

2
+6
x2

y=

y = 5

1
( x + 2)2
y

x=2

3 MC Which of the following translations took place, so that the


1
graph of y = 2 was changed into the one shown at right?
x
A m units to the left and p units up
B m units to the right and p units up
C m units to the left and n units up
D m units to the right and n units up
E m units to the left and n units down

y
p
y=n
x
x=m

4 WE12 For each of the following state:


i the equations of the asymptotes
ii the domain
iii the range.

a y= 2
b y= 4
x2
3x 2
5
2
d y=
e
y
=
( x + 1)2
(4 + x ) 2
4 1
1 3
g y= + 2
h y= 2
5 x
2 x

c y=
f
i

1
( x 2)2

2
3
x2
2
y=
+4
( x 1)2
y=

Questions 5 t 7 refer to the following diagrams.


i

ii

iii

3
0

84

Maths Quest 12 Mathematical Methods CAS for the TI-Nspire

iv

Digital doc
Function
grapher

0
3

Spreadsheet 236

eBoo
k plus
eBook

vi

5 MC Which of the above functions has the domain R\{3}?


A i i only
B i i i only
D i i , i i i and v i
E v and vi

i i and i i i

6 MC Which of the above functions has the range y < 3?


A i , i i and i v
B i i i , v and vi
E i i i and vi
D v only

i v only

7 MC Which of the graphs has asymptotes y = 0 and x = 3?


A i
B ii
D iv
E v

iii

8 WE13 Sketch each of the following, clearly showing the position of the asymptotes and the
intercepts with the axes (correct to 1 decimal place where appropriate).

1
2
a y= 2
b y=
c y=
2
2
( x 3)
(4 + x ) 2
5x
d y=

( x 1)2

2
2
( x 1)2
3
1
y=

2
4
4( x + 1)

g y=
j

e y = 1 4
x2

2
(3 + x )2
1
+3
k y=
(2 x ) 2

h y = 4

9 Find the equations for each of the following


curves, if it is known that all of them are
a
of the form y =
+ c and a is
( x b) 2
+

either 1 or 1.
a

1
3
2x2
2
1
y= +
3 ( x 2)2
4
y=
1
(2 x 4) 2
y=

EXAM TIP
Always check you have met the
requirements of the answer format, for example,
ensure that you have rounded correctly and show the
correct number of decimal places.
[Assessment report 1 2006 VCAA]

p
q

n
m

cc
d
a
0

t
0

Chapter 2

f
f

Functions and transformations

85

y
x

0
l
i

k
[ VCAA 2006]

10 WE14 Find the equation for each of the following.


a
b
y
y

y=

34

x y=0

y=4
x

2.5

y = 3

y
0
2

11

y=1
x

x = 2

x=

x=2

1 0

y
1

x=0

x=4

y = 2

7
x=1

The domain of a truncus is R\{2}; its range is y > 3 and its graph cuts the x-axis at
1 and x = 3. Find the equation of the function.

12 The domain of a truncus is R\{1}; its range is (2, ) and its graph cuts the y-axis at
y = 5. Find the equation of the function.

2E

The square root function


in power form
The square root function is given by y = x (or y = x ).
y
Power functions are functions of the form f (x) = xn, n R. The
value of the power, n, determines the type of function. We saw earlier
that when n = 1, f (x) = x and the function is linear. When n = 2,
f (x) = x2 and the function is quadratic. When n = 3, f (x) = x3 and the
x
0
function is cubic. When n = 4, f (x) = x4 and the function is quartic.
When n = 1, f (x) = x1 and the power function produces the graph of
a hyperbola. When n = 2, f (x) = x2 and the power function produces the graph of a truncus.
1
The power function that produces the graph of the square root function has a value of n = .
2
1
2

Thus, the function f ( x ) = x can also be expressed as the power function f ( x ) = f ( x ) = x .


The function is defined for x 0; that is, the domain is R+ {0}, or [0, ).
1
2

86

Maths Quest 12 Mathematical Methods CAS for the TI-Nspire

As can be seen from the graph, the range of the square root function is also R+ {0}, or [0, ).
Throughout this section we will refer to the graph of y = x as the basic square root curve.
Let us now investigate the effects of various transformations on the basic square root curve.
a( x b) + c.
Consider the function y = a x b + c, or y = a(
1
2

Dilation

a=3

a=2

The value a is a dilation factor; it dilates the graph from the x-axis.
The domain is still [0, ).

a=1
a = 12
0

y=a x

Reflection
If a is negative, the graph of a basic square root curve is
reflected in the x-axis. The range becomes (, 0]. The
domain is still [0, ).

y
y=
x
(1, 1)

(0, 0)

(1, 1)
y =
x

If x is replaced with x, the graph is reflected in the y-axis.


For example, the graphs with equations y = x and
y = x are reflected across the y-axis. The domain
becomes (, 0] and the range is [0, ).

y
y = x

y=
x

(1, 1)

(1, 1)
x

(0, 0)

Translation
Horizontal translation
The value b translates the graph horizontally. If b > 0, the graph is
translated to the right, and if b < 0, the graph is translated to
the left. The graph with the equation y = x 3 results when the
basic curve is translated 3 units to the right. This translated graph
has domain [3, ) and range [0, ). If the basic curve is translated
2 units to the left, it becomes y = x + 2 and has domain [2, )
and range [0, ). The domain of a square root function after a
translation is given by [b, ).
Vertical translation
The value c translates the graph vertically. If c > 0, the graph is
translated vertically up, and if c < 0, the graph is translated
vertically down. If y = x is translated 2 units vertically up, the
graph obtained is y = x + 2, with domain [0, ) and range [2, ).
If the basic curve is translated 4 units down, it becomes y = x 4,
with domain [0, ) and range [4, ).
The range of the square root function is [c, ) for a > 0.

Chapter 2

y
b = 2
(2, 0) 0

(3, 0)

b=3
x

y= xb

y
(0, 2)
0
(0, 4)

c=2

c = 4

y= x+c

Functions and transformations

87

Combination of transformations

The graph of y = a x b + c shows the combination of these


transformations.
The point (b, c) is the end point of the square root curve. For
example, the end point of y = x 2 + 1 is (2, 1).
It is always good practice to label the end point with its
coordinates. Make sure it is an open circle if the x-value is
not in the required domain and a closed circle if its x-value
is within the functions domain.
Consider the function y = a b x + c.
The graph of y = a x + b + c has (b, c) as its end
point. If this function is reflected in the y-axis, it becomes
y = a x + b + c with end point (b, c). The equation
y=a

y=a xb+c

(b, c)
x

y=a bx +c y=a x+b +c

+ b + c can then be rewritten as


(b, c)

y = a b x + c.

(b, c)
x

For example, the graph of y = 2 x + 1 can be


x

rewritten as y =

+ 2 + 1, which has an

end point of (2, 1) and bends to the left. The domain is (, 2] and the range is [1, ). The
equation y = x + 2 + 1 results in y =
changes from

[2,

) to

(,

be rewritten as y = 2( x +

3
)
2

+ 2 + 1 when it is reflected in the y-axis. The domain

2] and the range remains [1, ). The equation y = 2 x + 3 1 can

1;
1 the domain is [ 2 , ) and the range is [1, ).

WORKED EXAMPLE 15

State the transformations required to change y = x to y = 3 x + 5 + 3.


THINK

WRITE

Write the general formula for the square root curve.

y=a xb +c

Identify the value of a.

a = 3

State the effect of a on the graph.

The graph is dilated by a factor of 3 from the


x-axis and reflected in the x-axis.

Identify the value of b.

b = 5

State the effect of b on the graph.

The graph is translated 5 units to the left.

Identify the value of c.

c=3

State the effect of c on the graph.

The graph is translated 3 units up.

WORKED EXAMPLE 16

eBoo
k plus
eBook

For each of the following functions find the domain and range.

88

y = 2 x 3 +1

y= 4 x +2

y = 4 3x + 2 4

Maths Quest 12 Mathematical Methods CAS for the TI-Nspire

Tutorial

int-0523
Worked example 16

THINK

WRITE

Write the general formula.

y=a xb +c

a y = 2 x 3 +1

Write the question.

Identify the values of b and c.

b = 3, c = 1

State the domain x b.

The domain is [3, ).

State the range (y c for a > 0).

The range is [1, ).


b y = 4 3x + 2 4

Write the question.

Factorise the expression under the square root


sign.

y = 4 3( x + 23 ) 4

State the domain.

).
The domain is [ 3 , )

Identify the value of c and check whether a is


positive or negative.

c = 4, a < 0

State the range.

The range is (, 4].

Write the question.

2
3

c y=

4 x +2

Identify the values of b and c.

b = 4, c = 2

Since the function is of the form

The domain is (, 4].

y = a b x + c, the domain is x b.
4

State the range (y c).

The range is [2, ).

To sketch the graph of the square root function, we


a>0
a>0
need to compare the given formula with
y=a bx+c
y=a xb+c
y = a x b + c. This will give us an idea of the
(b, c)
a<0
a<0
changes required to transform the basic square root
curve into the one we want. It will also let us know the
way the curve will look. The diagram above illustrates
the idea.
Once the coordinates of the end point and the direction of the curve are known, the intercepts
with the axes (if any) should be found before sketching.

WORKED EXAMPLE 17

Sketch the graph of y = 2 x 3 + 1 , clearly marking intercepts and the end points.
THINK

WRITE/DRAW

Write the equation.

y = 2 x 3 + 1

Write the coordinates of the end point.

End point: (3, 1)

State the shape of the graph.

Shape:

Chapter 2

Functions and transformations

89

Find the x-intercept by letting y = 0.

x-intercept: y = 0
0 = 2 x 3 +1
2 x3 =1
1
x3 = 2
1

x 3 = ( 2 )2
=

1
4

x =34
5
6

Find the y-intercept if there is one.


Sketch the graph by plotting the end point, marking
the x-intercept, and drawing the curve so that it starts
at the end point and passes through the x-intercept.

There is no y-intercept.
y
1
0

(3, 1)
(3 14 , 0)
x
3

WORKED EXAMPLE 18

Given f : [0, ) R, where f ( x ) = x and g(x


( ) = af (x
(x
(x) + b, where a and b are positive real constants,
consider the effect on g(x
( ) as a and b increase individually.
(x
THINK
1

WRITE

On a Graphs page, complete the


function entry line as:
f 1( x ) = v1 x + v 2
Insert a slider by pressing:
MENU b
1:Actions 1
A:Insert Slider A
Then press ENTER .
Repeat to insert a second slider.
Grab the slider for each variable and move it
back and forth taking note of the effect of each
variable increasing.

Write your description in words.

As a increases, the graph is dilated away from the


x-axis, with the graph stretched further from the x-axis.
As b increases, the graph is translated up parallel
to the y-axis.

REMEMBER

1. The graph of the function y = a x b + c is the graph

2.
3.
4.
5.

of y = x,
x dilated by the factor of a from the x-axis and
translated b units along the x-axis and c units along the y-axis.
If a < 0, the basic graph is reflected in the x-axis.
The end point of the graph is (b, c).
The domain is x b.
The range is y c for a > 0, or y c for a < 0.

y=a xb+c

(b, c)
0

6. If y = a b x + c, the domain is x b; the graph of y = a x is reflected in the y-axis.

90

Maths Quest 12 Mathematical Methods CAS for the TI-Nspire

EXERCISE

2E

The square root function in power form


State the transformations required to change y = x to each of the following.

1 WE15
a y=

b y=

1
3

y = 3 x 1

d y = 2 4 + x

e y = x 1

y = 23 x

g y = x 4 +3

h y = 62 3+ x

y=

1
2

2 x + 3

2 For each of the functions in question 1 write the coordinates of the end point.
3 MC The graph shown below was obtained by translating the graph of y =
A 3 units up and 9 units to the right
y
B 3 units down and 9 units to the right
C 9 units up and 3 units to the right
3
D 3 units down and 9 units to the right
E none of the above

x:

(2, 4)

e y = 5 x

f y = x 1+3

g y = 2 + x 1

h y = 4 2 2x + 1

j y= 3 x 7

k y = 6 + 4 2x

l y = 1 2 x

Questions 6 t 7 refer to the diagram at right.


6 MC The equation of the graph is of the form:

3
5

d y = 4+2 x

y=

4 MC To obtain the graph in the diagram at right, the graph of


y = x was:
A translated 2 units to the right and 4 units up
B translated 4 units to the right and 2 units up
C translated 2 units to the right, 4 units up and reflected in the x-axis
D reflected in the y-axis, translated 4 units up and 2 units to the right
E reflected in the x-axis, translated 4 units up and 2 units to the left.
5 WE16 Find the domain and range for each of the following functions.
a y = x +1
b y= x3
c y = x 3

3x 4 + 2

A y = a x b + c, a > 0

B y = a x b + c, a < 0

C y = a b x + c, a > 0

D y = a b x + c, a < 0

(2, 2)
0

E could be either B or C
7 MC The domain and range (in that order) of the function are:
A (, 2] and (, 2]
B (, 2) and (, 2)
C (, 2} and {2, +)

D ( , 2] and ( , 2]
E ( , 2] and ( , 2]
8 WE17 Sketch the graph of each of the following, clearly marking intercepts and end points.
a y= x+2
b y = 13 x + 3
c y = 2 x

d y = x 6 +1

e y= 3+ x +2

f y = 12 4 + x

g y = 2x 3

h y = 6 + 3x + 2

i y = 2 x 1

EXAM TIP
Graphs should be drawn showing
correct features, such as smoothness and end points.
[Assessment report 2 2007]

[ VCAA 2007]

Chapter 2

Functions and transformations

91

eBoo
k plus
eBook

9 WE18
A

y = 2 x 1

y = 2 2 1 x

Spreadsheet 236

y = x 12

y = x 2 +1

Function
grapher

y = 22 x 1

Digital doc

MC The equation of the graph shown at right is:

(1, 2)
x

The graph of y = x was dilated by the factor of 2 from the x-axis and translated m units
to the right and 4 units down. It intersects with the x-axis at x = 5. Find:
a the value of m
b the equation of the curve.
11 The end point of the square root curve is at (4, 3) and its y-intercept is 9. Sketch the graph of
the curve and hence establish its equation.
10

12 The graph of y = x was dilated by the factor of 4 from the x-axis, reflected in the x-axis,
translated 1 unit to the left and p units up. Find:
a the value of p, if the graph cuts the y-axis at y = 4
b the equation of the curve
c the x-intercept
d the domain
e the range.
f Hence, sketch the graph, showing the coordinates of the end points and the intercepts
with the axes.

2F

The absolute value function


The function f (x) = | x | is called an absolute value function or modulus function.
The domain of this function is R and its range is R+ {0}. Its graph
is symmetrical in the y-axis and has a cusp (a sharp point) at the origin.
The symbol | x | represents the magnitude of x, (that is, the size of x),
regardless of its sign.
x, if x 0
Therefore,
|x| =

x, if x < 0
Compare the graphs of y = x and y = | x |.
For x 0, the graphs of the two functions
are identical, while for x < 0 the graph of
y = | x | is the reflection of y = x in the x-axis.
In general, any graph of the form
y = |f|f (x)| is called an absolute value
function. To sketch the graph of y = |f|f (x)|,
we need to sketch the graph of y = f (x) first
and then reflect in the x-axis the portion of
the graph which is below the x-axis.

0
y = |x|
y

0
y=x

0
y = |x|

WORKED EXAMPLE 19

Sketch the graph of y = | x2 1|.


THINK
1

92

We first need to sketch the graph of y = x2 1.


State the shape of this graph.

WRITE/DRAW

Let y = x2 1
Shape: positive parabola, translated 1 unit down

Maths Quest 12 Mathematical Methods CAS for the TI-Nspire

State the coordinates of the turning point (note


that it is also the y-intercept).

Turning point: (0, 1)

Find the x-intercept by letting y = 0.

x-intercept: y = 0
x2 1 = 0
x2 = 1
x= 1
x = +1 or 1

4
5

Sketch the graph of the parabola (Figure A).

Reflect the portion of the parabola for < x < 1


in the x-axis; mark the new y-intercept (Figure B).

(0, 1)
(1,
1, 0)

(1, 0) x

(1, 0) 0 (1, 0)

(0, 1)

y = |x2 1|

Figure A

Figure B

Similar to the graphs discussed in the previous sections, the graph of


the absolute value function can be transformed through dilations,
translations and reflections. If y = a|| f (x)| + c, a is the dilation factor. It
dilates the graph from the x-axis. The larger a is, the thinner the graph.
If a < 0, the graph is reflected in the x-axis.

a=2
a=1

The value c translates the graph along the y-axis. If c > 0, the graph
is moved c units up and if c < 0, it is moved c units down.

a = 1

c=1
c = 2

1
0
2

WORKED EXAMPLE 20

y = |x| + c

eBoo
k plus
eBook

Sketch the graph of y = |x 2| + 1.


THINK

Tutorial

WRITE/DRAW

int-0524
Worked example 20

Compare the given function with y = a|f|f (x)| + c, and


write a short comment.

The graph of y = |x 2|
is translated 1 unit up.

To sketch the required shape we first need to sketch


y = x 2 (a straight line). Find the y-intercept by
letting x = 0.

Let y = x 2.
y-intercept: x = 0
y=02
= 2

Find the x-intercept by letting y = 0.

x-intercept: y = 0
x2=0
x=2

Chapter 2

Functions and transformations

93

Sketch the line.

(2, 0) x

0
(0, 2)

Reflect the portion below the x-axis in the x-axis;


mark the new y-intercept.

y
(0, 2)
0

Move the graph 1 unit up; mark the new y-intercept


and the coordinates of the cusp.

(2, 0)

y
(0, 3)
(2, 1)
0

Absolute value functions as hybrid functions


An absolute value expression can be thought of as two separate expressions, depending on
whether it is negative or positive.
|2x
2 + 3| can be written as (2
2x
(2x + 3) or (2x
(2 + 3) depending upon the value that x takes.
To determine these particular values of x, we need to solve the two inequalities
3
3
2 + 3 > 0 and 22x + 3 < 0, giving x >
2x
and x < , respectively.
2
2
This gives us a domain for the two expressions above, so we can write a representation for the
absolute value expression as:
3

2 x + 3, wh ere x
2
|2x
2 + 3| =
2x

(2 x + 3), wh ere x < 3

2
This is a useful process when used to rewrite an absolute value function that is to be graphed,
because it gives us a rule for each part of the graph in the form of a hybrid function. It is also
important when needing to differentiate a function of this type in a later chapter.
WORKED EXAMPLE 21

k plus
eBook
eBoo

Express f (x) = |5x


5 4| as a hybrid function, defining the domain of each part
5x
and graphing the function.
THINK
1

Break the function into two parts: a negative


and positive part.

Simplify the domain and function for each.

Tutorial

int-0525
Worked example 21

WRITE/DRAW

5 x 4, wh ere 5 x 4 0
f ( x) = | 5x 4 | =
(5 x 4), wh ere 5 x 4 < 0
First function: 5x 4
First domain: 5x 4 0
4

x 5

Second function: (5x 4) = 5x + 4


Second domain: 5 x 4 < 0
x<

94

Maths Quest 12 Mathematical Methods CAS for the TI-Nspire

4
5

Rewrite the function in hybrid form with the


two rules with their respective domains.

Graph the two functions for the specific


domains.

5 x 4, wh ere x 5
f ( x) =
5 x + 4, wh ere x < 4

5
y

5
4 (0, 4)
3
2
1
2

f x) = 5x 4
f(

1
( 45 , 0)

The absolute value function can easily be


drawn using a CAS calculator. On a Graphs
page, complete the function entry line as:
f 1(x) = |5x 4|
Then press ENTER .

REMEMBER
y

1. The symbol | x | denotes the magnitude of x.


2. | x | = x, if x 0
| x | = x, if x < 0
3. To sketch the graph of y = |f|f (x)|:
(a) sketch the graph of y = f (x)
(b) reflect the portion of the graph which is below the x-axis
in the x-axis.
EXERCISE

2F

0
y = |x|

The absolute value function


1

Sketch the graph of each of the following, showing exact values of intercepts

WE19

of axes.
a y = |2x
2 |
2x
d y=

|x2

b y = |x 1|

6|

e y = |4

g y = |3x3|
h
i

y = |(x +
y=

2)3

2
x 1

x2|

y = |3 6x|

f y = |(x 3)2 4|
EXAM TIP
Always give answers in exact form
unless the question asks for a numerical
approximation (for example, give your answer to
2 decimal places).

1|

[Assessment report 2 2007]

[ VCAA 2007]

Chapter 2

Functions and transformations

95

MC Which of the following functions best describes this graph?

A y = |(x 1)3|

B y = |(x + 1)3|

C y = |x3 + 1|

D y = |x3 1|

E y = |(x +

1)3

+ 1|

1 0

3 For each of the following functions state the domain and range.

eBoo
k plus
eBook

a y = 2|x|

b y = |x| + 1

d y = |x2 3| 2

Spreadsheet 002

d y=

Absolute value
function

1
+1
x +1

y = 4 3|x|

y= 2

1
x2

WE20 Sketch the graphs of each of the following.

a y = 2|x|

Digital doc

y=

|x2

b y = |x + 5| 6

1| + 1

1 3

x 4

y=

y=

y=2

1
1
x2

y = 2|3 x| + 1

2|

y = |(x + 1)2 1| 2

2
+3
6 x

y=

e y=2
h

y=

y=

|x2

1 1
4
4 x2

2 x 2 + 3

x +1 8

EXAM TIP

Use a ruler for linear and absolute

value graphs.
[Assessment report 2 2007 VCAA]

WE21 Given the function f (x


(x) = | 3x 1|:

a rewrite the function as a hybrid function with appropriate domains


b find f (0) and f (2)
c sketch the graph, labelling any significant points.

6 Given the function f (x


(x) = | x2 3x |+ 2:
a rewrite the function as a hybrid function with appropriate domains
b find f (1) and f (2)
c sketch the graph, labelling any significant points.

eBoo
k plus
eBook
Digital doc

WorkSHEET 2.2

2G
eBoo
eBook
k plus
Interactivity

int-0247
Transformations
with matrices

96

7 The design shown at right is to be embroidered on the outer side of a pair of childrens socks.
The total length of the design is 12 cm and its width is 8 cm.
y
6
If we draw the set of axes through the centre of the design, the
red section can be thought of as the absolute value function on a
restricted domain.
a Find the rule for the red section and specify the domain.
b Using your knowledge of the transformations, and the rule for
4
4x
the red section, find the rules for the blue, green and yellow
sections of the design.
c Using a graphics calculator, sketch the 4 functions that were
obtained in a and b . Have you obtained the right design?
6

Transformations with matrices


Transformations of graphs can be described using matrices as an alternative to function notation.
The transformations that have been considered so far (dilations, reflections and translations)
can be represented in matrix form. This describes how a particular point on a graph will
be moved (or mapped) to a resultant location by the application of a dilation, reflection or
translation, or a combination of the three. Remember the definition of a transformation is a rule
that links each point in the Cartesian plane to another point. So the matrix can be used for any

Maths Quest 12 Mathematical Methods CAS for the TI-Nspire

point on a curve, or in fact used to determine the new rule for a function that has undergone one
or a series of transformations. The new points or rules are termed images of the original.

Reflections and dilations

x
We can summarise the use of matrices to map these transformations, T , of points on a
y
curve as follows:
(Let x' be the transformed value of x, and y' be the transformed value of y.)
x x' 1 0 x x
T = =
= represents a reflection in the y-axis.
y y' 0 1 y y
x x' 1
T = =
y y' 0

0 x

x
= represents a reflection in the x-axis.
y

1 y

x x' a 0 x ax
T = =
= represents a dilation of a factor of a from the y-axis.
y y' 0 1 y y
x x' 1 0 x x
T = =
= represents a dilation of a factor of a from the x-axis.
y y' 0 a y ay
These operations can be combined to represent more than one transformation, for example
x x' 2 0 x 2 x
T = =
=

y y' 0 3 y 3 y
represents a reflection in the y-axis, a dilation of a factor of 2 from the y-axis, and a dilation of a
factor of 3 from the x-axis.
WORKED EXAMPLE 22

Using matrices, find the location of the point ((x, y) under the following
transformations of the point (1, 3):
dilation by a factor of 2 from the y-axis
reection in the x-axis.
THINK
1

Construct the correct matrix that represents


the transformations described.
x x' 2
T = =
y y' 0

WRITE/DISPLAY

0 x


1 y

On a Calculator page, press:


Ctrl /
r
Select the appropriate matrix templates to
construct and solve the matrix equation using
the point (x, y) = (1, 3):
x x' 2
T = =
y y' 0

0 1

3

Chapter 2

Functions and transformations

97

Write the matrix equation and interpret this to


answer the question.

0 1

2
=
3

1 3

The image of the point (1, 3) is at (2, 3).

Matrix operations can be done using a CAS calculator, but as the matrix multiplication required here is simple, it is
recommended it be done by hand.

The difficult part is to correctly identify the transformation matrix. Once you have done that it is
a matter of performing a matrix multiplication.

Translations
Translations require a slightly different process. The transformation matrix is a 2 1 matrix, and
finding the new image requires addition of the matrices rather than multiplication.
b
Matrices describing translations are of the form .
c
This represents:
a translation of b units in the positive direction of the x-axis and
a translation of c units in the positive direction of the y-axis.
Note b and c > 0:
If either of the terms is negative, the translation is in the negative direction.
A zero entry indicates there is no translation in a particular direction.
So a translation of a point (x, y) can be described as follows:
x x' x b
T = = +
y y' y c
x + b
=

y + c
WORKED EXAMPLE 23

Find the location of the point ((x', y') under the following transformations of the point (2, 4):
translation of 3 units in the x direction
translation of 5 units in the y direction.
THINK
1

WRITE

Construct the correct matrix that represents the


transformations described.
x x' x 3
T = = +
y y' y 5

Construct and solve the appropriate matrix equation.


x x'
3
T = = +
y
y
'
4 5
2

Interpret this to answer the question.

3

5

2 3 1
+ =
4 5 1
2 3 1
+ =
4 5 1
The image of the point (2, 4) is at (1, 1).

Matrix operations can be done using a CAS calculator, but as the matrix addition required here is simple, it is
recommended it be done by hand.

98

Maths Quest 12 Mathematical Methods CAS for the TI-Nspire

Note it is very important to use the correct mathematical language, both for the size and the
direction of the transformation. For example, dilations are by a factor of, in other words a
multiple of the original value, as distinct from translations, which are described as of n units,
which is a set distance. In terms of direction, the expression from the y-axis can also be
expressed as parallel to the x-axis, in the x direction or even horizontally. The same applies
for from the x-axis.

Putting it all together


The formal notation often used to describe a transformation begins as T : R2 R2, which is
saying the transformation that maps a point (x, y) to another point (x, y) is . and then the
transformation is described. For example, a transformation involving a dilation by a factor of
3 from the x-axis followed by a translation of 2 in the x direction and 1 in the y direction could
be defined as T : R2 R2, T(
T x, y) = (x 2, 3y + 1), or simply (x, y) (x 2, 3y + 1).
When more than one transformation is described, it is known as a composition of the
transformations.
When a series of transformations are described, they need to be done in the correct order as
stated in the question.
WORKED EXAMPLE 24

Find the location of the point ((x', y') under the following transformation of the point (3,2):
1
from the y-axis
2
dilation by a factor of 3 from the x-axis
reection in the x-axis
translation of 3 units in the x direction.
dilation by a factor of

THINK

WRITE

Perform the first three transformations together. Construct the 1


correct matrix that represents these transformations described. 2
0
x x' 1
0 x
T = = 2

y y' 0 3 y

Construct and solve the appropriate matrix equation.


x x' 1
0 3
T = = 2

y y' 0 3 2

1
2
0

0 3 3
= 2
3 2
6

Now perform the translation. Construct and solve the


appropriate matrix equation. Note we are using the
product matrix from step 2.

3 3 9
2 + = 2
6 0 6

x x' 1
T = = 2
y y' 0
4

0 3 3
+
3
2 0

Interpret this to answer the question.

The image of the point (3, 2) is at

( , 6)
9
2

Remember that a transformation maps any point on a curve to another by the same rule. Rather
than mapping a series of individual points on the same curve, we can simply find a new rule
under a transformation (or a series of them) and use this new rule to determine the location of
any points from the curve described by the original rule.

Chapter 2

Functions and transformations

99

WORKED EXAMPLE 25

k plus
eBook
eBoo

Write the resultant equation from the following transformations of the curve
described by y = x3:
dilation by a factor of 2 from the y-axis
reection in the x-axis
translation of 2 units in the negative y direction.
THINK
1

Construct and solve the appropriate matrix


equation.

1 y

Now perform the translation. Construct the correct


matrix that represents this transformation described.

0

2

Perform the first two transformations together.


Construct the correct matrix that represents these
transformations described.

0 x

y

x x' 2 0 x 0
T = = +
y y' 0 1 y 2
Construct and solve the appropriate matrix
equation. Note we are using the product matrix
from our first equation in step 2.
The transformed values of x and y are 22xx and y 2.

Express x' and y' in terms of x and y.

State the resultant equation.

int-0526
Worked example 25

WRITE

x x' 2
T = =
y y' 0
2

Tutorial

0 x

2x
=
y

x' 2 x 0 2 x
= + =

y' y 2 y 2
Therefore x' = 2x
2x and y' = y 2.
x' and
2
y = y' + 2
y = y' 2
x=

x
y= 2
2

So for any point on the graph of the original function, y = x3, we can map the corresponding
point under the transformations above by substituting the values into this transformed equation.
Lets have a look at another example. This time we will complete reflections/dilations and
translations in the one step.
WORKED EXAMPLE 26

Find the image of the curve with equation y = x after a reflection in the x-axis, followed by a
dilation of a factor of 2 from the x-axis, and then a translation by +3 in the x direction.
THINK
1

100

Construct the correct matrices that represent the


transformations described.
x x' 1 0 x 3
T = = +
y y' 0 2 y 0

Maths Quest 12 Mathematical Methods CAS for the TI-Nspire

WRITE

3
and 0

Construct and solve the appropriate matrix equation.

x' 1

y' 0

0 x 3 x + 3
+ =
y 0 2y

The transformed values of x and y are x + 3 and 2y.

So x' = x + 3 and y' = 2y.

Express x' and y' in terms of x and y.

x = x' 3 and y = y'


2

Substitute new y- and x-terms for y = x,


x and drop the
primes.

2
y

= x3
= 2 x3

y = 2 x 3
6

State the resultant equation showing all transformations.

y = 2 x 3

REMEMBER

a 0
1.
represents a dilation of a from the y-axis and a dilation of b from the x-axis.
0 b
1 0
1
2.
represents a reflection in the y-axis, and
0
0 1
the x-axis.

represents a reflection in

b
3. represents a horizontal translation of b and a vertical translation of c.
c
EXERCISE

2G

Transformations with matrices


1

Identify the dilations and/or reflections described by the following matrices.


i

1 0

0 2

ii

1
2
0

iii

2 0

0 3

iv

2 WE22 WE24 Find the image of the point (3, 5) under the above transformations in question 1 .
3 Find the image of the graphs of the following equations under the transformations in
1 i and 1 ii.
a y=

1
x2

b y = x3 5

y= x

4 Identify the translations described by the following matrices.


i

3

2

ii

2

2

iii

Chapter 2

1
5
0

Functions and transformations

101

5 WE23 Find the image of the point (1, 2) under the transformations given in question 4 .
6 Find the image of the following equations under each of the transformations defined in
questions 4 i and 4 ii.
a y = |x|
b y = x2 3x
7 The transformation T : R2 R2 which maps the curve with the equation y = x3 to the curve
with the equation y = (3x 6)3 + 1, could have:
A

x 1 0 x 6
T =
+
y 0 3 y 1

x 1 0 x 6
T = 3
+
y 0 1 y 1

x 1 0 x 2
T = 3
+
y 0 1 y 1

x 1 0 x 3
T =
+
y 0 3 y 1

x 3 0 x 6
T =
+
y 0 1 y 1

For the following transformations, where T : R2 R2, state what the transformation T
1
represents and determine the image of the equation f ( x ) = .
x
x 1 0 x 6
x 2 0 x 3
a T =
b T = 1 +
+
y 0 2 y 1
y 0 2 y 1
x 1
c T = 3
y 0

0 x 1
+
1
y 2

9 A function g(x
( ) is mapped to the curve h(x
(x
( ) = g(4(x
(x
(4( + 1)) + 3. Create a matrix equation that
(4(x
will map g(x
( ) to h(x
(x
( ).
(x
10 WE25 The following transformations are applied, in order, to the graph of
y = x3 4x
4 :
dilation by a factor of 2 from the x-axis
reection in the y-axis
translation of 1 unit in the y direction.
a Use matrices to determine the image equation under these transformations.
b Find the image of the point (2, 0) and check whether this point lies on the curve of the
equation from a.
3
1
+ 1 , describe, in order, the transformations
and g( x ) =
2
( x 2)2
x
performed to the graph of f (x) to give g(x) and create a matrix equation which would map
f (x) to g(x).

11 WE26

If f ( x ) =

12 If f (x
(x) = g(2(x
(2( + 1)) + 1 and g( x ) = x , find f (x
(2(x
(x) in terms of x only, using:
a an algebraic method without the use of a CAS calculator
b matrices and a CAS calculator.
13 If f (x
(x) = 2g(x
( 1) 2 and g(x
(x
( ) = x2 3x, find f (x
(x
(x) in terms of x only, using:
a an algebraic method without the use of a CAS calculator
b matrices and a CAS calculator.
14 If

102

x3
1
1
( ) using matrix methods.
(x
h( x + 2) + 1 =
3 x 2 6 x , find h(x
2
2
2

Maths Quest 12 Mathematical Methods CAS for the TI-Nspire

2H

Sum, difference and product


functions
Sum functions

A sum function is of the form y = f (x) + g(x), or alternatively


y = x2
4
y = (f
( + g)(x).
3
Many functions include two or more terms added (or
2
y = 1x
1
1
2
subtracted) together. For example the function y = x + can be
x
4 3 2 110 1 2 3 4 x
1
2
thought of as the sum of the functions y = x and y = .
2
x
These graphs can be drawn by sketching the two individual functions on the same set
of axes then adding the y-values (ordinates) for each x-value and plotting the resulting
points.
This is a useful method when we know the basic shape of the individual functions but
do not recognise the whole function. We would not use this method for a familiar function
such as y = x2 + 3x, as we have learnt ways of sketching this without breaking it up into
parts.
1
y
Using the example in the first paragraph, y = x 2 + , we
4
y = x2
x
do not recognise the shape of this function, but we know the
3
2
two individual functions are the basic positive parabola and
y = x2 + 1x 1
y = 1x
the hyperbola. We could therefore sketch the graph of the
parabola and the hyperbola and add the y-values together for
44 3 22 1110 1 2 3 4 x
corresponding x-values to obtain points on the curve of the sum
2
function which can be joined together to obtain the graph of
the sum function.
Note that the domain of the hyperbola is restricted to R \ {0} so the y-value at x = 0 is
undefined. As you cannot add an undefined number, this x-value is also undefined for the sum
function. A general rule is that the sum function is only defined for the domain over which
both of the individual functions are defined. The domain of the sum function is, therefore, the
intersection of the domains of the individual functions.
If h(x) = f (x) + g(x), then the domain h(x) = domain f (x) domain g(x).

Difference functions
A difference function is of the form y = f (x) g(x), or alternatively y = (f
( g)(x). It is essentially
the same as a sum function except that one of the individual functions is subtracted from the
1
other. So y = x 2 could be sketched by the same method as described above but instead of
x
adding the y-ordinates, we would subtract one from the other.
The domain of a difference function is determined in the same way as a sum function. We
could extend our rule above to include difference functions.
y
If h(x) = f (x) g(x), then the domain h(x) =
4
y = x2
domain f (x) domain g(x).
3
y = x2 1x
2
We can also think of a difference function as adding a
y = 1x
1
1
negative and it could be written y = x 2 + . With this
x
4 3 2 110 1 2 3 4 x
in mind, an alternative method of sketching the graph of a
2
difference function is to reflect the graph of the second function
1
(in this example, ) in the x-axis and then add the ordinates as for a sum function.
x

Chapter 2

Functions and transformations

103

When sketching graphs of sum/difference functions, there are key points that can be
found on either individual function to easily identify the value of the ordinate of the sum or
difference function. These are the x-intercepts and any point of intersection of the individual
functions.
The x-intercept is where the ordinate of that particular function is zero, so the graph of the
sum or difference function is actually the ordinate of
y
the other function for that value of x.
2.5
At the intersection, the ordinate of the sum function
2
will be double that of the two individual functions.
1.5
For a difference function, an intersection of the two
individual functions corresponds to on x-intercept
1
(y = 0) of the difference function.
0.5
Another useful y-value to look for is where the
graphs of individual functions have y-values that are
1.5 1 0.5 0 0.5 1 1.5 x
of the same magnitude but one is positive and one
0.5
is negative. This point is an x-intercept of the sum
function.

WORKED EXAMPLE 27

eBoo
k plus
eBook

Using addition of ordinates, sketch the graph of

Tutorial

f ( x ) = log
log e ( x + 2 ) + x , x [
[ 1, 2 ].

int-0527
Worked example 27

THINK
1

WRITE/DRAW

Sketch the graphs of y = loge (x + 2) and


y = |x| on the same set of axes over the
required domain x [1, 2].

y
3
2 y = loge (x + 2)
(1, 1)

1.5 1 .5 0
2

104

Moving from left to right, add the


y-coordinates of the two graphs for the key
points and plot the resultant points. The key
points are the:
end points
y-intercepts
points of intersection.
The new points on the graph are marked by
an asterisk.

y =x (2, loge (4))


0.5 1 1.5 2 2.5 x

3
(1, 1)

2 y = log (x + 2)
e
1

1.5 1 0.5 0

(2, 2)

(2, 2)

y =x (2, loge (4))


0.5 1 1.5 2 2.5 x

Left end points (1, 0) and (1, 1), so the new point
will be at (1, 1).
Right end points (2, loge (4)) and (2, 2), so the new
point will be at (2, 2 + loge (4)).
y-intercepts (0, 0) and (0, loge (2)).
Points of intersection (0.44, 0.44) and
(1.15, 1.15), so the new points will be (0.44, 0.88)
and (1.15, 2.30).

Maths Quest 12 Mathematical Methods CAS for the TI-Nspire

Join these points with a smooth curve to


create f (x).

y f(
f x) = loge (x + 2) +x
3
(2, 2)
y
=
x
(1, 1)
1
y = loge (x + 2) (2, loge (4))
1.5 1 0.5 0 0.5 1 1.5 2 2.5 x
2

Remove the two individual graphs to leave


the sum function.

(2, 2 + loge (4))

y
3
2
(1, 1)

f x) = loge (x + 2) +x
f(

1.5 1 0.5 0

0.5

1 1.5

2 2.5 x

Product functions
A product function is of the form y = f (x) g(x), or alternatively y = (fg
( )(x).
When graphing product functions, it is useful to graph the individual functions, f and g, and
for any relevant values of x, to identify the y-values, or ordinates and multiply these together to
obtain the y-value of the product function. If the y-value is undefined at a particular value of x
for either of the individual functions, then the product function is undefined for that value. We
cannot multiply by an undefined number.
If h(x) = f (x)g(x), then the domain h(x) = domain f (x) domain g(x).
When examining the graph of the two individual functions, it is useful to look at x-intercepts
and points where the value of either function is 1. The product function will also have an
x-intercept at a point where either individual function has an intercept (as multiplying by zero
gives zero). At a point where a function = 1, the product function will have a value equal to the
value of the other function, or its negative.
It is also useful to observe that where the individual functions are both above the x-axis,
or both below the x-axis, the value of the product function will be positive, that is, above the
x-axis. This is because the product of two positive numbers or two negative numbers is positive.
Alternatively, where one function is above and one below the x-axis, the value of the product
function will be negative, that is, below the x-axis.
WORKED EXAMPLE 28

If f (x
(x) = 2x
2 and g( x ) = x + 1 , sketch the graph of f ( x ) g( x ) = 2 x x + 1 .
THINK
1

Sketch the graphs of f (x) and g(x).

WRITE/DRAW
y

y = 2x
2
y= x+1

(0, 1)
(1, 0) 0

Chapter 2

Functions and transformations

105

Find the domain of f (x) and the domain of g(x).

Dom f = R and dom g = [1, )

Find the domain of f (x) g(x).

Dom fg = [1, )

Find the x-intercepts of both f and g and hence find


the x-intercepts of the product fg.

x-intercept for f (x) is when x = 0 and f (x) = 0


x-intercept for g(x) is when x = 1 and g(x) = 0
Hence, the x-intercepts for the product are
when x = 0 and x = 1.

Find the values of x for which the product is


negative.

f (x) is negative and g(x) is positive


for x (1, 0), so fg is negative for x (1, 0).

Find the values of x for which the product is


positive.

Find the turning point using a CAS calculator.


Round the answer to 2 decimal places as
appropriate.

f (x) and g(x) are both positive


for x (0, ), so fg is positive for x (0, ).
The turning point is ( 32 , 0.77).

Sketch the graph of the product.

y = 22xx x + 1

(1, 0)
(0, 0)

2
(
, 0.77)
3

REMEMBER

1. For the sum/difference function, dom(


dom(f (x) g(x)) = dom f (x) dom g (x). The graph
of the sum/difference function can be obtained by using the addition of ordinates
method.
2. For the product function, dom ((ff (x) g(x)) = dom f (x) dom g(x). Some features of the
graph of the product function are as follows:
(a) the x-intercepts of f (x) g(x) occur where either f (x) or g(x) have their x-intercepts
(b) f (x) g(x) is above the x-axis where f (x) and g(x) are either both positive or both
negative
(c) f (x) g(x) is below the x-axis where one of the functions f (x) or g(x) is positive and
the other is negative.
EXERCISE

2H

Sum, difference and product functions


1

106

Sketch the graphs of f (x


(x) = g(x
( ) + h(x
(x
( ) using addition of ordinates, given the following
(x
functions g(x) and h(x). State the domain of f (x) in each case.
a g(x) = x2, h(x) = x
x3
b g( x ) = , h( x ) = x
2
c g(x) = 3x2, h(x) = | x |
1
x) = x + 2
d g( x ) = , h( x)
x

Maths Quest 12 Mathematical Methods CAS for the TI-Nspire

Determine the equation of g(x


( ) h(x
(x
( ) in each of the following cases then, using addition
(x
of ordinates, sketch the graph of g(x) h(x).
x + 1, h( x ) = x
a g( x ) =
b g(x) = | x |, h(x) = | x + 1| 2
3 For each of the following, find the domain of f (x
(x)g(x
( ).
(x
2
a f ( x ) = x , g( x ) = 3 x
b f (x) = x + 2, g(x) = |2x
2 + 1|
2x
c f ( x ) = x , g( x ) = 1 x
d f (x) = | x |, g(x) = x2 1
e f ( x ) = x 3 , g( x ) = x + 2
4

WE27 Sketch the graph of f ( x ) = x +

the general shape and any asymptotes.

2
for 4 x 4, by the addition of ordinates, showing
x

5 Two functions are defined as f (x


( ) = x2 and g( x ) = x .
a Sketch the graph of each on the same set of axes for 2 x 2.
b Find the smallest possible value of a given that the domain of the function h,
where h(x) = (f
( + g)(x), is a x 2.
c Find f (0) and g(0), and hence find h(0).
d Find f (1) and g (1), and hence find h (1).
e Find f (2) and g (2), and hence find h (2).
f Using this information, sketch the graph of h(x
( ) (on the same set of axes as in a).
(x
Given the functions f (x
( ) = x3, g(x
( ) = | x | and h(x
(x
( ) = f (x
(x
( ) + g(x
( ):
(x

a Sketch the graph of each on the same set of axes for 2 x 2.


b Find f (2)and g (2), and hence find h (2).
c Find f (0) and g (0), and hence find h (0).
d Find f (1) and g (1), and hence find h (1).
e Find f (2) and g (2), and hence find h (2).
f What is the range of the function h (in exact form).
g Using this information, sketch the graph of h(x) (on the same set of axes as in a).
7 WE28 Two functions are defined as f (x
( ) = x 3 and g( x ) = x . Let h( x ) = f ( x ) g( x ).
a Find the domain of h.
b Sketch the graph of each on the same set of axes.
c Find f (0) and g (0), and hence find h (0).
d Find f (1) and g (1), and hence find h (1).
e Find f (2) and g (2), and hence find h (2).
f What is the range of the function h (in exact form)?
g Using this information, sketch the graph of h(x) (on the same set of axes as in a).
6

8 Sketch the graphs of the functions f ( x ) = x + 5 and g( x ) = 8 x and use these to find the
domain of the function h(x
( ) = f (x
(x
( ) + g (x
( ). On the same axes, sketch the graph of h(x
( ), including
(x
the coordinates of any end points.
x2
4, for
9 Use a CAS calculator to view and sketch the graphs of f (x
( ) = | x + 2 | and g( x ) =
2
2.5 x 2.5. Then, without using the calculator, use these graphs to sketch the graph of
h, if h(x) = (f
( g)(x) on the same set of axes. Using the calculator, check the shape of the
graph you have drawn and use it to identify any significant points such as intercepts and cusp
points to 2 decimal places. (You may need to adjust your window settings in order to clearly
identify these points.)

Chapter 2

Functions and transformations

107

2I

Composite functions and functional


equations
Composite functions
A composite function is formed from two functions in the following way. If f (x) = x + 5 and
g(x) = 2x
2x are two functions, then we combine the two functions to form the composite function
g(f
( (x)) = 2f
(f
2 (x) = 2(x + 5). That is, f (x) replaces x in the function g(x).
The composite function reads g of f and can be written g f.
f
Another composite function is f (g(x)) = g(x) + 5 = 2x
2 + 5. In this case, g(x) replaces x in f (x).
This composite function reads f of g and can be written f g.
For the composition function f (g(x)) to be defined, the range of g must be a subset of (or
equal to) the domain of ff, that is ran g dom ff. It is easiest to list the domain and function of
both f (x) and g(x) first when dealing with composite function problems. For example: f (x) = x2
and g( x ) = x :

Domain
Range

f (x
( )

g(x
( )
(x

[0, )

[0, )

[0, )

Composite functions can be rather complex to graph by hand, so a CAS calculator can be
used for assistance when sketching.

WORKED EXAMPLE 29

For the pair of functions f ( x ) =


a show that f (g
( (x
( )) is defined
c state its domain.

1
and g( x ) = x :
x+2
b find f (g
( (x
( ))

THINK
a

WRITE

Create a table showing the domain


and range of both functions.

For f (g(x)) to exist the range of g


must be a subset of f.
f

Form the composition function f (g(x)) by


substituting g(x) into f (x).

f (x
(x)

g(x
( )
(x

Domain

R \{2}

R+

Range

R \{0}

R+

ran g(x) dom f (x)


f (g(x)) is defined.
f ( g( x )) = f ( x )
f ( g( x )) =

108

1
x +2

The domain of f (g(x)) must be the same as


Domain of f (g(x)) = R+
the domain of g(x). Since the domain of g(x)
is R+, it is the domain of f (g(x)).

Maths Quest 12 Mathematical Methods CAS for the TI-Nspire

Functional equations
Sometimes we are required to solve or analyse equations that are in terms of unknown functions,
for example, f (x) or f (y), rather than being in terms of unknown variables, for example x or
y. An example of the type of problem you might encounter is to find a function that satisfies
f (x + y) = f (x) + f (y). Alternatively, you might be required to determine if a particular function
satisfies the rule f (2x
(2x) = 2f
2f (x).
Equations such as f (x + y) = f (x) + f (y) are called functional equations.
There are generally two ways to solve these types of problems: algebraically or using a CAS
calculator.
WORKED EXAMPLE 30

Determine if f (x
( ) = 3x satisfies the equation f (x
( + y) = f (x
( ) f (y
( ).
THINK
1

WRITE

Substitute the function into the LHS and


RHS of the equation separately.
Simplify the LHS of the equation to determine
if it equals the RHS of the equation.
Answer the question.

2
3

LHS = f (x + y) = 3x + y
RHS = f (x) f (y) = 3x 3y
LHS = f (x + y) = 3x + y
LHS = f (x + y) = 3x 3y
LHS = RHS
x
ff (x) = 3 satisfies the equation f (x + y) = f (x) f (y)

WORKED EXAMPLE 31

Determine if g(x
( ) = 10x
(x
10 satisfies the equation g(2x
(2 ) = 2g
(2x
2 (x
( ).
THINK
1

On a Calculator page, press:


MENU b
1:Actions 1
1:Dene 1
Complete the entry line as:
Define g(x) = 10x
Then press ENTER .

To calculate g(2x) and 2g(x), complete


the entry lines as:
g(2x)
2g(x)
Press ENTER after each entry.
Examine the solutions to determine if
the expressions are equal.

WRITE/DISPLAY

LHS = g(2x
(2 ) = 20x
(2x
20

Chapter 2

RHS = 2g(x) = 20x


20

Functions and transformations

109

Check that the equation holds true for


all values of x by completing the entry
line as: g(2x) = 2g(x)
Then press ENTER .
As the statement is true the equation
must hold for all values of x.

Write the solutions for g(2x


(2 ) and 2g(x)
(2x
and answer the question.

g(2x
(2 ) = 20x
(2x
20
2g(x) = 20x
20
g(2x
(2 ) = 2g(x)
(2x
When g(x) = 10x
10 it satisfies the equation g(2x
(2 ) = 2g(x).
(2x

If we consider the same equation f (2x


(2x) = 2f
2f (x
(x) for a different function, for example, f ( x ) = x ,
we obtain two different equations, f (2 x ) = 2 x and 2 f ( x ) = 2 x , which are not equal.
However, if we define this function on a CAS calculator and enter the statement f (2x
(2x) = 2f
2f (x), the
result is x = 0. This means this equation holds true when x = 0 but not for any other values of x.
REMEMBER

1. For the composite function f (g(x)) to be defined, the range of g must be a subset of the
domain of ff. Furthermore, if f (g(x)) is defined, the domain of f (g(x)) equals the domain
of g(x).
2. For equations involving algebra of functions, determine if the equation is true for the
particular function by considering the LHS and RHS of the equation separately to test
if the equation holds true for all values of x.
EXERCISE

2I

Composite functions and functional equations


1

WE29 For each of the following pairs of functions:

i show that f (g(x)) is defined

i i find f (g(x)) and state its domain.

a f (x) = 2x
2 1 and g( x ) = x + 3
c f (x) = 3(x 2)3 and g(x) = x2
e f (x) = (x + 1)(x + 3) and g(x) = x2
2 Show that the function f (x
( ) = 3xx satisfies
x + y f ( x) + f ( y)
=
the equation f
.
3
3
3

WE30 Show that f ( x ) =

equation

110

[ VCAA 2007]

2
satisfies the
x

f ( x) + f ( y)
= x + y.
f ( xxyy )

Maths Quest 12 Mathematical Methods CAS for the TI-Nspire

1
and g(x) = | x | + 1
x+2
d f (x) = | x | and g(x) = x3
b f ( x) =

EXAM TIP
Show that questions require detailed
working out to be given, so provide as many steps as
possible to attain full marks. Write as required at
the end of your solution once you have shown the
problem is correct.
[Assessment report 2 2007]

4 WE31 Determine which of iv


v hold for the following functions:
v
f ( x)
ii f ( x y ) =
i f (x y) = f (x) f (y)
f ( y)
x f ( x)
f
=
iii f (x) + f (y) = (x2 + y2) f (xy)
iv y f ( y )
v f (xy) = f (x) f (y)

5
6
7
8

10

b f (x) = | x |
a f ( x) = x
1
1
c f ( x) =
d f ( x) = 2
x
x
e f (x) = x2
f f (x) = 2x
1
f ( x) =
and g( x ) = x , determine the values of a such that f (g(x
( )) exists.
(x
( x + a) 2
1
+ 2, determine if f g
If f:
f RR, where f ( x ) = x 2 and g:R {1} R, where g( x ) =
x +1
and g f exist and, if so, find the composition functions.
If f:
f RR, where f ( x ) = 3 x and g:RR, where g(x
( ) = x2 1, show that f g is not defined.
(x
By restricting the domain of g, find a function h such that f h is defined.
Given w(x
( ) = x + 3, x > 3 and v(x
(x
( ) = | x | 2, x R+, state the domain and range of each
(x
function. Hence, find if w v and v w exist and, if so, state their rules including their
domains.
Show that the equation g(x
( ) = x3 satisfies the equation g(x) = g(x
(x
( ). Show that
(x
n
this statement is true for all functions of the form g(x) = x , where n is an odd natural
number.
Show that g(x
( ) = x4 satisfies the equation g(xy
(x
( ) = g(x
(xy
( )g(y
(x
( ). Show that this equation is true
(y
for all functions of the form g(x) = xn, where n is a natural number.

11 Given the two functions:


1
f : [0, 6] R
R,, f ( x)
x ) = 4 x ( x 6)2
g: [0, 6] R, g (x) = x2 + 6x
find the maximum value of | f (x) g(x)| for
0 x 6, correct to 2 decimal places, and
determine the values of x in this interval for
which the maximum occurs. [ VCAA 2007]

EXAM TIP
Be careful to answer all aspects of the
question (this question requires the x-value and the
maximum). Always re-read the question to check
you have met all requirements before moving on to
the next question.
[Assessment report 2 2007]

12 Consider f : [4, ] R
( ) = 1 x. What transformations are
(x
R,, f ( x)
x ) = x 4 and g: R R, g(x
required to obtain f (g(x
( )) from f (x
(x
( )?

2J

Modelling
People such as scientists, financial advisers, business analysts, economists, statisticians and
others often have to deal with large and small sets of data.
Once the data are collected, we are often interested in finding the rules that link features of
the data. The process of finding such a rule is called modelling and the rule itself is known as
the mathematical model. When finding the model, the best way to start is to plot the data, as the
shape of the graph might suggest the type of relationship between the variables.

Types of graphs
By recognising the shape of a graph, it is possible to find the rule or mathematical model that
describes it. Throughout this chapter, several types of graphs have been investigated.

Chapter 2

Functions and transformations

111

The parabola: y = x2

The graph of a cubic function: y = x3

The hyperbola: y =

y=0

1
x

x=0

The truncus: y =
y

y=0

The graph of a square root function:

1
x2

y= x
y

0
x=0

Reflections and translations can be applied to each of these graphs, but the basic shape of each
graph remains the same.

WORKED EXAMPLE 32

Match each of the following graphs with the appropriate model.


i y = ax2
a

ii y = ax3
b

y
x

iii y =
c

a
x

iv y =
d

THINK

a
x2

v
e

y= a x
y

WRITE

Match the graphs using the information in the


summary above.

i is a parabola; it matches graph b .


i i is a cubic; it matches graph e .
i i i is a hyperbola (the graph is in opposite quadrants);
it matches graph c .
iv is a truncus (the graph is in adjacent quadrants); it
matches graph a .
v

112

is a square root function; it matches graph d .

Maths Quest 12 Mathematical Methods CAS for the TI-Nspire

WORKED EXAMPLE 33

2.5

3.54

4.33

5.59

The data in the above table exactly fit one of these rules: y = ax 2 , y = ax 3 , y =
a Plot the values of y against x.

b Select the appropriate rule and state the value of a.

THINK

WRITE/DRAW

a Plot the values of y against x.

y
5
4
3
2
1
0

a
a
, y=
or y = a x .
x
x2

1 2 3 4 5

b Assume that y = a x .

Study the graph. It appears to be a square


root curve. Write the appropriate rule.

To find the value of a: select any pair of


corresponding values of x and y. (Since we
need to take a square root, the best to choose
is the one where x is a perfect square.)

Using (1, 2.5):

Substitute selected values into the rule


and solve for a.

2.5 = a 1
=a1
a = 2.5

We need to make sure that the selected


rule is the right one. Replace a with 2.5
in the rule.
Substitute the values of x from the table
into the formula and check if you will
obtain the correct values of y.

Verifying:
y = 2.5 x

As the values of y obtained by using the


rule match those in the table, the choice
of model is correct.

The rule that fits the data is y = a x ,


where a = 2.5.

y = 2.5 0
=0
(2, 3.54): y = 2.5 2
= 3.54
(3, 4.33): y = 2.5 3
= 4.33
y = 2.5 4
(4, 5):
=5
(5, 5.59): y = 2.5 5
= 5.59
(0, 0):

Chapter 2

Functions and transformations

113

The process of fitting a straight line to a set of points is often referred to as regression. Statistical
data is easiest to deal with in linear form. If the data is not linear, then a linear relationship can
still be found by transforming the x scale. A regression line can then be fitted.
m
For example, y = x + c is a hyperbola. However, if we substitute X for 1 , the rule becomes
x
linear: y = mX + c. The graph of y versus X will be a straight line with a gradient of m and a
y-intercept of c. These values (m and c) can then be established from the graph and thus the
hyperbolic model can be determined.
Note: In a quadratic relationship, X is substituted for x2; in a cubic relationship, X is substituted
for x3.
WORKED EXAMPLE 34

It is believed that, for the data in the table below, the relationship between x and y can be modelled
by y = aaxx 2 + bbxx + c.
x

5.3

8.6

14.8

23

34.4

a Plot the values of y against x.

THINK
a

114

b Calculate the values of a, b and c (correct to


3 decimal places) and write the equations.
WRITE/DISPLAY

On a Lists & Spreadsheet page, enter


x-values into column A and y-values
into column B. Label each column x
and y respectively.

On a Data & Statistics page, move


the pointer to the horizontal and
vertical axes and select the x-values
variable and y-values variable
respectively.

Maths Quest 12 Mathematical Methods CAS for the TI-Nspire

Return to the Lists & Spreadsheet


page, then press:
MENU b
4:Statistics 4
1:Stat Calculations 1
6:Quadratic Regression 6
Change the X List to x, and the Y
List to y.
Select OK and press ENTER

Interpret the variables given on the


screen.

a = 1.252
b = 0.222
c = 4.096
y = 1.252x
1.252 2 0.222x
0.222 + 4.096
Correct to 3 decimal places.

If the relationship between the variables is not given, we have to make an assumption of a model
from the graph of the data. We then have to transform the data according to our assumption.
If the assumption was correct, the transformed data, when plotted, will produce a perfectly
straight, or nearly straight, line.
Note: In this section we will consider only the rules of the type y = ax2 + b, y = ax3 + b,
a
y = + b and so on (we will not allow for a horizontal translation), so that the appropriate
x
substitution can be made.
WORKED EXAMPLE 35

35

21

16

12

11

10

Establish the rule connecting x and y that fits these data.


THINK
1

Using either graph paper or a CAS


calculator, plot y against x.

WRITE/DRAW
y
35
30
25
20
15
10
5
0

The scatterplot appears to be a


hyperbola. Write the appropriate
formula (remember that we do not
consider horizontal translations in this
section).

1 2 3 4 5 6

Assumption: y =

a
+b
x

Chapter 2

Functions and transformations

115

Check your assumption: prepare a new


1
table by replacing values of x with
x
(leave the values of y unchanged).
1
Plot y against .
x

1
x

35

116

Comment on the shape of the graph.

If we replace

1
with X, the rule
x
becomes y = aX + b, which is the
equation of the straight line, where a
is the gradient and b is the y-intercept.
These (a and b) can be found from the
graph as follows: draw in the line of
best fit.

21

0.33

0.25

16

12

0.2
11

0.17
10

y
35
30
25
20
15
10
5
0

0.5

0.2 0.4 0.6 0.8

1
x

The graph is very close to a straight line,


therefore the assumption of a hyperbolic model is
correct.
y
35
30
25
20
15
10
5
0

0.2 0.4 0.6 0.8

y2 y1
x2 x1

Write the formula for the gradient.

m=

Select any 2 points on the line.

Using (0.17, 10) and (1, 35):

Substitute the coordinates of the points


into the formula and evaluate.

m=

10

Write the value of a.

Since a is the gradient, a = m


= 30.12.

11

Write the general equation of the


straight line.

y = mx + c

12

Substitute the value of m and the


coordinates of any of the 2 points, say
(1, 35) into the equation.

35 = 30.12 1 + c

13

Solve for c. (Alternatively, read the


y-intercept directly from the graph.)

14

State the value of b.

35 = 30.12 + c
c = 35 30.12
= 4.88
Since b is the y-intercept, b = c = 4.88
88.

15

Substitute the values of a and b into


a
y = + b to obtain the rule that fits
x
the given data.

35 10
1 0.17
25
=
0.883
= 30.12

The rule for the given data is:


y=

30.12
+ 4.888
x

Maths Quest 12 Mathematical Methods CAS for the TI-Nspire

REMEMBER

1.
2.
3.
4.

Modelling is the process of finding the rule that fits the given data.
The rule itself is called a mathematical model.
The best way to start modelling is to produce a scatterplot of the original data.
Use the scatterplot of the data to make an assumption of the model of the relationship.
a
It should be of the type y = ax2 + b, y = ax3 + b, y = + b and so on. To test the
x
assumption, transform the data accordingly. If the assumption is correct, the
transformed data when plotted will produce a straight, or nearly straight, line.
5. To find the values of a and b in the model, draw a line of best fit; a is the gradient of
the line and b is the y-intercept.
EXERCISE

2J

Modelling
1

WE32 Match each of the graphs with the appropriate model:

i y = ax2 + b
a
iii y = + b
x

ii y = ax3 + b
a
iv y = 2 + b
x

v y=a x +b
a

x
x

2
eBoo
k plus
eBook
Digital doc

Spreadsheet 076
Modelling

WE33 The data in each of the tables below exactly fit one of these rules: y = ax2, y = ax3,

a
a
,y =
or y = a x . For each set of data, plot the values of y against x and draw the
x
x2
graph. Select the most appropriate rule, and find the value of a.
y=

8.1

2.4

0.3

0.3

2.4

8.1

24

24

54

Chapter 2

Functions and transformations

117

0.5

0.08

0.08

0.5

0.5

1.5

1.13

1.6

1.96

2.26

2.5

1.25

1.5

40.5

12

1.5

10
0.5
2
12

a
+ b?
x2
iii
y

3 MC Which of the graphs below could be modelled by y =


i

ii

iv

y
x

A i only
D i , i i and i v
4

B i , i i and i i i
E i , i v , and v

C i v and v

WE34 It is believed that for the data in the table below, the relationship between x and y can
be modelled by y = ax2 + b.

3.2

4.9

14.5

29

46.8

a Plot the values of y against x.


b Plot the values of y against x2 and draw the line of best fit.
c Find the values of a and b and hence the equation describing the original data.
5 The table below shows the values of 2 variables, x and y.
x

28

13.5

12.5

2
10

4.3

41

Establish the mathematical model of the relationship between the variables, if it is known
that it is of the form y = ax3 + b.
6 The table below shows the results, obtained from an experiment, investigating the frequency of
a sound, ff, and the length of the sound wave, .

118

0.3
1130

0.5
680

10

340

110

70

40

35

Maths Quest 12 Mathematical Methods CAS for the TI-Nspire

a Plot f against .
b From the following relationships select the one which you think is suggested by the plot:
a
f = a2, f = , f = a .

1
c Based on your choice in part b, plot f against either 2,
or , draw in the line of best

fit and use it to find the rule that connects the 2 variables.
7 For her science assignment, Rachel had to find the relationship between the intensity of the
light, II, and the distance between the observer and the source of light, d. From the experiments
she obtained the following results.
d

270

1.5

120

2.5

68

43

30

3.5
22

4
17

a Use a graphics calculator to plot the values of I against d. What form of relationship does
the graph suggest?
b Nathan (Rachels older brother) is a physics student. He tells Rachel that from his studies
a
he is certain that the relationship is of the type I = 2 . Use this information to help
d
Rachel to find the model for the required relationship.
8 WE35 The table below gives the values of 2 variables, x and y. Establish the rule, connecting
x and y, that fits these data.
x

11

12

13

9 Joseph is a financial adviser. He is studying the prices of shares of a particular company over
the last 10 months.
Months

Price, $

6.00

6.80

7.45

8.00

8.50

8.90

9.30

9.65

9
10.00

10
10.30

a Represent the information graphically.


b Establish a suitable mathematical model, which relates the share price, P, and the number
of the month, m.
c Use your model to help Joseph predict the share price for the next 2 months.
eBoo
k plus
eBook
Digital doc

Investigation
Goal accuracy

Chapter 2

Functions and transformations

119

SUMMARY
Graphs of the power functions

Name
Parabola

Equation
y = a(x b)2 + c

Basic shape
y

(b, c)
0

Cubic

y = a(x b)3 + c

y=

a
+c
xb

or
y = a( x b) 1 + c
Truncus

a
+c
( x b) 2
or
y = a( x b) 2 + c

If a > 0
yc
If a < 0
yc

Turning point at (b, c)

Stationary point of
inflection at (b, c)

R \{b}

R \{c}

Horizontal asymptote
y = c, vertical asymptote
x=b

R \{b}

If a > 0
y>c
If a < 0
y<c

Horizontal asymptote
y = c, vertical asymptote
x=b

xb

If a > 0
yc
If a < 0
yc

End point at (b, c)

c
x

0
b
y

y=

y=a xb +c
or
y = a( x b) + c

Special feature

(b,
b, c)
c
x

c
0

Square root

Range

y
0

Hyperbola

Domain

1
2

(b, c)

The equation for any graph y = f (x) above can be written in the general form:
y = af (x b) + c.
This form can be used to describe transformations of all of the functions considered.
For all of the above functions:
1. a is the dilation factor: it dilates the graph from the x-axis.
2. When an equation for these types of graphs is put into its general form of y = af (x b) + c, the horizontal
dilation can be described in terms of a vertical dilation.
3. If a < 0, the basic graph is reflected in the x-axis.
4. f (b x) or f (x + b) is the reflection of f (x + b) in the y-axis.
5. b translates the graph b units along the x-axis (to the right if b > 0, or to the left if b < 0).
6. c translates the graph c units along the y-axis (up if c > 0, or down if c < 0).

120

Maths Quest 12 Mathematical Methods CAS for the TI-Nspire

To put equations into general form:


7. If the coefficient of x is a number other than 1, to find the value of b and a, the equation should be
transposed to make the coefficient of x equal to 1.
For example,
y = (3x + 5)2 + 4
= [3( x + 35 )]2 + 4
= 32 ( x + 35 )2 + 4
= 9( x + 35 )2 + 4
Hence, a = 9, b =

5
.
3

The absolute value function

y = |x| means

y = x, if x 0
and y = x, if x < 0
y
y = |x|
x

Domain: R Range: R+ {0}


To sketch y = | f (x)|:
1. Sketch the graph of y = f (x).
2. Reflect the portion of the graph that is below the x-axis in the x-axis.
Or:
1. Express the function in hybrid form with specific domains where the absolute value expression is positive
and negative.
2. Sketch each rule for the specified domain.
For functions of the form y = a|| f (x)| + c, a and c have the same impact on the graph of the absolute value
function, as on the graphs of all other functions discussed in this section.
Transformations with matrices

The use of matrices to map transformations of points and equations can be summarised as follows, where
(x, y) is the image of the point (x, y) under the transformation.
x
T
y
x
T
y
x
T
y
x
T
y

x' 1
= =
y' 0
x' 1
= =
y' 0

0 x x
= represents a reflection in the y-axis.
1 y y
0 x

x
= represents a reflection in the x-axis.
y
x' a 0 x ax
= =
= represents a dilation of a factor of a from the y-axis.
y' 0 1 y y
x' 1 0 x x
= =
= represents a dilation of a factor of a from the x-axis.
y' 0 a y ay
1 y

Transformations can be combined to represent more than one transformation. For example,
4x + 2
x ' 4 0 x 2

=
+
=

0 1 y

y'
y
3
+
3


2
2

x = 4x + 2
y
y =
+3
2

Chapter 2

Functions and transformations

121

describes the following: dilation by a factor of 4 from the y-axis, a dilation by a factor of 2 from the x-axis,
reflection in the x-axis, a horizontal translation of +2 and a vertical translation of +3.
Sum and difference of functions

For the graph of the sum/difference function, dom ((f (x) g(x)) = dom f (x) dom g(x). The graph of the
sum/difference function can be obtained by using the addition of ordinates method.
For the product function, dom ((ff (x)g(x)) = dom f (x) dom g(x). Some features of the graph of the product
function are as follows:
the x-intercepts of f (x)g(x) occur where either f (x) or g(x) have their x-intercepts
f (x)g(x) is above the x-axis where f (x) and g(x) are either both positive or both negative
f (x)g(x) is below the x-axis where one of the functions f (x) or g(x) is positive and the other is negative.
Composite functions and functional equations

For the composite function f (g(x)) to be defined, the range of g must be a subset of the domain of f.
f
Furthermore, if f (g(x)) is defined, the domain of f (g(x)) equals the domain of g(x).
Equations involving algebra of functions, for example f (2x
(2 ) = 2f
2 (x), are generally tested to determine if they
are true for particular functions.
To determine if an equation is true for a particular function, consider the LHS and RHS of the equation
separately to determine if the equation holds true for all values of x.
Alternatively, you may find a particular x-value for which the equation does not work; that is, a
counterexample.
These types of equations can be investigated by defining the functions on the CAS calculator and then
testing the algebraic function equation.
Modelling

Modelling is the process of finding the rule (mathematical model) that fits the given data.
To model:
1. Plot the original data on graph paper or use a CAS calculator.
2. Make an assumption of the model.
3. Transform the data in accordance with your assumption.
4. Check the assumption by plotting the transformed data (if correct, the graph will be a straight or nearly
straight line).
5. Draw in a line of best fit.
6. Find the equation of the line (y = mx + c).
1
7. Replace x in the equation with the transformed variable (for example, x2, ).
x

122

Maths Quest 12 Mathematical Methods CAS for the TI-Nspire

CHAPTER REVIEW
SHORT ANSWER

1 For the function


x 3)2 4:
a state the coordinates of the turning point
b state the domain and range
c sketch the graph.
a
2 The graph of y =
+ c has a vertical
xb
asymptote at x = 2 and a horizontal asymptote at
y = 1.
a Find the values of b and c.
This graph then undergoes the following
transformations:
reflection in the x-axis
dilation by a factor of 3 from the x-axis
horizontal shift of 2 units right.
b If the intersection of the two graphs is at (m, 2),
find the value of m.
c Hence find the equation of the transformed
graph.
6x 5
3 Express the function f ( x ) =
in the form
3x + 1
b
a+
. Hence describe the transformations
x+c
required to produce this curve from the graph of
1
y= .
x
4 The graph of a cubic function has a stationary point
of inflection at (1, 1). It cuts the y-axis at y = 4.
Find the equation of the graph.
1
5 The graph of y = was dilated by the factor of
x
4 from the x-axis, reflected in the x-axis and then
translated 2 units to the left and 1 unit down.
a State the equation of the asymptotes.
b State the domain and range.
c State the equation of the new graph.
d Sketch the graph.
6 a State the changes necessary to transform the
1
y
graph of y = 2 into
x
the one shown.
b Find the equation of
the graph.

1
23

7 The domain of a truncus is R \{1}, the range is


(, 2) and the graph cuts the y-axis at y = 3. Find
the equation of the function.
8 The basic square root curve was reflected in both
axes and then translated so that its intercepts at
the axes were (0, 1) and (5, 0). Find the size and
the direction of the translations; hence, find the
equation of the new graph.
2
2,
9 a Sketch the graph of y = 2
( x + 2)2
clearly showing the coordinates of the cusps,
the intercepts with the axes and the position of
the asymptotes.
b State the domain and range of the graph in a.
10 The graph of ff: [5, 1] R where
f (x
( ) = x3 + 6x
6 2 + 9x is shown.
y

a On the same set of axes sketch the graph of


y = /f (x)) /.
b State the range of the function with the rule
y = /f (x)) / and domain [5, 1].
[ VCAA 2006]

11 The point (1, 3) undergoes a translation given by


a
the matrix to (2, 0). Find a and b and describe
b
the transformations involved.
12 The point (1, 2) undergoes a series of a 0

transformations given by the matrices


and
0
b

1
then to (7, 4).
2
a Find the values of a and b.
b Find the image under the transformations of:
i y=2 x
ii y = x3 + x
13 A point on a curve ((x, y) undergoes a
a 0
transformation descibed by
to (x', y'),
0 2
where a is a real constant such that a > 0.

Chapter 2

Functions and transformations

123

a Find the values of x and y in terms of a, x' and y'.


b If the point is on the curve y = 2x
2 2 x, find
the image of the curve in terms of a under this
transformation.
c If the point (3, 6) is on the transformed curve,
find the value(s) of a and hence the rule of this
image.
14 The graph of the function
f : (2, 1) R, f (x
( ) = x3 + 2x
2 2 is shown below.
y

f x) = x3 + 2x
f(
2 2

2
1

(2, 0)

2 1.5 1 0.5 0
1

0.5

Let g(x) = f (x) + 1, and sketch this graph on the same


set of axes. Hence, sketch ((f + g)(x).
15 The graph of the function f:(
f 2, 2), f (x
( ) = /x / is
shown below.
(2, 2)

f x) =x
y f(
2
(2, 2)
1

0
2 1
1

1 2 x

Let g(x) = f (x) + 1 and sketch the graph of this


function on the same set of axes. Hence, sketch
f (g(x)).
16 Let f (x
( ) = x2 determine which of the following
relationships are true.
a f (x) f (y) = f (x) f (y)
b f (x) f (y) = f (x) + f (y)
c f (x) + f (y) = f (x) + f (y)
d f (x) f (y) = f (x) + f (y)
17 The data in the table below exactly fit one of these
a
models: y = ax3, y = 2 or y = a x .
x
x

25

10

6.25

20
0.25

25
0.16

a Plot the values of y against x and use the


scatterplot to choose a suitable model.
b Plot the values of y against either x3, 1 or x
x2
(depending on your choice in part a ). Did you
choose the right model? Explain your answer.
c Find the value of a.

124

1 The equation of a parabola is given by


y = m 2(x
2( + 3)2, where m > 0. The increase in
m will result in:
A the graph being thinner
B the graph being wider
C the increase of the domain
D the increase of the range
E the graph being shifted further to the right
2 The coordinates of the turning point of the parabola
y = 2(3x + 6)2 3 are:
A (6, 3)
B (6, 3)
C (2, 3)

D ( 2, 3)
E ( 2, 3)

(1, 3)

MULTIPLE CHOICE

bx 3)3 + 1 is dilated in the


3 The graph of y = 23 (bx
y direction by the factor of:
3
C 2b
B 23 b
A 23
3
2
3
D b
E
3b3
4 The graph of y = 2 (3 + 4x
4 )3 has a stationary point
of inflection at:
A ( 43 , 2)

B (

3
, 2)
4

3
, 2)
4

C (3, 2)
D (
E (3, 2)
2
5 If f ( x ) = + 1, then f (x
( ) + 2 will have:
x
A the horizontal asymptote y = 2
B the horizontal asymptote y = 1
C the horizontal asymptote y = 3
D the vertical asymptote x = 2
E the vertical asymptote x = 1
6 The equation of the graph shown is likely to be:
2
y
A y=
1
x2
2
B y =1
x+2
2
x
2
1
C y=
2
x +1
2
1
D y=
x+2
2
E y=
1
x+2
7 If the graph of y = 1 is reflected in the y-axis,
x
translated 43 units to the right and 2 units up, the
resulting graph would have the equation:
1
3
A y=
+
x2 4
3
+2
B y=
4x

Maths Quest 12 Mathematical Methods CAS for the TI-Nspire

1
3 4x
1
+2
D y=
4x 3
1
+2
E y=
3 4x
C y = 2

8 Which of the following is not true for the graph at


right?
y
A The vertical asymptote
is x = 2.
B The horizontal
asymptote is y = 4.
x
2
C The domain is R\{2}.
D The range is R\{4}.
4
E The value of the
y-intercept is greater than 4.
9 To obtain the graph shown, we need to:
A translate the graph of
y
1
y = 2 one unit to the left
x
and reflect in the x-axis
x
1
B translate the graph of
1
y = 2 one unit to the left
x
and reflect in the y-axis
1
C translate the graph of y =
one unit to the
x2
left and reflect in both axes
1
D translate the graph of y = 2 one unit to the
x
left and dilate it in the x direction
E none of the above
10 The equation of this graph is of the form:
A y = a x m + n, a > 0
B y = a m x + n, a > 0

y
(m, n)
x

C y = a x m + n, a < 0
D y = a m x + n, a < 0
E y = a x + m + n, a < 0
11 The equation of this graph
could be:
A y=d ax

y
d

B y= xa +d
C y=d xa
D y = c a x
E y= a x c

b a
c

12 The domain of the function f ( x ) = 2 x 1 + 3 is:


A [1, )
B [3, )
C [0.5, )
D [2, )
E [1, )
1
13 The range of the function y = 3
2 is:
x2
A (1, )
B (, 5]
C [3, )
D (, 2)

E ( , 2]
14 The equation of the graph shown in the diagram
below is best described by:
y
A y = |x + 2| + 2
2
B y = 2 |x + 2|
C y = |2 x| + 2
x
2
D y = 2 |2 x|
E y = |x + 2| 2
15 The value(s) of k for which |2k + 1| = k + 1 are:

2
2
B
A 0 only
C 0 or
3
3

1
2
E
or 0
D 0 or
2
3
[ VCAA 2006]
16 Under the transformation T : R2 R2 of the plane
x 2 0 x 1
defined by T =
+ , the
y 0 1 y 3
image of the curve y = | x | has the equation:
A
B
C
D
E

y=

1
2

x 1 + 3

1
2

x 1 3

y = | 22x + 1| 3
y = 2 | x + 1| 3
y = 2 | x 1| + 3
y=

17 If g:[2, 4] R, where g(x


( ) = x2(x
(x
( 3), and
h:(0, 3] R, where h(x
( ) = 3 x, then the
(x
function f (x
( ) such that f (x
( ) = g(x
( ) h(x
(x
( ) is defined
(x
by the rule:
A f:
f R R, where f (x) = x2(x 3)2
B ff: (0, 3] R, where f (x) = x2(x 3)2
C ff: [2, 3] R, where f (x) = x2(x 3)2
D ff: (0, 4] R, where f (x) = x2(x 3)2
E ff: [2, 3] R, where f (x) = x2(x 3) (3 + x)
18 Given the function f ( x ) = x 3 then f h could
exist if h(x
( ) was defined as:
(x
A h(x) = x + 2
B h(x) = x2 3
C h(x) = x3
D h(x) = x2 + 3
E h( x ) = x 2

Chapter 2

Functions and transformations

125

19 The data in the following table exactly fit one of


these models: y = ax2, y = ax3 or y = a x .
x

0.3

2.4

8.1

The value of a is:


A 2.4
C 2.7
E 0.3

4
19.2

B 1.2
D 0.9

20 For certain data the values of y were plotted against


1
and the line of best fit was drawn as seen on
x
the diagram below. The model that relates the
variables x and y is:
y
A y = 20x
20 1
(1, 19)
B y = 19x + 1
x
1
C y=
20
19
D y = 1
x
(0.1, 1)
20
1
1
E y=
x
x

EXTENDED RESPONSE

1 The graph of y = f (x
( ) is shown at right.
a Sketch the graph of each of the following functions on the same set of axes
with the original graph and give the coordinates of the points A, B, C and D.
i y = f (x)
ii y = f (x)
iii y = f (xx 2)
iv y = f (x) + 3
v y = 2f
2 (x)
vi y = 1 f (x + 1)
b Maya, a fabric designer, wishes to use the curve of y = f (x) (red)
to create a wavy pattern as shown in the diagram at right.
If she wants the waves to be 2 units apart vertically, suggest the best way
she could alter the equation of y = f (x). (Remember a fabric has a fixed
width!)

D(4, 6)

B
C(2, 3)
A 2
3 2
7 x

2 units
apart

2 Consider the function f:R R, f(


f x) = (x 1)2 (x 2) + 1.
a The coordinates of the turning points of the graph of y = f(
f x) are (a, 1)
25
and (b, 27 ). Find the values of a and b.
b Find the real values of p for which the equation f(
f x) = p has exactly one solution.
c For the following, k is a positive real number.
x
i Describe a sequence of transformations that maps the graph of y = f (x) onto the graph of y = f 1.
k
x
y
=
f

1
ii Find the x-axis intercepts of the graph of
in terms of k.
k
d Find the real values of h for which only one of
EXAM TIP
the solutions of the equation f (x + h) = 1 is positive.
Students must ensure that they show
[ VCAA 2004]

3 The graph of the function f:


1
f ( x ) = 2 is shown below.
x
(0.5, 4)

y
4

(2, 0.5)

(0.5, 2) R,

(0.5, 4)

[Assessment reports 1 and 2 2007 VCAA]

f x) = 12
f(

2
(2, 0.25)

126

2.5 2 1.5 1 0.5 0

their working if a question is worth more than one


mark, students risk losing all available marks if only
the answer is given and it is incorrect. The
instructions at the beginning of the paper state that if
more than one mark is available for a question,
appropriate working must be shown.
When students present working and develop their
solutions, they should use conventional mathematical
expressions, symbols, notation and terminology.

(2, 0.25)
0.5

1 1.5

2 2.5 x

Maths Quest 12 Mathematical Methods CAS for the TI-Nspire

The graph of y = f (x) is to be transformed to become the graph of y = f (2x


(2 ) + 1.
a Describe these two transformations.
b Create matrices to represent these transformations.
c Use these matrices to find the images of the points (1, 1) and (2, 0.25) under these transformations, and
use these values to deduce the images of the points (1, 1) and (2, 0.25).
d On the same axes, sketch this transformed function, showing the coordinates of the four points from c
above.
e Using any method, find a rule for this transformed function.
4 A proposed section of a ride at an amusement park is to be modelled on the curve y = 1 (600 x + 25 x 2 x 3 ),
500
where y is the height (in metres) of the ride above ground level and x is the horizontal distance (in metres).
The x-axis represents ground level. It will travel through a tunnel from A to C; B is the lowest point in the
tunnel and D is the highest point on the ride.
y

Ex

C
B

a Find the horizontal distance from A to E.


b Find the greatest depth below ground level and the maximum height above ground level that the
rollercoaster will reach in this section (correct to 2 decimal places).
c Describe the impact that a dilation by a factor greater than 1 from the x-axis would have on:
i the maximum depth and maximum height from b
ii the point at which the rollercoaster would emerge from the tunnel
iii the gradient of the slope at this point.
5 Lena and Alex are planning to buy a new house. Theyve been watching the prices of 3-bedroom houses in a
specific area, where they want to live, for the whole year. During each month they collected the data and then,
at the end of the month, they calculated the average price for that month. The results of their calculations are
shown in the table below. (The prices given are in thousands of dollars.)
Month
Price

10

11

12

240 248 255 261 266 271 273 274 275 274 272 270

a Plot the prices against the months. What model does the graph
suggest?
b If the model of the form y = a(x b)2 + c is to be used for these
data, what is (judging from the graph) the most suitable value
for h?
c Plot the values of y (the prices) against (x b)2, where b is the
value youve selected in part b . Comment on the shape of the
graph.
d Draw a line of best fit and find its equation. Hence, state the
values of a and c in the model.
e Write the equation of the model.
f According to the Real Estate Institute, the property market is on a steady rise (that is, the prices are
going up and are likely to rise further). Do the data collected by Lena and Alex support this theory?
g Use the model to predict the average price for the next 2 months.

Chapter 2

Functions and transformations

127

h Lena and Alex were planning to spend no more than 250 000 for their new house. Several months ago the
prices were in their range, but they could not find what they wanted. If the prices are going to behave
according to our model, how long do they have to wait until the prices fall back into their range?
6 An eagle soars from the top of a cliff that is 48.4 metres above the ground and then descends towards
unsuspecting prey below. The eagles height, h metres above the ground, at time t seconds can be modelled by
a
the equation h = 50 +
, where 0 t < 25 and a is a constant.
t 25
a Find the value of a.
b Find the eagles height above the ground after i 5 seconds ii 20 seconds.
c After how many seconds will the eagle reach the ground?
d Comment on the changes in speed during the eagles descent.
e Sketch the graph of the equation.
After 24 seconds, the eagle becomes distracted by another bird and reaches the ground exactly 2 seconds
later. For this second part of the journey, the relationship between h and t can be modelled by the equation
h = a(t 24)2 + c.
f Find the values of a and c.
g Fully define the hybrid function that describes the descent of the eagle from the top of the cliff to the
ground below.
eBoo
k plus
eBook
Digital doc

Test Yourself
Chapter 2

128

Maths Quest 12 Mathematical Methods CAS for the TI-Nspire

eBook plus

ACTIVITIES

Chapter opener
Digital doc

10 Quick Questions: Warm up with ten quick


questions on functions and transformations. (page 57)
2A

Transformations of the parabola

Digital docs

Spreadsheet 132: Investigate transformations.


(page 59)
Spreadsheet 108: Investigate the quadratic function
in power form. (page 61)
2B

The cubic function in power form

Digital docs

Spreadsheet 015: Investigate the cubic function in


power form. (page 65)
Spreadsheet 236: Investigate graphs of functions.
(page 70)
2C

The power function (the hyperbola)

Tutorial

WE 9 int-0522: Watch a worked example on


sketching the graph of a hyperbola. (page 74)
Digital docs

Spreadsheet 051: Investigate the hyperbola. (page 72)


Spreadsheet 236: Investigate graphs of functions.
(page 71)
WorkSHEET 2.1: Find the domain, range,
coordinates of turning points and equations of
asymptotes of various graphs. (page 78)
History of Mathematics: Investigate the history of
major curves. (page 78)
2D

The power function (the truncus)

Digital doc

Spreadsheet 236: Investigate graphs of functions.


(page 85)
2E

The square root function in power form

Tutorial

WE 16 int-0523: Watch a worked example on implied


domain and range. (page 88)
Digital doc

Spreadsheet 236: Investigate graphs of functions.


(page 92)
2F

The absolute value function

Tutorials

WE 20 int-0524: Watch a worked example on


sketching the graph of an absolute value function.
(page 93)
WE 21 int-0525: Watch a worked example on
expressing an absolute value function as a hybrid
function. (page 94)

Digital docs

WorkSHEET 2.2: Identify transformations, state


domain and range, sketch graphs of power functions
and absolute value functions. (page 96)
Spreadsheet 002: Investigate graphs of absolute
value functions. (page 96)
2G

Transformations with matrices

Interactivity

Transformations with matrices int-0247: Consolidate


your understanding of using matrices to transform
functions. (page 96)
Tutorial

WE 25 int-0526: Watch how to use matrices


to determine the resultant equation after
transformations. (page 100)
2H

Sum, difference and product functions

Tutorial

WE 27 int-0257: Watch how to use addition of


ordinates to sketch the sum of two functions.
(page 104)
2J

Modelling

Digital docs

Spreadsheet 076: Investigate modelling with


functions. (page 117)
Investigation: Goal accuracy. (page 119)
Chapter review
Digital doc

Test Yourself Chapter 2: Take the end-of-chapter test


to test your progress. (page 128)
To access eBookPLUS activities, log on to
www.jacplus.com.au

Chapter 2

Functions and transformations

129

EXAM PRACTICE 1
SHORT ANSWER

15 minutes

1 The functions f
g are graphed below. On the
same axes sketch a graph of f + g.
f

CHAPTERS 1 TO 2

1
a
xb
1
C y=
+b
x+a
1
E y=
+a
x+b
A y=

B y=

+b
ax
1
D y=
a
xb

2 The graph sketched below is best represented by


the rule:

1 mark

2 Write down the maximal domain of f : X R


where f ( x ) = x 2 3 .

1 mark

3 For y = (2x
(2 3)2 1:
a write down the y-coordinate of the turning
point
b determine the equation of the axis of
symmetry.
2 marks

4 a Using the factor theorem, show that x + 2 is a


factor of x3 7x 6.
b Given x3 7x 6 = (x + 2)Q(x) where Q(x) is a
quadratic factor, determine Q(x).
2 marks
2x
2

5 For what values of c does the graph of y =


+c
intersect the graph of y = x2 + x 2 at two distinct
points?
2 marks

6 Sketch the graph of the function


f :[2,5) R, f (x
( ) = | 3(x
3( 3)2 10 |.
2 marks

MULTIPLE CHOICE

10 minutus

Each question is worth 1 mark.


1 Given a, b R, this graph could have the rule:

A
B
C
D
E

y = (xx a)(xx b)2(xx c)2


y = (xx a)(xx c)2
y = (xx a)3(xx c)2
y = (xx a)(xx c)
y = (xx a)(xx b)(xx c)

3 The coefficient of the term in x4 in the expansion of


3
(2 x 2 )5 is:
x
A 240
B 72
C 60
D 72
E 720
4 What is the equation of the horizontal asymptote of
the graph of y =
A y = 4
3
D y=
4

3x 2
?
4x
B y = 3

C y = 1

E y=3

5 The quadratic function f : D R, f (x


( ) = 2(x
2( + 1)2 5
has a domain, D,, of [0, 3]. The range of f is:
A [1, 3]
C [5, 27]
B R
D [3, 27]
E [5, )

y=b
x=a
0

130

6 The simultaneous linear equations 3x + ay = 12 and


ax + 3y = 4a have infinitely many solutions for:
A a [3,3]
B a=3
C a = 3 or a = 3
D a=9
E a=0

Maths Quest 12 Mathematical Methods CAS for the TI-Nspire

EXTENDED RESPONSE

40 minutes

1 Severe tropical cyclone Vance devastated the town of Exmouth in northern WA in 1999. It produced a
measured wind gust of 267 km/h, which is the highest wind speed ever recorded on mainland Australia. In
order for it to be first categorised as a cyclone, its wind speed needed to exceed 119 km/h.
Several years later, it is a peaceful day, with no wind detectable at 1 pm. By 3 pm, however, the wind
speed is gusting to 200 km/h, and the residents know they are in trouble.
a Let t be the time in hours after noon and v be the wind speed. Establish a linear model of the form
v = at + b to represent the relationship between the wind speed and time.
2 marks
b Using this model, determine to the nearest minute when the wind speed will be high enough for
classification as a cyclone.
1 mark
c Predict to the nearest minute when the cyclone will break the record for the highest wind speed ever
recorded.
1 mark

d Explain why the linear model is unsatisfactory as a model for the cyclones behaviour.
e

1 mark

i The wind speed actually peaks at 256 km/h at 5 pm. Use this data and the wind speed at 1 pm to help
to create a quadratic model relating wind speed (v) to hours after noon (t). How well does this model
match all the data provided?

2 marks

ii The wind speed actually returns to zero at 1 am the following morning, 12 hours later. Discuss how
well this result matches the quadratic model.
f

1 mark

i Establish a second quadratic model that exactly matches the following data:
Time

Wind speed

1 pm

0 km/h

5 pm

256 km/h

1 am

0 km/h
1 marks

ii Evaluate how well this model represents the relationship between time and wind speed.
g It has been suggested that a cubic function would be a better model for the data. Use all the data
provided in order to establish a cubic model.

1 mark
2 marks

1020
1010
1000
990
980
970
960
950
0 km

Eye

Pressure
Wind speed

175
150
125
100
75
50
25

Wind speed (km/h)

Pressure (hPa)

h The graph below shows how air pressure in hectopascals (P hPa) and wind speed (v km/h) relate to
1
distance (x) across a cyclone. It has been suggested that a truncus (general form v ( x ) 2 ) could represent
x
the shape of the relationship between speed and distance across the cyclone.

500 km

i What is the equation to the vertical asymptote for


this relationship?
ii Determine a rule for the relationship between speed (v) and
distance (x).
iii Determine a rule for the relationship between air pressure (P) and
distance (x).

1 mark
2 marks
2 marks

eBoo
k plus
eBook
Digital doc

Solutions
Exam practice1

Exam practice 1

131

3A
3B
3C
3D
3E
3F

The index laws


Logarithm laws
Exponential equations
Logarithmic equations using any base
Exponential equations (base e)
Equations with natural (base e)
logarithms
3G Inverses
3H Literal equations
3I Exponential and logarithmic modelling

Exponential
and logarithmic
equations
arEas oF sTudy

Logarithm and exponent laws


Identification of appropriate solution processes
for solving logarithmic and exponential
equations
Numerical solutions of exponential and
logarithmic equations
Solution of literal equations such as emx + n = k

Application of algebraic and logarithmic


properties to the simplification of expressions
and the solution of equations
Finding the rule of an inverse function involving
logarithmic and exponential equations
Application of logarithmic and exponential
functions in modelling practical situations
eBook plus

3a

Digital doc

The index laws

10 Quick Questions

A number in index form has two parts, the base and the index, power, exponent or logarithm.
A number in index form is represented like this:
Index, power, exponent or logarithm

ax
Base

The index laws are summarised below.


ax = x a

ax ay = ax - y

a y = ax =

(ax)y = axy

(ab)x = axbx

a0 = 1 (a 0)

ax
a
b = b x , b 0

132

ax ay = ax + y

1
(a 0)
ax

maths Quest 12 mathematical methods Cas for the Ti-nspire

( a)
y

Worked Example 1

Simplify

( 2 x 2 y3 ) 3 3( xy4 )2
.
6 x 4 2 xy4

Think

Write

Remove the brackets by multiplying the indices.

Add the indices of x and add the indices of y.


Simplify 23 to 8 and multiply the whole
numbers.

Subtract the indices of x and y. Divide 24 by 12.

(2 x 2 y 3 )3 3( xy 4 )2 23 x 6 y 9 3 x 2 y8
=
6 x 4 2 xy 4
12 x 5 y 4
=

24 x8 y17
12 x 5 y 4

= 2x3y13

For negative indices and fractional or decimal indices, the same rules apply.

Worked Example 2

Write in simplest form:


2

0.4

a 64 3 b 32
2

c 125

using a CAS calculator.

THINK

WRITE/display
x

2
y

Rewrite using the index law a y = a x .

Rewrite using a x = ( y a ) x .

= ( 3 64 )2

Simplify by taking the cube root of 64.

= 42

Square 4.

a 64 3 =

64 2

= 16
0.4

Write as a fraction with a positive


index.

Change 0.4 to 104 .

Simplify the fractional index.

Rewrite using the index law a y = a x .

Simplify by taking the 5th root of 32.

Square 2.

b 32

=
=
=

1
320.4
1
4

3210
1
2

32 5
x

1
5

( 32 )2
1
= 2
2
1
=
4

Chapter 3 Exponential and logarithmic equations

133

To simplify 125 3 with a CAS


calculator, open a Calculator page.
2

Complete the entry line as: 125 3 .


Then press ENTER .
Note: If the calculator is set to
Approximate, the answer will be
displayed as a decimal.

2
3

Write the answer.

125

= 25

Worked Example 3

Simplify, leaving your answer with positive indices:


a

a 2 b4 ( a 3 b 4 )

1 1
a2 b
b
1 2
3 b

using a CAS calculator.

THINK
a

Remove the brackets by multiplying


the indices.

Add the indices of a and of b.


Place a5 in the denominator with a
positive index.

WRITE/display

On a Calculator page,
complete the entry line using
the fraction template
12 1
a b
as: 3 1 b 2

- -1

a a 2b4 (a3b 4)

= a 2b4 a 3b4
-

= a 5b8
=

b8
a5

Then press ENTER .

Write the answer.


The answer is expressed with
positive powers.

12 1
a b
1 2
3 b

3 a

If the expression contains different numbers that do not have the same base, write each number
as a product of prime factors.

134

Maths Quest 12 Mathematical Methods CAS for the TI-Nspire

WorkEd ExamplE 4

Simplify

3 n 6 n + 1 12 n 1
:
32 n 8 n

a showing working, and

Think
a

WriTE/display

Write each number as the product of


prime factors.

Remove the brackets.

In the numerator, add the indices of


numbers with base 3 and add indices of
numbers with base 2.
Subtract the indices.

3n 6 n + 1 12 n 1
32 n 8 n
3n (3 2) n + 1 (22 3) n 1
=
32 n ( 2 3 ) n
3n 3n + 1 2 n + 1 2 2 n 2 3n 1
32 n 2 3 n
3
n
3
n

1
3 2
= 2n
3 23 n
=

-1

Write the term with a negative index in


the denominator with a positive index.

= 3n 2
1
= 3n
2

Simplify.

On a Calculator page, complete the entry


line using the fraction template as:

b with a CAS calculator.

3n
2

3n 6 n + 1 12 n 1
32 n 8 n
Then press ENTER .

Write the answer.

3n 6 n + 1 12 n 1 3n
=
2
32 n 8 n

WorkEd ExamplE 5

eBook plus

Simplify, leaving your answer with positive indices.


3
3
1

a x 2
b
+ 1
2
1
x
x +2 x 2
Think
a

Tutorial

int-0528
Worked example 5

WriTE

Rewrite the expression with positive


indices.

a x

x 2
1
= 2 3x 2
x

Chapter 3

Exponential and logarithmic equations

135

Find the lowest common denominator.

1 3x 4
2
x2
x

1 3x 4
x2

1
3
b
+
=
1 1
x + 2 x 2

Simplify.

Rewrite the question using positive


indices.

Find the common denominator for the


terms in the brackets and simplify.

Follow the process for division of fractions


(change the division sign to a multiplication
sign and invert the second fraction).

Find the common denominator.

Expand the brackets on the numerator.

Simplify the numerator by adding like


terms.

Factorise the numerator and write the


answer.

1
x2
3x 2 2
2
x
x

3
x
+
1 + 2x 1 2x
x x

3x
x
+
1 + 2x 1 2x

3 x (1 2 x ) + x (1 + 2 x )
(1 + 2 x ) (1 2 x )

3x 6 x 2 + x + 2 x 2
(1 + 2 x ) (1 2 x )

4x 4x2
(1 + 2 x ) (1 2 x )

4 x (1 x )
(1 + 2 x ) (1 2 x )

REMEMBER

1. axay = ax + y
ax
2. ax ay = ax y or y = a x y
a
3. (ax)y = axy
4. a0 = 1, a 0
1
1
5. a x = x and x = a x
a
a

6. a y = a and a y = a x = ( a ) x
Exercise

3A

The index laws


1 WE1

136

Simplify:

x3

x4

( x 2 )3 x 5
( x 5 )2

b x7 x2
f

5x 2 y 4 4 x 5 y
22 x 3 y 2

Maths Quest 12 Mathematical Methods CAS for the TI-Nspire

c (x2)5
2 3
4 2
g (2 xy ) 5( x y )
4 x 5 y 3 3x 2 y 3

d (x 3)2

2 WE2
a

Simplify:
3

2
27 3

b 25
2

27 3
d
64

c 810.25

243
e
32

3
5

3 Simplify:
a

x4 x5
x3

c 361.5

b 16 4

e 256
81

d 9 2
49
4 WE3

3
4

Simplify, leaving your answer with positive indices.


1

3x 3y2

-4

(x2y)

3
1 2
x 2y2

b
5

3 2
9x 2 y2

1 2
2x 2 y 3

2
a2b c
d 1

3a 2 bc 2

1 1 2
9x 5 y 2

2
a 3 b3
3 1 2
a c

5 Simplify:
4y-1

a x

1
16 2

(x 2y3) 1

1
2 1 2
x5y 4

6 WE4 Simplify:
a 2n 4n + 1 8n - 1
d

32 2

3
92

1 2 1
83 x 3 y 2

3 3
a 2b4
d
ab 2

1
2 12
4x 5 y 2

1 3
5x 3 y 4

b 3n 9n - 1 27n + 1

9a 3 b 2
2 3
4a b

c 2n 3n + 1 9n

16

7 Simplify:
a 2n - 1 3n 6n + 1

52 3

125 9

27
5

8 WE5 Simplify, writing your answer as a single fraction with positive indices.

a x 1 +

b (x 1 + x 2) 2

x
1
1
+
c 1
x +1 x 1 1

-1

d 2x(x2 - y2) 1 - (x - y)

9 MC 3 x + 3x is equal to:
A 1

1 + 32 x
3x

C 3

x2

D 6

x
E 1 + 3
3x

Chapter 3 Exponential and logarithmic equations

137

3B

Logarithm laws
If a > 0, then N = ax loga (N) = x.
For example, an expression in index form can also be rewritten in logarithmic form.
8 = 23 log 2 (8) = 3
Since 0 then loga (0) is undefined.
a0 = 1 loga (1) = 0
a1 = a loga (a) = 1
Let m = ax loga (m) = x and n = ay loga (n) = y.
mn = ax ay loga (mn) = x + y
= ax + y
= loga (m) + loga (n)
x
m

m a
= log a = x y
n
n ay
= ax - y
= loga (m) loga (n)
mp = (ax)p loga (mp) = px
= axp
= p loga (m)
ax

Change-of-base rule
Suppose b = ax, then loga (b) = x.
Consider N = by, then logb (N) = y.
But N = by = (ax)y = axy.
Therefore, loga (N) = xy
= loga (b) logb (N).
Thus, log b ( N ) =

log a ( N )
.
log a (b)

This is called the change-of-base rule.

Worked Example 6

Evaluate:
a log2 (1) b log5 (5).
Think

Write

Log of 1 to any base is equal to zero:


loga (1) = 0.

a log2 (1) = 0

If the number and base are equal the answer is 1:


loga (a) = 1.

b log5 (5) = 1

Worked Example 7

Write in index form:


a log2 (8) = 3 b logx (81) = 4.
Think

138

Write

a Use ax = y loga (y) = x.

a log2 (8) = 3 23 = 8

b Use ax = y loga (y) = x.

b logx (81) = 4 x4 = 81

Maths Quest 12 Mathematical Methods CAS for the TI-Nspire

WorkEd ExamplE 8

Simplify:
a log10 (5) + log10 (2)

b log4 (20) - log4 (5)

c log2 (16)

Think
a

d log 5

( x ).
5

WriTE
a log10 (5) + log10 (2) = log10 (5 2)

Rewrite using loga (mn) = loga (m) + loga n.

Simplify.

= log10 (10)

Simplify using loga (a) = 1.


m
Rewrite using log a = log a (m) log a ( n).
n

=1

20
5

b log 4 (20) log 4 (5) = log 4

Simplify.

= log 4 (4)

Simplify using loga (a) = 1.

=1

Rewrite 16 as a number with base 2.

Rewrite using loga (mp) = p loga (m).

= 4 log2 (2)

Simplify using loga (a) = 1.

=4

Rewrite using y a = a y .

Rewrite using loga (mp) = p loga (m).

c log2 (16) = log2 (24)

d log 5

( x ) = log x
5

1
5

= 5 log 5 ( x )

WorkEd ExamplE 9

Simplify log3 (27) + log3 (9) - log3 (81).


Think

WriTE/display

On a Calculator page, complete the entry line


as: log3 (27) + log3 (9) + log3 (81)
Then press ENTER .

Write the answer.

log3 (27) + log3 (9) +log3 (81) = 1

WorkEd ExamplE 10

eBook plus

Simplify:
a 2 + log10 (3)

b 3 log3 (6) - 3 log3 (18)

log 3 ( 9 )
c
.
log 3 ( 27 )

Chapter 3

Tutorial

int-0529
Worked example 10

Exponential and logarithmic equations

139

Think
a

Write

Write 2 as 2 log10 (10) because log10 (10) = 1.


(mp)

a 2 + log10 (3) = 2 log10 (10) + log10 (3)

= log10 (102) + log10 (3)

= p loga (m).

Rewrite using loga

Rewrite using loga (mn) = loga (m) + loga (n).

= log10 (102 3)

Write 102 as 100.

= log10 (100 3)

Multiply the numbers in the brackets.


(mp)

= log10 (300)
b 3 log3 (6) 3 log3 (18) = log3 (63) log3 (183)

= p loga (m).

Rewrite using loga

Rewrite using
m
log a = log a (m) log a ( n).
n

Write 63 as 6 6 6 and 183 as


18 18 18.

Simplify.

1
= log3 3
3

Write the numbers with the base 3.

= log3 (3 3)

Rewrite using loga (mp) = p loga (m).

= 3 log3 (3)

Simplify using loga a = 1.

= 3 1 = 3

Write the numbers with the same base. It is


not possible to cancel the 9 and the 27 because
they cannot be separated from the log.

Rewrite using loga (mp) = p loga (m).

2 log3 (3)
3 log3 (3)

Cancel the logs because they are the same.

2
3

63
= log3 3
18
666
= log3
18 18 18

log3 (9) log3 (32 )


=
log3 (27) log3 (33 )

Worked Example 11

Calculate the value of log2 (18), correct to 2 decimal places.


Think

140

On a Calculator page, complete the entry


line as: log2 (18)
Then press ENTER .
Pressing Ctrl / ENTER will give a
decimal approximation.

Write the answer.

Write/display

log2 (18) = 4.17, correct to 2 decimal places.

Maths Quest 12 Mathematical Methods CAS for the TI-Nspire

REMEMBER

For a > 0:
1. loga (1) = 0
2. loga (a) = 1
3. loga (0) is undefined.
4. loga (x) does not exist when x 0.
5. loga (mn) = loga (m) + loga (n)
Exercise

3B

m
6. log a = log a (m) log a ( n)
n
7. loga (mp) = p loga (m)
log a ( N )
8. log b ( N ) =
log a (b)

Logarithm laws
1 WE6
Evaluate the following.
a log3 (1)
b log5 (1)

c log2 (2)

2 WE7
Write the following in index form.
a log2 (16) = 4
b logx (25) = 2
d log3 (x) = 5
3

c log5 (125) = x

1
e log 5 5 = 1

Write the following in logarithmic form.


a 23 = 8
b 34 = 81

d 5x = 125
e 2 1 = 12

4 WE8

c 43 = x
f x3 = 27

Simplify:

a log6 (3) + log6 (2)


d log3 (81)
5

d log6 (6)

b log2 (10) log2 (5)

c log2 (32)

log 5 1
5

f log3

b 3 log3 ( 3 x )

c log 2

27

Simplify:
a log 2 ( x )

6 WE9 Simplify:
a log4 (10) + log4 (2) log4 (5)
c 1 log10 (16) + log10 (52 )

x4
y 2

b log5 (25) + log5 (125) log5 (625)


d log3 (2) log3 (10) + log3 (15)

e log2 (16) + log2 (8) + log2 (4)


7 WE10
Simplify:
a 4 log2 (12) 4 log2 (6)
d

log 2 (64)
log 2 (8)

b 2 + log5 (10) log5 (2)


e

log a ( x )
log a ( x )

8 WE11 Evaluate correct to 3 decimal places.


a log10 (3)
b log5 (4)
d log2 (0.8)
9

c 1 + log2 (5)

e log4 (20)

c log10 (0.5)
f log3 (60)

Simplify:
a 5 log3 (x) + log3 (x2) log3 (x7)

b 4 log2 (x) + log2 (x3) log2 (x6)

c 3 log4 (x) 5 log4 (x) + 2 log4 (x)


e log10 (x2) + 3 log10 (x) 2 log10 (x)

d 4 log6 (x) 5 log6 (x) + log6 (x)


f 4 log10 (x) log10 (x) + log10 (x2)

g log5 (x + 1) + log5 (x + 1)2

h log4 (x 2)3 2 log4 (x 2)


Chapter 3 Exponential and logarithmic equations

141

10 MC 2 log10 (5) log10 (20) + log10 (8) is equal to:


B log10 (2)
C 1
A log10 (2)

D 1

E log10 (4)

11 MC If loga (b) = 2, then b is equal to:


A 0
B 1
C 2
D a
E a2
12 If y = a log10 (x), find x when a = 2 and y = 3. Give your answer correct to 3 decimal places.

3C

Exponential equations
The equation ax = b is an example of a general exponential (or indicial) equation and 2x = 32 is
an example of a more specific exponential equation.
To solve one of these equations it is necessary to write both sides of the equation with the
same base if the unknown is an index or with the same index if the unknown is the base.

Worked Example 12

Solve for x in each of the following.


1
a 2x = 32 b 3 x =
c 2 3x = 162 d 2(1 x) = 16
27
Think
a

The indices are equal because the base is


2 on each side of the equation.

Write 27 with base 3.

3
1

Write 32 with base 2, the same as the


left-hand side.

Write
a 2x = 32

2x = 25
x=5

1
27
1
= 3
3

3x = 3 3

b 3x =

Write 1 as a number with base 3.


33
Equate the indices.
Divide both sides by 2 to leave 3x on the
left-hand side.

Write 81 as a number with base 3.

Equate the indices.

Write 16 with base 2.

Equate the indices.

Solve for x.

x = 3
c 2 3x = 162

3x = 81

3x = 34
x=4
d

= 16
21 x = 24

21 x

1x=4
x = 3

Worked Example 13

Solve 5x 252x 3 = 625 for x: a using index laws b with a CAS calculator.
Think
a

Write/display

Write all numbers with the same base.

5x 252x 3 = 625
5x (52)2x 3 = 54

142

Simplify.

5x 52(2x 3) = 54

Maths Quest 12 Mathematical Methods CAS for the TI-Nspire

5x 54x - 6 = 54

Remove the brackets in the index.

Add the indices on the left-hand side.

55x - 6 = 54

Equate the indices.

5x - 6 = 4

Solve the equation.

5x = 10
x=2

On a Calculator page, press:


MENU b
3:Algebra3
1:Solve1
Complete the entry line as:
solve(5x 25(2x - 3) = 625,x)
Then press ENTER .

Write the solution.

Solving 5x 25(2x - 3) = 625 for x gives x = 2.

Sometimes it is possible to use the methods for solving quadratic equations to help solve indicial
equations. Remember that 22x = (2x)2.
WorkEd ExamplE 14

eBook plus

Solve for x in the following.


b 32x - 12 3x + 27 = 0

a (2x - 16)(2x + 4) = 0
Think
a

c 4x - 2x + 3 + 16 = 0
WriTE

Use the Null Factor Law to solve by


making each bracket equal to zero.

(2x - 16) = 0 or (2x + 4) = 0

Solve each equation.

2x = 16 or 2x = -4

Write 16 as a number with base 2 but -4


can not be written with base 2.

2x = 24 or no real solution

Solve by equating the indices.

Write

as

(3x)2.

int-0530
Worked example 14

a (2x - 16)(2x + 4) = 0

32x

Tutorial

x=4
b 32x - 12 3x + 27 = 0

(3x)2 - 12 3x + 27 = 0

Let 3x = a to make a simpler quadratic


equation to solve.

a2 - 12a + 27 = 0, where a = 3x

Factorise.

(a - 3)(a - 9) = 0

Use the Null Factor Law by making each


bracket equal to zero.

a - 3 = 0, a - 9 = 0

Solve for a.

Substitute back a = 3x.

3x = 3, 3x = 9

Write numbers with base 3.

3x = 31, 3x = 32

a = 3, a = 9

Chapter 3

Exponential and logarithmic equations

143

Equate the indices.

4x

Rewrite

as

(2x)2

x = 1, x = 2
and

2x + 3

as

2x

23.

4x 2x + 3 + 16 = 0
2x 23 + 16 = 0

(2x)2

(2x)2 2x 8 + 16 = 0

Rewrite 23 as 8.

Let 2x = a to make a simpler quadratic


equation to solve.

Replace a 8 with 8a because the


coefficient precedes the pronumeral.

Factorise.

(a 4)(a 4) = 0

Use the Null Factor Law.

a 4 = 0, a 4 = 0

Solve for a.

a = 4 and a = 4

The two factors are equal because


a2 8x + 16 = 0 is a perfect square.

Substitute back a = 2x.

2x = 4

10

Write 4 as a number with base 2.

2x = 22

11

Solve by equating the indices.

a2 a 8 + 16 = 0 where a = 2x
a2 8a + 16 = 0

x=2

Remember to always make the right-hand side equal to zero when solving quadratic equations.
It is a good idea to substitute your answer back into the original equation to check the
accuracy of your work.
If the base is not the same and the numbers cannot be written with the same base, then
logarithms can be used. It is possible to take the logarithm of both sides of an equation provided
the same base is used.

Worked Example 15

Solve for x in the following. Give your answers in exact form using base 10 and correct to 3 decimal
places.
a 5x = 10 b 2(x + 1) = 12
Think
a

144

Write
a

5x = 10
log10 (5x) = log10 (10)

Take the logarithm of both sides to


base 10.

Use loga (mp) = p loga (m) and loga (a) = 1.

x log10 (5) = 1

Divide both sides by log10 (5).

x=

Use a calculator to simplify.

x 1.431, correct to 3 decimal places.

Take the logarithm of both sides to


base 10.

Use loga (mp) = p loga (m) to simplify.

Divide both sides by log10 (2).

1
(exact form)
log10 (5)

2(x + 1) = 12
log10 (2(x + 1)) = log10 (12)

(x + 1) log10 (2) = log10 (12)

Maths Quest 12 Mathematical Methods CAS for the TI-Nspire

( x + 1) =

log10 (12)
log10 (2)

Use a calculator to simplify the right-hand


side.

Solve for x.

x=

log10 (12)
1 (exact form)
log10 (2)

x 2.585, correct to 3 decimal places.

Note: Logarithms in bases other than 10 may be used.


Inequalities are worked in exactly the same way except that there is a change of sign when
dividing or multiplying both sides of the inequality by a negative number.
Worked Example 16

Solve the following equations for x, giving your answers both in exact form and correct to 3 decimal
places.
a 2x > 5 b 0.5x 1.4
Think

Write/display

On a Calculator page, press:


Menu b
3:Algebra 3
1:Solve 1
Complete the entry lines as:
solve(2x>5,x)
solve(0.5x1.4,x)
Press ENTER after each entry.

Write the answers in exact form.


The calculator defaults to base e when
solving exponential equations in exact form.
Note: ln (A) loge (A). This will be
discussed later in this chapter.
Note: In part b , the inequality has been
1
changed to because log e 2 < 0.

a Solving 2x > 5 for x gives

Press Ctrl / ENTER after solving each


equation to obtain a decimal approximation.
Write the answers correct to 3 decimal places.

a Solving 2x > 5 for x gives x > 2.322,

x>

log e (5)
.
log e (2)

b Solving 0.5x 1.4 for x gives

7
log e
5
log e (2)

correct to 3 decimal places.

b Solving 0.5x 1.4 for x gives x 0.485, correct

to 3 decimal places.

REMEMBER

1. The equations ax = y and 2x = 32 are exponential equations.


2. Write numbers with the same base to help simplify problems. The most commonly
used bases are 2, 3 and 5.
3. If the base is the same, equate the indices.
4. If the indices are the same, equate the bases.

Chapter 3 Exponential and logarithmic equations

145

5. Use the Null Factor Law to solve quadratic equations.


6. A negative number cannot be expressed in index form, for example, -4 cannot be
expressed with base 2.
7. a2x = (ax)2
8. Take the logarithm of both sides of an equation or inequation using the same base.
9. Change the sign of an inequality when multiplying or dividing by a negative
number.
10. loga (x) > 0 if x > 1
11. loga (x) < 0 if 0 < x < 1
ExErCisE

3C

Exponential equations
1

eBook plus

Index
form

3x

b 10-x = 1000

= 81

d 7x = 1
49

Digital doc

SkillSHEET 3.1

Solve for x in each of the following.

WE12

Digital doc

c 2x 4x - 1 = 16
4

Digital doc

b 5x 52x + 1 = 625

243

Solve for x in the following.


a
- 1) = 0
b 22x - 6 2x + 8 = 0
2x
x
c 6 -76 +6=0
d 4x - 6 2x - 16 = 0
x
x
e 9 =23 +3
b 42x - 20 4x = -64

WE15 Solve for x correct to 3 decimal places.

a 2x = 5

b (0.3)x - 1 = 10

c (1.4)2 - x = 6

d 3 5x = 27

e 5 7x = 1

f 2x 3x + 1 = 10

WE16 Solve for x correct to 3 decimal places.


a 3x > 5
b 22x 7

d 7x 0.5
8

33 x + 1
= 81
9x 2

9)(3x

5 Solve for x in the following.


a 25x + 4 5x - 5 = 0

SkillSHEET 3.3
Solving
indicial
equations
by equating
the bases

3x - 1 =

WE14

(3x

eBook plus

mC The value of x for which 5 2x = 1255, correct to 3 decimal places, is:

A 7.971

B 897.750

Digital doc

D 7.972

E 2.059

Solving liner
inequations

146

c (0.2)x > 3

e (0.4)x > 0.2

eBook plus
SkillSHEET 3.4

d 22x - 6 = 1

Solve for x in each of the following.

WE13

3x

c 52 x 1 = 1
125

b 6x - 2 = 216

Solving
equations

= 32

Solve for x in each of the following.

eBook plus
SkillSHEET 3.2

1
2x

e 243x = 3

a 3 2x = 48
3

C 897.749

mC The solution to the equation 102x = 3 10x + 4 is:

A log10 (-1), log10 (4)

-1,

D 0, 0.602

E log10 (4)

maths Quest 12 mathematical methods Cas for the Ti-nspire

C 10x + 1, 10x - 1

3d

logarithmic equations using


any base
The equation loga (y) = x is an example of a general logarithmic equation. Laws of logarithms
and indices are used to solve these equations.

WorkEd ExamplE 17

Solve for x in the following equations.


a log2 (x) = 3
b log3 (x4) = -16

c log5 (x - 1) = 2

Think
a

WriTE

Rewrite using ax = y loga (y) = x.

23 = x

x=8

Rearrange and simplify.

Rewrite using loga

(mp)

Divide both sides by 4.

Rewrite using ax = y loga (y) = x.

Rearrange and simplify.

Rewrite using ax = y loga (y) = x.

a log2 (x) = 3

= p loga (m).

log3 (x4) = -16


4 log3 (x) = -16
log3 (x) = -4
-

3 4 = x
1
34
1
=
81

x=

c log5 (x - 1) = 2

52 = x - 1

x - 1 = 25
x = 26

Solve for x.

The base of a logarithmic function and the base of an exponential function must be a positive
real number other than 1. In the expression loga (x), a R+\{1}.

WorkEd ExamplE 18

eBook plus

Solve for x in each of the following:


a logx (4) = 2

without technology

Think
a

125

b log x

= 3 with a CAS calculator.

Tutorial

int-0531

Worked example 18

WriTE/display

Rewrite using ax = y loga (y) = x.

Solve the quadratic equation.

logx (4) = 2
x2 = 4

x2 - 4 = 0
(x - 2)(x + 2) = 0
x - 2 = 0 or x + 2 = 0
x = 2

Chapter 3

Exponential and logarithmic equations

147

Check to see if solutions


are valid.

x = 2 is the only solution.

This is the only solution. The solution x = 2 is not valid because the base of a logarithmic function
must be a positive real number other than 1.
b

On a Calculator page, press:


Menu b
3:Algebra 3
1:Solve 1
Complete the entry line as:
solve(log x

125

= 3, x )

Then press ENTER .

Solving log x

Write the solution.

125

= 3 for x gives x = 5.

Worked Example 19

Solve for x in the following.


a log2 (16) = x b log 3 1 = x c log9 (3) = x
3

Think
a

Write

Rewrite using

ax

= y loga (y) = x.

Write 16 with base 2.

Equate the indices.

Rewrite using ax = y loga (y) = x.

a log2 (16) = x

2x = 16
2x = 24
x=4

b log 3 1 = x
3

1
3
1
= 1
3

3x =

148

3x = 3

Write 3 with base 3.

Equate the indices.

Rewrite using ax = y loga (y) = x.

x = 1
c log9 (3) = x

9x = 3

(32)x = 3

Write 9 with base 3.

Remove the brackets.

32x = 31

Equate the indices.

2x = 1

Solve.

Maths Quest 12 Mathematical Methods CAS for the TI-Nspire

x=

1
2

WorkEd ExamplE 20

Solve for x in the following.


a log2 (4) + log2 (x) - log2 (8) = 3

b log10 (x) + log10 (x - 3) = log10 (4)

Think
a

WriTE

Simplify the left-hand side.


Use loga (mn) = loga (m) + loga (n) and
m
log a = log a (m) log a ( n) .
n

Rewrite using ax = y loga (y) = x.

Solve.

Simplify the left-hand side by using


loga (mn) = loga (m) + loga (n).

a log2 (4) + log2 (x) - log2 (8) = 3

4 x
log 2
=3
8
x
log 2 = 3
2
23 =

x
2

x = 2 23
=28
= 16
b log10 (x) + log10 (x - 3) = log10 (4)

log10x (x - 3) = log10 (4)


x(x - 3) = 4

Equate the logs.

Expand.

Solve the quadratic equation.

x2 - 3x - 4 = 0
(x - 4)(x + 1) = 0
x = 4 or x = -1

x > 0, x - 3 > 0 because it is not possible to


take the logarithm of a negative number, x > 3.

x = 4 is the only solution.

x2 - 3x = 4

rEmEmBEr

1. In a logarithmic equation, the unknown can be:


(a) the number, log2 (x) = 5
(b) the base, logx (8) = 3
(c) the logarithm, log2 (4) = x.
2. In the expression loga (x), a is a positive real number other than 1.
3. Use laws of logarithms and indices to solve the equations.
4. Check that all solutions are valid.
ExErCisE

3d
eBook plus
Digital doc

WorkSHEET 3.1

logarithmic equations using any base


1

WE17 Solve for x in the following.

i log5 (x) = 2

i i i log10 (x2) = 4
v log4 (2x - 3) = 0

i i log2 (x) = -3
i v log3 (x + 1) = 3
vi log2 (-x) = -5

v i i log5 (1 - x) = 4

Chapter 3

Exponential and logarithmic equations

149

i i log4 (x) = 2

i log3 (x) = 4

i i i log2 (x3) = 12

i v log5 (x 2) = 3
vi log3 (x) = 2

v log10 (2x + 1) = 0
v i i log10 (5 2x) = 1

2 WE18 Solve for x in the following.


a
i logx (9) = 2

2
3

i i i log x 1 = 3

i v logx (62) = 2

i logx (16) = 4

i i log x (125) =

i i log x (25) =

i i i log x

64

3
4

i v logx (43) = 3

= 2

3 WE19 Solve for x in the following.


a

i log2 (8) = x

i i log 5 1 = x

i i i log4 (2) = x

i v log6 (1) = x

v log 1 (2) = x
2

16

i log3 (9) = x

i i log 4

i i i log8 (2) = x

=x

i v log8 (1) = x

v log 1 (9) = x
3

4 WE20 Solve for x in the following.


a

i log2 (x) + log2 (4) = log2 (20)


i i i log3 (x) log3 (2) = log3 (5)

i i log5 (3) + log5 (x) = log5 (18)


i v log10 (x) log10 (4) = log10 (2)

v log4 (8) log4 (x) = log4 (2)


b

i log3 (10) log3 (x) = log3 (5)


i i i log2 (x) + log2 (5) = 1

i i log6 (4) + log6 (x) = 2


i v 3 log10 (x) = log10 (2)

v 5 log4 (8) = log4 (x)

5 Solve for x in the following.


i log2 (x) + log2 (6) log2 (3) = log2 (10) i i log2 (x) + log2 (5) log2 (10) = log2 (3)
a
i i i log3 (5) log3 (x) + log3 (2) = log3 (10) i v log5 (4) log5 (x)+ log5 (3) = log5 (6)
v log5 (x) + log5 (x 2) = log5 (3)
b

i log3 (x) + log3 (x + 2) = log3 (8)


i i i log5 (x) + log5 (x + 20) = 3

i i log4 (x) + log4 (x 6) = 2


i v log5 (x + 1) + log5 (x 3) = 1

v log6 (x 2) + log6 (x + 3) = 1

6 MC If loga (x) = 0.7, then loga (x2) is equal to:


A 0.49

B 1.4

D 0.837

E 0

C 0.35

7 MC If log10 (x) = (a), then (log10x)2 + log10 (x) 6 becomes:

150

A (log10 (a))2 + log10 (a) 6

B a2 + a + 6

D (a 2)(a + 3)

E log10 (106x3)

Maths Quest 12 Mathematical Methods CAS for the TI-Nspire

C log10 (x3) 6

8 Solve for x in the following.


a (log10 (x))2 + log10 (x) 2 = 0 (Hint: Let a = log10 (x))
c (log2 (x))2 2 log2 (x) = 8
e (log3 (x))2 log3 (x4) + 3 = 0
g log2 (x4) = (log2 (x))2
i log10 (x2 + 2x 5) = 1
log10 ( x )
= 4, find x.
9 If
log10 (2)

3E

b
d
f
h
j

(log10 x)2 2 log10 (x) 3 = 0


(log2 (x))2 + 3 log2 (x) = 4
(log5 (x))2 log5 (x3) + 2 = 0
log3 (x3) = (log3 (x))2
log3 (x2 3x 7) = 1

Exponential equations (base e)

Eulers number e, named after an 18th century Swiss mathematician, is a very important number
used in problems involving natural growth and natural decay. Like , it is irrational and has to
be approximated: e = 2.718 281 828 459 . . . The number e can be used to find the value of an
investment after a period of time, or the temperature of a liquid after it has been cooling.
n
1
To find the value of e, take the expression 1 + and evaluate it for increasing values of n.
n
n
1
1
1

n=1
1 + n = 1 + 1 = 2
n

n = 2

1
1

1 + n = 1 + 2 = 2.25

n = 3

1
1

1 + n = 1 + 3 = 2.370 37

n = 5

1
1

1 + n = 1 + 5 = 2.48832

n = 10

1
1

1 + n = 1 + 10

n = 100

1
1

1 + n = 1 + 100

10

= 2.59374
100

1
1

n = 1000 1 + = 1 +
1000

n
n

= 2.70481
1000

1
1

n = 10000 1 + = 1 +

n
10 000

= 2.716 92
10 000

= 2.71815

As n increases, 1 + becomes closer and closer to 2.718 281 or e, or

n
n
1

e = lim 1 +
n
n
An answer given in terms of e is an exact answer.
The laws of indices apply in the same way if e is the base, that is:
ex ey = ex + y
ex ey = ex y
(ex)y = exy
e0 = 1
1
x
e = x
e

x
ey

= ex

Chapter 3 Exponential and logarithmic equations

151

Worked Example 21

Solve for x in e3x = e.


Think

Write

Write the equation.

e3x = e

Write e with a power of 1.

e3x = e1

Equate the indices.

Solve for x.

3x = 1
1
x=
3

CAS calculators have an ex function, which is treated in the same way as any other number.

Worked Example 22

Solve for x, showing working and with a CAS calculator. Express your answers in exact form and
correct to 3 decimal places.
a ex = 3 b ex 3ex = 2
Think

Write/display

Method 1: Technology-free
a

Write the equation.

As loge (e) = 1, take loge of both sides


of the equation.

Rewrite using loga (xp) = p loga (x).

Solve for x.

Write the equation.


1

Write e x as .
ex

152

ex = 3

loge (ex) = loge (3)


x loge (e) = loge (3)
x = loge (3)
1.099, correct to 3 decimal places.
x

b ex 3e

ex

=2
3
x =2
e

Multiply every term by ex.

(ex)2 3 = 2ex

Make the right-hand side equal to zero.

(ex)2 2ex 3 = 0

Let ex = a.

a2 2a 3 = 0 where a = ex

Factorise and solve for a.

(a 3)(a + 1) = 0
a 3 = 0 or a + 1 = 0
a = 3 or a = 1

Substitute ex for a.

ex = 3 or ex = 1

Solve for x by taking the log of both


sides to base e.

loge (ex) = loge (3)

ex = 3 is the only solution because


ex = 1 has no real solution.

x 1.099, correct to 3 decimal places.

Maths Quest 12 Mathematical Methods CAS for the TI-Nspire

Method 2 : Technology-enabled
1

On a Calculator page, press:


Menu b
3:Algebra 3
1:Solve 1
Complete the entry lines as:
solve(ex = 3,x)
solve(ex 3ex = 2,x)
Press ENTER after each entry.

Write the answers in exact form.

a Solving ex = 3 for x gives x = loge (3).


b Solving ex 3ex = 2 for x gives

x = loge (3).

Write the answers in approximate form, correct


to 3 decimal places.

a Solving ex = 3 for x gives x = 1.099, correct to

3 decimal places.

b Solving ex 3ex = 2 for x gives x = 1.099,

correct to 3 decimal places.

REMEMBER

1. Eulers number e is an irrational number, which is approximated to 2.718 (correct to


3 decimal places).
2. Evaluate e by using the ex button on the calculator.
3. The number e is an exact answer. Use the calculator to give an approximation if required.
4. The laws of indices apply in the same way if e is the base.
5. On a calculator, use the LN button to take the log of a number to base e. The LOG
button defaults to base 10 if not specified.
6. loge (x) = ln (x).
7. ex > 0, that is, ex = 1 has no real solution.
Exercise

3E

Exponential equations (base e)


1 Evaluate the following, giving your answer correct to 3 decimal places.
1

a e2

b e4

c e 2

d e 3

g ln (4)

h ln (5)

i loge (1.5)

j loge (3.6)

2 WE21
a ex = e

Solve for x in each of the following.


b ex = e2
c ex 2 = e4
1

1
g e3 x + 6 = e
e e x + 1 =
f e x 2 = 2
e
e

d e2x = e

h e 2 x 1 = e3

Chapter 3 Exponential and logarithmic equations

153

3 WE22 Solve for x in each of the following, giving your answer correct to 3 decimal places.
a ex = 2

b ex = 5

c e x =

e ex = 1.3

f ex = 2.6

g 2ex = 6

1
2

d e x =

1
4

h 3ex = 12

4 Solve for x in each of the following, giving your answer correct to 3 decimal places.
a (ex 1)(ex + 2) = 0
b (ex 2)(e2x 3) = 0
c (3ex 2)(2ex 1) = 0
d (ex)2 ex = 0

e (ex)2 e ex = 0

g 6 11ex + 3e2x = 0

h 18 23ex + 7e2x = 0

f (ex)2 7ex + 10 = 0

5 Solve for x in each of the following, giving your answer correct to 3 decimal places.
a ex 4ex = 0

b ex 15ex 2 = 0

c 5ex 12ex 11 = 0

d 3ex + 6ex 11 = 0

e 4ex + 6ex 11 = 0

f ex + 2ex = 3

6 Solve for x in each of the following, giving your answer correct to 3 decimal places.
a ex > 1

b ex < e

c ex < 2

d e2x 4

e ex + 1 6

f e1 x 10

g e > 0.75
7 If y = Aekt, and y = 19.6 when t = 2, and y = 19.02
when t = 5, find the value of the constants A and k. Give your
answers correct to 2 decimal places.
8 For a body that has a higher temperature than its surroundings,
Newtons Law of Cooling is given by the formula = 0ekt,
where is the difference between the temperature of the
body and its surroundings after t minutes and 0 is the
difference between the original temperature of the body and
its surroundings. If the temperature of a freshly poured cup of
coffee is 90C in a room with a constant temperature of 18C,
and it cools to 65C after 10 minutes, find the value of k. Give
your answer correct to 2 decimal places.

3F

Equations with natural (base e)


logarithms
CAS calculators have an LN function that can be used to find the log to base e in the same way
that the LOG key is used to find the log of a number to base 10. CAS calculators default to
base 10 when using the log key.
In general, aloga(x) = x, where a R+\{1}.
The laws of logarithms apply in the same way for base e as they do for base 10.
ex = y loge (y) = x.

Worked Example 23

Solve for x, giving your answer both in exact form and correct to 3 decimal places, given that
loge (x) = 3.
Think
1

154

Rewrite using ex = y loge (y) = x.

Write/display

loge (x) = 3
e3 = x

Maths Quest 12 Mathematical Methods CAS for the TI-Nspire

Write the answer in exact form.

On a Calculator page, press:


Menu b
3:Algebra 3
1:Solve 1
Complete the entry line as:
solve(ln (x) = 3,x)
Then press ENTER .

Write the solution in approximate form,


correct to 3 decimal places.

x = e3

Solving loge (x) = 3 for x gives x = 20.086, correct


to 3 decimal places.

Worked Example 24

Solve for x giving your answer correct to 3 decimal places where appropriate.
a loge (3) = loge (x) b loge (x) + loge (3) = loge (6)
Think
a

Write
a loge (3) = loge (x)

Since the base is the same, equate the numbers.


1
2
3

x=3

Rewrite using loge (mn) =


loge (m) + loge (n).

b loge (x) + loge (3) = loge (6)

loge (3x) = loge (6)


3x = 6

Equate the number parts.

x=2

Solve for x.

REMEMBER

Exercise

3F

1. ex = y loge (y) = x

2. e log

4. loge (1) = ln (1) = 0

5. loge (e) = ln (e) = 1

( x)

=x

3. 10 log

10

( x)

=x

6. loge (0) = ln (0) is undefined.

7. loge (mn) = loge (m) + loge (n) or

ln (mn) = ln (m) + ln (n)

m
8. log e = log e (m) log e ( n)
n

or

m
ln = ln (m) ln ( n)
n

9. loge (mp) = p loge (m)

or

ln (mp) = p ln (m)

Equations with natural (base e) logarithms


1 WE23 Solve for x in each of the following giving, exact answers.
a loge (x) = 1
b loge (x) = 2
c loge (x) = 2

d loge (x) = 1

Chapter 3 Exponential and logarithmic equations

155

2 Solve for x, giving exact answers when appropriate, otherwise, correct to 3 decimal places.
a ln (2x) = 2
b ln (3x) = 1
c ln (x3) = 3
d ln (x2) = 2

e ln (x2) = 0.4

f ln (x3) = 0.9

g ln (x - 1) = -1

h ln (2x + 1) = -2

3 WE24 Solve for x, giving exact answers when appropriate, otherwise, correct to 3 decimal
places.
a loge (x) = loge (2)
b loge (x) = loge (5)
c loge (x) + loge (3) = loge (9)

d loge (x) + loge (2) = loge (8)

e loge (x) - loge (5) = loge (2)

f 1 + loge (x) = loge (6)

4 Solve for x, giving exact answers.


a loge (x) + loge (5) - loge (10) = loge (3)

b 2 loge (3) + loge (x) - loge (2) = loge (3)

c 3 loge (2) + loge (x) - loge (4) = loge (5)

d loge (4) + loge (3) - loge (x) = loge (2)

e loge (x) + loge (x + 1) = loge (2)

f ln (x + 1) + ln (2x - 1) = ln (5)

5 mC If ln (y) = ln (x) + ln (a), then an equation relating x and y that does not involve
logarithms is:
x
a
A y=x+a
B y = ax
C y=x-a
D y=
E y=
a
x
6 mC In the equation 2 loge (x) - loge (3x) = a, x =
A 3ea
B -a
C 3a

D loge (6a)

E no solution

7 Write the following equation without logarithms and with y as the subject.
2 loge (x) + 1 = loge (y)
8 If loge (x) = a and y = ea, express y in terms of x.
9 Solve for x the equation eln (x) = 2.
eBook plus
Digital doc

WorkSHEET 3.2

3G
eBook plus
Interactivity

int-0248
Inverses

156

10 Five grams of a radioactive substance is decaying so that


the amount, A grams, that is left after t days, is given by
the formula A = 5e-kt.
a Find the value of A when the number of grams of the
radioactive substance has been halved.
b Rewrite the equation with the new value of A.
c Rearrange the equation so that t is the subject of the
equation.
d If k = 0.005, find how long it will take for the number
of grams of the radioactive substance to be halved.
Give your answer correct to the nearest day.

inverses
Inverse operations are opposite operations. Addition and subtraction are inverse operations and
multiplication and division are inverse operations.
Squaring and taking the square root are also inverse operations. The equation of the inverse of
the function y = ex can be found by interchanging the x and y so that y = ex becomes x = ey. Using
ax = y loga (y) = x, x = ey becomes loge (x) = y or y = loge (x). Therefore y = ex and y = loge (x)
are the equations of inverse functions.
Thus two important properties follow:
aloga(x) = x, x R+
and
loga a(x) = x, x R.

maths Quest 12 mathematical methods Cas for the Ti-nspire

Worked Example 25

Calculate the inverse of y = 3ex + 1:


a without the use of technology b using a CAS calculator.
Think
a

Write/display

Interchange x and y to write the inverse


equation.

Divide both sides by 3.

In order to make y the subject, begin by


rewriting the equation using
ax = y loga (y) = x.

Make y the subject.

Interchange x and y to write the inverse


equation.

On a Calculator page, press:


Menu b
3:Algebra 3
1:Solve 1
Complete the entry line as:
solve(x = 3ey+1,y)
Then press ENTER .

Write the equation of the inverse.

y = 3ex + 1
Inverse is x = 3ey + 1
ey + 1 =

x
3

x
log e = y + 1
3
x
y = log e 1
3
x = 3e y + 1

The inverse of y = 3ex + 1 is


x
y = log e 1 where x > 0
3

Worked Example 26

Calculate the inverse of f(x) = 2 log10 (x 1) + 1:


a without the use of technology b using a CAS calculator.
Think
a

1
2

Write

Interchange x and y to write the inverse


equation.
In order to make y the subject, begin by
subtracting 1 from both sides.

Divide both sides by 2.

Rewrite using ax = y loga (y) = x.

Let y = 2 log10 (x 1) + 1
Inverse is x = 2 log10 (y 1) + 1
2 log10 (y 1) = x 1
log10 ( y 1) =
10

x 1
2

x 1
2

= y 1

Chapter 3 Exponential and logarithmic equations

157

Add 1 to both sides.

Write the answer.

f 1 ( x ) = 10

Interchange x and y to write the inverse


equation.

x = 2 log10 (y - 1) + 1

On a Calculator page, press:


MENU b
3:Algebra3
1:Solve1
Complete the entry line as:
solve(x = 2 log10 (y - 1) + 1,y)
Then press ENTER .

y = 10

x 1
2

Write the equation of the inverse.

The solution from the CAS can be simplified


to obtain the solution found in a . Rewrite the
answer with a negative power.

y = 10

Add the powers when multiplying


indices of the same base.

y = 10

Simplify the powers and write the


answer.

1
2

10 2
y =
10

x 1
2

10 2

1+

+1

+1
x

x 1
= 10 2

f 1 ( x )

+1

x 1
+
2 2

1
2

+1

+1

+1

x 1
= 10 2

+1

rEmEmBEr

1. The equation y = loge (x) is an inverse function of y = ex.


2. To find the inverse of a function, interchange x and y and then make y the subject.
ExErCisE

3G

inverses
1

eBook plus
Digital doc

SkillSHEET 3.5
Inverses

158

WE25 Find the inverse of the following.


a y = 2ex
b y = ex + 1
c y = ex - 1

d y = e2x - 1

2 Find the equation of the inverse of the following.


a y = 2 + ex
b y = 2 - ex
d y = 2 + ex + 1
e y = 3 - 2ex - 2
3

WE26 Find the inverse of the following.


a f (x) = 2 loge (x)
b f (x) = loge (x + 1)
d f (x) = loge (2x - 1)
e f (x) = loge (2 - x)

maths Quest 12 mathematical methods Cas for the Ti-nspire

e y = e2 - x

f y = e2 - 3x

c y = 1 - 2ex
f y = 2 - 3ex + 1
c f (x) = loge (x - 1)
f f (x) = loge (2 - 3x)

4 Find the equation of the inverse of the following.


b y = 2 - loge (x)
a y = 2 + loge (x)
d f (x) = 2 - ln (x - 1)
5

e y = 3 + 2 ln (x - 1)

x 1
5

x 1
e 5

B e

+2
2

x 1
5

x+2
e 3

C e

x 1
5

+2

mC If y = 5e 2x + 1 - 1, the equation of the inverse is:

1 x + 1
+1
ln
2 5

1 x + 1
1
ln
2 5

1 x + 1 1
+
ln
2 5 2

1 x + 1 1

ln
2 5 2

(2x)

mC If eloge

A x2

C 5 ln ( x 1) + 1
2

= y , then y equals:
B loge (2x)

C e2x

E 2ex

D 2x

3h

f f (x) = 1 - 3 ln (x + 2)

mC If y = 5 loge (3x - 2) + 1, the equation of the inverse is:

A e

c y = 2 + 3 loge (x)

literal equations
An equation such as ekx = a, where k R and a R+, is called a literal equation. It does not have
a numerical solution. The solution will be expressed in terms of the other pronumerals, in this
case a and k, often called parameters.
For this equation, the solution is: kx = ln (a)
x =

1
ln (a), k 0, a R +
k

WorkEd ExamplE 27
-kx

Solve ekx = 5 + 2e

eBook plus

for x, where k R\{0}.

Think

Tutorial

WriTE/display

2
e kx

Rewrite the equation with positive powers.

e kx = 5 +

Multiply both sides by ekx.

(ekx)2 = 5ekx + 2

Let y = ekx and make the right-hand side zero


to obtain a quadratic equation in terms of y.

y2 - 5y - 2 = 0

Solve for y using the quadratic formula.

y=
y=

int-0532
Worked example 27

( 5)

( 5)2 4 1 2
2 1

5 33
2

y =

Chapter 3

5 + 33 or
5 33
y=
2
2

Exponential and logarithmic equations

159

Substitute ekx for y.

e kx =

5 + 33 or kx 5 33
e =
2
2

Only the first solution is valid as ekx > 0.

e kx =

5 + 33
2

Re-write the exponential equation in


logarithmic form using ax = y loga (y) = x
or take logs of both sides. Divide both sides
by k.

Write the solution and state the restriction


for k.

On a Calculator page, press:


Menu b
3:Algebra 3
1:Solve 1
Complete the entry line as:

solve(ek x = 5 + 2e k x,x)
Note: You must put in a multiplication sign
between the k and the x.
Then press ENTER .

10

Write the answer.

5 + 33
kx = log e
2

x =

5 + 33
1
log e
, k R \ {0}.
k
2

x =

33 + 5
1
log e
k
2

Worked Example 28

1
Solve for x, given that log 2 ( x ) 5 log 2 ( p) = log 2 ( 6 ) where p > 0,
2
a manually b with a CAS calculator.
Think
a

160

Write/display

Simplify the left-hand side using


p loga (m) = loga (mp) and
m
log a (m) log a ( n) = log a
n

1
log 2 x 2 log 2 ( p5 ) = log 2 (6)

x
log 2 5 = log 2 (6)
p
x
=6
p5

Equate both sides.

Multiply both sides by p5.

x = 6 p5

Square both sides to obtain x.

x = (6p5)2

Write the solution and state the


restriction for p.

x = 36p10 where p > 0

Maths Quest 12 Mathematical Methods CAS for the TI-Nspire

On a Calculator page, press:


Menu b
3:Algebra 3
1:Solve 1
Complete the entry line as:
1

solve log 2 ( x ) 5 log 2 ( p) = log 2 (6),x


2

Then press ENTER .

Write the answer. Note that the CAS


gives the incorrect restriction p 0. The
correct restriction is p > 0 for log2 (p) to
be defined.

x = 36p10

Some equations can only be solved using a CAS calculator, that is, no algebraic method exists.
The following example demonstrates this.
Worked Example 29

Solve the following equations using a CAS calculator. Give your answers correct to 3 decimal places.
a ex = x3 b loge (x) = x 2
Think

Write/display

On a Calculator page, press:


Menu b
3:Algebra 3
1:Solve 1
Complete the entry lines as:
solve(ex = x3,x)
solve(ln (x) = x 2,x)
Press ENTER after each entry.

Write the solutions correct to 3 decimal


places.

a Solving ex = x3 for x gives x = 1.857 or x = 4.536.


b Solving loge (x) = x 2 for x gives x = 0.159 or

x = 3.146.

REMEMBER

1. An equation such as ekx = a, where k R and a R+, is called a literal equation.


2. Literal equations do not have numerical solutions.
3. The solution of a literal equation is expressed in terms of the other pronumerals, in this
case a and k, often called parameters.

Chapter 3 Exponential and logarithmic equations

161

ExErCisE

3h

literal equations
1 Solve 5e2x = a for x, where a R+.
2 If log3 (D) = cy + log3 (Z), solve for D.
3 Solve emx + n = 3k for x, where m R\{0} and k R+.
4 Solve for q given that 2 log3 (p + 5q) = 4.
x2
z2 x5
y4
y
5 Prove that log10 3 2 + log10 3 log10 3 4 log10 4 = 0.
z x
y z
z
z x
6 Consider the exponential equation 9xb 273a = 81.
a Find x in terms of a and b, where a R, b R\{0}.
b Hence find the value of x if a = 2 and b = -3.
7 Solve 42x - b = 20 for x, where b R.
8 Solve 2x - 1 = 3x + a for x, where a R.
9 If y = m + Rebx, solve for x.
10 Solve for a given that (log2 (5a))2 = 16b2.
a
11 WE27 Solve e kx = 3 + kx for x, where a 0, k R\0.
e
1

12 WE28 Solve for x given that 2 log 4 ( x ) 3 log 4 ( y ) = log 4 (3).


13 Solve for b given that 2 loge (a) - 5 loge (b) - 2 = 0, where a, b R+.
14 WE29 Solve the following equations using a CAS calculator. Give your answers correct to
3 decimal places.
a ex = 3x
b x+2=e x
c x2 - 1 = e2x
15 WE29 Solve the following equations using a CAS calculator. Give your answers correct to
3 decimal places.
a ln (x) = 2 - x
b ln (x - 2) = x - 4
c x2 - 1 = ln (2x)
16 Solve for x given that log7 (x) = log4 (p).

3i
eBook plus
eLesson

eles-0091
Exponential
and logarithmic
modelling

162

Exponential and logarithmic


modelling
Exponential and logarithmic functions can be used to model many real situations involving
natural growth and decay.
Continuous growth and decay can be modelled by the equation A = A0ekt, where A0 represents
the initial value, t represents the time taken and k represents a constant.
For continuous growth, k is positive, but for continuous decay, k is negative.
Logarithms to base 10, often called common logarithms, are used in scientific formulas for
measuring the intensity of earthquakes, the acidity of solutions and the intensity
of sound.

maths Quest 12 mathematical methods Cas for the Ti-nspire

WorkEd ExamplE 30

eBook plus

In the town of Ill Ness, the number of cases of a particular disease,


Tutorial
D, can be modelled by the equation D = D0ekt, where t is the time in years.
int-0533
Using available medication the number of cases is being reduced
Worked example 30
by 20% each year. There are 10 000 people with the disease today.
a How many people will have the disease after one year?
b Find the value of k correct to 3 decimal places.
c Write the equation substituting values for k and D0.
d Find how long it would take for the number of people with the disease to be halved. Give your
answer correct to the nearest year.
e How long would it take for the number of people with the disease to be reduced to 100? Give your
answer correct to the nearest year.
Think
a

a (100 - 20)% = 80%

Find the percentage of people with the


disease after one year.

Find 80% of the original number.

80% of 10 000 = 8000

Write a sentence.

Therefore, 8000 people will have the disease after


one year.

Substitute t = 0 and D = 10 000 into the


given equation.

WriTE

Substitute t = 1 and D = 8000 into [1], and


solve for k.

Use the given equation D = D0ekt.


1

Substitute 5000 for D.

Simplify by dividing both sides by 10 000.

Take loge of both sides.

Solve for t.

D = D0 ekt
When t = 0 and D = 10 000,
10 000 = D0 ek 0
10 000 = D0 1
10 000 = D0
So
D = 10 000ekt

[1]

When t = 1 and D = 8000,


8000 = 10 000ek 1
8000
= ek
10 000
0.8 = ek
loge 0.8 = loge (ek)
-0.223 = k log (e)
e
-0.223 = k 1
k = -0.223
-0.223t

c D = 10 000e

-0.223t

d D = 10 000e

When D = 5000,
5000 = 10 000e 0.223t
-0.223t

0.5 = e

loge (0.5) = -0.223t


t=

log e (0.5)
0.223

3.108 (3 decimal places)


5

Write a sentence.

It would take about 3 years.

Chapter 3

Exponential and logarithmic equations

163

0.223t

D = 10 000e

Write the equation.

Substitute 100 for D.

Simplify by dividing by 10 000.

Take loge of both sides.

Solve for t.

Write the answer in a sentence.

When D = 100,

100 = 10 000e 0.223t


0.223t

0.01 = e

loge (0.01) = 0.223t


t 20.651 (3 decimal places)
It would take about 21 years.

REMEMBER

1. Make a note of all the information that has been given.


2. Give your answers to the correct number of decimal places.
3. Continuous growth and decay is modelled by the equation A = A0ekt, A0 represents the
initial value (that is, when t = 0) and k represents a constant.

Exercise

3I

Exponential and logarithmic modelling


1 WE30 Changing -gluconolactone into gluconic acid can be modelled by the equation

y = y0e 0.6t, where y is the number of grams of -gluconolactone present t hours after the
process has begun. Suppose 200 grams of -gluconolactone is to be changed into gluconic
acid.
a Find the value of y0.
b Write the equation replacing y0 with your answer.
c How many grams of -gluconolactone will be present after 1 hour? Give your answer
correct to the nearest gram.
d How long will it take to reduce the amount of -gluconolactone to 50 grams? Give your
answer correct to the nearest quarter of an hour.

2 The decay of radon-222 gas is given by the equation y = y0e 0.18t, where y is the amount of radon
remaining after t days. When t = 0, y = 10g. Give all answers to the nearest whole number.
a Find the value of y0.
b Write the equation substituting your value of y0.
c What will be the mass after 1 day?
d How many days will it take for the mass to reach 1 g?
3 The equation y = A + B loge (x) relates two variables x and y. The table below shows values of
x and y.
x
y
a
b
c
d

164

1
3

2
4.386

3
m

Find the value of A and B correct to the nearest whole number.


Write the equation relating x and y substituting values for A and B.
Using your new equation, find the value of m correct to 3 decimal places.
If y = 7.6, find x correct to the nearest whole number.

Maths Quest 12 Mathematical Methods CAS for the TI-Nspire

4 An amount of $1000 is invested in a building society where the 5% p.a. interest paid is
compounded continuously. The amount in the account after t years can be modelled by the
equation A = A0ert, where r is the continuous interest rate.
a Find the value of A0 and r.
b Write the equation substituting values of A0 and r.
c Find the amount in the bank after i 1 year ii 10 years. Give your answer correct to
the nearest dollar.
d How long will it take for the investment to double in value? Give your answer to the
nearest year.
5 The number of people living in Boomerville at any time, t years, after the first settlers arrived
can be modelled by the equation P = P0ekt. Suppose 500 people arrived on 1 January 1850, and
by 1 January 1860 there were 675 people.
a What is the value of P0?
b Find the value of k correct to 2 decimal places.
c Write the equation substituting values for P0 and k.
d What will be the population on 1 January 1900? Give your answer to the nearest
10 people.
e When will the population be 2000?
6 A cup of soup cools to the temperature of the surrounding air. Newtons Law of Cooling can be

written as T TS = (T0 TS)e kt, where T is the temperature of the object after t minutes, and TS
is the temperature of the surrounding air. The soup cooled from 90C to 70C after 6 minutes
in a room with an air temperature of 15C.
a Find the values of TS, T0 and k correct to 2 decimal places.
b Write the equation substituting the values for TS, T0 and k.
c Find the temperature of the soup after 10 minutes. Give your answer to the nearest degree.
d How long would it take for the soup to be 40C? Give your answer to the nearest minute.
e If the soup is placed in a refrigerator in which the temperature is 2C, how long will it
take for the soup to reach 40C? Use the same value of k and give your answer to the
nearest minute.
7 The diameter of a tree for a period of its growth can be modelled by the equation D = D0ekt,
where t is the number of years after the beginning of the period. The diameter of the tree grew
from 50 cm to 60 cm in the first 2 years that measurements were taken.
a Find the values of D0 and k.
b Write the equation using these values.
c How much will it have grown in the first 5 years? Round to the nearest centimetre.
d How long will it take for the trees diameter to double? Round to the nearest year.

8 The decay of a radioactive substance can be modelled by the equation M = M0e kt, where
M grams is the mass of the substance after t years. After 10 years the mass of the substance is
98 grams and after 20 years the mass is 96 grams.
a What was the mass of the substance initially? Give your answer to the nearest gram.
b Find the value of k. Give your answer to 3 decimal places.
c Write the equation using these values.
d Find the mass of the substance after 50 years.
e How long would it take for the mass to be halved?
9 The number of bacteria present in a culture at any time, t hours, can be modelled by the
equation N = N0ekt.
a If the original number is doubled in 3 hours, find k correct to 2 decimal places.
b Write the equation substituting the value of k.
c Find the original number of bacteria if there were 2500 bacteria after 4 hours. Give the
answer correct to the nearest thousand.

Chapter 3 Exponential and logarithmic equations

165

d Write the equation substituting your value for the original population.
e Find the number of bacteria present after 8 hours. Give your answer correct to the nearest
thousand.
10 The intensity of light d metres below the surface of the sea can be modelled by the equation

I = I0e kd. Divers in the Sea of Loga have found that the intensity of light is halved when a
diver is 5 metres below the surface of the water.
a Find the value of k correct to 4 decimal places.
b Write the equation substituting the value of k.
c Find the percentage of light available at a depth of 10 metres.
d If artificial light is necessary when the intensity of light is less than 0.1 of the intensity at
the surface (I<0.1I0), find how deep a diver can go before artificial light is necessary.

166

Maths Quest 12 Mathematical Methods CAS for the TI-Nspire

Summary
Index laws

ax ay = ax + y
ax ay = ax y
x
y
xy
(a ) = a
a0 = 1
1
1

a x = x and x = a x
a
a
x

a y = y a and a y = y a x
ax = y loga (y) = x
Logarithm laws

loga (1) = 0
loga (a) = 1
loga (0) is undefined.
loga (mn) = loga (m) + loga (n)
m
log a = log a (m) log a ( n)
n
loga (mp) = p loga (m)
log a ( N )
log b ( N ) =
log a (b)



Exponential equations

To solve exponential equations:


1. write all terms with the same base, write terms with the smallest possible base or take the logarithm of
both sides of the equation
2. then solve the equation.
A negative number cannot be expressed in index form.
If 0 < x < 1, then loga (x) < 0 and if x > 1 then loga (x) > 0.
It is not possible to take the logarithm of a negative number.
Exponential equations (base e)
n

1
Eulers number e = lim 1 + = 2.718 281 828459 ...
h
n
The laws of indices and logarithms apply in the same way when using e.
Equations with natural (base e) logarithms

To solve logarithmic equations use the laws of logarithms and indices.


Inverses

The equation y = loge (x) is an inverse function of y = ex.


To find an inverse, interchange x and y.
a log ( x ) = x, x R +
log a (a x ) = x, x R
a

Literal equations

An equation such as ekx = a, where k R and a R+, is called a literal equation.


Literal equations do not have numerical solutions.
The solution of a literal equation is expressed in terms of the other pronumerals, in this case a and k, often
called parameters.

Chapter 3 Exponential and logarithmic equations

167

chapter review
2 2 log3 (x) + 4 log3 (x) log3 (x6) is equal to:

Short answer
2

3 2
4
1 Simplify 4 x 5 y 3 2 x 3 y 5 , leaving your

answers with positive indices.

A 0

6x
B log3 x 6

2 If log2 (5) = 2.321 and log2 (9) = 3.17, find

C log3 (6x - x6)

8x
D log3 x 6

E 6log3 (x - x6)

log 2 95 .
3 Solve 3 2x 7 = 17 for x.
log 2 (32)
.
log 2 (8)
1
5 Solve log x (2) = for x.
3
6 If 4e(2 x) = 128, find x, giving your answer in exact
form.
4 Evaluate

7 Solve for x in
loge (5) + loge (x) loge (2) = loge (10).
8 Find the rule of the inverse function to y=3e2xa.
9 Find the rule of the inverse function of
y = loge (1 x) + 3.
10 Solve the equation loge (3x + 5) loge (2) = 2 for x.


Exam tip Common mistakes include cancelling


out the logarithms, adding to obtain 3x + 7, or
multiplying to get 6x + 5.

[Assessment report 1 2007]

[ VCAA 2007]

11 Solve 6e3x = k for x, where k R+.


12 Solve 3eax + b 6k = 0 for x and state the necessary
restrictions for the parameters a, b and k.
13 Solve for x given that 4 log2 (ax + b) = 12.
14 Solve for x given that log2 (x) = y + log2 (z). State
the restrictions for the parameters.
Multiple choice

1 If a > 1, the solution of x for the equation x = a2 is:


A 1
B a negative number less than 1
C a positive number less than 1
D a negative number greater than 1
E a positive number greater than 1

168

3 The solution of the equation 3e2x = 4 is closest to:


B 0.405
A 0.406
C 0.143
D 0.144
E 0.575

[ VCAA 2005]
4 Evaluated to 3 decimal places, log3 (24) is:
A 2.892
B 2.893
C 0.345
D 0.346
E 1.380
5 The solution set of the equation (2x 1)(22x 4) = 0
over R is:
A {0, 1}
B {0, 2}
C {1, 2}
D {1, 4}
E {2, 4}

6 The solution set of the equation ex 12e x = 4 over


R is:
A loge (2), loge (6)
B ex 2, ex + 6

C 2, 6
D loge 2
E loge 6
7 If loge (x) = a, then e2a + 3ea 2ea is equal to:
2
a
2
B x 2 + 3 x
x

A a 2 + 3a

C 2 log e (a) + 3 log e (a)

2
log e (a)

d log e ( x 2 ) + 3 log e ( x )

2
log e ( x )

E (ea + 2)(e + 1)
8 If loge (2x) = a, then x is equal to:
A 2ea
ae
D
2

Maths Quest 12 Mathematical Methods CAS for the TI-Nspire

B 2ae
E e2a

ea
2

9 If ex + 4 = e2x 1, then x is equal to:

5
B 5
A
3
D -5

C e5

14 If y = loga (7x b) + 3, then x is equal to:


1
A 1 a y 3 + b
B (a y 3) + b
7
7
b
1 y3
+ b)
D a y 3
C (a
7
7
y3
E
log a (7 b)
[ VCAA 2007]

E e 3
10 The solution(s) to the equation
2ln (x) = ln (x + 4) + ln 2 is/are:
B 2, 4
A -2, 4
C 1
D 2
E 4
11 If 2a =

x2
, then y is equal to:
y

x
2a
x2
C a
2
A

x
B
2a
x
D
22 a

x2
4a
12 The equation which is the inverse of y = ex 1 is:
A y = loge (x) 1
B y = loge (x 1)
C y = loge (x) + 1
D y = loge (x + 1)

E y = e x 1
E

13 The air pressure P cm of mercury at h km


above sea level can be modelled by the equation

P=76e 0.13h. One kilometre above sea level the


pressure has:
A increased by approximately 9 cm
B decreased by approximately 9 cm
C increased by approximately 41 cm
D decreased by approximately 41 cm
E neither increased nor decreased significantly

15 If 2loge (x) loge (x + 2) = 1 + loge (y), then y is


equal to:
x2
A
10( x + 2)
2x
1
B
x+2
x2
C
x+2
2x
D
x+2
x2
E
e( x + 2)

[ VCAA 2003]
16 If 3log ( x + 4 ) = y , then y equals:
A log (x + 4)
B x + 4
C (x + 4)3
D 3 log (x + 4)
E 4
3

17 The solution set of the equation e4x 5e2x + 4 = 0


over R is:
A {1, 4}
B {4, 1}
C {2, 1, 1, 2}
D {loge (2), 0, loge (2)}
E {0, loge (2)}
[ VCAA 2007]

Extended response

a
1 log e = x . If loge (a) = 0.6932, find the value of x, giving your answer correct to 2 decimal places.
3
x x 2 x3
+ + ...
2 e x = 1 + +
1! 2! 3!
a Write the next 3 terms.
b Substitute x = 1 in the equation using the first 7 terms.
c Show that e 2.7182.
3 The apparent brightness of a star can be found using the formula B = 6 - 2.5 log10A, where A is the actual
brightness of that star. Find the apparent brightness of a star with actual brightness of 3.16.
a
4 Earthquake magnitude is often reported on the Richter scale. The magnitude, M, is given by M = log10 + B,
T
where a is the amplitude of the ground motion in microns at the receiving station, T is the period of the
seismic wave in seconds, and B is an empirical factor that allows for the weakening of the seismic wave
with the increasing distance from the epicentre of the earthquake.

Chapter 3 Exponential and logarithmic equations

169

Find the magnitude of the earthquake if the amplitude of the ground motion is 10 microns, the period is
1 second and the empirical factor is 6.8.
5 Five grams of a radioactive substance is decaying so that the amount, A grams, that is left after t days, is
given by the formula A = 5e kt.
a Find the value of A when the number of grams of the radioactive substance has been halved.
b Rewrite the equation with the new value of A.
c Rearrange the equation so that t is the subject of the equation.
d If k = 0.005, find how long it will take for the number of grams of the radioactive substance to be halved.
Give your answer correct to the nearest day.
6 A school in the suburb of Bienvenue opened with 30 students in February 1995. It has been found for the first
years after opening that the number of students enrolled in the school t years after opening can be modelled by
the equation N = N0ekt. There were 45 students enrolled in February 1996.
a Find the values of N0 and k.
b Write the equation substituting the values for N0 and k.
c How many students will there be 5 years after the opening?
d How many years will it take for the school to have 1000 pupils?
Another school in the suburb of Enbaisse has a declining student population. The number of students
enrolled at any one time can be modelled by the equation E = E0e rt. There are 1000 students enrolled in
February 1995 and 900 in February 1996.
e Find the values of E0 and r.
f Write the equation substituting the values for E0 and r.
g How many students will be enrolled after 5 years?
h How many years will it take for the two schools to have approximately the same number of pupils?
i What will the population be then? Use the calculator value in the working and do not round off until the
final answer.
-kx

7 Solve ekx = 4 + ke

for x, where k R+.

eBook plus
Digital doc

Test Yourself
Chapter 3

170

maths Quest 12 mathematical methods Cas for the Ti-nspire

eBook plus

aCTiviTiEs

Chapter opener
Digital doc

10 Quick Questions: Warm up with ten quick


questions on exponential and logarithmic equations.
(page 132)
3A

The index laws

Tutorial

WE 5 int-0528: Watch a worked example on writing


expressions with positive indices. (page 135)
3B

Logarithm laws

Tutorial

WE 10 int-0529: Watch a worked example on


simplifying logarithmic expressions. (page 139)
3C

Exponential equations

Tutorial

WE 14 int-0530: Watch a worked example on solving


exponential equations. (page 143)
Digital docs

SkillSHEET 3.1: Practise writing expressions using


index form. (page 146)
SkillSHEET 3.2: Practise solving equations.
(page 146)
SkillSHEET 3.3: Practise solving indicial equations
by equating the bases. (page 146)
SkillSHEET 3.4: Practise solving linear inequations.
(page 146)
3D

Logarithmic equations using any base

Tutorial

WE 18 int-0531: Watch a worked example on solving


logarithmic equations using a CAS calculator.
(page 147)
Digital doc

WorkSHEET 3.1: Simplify exponential and


logarithmic expressions, and solve logarithmic and
exponential equations. (page 149)
3F

Equations with natural (base e)


logarithms

3G

Inverses

Interactivity

Inverses int-0248: Consolidate your understanding of


inverses using the interactivity. (page 156)
Digital docs

SkillSHEET 3.5: Practise finding inverses. (page 158)


3H

Literal equations

Tutorial

WE 27 int-0532: Watch a worked example on solving


literal equations. (page 159)
3I

Exponential and logarithmic modelling

eLesson

Exponential and logarithmic modelling eles-0091:


Learn about how exponential and logarithmic
modelling is used. (page 162)
Tutorial

WE 30 int-0533: Watch a worked example on


exponential modelling. (page 163)
Chapter review
Digital doc

Test Yourself Chapter 3: Take the end-of-chapter test


to test your progress. (page 170)
To access eBookPLUS activities, log on to
www.jacplus.com.au

Digital doc

WorkSHEET 3.2: Solve logarithmic and exponential


equations and application questions. (page 156)

Chapter 3

Exponential and logarithmic equations

171

4A Graphs of exponential functions with


any base
4B Logarithmic graphs to any base
4C Graphs of exponential functions with
base e
4D Logarithmic graphs to base e
4E Finding equations for graphs of
exponential and logarithmic functions
4F Addition of ordinates
4G Exponential and logarithmic functions
with absolute values
4H Exponential and logarithmic modelling
using graphs

Exponential
and logarithmic
graphs
AREAS OF STUDY

Graphs and identification of key features of


graphs of the following functions:
exponential functions: exponential equations,
y = ax
logarithmic functions, y = loge (x) and
y = log10 (x), the relationship
a = ek, where k = loge (a)
Key features of functions, including:
axis intercepts
domain (including maximal domain) and
range
asymptotic behaviour
symmetry
Transformations for y = f (x) to
y = Af (n(x + b)) + c, where A, n and c R
and f is an exponential or logarithmic function
and relation between the graph of the original

function and the graph of the transformed


function
Families of transformed functions for a single
transformation parameter
Graphs of sum, difference, product and
composite functions of f and g, where f and g
are exponential or logarithmic equations such
as y = x2ekx, k R
Graphical and numerical solutions of equations
Recognition of the general form of possible
models for data presented in graphical or
tabular form using exponential and logarithmic
functions
Applications of simple combinations of
exponential or logarithmic functions and
interpretation of features of the graphs of these
functions in modelling practical situations
eBook plus

4A

Graphs of exponential
functions with any base

Digital doc

10 Quick Questions

The function f (x) = ax is an exponential function where a is a positive, real number which is
not 1 (that is, a R+\{1}). In this chapter, a will take the values of 2, 10 and Eulers number e.
An exponential function f (x) can be written f: R R where f (x) = ax and a R+\{1}. In the
previous chapter it was seen that if x has a positive coefficient, f (x) is an increasing function
and may be used to describe physical growth. Examples of this include population and bacterial
growth, and increases in investment values, light intensity and temperature.
If x has a negative coefficient, f (x) is a decreasing function and may be used to describe
physical decay. Examples of this include population and bacterial decline, radioactive decay,
temperature cooling and decreases in light intensity and vehicle values.
Graphs with a positive coefficient of x are considered first.

172

Maths Quest 12 Mathematical Methods CAS for the TI-Nspire

Graphs of f (x) = 2x and f (x) = 10x


The graphs of f (x) = 2x and f (x) = 10 x are shown below.
y
12
10
8
6 (0, 1)
4
Asymptote 2
(1, 2)
y= 0
x
0
3 2 1
1 2 3

y
12
(1, 10)
10
8
6
4
Asymptote 2
(0, 1)
y= 0
x
3 2 1 0 1 2 3

f (x) = 2x
f (x) = 10x
These graphs have a number of common features:
1. They both cross the y-axis at the point (0, 1) because a0 = 1.
2. The graph does not cross the x-axis; therefore, there are no x-intercepts.
3. There is a horizontal asymptote along the x-axis (y = 0).
4. For f (x) = a x, another point on the graphs is (1, a).
5. The maximal domain is R.
6. The range is R+.
7. They are both increasing functions. That is, as x , y .
8. It can be seen that the greater the value of a, the steeper the graph.

Dilation
A dilation changes the shape of the graph, making it wider or narrower.

Dilation from the x-axis


If the coefficient of ax is changed to a positive real number greater than 1, the graph is stretched
vertically and is said to be dilated from the x-axis.
This could be written f (x) = Aax where A is the dilation factor. The graph is stretched
vertically (along the y-axis) away from the x-axis because each y-value is being multiplied by
the constant A. In mapping notation, a dilation factor of A from the x-axis, where A > 0, is given
by (x, y) (x, Ay). If A = 3 and a = 2 the function becomes f (x) = 3 2x.
A comparison can be made more easily if graphs are drawn on the same axes.
The graphs of f (x) = 2x, g(x) = 3 2x and h(x) = 4 2x are shown. The dilation factor, A, is 1,
3 and 4 respectively.
f (x
( ) = 2x

g(x
( ) = 3 2x
(x

h(x
( ) = 4 2x
(x

x-intercept

y-intercept

(0, 1)

(0, 3)

(0, 4)

R+ = (0, )

R+ = (0, )

R+ = (0, )

y=0

y=0

y=0

(1, 2)

(1, 6)

(1, 8)

Domain
Range
Horizontal asymptote
Dilation factor from the
x-axis
A point on the graph
Mapping

y
(0, 4)

h(x) = 4 2x
g(x) = 3 2x

4
3 (0, 3) f(
f x) = 2x
2
1 (0, 1)
Asymptote
x y= 0
3 2 1 0 1 2 3

(x, y) (x, 3y) (x, y) (x, 4y)

Chapter 4

Exponential and logarithmic graphs

173

If A is a real number between 0 and 1, the y-values are multiplied by a constant which is less
than 1 and the graph becomes less steep.
A comparison can be made more easily if the graphs are drawn on the same axes.
The graphs of f (x) = 2x, g(x) = 12 2x and h(x) = 14 2x are drawn below.
y
(1, 2)

The dilation factor is 1,

1
2

and

1
4

(0, 12)
1

1
2

2x

h(x) = 14 2x

(1, 1)

(0, 1)
Asymptote
y= 0

f x) = 2x
f(
g(x) =

(0, 14) (1, 2 )


x
0
1

respectively.

f (x
( ) = 2x

g( x ) = 12 2 x

h( x ) = 14 2 x

x-intercept

y-intercept

(0, 1)

1
0, 2

1
0, 4

Domain
Range

R
R+

Horizontal
asymptote

= (0, )

R
R+

R
R+

= (0, )

= (0, )

y=0

y=0

y=0

Dilation factor
from the x-axis

1
2

1
4

A point on the
graph

(1, 2)

(1, 1)

Mapping

(x, y) x, 1 y
2

(1, )
1
2

(x, y) x, 1 y
4

For any positive real value of A, as A increases, the graph of f (x


(x) = A 2x becomes
steeper and closer to the y-axis. Similarly, as A decreases the graph becomes less steep and
further from the y-axis. The domain, range and horizontal asymptote stay the same as for
f (x
(x) = 2x. The mapping is ((x, y) (x
( , Ay).

Dilation from the y-axis


If the coefficient of x changes, the graph is stretched horizontally
and is said to be dilated from the y-axis. This can be written
f (x) = akx where 1 is the dilation factor. The dilation factor is the
k
reciprocal of the coefficient of x. In mapping notation, a dilation
factor of 1 from the y-axis, where k > 0, is given by (x, y) ( 1 x, y).
k
k
To show the effect of a dilation
from the y-axis, the three graphs
x
2x and h(x) = 2 are drawn at right. The dilation
f (x) = 2x, g(x) = 22x
2
1
factors are 1, 2 and 2, respectively.

174

Maths Quest 12 Mathematical Methods CAS for the TI-Nspire

y
4
2
(0, 1)
3 2 1

2
g(x) = 22x
f x) = 2x x
f(
h(x) = 22

Asymptote
y=0
01 2 3 x

f (x
( ) = 2x

2
g(x
( ) = 22x
(x

h(x
( ) = 22
(x

x-intercept

y-intercept

(0, 1)

(0, 1)

(0, 1)

R+ = (0, )

R+ = (0, )

R+ = (0, )

y=0

y=0

y=0

1
2

(2, 4)

(1, 4)

(4, 4)

Domain
Range
Horizontal asymptote
Dilation factor from the y-axis
A point on the graph

(x, y)

Mapping

1
2

x, y

(x, y) (2x
(2 , y)

For all the graphs discussed so far (that is, of the form f (x
(x) = A akx, a R+\{1}, k > 0), the
+
maximal domain is R, the range is R , the x-axis is the horizontal asymptote and they are all
increasing functions. The dilations have affected the steepness of the graphs. The mapping is
( , y) ( 1 x, Ay).
(x
k
WORKED EXAMPLE 1
x

Sketch the graph of f (x


(x) = 2 2 2 , showing the intercepts and the asymptote, and stating the domain
and the range.
THINK

WRITE/DRAW
x

Write the rule.

State the basic shape and transformations.

An exponential curve with basic shape f (x) = 2x. Dilation


of 2 units from the x-axis and 2 units from the y-axis.

Find the y-intercept. Either let x = 0 or


use the fact that the y-intercept is A in the
function

If x = 0, then

f (x) = A a k .

f (x) = 2 2 2

y = 2 22
= 2 20
=21
=2
so the y-intercept is 2.

Find the horizontal asymptote.

The horizontal asymptote is the x-axis. There are no


x-intercepts.

Find another point on the graph.

If x = 2, y = 2 2 2 = 4.

Sketch the graph.

y
4

(2, 4)

(0, 2)

x
3 2 1 0 1 2 3
7

State the domain and the range.

f (x) = 2 2 2
Asymptote y = 0

The domain is R and the range is R+.

Chapter 4

Exponential and logarithmic graphs

175

Translation
Vertical translation
If a constant is added to the function, the graph is moved up or down and is said to be
translated vertically. In mapping notation, a vertical translation of B units is given by
(x, y) (x, y + B).
The graph of g(x) = 2x + 1 is shown below with the graph of f (x) = 2x. The graph of f (x) = 2x
has been moved up 1 unit. Every y-value has been increased by 1. The line y = 1 is the horizontal
asymptote.
f (x
( ) = 2x

g(x
( ) = 2x + 1
(x

x-intercept

y-intercept

(0, 1)

(0, 2)

Range

(0, )

(1, )

Horizontal asymptote

y=0

y=1

A point on the
graph

(1, 2)

(1, 3)

Domain

g(x) = 2x + 1
f x) = 2x
f(

4
3
(0, 2)

(1, 3)

(0, 1)
0
1

(1, 2) Asymptote
y=1
2

(x, y) (x, y + 1)

Mapping

The graph of g(x) = 2x 1 is the graph of f (x) = 2x translated 1 unit down. This means that the
horizontal asymptote is now the line y = 1 and the graph crosses the x-axis. Therefore, there is
an x-intercept. Every y-value has been decreased by 1. The graph of y = 2x 1 is shown with the
graph of y = 2x.
y

f (x
( ) = 2x

g(x
( ) = 2x 1
(x

x-intercept

(0, 0)

y-intercept

(0, 1)

(0, 0)

2
(0, 1)

Range

(0, )

(1, )

Horizontal asymptote

y=0

y = 1

A point on the
graph

(1, 2)

(1, 1)

Domain

33 2 11

(1, 2)

g(x) = 2x 1

(1, 1)
x
0 1 2 3
Asymptote
y = 1

(x, y) (x, y 1)

Mapping

Horizontal translation
In mapping notation, a horizontal translation of b units
is given by (x, y) (x + b, y).
The graph of g(x) = 2x 1 is the graph of f (x) = 2x
translated 1 unit to the right.
The graph of h(x) = 2x + 1 is the graph of f (x) = 2x
translated 1 unit to the left.
The graph of g(x) = 2x 1 is shown at right with the
graph of f (x) = 2x.
176

f x) = 2x
f(

Maths Quest 12 Mathematical Methods CAS for the TI-Nspire

h(x) = 2x + 1
f(x) = 2x
f(

4
2
(0, 1)
3 2 1

(1, 1)
(0, 12 )

01 2 3

g(x) = 2x 1
Asymptote
y=0
x

f (x
( ) = 2x

g(x
( ) = 2x 1
(x

h(x
( ) = 2x + 1
(x

x-intercept

y-intercept

(0, 1)

(0, )

(0, 2)

R+ = (0, )

R+ = (0, )

R+ = (0, )

Horizontal asymptote

y=0

y=0

y=0

A point on the graph

(1, 2)

(2, 2)

(0, 2)

(x, y) (x + 1, y)

(x, y) (x 1, y)

Domain
Range

Mapping

1
2

Horizontal and vertical translations have the same effect regardless of the base.
g(x
( ) = 2 + 10x + 1 is f (x
(x
(x) = 10 x translated 1 unit to the left and 2 units up. Horizontal
translations keep the same asymptote, domain and range as the original function; vertical
translations keep the same domain, but the range changes.
It is interesting to note that f (x) = 0.5 2x can be written as f (x) = 2x 1 because
f (x) = 0.5 2x
1

= 2 2x

= 2 1 2x
= 2x 1
A dilation of 0.5 from the x-axis has the same effect on f (x) = 2x as a translation of 1 unit to
the right.
WORKED EXAMPLE 2

eBoo
k plus
eBook

Sketch the graph of f (x


(x) = 2x 1 2, showing intercepts and asymptotes, and
stating the domain and range.
THINK

WRITE/DRAW

Tutorial

int-0534
Worked example 2

f (x) = 2x 1 2

Write the rule.


State the basic shape.

State the translations.

Horizontal translation of 1 unit to the right


Vertical translation of 2 units down

Find the horizontal asymptote by translating


y = 0 down 2 units.

The horizontal asymptote is y = 2.

Find the y-intercept. Make x = 0.

y-intercept:
If x = 0, then

An exponential curve with the same shape as


f (x) = 2x

y=2 12
= 12 2
= 1 12

Find the x-intercept. Make y = 0.

Equate the indices.

Solve for x.

x-intercept:
If y = 0, then

Chapter 4

2x 1 2 = 0
2x 1 = 21
x1=1
x=2

Exponential and logarithmic graphs

177

Sketch the graph.

y
1
3 2 1
1
2

10

(2, 0)
0 1 2 3 x

f (x) = 2x 1 2

1
(0, 112 ) Asymptote
y = 2

The domain is R and the range is (2, ).

State the domain and the range.

For all the graphs of the form f (x


(x) = ax + b + B, where b, B R and a R+\{1}, the maximal
domain is R, the range is (B, ), the horizontal asymptote is y = B and they are all
increasing functions. The graph remains exactly the same shape as f (x
(x) = ax. The mapping
is ((x, y) (x
( b, y + B).

Reflections
If a negative sign is in front of the a the graph is reflected in the x-axis. (Remember a > 0.) The
mapping is (x, y) (x, y).
The graph of g(x) = 2x is shown with the graph of f (x) = 2x.
f (x
( ) = 2x

g(x
( ) = 2x
(x

x-intercept

y-intercept

(0, 1)

(0, 1)

R+ = (0, )

R = (, 0)

Horizontal asymptote

y=0

y=0

A point on the graph

(1, 2)

(1, 2)

Domain
Range

f x) = 2x
f(

Asymptote
(1, 2) y = 0
3 2 1 0 1 2 3 x
(1, 2)
2
(0, 1)

(0, 1)
g(x) = 2x

(x, y) (x, y)

Mapping

If there is a negative sign before the x term, the graph is reflected in the y-axis. The graph of
g(x) = 2x is shown with the graph of f (x) = 2x.
f (x
( ) = 2x

g(x
( ) = 2 x
(x

x-intercept

y-intercept

(0, 1)

(0, 1)

R+ = (0, )

R+ = (0, )

Horizontal asymptote

y=0

y=0

A point on the graph

(1, 2)

(1, 2)

Domain
Range

Mapping

f x) = 2x
f(

y
g(x) = 2x

3
(1, 2) 2
1
3 2 1

(1, 2)
(0, 1)
0

Asymptote
y= 0
x
1 2 3

(x, y) (x, y)
For all the graphs of the form g(x) = ax, where a R+\{1}, the maximal domain is R and
the range is R. The horizontal asymptote is y = 0 and the functions are all decreasing. It is the
reflection of f (x) = ax in the x-axis. The mapping is (x, y) (x, y).

178

Maths Quest 12 Mathematical Methods CAS for the TI-Nspire

For all the graphs of the form g(x) = ax, where a R+\{1}, the maximal domain is R and
the range is R+. The horizontal asymptote is y = 0 and the functions are all decreasing. It is the
reflection of f (x) = ax in the y-axis. The mapping is (x, y) (x, y).
WORKED EXAMPLE 3

Sketch the graph of f (x


(x) = 2 2x 1, showing intercepts and asymptotes, and stating the domain and
the range:
a showing all working
b with a CAS calculator.
THINK
a

WRITE/DRAW

Write the rule.

f (x) = 2 2x 1

State the basic shape.

An exponential curve with basic shape f (x) = 2x

State the transformations.

Horizontal translation of 1 unit to the right


Vertical translation of 2 units up
Reflection in the x-axis

Find the horizontal asymptote by translating The horizontal asymptote is y = 2.


y = 0 up 2 units.

If x = 0,
f (x) = 2 2 1
Find the y-intercept by making x = 0.

=2

1
2

= 1 12
The y-intercept is 1 12.
6

Find the x-intercept by making y = 0.

Sketch the graph.

2 2x 1 = 0
2x 1 = 21
x1=1
x=2
The x-intercept is 2.
If y = 0,

y
2

Asymptote
y=2
(0, 32 )

1
0

2 1

(2, 0)
x
2

f (x) = 2 2x 1

State the domain and the range.

To graph y = 2 2x 1 on a Graphs page,


complete the function entry line as:
f1(x) = 2 2x 1
Press ENTER .
Note: The horizontal asymptote at y = 2 is
not displayed.

The domain is R and the range is (, 2).

Chapter 4

Exponential and logarithmic graphs

179

To locate the x-intercept on a Calculator


page, press:
MENU b
3:Algebra 3
1:Solve 1
Complete the entry line as:
solve(f 1(x) = 0, x)
Press ENTER .

Write the x-intercept in coordinate form.

(2, 0) are the coordinates of the x-intercept.

To calculate the y-intercept, complete the


entry line as: ff1(0) and press ENTER .
Write the y-intercept in coordinate form.

0,

( ) are the coordinates of the y-intercept.


3
2

REMEMBER

1. Graphs of the form f (x) = ax, where a R+\{1}


(a) The maximal domain is R.
(b) The range is R+.
(c) The x-axis is the horizontal asymptote.
(d) The y-intercept is 1.
(e) They are all increasing functions.
2. Dilation
(a) Graphs of the form f (x) = Aax, where a, A R+\{1}, have a y-intercept of A.
A changes the steepness and proximity of the graph to the y-axis when compared
with the graph of f (x) = ax. As A increases, the graph becomes steeper.
So f (x) = ax is dilated by a factor of A from the x-axis.
(b) Graphs of the form f (x) = akx, where a R+\{1} and k > 0, have a different
steepness than graphs of the form f (x) = ax. As k increases, the graph becomes
more steep (that is, it gets closer to the y-axis). The y-intercept remains the same.
So f (x) = akx is dilated by a factor of 1 from the y-axis.
k
3. Translation
(a) Graphs of the form f (x) = ax + B, where a R+\{1}, B R, have a y-intercept of
1 + B, a horizontal asymptote of y = B and a range of (B, ). If B < 0 there is one
x-intercept. If B > 0 there are no x-intercepts.
(b) Graphs of the form f (x) = ax + b, where a R+\{1}, b R, have a y-intercept of ab.
They are the same shape as f (x) = ax but have been translated b units to the left
if b > 0 and to the right if b < 0.
4. Reflection
(a) Graphs of the form f (x) = ax, where a R+\{1}, have a y-intercept of 1, a range
of R and they are decreasing functions. They are the same shape as
f (x) = ax but have been reflected in the x-axis.
(b) Graphs of the form f (x) = ax, where a R+\{1}, have the same key features as
f (x) = ax but are decreasing functions; they have been reflected in the y-axis.
5. Combinations of transformations
For all the graphs of the form f (x) = A ax + b + B, where b, B, A R and a R+\{1},
the maximal domain is R, the range is (B, ), the horizontal asymptote is y = B and
they are all increasing functions.

180

Maths Quest 12 Mathematical Methods CAS for the TI-Nspire

EXERCISE

4A
eBoo
k plus
eBook
Digital doc

Spreadsheet 034
Exponential
graphs

Graphs of exponential functions with any base


1 WE1
Sketch the graph of each of the following functions, showing the intercepts and the
asymptote, and stating the domain and range of each one.
a f (x) = 2x
b f (x) = 10x
c f (x) = 2 10x
x
x
d f (x) = 3 2
e f (x) = 0.3 2
f f (x) = 0.5 10x
3x
2
2x
2
g f (x) = 2
h f (x) = 10
i f (x) = 5 32x
x
x
3x
j f (x) = 4 2
k f (x) = 2 10 2
l f (x) = 2 2 3
2 WE2 Sketch the graph of each of the following functions, showing intercepts and asymptotes,
and stating the domain and range of each one.
a f (x) = 2x + 1
b f (x) = 3x + 2
c f (x) = 3x 3
x
x
+
2
d f (x) = 2 4
e f (x) = 2
f f (x) = 10x + 1
x

4
x

3
g f (x) = 3
h f (x) = 2
i f (x) = 2x + 1 8
x

2
j f (x) = 10
+1
3 WE3
Sketch the graph of each of the following functions, showing intercepts and
asymptotes, and stating the domain and range of each one.
a f (x) = 2x
b f (x) = 10x
x
c f (x) = 10
d f (x) = 2x
x
e f (x) = 1 3
f f (x) = 10 10x
x
g f (x) = 2 + 10
h f (x) = 1 + 2x
1

x
i f (x) = 2 2
j f (x) = 1 32 x
4 Sketch the graph of f (x
(x) = 2 31 x, showing the intercepts and asymptotes and stating the
domain and range.
5 Sketch the graph of the function f:
f R R where f (x
(x) = 3 2x 1, showing the intercepts and
asymptotes and stating the domain and range.
6 Sketch the graph of the function f:
f R R where f (x
(x) = 5 4 31 x showing the intercepts and
asymptotes and stating the domain and range.
7 State the transformation of y = 2x needed to sketch the graphs of the following functions. Give
details of each transformation.
a y = 23x
b y = 24x
c y = 2 2x
x

x
d y=32
e y= 2
f y = 2x
x
x
g y=2 +1
h y=2 3
i y = 2x 1
x
+
5
j y=2
8 Each of the following functions is a translation of f (x
(x) = 10x. State how far each graph is
translated, and in which direction.
a f (x) = 10 x + 4
b f (x) = 10 x 2
c f (x) = 10 x + 2
x

3
x
d f (x) = 10
e f (x) = 10 3
f f (x) = 2 + 10 x
x

1
2
+
x
g f (x) = 10
h f (x) = 10
i f (x) = 5 + 10 x + 1
x

4
x

3
j f (x) = 10
+2
k f (x) = 10
4
l f (x) = 10 x + 2 3
9 Write down the domain and range for each of the following graphs and the equations for the
horizontal asymptotes.
a

6
4
2 (0, 2)
y=0
x
0
3 2 1
1 2

6
4
4 (0, 4)
2

(0, 2)
y=1

3 2 1

0 1

Chapter 4

y=3

2
3 2 1

0 1

Exponential and logarithmic graphs

181

y
2

e
y=1

3 2 1 0

1 0

3 2 1

y=1

(1, 0)

4 (0, 4)
y=2

1 2

2 (0, 2)
0

1 2 3

10 If the graph of f (x
(x) = 2x is translated 1 unit up and reflected in the x-axis, what is the new
equation?
11 If the graph of f (x
(x) = 1 2x + 3 is translated 2 units to the right and dilated by factor 3 from the
x-axis, what is the new equation?
2 1 + 3 are respectively:
12 MC The domain and range of the graph of f (x
(x) = 102x

A R, R
B R, [ 1, )
C [3, ), R
D [1, ), R
E R, (3, )

13 MC The y-intercept of the graph of f (x


(x) = 10x + 1 is:
A 1
B 2
C 1

D 2

E 11

2x 1

14 MC When the graph of f (x


(x) =
4 is translated 3 units down and 2 units to the left, it
becomes:
A f (x) = 2x 3 7
B f (x) = 2x 3 1
x
+
1
C f (x) = 2
7
D f (x) = 2x + 1 1
x

2
E f (x) = 2
6
15 Under certain conditions a mathobacillus bacterial colony doubles its numbers every minute.
The population can be modelled by:
P = 50 2t
where P is the number of bacteria t minutes after counting has begun.
a Find the number of bacteria when t = 0.
b Find the number of bacteria after 3 minutes.
c Sketch the graph of the population as a function of time.

4B

Logarithmic graphs to any base


The function f (x) = loga (x) is a logarithmic function where a is a positive, real number which
is not 1 (that is, a R+\{1}). Logarithms to base 10 appear in many scientific formulas. An
example of this is the intensity of earthquakes which is measured on the Richter scale.
Remember that loga (x) does not exist when x < 0.
In this course only graphs of functions where a > 1 are considered. Graphs with a positive
coefficient of x will be considered first.

Graphs of f (x) = log2 (x) and f (x) = log10 (x)


The graphs of f (x) = log2 (x) and f (x) = log10 (x) are shown below.
Asymptote
y x= 0
(2, 1)
2
(1, 0)
1
1 0
1
2

2 3

f (x) = log2 (x)


182

Asymptote
x= 0

(10, 1)

(1, 0)
2 4 6 8 10

f (x) = log10 (x)

Maths Quest 12 Mathematical Methods CAS for the TI-Nspire

These graphs have a number of common features:


1. They are both increasing functions; that is, as x , y .
2. Both graphs cross the x-axis at the point (1, 0), as loga (1) = 0
3. The y-axis is a vertical asymptote, so there is no y-intercept. This is because loga (0) is
undefined.
4. There are no negative values of x because the log of a negative number does not exist.
5. Another point on the graph is (a, 1), where a is the base because loga (a) = (1).
6. The domain is R+.
7. The range is R.
8. It can be seen that the smaller the value of a, the steeper the graph or the more rapidly the
graph rises.

Dilation
Dilation from the x-axis
If the coefficient of loga (x) is changed to a positive real number greater than 1, the graph is
stretched along the y-axis and is said to be dilated from the x-axis. This could be written
f (x) = A loga (x) where A is the dilation factor. The graph is stretched along the y-axis
because each y-value is being multiplied by the constant, A. In mapping notation, a dilation
factor of A from the x-axis, where A > 0, is given by
Asymptote
(x, y) (x, Ay).
y x= 0
(10, 2) g(x) = 2 log10 (x)
The asymptote, x-intercept, domain and range are
the same for all graphs of the form f (x
(x) = A loga (x
( ).
Graphs with different values of A can be more easily
compared by drawing them on the same axes. The
graphs of g(x) = 2 log10 (x), f (x) = log10 (x) and
h(x) = 12 log10 (x) are drawn at right. Notice that, as
A increases, the graph becomes steeper.

1
0

f x) = log10 (x)
(10, 1) f(
h(x) = 12 log10 (x)
(10, 12 )
x
2 4 6 8 10

f (x
( ) = log10 (x
( )

g(x
( ) = 2 log10 (x
(x
( )

h(x
( ) = 2 log10 (x
(x
( )

x-intercept

(1, 0)

(1, 0)

(1, 0)

y-intercept

R+ = (0, )

R+ = (0, )

R+ = (0, )

x=0

x=0

x=0

1
2

(10, 1)

(10, 2)

(10, )

Domain
Range
Vertical asymptote
Dilation factor from
the x-axis
A point on the
graph
Mapping

(x, y) (x, 2y)

1
2

(x, y) x, 12 y

Note: h(x) = 12 log10 (x) can be written as h(x) = log10 ( x ) and is drawn in the same way.
1
2

Chapter 4

Exponential and logarithmic graphs

183

Dilation from the y-axis


If the coefficient of x is changed to a
positive real number not equal to 1,
the graph is stretched along the x-axis
and is said to be dilated from the
y-axis. This could be written
f (x) = loga (kx), where 1 is the dilation
k
factor. When k > 1, the graph moves
away from the x-axis.
To see the effect of a dilation from
the x-axis the graphs g(x) = log10 (2x
(2 ),
f (x) = log10 (x) and h(x) = log10 ( 12 x)
are drawn at right.

y Asymptote
x= 0
1

h(x) = log10 (2 xx)


2 (2, 0)
(1, 0)

(1, 0)

( , 0)

= (0, )

h(x
( ) = log10
(x

R+

= (0, )

= (0, )
R

x=0

x=0

x=0

Dilation factor from


the y-axis

1
2

Another point on the


graph

(10, 1)

(5, 1)

(20, 1)

(x, y)

1
2

R+

Mapping

( x)

(2, 0)

Horizontal asymptote

1
2

R+

g(x
( ) = log10 (2x
(x
(2 )

y-intercept
Range

f x) = log10 (x)
f(

( 12 , 0)

f (x
( ) = log10 (x
( )
x-intercept

Domain

2 )
2x
g(x) = log10 (2x

( x, y )
1
2

(x, y) (2x
(2 , y)

For all graphs discussed so far (that is, of the form y = A loga (kx), A R+\{1}, k > 0), the
maximal domain is R+, the range is R, the y-axis is the vertical asymptote and they are all
increasing functions. The graphs have varied in their x-intercepts. The dilation factor from
the x-axis is A and the dilation factor from the y-axis is 1 . In mapping notation,
k
Asymptote
( , y) 1 x, A
(x
Ay .
y x=0
k
f x) = 2 log2 (3x)
f(
If the base is 2, the graph is steeper than when the base
is 10 but the vertical asymptote, intercept, domain and range
2
stay the same as for f (x) = A log10 (kx).
f x) = 2 log10 (3x)
f(
1
The graphs of f (x) = 2 log10 (3x) and f (x) = 2 log2 (3x) are
( 3 , 0)
x
shown at right.
0
1
2
3
1. The vertical asymptote is the y-axis.
2
2. The x-intercept is 13 .
3. The domain is R+ and the range is R.
4. The basic graph is dilated by factor 2 from the x-axis and
factor 13 from the y-axis.

184

Maths Quest 12 Mathematical Methods CAS for the TI-Nspire

WORKED EXAMPLE 4

Sketch the graph of f (x


(x) = 3 log10 (2x
(2 ), showing the intercepts and the asymptote, and state
the domain, range and the kind of transformation.
THINK

WRITE/DRAW

Write the rule.

f (x) = 3 log10 (2x


(2 )

State the basic shape.

A logarithmic graph with basic shape


f (x) = log10 (x)

Find the vertical asymptote (log10 (0) is undefined). The vertical asymptote is the y-axis.

Find the x-intercept.


Remember that log10 (1) = 0.

x-intercept:
Let y = 0,
3 log10 (2x
(2 ) = 0
2 =1
2x
x = 12
The x-intercept is 12.

Find the y-intercept.

There are no y-intercepts.

Mark another point on the graph. Choose one


which makes 22xx equal the base.

Let x = 5, y = 3 log10 (10)


=31
=3
Coordinates are (5, 3).

Sketch the graph.

Asymptote
y x=0
3

f x) = 3 log10 (2x
f(
(2 )

(5, 3)

( 2 , 0)
1

3
8

State the domain and the range.

The domain is R+ and the range is R.

State the transformations.

The dilations are of factor 3 from the x-axis and


factor 12 from the y-axis.

Translation
Translations may be either vertical or horizontal.

Vertical translation
If a constant is added to the function, the graph is moved up or down and is said to be translated
vertically.
The graphs of f (x
(x) = log10 (x
( ), g(x
( ) = log10 (x
(x
( ) + 1 and h(x
( ) = log10 (x
(x
( ) 1 are drawn and compared.
y Asymptote
x=0
2

g(x) = log10 (x) + 1


f x) = log10 (x)
f(

1
0
1

h(x) = log10 (x) 1


2

10

12 x

2
3

Chapter 4

Exponential and logarithmic graphs

185

f (x
( ) = log10 (x
( )

g(x
( ) = log10 (x
(x
( )+1

1
,
10

h(x
( ) = log10 (x
(x
( )1

x-intercept

(1, 0)

y-intercept

R+ = (0, )

R+ = (0, )

R+ = (0, )

Vertical asymptote

x=0

x=0

x=0

Vertical translation

1 unit up

1 unit down

(1, 0)

(1, 1)

(1, 1)

(x, y) (x, y + 1)

(x, y) (x, y 1)

Domain
Range

A point on the
graph
Mapping

(10, 0)

Horizontal translation
If a constant is added to x, the graph of f (x) = log10 (x) is translated horizontally.
The graphs of f (x) = log10 (x), g(x) = log10 (x + 1) and h(x) = log10 (x 1) are drawn.
f (x
( ) = log10 (x
( )

g(x
( ) = log10 (x
(x
( + 1)

h(x
( ) = log10 (x
(x
( 1)

(1, 0)

(0, 0)

(2, 0)

x-intercept
y-intercept
Domain

R+

(1,

= (0, )

Range

Horizontal translation
A point on the
graph

(1, )

x=0

Vertical asymptote

x=

x=1

1 unit to left

1 unit to right

(10, 1)

(9, 1)

(11, 1)

(x, y) (x 1, y)

(x, y) (x + 1, y)

Mapping
y
2

g(x) = log10 (x + 1)

1
2 1

0 1

f x) = log10 (x)
f(
h(x) = log10 (x 1)

2
x = 1

x=1

For all graphs of the form f (x


(x) = loga (x
( + b) + B, where b and B R and a R+\{1},

the maximal domain is ( b, ), the range is R, the vertical asymptote is x = b and they
are all increasing functions. The graph has exactly the same shape as f (x
(x) = loga (x
( ).
The horizontal translation is b and the vertical translation is B. In mapping notation,
( , y) (x
(x
( b, y + B).

186

Maths Quest 12 Mathematical Methods CAS for the TI-Nspire

WORKED EXAMPLE 5

eBoo
eBook
k plus

Sketch the graph of f (x


(x) = 1 + log10 (x
( + 2), showing intercepts and asymptotes,
and stating the domain, range and transformations. Give intercepts to
1 decimal place.
THINK

Tutorial

int-0535
Worked example 5

WRITE/DRAW

Write the rule.

f (x) = 1 + log10 (x + 2)

State the basic shape.

Logarithmic graph, basic shape


f (x) = log10 (x)

Find the vertical asymptote (log10 (0) is undefined). x + 2 = 0


x = 2 is the vertical asymptote.

Find the x-intercept.

If y = 0,

Use the law ax = y loga(y) = x.

x + 2 = 10 1
x = 0.1 2
= 1.9

The x-intercept is 1.9.

Find the y-intercept.

If x = 0, y = 1 + log10 (0 + 2) (exact answer)


1 + 0.301 03
1.3, correct to 1 decimal place
The y-intercept is 1.3.

Sketch the graph.

1 + log10 (x + 2) = 0
log10 (x + 2) = 1

Asymptote y
x = 2
2

(0, 1.3)

(1.9, 0)
2

x
4 6 8
f x) = 1 + log10 (x + 2)
f(

State the domain and the range.

The domain is (2, ) and the


range is R.

State the translations.

The horizontal translation is 2 units to the


left and the vertical translation is
1 unit up.

Reflections
If there is a negative sign in front of the loga (x) term, the graph is
reflected in the x-axis. The mapping is (x, y) (x, y).
The graphs of f (x) = log10 (x) and g(x) = log10 (x) are
shown at right.

Asymptote
y x=0
f x) = log10 (x)
f(
1
(10, 1)
(1, 0)
0
1

Chapter 4

g(x) =

6 8 10

(10, 1)
10 (x))

log

Exponential and logarithmic graphs

187

f (x
( ) = log10 (x
( )

g(x
( ) = log10 (x
(x
( )

(1, 0)

(1, 0)

x-intercept
y-intercept

R+

Domain

= (0, )

Range

= (0, )

x=0

x=0

(10, 1)

(10, 1)

Vertical asymptote
A point on the graph

R+

(x, y) (x, y)

Mapping

If there is a negative sign in front of the x term, the graph is reflected in the y-axis. The
mapping is (x, y) (x, y).
The graphs of f (x) = log10 (x) and g(x) = log10 (x) are shown below right.
f (x
( ) = log10 (x
( )

g(x
( ) = log10 (x)
(x

(1, 0)

(1, 0)

x-intercept
y-intercept
Domain
Range
Vertical
asymptote
A point on the
graph

R+

= (0, )

= (, 0)

x=0

x=0

(10, 1)

(10, 1)

Mapping

Asymptote
y x=0
f x) = log10 (x)
g(x) = log10 (x) f(
1
(10, 1)
(10, 1)
(1, 0)
10

(1, 0)
2

(x, y) (x, y)

WORKED EXAMPLE 6
a Sketch the graph of f (x
(x) = 2 log10 (3 x) 2, showing intercepts and asymptotes, and stating

the domain, range and transformations. Give exact values or round to 3 decimal places.

b Sketch the same graph using a CAS calculator.


THINK
a

188

WRITE/DRAW

Write the rule.

f (x) = 2 log10 (3 x) 2

State the basic shape.

Logarithmic graph, f (x) = log10 (x)

Find the vertical asymptote (log10 (0)


undefined).

For the vertical asymptote,


3x=0
x = 3 is the vertical asymptote.

Find the x-intercept.

If y = 0,

2 log10 (3 x) 2 = 0
2 log10 (3 x) = 2
log10 (3 x) = 1
3 x = 101
x = 7

The x-intercept is 7.

Maths Quest 12 Mathematical Methods CAS for the TI-Nspire

10

Find the y-intercept.

Sketch the graph.

If x = 0, y = 2 log10 (3) 2
= 1.046 (to 3 decimal places)
The y-intercept is 1.046.
(7, 0)

x
6 44 2 0 2
Asymptote
(0, 2 log10 (3) 2)
x=3
22

f (x
(x)
x) = 2 log10 (3 x)
x 2
7
8

State the domain and the range.


State the transformations.

To graph y = 2 log10 (3 x) 2 on a
Graphs page, complete the function entry
line as:
f1(x) = 2 log10 (3 x) 2
Press ENTER .
Note: The vertical asymptote at x = 3 is
not displayed.

To locate the x-intercept on a Calculator


page, press:
MENU b
3:Algebra 3
1:Solve 1
Complete the entry line as:
solve (f 1(x) = 0, x)
Press ENTER .

Write the x-intercept in coordinate form.


To calculate the y-intercept, complete the
entry line as: ff1(0) and press ENTER .
Write the y-intercept in coordinate form.

(,

The domain is
3) and the range is R.
Reflection in the y-axis, dilation 2 units from the
x-axis, vertical translation 2 units down, horizontal
translation 3 units to the right

The coordinates of the x-intercept are (7, 0).


The coordinates of the y-intercept are (0, 1.046)
(correct to 3 decimal places).

REMEMBER

1. The function f (x) = loga (x) is a logarithmic function where a R+\{1}.


(a) The vertical asymptote is the y-axis, so there are no y-intercepts.
(b) The graph crosses the x-axis at (1, 0) because loga (1) = 0.
(c) The domain is R+; that is, there are no negative values of x.
(d) The range is R.

Chapter 4

Exponential and logarithmic graphs

189

2. Dilation
(a) The function f (x) = A loga (x) dilates the graph of f (x) = loga (x) by a factor of A
units from the x-axis. The vertical asymptote, x-intercept, domain and range remain
the same. As A increases, the graph becomes steeper.
(b) The function f (x) = loga (kx) dilates the graph of f (x) = loga (x) by a factor of 1
k
from the y-axis. The vertical asymptote, domain and range stay the same, but the
x-intercept is 1 . As k increases, the graph becomes steeper and the x-intercept
k
becomes smaller.
3. Translation
(a) The function f (x) = loga (x) + B translates the graph of f (x) = loga (x) vertically
B units. The vertical axis, domain and range remain the same but the x-intercept
changes. There is no change in the shape of the graph.
(b) The function f (x) = loga (x + b) translates the graph of f (x) = loga (x) horizontally
b units. The shape and the range remain the same but the vertical asymptote, the
x-intercept and the domain change:
(i) The vertical asymptote becomes x = b.
(ii) The graph crosses the x-axis at (1 b, 0).
(iii) The domain is (b, ).
4. Reflection
(a) The function f (x) = loga (x) reflects the graph of f (x) = loga (x) in the x-axis.
All key features remain the same but the graph is a decreasing function instead of
an increasing function.
(b) The function f (x) = loga (x) reflects the graph of f (x) = loga (x) in the y-axis.
The vertical asymptote and the range remain the same but the x-intercept and the
domain change.
(i) The graph crosses the x-axis at (1, 0).
(ii) The domain is (, 0).

EXERCISE

4B

Logarithmic graphs to any base


1
a
c
e
g

Find the vertical asymptote of each of the following functions.


f (x) = log2 (x)
b f (x) = log10 (x)
f (x) = 2 log10 (x)
d f (x) = 5 log2 (x)
f (x) = log2 (3x)
f f (x) = log10 (4x)
f (x) 3 log10 (2x
(2 )
h f (x) = 2 log2 (3x)

2 WE4
Sketch the graph of each of the functions in question 1, showing the intercepts and
asymptotes, and stating the domain and range of each one.
3

State the horizontal and vertical translations required to transform f (x


(x) = log2 (x
( ) into the
following functions.
a f (x) = log2 (x + 2)
b f (x) = log2 (x + 1)
c f (x) = log2 (x 3)
d f (x) = log2 (x 4)
e f (x) = 1 + log2 (x 5)
f f (x) = log2 (x 3) + 2
g f (x) = 2 + log2 (x + 1)
h f (x) = 3 + log2 (x + 1)
i f (x) = log2 (x + 3) 2
j f (x) = log2 (x + 1) 2

4 WE5 Sketch the graph of each of the functions in question 3, showing the intercepts and
asymptotes, and stating the domain and the range of each one.

190

Maths Quest 12 Mathematical Methods CAS for the TI-Nspire

eBoo
k plus
eBook
Digital doc

Spreadsheet 069
Logarithmic
graphs

5 Sketch the graph of each of the following functions showing intercepts and asymptotes, and
stating the domain, range and the transformation of f (x
(x) = log10 (x).
a f (x) = log10 (x)
b f (x) = 2 log10 (x)
c f (x) = log10 (x)
d f (x) = 3 log10 (x)
e f (x) = 1 log10 (x)
f f (x) = 1 + log10 (x)
g f (x) = 2 + log10 (x)
h f (x) = 2 log10 (x)
6 Sketch the graph of f (x
(x) = log2 (2x
(2 1), showing intercepts and asymptotes, and stating the
domain, range and transformations of the graph f (x
(x) = log2 (x
( ).
7 WE6 Sketch the graph of each of the following functions, showing intercepts and asymptotes,
and stating the domain and range of each one. Give exact values or round to 1 decimal place.
a f (x) = 2 log2 (x) + 3
b f (x) = 3 log10 (x) + 1
c f (x) = log10 (1 x)
d f (x) = log10 (3 x)
e f (x) = 1 2 log2 (x)
f f (x) = 2 5 log10 (x)
g f (x) = 3 log10 (x + 1) 2 h f (x) = 4 log2 (x 2) 1
i f (x) = 1 + 2 log10 (3x)
j f (x) = 3 4 log10 (2x
(2 )
8 Each graph in question 7 is a transformation of the graph of f (x
(x) = loga (x), a = 2, 10. State the
base of each graph and the kinds of transformation that have taken place.
9 Sketch the graph of f:
f R R where f (x
(x) = 3 log2 (2 x) + 1, showing all key features and
stating the domain and range.
10 Sketch the graph of ff: [0, 10] R where f (x
(x) = 3 log10 (x
( + 1) 2, showing key features.

eBoo
k plus
eBook
Digital doc

WorkSHEET 4.1

4C

11 MC When the function f (x


(x) = log2 (x
( + 2) is translated 2 units up and 3 units to the left,
the function becomes:
A f (x) = log2 (x + 4) + 3
B f (x) = log2 (x + 5) + 2
C f (x) = log2 (x 1) + 2
D f (x) = log2 (x + 5) 3
E f (x) = log2 (x 1) + 2

Graphs of exponential functions


with base e
Graphs of exponential functions with base e are drawn in exactly
the same way as with any other base. The graph of f (x) = ex can
be dilated, translated and reflected in the same way. The graphs
of f (x) = ex, g(x) = 2x and g(x) = 10x are shown at right. The
graph of f (x) = ex is shown in red.
It can be seen that h(x) = 10x is steeper than f (x) = ex and
h(x) = 2x is less steep than f (x) = ex.

Features common to all three graphs

y
4

h(x) = 10x
f x) = ex
f(

g(x) = 2x
3
(1, e)
2
(1, 2)
Asymptote
(0, 1)
y=0
x
0
2
1
1

1. The graphs all cross the y-axis at the point (0, 1).
2. The horizontal asymptote is the x-axis or the line y = 0.
3. The domain is R.
4. The range is R+.
5. They are all increasing functions.

WORKED EXAMPLE 7
3 1.
State the transformations of f (x
(x) = ex needed to form the graph of f (x
(x) = 2 e3x

THINK
1

Write the rule.

WRITE/DRAW

f (x) = 2e3x 1

Chapter 4

Exponential and logarithmic graphs

191

State the dilation. The coefficient of ex gives


the dilation from the x-axis.
The reciprocal of the coefficient of x gives
the dilation from the y-axis.

Dilation by a factor of 2 units from the x-axis

State the translation. The vertical translation is


given by the constant added to the ex term.

The graph is translated 1 unit down.

Dilation by a factor of

1
2

units from the y-axis

WORKED EXAMPLE 8

Sketch the graph of f (x


(x) = ex. On the same set of axes sketch the graph of f (x
(x) = ex 2, marking
the asymptote and y-intercept, and state the transformation, the domain and the range.
Give exact answers.
THINK

WRITE/DRAW

Write the rule for the first graph.

f (x) = ex

State the basic shape.

Exponential curve

State the horizontal asymptote.

The horizontal asymptote is the x-axis.

Find the y-intercept by making x = 0.

If x = 0, y = e0
y = 1 is the y-intercept.

Draw the graph.

f x) = ex
f(

Asymptote y = 0

(0, 1)
3 2 1 0

Write the rule for the second graph.

f (x) = ex 2

State the transformation. The horizontal


translation is given by the constant added to
the x term.

The horizontal translation is 2 units to the right.

State the horizontal asymptote, which is the


same as for f (x) = ex.

The horizontal asymptote is the x-axis.

Find the y-intercept by making x = 0.

If x = 0, y = e0 2

y=e 2
=

10

Sketch the graph on the same set of axes.

y
2
(0, 1)

1
e2
f x) = ex
f(
f x) = e (x 2)
f(
(0, e2)
Asymptote y = 0.
(2, 1)

3 2 1 0

192

2 3

2 3

11

Check the accuracy of your graph by


transforming the point (0, 1) and marking it on
your graph.

(0, 1) (2, 1)

12

State the domain and the range which are the


same for both graphs.

The domain is R and the range is R+.

Maths Quest 12 Mathematical Methods CAS for the TI-Nspire

For graphs of y = Aekx, k > 0 the coefficient of the ex term shows the dilation of A from the
x-axis and the reciprocal of the coefficient of the x term shows the dilation of 1 from the y-axis.
k
The mapping is (x, y) ( 1 x, Ay).
k
WORKED EXAMPLE 9
2 , marking the asymptote and intercept, and state the transformations,
Sketch the graph of y = 3e2x
domain and range.

THINK

WRITE/DRAW

Write the rule.

2
f (x) = 3e2x

State the basic shape.

Exponential curve

State the transformations.

Dilation by a factor of 3 units from the x-axis and


dilation by a factor of 12 unit from the y-axis

Find the asymptote, recognising that it is


unchanged by dilation.

The horizontal asymptote is the x-axis.

Find the y-intercept by making x = 0 or


by multiplying y-values of f (x) = ex by
3. (0, 1) (0, 3)

If x = 0,

Sketch the graph.

y = 3e2 0
=31
=3
so y-intercept is 3.
y
4
2

(0, 3)

3 2 1 0 1 2 3
7

State the domain and range.

2
f (x) = 3e2x
Asymptote y = 0

The domain is R and the range is R+.

The vertical translation is given by the constant added to the ex term. The horizontal translation
is given by the constant added to the x term.
WORKED EXAMPLE 10

Sketch the graph of f (x


(x) = ex 2 + 1, marking the asymptote and intercept, and
state the transformations, domain and range. Find the y-intercept correct to
2 decimal places.
THINK

eBoo
k plus
eBook
Tutorial

int-0536
Worked example 10

WRITE/DRAW

Write the rule.

f (x) = ex 2 + 1

State the basic shape.

Exponential curve

State the transformations.

The horizontal translation is 2 units to the right and


the vertical translation is 1 unit up.

Find the asymptote by translating the asymptote


of f (x) = ex up one unit.

The horizontal asymptote is y = 1.

Find the y-intercept.

If x = 0, y 1.14 (to 2 decimal places)


so the y-intercept is 1.14.

Find another point by translating the y-intercept of (0, 1) (2, 2)


f (x) = ex to the right by 2 units and up by 1 unit.

Chapter 4

Exponential and logarithmic graphs

193

Sketch the graph.

(2, 2)

(0, 1.14) 2
1 0
8

State the domain and the range.

Asymptote
y=1

f (x) = ex 2 + 1

The domain is R and the range is (1, ).

A reflection in the x-axis is shown by a negative sign before the ex term.


A reflection in the y-axis is shown by a negative sign before the x term.
WORKED EXAMPLE 11

Sketch the graph of f (x


(x) = 2 ex, marking the asymptote and intercepts. State the transformations,
domain and range. Give exact answers. Check using a CAS calculator.
THINK

WRITE/DRAW

State the rule.

f (x) = 2 ex

State the basic shape.

Exponential curve

State the transformations.

A reflection in the x-axis and a reflection in the


y-axis. The vertical translation is 2 units up.

Find the horizontal asymptote by translating


the asymptote of f (x) = ex up 2 units.

The horizontal asymptote is y = 2.

Find the y-intercept by making x = 0 or by


reflecting (0, 1) in the x-axis and translating it up
2 units.

If x = 0,

Find the x-intercept by making y = 0


and solving the equation.
1
ex = x
e
loge (eex) = x loge (e)
=x1

If y = 0,

y=2e 0
=21
=1
or (0, 1) (0, 1) (0, 1)
The y-intercept is 1.
2 ex = 0
=2
ex
1
=2
ex
ex = 12
loge (eex) = loge 1
2

x=

loge 1
2

so the x-intercept is loge 1 .


2

Sketch the graph.

y
(loge( 12 ), 0)

Asymptote
y=2
(0, 1)
0
1

f (x) = 2 ex
x

2
8

194

State the domain and the range.

The domain is R and the range is (, 2).

Maths Quest 12 Mathematical Methods CAS for the TI-Nspire

To graph y = 2 ex, on a Graphs page, complete


the function entry line as:
f1(x) = 2 ex
Then press ENTER .
Note: The horizontal asymptote at y = 2 is not
displayed.

2 , g(x) = e2x
2 1 and h(x) = e2x
2 2
The graphs of f (x) = e2x
are drawn at right.

2
f x) = e 2x
f(
2 1
g(x) = e 2x

2 2
h(x) = e 2x

(0, 1)
1
(0, e1)
0

1
2
f (x
( ) = e2x

2 1
g(x
( ) = e2x
(x
= e 2 x

x-intercept

y-intercept

(0, 1)

Domain
Range
Horizontal asymptote

1
1
0, e = 0, e

2 2
h(x
( ) = e2x
(x
2(
2(x
= e 1)

1
2
0, 2 = 0, e
e

R+ = (0, )

R+ = (0, )

R+ = (0, )

y=0

y=0

y=0

1
2

Horizontal translation
A point on the graph

1
2

(0, 1)

unit to right

( , 1)
1
2

(1, 1)
( 12 , 1)
Asymptote
(0, e2) y = 0
x
1

1 unit to right
(1, 1)

In summary: Transformations can be represented in the following way:


1. Dilation by factor A from the x-axis can be written as y = Af (x), A > 0.
2. (a) Dilation by factor 1 from the y-axis can be written as y = f (kx), k > 0.
k
(b) Dilation by factor k from the y-axis can be written as y = f 1 x , k > 0.
k
3. (a) Translation of B units up can be written as y = f (x) + B, B > 0.
(b) Translation of B units down can be written as y = f (x) B, B > 0.
4. (a) Translation of b units to the right can be written as y = f (x b), b > 0.
(b) Translation of b units to the left can be written as y = f (x + b), b > 0.
5. (a) Reflection in the x-axis can be written as y = f (x).
(b) Reflection in the y-axis can be written as y = f (x).

( )

Chapter 4

Exponential and logarithmic graphs

195

REMEMBER

For all the graphs of the form f (x) = Aex + b + B, where A R+, b, B R, the maximal
domain is R, the range is (B, ), the horizontal asymptote is y = B. When A R
the range changes to (, B).
1. A is the dilation factor from the x-axis. As A increases, the graph becomes steeper.
2. B is the vertical translation. When B > 0 the graph is translated up B units. When B < 0
the graph is translated down B units.
3. b is the horizontal translation. When b > 0 the translation is to the left. When b < 0 the
translation is to the right.
4. When A < 0 the graph is reflected in the x-axis.
5. When f (x) = ex the graph is reflected in the y-axis.
6. When f (x) = ekx the graph is dilated by factor 1 from the y-axis.
k
7. Consider transformations in the following order: reflection, dilation, translation.
EXERCISE

4C

Graphs of exponential functions with base e


1 WE7 State the transformations of f (x
(x) = ex needed to form each
of the following functions.
2
a f (x) = e3x
b f (x) = e2x
c f (x) = 4 e
d f (x) = 2e
e f (x) = 1 + ex 2
f f (x) = 2 + ex + 5
2
2x
g f (x) = 3 e
h f (x) = 1 e3x
2
i f (x)
1
x
2

x
3

eBoo
k plus
eBook
Digital doc

Spreadsheet 034
Exponential functions

2 WE8
For each of the following examples sketch the graph of f (x
(x) = ex. On the same set
of axes, sketch the graph of the given functions, marking the asymptote and y-intercept, and
state the transformation, the domain and the range. Give exact answers.
a f (x) = ex 1
b f (x) = ex 3
c f (x) = 2eex
d f (x) = 3eex
e f (x) = 1 + ex
f f (x) = ex 2
g f (x) = e2 x
h f (x) = e1 x
i f (x) = 3 ex
3 WE9 Sketch the graph of each of the following functions, marking the asymptote and
intercept, and stating the transformations, domain and range. For the intercepts give exact
answers or correct to 1 decimal place where appropriate.
2
a f (x) = e3x
b f (x) = e2x
c f (x) = 4eex
1
1 4x
x
d f (x) = 2ee
e f (x) = 2 e
f f (x) = 4 e 2 x
g f (x) = 2e3x
h f (x) = 3e4x
4 WE10
Sketch the graph of each of the following, marking the asymptote and intercepts,
and stating the transformations, domain and range. Mark the y-intercepts as exact values.
a f (x) = ex + 3
b f (x) = ex 3
c f (x) = ex + 1
x
x
+
1
d f (x) = e + 2
e f (x) = e
+2
f f (x) = ex + 2 + 1
x

1
x

2
g f (x) = e
+3
h f (x) = e
+5
i f (x) = ex + 2 1
5 WE11 Sketch the graph of each of the following functions, marking the asymptote and
intercepts, and stating the transformations, domain and range. For intercepts, give exact
answers or correct to 1 decimal place where appropriate.
a f (x) = ex
b f (x) = ex
c f (x) = 1 ex
x
x
d f (x) = 2 + e
e f (x) = 1 + e
f f (x) = 1 + ex
x
x
g f (x) = 3 e
h f (x) = 5 e
i f (x) = 2ex
196

Maths Quest 12 Mathematical Methods CAS for the TI-Nspire

2 3 1, marking all the key features and stating the domain


6 Sketch f:
f R R where f (x
(x) = e2x
and range. Write answers correct to 2 decimal places where appropriate.

7 Sketch f:
f [0, ) R where f (x
(x) = ex 2 1, marking all the key features and stating the domain
and range, giving answers correct to 2 decimal places where appropriate.
8 If f (x
(x) = ex, sketch the following functions.
a y = f (x) + 1
b y = f (x) 2
1
d y = 2 f (x)
e y = 1 f (x)
g y = f (x + 1)
h y = f (x 2)
j y = 1 + f (x)

c y = 2f
2f (x)
f y = 2 f (x)
i y = f (x)

2 + 3 is translated so that the new graph has a horizontal


9 MC The graph with equation y = e2x
asymptote of y = 1. The new graph has undergone a translation of:
A 1 unit down
B 2 units down
C 3 units down
D 4 units down
E 5 units down

10 MC An increasing exponential function has a horizontal asymptote of y = 2 and a y-intercept


of 4. A possible equation for this exponential is:
2 +4
2 4
A y = e2x
B y = e4xx + 2
C y = e2x
x
x
y
D y = 2ee + 4
E y = 2ee + 2

11 The graph shown is modelled by the equation y = 2eex b + B.


a Find the values of B and b.
b If the graph was translated down 1 unit, translated 3 units
to the left and dilated by factor 12 from the x-axis, write the
equation of the new graph.

4D

4
3
2
2 1

(3, 3)
Asymptote
y=1
0 1 2 3

Logarithmic graphs to base e


Graphs of logarithmic functions with base e are drawn in exactly the same way as with any other
base. The function f (x) can be dilated, translated and reflected in the same way.
The graphs of f (x) = log2 (x), g(x) = log10 (x)
y Asymptote
f x) = log2 (x)
f(
x=0
and h(x) = loge (x) are shown at right. The graph of
2
h(x) = loge (x)
h(x) = loge (x) is in blue.
(1, 0)
1
f (x) = log2 (x) and h(x) = loge (x) are steeper than
g(x) = log10 (x)
g(x) = log10 (x).
x
Remember h(x) = loge (x)
1 0 1 2 3 4 5
1
h(x) = ln (x)

Common features
1. The graphs all cross the x-axis at (1, 0) because loga (1) = 0.
2. The vertical asymptote is the y-axis (x = 0) because loga (0) is undefined.
3. The domain is R+.
4. The range is R.
5. They are all increasing functions.
WORKED EXAMPLE 12

State the transformations of f (x


(x) needed to form the graph of f (x
(x) = 2 loge (x
( 3) + 1.
THINK

WRITE

State the rule.

f (x) = 2 loge (x 3) + 1.

State the dilation. The coefficient of loge (x) gives the


dilation from the x-axis.

Dilation is 2 units from the x-axis.

Chapter 4

Exponential and logarithmic graphs

197

State the translations.


(a) Horizontal translation is given by the constant added
to the x term.

Horizontal translation is 3 units to the


right.

(b) Vertical translation is given by the constant added to


the log term.

Vertical translation is 1 unit up.

Dilations do not change the vertical asymptote, the domain or the range of a logarithmic graph.
The x-intercept does change, however.
WORKED EXAMPLE 13

Sketch the graph of f (x


(x) = 3 loge (2x
(2 ), marking all key features and stating the domain and range.
THINK

WRITE/DRAW

Write the rule.

f (x) = 3 loge (2x


(2 )

State the basic shape of the curve.

A log graph with basic shape f (x) = loge (x).

State the vertical asymptote, which occurs at loge (0). Vertical asymptote is the y-axis.

Find the x-intercept by letting y equal 0.


Solve for x. Divide both sides by 3.
Use loge (x) = a ea = x.
Use e0 = 1.
Divide both sides by 2.

Sketch the graph of f (x) = 3 loge (2x


(2 ).

If y = 0, 3 loge (2x
(2 ) = 0
loge (2x
(2 ) = 0
e0 = 2x
2
2 =1
2x
x = 12
The x-intercept is 12.
y Asymptote
x=0
6
4
2

State the domain and the range.

1 0
2

f (x) = 3 loge (2x


(2 )
( 12 , 0)
1

2 3

The domain is R+, and the range is R.

Translations do not change the shape of the basic graph, only the position. The asymptotes,
intercepts and domain may change but the range stays the same.
WORKED EXAMPLE 14

Sketch the graph of f (x


(x) = ln ((x + 1) 2, marking the vertical asymptote and the intercepts.
THINK

198

WRITE/DRAW

State the rule.

f (x) = ln (x + 1) 2

State the basic shape.

Log graph with shape f (x) = ln (x).

Find the vertical asymptote by making


(x + 1) = 0.

x + 1 = 0 for asymptote
so x = 1 is the vertical asymptote.

Find the y-intercept by making x equal to 0.


Remember that ln (1) = 0.

If x = 0, y = ln (0 + 1) 2
= ln (1) 2

= 2

y-intercept = 2.

Maths Quest 12 Mathematical Methods CAS for the TI-Nspire

Find the x-intercept by making y equal to 0.


Use ln (x) = a ea = x.

Sketch the graph.

If y = 0,

ln (x + 1) 2 = 0
ln (x + 1) = 2
e2 = x + 1
x = e2 1
x 6.39
x-intercept 6.39 (to 2 decimal places)
Asymptote
x = 1
y
2

0 2
2 (0, 2)

(e2 1, 0)
4 6 x

f (x) = ln (x + 1) 2

A reflection of f (x) = loge (x) in the x-axis does not change the graphs asymptote, x-intercept,
domain or range. If there is a y-intercept, the sign changes.
A reflection in the y-axis does not change the vertical asymptote or the range, but the
x-intercept and the domain change.

WORKED EXAMPLE 15

eBoo
k plus
eBook

Sketch the graph of f (x


(x) = 2 3 loge (1 x), marking the asymptote and intercepts.
State the domain and range and check using a CAS calculator.
THINK

Tutorial

int-0537

Worked example 15

WRITE/DRAW

State the rule.

f (x) = 2 3 loge (1 x)

Find the vertical asymptote by translating the


line x = 0 one unit to the right or by making
1 x = 0.

Vertical asymptote is x = 1.

Find the y-intercept by making x equal to 0 and


solving the equation.

If x = 0, y = 2 3 loge (1)
=2

Find the x-intercept by making y equal to 0.

If y = 0, 2 3 loge (1 x) = 0
3 loge (1 x) = 2
loge (1 x) = 23
2

e3 = 1 x

x = 1 e3
x 0.95 (to 2 decimal places)
5

Sketch the graph, remembering that there is a


reflection in both the x- and the y-axes.

y
4
(0, 2)
2
(1 e 3 , 0)

f (x) = 2 3 loge (1 x)

x
22 1 0 1
2 Asymptote
x=1
6

State the domain and the range.

The domain is (, 1) and the range is R.

Chapter 4

Exponential and logarithmic graphs

199

To graph f (x) = 2 3 loge (1 x) on a Graphs


page, complete the function entry line as:
f1(x) = 2 3 ln (1 x)
Then press ENTER .
Note: The vertical asymptote at x = 1 is not
displayed.

To sketch a graph by using transformations it is necessary to dilate, reflect and then translate.
REMEMBER

1. The function f (x) = loge (x) is an increasing logarithmic


Asymptote
y x=0
function:
y = loge (x)
(a) The vertical asymptote is the y-axis so there are no
y-intercepts.
(b) The graph crosses the x-axis at (1, 0) because
loga (1) = 0.
x
0 (1, 0)
(c) The domain is R+; that is, there are no negative
values of x.
(d) The range is R.
(e) After transformations, the function becomes
f (x) = A loga k(x + b) + B.
2. Dilation
(a) Dilations from the x-axis are given by the coefficient of loge (x). Then, y = f (x)
becomes y = Af (x).
(b) Dilations from the y-axis are given by the reciprocal of the coefficient of x. Then,
y = f (x) becomes y = f (kx).
(c) Dilations do not change the vertical asymptote, the domain or the range of a
logarithmic graph. Dilations from the y-axis change the x-intercept.
3. Translation
(a) Vertical translation is given by the term added to loge (x). Then, y = f (x) becomes
y = f (x) + B. If B > 0 the graph is moved up, and if B < 0 the graph is moved down.
(b) Horizontal translation is given by the term that is added to x. y = f (x) becomes
y = f (x + b). If b > 0 the graph is translated to the left, and if b < 0 the graph is
translated to the right.
(c) Translations do not change the shape of the basic graph, only the position. The
asymptotes, intercepts and domain may change but the range stays the same.
4. Reflection
(a) Reflection in the x-axis occurs if the coefficient A is negative. Then, y = f (x)
becomes y = f (x).
(b) Reflection in the y-axis occurs if the coefficient of x is negative. Then, y = f (x)
becomes y = f (x).
(c) When following a sequence of transformations to sketch a graph, it is necessary to
dilate, reflect and then translate.

200

Maths Quest 12 Mathematical Methods CAS for the TI-Nspire

EXERCISE

4D

Logarithmic graphs to base e


1 WE12 State the transformations of f (x
(x) needed to form the graph of each of the following.
a f (x) = 5 loge (2x
(2 )
b f (x) = 2 loge (4x)
c f (x) = loge (x + 1) 3
d f (x) = loge (x 2) + 1
e f (x) = loge (x)
f f (x) = loge (2x
(2 )
g f (x) = 1 loge (x 2)
h f (x) = 2 loge (x + 3)
i f (x) = 3 loge (x + 4) 1
j f (x) = 1 loge (x 4)
k f (x) = 2 loge (1 x) + 3
l f (x) = 3 loge (2 x) 1

eBoo
k plus
eBook
Digital doc

Spreadsheet 069
Logarithmic
graphs

2 WE13
Sketch the graph of the following functions, marking all key features and stating
the domain and range. Give exact values.
a f (x) = loge (2x
(2 )
b f (x) = loge (3x)
c f (x) = 3 loge (x)
x
d f (x) = 2 loge (x)
e f (x) = 2 loge
f f (x) = 3 loge (2x
(2 )

(3)

3 WE14 Sketch the graph of the following functions, marking the vertical asymptote and the
intercepts. Give exact answers for a f, otherwise round to 1 decimal place.
a f (x) = 1 + loge (x)
b f (x) = 2 + loge (x)
c f (x) = loge (x 1)
d f (x) = loge (x 2)
e f (x) = loge (x + 2)
f f (x) = loge (x + 3)
g f (x) = loge (x 3) + 2
h f (x) = loge (x 1) + 1
i f (x) = loge (x + 3) 2
j f (x) = loge (x + 1) 2
4 WE15 Sketch the graph of the following functions, marking the asymptote and intercepts, and
stating the domain and range. Give intercepts correct to 2 decimal places where appropriate.
a f (x) = loge (x)
b f (x) = loge (x)

c f (x) = 2 loge (x)


d f (x) = loge (2x
2 )

e f (x) = 3 loge ( 2x
2 )
f f (x) = 2 loge ( 4x)
g f (x) = loge (1 x)
h f (x) = loge (2 x)
i f (x) = loge (2 x)
j f (x) = loge (3 x)
5 The graph of f (x
(x) = loge (x
( ) undergoes the following transformations. Find the equation of the
image of f (x
(x) in each case.
a Translation of 1 unit to the right.
b Translation of 2 units down.
c Dilation of 5 units from the x-axis.
d Dilation of 3 units from the y-axis.
e Reflection in the x-axis.
f Reflection in the y-axis.
g Reflection in the x-axis and translation of 3 units to the left.
h Dilation of 4 units from the y-axis and translation 1 unit up.
i Reflection in the x-axis, dilation of 2 units from the x-axis and translation of 3 units
down.
6 Sketch the graphs of the equations found in question 5, showing the asymptote and intercepts,
and stating the domain and the range. Give intercepts correct to 2 decimal places where
appropriate.
7 State the transformations of f (x
(x) needed to form the graph of f (x
(x) = 2 loge (3x + 6) 1 and
sketch the graph, showing the asymptote and intercepts. Give intercepts correct to 2 decimal
places.

8 Sketch the graph of ff: ( , 1] R where f (x


(x) = 3 loge (2 x), showing key features.
9 MC The graph of y = loge (x) is transformed into the graph of y = 5 loge (2x
(2 ) by:
A A dilation of factor 5 from the x-axis and a dilation of factor 2 from the y-axis
B A dilation of factor 2 from the x-axis and a dilation of factor 5 from the y-axis
C A dilation of factor 15 from the x-axis and a dilation of factor 2 from the y-axis
D A dilation of factor 5 from the x-axis and a dilation of factor 12 from the y-axis
E A dilation of factor 12 from the x-axis and a dilation of factor 5 from the y-axis.
Chapter 4

Exponential and logarithmic graphs

201

10 MC For the function f (x) = 3 loge (x 2), the vertical asymptote and the coordinates of the
x-intercept are respectively:
A x = 3, (2, 0)
B x = 2, (3, 0)
C y = 3, (2, 0)
D y = 2, (3, 0)
E x = 3, (3, 0)
a
11 MC The function ff: [0.5, 6] R where f (x) =
is undefined when x is equal to:
logg e ( x )
A 0
B 0.5
C 1
D 6
E a
12 a When the function f (x
(x) = a + b loge (x
( ) is reflected in the x-axis and
translated 2 units to the right, find g(x
( ), the equation of the image of f (x
(x
(x).
b State the domain and range of g(x) and write it using correct function
notation.
c If g(x) is reflected in the y-axis and dilated by factor 4 from the
x-axis, write in correct function notation the equation of h(x),
the image of g(x).

4E

eBoo
k plus
eBook
Digital doc

WorkSHEET 4.2

Finding equations for graphs


of exponential and logarithmic
functions
As with other functions it is sometimes necessary to be able to find the equation of an
exponential or logarithmic function from a graph.
If we know points on the curve, we can substitute the values into the most suitable general
equation:
1. For an exponential graph the general equation is y = Ae(x + b) + B.
2. For a logarithmic graph the general equation is y = A loge (x + b) + B.
Both of these examples are written with base e but they could be written with any base, for
example y = A 2(x + b) + B or y = A log10 (x + b) + B.
If there are two unknowns, two pieces of information are necessary. The coordinates of two
points, substituted into a general equation, will give two equations and enable two unknowns to
be found.

WORKED EXAMPLE 16
y
4

The equation of the graph shown is of the form f (x


(x) = Aex + B.
Find the values of A and B correct to 2 decimal places and hence
find the equation of the function.

(0, 2)
(2.44, 0)
4 3 2 1

THINK
1

2
3

202

Use the point on the y-axis and substitute values


into the given equation.
Substitute the coordinates of another point
into the given equation.
Solve simultaneous equations by subtracting
[1] from [2].

0 1 x

WRITE

For (0, 2):


2 = Ae0 + B
2=A+B
(2.44,
2.44

For
0 = Ae

[2] [1]:

Maths Quest 12 Mathematical Methods CAS for the TI-Nspire

[1]

0):
+B

[2]
2

2.44

= Ae
A

= A(e 2.44 1)

Find A, rounding the answer correctly.

A=

2.444

Substitute in [1]: 2 =

2.444

e
1
2.19 (to 2 decimal places)

Find B by substituting the exact value of A in


equation [1] or [2] to find B.

B=2

0.19
x
f (x) = 2.19ee 0.19

1
2
2.444

+B
1

Rewrite the original equation, substituting


values for A and B.
Note: You can use a CAS calculator to solve equations [1] and [2].
1 On a Calculator page, press:
MENU b
3:Algebra 3
1:Solve 1
Press /r to obtain the expression
template and choose the simultaneous
equation template for two unknowns.
Complete the entry line as:
a + b = 2

solve
,
a

2.44
+b=0
a e
Press ENTER .
2

Write down the solutions correct to 2 decimal


places.

A = 2.19 and B = 0.19

The horizontal asymptote of an exponential equation gives the vertical translation and hence the
value of B in the general equation f (x) = Aex + B.
WORKED EXAMPLE 17

The equation of the graph shown is of the form f (x


(x) = aex + b. Find the
values of a and b and hence find the equation of the function.

y
10
(0, 5)

THINK

WRITE

3 2 1

Asymptote
y=2
0 1

Use the horizontal asymptote to find the value


of b.

The graph of ex has been translated up 2 units, so


b = 2.

2
3

Substitute the value of b in the equation.


Use the y-intercept to find a by substituting in
the given equation.

Write the equation, using the values of a and b.

f (x) = aex + 2
For (0, 5):
5 = ae0 + 2
ae0 = 3
a=3
f (x) = 3eex + 2

If there are three unknowns, three pieces of information are necessary to solve the equation.

Chapter 4

Exponential and logarithmic graphs

203

The vertical asymptote of a logarithmic graph gives the horizontal translation and hence the
value of b in the equation y = A loge (x + b) + B. A translation to the left gives a positive value of
b and a translation to the right gives a negative value of b.
WORKED EXAMPLE 18

The equation of the graph shown is of the form


y = A loge (x
( + b) + B. Find the values of A, b and B
and hence find the equation.

eBoo
k plus
eBook

x = 1
5 (0, 5)
(2, 0)
1

Use the vertical asymptote to find the value of b.

2
3

Substitute the value of b into the equation.


Use the y-intercept to find an equation.

Simplify using loge (1) = 0.

Substitute the value of B in the equation.


Use the x-intercept to find A.

The graph of f (x) = loge (x) is translated 1 unit to


the left, so b = 1.
f (x) = A loge (x + 1) + B.
For (0, 5):
A loge (0 + 1) + B = 5
A loge (1) + B = 5
=5
f (x) = A loge (x + 1) + 5
For (2, 0):
A loge (2 + 1) + 5 = 0
A loge (3) + 5 = 0
A loge (3) = 5

Substitute values back into the original equation.

5
log e (3)
A 4.55
(to 2 decimal places)
f (x) = 4.55 loge (x + 1) + 5
A=

WRITE

int-0538
Worked example 18

0
THINK

Tutorial

REMEMBER

1. The general equation for an exponential graph is f (x) = Aek(x + b) + B, where A is


the dilation factor from the x-axis, 1 is the dilation factor from the y-axis, b is the
k
horizontal translation and B is the vertical translation.
2. The general equation for a logarithmic graph is f (x) = A loge k(x + b) + B, where A
is the dilation factor from the x-axis, 1 is the dilation factor from the y-axis, b is the
k
horizontal translation and B is the vertical translation.
3. The coordinates of points on the curve can be substituted into the general equation to
find the values of unknowns in the equation.
4. One piece of information is required for each unknown in the general equation.
5. The horizontal asymptote of an exponential graph gives the vertical translation and
hence the value of B. If the asymptote is above the x-axis B is positive and if it is below
the x-axis the value of B is negative.
6. The vertical asymptote of a logarithmic graph gives the horizontal translation and
hence the value of b. If the asymptote is to the left of the y-axis b is positive and if it is
to the right of the y-axis b is negative.

204

Maths Quest 12 Mathematical Methods CAS for the TI-Nspire

EXERCISE

4E

Finding equations for graphs of exponential and


logarithmic functions
1 WE16 The equation of the graph shown is of the form
f (x
(x) = Aex + B. Find the values of A and B correct to 2 decimal
places and hence find the equation of the function. State the
equation of the horizontal asymptote.

y
4
2

(1, 4)
(0, 3)
0 1

2 Find the values of b and B if the general equation of the


graph shown is y = ex + b + B. Give your answers correct to
2 decimal places.

y
6

(1, 6)

4
2 (0, 2)
10.5 0 0.5 1 1.5

3 Find the values of A and k if the general equation of the


graph shown is y = Aekx. Give your answers correct to
2 decimal places.

y
0
2 1
(1, 4)
5

10
(2, 10)

4 Find the values of A and B if the general equation of the


graph shown is y = A 2x + B. Give exact answers.

y
4
(1, 3)
2
x
33 2 1 0 1 2
Asymptote
(3, 1)
2

y = 19
15

5 WE17 The equation of the graph shown is of the form


f (x
(x) = a 2x + b. Find the values of a and b and hence
find the equation of the function. Give exact answers.

4
2

(0, 112 )
1 Asymptote
y=1
0 1 2 x
3 2 1

6 MC If the horizontal asymptote is y = 1 and the y-intercept is 2, the equation for the
exponential function of the form y = e x + b + B is:
A y = ex + 1 2
B y = ex + 2 + 1

x
+
1
C y= e
2
D y = ex 1 2

x
+
1.1
E y= e
+1

Chapter 4

Exponential and logarithmic graphs

205

7 Find the values of b and B if the general equation of the graph


shown is y = e x + b + B. Give exact answers.

y
6
4

(2, 4)
Asymptote
y=3

2
0 1

8 Find the values of A and B if the general equation of the


graph shown is y = Aex + B . Give exact answers.

4
(0, 3)
2
0 1 2 x
Asymptote
2
y = 1

3 2 1

9 WE18 The equation of the graph shown is of the form


y = A loge (x
( + b) + B. Find the values of A, b and B and hence
find the equation. Give answers correct to 1 decimal place.

y
Asymptote
6

x= 2

10 Find the values of A and k, given that the graph of


y = A log10 kx passes through the points (1, 1) and (3, 2). Give
your answers correct to 4 decimal places where appropriate.

11 If the horizontal translation is 2 units to the left, the vertical


translation is 3, the graph passes through the point (2, 6), and the
equation is of the form y = A log2 (x
( + b) + B, find the values
of A, b and B.

4F

Addition of ordinates

(2, 6)

2
3 2 1

0 1

eBoo
k plus
eBook
Interactivity

Sometimes we need to sketch the graph of a function that can be thought


int-0249
of as the sum of two functions. For example, the function y = x2 + ex can Addition of ordinates
be thought of as the sum of the functions y = x2 and y = ex.
Such a graph can be drawn by sketching the two individual functions on the same set of
axes, then adding the y-values (ordinates) for each x-value and plotting the resulting points.
This method is convenient to use when we know the basic shape of the individual functions
but cannot recognise the basic shape of the given function. We can sketch both graphs on the
same set of axes, then plot the resulting points by adding the y-values for each x-value, and
hence sketch the new curve.
For example, the graph of y = x2 + ex can be sketched using the addition of ordinates
technique, since the basic shape of the function is not known, but the two individual functions
are a basic positive parabola and a basic exponential curve. The graph of y = ex + 1 can also
be thought of as the sum of two functions, but since we can recognise its shape as the basic
exponential curve translated 1 unit up, there is no need to use the addition of ordinates method.
Note that the sum function can only be defined for the domain over which both of the
individual functions are defined. So the domain of the sum function is obtained by finding
the intersection of the domains of the individual functions. That is, if h(x) = f (x) + g(x),
domain h(x) = domain f (x) domain g(x).
Sometimes a function is defined as the difference between two individual functions. We
can still use the method of addition of ordinates to graph such a function, because the second
function can be expressed as the addition of a negative function. That is, h(x) = f (x) g(x) can
be written as h(x) = f (x) + (g(x)).
206

Maths Quest 12 Mathematical Methods CAS for the TI-Nspire

So if, for example, we need to graph y = x2 ex, we could think of it as y = x2 + (ex). We then
would sketch the basic parabola and the basic negative exponential curves on the same set of
axes and use the addition of ordinates technique to obtain the graph of the original function.
WORKED EXAMPLE 19

State the domain of f (x


(x) = 3x
3 2 + loge (x
( ).
THINK

WRITE

Write f (x) as the sum of the two individual


functions g(x) and p(x).

Let f (x) = g(x) + p(x), where g(x) = 3x2


and p(x) = loge (x).

State the domain of g(x).

Domain g(x) = R

State the domain of p(x).

Domain p(x) = R+

State the domain of f (x) by finding the intersection


of the domains of individual functions.

Domain f (x) = domain g(x) domain p(x)


= R R+
= R+

When sketching the graph by hand, it is important to select wisely the values of x for which the
ordinates should be added. As a guide, good points to select are:
1. the end points of the graph
2. the points of intersection of the graphs
3. the x-intercepts of either of the graphs.
WORKED EXAMPLE 20

Using the same scale and axes, sketch the graphs of y1 = ex and y2 = ex over the domain [2, 2).
Hence, sketch the graph of y = ex + ex, rounding coordinates to 1 decimal place.
THINK
1

Sketch y1 = ex by finding the horizontal asymptote,


the y-intercept, and the end points.

WRITE/DRAW

The horizontal asymptote is the x-axis.


x = 0, y = e0, y = 1
x = 2, y = e2, y = 7.4

x = 2, y = e 2, y = 0.1
The y-intercept is 1.
y
8

(2, 7.4)

y1 = ex

4
(2, 0.1)

(0, 1)
0

2 1
2

On the same axes, sketch y2 = ex by reflecting


y1 in the y-axis and finding the horizontal
asymptote, the y-intercept, and the end points.

The horizontal asymptote is the x-axis.


The y-intercept is 1.

y2 = ex
(2,
2, 7.4)

y1 = ex

y
8

(2, 7.4)

6
4
(2, 0.1)
2 1

Chapter 4

(0, 1)
0

(2, 0.1)
x

Exponential and logarithmic graphs

207

Add ordinates at the end point with the lowest


x-value (y = y1 + y2 = ex + ex).

When x = 2, y = e 2 + e2
7.5
Point (2, 7.5)

Add ordinates when the graphs intersect.

When x = 0,

y=1+1
=2

Point (0, 2)
When x = 2,
Point (2, 7.5)

Add ordinates at the other end point.

Plot the points (which were obtained by adding


ordinates), and join them to sketch the graph.
It is necessary to label only end points and
intercepts.

y2 = e

(2, 7.5)

y 7.5
y
8
6
4
2

2 1

y1 = ex

(2, 7.5)

y = ex + e x

(0, 2)

0 1

WORKED EXAMPLE 21
y

Given the graphs of f (x


(x) and g(x
( ),
(x
sketch the graph of h(x
( ) = f (x
(x
(x) + g(x
( ).
(x

f x)
f(
g(x)
x

0
THINK
1

Add the ordinates at the point where g(x)


has the x-intercept and mark the resulting
point on the set of axes. Note that g(x) = 0
at this point; therefore f (x) + 0 = f (x) (that
is, if one of the functions cuts the x-axis,
the sum is equal to the y-value of the other
function).

Add the ordinates at the first point of


intersection of the 2 functions. Note that
at the point of intersection the value of
y is the same for both functions, so the
resulting point is double the y-value.
Mark the point on the set of axes.

208

Add the ordinates at the LHS end points


of the graph: a large positive value plus
a smaller negative value should give a
smaller positive value. Mark this point on
the axes.

WRITE/DRAW
1

Add the ordinates at the point where


f (x) has the x-intercept and mark the
result on the set of axes.

Maths Quest 12 Mathematical Methods CAS for the TI-Nspire

f x)
f(

f x)
f(

g(x)

g(x)

f x)
f(

f x)
f(

g(x)
0

g(x)
0

Add the ordinates at the second point


of intersection of the 2 graphs and mark
the resulting point on the set of axes.

Add the ordinates at the RHS end points


of the graph: 2 positive values together
give an even larger positive value.

Join the points with a smooth curve to


produce the graph of h(x).
Note that the turning point of h(x) is
between the y-axis and the point with
the same y-value. The graph begins to
turn at the left end point because the
graph of g(x) turns there.

f x)
f(

f x)
f(

g(x)

g(x)

h(x)

f x)
f(
g(x)
0

WORKED EXAMPLE 22

Sketch the graph of f (x


(x) = x + x , using addition of ordinates.
THINK
1

State the two individual functions.

State the domain of f (x) by finding the


intersection of the domains of individual
functions.

On the set of axes sketch the graph of g(x)


(a straight line, passing through the origin
and bisecting the first quadrant) and p(x) (a
basic square root curve). Note that only the
first quadrant is needed, since the domain
is R+ {0}.

WRITE/DRAW

Let f (x) = g(x) + p(x), where g(x) = x and p(x) = x


Domain g(x) = R
Domain p(x) = R+ {0}
Domain f (x) = domain g(x) domain p(x)
Domain= R R+ {0}
Domain= R+ {0}
1

y
g(x)
p(x)

0
4

The first point of intersection of g(x) and


p(x) is at the origin, f (x) = 0 + 0 = 0 (that
is, f (x) will also start at the origin). Mark
this point on the set of axes.
Add the ordinates at the second point of
intersection of the two graphs and mark the
resulting point on the set of axes.

g(x)

y
g(x)

p(x)

p(x)

Add the ordinates at the RHS end points of


the graph and mark the resulting point on
the set of axes.

Chapter 4

Exponential and logarithmic graphs

209

Sketch the graph of f (x) by joining the points.


Note, as x increases, f (x) approaches g(x)
because p(x) is increasing slowly.

f x)
f(
y
g(x)
p(x)

Further graphs
The example below shows how to draw the graph of a product function.
WORKED EXAMPLE 23

Sketch the graph of y = x2ex using a CAS calculator. Show all axis intercepts and any asymptotes.
THINK

WRITE/DISPLAY

To graph y = x2ex on a Graphs page, complete


the function entry line as:
f1(x) = x2ex
Then press ENTER .
Note: y = x2ex is asymptotic to the negative
x-axis, as when x , x2ex 0.

Substitute x = 0 to locate the y-intercept.


Write the y-intercept in coordinate form
(which is also the x-intercept).

y = 02e0
y=0
(0, 0)
y = 0 is an asymptote.

REMEMBER

1. A graph of the sum of 2 functions can be drawn by sketching the 2 functions on the
same set of axes and adding the y-values for each value of x.
2. The domain of the sum function is obtained by finding the intersection of the
domains of individual functions: if h(x) = f (x) + g(x),
domain h(x) = domain f (x) g(x).
3. Suitable points at which to add ordinates are:
(a) left end points and right end points
(b) the points of intersection of the 2 graphs
(c) the x-intercepts of either of the 2 functions.
4. Exact values of y can be obtained by substituting the required x-values into the given
function.
5. The difference function can be rewritten as the sum function as follows:
h(x) = f (x) g(x)
= f (x) + [g(x)],
so that addition of ordinates can be used.

210

Maths Quest 12 Mathematical Methods CAS for the TI-Nspire

EXERCISE

4F

Addition of ordinates
1 WE19 State the domain of each of the following functions.
1
a y=x+
b y=3 x x
x
d y= x+2 x
g y=

e y = x2 3x3

2
x2
( x + 1)2

h y = 3x 2 1 x

c y = x2 + 1 x 1
2
1
f y = 2x
2 3
x 3
2
i y = 3(2 x)2 +
3x

j y = 2 3 x 3 x +1
2 MC If f (x) = g(x) + p(x), and if f (x) is defined over the domain [2, 7] and g(x) is defined
for x 7, the domain of p(x) could not be:
A [2, 7]
B [2, )
C [2, 7)
D [2, 7.5)
E [2, 15]
3 WE20 Using the same scale and axes, sketch the graphs of y1 and y2 over the given domain.
Hence, sketch the graph of y = y1 + y2, rounding coordinates to 1 decimal place as this is
accurate enough for most sketch graphs. Check all graphs using a CAS calculator.
a y1 = 2eex, y2 = 2ex, [1, 1]
b y1 = 3ex, y2 = 3eex, [1, 1]
x

c y1 = e , y2 = x, ( 2, 2]
d y1 = ex, y2 = x, (2, 2]
x
2

e y1 = e , y2 = x , ( 2, 2)
f y1 = ex, y2 = x2, (2, 3)
4 Using the same scale and axes, sketch the graphs of f (x
(x) and g(x
( ). Hence, sketch the graph of
(x
h(x
( ) = f (x
(x
(x) + g(x
( ), rounding coordinates to 1 decimal place as this is accurate enough for most
(x
sketch graphs. State the domain and range of h(x
( ). Check all graphs using a CAS calculator.
(x
a f (x) = 2 loge (x), g(x) = x
b f (x) = 3 loge (x), g(x) = x
c f (x) = loge (x), g(x) = 2x
2
d f (x) = loge (x), g(x) = 12 x

e f (x) = loge (x), g(x) = x


f f (x) = 2 loge (x), g(x) = x
5 Using the addition of ordinates, sketch the graph of ff: [2, 2] R, where f (x
(x) = 12 ex + 12 ex.
State the domain and the range, giving answers to 1 decimal place where rounding is
necessary.
6 MC If the domain of y1 is (2, 2] and the domain of y2 is (0, ), the domain of y1 + y2 is:
A (2, )
B [2, )
C (0, )
D (2, 0)
E (0, 2]
7 WE21 Given the graphs of f (x
(x) and g(x
( ), sketch the graph of h(x
(x
( ) = f (x
(x
(x) + g(x
( ).
(x
y
y
y
a
b
c
f x)
f(

g(x)

g(x)

f x)
f(

x
y

x
f x)
f(
g(x)

g(x)

f x)
f(

f x)
f(

g(x)

x
0

f x)
f(
g(x)

8 On the same set of axes sketch the graphs of f (x


(x) = x2 and g(x
( ) = 5x + 6. Use the addition-of(x
2
ordinates method to sketch the graph of y = x + 5x + 6.

Chapter 4

Exponential and logarithmic graphs

211

9 On the same set of axes sketch the graphs of f (x


(x) = x3 and g(x
( ) = x2 1 and hence sketch the
(x
graph of y = x3 + x2 1, using the addition-of-ordinates technique.
10 WE22 Sketch the graph of each of the following functions, using addition of ordinates. Check
your answers using a CAS calculator.
a y= x + 2 x

b y = 2x
2 x

c y = x 3 x2

d y= x+5 + 5 x
y

11 MC The graph at right is likely to represent the sum of which


2 functions?

D
0

x
0

12 Sketch the graph of y = loge x + x by sketching the 2 individual functions on the same set of
axes and then adding the ordinates. State the domain of the function. Verify your answer with a
CAS calculator.
13 If f (x
(x) = 3x 3 and g(x
( ) = loge x, sketch the graph of h( x ) = f ( x ) + g( x ), using addition of
(x
ordinates. Check your answer using a CAS calculator.
14 WE23 For each of the following, sketch the graph using a CAS calculator. Show all axis
intercepts and any asymptotes. State the domain and range.
2
a y = x2e2x

b y = x2ex
c y = ex
2

15 For each of the following, sketch the graph using a CAS calculator. Show all axis intercepts
and any asymptotes. State the domain and range.
a y = x2 loge (x)
b y = x2 loge (3x)
c y = x loge (x)

4G

Exponential and logarithmic


functions with absolute values
x for x 0
The modulus, or absolute value, function is defined as f ( x ) = x =
.
x for x < 0
To obtain the graph of y = f x , the graph of y = f (x),
where x 0, is reflected in the y-axis.

212

Maths Quest 12 Mathematical Methods CAS for the TI-Nspire

The rule of the composite function y = f x ,


where f (x) = loge (x), x > 0, can be written as:
logg e ( x )
y = log e x =

logg e ( x )

2
1

for x > 0
for x < 0

0
1

2 x

2
3

To obtain the graph of y = f ( x ) , negative y-values


of y = f (x) are reflected in the x-axis. The rule
of the composite function y = f ( x ) , where
f (x) = loge (x), x > 0, can be written as:
logg e ( x )
y = log e ( x)
x) =
logg e ( x )

y
4
3
2
1

for 0 < x < 1

for x 1

2
y

The rule of the composite function y = f x ,


where f (x) = ex, can be written as:
e x for x 0
y = ex =
x for x < 0
e

(0, 1)
2

y=

2 x

The rule of the composite function y = f ( x ) ,


where f ( x ) = e x k, k R+, can be written as:
ex

4 x

y=k

(e x k ) for x < log e k


k =
for x log e k
e x k

(0, |1 k|)

2 x

(loge(k), 0)
WORKED EXAMPLE 24

For the function y = 2 log e x + 2 3:


a sketch the graph of y = 2 log e x + 2 3, showing any asymptotes
b calculate all axis intercepts both in exact form and correct to 2 decimal places
c state the domain and range.
THINK
a

WRITE/DRAW
y

3
Sketch the graph of y = 2 log e x + 2 3,
x > 2. The graph is not defined when x = 2,
so there is a vertical asymptote at x = 2.

2
1
7 6 5 4 3 22 110

1 2 3 4 5 x

2
3
Asymptote
x = 2

Chapter 4

Exponential and logarithmic graphs

213

Reflect y = 2 log e x + 2 3 in the vertical


asymptote at x = 2.

2
1
7 6 5 4 3 22 110

1 2 3 4 5 x

2
3
Asymptote
x = 2

To graph y = 2 log e ( x + 2) 3 with a CAS


calculator, open a Graphs page and complete
the function entry line as:

f1(x) = 2 log e x + 2 3
Then press ENTER . Press /r and
select the absolute value template.

To locate the x-intercepts, express


y = 2 log e x + 2 3 as a hybrid function.

Substitute y = 0 and solve each equation.

Write the equation in exponential form and


solve for x.

2 log e ( x + 2) 3
y=
2 log e ( ( x + 2)))) 3
2 loge (x + 2) 3 = 0
3
logg e ( x + 2) =
2

for x > 2
for x < 2

Repeat for the other x-intercept.

x + 2 = e2
3

x = e2 2
x = 2.481 69
2 loge ((xx + 2)) 3 = 0
3
logg e ( ( x + 2))
2)) =
2
( x + 2)

= e2

x = e2 2
x = 6.481 69
5

Write down the coordinates of the


x-intercepts in exact form and correct to
2 decimal places.

3
3

e 2 2, 0 and e 2 2, 0

(2.48, 0) and ( 6.48, 0)

Substitute x = 0 to obtain the y-intercept


and simplify.

y = 2 log e 0 + 2 3

y = 2 log e 0 + 2 3

y = 2 log e 2 3

y = 2 log e 2 3

y = 1.613 71
7

214

Write down the coordinates of the y-intercept (0, 2 loge (2) 3)


in exact form and correct to 2 decimal places. (0, 1.61)

Maths Quest 12 Mathematical Methods CAS for the TI-Nspire

State the domain.

x R\{2}

State the range.

yR

WORKED EXAMPLE 25

eBoo
k plus
eBook

a Sketch the graph of y = e 1 showing all axis intercepts and asymptotes.


b State the domain and range.
x

THINK
a

Note that the graph of y =


1 now has a
horizontal asymptote at y = 1.

Substitute x = 0 to obtain the y-intercept and


simplify.

int-0539
Worked example 25

WRITE/DRAW

Sketch the graph of y = e x 1 .


The graph has a horizontal asymptote at
y = 1.
Reflect the negative part (below the x-axis)
of y = e x 1 and the asymptote about the
x-axis.

Tutorial

y
2
1
2

0
11

Asymptote
y=1
x
2 Asymptote
y = 1

ex

y = e0 1
= 11
=0

Write the coordinates of the y-intercept


(which is also the x-intercept).

The coordinates of the y-intercept are (0, 0).

State the domain.

xR

State the range.

y R+{0}

REMEMBER

x for x 0
1. The modulus, or absolute value, function is defined as f ( x ) = x =
.
x for x < 0
2. For y = f x , the graph of y = f (x), where x 0, is reflected in the y-axis.
3. For y = f ( x ) , negative y-values of y = f (x) are reflected in the x-axis.
EXERCISE

4G

Exponential and logarithmic functions with


absolute values
1

WE24 Sketch the graphs of the following functions, showing all axis intercepts and asymptotes
in exact form. For each graph, state the maximal domain and the range of the function.

b y = log e ( x + 1) 2
c y = log10 3 x + 1
y = 3 log e x 2 + 3
2 Sketch the graphs of the following showing all axis intercepts and asymptotes in exact form.
For each graph, state the maximal domain and the range of the function.
y = log e ( x 2)

b y=

log

e (1

x ) 2)
x)

Chapter 4

c y = log10 ( x + 3) + 1

Exponential and logarithmic graphs

215

3 Sketch the graphs of the following showing all axis intercepts in exact form. For each graph,
state the maximal domain and the range of the function.
a

x 1
y = e x1
+4

c y = e
4

x 1

+2

WE25 Sketch the graphs of the following showing all axis intercepts and asymptotes in exact
form. For each graph, state the maximal domain and the range of the function.

y = ex 3
c y= e

4H

b y = e 4 x 3

b y = ex 2 + 4

1 + 2

Exponential and logarithmic


modelling using graphs
As seen in chapter 3, exponential and logarithmic functions can be used to model real situations.
Graphs of these functions can be used to illustrate the model and make predictions for future
changes.
In most cases when modelling real life situations, the domain is restricted to [0, ) because
t = 0 when the model begins.

WORKED EXAMPLE 26

The population of wombats in Snubnose Gully is increasing according


to the equation:
W = 100e0.03t
where W is the number of wombats t years after 1 January 1998.
a Find the initial size of the population.
b Find the population 2 years and 10 years after the number of wombats was first recorded. Give
answers to the nearest whole wombat.
c Graph W against t for 0 t 30.
d Find the expected size of the population in the year 2020.
e Find the year in which the wombat population reaches 250.
WRITE/DRAW

THINK
a

216

a W = 100e0.03t

State the rule.

Find W when t = 0.

When t = 0, W = 100

Write the answer in a sentence.

The initial size of the population is


100 wombats.

Find W when t = 2.

b When t = 2, W = 100e0.03

= 100 1.0618
106 (nearest whole number)

Write the answer in a sentence.

After 2 years there are 106 wombats.

Find W when t = 10.

When t = 10, W = 100e0.03 10


= 100 1.3499
135 (nearest whole number)

Write the answer in a sentence.

After 10 years there are 135 wombats.

Maths Quest 12 Mathematical Methods CAS for the TI-Nspire

To graph f (x) = 100e0.03x on a Graphs


page, complete the function entry
line as:
f1(x) = 100e0.03x
Then press ENTER .
Note the viewing window settings:
Xmin:0, Xmax:30, Ymin:0 and
Ymax:250
When drawing your graph, label the
axes with the given variables.

Convert the year 2020 to the correct


number of years.
Find W by substituting t = 22 into the
equation.

d t = 2020 1998

= 22 years

When t = 22, W = 100e0.03 22


= 193.479
= 193 (nearest whole number)
In the year 2020, there are approximately
193 wombats.

Write the answer as a sentence.

Let W = 250.

Divide both sides by 100.

2.5 = e0.03

Take natural logs of both sides.

loge (2.5) = loge (e0.03 t)


0.03t = loge (2.5)

Divide both sides by 0.03.

t=

Evaluate the answer using a CAS


calculator.

t = 30.543
t = 31 (nearest year)

Express t = 31 as a year and write the


answer as a sentence.

There will be 250 wombats in the year 2029.

e 250 = 100e0.03

1
logg ( 2.5)
0.003 e

REMEMBER

1. In general the number of unknowns indicates the number of points required to


substitute into the equation.
2. The equation for exponential growth is A = A0ekt, where A0 represents the initial
value, t represents the time taken and k represents the rate constant. This is a rapidly
increasing function.
3. The equation for exponential decay is A = A0ekt, where A0 represents the initial
value, t represents the time taken and k represents the rate constant. This is a rapidly
decreasing function.
EXERCISE

4H

Exponential and logarithmic modelling using


graphs
1

WE26 The population of a species of koala found on Eucalyptus Island is increasing according
to the equation K = 50e0.04t, where K is the number of koalas t years after 1 January 1998.

Chapter 4

Exponential and logarithmic graphs

217

eBoo
k plus
eBook
Digital doc

Spreadsheet 041
Function
grapher

a Find the initial size of the population.


b Find the population 2 years and
10 years after the number of koalas was first
recorded. Give answers to the nearest whole
number.
c Plot a graph of K against t.
d Use the graph to find the size of the population after
15 years.
2 The population of a species of wallaby found on a reserve is increasing according to the equation
W = 150 1.08t, where W is the number of wallabies t years after records were first kept.
a Find the initial size of the population.
b Find the population 1 year and 5 years after records were first kept. Give answers to the
nearest whole number.
c Plot a graph of W against t.
d Use the graph to find the size of the population after 15 years.
e Use the graph to find how long it would take for the population to double.
3 A student invests $500 with a company that pays interest of 6% compounded continuously.
(Interest paid according to the formula A = A0ert is said to be compounded continuously and r
is called the continuous interest rate.)
a How much money made up the initial investment?
b How much did the student have with the company after 1 year? Give your answer correct
to the nearest 5 cents.
c How much interest did the student have after 5 years? Give your answer correct to the
nearest 5 cents.
d Plot a graph of A against t.
e Use your graph to find how much the student would have in the account after 8 years.
4 The decay of a radioactive element, E, is given by the equation E = E0 ekt, where E is the
number of radioactive nuclei present t days after the experiment begins.
a If 200 radioactive nuclei are present in the element at the beginning of the experiment
and there are 33 radioactive nuclei present after 10 days, find the value of k correct
to 2 decimal places. Use this rounded value for all working in the other parts of this
question.
b Find the number of radioactive nuclei, E, present after 1 day and after 5 days.
c Plot a graph of E against t.
d Use the graph to find how long it would take before 50 radioactive nuclei are left.
e Would there ever be no radioactive nuclei left? Give reasons for your answer.
f The half-life of a radioactive element is the time taken for half of the radioactive nuclei to
decay. Use your graph to find the half-life of this element.
5 A hard-boiled egg is placed in water to cool. The cooling process can be modelled by the
equation T T1 = (T
T0 T1)ekt, where T is the temperature of the egg t minutes after it was
placed in the water and T1 is the temperature of the water. Assume that the temperature of the
egg is 98 C when it is first placed in the 18 C water and it takes 5 minutes for it to cool to
38 C.
a Substitute the values of T0 and T1 into the equation and simplify, making T the subject.
b Find the value of k to 3 decimal places.
c Substitute it into the equation.
d Find how long it would take to reach a temperature of 25 C. Give your answer to the
nearest minute.
e Draw a graph of T against t, marking in asymptotes.
f Use the graph to find the temperature after 15 minutes.
g Assuming that the water does not become significantly warmer, use your graph to
determine whether the egg will ever reach the temperature of the water.

218

Maths Quest 12 Mathematical Methods CAS for the TI-Nspire

6 The population of a species of fish in Lake


Underwater is declining due to an unknown
cause. The number of fish t weeks after the
first dead fish was found on the surface of the
water can be modelled by the equation
P = A loge (t) + B. One week after the first
dead fish was discovered, the population was
10 000. The population was 8000 after
5 weeks.
a Find the values of A and B. Give answers
correct to 2 decimal places where
appropriate.
b Find how many fish there are after 10
weeks.
c Find how long it would take for the
population to be 3000. Give your answer
to the nearest week.
d Sketch a graph of P against t, clearly
indicating the asymptote.
e Use the graph to find how many fish there
would be after 20 weeks.
7

MC The relationship between m and n is modelled by the


equation m = log10 (an + b). The graph is shown at right.
The values of a and b are:
A 3 and 2
B 2 and 3
C 3 and 2

D 1 and 2
E 1 and 3

m
2
1
1

(4, 1)

(1, 0)
0 1

8 It costs a clothing company $20 to produce a jacket. Production costs are proportional to the
number of jackets produced.
a If the company produces n jackets, write an equation for the companys production costs
for the jackets, $C.
As the company produces more jackets they find that they have to sell them at a lower price.
The companys revenue, $R, is modelled by the equation

R(n) = 2000(1 e 0.1n)


b Show that R(0) = 0.
c According to this model, revenues plateau as costs increase. What is the value that the
revenue approaches?
d On the same axes, sketch the graphs of the cost equation and the revenue equation for
0 n 55.
e Use addition of ordinates to sketch the graph of the profit the company can expect to
make from selling the jackets (profit = revenue cost).
f Write an equation for this profit.
g Use a CAS calculator to find the number of jackets that must be sold in order to maximise
the profits and what that profit would be.
h Use a CAS calculator to find how many jackets the company could make before they
began to make a loss.
i Is this a reasonable model for a company to use? Explain your reasons.

Chapter 4

Exponential and logarithmic graphs

219

SUMMARY
Exponential functions
y
For graphs of the form f (x) = ax, where a R+\{1}:
1. The maximal domain is R.
4
2. The range is R+.
3. The x-axis is the horizontal asymptote.
2
4. The y-intercept is 1.
(0, 1)
5. They are all increasing functions.
x
0
1 2
2 1
Reflection:
1. If f (x) = ax is reflected in the x-axis the result is the graph of f (x) = ax. The graph is a decreasing function
instead of an increasing function. The y-intercept changes to (0, 1) and the range becomes R.
2. If f (x) = ax is reflected in the y-axis the result is the graph of f (x) = ax. All key features stay the same but
the graph is a decreasing function instead of an increasing function.
Translation:
For all of the graphs of the form f (x) = ax + b + B, where b, B R, and a R+\{1}, the maximal domain is R,
the range is (B, ), the horizontal asymptote is y = B and they are all increasing functions. b translates the
graph horizontally, B translates the graph vertically.
Dilation:
If f (x) = Aakx, where A, k R+, the graph of f (x) = ax is dilated by factor A from the x-axis and by factor 1
k
from the y-axis.
Combinations of transformations:
For all the graphs of the form f (x) = A ak(x + b) + B, where b, B R, A, k R+ and a R+\{1}, the maximal
domain is R, the range is (B, ), the horizontal asymptote is y = B and they are all increasing functions. b
translates the graph horizontally, B translates the graph vertically and A dilates the graph by factor A from the
x-axis, k dilates the graph by factor 1 from the y-axis. If A, k < 0 the graphs are reflected in the x- and
k
y-axes, respectively.
Logarithmic functions

For graphs of the form f (x) = loga (x), where a R+\{1}:


y
1. The maximal domain is R+; that is, there are no
negative values of x.
2. The range is R.
3. The vertical asymptote is the y-axis so there are no
x
0 (1, 0)
y-intercepts.
4. The graph crosses the x-axis at (1, 0) because
loga (1) = 0.
5. They are all increasing functions.
Reflection:
1. If f (x) = loga (x) is reflected in the x-axis the result is the graph of f (x) = loga (x). All key features remain
the same but the graph is a decreasing function instead of an increasing function.
2. If f (x) = loga (x) is reflected in the y-axis the result is the graph of f (x) = loga (x). The vertical asymptote
and the range remain the same but the x-intercept and the domain change.
(a) The graph crosses the x-axis at (1, 0).
(b) The domain is (, 0).
Dilation:
1. The function f (x) = A loga (x) dilates the graph of f (x) = loga (x) by a factor of A from the x-axis. The
vertical asymptote, x-intercept, domain and range remain the same. As A increases, the graph becomes
steeper.

220

Maths Quest 12 Mathematical Methods CAS for the TI-Nspire

2. The function f (x) = loga (kx) dilates the graph of f (x) = loga (x) by a factor 1 from the y-axis. The vertical
k
asymptote, domain and range stay the same, but the x-intercept is 1 . As k increases, the graph becomes
k
steeper and the x-intercept becomes smaller.
Translation:
1. The function f (x) = loga(x) + B translates the graph of f (x) = loga (x) vertically, B units. The vertical axis,
domain and range remain the same but the x-intercept changes. There is no change in the shape of the
graph.
2. The function f (x) = loga (x + b) translates the graph of f (x) = loga (x) horizontally b units. The shape and
the range remain the same but the vertical asymptote, the x-intercept and the domain change:
(a) The vertical asymptote becomes x = b.
(b) The graph crosses the x-axis at (1 b, 0).
(c) The domain is (b, ).
Combinations of transformations:
The function f (x) = A loga (x + b) + B has dilation factor A from the x-axis and is translated b units
horizontally and B units vertically. If A < 0 there is a reflection in the x-axis and if x is used there is a
reflection in the y-axis.
Addition of ordinates

A graph of the sum of two functions can be drawn by sketching the two functions on the same set of axes
and then adding the y-values for each value of x.
If h(x) = f (x) + g(x), domain h(x) = domain f (x) domain g(x).
Suitable points at which to add ordinates are:
1. the end points of the graph
2. the points of intersection of the two graphs
3. the x-intercepts of the two graphs.
The technique can be used for the difference of the two functions, if it is rewritten as a sum:
h(x) = f (x) g(x) = f (x) + [g(x)].
Further graphs

Use a CAS calculator to graph the product of two functions or composite functions.
Obtain the equation of any asymptote of the function by considering asymptotic behaviour of the individual
functions.
On your graph, clearly label:
1. asymptotes
2. axis intercepts.
Absolute value graphs

x for x 0
The modulus, or absolute value, function is defined as f ( x ) = x =
.
x for x < 0
For y = f x , the graph of y = f (x), where x 0, is reflected in the y-axis.
For y = f ( x ) , negative y-values of y = f (x) are reflected in the x-axis.

Chapter 4

Exponential and logarithmic graphs

221

CHAPTER REVIEW

SHORT ANSWER
( + 1) 4, showing
1 Sketch the graph of x) = 2(x
intercepts and asymptotes, and stating the domain
and range.

2 Sketch the graph of f (x


(x) = 3 log10 (2x
(2 ), showing
intercepts and asymptotes, and stating the domain
and range.
3 Find the equation of the graph below, given that it
is of the form f (x
(x) = Aex + B.
y
4
2
x
4 3 2 1 0 1 2
(0,
1)
2
y = 4
44
5

4 The graphs of f (x
(x) and g(x
( ) are shown below.
(x
Sketch the graph of ((f + g)(x
)( ), using the addition)(x
of-ordinates technique.
y
a
g(x)
f x)
f(
x

g(x)

b determine the rule for g 1(x), and hence state

the domain and range of g 1(x).


8 If h(x
( ) = f (x
(x
(x) + g(x
( ) where f (x
(x
(x) = x2 + 1 and
g(x
( ) = loge (x
(x
( ), sketch the graph of h(x
( )=
(x
f (x
(x) + g(x
( ). State the domain and range
(x
of h(x
( ).
(x
9 Describe a sequence of transformations that
maps the graph of y = f (x
(x) on to the graph of
y = 2f
2f (3 x) + 5.
10 After a protection program for tigers was
introduced in a province of India on 1 January
2006, the population of the tigers is modelled by
T t) = 50 20.4t, where t is the number of months
T(
after the start of the program. At the same time, the
number of elephants in the province is modelled by

E(t) = 400 4 0.1t.


Find:
a the number of tigers and elephants in the
province on 1 January 2006
b which of the two animals has the
highest numbers in the province on
1 April 2007
c the date when the population of the tigers
will equal the population of elephants in the
province.
If the number of elephants falls below 25, they are
at risk of extinction in this province.
d According to the model, will this happen? If
so, when will it happen?

f x)
f(
x

5 a Use the law loga (m


mp) = p loga (m) to simplify
2
f (x
(x) = log10 (x
( ) and hence sketch the graph of
the function ff: (0, ) R where f (x
(x) = log10 (x
( 2).
b Sketch the graph over the domain, R, noting that
it is now possible to take negative values of x.
6 P = P0et. If P = 120 when t = 0, find the exact
value of P when t = 20.
7 For the function g(x
( ) = 2eex + 1 4:
(x
a sketch the graph of g(x) by finding the equation
of any asymptotes and the coordinates of all
intercepts

222

11 Sketch the graph of y = log e ( x + 3) , showing all


intercepts and asymptotes in exact form. State the
maximal domain and the range.

Maths Quest 12 Mathematical Methods CAS for the TI-Nspire

12 Let f : D R, f ( x ) = 2 log
log e x + 3 + 1, where D is
the maximal domain of f.
f
a State D.
b Find the exact coordinates of the points where
the graph of y = f (x) intersects the x- and
y-axes.
c Sketch the graph of y = f (x). Indicate any
asymptote with its equation.

6 Which of the following graphs best represents the


function f (x
(x) = log10 (x
( 2) + 1?
A
y
x=2

2
6 4 2 0
2

2 4

2 4

2 4

EXAM TIP

R/{3}

R\{3}.

a Some students wrote


instead of
Some also wrote R3, or just 3, which are
insufficient when specifying the domain.
b Some students did not give exact values and
some missed the second x-intercept, because
they were not aware of the absolute value
function inside the natural logarithm. Instead
of writing loge (3), some students appeared to
write 3.
c Some students only drew the right-hand branch
of the function. Some did not show asymptotic
behaviour correctly, particularly by having
branches suddenly stop at a circle somewhere
on the asymptote. The function should not curve
away from the asymptote, nor touch it.

x = 2 y
2
6 4 2 0
2

x = 2 y
2

[Assessment report 1 2003]

[ VCAA 2003]

6 4 2 0
2

MULTIPLE CHOICE

1 The horizontal asymptote for the graph of


f (x
(x) = 2x 1 is:
A x=2
B x=1
C y=0
D y = 1
E y = 2

4 When the function f (x


(x) = log10 (x
( 1) is translated
2 units up and 1 unit to the left the function
becomes:
A log10 (x 2) + 2
B log10 (x 3) + 3
C log10 (x) + 2
D log10 (x + 1) + 2
E log10 (x + 2) + 1
5 The function f (x
(x) = log2 (x
( + 1) has as its domain:
A (1, )
B [1, )
C (1, )

D [ 1, )
E R

2 The domain and range of the graph of


f (x
(x) = 3 10x + 1 are respectively:
A R, [1, )
B R, (1, )
C R, [3, )
D R, (3, )
E R, R
3 The y-intercept for the graph of f (x
(x) = 1 2x is:
A 0
B 1
C 2
D 3
E 4

y
2

x=2

1 2 3 4 5 6 7

x = 2 y

3 2 1

0 1 2 3 x

7 The graph of f (x
(x) = 2eex + 1 is obtained from the
graph of f (x
(x) = ex by:
A a dilation of 2 units from the y-axis and a
translation of 1 unit to the left

Chapter 4

Exponential and logarithmic graphs

223

B a dilation of 2 units from the y-axis and a


translation of 1 unit to the right
C a dilation of 2 units from the x-axis and a
translation of 1 unit to the left
D a dilation of 2 units from the x-axis and a
translation of 1 unit to the right
E a dilation of 1 unit from the x-axis and a
translation of 2 units to the right
8 If f (x
(x) = ex, the function in the graph below is:
y

14 If f (x
(x) = loge (x
( + 2) + 1, then f (1) and f (0) are
respectively (correct to 2 decimal places):
A 2.10, 1.69
B 2.09, 0.69
C 2.098, 1.693
D 1.10, 1.69
E 2.10, 0.69
15 The domain of {{ff (x
(x) + g(x
( )} is:
(x
A dom f dom g
B dom f dom g
C dom f
D dom g
E R

16 If f (x
(x) = g(x
( ) + p(x
(x
( ), and dom g(x
(x
( ) = ( , 5] and
(x

dom p(x
( ) = ( 2, ), then the domain of f (x
(x
(x) is:

A ( 2, 5]
B [ 2, 5)

C [ 2, 5]
D ( 2, 5)
E none of the above

2
(0, 1)
x
3 2 1 0 1 2 3
Asymptote
2
y = 2

17 Which of the following shows the graph of


y=

A 3f
3f (x) + 2
C 2f
2f (x) + 3
E 2 + 3f
3f (x)

B 3f
3f (x) 2
D 2 3f
3f (x)

x+4 +
y

10 The vertical asymptote for the graph of


f (x
(x) = 3 loge (x
( 2) + 1 is:

A x= 1
B x=0
C x=1
D x=2
E x=3

4 x

13 The general equation of the graph shown


y
is y = Aex + B. The values of
4
A and B are:

(0, 3)
A 3, 1
B 2, 1

2
C 4, 1
D 1, 3
E 4, 1

4 x

4 x

y
4

12 If f (x
(x) = loge (x
( + 1), the y-intercept
-intercept of 22ff (x
(x) + 3 is:
A 0
B 1
C 2
D 3
E 4

4 x

C
4

11 For the function ff: [ 1, ] R where


f (x
(x) = 3 loge (x
( + 2), the domain and range are
respectively:

A ( 2, ), (0, )
B (2, ), R

D [ 1, ), [0, )
C ( 1, ), R+
E R+, R

18 The decay of uranium-235 is modelled by the


equation U = U0ekt, where U is the number of
grams of uranium-235 after t million years. If
a 1000 g mass of uranium-235 decays to 907 g
in 100 million years, the values of U0 and k are
respectively:

A 100, 0.000 976


B 1000, 0.000 976

C 10 000, 0.000 907


D 1000, 0.000 907
E 100, 907

19 The function g has the rule g(x


( ) = logex b,
(x
where b is a real constant. The maximal domain
of g is:
A R+
B R\{b}
0 2 x
C R
D (b, b)
Asymptote
[ VCAA 2006]
E (b, )

2 y = 1

224

9 If the horizontal asymptote is y = 2 and the


y-intercept is 1, a possible equation for the graph is:
A y = ex + 2
B y = 2eex + 1
C y = 2eex 1
D y = ex 2
x
E y=2e

6 4 2

4x ?

Maths Quest 12 Mathematical Methods CAS for the TI-Nspire

20 The maximal domain D of the function f:


f DR
with rule f (x
( ) = loge (x 3) + 6 is:
B (3, )
A R\{3}
D (3, )
C R

[ VCAA 2007]
E ( , 3)
21 The function f satisfies the functional equation
x + y f ( x) + f ( y)
f
=
where x and y are any
2
2
non-zero real numbers. A possible rule for the
function is:
A f ( x ) = log e x
B f ( x) = 1
x
C f ( x) = 2 x
D f ( x) = 2 x
E f ( x ) = sinn (2 x )

22 The function f has the property that


f (x
(x) + f (y
(y) = (x
(x + y) f (xy
(xy) for all non-zero real
numbers x and y. A possible rule for f is:
A f (x
(x) = 2x
2
B f (x
(x) = 5 2x
2
1
C f ( x) = x
D f (x
(x) = ex
E f ( x ) = log e x
[ VCAA 2005]

23 Let f (x) = ex. For all positive real numbers x and y,


f (x + y) is equal to:
A f (x
(x) + f (y
(y)
B f (x
(x) f (y
(y)
C f (xy
(xy)
D (f
(f (x
(x)) y
y
[ VCAA 2003]
E f (x
(x )

[ VCAA 2006]

EXTENDED RESPONSE

1 Sketch the graph of f (x


(x) = 2eex 1 + 1, showing all key features. State the domain and the range.
2 Sketch the graph of f (x
(x) = 1 loge (2 x), clearly showing intercepts and asymptotes, and state the domain
and range.
3 By adding ordinates, sketch the graph of f (x
(x) = log10 (2x
(2 ) + log10 (x
( ). State the domain and the range.
4 N is the number of bacteria in a culture where N = 10 000e0.04tt is the formula for the number of bacteria in the
culture after t hours. Find:
a the initial number of bacteria
b the number of bacteria after 10 hours. Give your answer to the nearest thousand.
5 Sketch the graph of y = 4 e x 1 showing all axis intercepts in exact form and any asymptotes. State the
maximal domain and the range of the function.
6 Part of the graph of the function f : R R, f (x
(x) = a + bex is shown at right.
The line with equation y =1 is an asymptote and the graph passes
through the origin.
a Explain why a = 1 and b = 1.
Let h be the function h:[0, 2] R, h(x) = f (x).
b State the range of h using exact values.

c Find the inverse function h 1.

y
y=1

EXAM TIP

a Most students were able to explain why a = 1 by discussing the asymptote; however, many of them were unable to explain
why b = 1. The most common error occurred when students tried to discuss reflections and translations but failed to
indicate which graph they were transforming.

b This question was generally well done. Some students were unable to obtain full marks because they rounded up 1 e 2 to
0.8647. Others used round brackets instead of square brackets.
c When the inverse function is asked for, the domain must be given. Students will be penalised if the domain is left out. If
only the rule for the inverse function is asked for, the domain does not have to be given. Some students gave the incorrect

answer h 1 ( x ) = log e 1 x .

[Assessment report 2 2007]

[ VCAA 2007]

Chapter 4

Exponential and logarithmic graphs

225

log e (5
(5 x)
x ) + 1.
7 A function f is defined by the rule f ( x ) = log
a Sketch the graph of f over its maximal domain. Clearly label any intersections with the axes with their
exact coordinates and any asymptotes with their equations.

b Find the rule for the inverse function f 1.


EXAM TIP

The graph on a calculator screen is of limited use. Many students did not give exact intercepts. Some tried to cheat by
finding numerical approximations and then writing these as fractions (which were incorrect). Many students failed to identify
the vertical asymptote at all, while some identified a horizontal asymptote. If they failed to identify the vertical asymptote, the
right-hand end of the graph frequently ended abruptly on or close to the x-axis. Many students were also unable to represent
asymptotic behaviour correctly. A few students still wrote a circle where their graph ended on the asymptote this should
not be encouraged, as students will lose marks for doing it.
[Assessment report 1 2005]

[ VCAA 2005]

8 Kerri invested $5000 with a company that pays interest of 5% compounded continuously. To answer the
questions below, use the formula A = A0ert, where A is the amount of the investment, A0 is the original
investment, r is the continuous interest rate and t is the number of years since the money was originally
invested. Round the answer to the nearest 5c.
a How much money did Kerri first invest with the company?
b How much money did Kerri have with the company after 1 year?
c How much did she have after 5 years?
If the company had been paying interest compounded quarterly, the formula used would have been

( )

4t

A = A0 1 + r .
4
d Using this system, how much would Kerri have after 5 years of the investment?
e How long would it take to double the investment? Give your answer in years.
f Which is the better investment? Give reasons.
g What is the difference in the amount of interest after 5 years?
h What would be the difference in 5 years if Kerri had invested $10 000?
9 A local council decided to build a new road along the coast. To make it safer it was decided to design the road
so that it followed the curve represented by the equation
y= 2 log10 (2x
(2 a) + 3 where a > 0.
y
The grid at right shows the road.
In each direction, 1 unit represents
Bridge
1 kilometre.
Ship
a If the new section of road goes
4
over the bridge marked on the
graph, find the value of a.
b Find the x-coordinate of the
point where the road begins.
Give the answer correct to
2
2 decimal places. How far would
this be from the vertical axis to
the nearest metre?
c What is the shortest distance
x
0
2
4
from the beginning of the road
6
8
1
3
5
7
9
to the coastline if the coastline is
on the vertical asymptote at that
point? Give the answer correct to the nearest metre.
d The main highway is along the x-axis. How far is the road from the main highway when it is in line with
the ship marked on the grid? Give the answer in kilometres, correct to 1 decimal place.

226

Maths Quest 12 Mathematical Methods CAS for the TI-Nspire

10 The air pressure P in kilopascals (kPa) at a height of x kilometres (km) above sea level may be modelled by
the equation P = aebx.
A mountain climber uses an altimeter to record air pressures at known heights on a climb of Mount
Kosciuszko (height 2.228 km). These pressures are shown in the table below.
x

0.5

1.0

1.5

2.0

101.3

95.2

89.4

84.0

78.9

a If b is between 0.1 and 0.2, find the value for b (to 3 decimal places) which produces the best fitting
model of the form P = aebx for the above data.
b Use your model to predict the air pressure at the top of Mount Kosciuszko.
2 bex + c is shown below.
11 The graph of the function f (x
(x) = e2x

y
y=6

(0, 2)
0

a
b
c
d
e
f
g
h

Find the values of b and c.


Show that the exact values of the x-intercepts are x = loge (2) and x = loge (3).
Use a CAS calculator to find the coordinates of the turning point. Round answers to 2 decimal places.
Find the exact values of the coordinates of the point of intersection of the function and the horizontal
asymptote.

eBoo
k plus
eBook
If the function is reflected in the x-axis, fully define the new function g(x) = f (x).
If the function is reflected in the y-axis, fully define the new function, h(x).
Digital doc
If the function is reflected in both the x- and the y-axis, sketch the graph of
Test Yourself
the new function, k(x), write its equation and state the domain and the range.
Chapter 4
Find the equation of f (2 x) + 1. State the domain and range, rounding to
2 decimal places where appropriate.

Chapter 4

Exponential and logarithmic graphs

227

eBook plus

ACTIVITIES

Chapter opener
Digital doc

10 Quick Questions: Warm up with ten quick


questions on exponential and logarithmic graphs.
(page 172)
4A

Graphs of exponential functions with


any base

Tutorial

WE 2 int-0534: Watch how to sketch a graph of an


exponential function. (page 177)
Digital doc

Spreadsheet 034: Investigate exponential graphs.


(page 181)
4B

Logarithmic graphs to any base

Tutorial

WE 5 int-0535: Watch how to sketch the graph of a


logarithmic function. (page 187)
Digital docs

Spreadsheet 069: Investigate logarithmic graphs.


(page 191)
WorkSHEET 4.1: Sketch graphs of exponentials and
logarithms, identify transformations and determine
rules for graphs. (page 191)
4C

Graphs of exponential functions


with base e

Tutorial

WE 10 int-0536: Watch how to sketch the graph of an


exponential function and state the transformations.
(page 193)
Digital doc

Spreadsheet 034: Investigate the exponential


function. (page 196)
4D

Logarithmic graphs to base e

Tutorial

WE 15 int-0537: Sketch the graph of a logarithmic


function stating the domain and range using a CAS
calculator to check. (page 199)
Digital docs

Spreadsheet 069: Investigate logarithmic graphs.


(page 201)
WorkSHEET 4.2: Sketch graphs of exponentials
and logarithms and determine points of intersection
between graphs. (page 202)

228

4E

Finding equations for graphs of exponential


and logarithmic functions

Tutorial

WE 18 int-0538: Watch how to find the equation of an


exponential given the graph. (page 204)
4F

Addition of ordinates

Interactivity int-0249:

Addition of ordinates: Consolidate your


understanding of addition of ordinates using the
interactivity. (page 206)
4G

Exponential and logarithmic functions


with absolute values

Tutorial

WE 25 int-0539: Watch a worked example on how to


sketch the graph of an absolute function. (page 215)
4H

Exponential and logarithmic modelling


using graphs

Digital doc

Spreadsheet 041: Invesigate graphs using a function


grapher. (page 218)
Chapter review
Digital doc

Test Yourself: Take the end-of-chapter test to test


your progress. (page 227)
To access eBookPLUS activities, log on to
www.jacplus.com.au

Maths Quest 12 Mathematical Methods CAS for the TI-Nspire

5A
5B
5C
5D

Relations and their inverses


Functions and their inverses
Inverse functions
Restricting functions

inverse
functions
AREAS oF STuDy

Graphs and identification of key features of


graphs of functions studied
Graphs of inverse functions
Functions and their inverses, including
conditions for the existence of an inverse
function, and use of inverse functions to solve
equations involving exponential, logarithmic
and power functions

The concept of an inverse function, connection


between the domain and range of the original
function and its inverse and the conditions for
the existence of an inverse function, including
the form of the graph of the inverse function for
specified functions
Finding the rule of an inverse function and
giving its domain and range
eBook plus

5A
eBook plus
Interactivity

int-0248
Relations
and their
inverses

Relations and their inverses

Digital doc

10 Quick Questions

You will recall from Maths Quest 11 Mathematical Methods CAS that a relation is a set of
ordered pairs that can be graphed or may be described by a rule.
As we have seen, the inverse of a relation can be found by:
1. interchanging the x- and y-coordinates of an ordered pair; for example, (1, 2) becomes (2, 1)
2. reflecting the relation in the line y = x
y
3. interchanging x and y in the rule and rearranging
y=x
3
the equation to make y the subject; for example,
(1, 2)
y = x 1 becomes x = y 1, giving y = x + 1.
2
(2, 1)
(0,
1)
The domain of a relation becomes the range of its inverse
1
and the range of a relation becomes the domain of its inverse.
(1, 0)
(1, 0)
x
0 1 2 3
The diagram at right shows a set of ordered pairs
3 2 1
1 (0, 1)

(2, 1)
A = {( 1, 2), (0, 1), (1, 0), (2, 1)}, the line y = x and the
2
inverse B = {(2, 1), (1, 0), (0, 1), (1, 2)}.
(1, 2)

3
The domain of A is { 1, 0, 1, 2} and the range of A is
{2, 1, 0, 1}.
The domain of B is {2, 1, 0, 1} and the range of B is {1, 0, 1, 2}.

using matrices to describe a reection in the line y = x


In chapter 2, we looked at how matrices can be used to find the image of a point that is
transformed on a plane, or to find the rule for the graph of a relation that undergoes a
transformation or series of transformations.

Chapter 5

inverse functions

229

As the graph of an inverse is the reflection in the line y = x of the original, we can use a matrix
to describe this transformation and find the image of a point on f (x), or the rule of the inverse

f 1(x) given f (x).


0 1 x
The matrix operation that produces a reflection in the line y = x is
.
1 0 y
Finding the image of a point using this method is trivial, as we know that the points on the
graph of an inverse are found by interchanging the (x, y) values of any point on the original. This
can be shown as follows:
The graph of a relation passes through the point (3, 4). Find the image of this point on the
graph of the inverse.
0 1 3 4

=
1 0 4 3
That is, (3, 4) maps to (4, 3).
A more general application will be to find the rule of the inverse of a relation. This can be
done using matrices but in practice this is unnecessary as it simply involves swapping the
variables and rearranging the new equation.
Again, it is not time effective to employ this method, but it is shown as a demonstration of the
relationship of the inverse graph to its original.
x x 0 1 x y
T = =
=
y y 1 0 y x
That is, (x, y) maps to (y', x' ).
Therefore, we can write y' = x and x' = y, which relates to the practice of reversing the
variables to find the rule of the inverse.
WoRkED ExAMplE 1

Sketch the graph of each of the following relations. State the domain and range of each.
a {(3, 1), (1, 1), (1, 3), (3, 5)}
b y = x2 + 2
Think
a

WRiTE/DRAW

Plot each coordinate pair on a set of axes.

The domain is the set of first elements of


the ordered pairs.

The range is the set of second elements of


the ordered pairs.

The relation is a parabola with a minimum


turning point (0, 2).

Sketch the parabola.

The domain is the set of real numbers.

The range is any real number greater than


or equal to 2.

y
5
4
3
2
1
321 0
1

Domain
= {3, 1, 1, 3}
1 2 3

Range
= {1, 1, 3, 5}

y = x2 + 2

(0, 2)
0

Domain = R
Range = [2, )

WoRkED ExAMplE 2

Find the inverse of each relation in worked example 1. Sketch the graph of each inverse relation,
stating its domain and range.

230

Maths Quest 12 Mathematical Methods CAS for the Ti-nspire

Think
a

WRiTE/DRAW
a {(3, 1), (1, 1), (1, 3), (3, 5)}

Write the original relation.

Interchange the x and y elements to


obtain the inverse.
Plot these points on a set of axes.

The inverse is {(1, 3), (1, 1), (3, 1), (5, 3)}
y
3
2
1
1 0 1 2 3 4 5 x
1
2
3

4
5

1
2
3
4
5

Domain = {1, 1, 3, 5}

State the domain, which is the set


of first elements of the ordered pair.
State the range, which is the set of
second elements of the ordered pair.
Write the original relation.
Interchange x and y in the rule.

Range = {3, 1, 1, 3}
b y = x2 + 2

The inverse is:


x = y2 + 2
y2 = x 2

Subtract 2 from both sides.


Take the square root of both sides to
make y the subject.

y= x2
y

The graph of y = x 2 is a
sideways parabola with a turning
point (2, 0).
(use a CAS calculator to verify this
graph.)

+ x 2
y =

Sketch the graph of the relation.


State the domain.

Domain = [2, )

State the range.

Range = R

Note that the domain of the original relation is the range of its inverse, and the range of the
original relation is the domain of its inverse.
WoRkED ExAMplE 3

eBook plus

For each relation graphed below, sketch the graph and its inverse on the same
axes. Draw in the line y = x.
a

(0, 5)

(2, 0) x

4
(2, 2)

(2, 4)

(2, 2)

Tutorial

int-0540
Worked example 3

(1, 4)

(3, 0) 0

Chapter 5

inverse functions

231

Think
a

DRAW

Copy the graph shown.

Interchange the points (2, 0) and (0, 5) to


(0, 2) and (5, 0), respectively, and mark them
on a set of axes.

Draw a straight line through each pair of


points.

Draw the line y = x, noting that the inverseis a


reflection of the original function in y = x.

Copy the graph shown.

Interchange the points (2, 2), (0, 0), (0, 4) and


(2, 2) to (2, 2), (0, 0), (4, 0) and (2, 2) and
mark on a set of axes.

Draw a circle through the second set of


4 points.

Draw the line y = x, noting that the inverse is a


reflection of the original relation in y = x.

1
2

Copy the graph shown.

(0, 2)
(5, 0)

(2, 0)

(0, 5)

y=x
b

y
(0, 4)

(2, 2)

y=x
(2, 2)

(4, 0) x

0
2
c

Interchange the points (3, 0), (2, 4), (0, 0)


and (1, 4) to (0, 3), (4, 2), (0, 0) and (4, 1)
and mark them on the same set of axes.
Draw a sideways cubic through the second
set of points.
Draw the line y = x, noting that the inverse is a
reflection of the original function in y = x.

(2, 4)

(2, 2)
y
4

(1, 4)

y=x

2
(4, 1)
(3, 0)
2

0
2
(0, 3)

(4, 2)

Note: This example shows that the graph of the inverse relation can be obtained by reflecting the
graph of the original relation in the line y = x. Some specific points can be marked and used as a
guide; intercepts are particularly useful. Note also that a relation and its inverse intersect on the
line y = x.

How to visualise the graph of an inverse


The following activity can help you to quickly see the shape of an inverse from the graph of a
relation.
Make a sketch of the graph of a relation in the right-hand bottom corner of a blank page. This
should be 4 or 5 cm square. You will need to use a pen or a dark pencil for best results.
Fold the corner in to the centre at 45. Keeping the page orientation the same, view the axes
through the paper and you will see the graph of the inverse!

232

Maths Quest 12 Mathematical Methods CAS for the TI-Nspire

Looking through
paper

Fold in along
this line at 45

REMEMbER

1.
2.
3.
4.

A relation is a set of ordered pairs which may be described by a rule.


The inverse of a relation can be found by interchanging the x and y elements.
The graph of a relation and its inverse are reflected in the line y = x.
If the rule for a relation is known, then the rule for its inverse is obtained by
interchanging x and y in the rule and then making y the subject.
5. The domain of a relation is the same as the range of its inverse and the range of a
relation is the same as the domain of its inverse.
ExERCiSE

5A
eBook plus
Digital doc

SkillSHEET 5.1
Domain and
range

eBook plus
Digital doc

Spreadsheet 041
Function grapher

Relations and their inverses


1 WE 1
Sketch the graph of each of the following relations. State the domain and range of
each.
a {(0, 1), (1, 2), (3, 2), (3, 5)}
b {(8, 7), (5, 2), (2, 1), (1, 1)}
c y=x
d y = 2x 5
e 2x + 4y = 8
f y = x2 + 4x
3
g y = x2 1
j y=
h y = (x + 1)2
i x2 + y2 = 4
x
k y=2
l x = 4
m y = 2x3
2 WE 2 Find the inverse of each relation in question 1.
Sketch the graph of each inverse relation, stating its
domain and range.
(Check your graphs using a CAS calculator.)

ExAM Tip
The best procedure for
determining the range of a function is
through sketching a graph for the given
domain.

3 MC
The graph of a relation passes through the
points (0, 1), (1, 2) and (3, 3). The graph of the inverse of this relation must pass through the
points:
A (2, 1) and (3, 3)
B (0, 1) and (3, 3)
C (0, 1) only
D (1, 2) and (3, 3)
E (3, 3) and (4, 10)

Chapter 5

inverse functions

233

4 MC
A relation has x-intercepts 2 and 3. The y-intercepts of the inverse of this relation
are:
B 2 and 3
C 2 and 3
A 2 and 3
D 2 and 3
E cannot be determined
5 WE3
For each relation, sketch the graph and its inverse on the same axes. Draw in the
line y= x.
a

4
3

(1, 1)

(2, 3)

5b

2 x

(1, 2)

Functions and their inverses


Recall that a function is a relation which has
only one y-value for each x-value. The graph of a
function can be crossed only once by any vertical
line.
To find the inverse of a function, we use the
same procedures that we used for relations in the
previous exercise.
1. The rule for the inverse of a function is
obtained by either:
(a)interchanging the x- and y-values of its
ordered pairs or
(b)interchanging x and y in the rule for the
function, then making y the subject.

234

2
4

y
0

1 0

(1, 1)
0

2 x

2
x

y
2

(1, 3)

Maths Quest 12 Mathematical Methods CAS for the TI-Nspire

y
y = f(x)

Vertical lines cross the curve only once.

2. The graph of the inverse of a function is obtained by either:


(a) interchanging the x- and y-values of its ordered pairs, or
(b) reflecting the graph of the function through the line y = x, or
(c) finding the rule of the inverse and graphing the new function.
The domain of a function is the range of its inverse. The range of a function is the domain of
its inverse.
WoRkED ExAMplE 4

State whether or not each of the following is a function by using the vertical line test.
a y = (x + 1)2 3
b x2 + y2 = 1
4
+1
c y=
x2
Think
a

WRiTE/DRAW

Write the equation.

State the type of graph.

Do a rough sketch and use the vertical line test


to determine if the relation is a function.

a y = (x + 1)2 3

A parabola with minimum turning point


(1, 3)
y

0
(1, 3)

State whether or not it is a


function.

Write the equation.

State the type of graph.

Do a rough sketch and use the vertical line test


to determine if the relation is a function.

(0, 2)

y = (x + 1)2 3 is a function.
b x2 + y2 = 1

A circle, centre (0, 0), radius 1


y
(0, 1)
(1, 0)

(1, 0)
0

(0, 1)

State whether or not it is a


function.

Write the equation.

State the type of graph.

x2 + y2 = 1 is not a function.
c y=

4
+1
x2

A hyperbola with asymptotes x = 2 and


y=1

Chapter 5

inverse functions

235

Do a rough sketch and use the vertical test to


determine if the relation is a function.

y (x = 2)

(2, 0) 0 (0, 1)

State whether or not it is a function.

y=

eBook plus

For each of the following functions, sketch, on the same set of axes, the graph of
the function, its inverse and the line y = x. State the domain and range for the
function and its inverse.
a y = x2 2x
b y = loge (x + 1)
a

(y = 1)

4
+ 1 is a function.
x2

WoRkED ExAMplE 5

Think

Tutorial

int-0541
Worked example 5

WRiTE/DRAW
a y = x2 2x

Write the original function.

Change it to turning point form by completing the


square.

= x2 2x + 12 12
= (x2 2x + 1) 1
= (x 1)2 1

Determine the turning point of the parabola.

This is a parabola with turning point


(1, 1).

Find the y-intercept by letting x equal 0 and


substituting in the original equation.

If x = 0, y = 02 2(0)
y=0
The y-intercept is 0.

Find the x-intercepts by letting y equal 0.

If y = 0, x2 2x = 0

Solve the equation by factorising the left-hand


side.

x(x 2) = 0
x = 0 and x = 2
The x-intercepts are 0 and 2.

Sketch the graph of the function.

0
1

x
1 2
(1, 1)
y = x2 2x

Interchange x and y to find the equation of the inverse.

x = (y 1)2 1

Rearrange to make y the subject.

x + 1 = (y 1)2
y 1= x +1
y = 1 x + 1 which is a sideways
parabola.

236

Maths Quest 12 Mathematical Methods CAS for the Ti-nspire

10

Interchange the x- and y-coordinates of the


turning point and intercepts in the original
equation to find the turning point and intercepts of
the inverse.

11

On the original axes, sketch the graph of the


inverse and the line y = x.

(1, 1) becomes (1, 1),


(2, 0) becomes (0, 2) and
the origin remains the same.
y

y=x
y = x2 2x

x
y=1 x+1

12

State the domain and range of the function.

The domain is R and the range is


[1, ).

13

State the domain and range of the inverse.

The domain of the inverse is [1, )


and the range is R.
b y = loge (x + 1)

Write the function.

Find the vertical asymptote using the fact that


loge (0) is undefined.

Vertical asymptote occurs where


x+1=0
x = 1 is the vertical
asymptote.

Find the x-intercept using loge (1) = 0.

x-intercept occurs where y = 0 or


x+1=1
x = 0 so the x-intercept is 0.

Sketch the graph of the function.

x = 1

y = loge (x + 1)

Interchange x and y to find the equation of the


inverse.

x = loge (y + 1)

Rearrange to make y the subject.

ex = y + 1
y = ex 1

Find the horizontal asymptote and intercepts from


the original function.

x = 1 is the original vertical


asymptote, so
y = 1 is the horizontal asymptote for
the inverse.
(0, 0) is on both graphs.

Chapter 5 Inverse functions

237

On the original axes, sketch the graph of the


inverse and the line
y = x.

x=

y = ex 1

y=x

y = loge (x + 1)
x
y = 1
9

State the domain and range of the function.

The domain is (1, ) and the


range is R.

10

State the domain and range of the inverse.

The domain of the inverse is R


and the range is (1, ).

Worked Example 6

If f(x) = ln (x + 1) + 1, use a CAS calculator to:


a find f 1(x) b draw the graph of f(x) and its inverse f 1(x).
Think
a

1
2
3

238

WRITE

Let y = f(x).
Interchange x and y.
To make y the subject, press:
Menu b
3:Algebra 3
1:Solve 1
Complete the entry line as:
solve(ln(y + 1) + 1 = x,y)
(This means to solve the equation
with respect to y).
Then press ENTER .

a y = ln (x + 1) + 1

x = ln (y + 1) + 1

Write the answer in the form f-1(x).


-

To draw the graphs of f(x) and f 1(x), open a


Graphs page.
Complete the function entry line as:
f1(x) = ln(x + 1) + 1
Then press ENTER .
Complete the function entry line as:
f2(x) = ex - 1 - 1
Then press ENTER .

f 1(x) = ex - 1 - 1
b

Maths Quest 12 Mathematical Methods CAS for the TI-Nspire

REMEMbER

1. A function is a relation which has only one y-value for each x-value. The graph of a
function can be crossed only once by a vertical line.
2. The graph of the inverse is obtained by:
(a) interchanging the x- and y-coordinates in the ordered pairs
(b) reflecting the graph in the line y = x
(c) finding the rule of the inverse and then graphing the inverse.
3. A horizontal asymptote y = a is the vertical asymptote x = a of the inverse, and vice
versa.
4. The domain of a function is the same as the range of its inverse. The range of a
function is the same as the domain of its inverse.
ExERCiSE

5b

Functions and their inverses


1 WE 4 State whether or not each of the following is a function by using the vertical line test.
a y = 2x 1
d y=

1
3
(2 x + 1)2

Digital doc

SkillSHEET 5.2
Matching graphs
with equations

c y=

e x2 + y2 = 4

f y = 3e(x 2) + 1

h y= x +2

g y = loge (x + 1) 2
eBook plus

2 Sketch the graph of each of the following functions and state the domain and range of each.
(Verify that it is correct using a CAS calculator.)
a (4, 2), (2, 0), (0, 1), (2, 4), (3, 6)}
c f (x) = 5 2x
d f (x) = x2 9
4
f f ( x) =
g f (x) = x2 + 8x
x

b 3x + 4y = 12
e f (x) = (x + 2)2
x3
h f ( x) =
2

i f (x) = 2ex

k f ( x) = 4 x 2

j f (x) = loge (2x)

3 WE 5
Find the inverse of each function in
question 2. Sketch the graph and state the domain and
range of each inverse. (Verify using a CAS calculator.)

ExAM Tip
When the inverse
function is asked for, the domain
must be given. Students will be
penalised in the future if the
domain is left out. If only the rule
for the inverse function is asked
for, the domain does not have to be
given.

4 For each function graphed below:


i copy the graph of each function and sketch its
inverse on the same axes
ii state the domain and range of f (x)
iii state the domain and range of the inverse of f (x).
eBook plus
Digital doc

Spreadsheet 059
Inverse graphs

4
1
x+3

b y = 3(x 1)2 + 2

y=x

f(x)

[Assessment report 2007]

f(x)

f(x)

1
0

x
0

y=x

y=x

Chapter 5

inverse functions

239

f(x)

y x=1

f(x)

3
f(x)

y=x

y=x

y=x
y

f(x)

y=x

f(x)
5

5x

y
y=x

(4, 1)
3

f(x)

y=x

3 x

y=x

y
x = 3

f(x)
6

3 2

eBook plus
Digital doc

WorkSHEET 5.1

2
y=x

Questions 5 to 8 relate to the following function.


f : R + R, f ( x ) =

(3, 4)

5 MC The range of f (x) is:


A R+
B R

y
4

f(x)

2
x

y=x

x
D R

E (, 0]

6 MC The domain of the inverse of f (x) is:


A R
B R
C R+

D [0, )

E (, 0]

7 MC The range of the inverse of f (x) is:


C [0, )
B R
A R

D (, 0]

E R+

C [0, )

8 MC The inverse of f (x) can be defined by:


A y = x2, where x [0, )
B y = x2, where x R
2
+
D y = x , where x R
E y = x,
x where x [0, )

C y = x2 where x R

9 WE 6 For each of the following functions, fully define the inverse.


a f : [2, ) R, f (x) = (x 2)2 3
b f : R R, f (x) = 3e x 1 + 2

5C

inverse functions
A one-to-one function is a function where for each x-value there is only one y-value and vice
versa.
The graph of a one-to-one function can be crossed only once by any vertical or horizontal
line.
A function that is not one-to-one is many-to-one.

240

Maths Quest 12 Mathematical Methods CAS for the Ti-nspire

A function will have an inverse that is also a function if and only if it is a one-to-one function.

If a function, f , is one-to-one, then its inverse function is denoted by f 1.


Furthermore:

dom f 1 = ran f
1
ran f = dom f
1
The graph of f is obtained by reflecting the graph of f through the line y = x. (Note that if f

crosses the line y = x at any point, f and f 1 will intersect at that point.)
The maximal domain of a
y
function is the largest domain
y = f(x)
for which its rule is defined. If a
function is given without its domain
specified, then it is understood that
the maximal or implied domain is Horizontal lines
x
0
intended.
cross the curve
only once.

Vertical lines cross the curve only once.


eBook plus

WoRkED ExAMplE 7

1
.
x+2
State whether or not it is a one-to-one function, and hence
determine if the inverse function exists. If the inverse function exists:
on the same set of axes, sketch f(x), its inverse and the line y = x

use a CAS calculator to find the points of intersection of f(x) and f 1(x)
find the maximal domain and range of the original function
find the domain and range of the inverse function.

Tutorial

Consider the function f ( x ) =


a
b
c
d
e

WRiTE/DRAW

Think
a

int-0542
Worked example 7

1
, a basic hyperbola
x+2
translated 2 units to the left.
f ( x) =

Write the function.

Find the vertical asymptote by making the


denominator of the fraction equal to 0.

Vertical asymptote occurs where


x+2=0
x = 2

Find the horizontal asymptote remembering


there is no vertical translation.

Horizontal asymptote occurs where


y = 0.

Sketch the graph of f (x) and use vertical and


horizontal line tests to decide whether or not
f (x) is a one-to-one function.

x = 2

y
(0, 12)

1
f(x) = x + 2
5

State whether or not the inverse function exists.

f (x) is a one-to-one function, so f 1(x)


exists.

Chapter 5

inverse functions

241

b On the same axes sketch f (x), f

b The horizontal asymptote is y = 2.

(x) and y = x.

The vertical asymptote is x = 0.


1
0, 2 becomes 1 , 0

( )

( )
2

The equation is y = 1x 2.
x = 2
1)
(0,
2
1
y =
x+2

y=x

0
1 , 0)
(
2

1 x 2
y=
x y = 2

c The points of intersection are (2.414,

c Find the points of intersection.

2.414)

and (0.414, 0.414). Note that


these points both lie on the line y = x.

d The domain is R\{2} and the range is

d Find the domain and range of f (x).

R \{0}.
e The domain of the inverse is the range of the original

e The domain of the inverse is R \{0} and

the range is R \{2}.

function. The range of the inverse is the domain of the


original function.

Functions are either one-to-one or many-to-one.


If a function, f, is not one-to-one but is many-to-one, it is possible to restrict its domain so that
the limited domain is one-to-one and hence the inverse function exists.
WoRkED ExAMplE 8

State the largest negative domain of f(x), shown in the figure, so that f 1(x) exists.
y
f(x)
(0, 9)

Think

242

Decide whether f (x) is a one-to-one function.

The parabola can be divided through its axis of


symmetry, x = 3, so that 2 one-to-one functions
are formed.

Maths Quest 12 Mathematical Methods CAS for the Ti-nspire

WRiTE/DRAW

f (x) is not one-to-one, because horizontal


lines cut the parabola in 2 places.

State the domain of the left part of the parabola


that is one-to-one.

f (x) is one-to-one if the domain is restricted


to (, 3].

State the domain of the right part of the parabola


that is one-to-one.

f (x) is also one-to-one if the domain is


restricted to [3, ).

State the largest negative domain of f (x) so that

f 1(x) exists.

The largest negative domain for f 1(x) to


exist is (, 3].

REMEMbER

A function f (x) has an inverse function f 1(x) if and only if the function is one-to-one.

The domain of a function f (x) is the range of its inverse f 1(x).

The range of a function f (x) is the domain of its inverse f 1(x).


1
The graph of f (x) is obtained by reflecting f (x) in the line y = x.
The maximal or implied domain is used when the domain of a function is not
specified. It is the largest domain for which the rule can be defined.
6. If a one-to-one function intersects with its inverse, the points of intersection lie on the
line y = x.
7. If a function is not one-to-one it is possible to restrict the domain so that the new
function is one-to-one and hence an inverse function exists.

1.
2.
3.
4.
5.

ExERCiSE

5C

inverse functions
1

WE 7 Consider the following functions.


For each function:
i state whether or not it is a one-to-one function and hence determine if the inverse
function exists.
If the inverse function exists:
ii on the same set of axes, sketch f (x), its inverse and the line y = x

iii use a CAS calculator to find the points of intersection of f (x) and f 1(x)
iv find the maximal domain and range of the original function
v find the domain and range of the inverse function.
a f (x) = 4x + 1
b f (x) = 6x
c f (x) = 5
d f (x) = x2 + 2
e f (x) = (x 3)2
f f (x) = (x + 1)3
2
g f ( x) =
i f (x) = x2 6x + 3
h f ( x ) = 16 x 2
x
4x 2
k f (x) = 2 loge (x 1)
j f (x) = e

2 Find: i which of the functions below

has an inverse function f 1(x)


1
ii f (x), if it exists.

ExAM Tip
When asked for the domain of an

inverse, simply stating that the domain of f 1 is the


same as the range of f was not sufficient to gain the
mark. Students were expected to explicitly state the
domain as a set.
[Assessment report 2006]

a f (x) = 4x

b f ( x) =

1
2
x2

c f (x) = 5 x2

d f (x) = (x 1)2

e f ( x) =

x3
2

f f (x) = (x + 5)2 28

Chapter 5

inverse functions

243

g f ( x) = x 2
j f (x) = 5 ex 2

h f ( x ) = 16 x 2
k f (x) = 3ex 2

i f (x) = 2ex + 1
l f (x) = loge (3x)

m f (x) = 2 loge (x 4)

n f (x) = 1 + 2 ln (x)

o f (x) = 3 loge (2x + 3)

3 Copy and complete the following table.


Function

Inverse of the function

Domain

Range

Domain

Range

[1, )

[3,

[0, 3]
R+

[10, )

3]

(0, )

g
h

[5, 5]

[0, 8]
R+

4 MC Consider the function in the figure shown at right.


a The function would be one-to-one if the domain were
restricted to:
A [3, 5]
B [2, 10]
C [1, 3]
D [0, 4]
E [0, 2.2]
b The largest domain that restricts it to a one-to-one function is:
B [0, )
A (, 4]
C R+

E ( , 2]
D R

2
+ 1.
5 MC Consider the function f ( x ) =
( x 3)2
a The function would be one-to-one if the domain were restricted to:
A (3, ]
B [3, )
C (3, )
D (1, )
E [3, )
b The inverse would be a function if the domain of f (x) were:
A [0, 4]
B (1, 4]
C [2, 4]
D [0, 3)
E [1, )
6 MC Consider the function f : S R, f (x) = x2 + 1.

a The maximal domain, S, for f 1(x) to exist is:


+
C [1, )
A R
B R
+

E (, 1]
D R {0} or R {0}

b For f 1 to exist, the largest possible positive set, S, is:


C [0, )
D R+
A R
B R
7 WE8
exists.
a

y
x

f(x)

y
f(x)

(3, 3)

y
f(x)

Maths Quest 12 Mathematical Methods CAS for the Ti-nspire

E [1, )

For each function graphed below, state the largest possible domain of f so that f

244

1 0

y
0

f(x)
2

f(x)

y
9

f(x)

f(x)
0 1
3
7

5D

x=5

9 x
f(x)

9x

f(x)

Restricting functions
As we have seen in the previous exercise, functions which are not one-to-one can have their
domains restricted so that they become one-to-one. As a result their inverses will then be

functions, that is, f 1 will exist.

WoRkED ExAMplE 9

eBook plus

Consider the function fully defined as follows:


f : [ 3, 3] R, f ( x ) = 9 x 2

a Find the largest possible positive domain so that f 1 exists.


1
b Use this restricted domain to fully define f (x).

int-0543
Worked example 9

WRiTE

Think
a

Tutorial

a Let y = f (x)

Write the function.

y = 9 x2
y2 = 9 x2

Square both sides.

Add x2 to both sides.

x2 + y2 = 9

State the shape and key features of the graph.

This is the equation of a circle with


centre (0, 0) and radius 3.

Describe f (x), remembering that a positive square


root implies the upper part of a semicircle.
Sketch the graph of f (x).

The graph of f ( x ) = 9 x 2 is an upper


semicircle.

y
3

3
7

State the possible domains if f (x) is one-to-one.

State the largest positive domain for f 1(x) to


exist.

f (x) is one-to-one if the domain is [3, 0]


or [0, 3].
The largest positive domain for f
exist is [0, 3].

Chapter 5

to

inverse functions

245

b If dom f = [0, 3], then ran f = [0, 3].

Find the range of f (x).


1

= ran f.

dom f

= ran f
= [0, 3]

ran f

= dom f
= [0, 3]

Find the domain of f

Find the range of f

Write the function.

y = 9 x2

Find the inverse by interchanging x and y.

Inverse is x = 9 y 2

Make y the subject.

Select the appropriate inverse function.

State the function f 1.

using dom f

using ran f

= dom f.

x2 = 9 y2
x2 + y2 = 9
y2 = 9 x2
y = 9 x2
Since ran f

= [0, 3], use y = 9 x 2 .

Therefore the inverse of f with the


largest positive domain is:

f 1: [0, 3] R, f 1 ( x ) = 9 x 2

WoRkED ExAMplE 10

If f: S R, f(x) = 3(x 1)2 2:

a find the largest positive set, S, for which f 1(x) exists


1
b fully define f and sketch its graph.
WRiTE/DRAW

Think
a

a y = 3(x 1)2 2

Write the function.

State the shape and key features of the graph.

Sketch the graph of f (x).

A parabola with turning point (1, 2),


y-intercept (0, 1) and x-intercepts
(0.184, 0) and (1.816, 0)
y

(0, 1) (0.184, 0) (1.816, 0)


x
0
(1, 2)

State whether or not it is a one-to-one function.

State the possible domains if f (x) is to be one-to-one.

State the largest positive domain, set S, for f


to exist.
Find the range of f (x).

1
2
3

246

Find the domain of f


Write the function.

using dom f

= ran f.

Maths Quest 12 Mathematical Methods CAS for the Ti-nspire

The parabola is not one-to-one so the


domain must be restricted.
The domain could be (, 1] or [1, ).
S = [1, )
b The range is [2, ).

The domain of f 1 is [2, ).


y = 3(x 1)2 2

x = 3(y 1)2 2
x + 2 = 3(y 1)2
x+2
= ( y 1)2
3

Interchange x and y and make y the subject.

x+2
Reject the negative
3
solution because the domain is [2, )
x+2
y = 1+
3
y 1=

f 1:[2, ) R, f 1 ( x ) = 1 +

Fully define the inverse function.

x+2
3

key feature of inverse functions


A feature of inverse functions is that by taking the inverse function of a function, or vice versa,
the independent variable (x) is always obtained. That is:

f 1 [ f (x)] = f [ f 1(x)] = x.
The following example illustrates how this works.
WoRkED ExAMplE 11

eBook plus

For f : R R, f(x) = x3:

a find f 1(x) and state its range

b find f [ f 1(x)] and f 1[f(x)] and show that f [ f 1(x)] = f 1[ f(x)] = x.


Think
a

1
2
3

int-0544
Worked example 11

WRiTE

Write the function.


Interchange x and y.
Make y the subject by writing both sides to the
power of 13 and simplifying.

a f(x) = y = x3

Its inverse is x = y3.


1

Write the rule for f 1(x).

Find the range of f 1(x), remembering that ran

f 1 is the same as dom f.

Write your answer in the same form as the question.

Find f [ f 1(x)] by rewriting f 1(x) as x 3 .

y = x3

Find f ( x 3 ) by replacing x by x 3 in f(x) = x3.

Simplify.

Find f

Find f

Simplify.

dom f = R

ran f 1 = R
f

f 1 ( x) = x 3

(y3 ) 3 = x 3

Tutorial

: R R, f 1 ( x ) = x 3
1

f [ f 1 ( x )] = f ( x 3 )
1
= x3

=x

f 1[ f ( x )] =

x3.

[f(x)] by rewriting f(x) as

(x3) by replacing x by x3 in f 1 ( x ) = x 3.

f 1 (x3 )
1

= (x3 ) 3
=x

Compare f [ f (x)] to f

[ f(x)].

f [ f 1 (x
( x )] =

f 1[ f ( x)]
x )] = x

Chapter 5

inverse functions

247

WoRkED ExAMplE 12

eBook plus

a Sketch the graph of f(x) = x2 3x + 3, showing the turning point and relevant
Tutorial
intercept(s).
int-0545
b Find the rule of the inverse by an algebraic method and sketch this graph
Worked example 12
on the same set of axes together with the line y = x.
c Is the inverse a function?
d The inverse is a reection in the line y = x of the original function f(x). Use this information to find
any points of intersection between the original curve and its inverse.

e Find the maximum value of a for f: (, a] R, f(x) = x2 3x + 3 so that f 1(x) exists.


WRiTE/DRAW

Think
a

use a CAS calculator to help in drawing a


graph of f (x) including the relevant points.

y
4
3
2
1

f(x) = x2 3x + 3

(1.5, 0.75)

0
b

248

1 2 3 4

b x = y2 3y + 3

To find the equation of f 1(x), let


y = x2 3x + 3. Interchange
x and y.

To make y the subject, on a


Calculator page, press:
MeNu b
3:Algebra3
1:Solve1
Complete the entry line as:
solve(y2 3y + 3 = x,y)
Then press eNTeR .

Write the answer in the form of f 1(x).

use the calculator to draw the graphs

of f(x), f 1(x) and y = x.

The inverse is a one-to-many relation and


therefore is not a function.

f 1 ( x) =

4x 3 + 3
2

y
4
3
2
1

f(x) = x2 3x + 3
y=x

1 2 3 4 5 6 7

c The graph of the inverse does not pass

the vertical line test, as it is a one-tomany relation, and therefore it is not a


function.

Maths Quest 12 Mathematical Methods CAS for the Ti-nspire

The points of intersection between f (x) and

f 1(x) will occur at the intersections of the


graph of f (x) and y = x.
To locate these points on a Calculator page,
press:
MeNu b
3:Algebra3
1:Solve1
Complete the entry line as:
solve(x2 3x + 3 = x, x)
Then press eNTeR .
The points of intersection are at (1, 1) and (3, 3).
Draw the graphs of f(x) and y = x.

For the left-hand branch of the parabola, we


need f (x) = x2 3x + 3, since the domain is
(, a). It is a one-to-one function.
Locate the turning point of the graph and
the x-coordinate will be the value of a.
To locate the minimum of f (x) = x2 3x + 3
on a Calculator page, press:
Menub
4:Calculus4
7:FunctionMinimum7
Complete the entry line as:
f Min(x2 3x + 3, x)
Then press eNTeR .
3
The x-value of the turning point is 2 , which
3
implies that a = 2 .

f(x) = x2 3x + 3

y
4
3
2
1

y=x

(1.5, 0.75)

1 2 3 4

The points of intersection are at (1, 1) and (3, 3).

f : ( , 2 ) R, f ( x ) = x 2 3 x + 3
a=

3
2

REMEMbER

1. Functions that are not one-to-one can have the domain restricted so that they become
one-to-one. Then their inverses will be functions.
2. A fully defined function is written in the form f : [domain] R, f (x) = rule.

3. The graph of f 1(x) is a reflection of f (x) in the line y = x.

4. The points of intersection of f (x) and f 1(x) lie on the line y = x.


5. By taking the inverse function of a function or vice versa, the independent variable (x)

is always obtained: f 1[f (x)] = f [f 1(x)] = x.


ExERCiSE

5D
eBook plus
Digital doc

SkillSHEET 5.3
Function
notation

Restricting functions
1 WE 9a Find the largest possible domain(s) of the following functions so that the inverse
function exists. (use a CAS calculator to sketch the graph if necessary.)
a f : R R, f (x) = x2 + 3
b f : R R, f (x) = 3x2 1
1
c f : R R, f (x) = (x + 3)2 2
d f : R \{0} R, f ( x ) = 2 3
x
1
1
f f : R \ {2} R, f ( x ) =
e f : R \{4} R, f ( x ) =
+1
2
x2
( x 4)

Chapter 5

inverse functions

249

g f : [ 5, 5] R
R,, f ( x)
x) =
i

25 x 2

R,, f ( x)
x) = 1 x 2
h f : [ 1, 1] R
j f : R R, f (x) = x2 2x + 5

f : [4, ) R
R,, f ( x)
x) = x 4

k f : R R, f (x) = e x + 2

l f : (5, ) R, f (x) = 2 loge (x 5)

2 WE 9b For each function in question 1 use the restricted domain to fully define f 1(x).
(If there are two possible domains, use the one which is to the right.)
3 MC use the function f : [0, ) R, f (x) = x2 to answer the following questions.

a Any points of intersection of f (x) and f 1(x) must lie on the line:
A y=0
B y=x
C y = 2x
D y=x+1
E x=0

b f 1(x) is correctly defined by:

A f 1 : [0, ) R, f 1(x) = x
B f 1 : [0, ) R, f 1(x) = x

R, f 1 ( x)
x = x
D f 1 : [0, ) R, f 1 ( x ) = x
C f 1 : [0, ) R,
1

R, f 1 ( x)
x = 2
E f 1 : (0, ] R,
x

c The range of f 1(x) is:


, 0]
D [0, )
A (
B R
C R+
E R
1
d The graphs of f and f intersect at the point(s):
A (0, 0) and (1, 1)
B (1, 1) only
C (0, 0) and (1, 1)
1
D (0, 0) and (3, 3)
E (1, 1) and (2, 4 )
4 Copy each of the following graphs, then on the same set of axes sketch the graphs of y = x and

y = f 1(x).
b
a
y
y
f(x)

3
5

f(x)

f(x)

y
3
f(x)

y
1

250

3
y

f(x)
x

f(x)

Maths Quest 12 Mathematical Methods CAS for the Ti-nspire

(2, 5)

y
2

4
f(x)

f(x)
4

5 WE 10 If f : S R, f ( x ) = 3 + x 1, find:

a the largest set, S, for which f 1(x) exists


1
b f and sketch its graph.
6 If f : [2, a] R, f (x) = (x 4)2 5, find:
a the largest possible value of a so that f will have an inverse which is a function

b f 1 and sketch its graph.


1
7 WE 11 For f : R+ R, f ( x ) =
x

a find f 1 and state its range


1
b find f [ f (x)]

c find f 1[ f (x)]

d show that f [ f 1(x)] = f 1[ f (x)] = x.


1
:
8
For f : R R, f ( x ) = 2
x +2
a sketch the graph of f (x)

b state the largest positive domain for f (x) so that f 1(x) exists
1
c sketch the graph f (x) on the same set of axes as f (x), using the positive domain

d show that f [ f 1(x)] = f 1[ f (x)] = x.


9 WE 12 If g : [b, 8] R, g(x) = 9 x2, find:
a the smallest value of b so that g(x) has an inverse that is a function

b g1

c the range of g 1(x).


10 Given that g( x ) = 3 + 4 x 2 , fully define two inverse functions,

g 1, using maximal domains.

Chapter 5

eBook plus
Digital doc

WorkSHEET 5.2

inverse functions

251

Summary
Functions and their inverses

A function is a relation which has only one y-value for each x-value. The graph of a function can be crossed
only once by a vertical line.
A one-to-one function is a function which has only one x-value for each y-value. The graph of a one-to-one
function can be crossed only once by any vertical or horizontal line.
A function which is not one-to-one is many-to-one.
The rule for the inverse of a relation, or a function, can be obtained by either:
1. interchanging the first and second elements of the ordered pairs, or
2. interchanging x and y in the rule and making y the subject.
Graphing the inverse The graph of the inverse of a relation, or a function, can
y
be obtained by either:
1(x)
f
1. interchanging the first and second elements of the ordered pairs of the
relation or function, or
2
(5, 2)
2. reflecting the graph of the relation or function through the line y = x, or
x
0
3. using the rule of the relation or function to find the rule of the inverse and
5
f(x)
then graphing the inverse.
y=x
The domain and range of a function and its inverse are interchanged.
(2, 5)
The graphs of a function and its inverse intersect on the line y = x.
Inverse functions

The inverse of a one-to-one function, f(x), is also a function and is denoted by f 1(x).

dom f 1 = ran f
1
ran f = dom f

f 1[f(x)] = f[f 1(x)] = x


The implied (or maximal) domain of a function is the largest domain for which the function has meaning.
When the domain is not mentioned, use the maximal domain.
Restricting functions

A function, f(x), which is not one-to-one can have its domain restricted so that f 1(x) exists.

The points of intersection of f(x) and f 1(x) lie on the line y = x.


A function is fully described using the notation
f: X Y, f(x) = rule
where X is the domain and Y is the co-domain.

The graph of f 1(x) is a reflection of f(x) in the line y = x.


1

f [f(x)] = f[f 1(x)] = x

252

Maths Quest 12 Mathematical Methods CAS for the TI-Nspire

ChApTER REViEW
c

ShoRT AnSWER

1 Sketch the graph of the inverse of each of the


following relations.
y
a

8 x

f(x)
(4, 4)
x

y
(4, 4)

2
6

2 Sketch the graph of each of the following functions


and state its domain and range.
a y = 36 x 2

b f (x) = 2e x 2

3 using the graph of the functions given below


sketch the graph of the inverse of each function.
y
a
x = 2
1
2

1 0

f(x)

y
f(x)

(1, 4)

4 State the largest possible domain for each of the


following functions to be one-to-one.
a f (x) = 3 loge (x + 4)
b g(x) = (x 2)2
1
c f ( x) = 5
x2
5 For the following functions:
i Sketch their graphs and, by inspection, define
them as one-to-one or many-to-one functions.
ii For the functions defined in i as many-to-one,
define the maximal domain for which the
inverse is a function, stating their domains
in formal function notation. If there is more
than one option, choose the right-hand
option.
iii Find the rules of their inverses.
iv For all of the one-to-one functions from i and
the redefined functions from iii, sketch all
these inverse functions.
a f (x) = 2x 1
b f (x) = 2(x 1)3 + 1
c f (x) = x2 + x
d f ( x) = 2 x 3
e f (x) = 3ex + 1
2
1
f f ( x) =
3( x 2)

1
+1
g f ( x) =
( x + 3)2
6 a Sketch the graph of f (x) = loge (x 2) + 1,
marking the intercepts and asymptotes and
stating the domain and the range.
b Find the rule for the inverse function.

c If f 1(m) = 3, find the value of m.

d Draw the graphs of y = x and f 1(x) on the


same axes as f (x) = loge (x 2) + 1, marking
the point (m, 3) and checking that it is on

f 1(x). State the domain and the range.


e Mark the image of (m, 3) on the original graph.

Chapter 5

inverse functions

253

7 If g : (, b) R, g(x) = x2 + 5x 1, find the

largest value of b for g 1 to exist.

8 Part of the graph of the function f : R R,

f (x) = a + be x is shown below.

y
y=1
0

3 The graph of the function with the equation y = f (x)


is shown below.
y
1

The graph of the inverse function f


be:
y

2,
4, 2
2, 4
0, 0

is most likely to
y
1
0

1 0

1 A relation has an x-intercept of 4 and y-intercept


of 2. The inverse of this relation has an x-intercept
and y-intercept, respectively, of:
4,

0
1

MulTiplE ChoiCE

[ VCAA 2007]

[ VCAA 2008]

The line with the equation y = 1 is an asymptote


and the graph passes through the origin.
a explain why a = 1 and b = 1.
Let h be the function h: [0, 2] R, h(x) = f (x).
b State the range of h using exact values.

c Find the inverse function h 1.


d Sketch and label the graph of the inverse

function h 1 on the graph axes shown


above.
9 Let f : R R, f (x) = e2x 1.
a Find the rule and domain of the inverse

function f 1.

b Sketch the graph of y = f (f 1(x)) for its


maximal domain.
ax

c Find f (f 1(2x)) in the form


where a, b
bx
+c
and c are real constants.

A
B
C
D
E

1 0

2 The graph that best represents the inverse of the


relation shown in the figure below is:
E

[ VCAA 2005]

254

Maths Quest 12 Mathematical Methods CAS for the Ti-nspire

4 The relation x2 + (y + 1)2 = 16 has an inverse that is:


A a one-to-one function
B a many-to-one relation
C a many-to-many relation
D a one-to-many function
E a one-to-one relation

-intercept of the inverse function f 1(x),


5 The x-intercept
where f ( x ) = 3 x 3 , is closest to:
A 1.27
B 6
C 3
D 1.27
E 6
6 The y-intercept of the inverse function f
2
+ 3, is closest to:
where f ( x ) =
( x 2)
4
A
3
B no x-intercept
4
C
3
D 2
E 2

1
(x),

7 Asymptotes exist on the graph of the inverse


2

+ 3, at:
function f 1(x), where f ( x ) =
( x 2)
A x = 1 only
B y = 1 only
C x = 3, y = 2
D x = 2, y = 1
E x = 3, y = 2
4
8 The inverse of the relation y = 2 is:
x
2
2
B y=
A y=
x
x
2
2
D y=
C y=
x
x
2
E y=
x
9 A function has a domain of [0, ) and a range of
R+. The domain of its inverse must be:
A R+
B R
D [0, )
C R+
E (, 0]
1
10 The implied domain of the function, f ( x ) = 2
x
1
is:
A R
C R \{0}
E R+

B R \{1}
D R \{1, 1}

Questions 11 to 13 can be answered by


considering the function
f (x) = 4 ln (x + 3) 2.
11 The maximal domain for f (x) is:
A [3, )
B (, 3)

D (3, )
C R \{ 3}
E R+

12 dom f 1 is equal to:


A R+
C R
E (3, )
13 The rule for f

B [0, )
D (, 3)

is completely described by:

A f 1 : R R, f 1 ( x ) = e 4 3
1
1
B f : R R, f ( x ) = e

x+2
4

R, f 1 ( x)
x) = e
C f : ( , 3) R,

1
1
D f : R R, f ( x ) = e

f 1 : R + R, f 1 ( x ) =

x+4
3

x+4
2

x4
e 2

14 At which one of the following points is it possible


for a function and its inverse to intersect?
A (2, 2)
B (1, 2)
C (2, 1)
D (0, 1)

E ( 1, 2)

15 The value of f [ f 1 ( 13 )] is equal to:


A 3
1
B 3
C x
1
D 3
E 3

16 The function shown in the figure is one-to-one if


the domain is restricted to:
A
B
C
D
E

[2, )
[3, )
[3, 2]
(, 2]
[0, )

f(x)
3

17 If f : S R, f (x) = x2 10x + 18, then the largest

possible set, S, for f 1 to exist is:


A
B
C
D
E

(, 0]
[0, )
[0, 5]
[5, 100]
(, 5]

Chapter 5

inverse functions

255

ExTEnDED RESponSE

1 using a CAS calculator:


a Find the rule for the inverse of the function f (x) = 0.213x2 + 1.127x 2.124.
b explain why this inverse is not a function.
c Find the value of a, correct to 2 decimal points, for [a, ), the maximal domain of f (x) so that the
inverse is a function. Let this new function be g(x).

d Find the domain and range of f 1(x).


1
e Solve g(x) = g (x) for x, correct to 2 decimal places.
2 The function, f (x), in the figure is a parabola with a turning point at (0, 4).
a Find the function which describes f (x).
b Find the coordinates of the point, A.
c State the domain and range of f (x).
d Sketch the graph of f (x) and its inverse on the same set of axes.
e What shape is enclosed between f (x) and its inverse?
f i State the domain for which the inverse of f (x) is a function.
ii Give the rule for the inverse of f (x) over this domain.
g i State the domain for which the inverse of f (x) is not a function.
ii Give the rule for the inverse of f (x) over this domain.
3 The graph of f (x) = exx2 is shown at right.
a use a calculator to find the turning points, rounding answers to
2 decimal places as appropriate.
b State the domain and range of f (x).
c On the same set of axes, sketch f (x), its inverse and the line y = x.
Mark all turning points and asymptotes.
d State the domain and range of the inverse of f (x).
e use a CAS calculator to solve exx2 = x to find the points of
intersection of f (x) and its inverse.

f Find the largest possible negative domain of f (x) so that f 1(x)


exists (that is, the inverse is a function).
g Find the domain of the inverse function.

y
4

y=x

A
2

f(x) = exx2
x

4 a use a CAS calculator to draw the graph of f (x) = 6 loge |x 3|.

b Find the value of a for (a, ), the maximal domain of f (x), such that f 1(x) exists. Let this new function
be g(x).

c Solve g(x) = g 1(x) for x, correct to 2 decimal places.

d Consider the function f (x) = a loge (x 3), where a > 0. Let f 1(x) = h(x). Given that
x
a
1 a
f ( x ) =
and h ( x ) = e , find the value of a (correct to 2 decimal places) for which the graph of
a
x3
f (x) and its inverse intersect only once.

eBook plus
Digital doc

Test Yourself
Chapter 5

256

Maths Quest 12 Mathematical Methods CAS for the Ti-nspire

eBook plus

ACTiViTiES

Chapter opener
Digital doc

10 Quick Questions: Warm up with ten quick


questions on inverse functions. (page 229)
5A

Relations and their inverses

Interactivity int-0248:

Inverses: use the interactivity to consolidate


your understanding of graphs of the inverses of
relations. (page 229)
Tutorial

WE 3 int-0540: Watch a worked example on how to


sketch relations and their inverses. (page 231)
Digital docs

SkillSHeeT 5.1: Practise identifying domain and


range. (page 233)
Spreadsheet 041: Investigate graphs of functions.
(page 233)

5D

Restricting functions

Tutorials

WE 5 int-0541: Watch a worked example on


sketching functions and their relations. (page 236)

WE 9 int-0543: Watch a worked example on defining


inverse functions. (page 245)
WE 11 int-0544: Watch a worked example
on identifying that the composite functions

f [ f 1(x)] = f 1[ f (x)] = x. (page 247)


WE 12 int-0545: Watch a worked example on
restricting a function to define an inverse function.
(page 248)

Digital docs

Digital docs

5B

Functions and their inverses

Tutorial

SkillSHeeT 5.2: Practise making graphs with


equations. (page 239)
Spreadsheet 059: Investigate inverse graphs.
(page 239)
WorkSHeeT 5.1: Sketch relations, functions and
their inverses. (page 240)
5C
Tutorial

Inverse functions

WE 7 int-0542: Watch a worked example on


sketching graphs of inverse functions. (page 241)

SkillSHeeT 5.3: Practise using function notation.


(page 249)
WorkSHeeT 5.2: Recognise types of functions and
sketch polynomials and power functions. (page 251)
Chapter review
Digital doc

Test Yourself: Take the end-of-chapter test to test


your progress. (page 256)
To access eBookPLUS activities, log on to
www.jacplus.com.au

Chapter 5

inverse functions

257

6
Circular
(trigonometric)
functions
areaS oF STudy

Graphs and identification of key features of


graphs of the circular functions:
y = sin (x), y = cos (x), y = tan (x)
Transformation from y = f (x) to
y = Af (n(x + b)) + c, where A, n, b and c R,
and f is one of the circular functions specified
above and the relation between the graphs of the
original and transformed functions for functions
such as: y = 10 sin ((x c)) 5, c R
Graphs of sum, difference, product and composite
functions of f and g, where f and g are functions
such as y = sin ((x) + 2x
2x and y = |cos
cos (2
(2x)|
Graphical and numerical solutions of
trigonometric equations

6a
6b
6c
6D
6E
6F

Revision of radians and the unit circle


Symmetry and exact values
Trigonometric equations
Trigonometric graphs
Graphs of the tangent function
Finding equations of trigonometric
graphs
6G Trigonometric modelling
6H Further graphs
6I Trigonometric functions with an
increasing trend

Recognition of the general form of possible


models for data presented in graphical or
tabular form, using circular functions
General solutions of equations such as
1
cos ( x ) + cos
cos (3 x ) = 2 , x R and the specification
of exact solutions or numerical solutions, as
appropriate, within a restricted domain
Applications of simple combinations of
functions and interpretation of features of the
graphs of these functions in modelling practical
situations, for example, y = ax + b + m sin (nx)
as a possible pattern for economic growth
cycles

eBook plus
Digital doc

nit
c

nit

revision of basic concepts

1u

revision of radians and


the unit circle

1u

6a

10 Quick Questions

Angles are measured in degrees or radians.


O
S x
To define a radian we can use a circle which has a radius of
one unit. This circle is called the unit circle. If we take a piece of
string which is the same length as the radius and place it along
the circumference of the circle from S to P to form an arc, then
A unit circle
the angle formed by joining S and P to O, the centre of the circle,
measures one radian.
The radius of the circle can be any length and can still be regarded as a unit. As long as the
arc is the same length as the radius, the angle will always measure one radian.
In general, therefore, a radian is the angle formed at the centre of any circle by radii meeting
an arc which is the same length as the radius of the circle. Note the following.

258

maths Quest 12 mathematical methods CaS for the Ti-nspire

y
1. One radian is written as 1c (or 1 radian can be written as 1).
A
2. The circumference of a circle is 2r units in length.
P
3. If the radius is one unit, as in the case of the unit circle, then the
r
r
circumference is 2 units, and the angle at the centre of the circle
c
is 2 radians.
1
r
4. 2 radians = 360
B
S x
5. The length of the semicircle from S through A to B is half the
circumference and is units.
6. radians = 180
7. An arc length of r units subtends an angle of 1 radian.
A radian
8. An arc length of 2r units subtends an angle of 2 radians.
r

2 r
9. An arc length of a quarter of a circle is
units that is,
and subtends an angle of

2
4

radians.
2

Finding the number of degrees in one radian


c = 180
180
1c =
= 57.296 (correct to 3 decimal places)

Since
we have

Converting radians to degrees


Radians are converted to degrees using the following equation.
180
1c =

Worked Example 1

Convert the following to degrees, giving the answer correct to 2 decimal places.
a 2c b 6.3c c 9

10

Think
a

1
2

Write

180
Multiply the number of radians by
.

Simplify where possible.

180

360
=

= 114.59, correct to 2 decimal places.

a 2c = 2

Write the answer correct to 2 decimal


places.

b
On a Calculator page, type:
6.3
Note: The radian sign does not need to be
entered if the calculator is already
in radian mode.
To convert 6.3c to degrees, press Catalog
k, press D to get to the items beginning
with the letter D.
Then select DD.
Then press ENTER .

Chapter 6 Circular (trigonometric) functions

259

Then press ENTER .


Then write the answer.

6.3c = 360.96 correct to 2 decimal


places.
c

Multiply the number of radians


180
by
.

Simplify by cancelling.

c
c 9 = 9 180

10

10

= 162

Converting degrees to radians


Degrees are converted to radians using the following equation.
180 = c
1 =
Worked Example 2

Convert the following to radians.


a 2 b 36.35 c 150
Think
a

260

Write

On a Calculator page, type:


2
The calculator is in radian mode
so you will need to type the degree
symbol before converting to radians.
You will find the degree symbol in
the Maths expression template by
pressing Ctrl / Catalog k.

Maths Quest 12 Mathematical Methods CAS for the TI-Nspire

c
180

Complete the entry line as:


2
Then press ENTER .
Then write the answer.

or 0.035c correct to 3 decimal places.


90

b 36.35 = 36.35
180
2 =

Multiply the number of degrees by

.
180

Simplify.

= 0.634c

Note: In this example it is not appropriate to


leave your answer in exact form.
c

Multiply the number of degrees by

.
180

Simplify, leaving your answer in


exact form.

150 = 150

180

150
180
5
=
6
=

Special cases (degrees to radians)


Note the following special cases.
180 =

90 =
2

60 =
3

45 =
4

30 =
6

Divide both sides by 2.


Divide both sides by 3.
Divide both sides by 4.
Divide both sides by 6.

Basic definitions of sine, cosine and tangent


Sine and cosine
In the unit circle the vertical distance PR is defined as
sine () or sin () and the horizontal distance OR is
defined as cosine () or cos ().
The coordinates of the point P are (cos (), sin ())
where can be in radians or degrees.
The x-coordinate of P is cos () and the y-coordinate of
P is sin ().

y
P (cos ( ), sin ( ))
sin ()

x
O cos () R

sin (q) and cos (q)

Chapter 6 Circular (trigonometric) functions

261

Special cases (sin, cos)


Note the following special cases for sin and cos.
sin (0) = 0

sin = 1
2
sin () = 0

3
sin = 1
2

cos (0) = 1

cos = 0
2
cos () = 1

sin (2 )
cos ( )
cos (0) 0

sin (2) = 0
sin (0) = 0
sin (90) = 1
sin (180) = 0
sin (270) = 1
sin (360) = 0

3
)
sin (
2

3
cos = 0
2

cos (2) = 1
cos (0) = 1
cos (90) = 0
cos (180) = 1
cos (270) = 0
cos (360) = 1

Special cases

Tangent

Using the unit circle, the vertical distance TS is defined as tan .


TS is the tangent to the circle which intersects with the x-axis and
TOS = .
Using Pythagoras theorem in triangle OPR (figure below),
PR2 + OR2 = OP2.
2
So,
sin () + cos2 () = 1
From the diagram below, OPR is similar to OTS (angle,
angle, angle).

Special cases (tan)

it

P
un

sin ( )
tan ( ) =
cos ( )

tan (q)

TS PR
Therefore: =
OS OR
tan ( ) sin ( )
=
1
cos ( )

tan ( )

tan ( )
sin ( )

cos ( ) R
1 unit
Identities

S x

Note the following special cases for tan.


tan (0) = 0

tan is undefined
2

tan (0) = 0

tan (90) is undefined

tan () = 0

3
tan is undefined
2

tan (2) = 0

tan (180) = 0

tan (270) is undefined tan (360) = 0


sin (90o ) 1
= , which is undefined.
It can be seen that tan (90) is undefined because tan (90o ) =
cos (90o ) 0
REMEMBER

1. Radians:
(a) 1c = the size of the angle formed where the length of an arc is equal to the radius of
the circle.
(b) c = 180
(c) Angles are in radians unless a degree symbol is shown.

262

Maths Quest 12 Mathematical Methods CAS for the TI-Nspire

180
.

3. To change degrees to radians, multiply by


.
180
4. Identities:

2. To change radians to degrees, multiply by

(b) tan ( ) =

(a) sin2 ( ) + cos2 ( ) = 1

exerCiSe

6a
eBook plus
Digital doc

Spreadsheet 142
Unit circle

eBook plus

revision of radians and the unit circle


1 We1 Convert the following to degrees, giving answers correct to 2 decimal places.
a 3c
7 c
e
20

b 5c
c 4.8c
d 2.56c
5 c
5 c
3 c
h
f
g
4
6
10
2 We2 Convert the following to radians. Give exact answers for a , b , c and d . Write other
answers correct to 2 decimal places.

Digital doc

SkillSHEET 6.1
Changing
degrees to
radians

sin ( )
cos ( )

a 5

b 15

c 120

d 130

g 235
h 260
i 310
f 78.82
3 Evaluate using a calculator. Give answers correct to 3 decimal places.

e 63.9
j 350

a sin (0.4)

b sin (0.8)

c cos (1.4)

d cos (1.7)

e tan (2.9)

f tan (2.4)

g sin (75)

h sin (68)

i cos (160)

j cos (185)

k tan (265)

l tan (240)

Evaluate the following.


a sin (0)
3
e tan
2

b sin ()

f tan
2

c cos (2)

d cos ()

g sin (90)

h sin (360)

i cos (180)

j cos (0)

k tan (270)

l tan (240)

Evaluate without using your calculator.


a sin2 (20) + cos2 (20)

b cos2 (50) + sin2 (50)

e sin 2 + cos 2
2
2

d sin2 (2.5) + cos2 (2.5)


g 2 sin2 () + 2 cos2 ()

c sin2 () + cos2 ()

f sin 2 + cos 2
2
2

h 5 sin2 () + 5 cos2 ()

6 Write the following in order from smallest to largest.


a sin (35), sin (70), sin (120), sin (150), sin (240)
b cos (0.2), cos (1.5), cos (3.34), cos (5.3), cos (6.3)
8

15

7 If sin ( ) = 17 and cos ( ) = 17 , find tan ().


8 If sin (A) = 0.6 and cos (A) = 0.8, find tan (A). Draw a triangle marking in
the position of angle A and possible lengths of the sides.

radians is equal to:


3
a 0
b 30
c 45

eBook plus
Digital doc

9 mC

SkillSHEET 6.2
Tangent ratios

D 60

E 90

Chapter 6

Circular (trigonometric) functions

263

10 MC The expression 1 sin2 () is equal to:


c cos ()
D tan ()
a 1
b cos2 ()

E tan2 ()

11 The temperature T C inside a shop t hours after 2 am is given by



T = 15 3 cos t
12
Calculate the exact temperature after 4 hours and the temperature to the nearest tenth of a
degree at 9.00 am.

6B

Symmetry and exact values


Exact values
Using the equilateral triangle (of side length 2 units) shown at
right, the following exact values can be found.

sin (30) = sin =


6

1
2

cos (30) = cos =


6

tan (30) = tan =


6

3
2
1

3
3

30
2

sin (60) = sin =


3

3
2

cos (60) = cos =


3

1
2

tan (60) = tan = 3


3

60
1
1
Exact values of sine,
cosine and tangent of
30 and 60

Using the right isosceles triangle shown, the following exact values
can be found.

sin (45) = sin =


4

1
2

cos (45) = cos =


4

1
2

2
2

45
1

45
1
Exact values of sine,
cosine and tangent
of 45

2
2

tan (45) = tan = 1


4

The unit circle and symmetry properties


The unit circle is symmetrical so that the magnitude of sine, cosine and tangent at the angles
shown are the same in each quadrant but the sign varies.
y

Symmetrical properties

In the first quadrant sin, cos and tan are all positive.
In the second quadrant only sin is positive.

264

Maths Quest 12 Mathematical Methods CAS for the TI-Nspire

In the third quadrant only tan is positive.


In the fourth quadrant only cos is positive.
This can be remembered as All Students To Class (ASTC).
1st quadrant

2nd quadrant

3rd quadrant

4th quadrant
sin

sin (2 ) = sin ()

sin ()

sin ( ) = sin ()

sin ( + ) =

cos ()

cos ( ) = cos ()

cos ( + ) = cos ()

cos (2 ) = cos ()

tan ( + ) = tan ()

tan (2 ) = tan ()

tan ()

tan ( ) =

tan

()

()

Worked example 3

eBook plus

Without using a calculator, find the value of:


a sin (150)
Think
a

Tutorial

5
b cos
.
4

int-0546
Worked example 3

WriTe
a sin (150) = sin (180 30)

Find the equivalent first quadrant angle.

As 150 is in the 2nd quadrant, sine is positive.

= sin (30)

Write the exact value.

Find the equivalent first quadrant angle using


5 4
=
+ .
4
4 4

Decide on the sign required. As it is in the 3rd


quadrant, cosine is negative.

= cos
4

Write the exact value.

b cos

1
2

= cos +

4
4

2
2

Angles are not restricted to values between 0 and 2; that is, the domain is not restricted to
[0, 2]. If an angle is greater than 2 radians, it is necessary to subtract multiples of 2 so that
the angle is within one turn of the unit circle. Each 2 radians is a complete turn of the circle.
Worked example 4

If sin (x) = 0.6, cos (x) = 0.8, and x is in the first quadrant, find:
a sin (3 x)
b cos (4 + x).
Think
a

WriTe

Write 3 as 2+ that is, one complete


cycle and then the angle x.

a sin (3 x) = sin (2 + x)

= sin ( x)

= sin (x)

As it is in the 2nd quadrant, sine is positive.

Chapter 6

Circular (trigonometric) functions

265

= 0.6

Substitute the given value.

4 = 2 2 that is, two complete cycles and


then the angle x.

b cos (4 + x)

y
x

As it is in the 1st quadrant, cosine is positive.

= cos (x)

Substitute the given value.

= 0.8

Worked example 5

eBook plus

< < calculate cos ( ) and hence find tan ( ).


2

If sin ( ) = 12
and
13
Think

Tutorial

int-0547
Worked example 5

WriTe

Method 1
1

2
3
4
5

In the given triangle, find the length of the third


132 = 52 + 122
side by using Pythagoras theorem. Let be the
first quadrant angle corresponding to .
adjacent
5
cos ( ) =
cos ( ) = 13
hypotenuse

is in the second quadrant, therefore cos ( )


is negative.
opposite
tan ( ) =
adjacent
is in the second quadrant, therefore tan ( )
is negative.

cos ( ) =
tan ( ) =
tan ( ) =

13
'

5
13
12
5
12
5

Method 2
1

Use the rule sin2 ( ) + cos2 ( ) = 1.

Rearrange.
12
13

cos2 ( ) = 1 sin2 ( )
for sin ( ).

Substitute

Evaluate.

Take the square root of both sides,


remembering that the answer could be
positive or negative.

cos is negative in the 2nd quadrant.

Use tan ( ) =
tan ( ).

sin2 ( ) + cos2 ( ) = 1
144

= 1 169
25

= 169

sin ( )
to find the value of
cos ( )

cos ( ) = 13
5

Take cos ( ) =
tan ( ) =
tan ( ) =

266

maths Quest 12 mathematical methods CaS for the Ti-nspire

5
13
12
13
5
13
12
5

as

< <
2

12
5

Method 3
1

On a Calculator page, press:


MENU b
3:Algebra 3
1:Solve 1
Complete the entry line as:
12

solve (sin ( ) = , ) | < <


13
2
Then press ENTER .

Complete the entry lines as:

12
cos = sin 1
13

12
tan = sin 1
13

Press ENTER after each entry.


Write the answers.

Given sin ( ) = 12
,
13
cos ( ) =
tan ( ) =

< < ,
2

5
13
12
5

Negative angles
For negative angles, move in a clockwise direction.
In the diagram,RQ = PR, and
OR = OR, so
T1S = TS
An alternative way to find tan () is:
tan


( )
()

( )

sin

sin
( )
=
=
= tan ( )
cos ( ) cos ( )

y
P

sin ( )

R S
sin ( )
Q

sin

sin
=
()
cos () = cos ()
tan () = tan ()

The diagram shows 0 < <

T1

Negative angles

; however, these relationships are true for all values of .


2

sin (150) = sin (150) cos (190) = cos (190) tan (280) = tan (280)

Chapter 6 Circular (trigonometric) functions

267

Worked Example 6

Find the exact value of:


a sin (135) b cos (240) c tan (330).
Think
a

Write

sin (135) = sin (135)

135 is in the 2nd quadrant.

= sin (180 45)

Sine is positive in the 2nd


quadrant.

= sin (45)

Give the exact value.

cos (240) = cos (240)

240 is in the 3rd quadrant.

= cos (180 + 60)

Cosine is negative in the 3rd


quadrant.

= cos (60)

Give the exact value.

tan (330) = tan (30)

Give the exact value.

268

1
2

c tan (330) = tan (30)

3
3

a sin
=
3

4
is in the 3rd quadrant.
3
Sine is negative in the 3rd
quadrant.

Maths Quest 12 Mathematical Methods CAS for the TI-Nspire

T
330

Write

sin () = sin ()

240

Method 1: Technology-free
1

135

b cos (240) = cos (240)

Find the exact value of:


4
5
a sin
b tan
.

3
6

135

2
2

Worked Example 7

Think

a sin (135) = sin (135)

sin

4
3

= sin +

= sin
3

= sin
3

S x

Give the exact value.

tan () = tan ()

5
is in the 2nd quadrant.
6

Tangent is negative in the 2nd


quadrant.

Give the exact value.

=
5

b tan
6 =

3
2

5
6

tan

= tan

= tan
6

= tan
6
=

3
3

4
sin
=
3

3
2

5
tan
=
6

3
3

Method 2: Technology-enabled
1

On a Calculator page, complete the entry


lines as:
4
a sin
3
b

5
tan
6
Press ENTER after each entry.

Write the answers.

Complementary angles

Complementary angles add to 90 or radians. Therefore, 30 and 60 are complementary angles.


2

In other words and are complementary angles, and and are also complementary

2
6
3
angles.

The sine of an angle is equal to the cosine of its complement. Therefore, sin (60) = cos (30).
We say that sine and cosine are complementary functions.
The complement of the tangent of an angle is the cotangent or cot that is, tangent and
cotangent are complementary functions (as well as reciprocal functions).
1
cot ( ) =
tan ( )

Chapter 6 Circular (trigonometric) functions

269

1st quadrant

2nd quadrant

3rd quadrant

sin = cos ( ) sin + = cos ( )

2
2

4th quadrant

3
3

sin
= cos ( ) sin
+ = cos ( )
2

3
3

cos = sin ( ) cos + = sin ( ) cos


= sin ( ) cos
+ = sin ( )
2

2
2
3

tan = cot ( ) tan + = cot ( ) tan


= cot ( )
2

2
2

tan
+ = cot ( )
2

Worked example 8

eBook plus

If sin ( ) = 0.4 and tan () = 0.6, find:

+ .
a cos +
b tan
2

Tutorial

int-0548
Worked example 8

Think
a

WriTe

+ = sin ( )
2

a cos

Cosine and sine are complementary


functions and cosine is negative in the
second quadrant.

Substitute the given value for


sin ( ).

Tangent and cotangent are complementary functions


and tangent is negative in the 4th quadrant.

= 0.4
3

+ =
b tan
2

cot (

tan ( )
1

Substitute the given value for


tan ().

Calculate.

= 1.667, correct to
3 decimal places

0.6

rememBer

1. Exact values can be determined by using an equilateral triangle and a right isosceles
triangle as shown.

30
3

45
2
60
1

270

maths Quest 12 mathematical methods CaS for the Ti-nspire

45

30

45

60

90

180

270
3
2

360

sin ( )

1
2

2
2

3
2

cos ( )

3
2

2
2

1
2

tan ( )

3
3

Undef.

Undef.

2. All angles in the unit circle are positive in the 1st quadrant, sine is positive in the
2nd quadrant, tangent is positive in the 3rd quadrant and cosine is positive in the
4th quadrant. Symmetry properties:
sin ( ) = sin ( )

sin ( + ) = sin ( )

sin (2 ) = sin ( )

cos ( ) = cos ( )

cos ( + ) = cos ( )

cos (2 ) = cos ( )

tan ( + ) = tan ( )

tan (2 ) = tan ( )

tan ( ) =

tan

( )

3. Negative angles
sin ()=sin ( ),
cos () = cos ( ),
tan ()=tan ( )
4. Complementary angles

(a) Complementary angles add to 90o or radians.


2
(b) The sine of an angle is equal to the cosine of its complement.
(c) The complement of the tangent of an angle is the cotangent.

exerCiSe

6B

sin
2

= cos ( )

sin +
2

= cos ( )

sin

2

= cos ( )

sin
+
2

= cos ( )

cos
2

= sin ( )

cos +
2

=sin ( )

cos

2

=sin ( )

cos
+
2

=sin ( )

tan
2

= cot ( )

tan +
2

=cot ( )

tan

2

= cot ( )

tan
+
2

=cot ( )

Symmetry and exact values


Note: Give answers as surds with rational denominators.

eBook plus

1 We3a

Without using a calculator, find the exact values of the following.

a sin (120)

b cos (135)

c tan (330)

SkillSHEET 6.3

d cos (225)

e sin (210)

f tan (150)

Rationalising
the
denominator

g sin (315)

h cos (300)

i tan (225)

j cos (390)

k sin (405)

l tan (420)

Digital doc

Chapter 6

Circular (trigonometric) functions

271

2 We3b
eBook plus
Digital doc

Spreadsheet 142
Unit circle

Find the exact values of the following.

3
a sin
4

5
b cos
6

2
c tan
3

4
3

5
e sin
4

7
f tan
6

g sin

11
6

5
h cos
3

7
i tan
4

9
j cos
4

13
k sin
6

7
l tan
6

d cos

3 We4
If sin (x) = 0.3, cos (a) = 0.5, tan (b) = 2.4 and x, a and b are in the first quadrant,
find the value of the following.
b cos ( a)
c tan (2 b)
d cos ( x)
a sin ( x)
e sin ( a)
f tan ( + b)
g sin (2 x)
h cos (2 a)
i tan ( b)
j cos (2 + x)
k sin (2 + a)
l tan (2 + b)
n cos (3 + a)
o tan (3 b)
m sin (3 x)
If sin ( ) =

7
25

and cos( ) =

24
,
25

find tan () and show that sin2 () + cos2 () = 1.

< x < , find cos (x) and hence find tan (x).
2
1
3
, find sin (x) and tan (x).
b If cos ( x ) =
, and < x <
2
2
If sin ( x ) = 12, and

5 a We5

c If sin ( x ) =

3
2

, and

3
< x < 2 , find cos (x) and tan (x).
2

3
, find sin (x) and cos (x).
2
Find the exact value of the following.

d If tan ( x ) = 3 , and < x <


6 We6
a
e
i
m

sin (30)
sin (120)
tan (240)
sin (420)

b
f
j
n

cos (45)
tan (135)
cos (330)
cos (390)

c
g
k
o

tan (60)
sin (225)
sin (315)
tan (405)

d cos (150)
h cos (210)
l tan (300)

Evaluate (sin + cos )2 + (sin cos )2.

7
8 We7

Find the exact values of:


a sin
b cos
3
6

7
6

d cos
4

2
e sin
3

5
f tan
6

g sin

5
h cos
4

4
i tan
3

5
j cos
3

13
k sin
6

9
l tan
4



Show that cos2 + sin 2 = 1.
4
4

10 We8

If sin ( ) = 0.3, cos (x) = 0.7 and tan () = 0.4, find the value of the following.

a sin x
2

272


c tan
4

+
b cos
2

maths Quest 12 mathematical methods CaS for the Ti-nspire

c tan
2

+
2

d cos

x
e sin
f tan +
2

2
3
3


+
i tan
j cos
2

11 mC If x = , 3 sin (2x) is equal to:


12
3
3 3
3 2
c
a
b
2
2
2


h cos
2

+
l tan
2

g sin + x

x
k sin
2

1
2

12 mC The expression 1 sin 2 is equal to:

13

a cos

b sin

D cos2
2

E 0

c sin 2
2

A weight on a spring moves so that its speed, v cm per second,


is given by the formula
t
v = 10 + 2 sin
6
a Find the initial speed.
b Find the speed of the weight after 5 seconds.
c What is the greatest speed that the weight can reach?

eBook plus
Digital doc

SkillSHEET 6.4
Problem solving
using trigonometry

14 The height, H, in metres, that sea water reaches up a particular


tree trunk at a Caribbean resort is
governed
t
by the equation H = 0.4 cos + 0.5,
12
where t is the number of hours past midnight. Find the
height of water up the trunk at:
a midnight
b 8 am
c 8 pm.

6C

Trigonometric equations
From the general equation sin (x) = a, we can find an
infinite number of solutions. An example of this general
equation is:
3
sin ( x ) =
.
2
3

One of the solutions is x = , because sin =


.
3
2
3

However, we also know that sin =


because sine

3
2
is positive in the second quadrant.

sin ( 3 )

sin ( 3 )
x

2
and
.
3
3
(There are no solutions in the third and fourth quadrants because here, sine is negative.)
For this equation there are two solutions between 0 and 2. They are

Chapter 6

Circular (trigonometric) functions

273

To find a greater number of solutions we can go around the unit circle as many times as we
wish, finding new solutions each time. Since

sin 2 + =

3
2

and sin 3 =

3
2

2 7
8
, ,
.
and
3 3 3
3
We can also go in a negative direction. In the domain [, ] there are only
2

2 solutions: and
.
3
3
in the domain [0, 4] there are 4 solutions:

Worked Example 9

Find all solutions to the equation cos ( x ) =

2
2

in the domain [0, 2].

Think

Write

Method 1: Technology-free
2
2

Write the question.

cos ( x ) =

Find the equivalent angle in the first quadrant,


ignoring the sign.

Basic angle is

In the 2nd and 3rd quadrants cos (x) is


negative. So write the appropriate values of x in
these quadrants.

2nd quadrant:

x=
4
3rd quadrant:

x= +
4
3 5
x=
,
4 4

Simplify.

Method 2: Technology-enabled
1

On a Calculator page, press:


MENU b
3:Algebra 3
1:Solve 1
Complete the entry line as:

, x ) | 0 x 2
2
Then press ENTER .
solve(cos( x ) =

274

Write the answer.

x=

Maths Quest 12 Mathematical Methods CAS for the TI-Nspire

3 5
,
4 4

.
4

x
5

Worked Example 10

Find all solutions to the equation sin () = 0.7 in the domain [0, 4]. Give your answers correct to
4 decimal places.
Think

Write

Method 1: Technology-free
1

Find the equivalent angle in the 1st quadrant,


ignoring the sign.

The basic angle


is 0.7754.

Find in which quadrants sin () is positive.

sin () is positive in the 1st and 2nd quadrants.

Find the values of in the 1st and 2nd


quadrants.

= 0.7754, 0.7754
= 0.7754, 2.3662

Since the domain is [0, 4], we need to go


around the circle twice, so add 2 to each of
the first two solutions.

= 0.7754, 2.3662,
0.7754 + 2, 2.3662 + 2

Simplify giving answers correct to 4 decimal


places.

= 0.7754, 2.3662, 7.0586, 8.6494, correct to 4


decimal places.

Method 2: Technology-enabled
1

On a Calculator page, press:


MENU b
3:Algebra 3
1:Solve 1
Complete the entry line as:
solve(sin (a) = 0.7,a) | 0 a 4p
Then press ENTER .

Write the answer.

If sin (a) = 0.7, 0 a 4p, then


a = 0.7754 or 2.3662 or 7.0586 or 8.6494, correct
to 4 decimal places.

Worked Example 11

Calculate all solutions to the equation 2 sin ( 2 ) =


Think

3 in the domain 0 x 360.


Write

Since the equation contains 2 rather than ,


multiply both end points of the domain by 2.

0 360
0 2 720

Simplify the trigonometric equation.

2 sin (2 ) = 3
sin (2 ) =

3
2

Chapter 6 Circular (trigonometric) functions

275

Find the first quadrant (basic) angle, ignoring the


negative sign.

Basic angle = 60

Sine is negative in the 3rd and 4th quadrants. Find


the angles in these quadrants equivalent to the
basic angle.

2 = 180 + 60, 360 60


2 = 240, 300

As the domain has been extended to 720, we need


to go around the circle two times, so add 360 to
each of the two initial values.

2 = 240, 300, 240 + 360, 300 + 360


2 = 240, 300, 600, 660

Simplify.

= 120, 150, 300, 330

Worked Example 12

Calculate the sum of the solutions between 0 and 2 for the equation sin (3x) = cos (3x).
Think

Write

Adjust the domain as shown.

0 x 2
0 3x 6

Divide both sides by cos (3x).

Find the basic angle.

Solve for x between 0 and 6. Note that tan is


positive in the 1st and 3rd quadrants.

Cancel where possible.

sin (3x) = cos (3x)


tan (3x) = 1

Basic angle =
4

3 x = , + , 2 + , 3 + , 4 + , 5 +
4
4
4
4
4
4
5 9 13 17 21
= ,
,
,
,
,
4 4 4
4
4
4
5 9 13 17 21
x= ,
,
,
,
,
12 12 12 12 12 12
5 3 13 17 7
x= ,
,
,
,
,
12 12 4 12 12 4

Check that all answers are between 0 and


24 8 . They are.
2 (2 =
=
)
12
4
Calculate the sum of the solutions.

Sum of the solutions


5 3 13 17 7

= +
+
+
+
+
12 12 4
12
12
4
11
=
2

General solutions of trigonometric equations

3
and x = .
4
4
However, if a domain is not specified, there are an infinite number of solutions as multiples of

3
2 can be added or subtracted indefinitely to and . In this situation a general solution is
4
4
obtained where the solutions are in terms of a parameter, n, where n is an integer (that is, n Z).
The solution to the equation sin ( x ) =

276

where x [0, 2] is x =

Maths Quest 12 Mathematical Methods CAS for the TI-Nspire

becomes x = 2 n + , where n Z.
4
4
3
3
The general solution for the second quadrant solution x =
becomes x = 2 n + ,
4
4
where n Z.
3
Note that the general solution x = 2 n +
can be expressed as
4

x = 2 n + = (2 n + 1) .
4
4
Substituting different integer values of n will give specific solutions, as shown in the table below.

3
n
x = 2 n + , n Z
x = 2 n +
, n Z
4
4
3 5
7
1
x = 2 +
=
x = 2 + =
4
4
4
4
3 3

0
x=0+ =
x=0+
=
4 4
4
4
9
3 11
1
x = 2 + =
x = 2 +
=
4
4
4
4
17
3 19
2
and so on
x = 4 + =
x = 4 +
=
and so on
4
4
4
4
The general solution for the first quadrant solution x =

In general:

If sin (x) = a, then x = 2n + sin 1 (a) and x = (2n + 1) sin 1 (a),


where a [1, 1] and n Z.

If cos (x) = a, then x = 2n cos 1 (a), where a [1, 1] and n Z.


1
If tan (x) = a, then x = n + tan (a), where a R and n Z.
Worked example 13

eBook plus

Find the general solution of the equation 2 cos ( x ) 1 = 0.


Hence, find all the solutions for x [2, 2].
Think

int-0549
Worked example 13

WriTe

Write down the general solution for cos (x) = a.

Substitute a =
1
cos 1
2

Tutorial

1
2

into the general equation and evaluate

recognising that it is an exact angle.

Write the two separate solutions and specify n Z.

Note the answer from step 2 could be further


simplified by combining the two terms. A CAS
calculator will give the answer in this form.

Substitute n = 1, n = 0 and n = 1 into each of the


general solutions.

Write down the solutions for x [2, 2].

x = 2n cos 1 (a)
x = 2 n cos

( )
1

x = 2 n
4

x = 2 n + and x = 2 n , n Z.
4
4
8n (8n 1)
,nZ
x=
=
4
4
7
9
and x =
4
4

n = 0: x = and x =
4
4
7
9
n = 1: x = 2 =
and x = 2 + =
4
4
4
4
7 7
x=
,
, ,
4
4 4 4

n = 1 : x =

Chapter 6

Circular (trigonometric) functions

277

Worked Example 14

Find the general solution of the equation 2 sin ( 2 x ) =


domain 0 x 2.
Think

278

3 and hence find all solutions for x in the

Write

On a Calculator page, press:


MENU b
3:Algebra 3
1:Solve 1
Complete the entry line as:
solve(2 sin (2 x ) = 3 , x )
Then press ENTER .

Write the two separate solutions and


specify n Z.

Solving 2 sin (2x) = 3 gives


(6 n 1)
(3n + 2)
x=
and x =
, n Z.
6
3

Substitute n = 0 and n = 1 and n = 2 into


each of the general solutions, to find the
solutions in the domain 0 x 2p.

n = 0: x =

This answer can be verified using a CAS


calculator.
Complete the entry line as:

solve(2 sin (2 x ) = 3 , x ) | 0 x 2
Then press ENTER .

2
6
3
5
5
n = 1: x =
and x =
6
3
11
8
n = 2: x =
and x =
6
3
2 5 5 11
,
,
,
For 0 x 2p, x =
3 3 6 6

Maths Quest 12 Mathematical Methods CAS for the TI-Nspire

and x =

Worked Example 15

Find the general solution of the equation sin (3x) = cos (3x) and hence find
all solutions for x in the domain 0 x 2.
Think
1

Divide both sides by cos (3x).

Write

sin (3 x ) = cos (3 x )
sin (3 x )
=1
cos (3 x )
tan (3x) = 1
-

Write down the general solution for tan (3x) = a. 3x = np + tan 1 (a)

Substitute a = 1 into the general equation and


evaluate tan 1 (1), recognising that it is an
exact angle.
Divide each side by 3 to solve for x.

To find the solutions in the domain 0 x


2p, substitute n = 0, 1, 2 ... into each of the
general solutions.

Write down the simplified solutions for


0 x 2p.

This answer can be verified using a CAS


calculator.
Note: 1. The general formula here is found using
3
the basic angle of
, which means
4
that values of n start at 1 rather than 0.
2. Remember to scroll to the right to view
the full set of solutions over the given
domain.

3x = np + tan 1 (1)

3 x = n +
4
x=

(4 n + 1)
, n Z
12

n= 0: x =

12

n = 1: x =

5
12

n = 2: x =

9
12

n = 3: x =

13
12

n= 4: x =

17
12

n = 5: x =

21
12

x=

5 3 13 17 7
, , ,
,
,
12 12 4 12 12 4

Chapter 6 Circular (trigonometric) functions

279

Worked Example 16

Find the solutions of the equation cos ( x ) + cos ( 3 x ) = 12 , x [ 2 , 2 ] .


Think

Write

On a Calculator page, press:


MENU b
3:Algebra 3
1:Solve 1
Complete the entry line as:
1
solve(cos ( x ) + cos (3 x ) = , x ) 2 x 2
2
Then press ENTER .
An exact solution is not available.

A numerical solution will be appropriate and can


be found by pressing Ctrl / ENTER .

Write the solutions.

Solving cos( x ) + cos(3 x ) =

1
2

for

x [-2p, 2p] gives


x = - 5.65487, -2.0944,
-0.628319, 0.628319,
2.0944, 5.65487

REMEMBER

1. Given a general equation such as sin (x) = a, there can be an infinite number of
solutions. The domain is usually restricted and it is important to find all values for x
within this domain.
2. If the domain is given in radians, then the solution(s) to x should be in radians. If the
domain is given in degrees, then the solution(s) to x should be in degrees.
3. Adjust the domain to match what has been done to the angle in the question.

280

Maths Quest 12 Mathematical Methods CAS for the TI-Nspire

4. Sine is positive in the 1st and 2nd quadrants, cosine is positive in the 1st and 4th
quadrants and tangent is positive in the 1st and 3rd quadrants.
5. If sin (x) = a, then the general solution is x = sin1 (a) + 2n, n Z.
6. If cos (x) = a, then the general solution is x = cos1 (a) + 2n, n Z.
7. If tan (x) = a, then the general solution is x = tan1 (a) + n, n Z.
8. Find all the solutions within the specified domain, by substituting integer values for n
into the general solution.
9. If given cos2 (x) = a, then you will need to find values in all 4 quadrants (as x will
have both positive and negative values).
10. If the equation is of the form sin (ax) = k cos (ax), divide both sides by cos (ax) to
change the equation to tan (ax) = k.

exerCiSe

6C

Trigonometric equations
Find all solutions to the equations below in the domain [0, 2].

1 We9

eBook plus
Digital doc

Spreadsheet 135
Trig equations

a cos ( ) = 0

b sin ( ) =

d sin ( ) = 1

e cos ( ) =

1
2

cos ( ) =

sin ( ) =

1
2

3
2

Find all the values of between 0 and 360 for which:


a sin ( ) = 1

b cos( ) =

1
2

d cos ( ) = 1

e sin ( ) =

3
2

3 We10 For each equation below, find all the values of x between 0 and 4. Give answers
correct to 4 decimal places.
a cos (x) = 0.6591
b sin (x) = 0.9104
c cos (x) = 0.48
d sin (x) = 0.371
4 Find all the values of x between 0 and 360 for which:
a sin (x) = 0.2686
b cos (x) = 0.7421

c sin (x) = 0.5432


d cos (x) = 0.1937
Give answers correct to 2 decimal places.
5

Find the solutions to the following equations in the domain 0 x 2.


a 2 sin (x) = 1
c 2 sin ( x ) =

b 3 cos (x) = 0
d

2 cos( x ) = 1

6 We11 Find the solutions to the following equations in the domain 0 x 360.
Give exact answers where possible, otherwise give answers correct to 2 decimal
places.
a cos (2x) = 1
b 2 sin (2x) = 1
c 2 cos (3 x ) =
e sin (3x) =
g 4 sin

0.1254

( x ) = 0.913
1
2

d 2 sin (3 x ) = 3
f 3 cos (2x) = 0.5787
h

2 cos( x ) = 0.2751

Chapter 6

Circular (trigonometric) functions

281

7 Find all the solutions between 0 and 2 to the following equations. Give exact answers where
possible, otherwise give answers correct to 4 decimal places.
x

a 4 sin (x) + 2 = 6

b 3 cos (x) 3 = 0

c cos

x
d sin + 5 = 5.32
3

e 2 sin (3x) 5 = 4

g 2 cos (2 x ) + 3 = 0

1
3

+ 4 = 4.21

2 cos (3 x ) + 2 = 3

sin 2 x 1 = 0.8039

8 We12 Calculate the sum of the solutions between 0 and 2for each of the following
equations. Give exact answers for questions a to d. Otherwise give answers correct to
4 decimal places.
a sin (x) =cos (x)
b sin (2x) =cos (2x)
c sin (2 x ) = 3 cos (2 x )

eBook plus
Digital doc

WorkSHEET 6.1

f sin (x) +3 cos (x) =0


e sin (3x) +2 cos (3x) =0
d 3 sin (3 x ) = cos(3 x )
9 A particle moves in a straight line so that its distance, x metres, from a point O is given
by the equation x =3 +4 sin (2t), where t is the time in seconds after the particle begins
to move.
a Find the distance from O when the particle begins to move.
b Find the time when the particle first reaches O. Give your answer correct to 2 decimal
places.
10 We13
Find the general solution of the following equations. Hence, find all solutions for
2 x 2 for each equation.
a 2 cos ( x ) 3 = 0

b tan ( x ) =

2 sin ( x ) 1 = 0

11 We14 Find the general solution of the equation 2 sin (2x) 1 = 0. Hence, find all solutions for
x .
12 Find the general solution of the equation 2 cos (3x) 1 = 0. Hence, find all solutions for
x .
13 We15 Find the general solutions for each of the following equations. Hence, find all solutions
for x [0, 2].
a

3 sin ( x ) = cos ( x )

b sin (2x) =cos (2x)

14 We16 Find the solutions of the equation sin (2 x ) + sin (3 x ) =

6d

3
,x
2

3 sin (3 x ) = cos (3 x )

[ , ] .

Trigonometric graphs
Graphs of the sine and cosine functions
The graph of the function f (x) = sin (x) is drawn below. It is drawn over the domain [2, 2].
The graph repeats itself every 2 radians. We say that it has a period of 2. Half the distance
between the maximum and minimum values is 1 so we say
y
that the amplitude is 1.
1
It is possible to take any value of x for the function
0.5
f (x) = sin (x), so the domain of the whole function is R.
The range is [1, 1]. The graph is shown at right.
3
3
0
2
2 x
2

0.5

1
f(x) = sin (x),

282

maths Quest 12 mathematical methods CaS for the Ti-nspire

x 2

The graph of the function f(x) = cos (x) is the same shape
as the sine graph, but the graph has been translated to a
different position. The period is also 2. The amplitude is
1, the domain is R and the range is [1, 1]. The graph is
shown at right.

y
1
0.5
3
2 0
2
2
0.5

2 x

1
f(x) = cos (x), -2p x 2p

Dilation
If we change the amplitude, the distance between the maximum value and the minimum value
also changes.
y
The graph of f(x) = 2 sin (x) is shown at right. The amplitude is
2
2. The period is still 2. The domain is R and the range is [2,2].
1
This graph is a dilation of the basic graph of f(x) = sin (x) by
a factor of 2 from the x-axis. It has been stretched vertically.
0
3

2 x
2
2
Generally, if f(x) = a sin (x) or f(x) = a cos (x), a is the dilation
1
factor in the direction of the y-axis. The amplitude of the
2
graph is a .
f(x) = 2 sin (x), 0 x 2p
If we change the coefficient of x, the period of the graph
changes. The graph of f(x) = cos (2x) from 0 to 2 is shown
at right. The amplitude is 1 and the period is . This can be
y
found by dividing 2 by the coefficient of x.
1
2
In this case
= . The domain is R. The range is [1, 1].
0.5
2
The x-intercepts can be found by solving the equation
0
3
5
3
7 2 x
4 2
2
4
4
4
cos(2x) = 0.
0.5
3 5 7
2 x = , , ,
1
2 2 2 2
f(x) = cos (2x), 0 x 2p
3 5 7
so x = , , ,
4 4 4 4
1
This graph is a dilation of the basic graph of f(x) = cos (x) by a factor of 2 from the y-axis.
The period has been halved or the graph has been squashed up.
The decimal approximation for these solutions can be found using a graphics calculator. The
graphics calculator can also be used to check the number and approximate value of the solutions
when solving trigonometric equations.
1
Generally, if f(x) = sin (nx) or f(x) = cos (nx), the graph is dilated by a factor of from the
n
2
y
y-axis. The period of the graph is
.
n
4
The graph of f(x) = 4 sin (3x) is shown at right.
3
2

It is drawn from 0 to 2. The amplitude is 4 and the


1
2
4
5

3 3
3
3
2

3
0
period is
. The domain is R. The range is [ 4, 4].

2 x
2
2
1
3
2
The x-intercepts are found by solving 4 sin (3x) = 0.
3
So sin (3x) = 0
4
3x = 0, , 2, 3, 4, 5, 6
f(x) = 4 sin (3x), 0 x 2p
4 5
2
x = 0, ,
, ,
,
, 2
3 3
3 3
This is an example of a sine graph dilated in both x and y directions. It has a dilation factor of
1
4 from the x-axis, and of 3 from the y-axis.

Chapter 6 Circular (trigonometric) functions

283

Worked Example 17

State the period and the amplitude of the graphs of each of the following functions.
a y = 2 sin

( x)
1
4

b y =

1
cos
3

(2 x ) .

Think
a

Write

Write the equation.

Find the period of the graph, using the formula.

State the amplitude it is the coefficient in


front of the sine function.

Write the equation.

Find the period of the graph.

a y = 2 sin

( x)
1
4

2
1
,n = 4
n
2
4
So period = 1 = 2 = 8
1
4
Period =

Amplitude = 2
b y=

1
cos (2 x )
3

Period =
So period =

Amplitude =

State the amplitude. Remember that


the amplitude is always positive, so
we need only the magnitude of the
coefficient.

2
, n=2
n
2
=
2
1
3

Worked Example 18
1
Sketch the graph of y = 2 sin (3 ) for one complete cycle stating the amplitude, period and range.

Think

Write/draw

Method 1: Technology-free

284

Write the equation.

Find the period of the graph.

State the amplitude.

y = 2 sin (3 )
2
,n=3
n
2
So period =
3
Period =

Amplitude =

Maths Quest 12 Mathematical Methods CAS for the TI-Nspire

1
2

Sketch the basic sine shape (one full cycle)


2
1
with the period of
and the amplitude of 2 .
3
1
The scale on the x-axis should be 4 of a
period (to reflect the 4 quadrants),
2
2
so it is
4=
= .
3
12 6

The minimum value of the function is

1
2

and

y
1
2

1
2

Range : [ 2 , 2 ]

the maximum value is 2 , so state the range.


Method 2: Technology-enabled
On a Graphs page, complete the function entry
line as:
1
2
f 1( x ) = sin (3 x) | 0 x
2
3
Then press ENTER .

Reflection
If the coefficient of the function is negative, the graph is turned upside down, that is, reflected in
the x-axis. This does not alter the amplitude, which is always positive.
y = -f(x) is the image of f(x) when reflected in the x-axis. y
y = f(-x) is the image of f(x) when reflected in the y-axis. 4
3
The graph of f(x) = 4 sin (3x) is shown at right. You
2
will notice that it is the graph of f(x) = 4 sin (3x) turned
1
3

upside down. (The graph of f(x) = 4 sin (3x) is shown


2
2
5
4
2
0
on page 283.)


2 x
3 3
3
3
1
2
3
4

f(x) = -4 sin (3x), 0 x 2p

2
. The domain is R. The range is [4, 4].
3
4 5
2
The x-intercepts are 0, ,
, ,
,
, 2 .
3 3
3 3
1
This graph has a dilation factor of 4 from the x-axis and of 3 from the y-axis.
If the function f(x) = 4 sin (3x) is reflected in the x-axis, the result is f(x) = 4 sin (3x). If we
reflected the graph of f(x) = 4 sin (3x) in the y-axis, the result would still be f(x) = 4 sin (3x).
The amplitude is 4 and the period is

Chapter 6 Circular (trigonometric) functions

285

If we reflect f(x) = 2 cos (3x) in the x-axis, the result is f(x) = 2 cos (3x), but if we reflect it in the
y-axis, the graph does not change. This is because f(x) = 2 cos (3x) is symmetrical about the y-axis.
Check this on a CAS calculator.
y
4
3

Translation
If we add a constant to the function, the graph is moved up or
down and is said to be translated parallel to the y-axis. The
number that is being added becomes the median value of the
function.
The graph of f(x) = 3 cos (2x) + 1 is shown at right.
Compared to f(x) = 3 cos (2x), the graph is shifted 1 unit up.
The amplitude is 3, the period is , the domain is R and the
range is [2, 4].
The x-intercepts are found by solving 3 cos (2x) + 1 = 0.
So cos 2 x =

1
3

2
1
1

2 x

2
f(x) = 3 cos (2x) + 1, 0 x 2p

(cosine is negative in quadrants 2 and 3).

The basic angle is inverse cosine of

1
3

= 1.231c .

2x = 1.231, + 1.231, 3 1.231, 3 + 1.231


2x = 1.911, 4.373, 8.194, 10.656 x = 0.955, 2.186, 4.097, 5.328
If we add a constant to x, the graph is translated parallel to
the x-axis, that is, left or right.

The graph of f ( x ) = sin x is the graph of

y
1
0.5

0
5

2 x
4
4
units to the right.
0.5
4
1

The graph of f ( x ) = sin x + is the graph of

f ( x ) = sin x + , x 2

4
f(x) = sin (x) translated units to the left.
4
This graph is a translation of the

basic graph of the function


The graph of f ( x ) = sin x + is shown at right.

f ( x ) = sin( x ) units parallel to


It is drawn between [, 2].
4
The amplitude is 1, the period is 2, the domain is R and
the x-axis in a negative direction.
the range is [1, 1]. The y-intercept occurs when x = 0.
2

.
So y = sin 0 + =

4
2

The x-intercepts occur when sin x + = 0.

4
3 7

So x + = 0, , 2 x =
,
,
4
4 4 4

3
Note: The graph of sin (x) is the same as the graph of cos ( x ) or cos ( x + ). That is, the
2
2

3
sine graph can be turned into the cosine graph by a translation of units left or
units
2
2
right. The cosine graph can be turned into the sine graph by the opposite translations.

f(x) = sin (x) translated

If we take a general trigonometric function f(x) = a sin n(x b) + c, it has an amplitude


2
of a , a period of
, a horizontal translation of b units and a vertical translation of
n
cunits.

286

Maths Quest 12 Mathematical Methods CAS for the TI-Nspire

The graph of f ( x ) = 2 cos 3 x 1 is shown at right for

3
0 x 2.
2
The amplitude is 2, the period is
, the domain is R and the
3
range is [3, 1].

1
0

2 x

This graph is a dilation of the basic graph of f(x) = cos (x) by a


1
factor of 2 from the x-axis and a factor of 3 from the y-axis,

along with a translation of units to the right and 1 unit down.


3

Worked Example 19

Sketch the graph of y = 5 cos x + + 5 for 0 x 2, and state the period and amplitude.

4
Think

Write/draw

y = 5 cos x + + 5

Write the equation.

Determine the period.


(The coefficient of x is 1.)

Period =

State the amplitude.

Amplitude = 5

Using a pencil, sketch a basic cosine shape with


the period and amplitude above.

2
= 2
1

y
5

y = 5 cos (x)

2 x

5
5

Apply the horizontal translation of +

(the + sign
4

means to the left). Again, use pencil.

Apply the vertical translation of +5 (+ means up).

5
3
7

4 2 4 2

y = 5 cos (x + 4 ) + 5

10

+5

+5

5
+5
+5

y = 5 cos (x)

5
6

5 y = 5 cos (x + 4 )

y = 5 cos (x + 4 )

+5

3
7

2 4

2 x

Chapter 6 Circular (trigonometric) functions

287

Erase the pencilled stages.

y
10

y = 5 cos (x + 4 ) + 5

2 x

rememBer

1. Graphs of the form y = a sin n(x b) + c and y = a cos n(x b) + c are transformations
of y = sin (x) and y = cos (x).
2. a is the amplitude and is a dilation from the x-axis. If a is negative, the amplitude is
still positive but the graph is reflected in the x-axis.
3. c is the vertical translation (the translation parallel to the y-axis). If c is positive,
the graph is translated c units up, and if c is negative, the graph is translated
c units down.
4. The range is [c |a|, c + |a|].
2
.
5. The period is
n
6. The factor n is the horizontal dilation where the graph has been dilated by a
1
factor of from the y-axis.
n
7. The value b is the horizontal translation (the translation parallel to the x-axis). If b is
positive, the graph is translated b units to the right, and if b is negative, the graph is
translated b units to the left.

exerCiSe

6d

Trigonometric graphs
1 We17

eBook plus
Digital doc

SkillSHEET 6.5
Period and
amplitude
of sine and
cosine graphs

eBook plus
Digital doc

Spreadsheet 124
Sine graphs

State the period and amplitude of the graphs of each of the following.

a y = cos (x)
1
3

d y = cos ( x )
g y = 3 sin

1
2

c y = 4 sin (x)

e y = 2 cos (3x)

f y = 3 sin (2x)

h y = 2 cos

( x)
1
3

y=

1
3

cos (2 x )

j y = 4 sin (3x)
2 We18
Sketch the graphs of each of the following for one complete cycle and state the
amplitude, the period and the range.
2

a y = 3 cos( )

b y = 4 sin ( )

d y = 2 cos (3 )

e y = 2 cos (3 )

g y = 4 sin

288

( x)

b y = sin (x)

( )
1
2

h y = 3 cos

maths Quest 12 mathematical methods CaS for the Ti-nspire

( )
1
3

c y = 3 sin (2 )
f y=

1
3

sin (2 )

eBook plus

Sketch the graph of the function f:R R where f (x) = 4 cos (3 ) for 0 2. State the
amplitude, period and range.

Sketch the graph of the function f:R R where y = 2 sin (2x) for x . State the
amplitude, period and range.

From the basic graphs of y = sin (x) and y = cos (x), state the horizontal translation and the
vertical translation for each of the following.

a y = sin x + + 3
b y = cos x + 1

3
2

Digital doc

Spreadsheet 012
Cosine graphs

c y = 3 cos x 2

d y = 2 sin x + 1

6 Sketch the graphs of the following for one complete cycle stating the amplitude, the period and
the range.
a y = sin (x) + 1
b y = cos (x) 1
c y = 2 cos (x) 2
d y = 2 sin (x) + 3
e y = sin (3x) 1
f y = cos (2x) + 1
1

h y = 2 sin (2 x ) + 3

g y = 3 cos (3x) 2
j y = 2 cos

( x) 1

y = 3 sin

( x) + 4
1
2

1
3

7 We19 Sketch the graphs of the following for 0 2. State the period and
amplitude.

a y = sin
b y = cos +
c y = 3 cos

3
4
2

d y = 2 sin

g y = cos 3 + 1

e y = 2 sin 2 +

h y = 2 sin 2 2

f y = 3 cos 3 +

i y = 2 sin 1

j y = cos 2 ( ) + 1
8

Write down the amplitude, period and range of the following graphs.
a

y
4

y
4

2
3
0


4
2 4
4
2

0
2

4 x

y
2

y
1

1
0
1

0.5

3 x

0.5

0 2 3 4 5 6 7 8 9 x

Chapter 6

Circular (trigonometric) functions

289

4
3
2
1

2 1 0
2

x
7

y
1.5
1
0.5
0.5
1
1.5

y
1
0.5

2 x

0.5

3 x

State the maximum and minimum values for each of the following.
a y = cos (x)
b y = sin (x)
c y = 3 sin (x)
d y = 2 cos (x)
e y = 2 cos (3x)
f y = 3 sin (2x)
g y = 4 sin (2x) + 1
h y = cos (4x) 2
i y = 2 cos (x ) 3
j y = 4 sin 3 (x + ) + 1

k y = 2 sin 3 ( x + 2 ) + 2

l y = 3 cos 2 ( x 2 ) 4

10 Sketch the graphs of the following over the domain [0, 2] and state the period, amplitude and
range.
a y = cos (x)
b y = sin (x)
c y = 2 sin (x)
d y = 3 cos (x)
e y = 3 cos (2x)
f y = sin (3x)
g y = 1 4 sin (x)
j y = 2 sin (x 2)

h y = 2 cos (2x) 2

i y =

1
2

cos 3 ( x + ) + 1

to the left, what is the new equation?


3

12 If the graph of y = 2 cos (3x) 2 is translated to the right and 3 units up, what is the new
4
equation?

13 If the graph of y = 3 sin (x ) + 1 is translated to the left and 3 units down, what is the new
3
equation?
11 If the graph of y = sin (x) + 1 is translated

Water level

14 The level of the water in the Banksia River was measured at hourly intervals from midnight
and the results recorded. The graph below shows the results.
Find:
y
a the amplitude
3
b the period
2
c the maximum height of the river
d the minimum height of the river
1
e at what times the river has maximum
height
0 2 4 6 8 10 12 14 16 18 20 22 24 x
f at what times the river has a minimum
Hours
height
g the equation of the curve which is of the
form y = A sin a(x) + B.

290

Maths Quest 12 Mathematical Methods CAS for the TI-Nspire

6E

Graphs of the tangent function


The graph of the function f(x) = tan (x) is shown at right.

x = 2
It is drawn over the domain [2, 2]. The graph repeats
y

x = 32
itself every radians, so its period is . There are vertical
x = 2 x = 32
3
asymptotes through half the period,
2
3

that is, at x = and , so the function is not


2
2
1
defined at these points. Generally, the vertical
0 3
2 32 2
2 x

2
2
asymptotes are given by the equation x = (2k + 1) ,
1
2
2
where k Z (that is, k = 0, 1, 2 ). Hence, the domain

3
of the tangent function is R \ {x : x = (2k + 1) , k Z}.
2
Unlike sine and cosine functions, the tangent graph does not have an amplitude; it extends
infinitely up and down,so its range is R.

Dilation

y = 2 tan (x)
y = tan (x)

Compared to f(x) = tan (x), the graph of f(x) = a tan (x) is dilated
by a factor of a from the x-axis. Vertical dilation does not affect the
period, domain or range, or the position of the asymptotes or the
x-intercepts. In fact, its effect can be seen only when two graphs
are sketched on the same set of axes. The diagram at right shows
the graphs of f(x) = tan (x) and f(x) = 2 tan (x) over the domain
[0, 2]. The graph of f(x) = 2 tan (x) is the dilation of the basic
tangent graph by a factor of 2 from the x-axis. It has been stretched
vertically.
Compared to f(x) = tan (x), the graph of f(x) = tan (nx) is dilated
1
by a factor of
from the y-axis. Horizontal dilation
y
n
x = 4
3
affects the period and domain of the graph, as well as
the position of the asymptotes and x-intercepts.
2

The period of f(x) = tan (nx) is and the general equation


1
n

of the asymptotes is x = (2k + 1) ,where k Z.


0
2n
1 4 2

The domain changes to R \ {x : x = (2k + 1) , k Z}


2
2n
accordingly. (The range is not affected by the value of n.)
3
At right is the graph of f(x) = tan (2x). This graph is a
dilation of the basic f(x) = tan (x) graph by a factor of 12

from the y-axis; its period is . As the period has been


2
halved, the resultant graph is compressed horizontally.

3
2
1
0
1

32

2
3

x = 2 x = 32

x = 34 x = 54 x = 74

3
7

2 4

2 x

x = 2 x = 32

3
2
1

Reflection
If the coefficient a in f(x) = a tan (x) is negative, the graph is
reflected in the x-axis. Reflecting the graph does not affect its period,
domain or range, or the position of the asymptotes. At right is the
graph of f(x) = 2 tan (x). This graph is a reflection in the x-axis of
the graph f(x) = 2 tan (x), shown previously.

0
1

2
3

Chapter 6 Circular (trigonometric) functions

291

Translation
y
If a constant is added to the function, the graph is translated
x = 2 x = 32
vertically, that is, up or down, parallel to the y-axis. Thus, the
3
graph of f(x) = tan (x) + c represents a translation of the graph
2
f(x) = tan (x) by c units in the y direction. If c > 0, the graph is
1
shifted up, and if c < 0, it is shifted down. Vertical translation
does not affect the period, domain or range, or the position of
0 3 2 x
2
2
the asymptotes. The axial intercepts, however, will change.
1
The diagram at right shows the graph of y = tan (x) + 2. It is
2
translated 2 units up, compared to the basic graph of
y = tan (x).
3
If a constant is added to x, the graph is translated
horizontally, that is, right or left, parallel to the
x-axis. Thus, the graph of f(x) = tan (x b),
y
x = 4
x = 54
represents a translation of the graph f(x) = tan (x)
3
by b units in the x direction. If b > 0, the graph is
2
shifted to the right, and if b < 0, it is shifted to the
left. Horizontal translation has no effect on the
1
period or range, but it does affect the domain and
3
5

the position of the asymptotes and the axial


2 x

3
0
4
4
2
4
4
2 4
1
intercepts.

2
The diagram at right shows the graph of y = tan ( x + ).
4

translated units to the left, compared to the basic


4
graph of y = tan (x).
Having considered each transformation individually, we can now summarise them as
follows.
Compared to the basic graph of y = tan (x), the graph of y = a tan [n(x b)] + c is:
dilated by a factor of a from the x-axis
1

dilated by a factor of from the y-axis (and hence has a period of )


n
n
reflected in the x-axis if a < 0
translated c units vertically (up if c > 0 and down if c < 0)
translated b units horizontally (to the right if b > 0 and to the left if b < 0).

Worked Example 20

x
State the period and sketch the graph of y = 2 tan , showing one full cycle.
4
Think

Write/draw

Write the equation.

x
y = 2 tan
4

Find the period.

Period =

; n=
n

So, period =

292

Maths Quest 12 Mathematical Methods CAS for the TI-Nspire

1
4

1
4

4
= 4
1

Sketch the tangent shape with a period of 4. The


graph is dilated by a factor of 2 parallel to the
y-axis, so in comparison to the graph of y = tan (x),
each y-coordinate will be doubled. The vertical
asymptote goes through half of the period, so it is
at x = 2.

x = 2

3
2
1
0
1

2
3

Worked example 21

eBook plus

State the period and sketch the graph of y = tan 2 ( x ) + 1


4
for 0 x .
Think

Tutorial

int-0550
Worked example 21

WriTe/draW

Write the equation.

y = tan 2 x + 1

Find the period.

Period =

Using pencil, sketch the basic tangent shape with

a period of .
2
(Since 0 x , we need to show two cycles.)
Remember that the asymptotes are at the middle
of each cycle (that is, halfway through the
period).

; n = 2. so period =
n
2
x = 4 x = 34

3
2
1
0
1

2
3
4

Since we need a negative tangent graph, reflect


the graph in the x-axis.

x = 4 x = 34

3
2
1
0
1

2
3

Chapter 6

Circular (trigonometric) functions

293

Translate the graph

units to the right.


4

y
3

x = 4 x = 2

x = 34

1
0
1

2
3
6

Translate the graph 1 unit up.

x = 2 x =

3
2
1
0
1

2
3

Erase the pencilled stages to see the final graph.

x = 2 x =

3
2
1
0
1

2
3

REMEMBER

1. The graph of y = tan (x) has a period of and does

y
x = 32 x = 2
x = 2 x = 32
not have an amplitude.
3
2. The equations of the vertical asymptotes of
2

y = tan (x) are given by x = (2k + 1) , where


1
2
k Z, that is, k = 0, 1, 2 ...
0 3
2 32 2
2 x
2
2
3. The domain of the tangent function is
1

R \ {x : x = (2k + 1) , k Z} and its range is R.


2
2

294

Maths Quest 12 Mathematical Methods CAS for the TI-Nspire

4. Compared to the basic graph of y = tan (x), the graph of y = a tan [n(x b)] + c is:
dilated by a factor of a from the x-axis
1

dilated by a factor of from the y-axis (and hence has period of )


n
n
reflected in the x-axis if a < 0
translated c units vertically (up if c > 0 and down if c < 0)
translated b units horizontally (to the right if b > 0 and to the left if b < 0).
exerCiSe

6e

Graphs of the tangent function


1

We20 State the period and sketch the graphs of each of the following, showing one full cycle.
a y = 4 tan (x)
b y = tan (2x)
c y = tan (3x)
x
1
f y = 2 tan
d y = 2 tan (4x)
e y = tan 4 x
3
x

h y = 5 tan (2x)
g y = 3 tan
i y = 21 tan (4 x )
2

( )

y = 13 tan

( x)
1
2

2 For each of the following, state:


i the period
ii the transformations, compared to the basic graph y = tan (x).
a y = 2 tan (3x)
d y = tan

( x) 2
1
2

1
g y = 4 tan 3 x +

b y = tan (4x) +1

e y = 5 tan x +

c y = 3 tan (2x) 4

f y = tan 2 x

x
1
h y = 6 2 tan
3

i y = tan 4 x 3

12

j y = 2 tan x + + 5

3
3

We21 State the period and sketch the graphs of each of the following for 0 x .

a y = tan (x) +2

d y = 2 tan x

2
x
1
g y = 2 tan 1
2
j y=

6F

1
3

b y = tan (2x) 3

c y = tan x +

1
e y = tan 4 ( x )

h y = 2 tan 2 x +

x
1
f y = 3 tan + 1 2
6
3


tan 4 x + + 1
i y=

4
12

tan 2 x 2

Finding equations of
trigonometric graphs

eBook plus
Interactivity

Sometimes it is necessary to be able to find the equation of a


trigonometric function from a graph. The following worked examples
illustrate how this can be done.

Chapter 6

int-0251
Finding equations of
trigonometric graphs

Circular (trigonometric) functions

295

Worked Example 22

The equation of the following graph is in the form


y = a sin (nx). Find the values of a and n. Hence, find
the equation of the function.

y
2
1
0
1

Think

Write

State the amplitude of the graph.

Amplitude = 2

The coefficient a represents the amplitude. Since


the graph is not reflected in the x-axis, a is positive.

The graph is not reflected, so a = 2.

State the period of the graph (it is the length of one


full curve).

Period = 4.

Use the formula for the period to find the value


ofn.

Period =
n=

Substitute the values of a and n into y = a sin (nx)


to find the equation of the function.

2
2
, so
= 4 ;
n
n

2
=
4

1
2

y = a sin (nx); a = 2, n =
y = 2 sin

1
2

( x)
1
2

Worked Example 23

The graph shown is a trigonometric function of the


form y = a sin (nx) + c. Find the values of a, n and c.
Hence, find the equation of the function.

y
2
1
1 0 4
2
3
4

Think
1

296

The amplitude (a) is half the distance between the


maximum and minimum values.
The period is the interval from one point on the
graph to the next point where the graph begins to
2
repeat itself. The period is .
n
The line through the centre of the graph
is y = 1, so the graph has been translated
down 1 unit.

Write

y = a sin (nx) + c
1

Amplitude a = 2 (2 + 4) = 3
2
= , so n = 2.
The period is
n

c = 1
So the equation is y = 3 sin (2x) 1.

Maths Quest 12 Mathematical Methods CAS for the TI-Nspire

2 x

Worked Example 24

This graph is a trigonometric function of the form y = c + a cos (nx). Find the values of a, n and c.
Hence, write the equation of the function.
y
3
2
1
0
1

Think

The amplitude (a) is half the distance between the


maximum and minimum values.

We know that the graph is a cosine graph so it must


be inverted; that is, a is negative.
The period is the amount of time taken to complete
the pattern once.

4
5

6 x

Write

c represents the vertical translation. This graph has


been translated up one unit.
The equation of this trigonometric function is in
the form y = c + a cos (nx).

y = c + a cos (nx)
1
Amplitude a = 2 (3 + 1)
=2
a = 2
Period =

2
=6
n

2
=n
6

so n =
3
c=1

y = c + a cos ( nx )

3

y = 1 2 cos x
3
c = 1, a = 2, n =

REMEMBER

1. Trigonometric functions can be expressed in the form of y = a sin [n(x b)] + c


ory=a cos [n(x b)] + c.
2. The amplitude a can be found by halving the distance between the maximum and
minimum values.
3. The period is the interval from one point on the graph to the next point where the
2
graph begins to repeat itself. The period is
.
n
4. The vertical shift is c.
5. The horizontal shift is b.
Exercise

6F

Finding equations of trigonometric graphs


1 WE22 The equations of the following graphs are of the form y = a sin (nx). Find the values of
a and n. Hence, find the equation of each function.

Chapter 6 Circular (trigonometric) functions

297

eBook plus
Digital doc

Spreadsheet 124
Sine graphs

Digital doc

y
2
1

1
2
3

eBook plus

y
3
2
1

0 1 2 3 4 5 6 7 8 9 10 11 12 x
1
2

Spreadsheet 012
Cosine graphs

2 The equations of the following graphs are of the form y = a cos (nx). Find the values of a and
n. Hence, find the equation of each function.
b
a
y
y
2

0
1

0 0.5 1 1.5 2 2.5 3 3.5 4 4.5 5 5.5 6 x


1
2

We23 The equations of the following graphs are of the form y = a sin (nx) + c. Find the
values of a, n, and c and hence write the equation of the function.
b
a
y
y

1.5

0 1 2 3 4 5 6 7 8x
1
2

1.0
0.5
0

3
4
5

2 x

4 The equations of the following graphs are of the form y = a cos [n(x )]. Find the values of a,
n and . Hence, write the equation of the function.
b
a
y
y
5

4
2
0
2

5 The equations of the following graphs are of the form y = a sin [n(x + )] + c. Find the values
of a, n, and c. Hence, write the equation of the function.
b
a
y
y
1
0
1


3 2

2
5

3 6

7
4
3
5 11

6 3 2 3 6

2 x

2
3

298

maths Quest 12 mathematical methods CaS for the Ti-nspire

5
4
3
2
1

0
1

We24 The equations of the following graphs are of the form y = c + a cos (nx). Find the
values of a, n and c. Hence, write the equation of the function.

y
4
3
2
1
0 1 2 3 4 5 6 7 8 9 10 11 12 x
1

y
4
3
2
1
0 1 2 3 4 5 6 7 8 9 10 11 12x
1

mC If the amplitude is 2, the period is 6 and there is a vertical translation of 2, then the

equation of the form y = a sin (nx) + b is:


a y = 2 sin (6x) 2

b y = 6 sin (2x) 2

D y = 2 sin
3

E y = 2 sin
6

x 2


c y = 2 2 sin x
3

x 2

mC If the period is , the range is [2, 4], and the horizontal

eBook plus

Digital doc
, the equation for the trigonometric function of the
WorkSHEET 6.2
4
form y = a cos [n(x + )] + b is:


a y = 3 2 cos x + + 1 b y = 3 cos 2 x + 1 c y = 2 cos 3 x + + 1

4
4
4

translation is

D y = 2 cos 3 x + + 1
4

6G

E y = 3 cos 2 x + 1
4

Trigonometric modelling
In real life there are many examples of periodic behaviour. Sine and cosine
functions such as y = a sin [n(x b)] + c and y = a cos [n(x b)] + c are
often used to model this behaviour.

eBook plus
eLesson

eles-0092
Trigonometric
modelling

Worked example 25

While out in his trawler John North, a fisherman, notes


that the height of the tide in the harbour can be found by
using the equation:

h = 5 + 2 cos t ,
6
where h metres is the height of the tide and t is the number
of hours after midnight.
a What is the height of the high tide and when does it
occur in the first 24 hours?
b What is the difference in height between high and low
tides?
c Sketch the graph of h for 0 t 24.
d John North knows that his trawler needs a depth of at
least 6 metres to enter the harbour. Between what hours is
he able to bring his boat back into the harbour?

Chapter 6

Circular (trigonometric) functions

299

Think

Write

Method 1: Technology-free
a

Write the given equation.

For high tide, find the maximum


value of h.


t
6

a h = 5 + 2 cos

For maximum h,

cos t = 1
6
So h = 5 + 2 1 = 7
Alternatively, maximum value =
median + amplitude so h = 5 + 2 = 7

Find when high tide occurs.

Find the minimum value of h.


cos t = 1, so
6

t = 0, 2 , 4 , . . .
6
t = 0, 12, 24, . . .
A high tide of height 7 m occurs at midnight, noon
the next day, and midnight the next night.
b For minimum h,


cos t = 1
6

So h = 5 + 2 1 = 3
Alternatively, min. value =
median amplitude so h = 5 2 = 3.
The difference between high and low
tides is 7 3 = 4 metres.

Find the difference between high and


low tides.

Use the information from the previous page


to sketch the graph.

h
6
4
2
0 2 4 6 8 1012 14 16 18 202224 t

300

Find t using the equation when h = 6.

Write the answer in words.

d When h = 6,


5 + 2 cos t = 6
6

2 cos t = 1
6
1
cos t = 2
6

5 7 11
t= ,
,
,
....
6
3 3 3
3
t = 2, 10, 14, 22, . . .
From the graph we can see that John North can
bring his boat back into harbour before 2 am,
between 10 am and 2 pm and between 10 pm and
2 am the next morning.

Maths Quest 12 Mathematical Methods CAS for the TI-Nspire

Method 2: Technology-enabled
1

On a Graphs page, complete the


function entry lines as:

f 1( x ) = 5 + 2 cos x
6
f 2( x ) = 6
Press ENTER after each entry.
Select an appropriate window.

To find the points of intersection,


press:
MENUb
7:Points&Lines7
3:IntersectionPoint(s)3
Use the NavPad to move the cursor
to the first point of intersection and
press Click x twice and the points
of intersection will appear.

t = 2, 10, 14, 22, . . .


rememBer

1. Read the question carefully to find out what is being asked.


2. Answer the questions in words where appropriate.
3. Ensure that graphs are sketched over the given domain.
exerCiSe

6G
eBook plus
Digital doc

Spreadsheet 041
Function
grapher

Trigonometric modelling
1

We25 Competition is severe, so Fred Greenseas decides that he will catch more fish in an
inlet several kilometres east of the place where John North fishes. There is a sandbar at the
entrance to the inlet and the depth of water in metres on the sandbar is modelled by the
t
function d (t ) = 6 + 2.5 sin where t is the number of hours after 12 noon.
6
a What is the greatest depth of the water on the sandbar and when does it first occur?
b How many hours pass before there is once again the maximum depth of water on the
sandbar?
c What is the least amount of water on the sandbar?
d Sketch the graph of d for 0 t 24.
e Fred Greenseas needs a depth of 7.25 metres to cross the sandbar. Between what hours is
he able to enter and leave the inlet?

Chapter 6

Circular (trigonometric) functions

301

2 A student wanting to catch fish to sell at a local market on Sunday has discovered that more
fish are in the water at the end of the pier when the depth of water is greater than 8.5 metres.

The depth of the water (in metres) is given by d = 7 + 3 sin t , where t hours is the
6
number of hours after midnight on Friday.
a
b
c
d
e

What is the maximum and minimum depth of the water at the end of the pier?
Sketch a graph of d against t from midnight on Friday until midday on Sunday.
When does the water first reach maximum depth?
Between what hours should the student be on the pier in order to catch the most fish?
If the student can fish for only two hours at a time, when should she fish in order to sell
the freshest fish at the market from 10.00 am on Sunday morning?

3 The mean daily maximum temperature in Tarabon, an experimental town in a glass dome, is

modelled by the function T (m) = 18 + 7 cos m , where T is in degrees Celsius and m is the
6
number of months after 1 January 2007.
a What was the mean daily maximum temperature in March 2007, and in August 2007?
b What is the highest mean daily maximum temperature in Tarabon? In which months does
it occur?
c What would the mean daily maximum temperature be in February 2008?
d If the pattern continued, how many months would pass before the mean daily maximum
temperature would be the same again as it was in February 2008?
4 The height above the ground of the middle of a skipping rope as it is being turned in a childs
game is found by using the equation h = a sin (nt) + c, where t is the number of seconds
after the rope has begun to turn. During the game, the maximum height the rope reaches is
1.8metres and it takes 2 seconds for the rope to complete a full turn.
a Find the values of a, n and c and hence write the equation of h in terms of t.
b Sketch the graph of h against t for 0 t 5.
c After how much time from the beginning of the turn will the rope be 25 cm above the
ground? Give your answer correct to the nearest tenth of a second.
y

5 The graph at right shows the path of a small lizard as


it runs over a hill.
a Calculate the height of the hill, in metres, and hence
find the amplitude of the trigonometric function.
b If the ground was flat, how far would the lizard run to
reach the same spot on the other side of the hill? Hence,
find the period of the function.
6 When Sloane and Michael were riding on a
Ferris wheel, they realised that as the wheel
turned clockwise, the height of their seat in
metres after t minutes could be modelled by
the function h(t) = a b cos (nt).
The graph of h against t is shown for the
whole ride.
a State the values of a, b and n.
b Write down the rule for h(t).
c How many times do Sloane and Michael
reach the highest point during their ride
and how far above the ground is it?
d How high are they when the ride begins
to move?
e How many minutes pass before they are
at this point again?

302

Maths Quest 12 Mathematical Methods CAS for the TI-Nspire

h = 34

30
25
20
15
10
5
0

h=2
9 12 15 t

y = 1 cos ( 4 x)

f How far above the ground are they after 1 minute?


g If the ride began when the boys were at the height found in part f , what would the
function become?
h Draw a graph of the new function for the first 6 minutes of the ride.
7 On a summer day the hourly temperature, which can be approximated to a cosine curve, was
recorded. The maximum temperature was 30C and occurred at 3.00 pm. The minimum
temperature was 10C and occurred at 3.00 am. The temperature was first recorded at
12midnight, then every hour for 24 hours.
a What is the amplitude of the function?
b What is the period of the function?
c What is the middle value of the function?
d How far has this middle value been translated upwards from the x-axis?
e When do the maximum and minimum temperatures occur?
f Using the above, write an equation that will model this function.
g Check the accuracy of your work by using your equation to find the temperature after 3,
9, 15 and 21 hours.
h What is the temperature at midnight? Give your answer correct to the nearest degree.
Check that your answer makes sense.

6H

Further graphs
Earlier in the book we discussed in detail how to graph the sum and difference offunctions. In
this section we will further apply these techniques to trigonometric functions.

Graphing the sum and difference of functions


The sum (and difference) function is defined over the intersection of the domains of the
individual functions. That is, dom (f(x) g(x)) = dom f(x) dom g(x).
To sketch the graph of the sum function, the method of addition of ordinates can be
used. This method involves sketching the graphs of the individual functions on the same
set of axes and then adding their y-coordinates. This technique is illustrated in the worked
example below.

Worked Example 26

Using addition of ordinates, sketch the graph of y = sin (x) + cos (x) for the
domain [0, 2].
Think

Draw

Method 1: Technology-free
1

Sketch the graphs of y = sin (x) and y = cos (x)


on the same set of axes over the required domain.
(Note that both y = sin (x) and y = cos (x) are
defined over [0, 2], so the sum function is
defined over the same domain.)

y
2
1
0
1

2 x

Chapter 6 Circular (trigonometric) functions

303

Moving from left to right, add the y-coordinates of


the two graphs and plot the resultant points. The
3
5 3 7
,
,
,
keypoints are at x = 0, , ,
4 2 4
4 2 4
and 2.

y
2
1
0
1

2 x

2
3

Join the points with a smooth curve.

y
2
1
0
1

2
4

Erase y = sin (x) and (y) = cos (x) to see the final
graph.

y
2
1
0
1

2 x

Method 2: Technology-enabled
On a Graphs page, complete the function entry
lines as:
f1(x) = sin (x)
f2(x) = cos (x)
f3(x) = f1(x) + f2(x)
Press ENTER after each entry.
Select an appropriate window.

Note that a difference function can be treated as a sum function where the second additive
is negative. For instance, the function y = sin (x) cos (x) can be viewed as y = sin (x) +
(cos (x)). So to obtain the graph of y = sin (x) cos (x), we can sketch the graphs of y = sin (x)
and y = cos (x) on the same set of axes and add their respective y-coordinates.

Graphing modulus (absolute value) functions


As was discussed previously, to sketch the graph of the modulus function y = | f(x)|, sketch the
graph of y = f(x) and then reflect all sections of the graph that are below the x-axis in the x-axis.
The technique is shown in the worked example below.

304

Maths Quest 12 Mathematical Methods CAS for the TI-Nspire

Worked Example 27

Sketch the graph of y = |3 cos (2x)| over the domain [0, 2].
Think

Draw

Method 1: Technology-free
1

First sketch the graph of y = 3 cos (2x). The


amplitude of this graph is 3 and its period is
2
= . Since the domain is [0, 2], we need
2
to sketch two full cycles.

y
3
2
1
0
1

2 x

2
3
2

Reflect all sections of the graph that are below


the x-axis in the x-axis.

y
3
2
1
0
1

2 x

2
3
3

Erase the sections below the x-axis to see the


final graph.

y
3
2
1
0
1

2 x

2
3

Method 2: Technology-enabled
On a Graphs page, complete the function entry
line as:
f1(x) = |3 cos (2x)|
Then press ENTER .
Select an appropriate window.

Chapter 6 Circular (trigonometric) functions

305

Graphing product functions


The product function is defined over the intersection of the domains of the individual functions.
That is, dom (f(x) g(x)) = dom f(x) dom g(x). Some features of the product function can be
established by using the following rules:
1. The x-intercepts of f(x) g(x) occur where either f(x) or g(x) have their x-intercepts.
2. f(x) g(x) is above the x-axis where f(x) and g(x) are either both positive, or both negative.
3. f(x) g(x) is below the x-axis where one of the functions f(x) or g(x) is positive and the other is
negative.
The general shape of the graph and the coordinates of the turning points can be found using a
CAS calculator.
Worked Example 28

Find the domain and sketch the graph of the product function y = x sin (x). Use a CAS calculator for
assistance.
Think

306

The required function can be viewed as


a product of two functions: f(x) = x and
g(x) = sin (x). The domain of the product
function is equal to the intersection of the
domains of the two individual functions.

On a Graphs page, complete the function


entry line as:
f1(x) = x sin (x)
Then press ENTER .
Select an appropriate window.

To locate the maximums and minimums,


press:
MENU b
5:Trace 5
1:Graph Trace 1
Trace along the graph to find a local
maximum.
Press ENTER to lock in the point.
This method can also be used to locate
minimums and intercepts.

write/Draw

Let f(x) = x, dom f(x) = R


Let g(x) = sin (x), dom g(x) = R
dom (f(x) g(x))
= dom f(x) dom g(x)
=R
So the domain of y = x sin (x) is R.

Maths Quest 12 Mathematical Methods CAS for the TI-Nspire

Of course the graphs of product functions are not limited to those involving trigonometric
functions.

Worked Example 29

If f(x) = 2x and g ( x ) = x + 1, sketch the graph of f ( x) g ( x ) = 2 x x + 1.


Think
1

write/Draw

Sketch the graphs of f(x) and g(x).

y = 2x
y= x+1

(0, 1)
x

(1, 0) 0

Find the domain of f(x) and the domain of g(x).

Dom f = R and dom g = [1, )

Find the domain of f(x)g(x).

Dom fg = [1, )

Find the x-intercepts of both f and g and hence find


the x-intercepts of the product fg.

x-intercept for f(x) is when x = 0 and f(x) = 0


x-intercept for g(x) is when x = 1 and g(x) = 0
Hence, the x-intercepts for the product are
when x = 0 and x = 1.

Find the values of x for which the product is


negative.

f(x) is negative and g(x) is positive for


x (1, 0), so fg is negative for x (1, 0).

Find the values of x for which the product is


positive.

f(x) and g(x) are both positive for x (0, ), so


fg is positive for x (0, ).

Find the turning point using a graphics calculator.


Round the answer to 2 decimal places as
appropriate.

The turning point is (

Sketch the graph of the product.

, 0.77).

y = 2x x + 1

(1, 0)
(0, 0)
3
(
,
3

0.77)

Graphing composite functions


A composite function is formed from two functions in the following way. If f(x) = x + 5 and
g(x) = 2x are two functions, then we combine the two functions to form the composite function
g(f(x)) = 2f(x) = 2(x + 5). That is, f(x) replaces x in the function g(x).

Chapter 6 Circular (trigonometric) functions

307

The composite function reads g of f and can be written g f.


Another composite function is f (g(x)) = g(x) + 5 = 2x + 5. In this case, g(x) replaces x in f (x).
This composite function reads f of g and can be written f g.
For the composite function f (g(x)) to be defined, the range of g must be a subset of the
domain of f. Furthermore, if f (g(x)) is defined, the domain of f (g(x)) equals the domain of g(x).
Composite functions can be rather complex to graph by hand, so a CAS calculator can be
used for assistance when sketching.
Worked example 30

eBook plus

For the pair of functions f(x) =cos (x) and g( x ) = x :


a show that f(g(x)) is defined
b find f(g(x)) and state its domain
c sketch the graph of f(g(x)), using a CAS calculator for assistance.
Think
a

Tutorial

int-0551
Worked example 30

WriTe/draW

For f(g(x)) to exist, the range of g must be a


subset of the domain of f. So find both the
range of g and the domain of f to show that
this condition is observed.
1

Form the composite function f (g(x))


by substituting g(x) into f (x).

The domain of f (g(x)) must be the


same as the domain of g(x). Since the
domain of g(x) is R+{0},so is the
domain of f (g(x)).

On a Graphs page, complete the function


entry line as:
f 1( x ) = cos( x )
Then press ENTER .
Select an appropriate window.

a f (x) = cos (x); domain of f (x) = R

g( x ) = x : range of g(x) = R+{0}


Range of g(x) domain off(x)
f (g(x)) is defined

b f ( g( x )) = cos ( x )

Domain of f (g(x)) =R+{0}

The graphs of composite functions are not limited to those involving trigonometric functions.
This is demonstrated in the following example.
Worked example 31

For f(x) =x2 2 and g(x) =x4:


a show that both f(g(x)) and g(f(x)) are defined
b find both f(g(x)) and g(f(x)), stating the domain and range of each one
c on separate axes, sketch the graphs of f(g(x)) and g(f(x)).

308

maths Quest 12 mathematical methods CaS for the Ti-nspire

Think

Write/draw

Show that ran f dom g and that ran g dom f.

a Dom f = R; ran f = [2, ).

Write f(x).

b f(x) = x2 2

Form the composite function f(g(x)) by


substituting g(x) into f(x). Simplify.

f(g(x)) = (x4)2 2
= x8 2

State the domain and range of f(g(x)).

Domain = R, range = [2, ).

Form the composite function g(f(x)) by


substituting f(x) into g(x).

g(f(x)) = (x2 2)4

State the domain and range of g(f(x)).

Sketch f(g(x)), rounding x-intercepts to


2decimal places.

Dom g = R; ran g = [0, ].


Range of f domain of g, so g(f(x))
exists.
Range of g domain of f, so f(g(x))
exists.

Domain = R, range = [0, )

c The y-intercept is 2, and the x-intercepts

are 1.09 and 1.09.


y

y = x8 2
(1.09, 0) 0

Sketch g(f(x)).

(1.09, 0)
x
(0, 2)

The y-intercept is 16, and the turning


point is (2, 0).
y
(0, 16)
y = (x2 2)4
0

(2, 0) x

REMEMBER

1. For the sum/difference function, dom (f(x) g(x)) = dom f(x) dom g(x). The graph
of the sum/difference function can be obtained by using the addition of ordinates
method.
2. The graph of the modulus function, y = | f(x)|, can be obtained by sketching the graph
of y = f(x) and then reflecting all of the sections of the graph that are below the x-axis in
the x-axis.
3. For the product function, dom (f(x) g(x)) = dom f(x) dom g(x). Some features of the
graph of the product function are as follows:
(i) the x-intercepts of f(x) g(x) occur where either f(x) or g(x) have their x-intercepts

Chapter 6 Circular (trigonometric) functions

309

(ii) f (x) g(x) is above the x-axis where f (x) and g(x) are either both positive or both
negative
(iii) f (x) g(x) is below the x-axis where one of the functions f (x) or g(x) is positive and
the other is negative.
4. For the composite function f (g(x)) to be defined, the range of g must be a subset of the
domain of f. Furthermore, if f (g(x)) is defined, the domain of f (g(x)) equals the domain
of g(x).
exerCiSe

6h
eBook plus

Further graphs
1

Digital doc

SkillSHEET 6.6
Addition of
ordinates

We26
Using addition of ordinates, sketch the following graphs for the domain [0, 2].
a y = sin (x) + cos (2x)
b y = cos (x) + sin (2x)
c y = 2 sin (x) + cos (x)
d y = 2 cos (x) + sin (x)
e y = 2 sin (x) + cos (2x)
f y = 2 cos (x) + sin (2x)
g y = 2 sin (2x) + cos (x)
h y = 2 cos (2x) + sin (x)

2 Apply the addition of ordinates method to sketch each of the following graphs over the domain
[0, 2]. Use a CAS calculator to check your answers.
a y = sin (x) +x
b y = cos (x) x
c y = 3 sin (x) 2x
1

d y = cos (2 x ) + 4 x 2

e y = 2 sin (4 x ) 8 x 3

g y = 4 sin (x) 5 loge (x +1)

h y = 3 cos (2 x )

f y = tan (x) 2x2

20
( x + 2)2

We27 Sketch each of the following graphs over the domain [0, 2].

a y =|sin (2x)|

b y = |2sin (4x)|

1
c y = cos x
2

d y = |3 cos (3x)|

e y = |2tan (x)|

f y = |4sin (x) + 2|

g y = |1 2 cos (2x)|

h y = |tan (2x) + 3|

4 Sketch each of the following graphs over the domain [0, 2]. Remember to observe the
appropriate order of transformations.
x
a y =3|sin (x)|
b y = |2cos (2x)|+ 1
c y = 2 cos + 3
2
5

We28 Find the domain and sketch the graph of each of the following product functions. Use
a CAS calculator for assistance.

a y = 0.5x sin (x)

b y = (x 1) cos (x)

c y = 3 sin (x) loge (x)

d y = 2 cos ( x ) x

e y = 8 cos (x) sin (x)

f y = (4 2x) sin (2x)

g y =(1.2)x cos (x)

h y = sin

d f (x) = |x |,g(x)

=x2

g f ( x ) = x , g( x ) = 1 x

310

2 x

We29 For each of the following functions f (x) and g(x), sketch the graph of f (x)g(x).

a f ( x ) = x, g ( x ) = x + 2

x

2

b f (x) = x 2, g(x) =ex

c f (x) = x 1, g(x) =loge (x)

e f ( x ) = x , g( x ) =

f f (x) = 1 x2, g(x) =ex

x+2

h f (x) = x2, g(x) =loge (x)

We30 For each of the following pairs of functions:

i show that f (g(x)) is defined


ii find f (g(x)) and state its domain
iii sketch the graph of f (g(x)), using a CAS calculator for assistance.

maths Quest 12 mathematical methods CaS for the Ti-nspire

a f(x) = cos (x) and g(x) = loge (x)


x2
c f ( x ) = 2 sin ( x ) and g( x ) =
4
e f(x) = x2 and g(x) = sin (x)
g f ( x ) = cos

x

4

b f ( x ) = sin (2 x ) and g( x ) = x
d f ( x ) = x + 2 and g( x ) = 2 cos ( x )
f f(x) = 2x and g(x) = cos (2x)

and g( x ) = 2 x 2

h f ( x ) = 2 sin ( x ) + 1 and g( x ) = x 3

8 WE31 For each of the following pairs of functions f(x) and g(x):

i state whether f(g(x)) and g(f(x)) are defined
ii for the composite functions that are defined, find f(g(x)) and g(f(x)), stating the
domain and range of each one
iii on separate axes, sketch the graphs of f(g(x)) and g(f(x)) that are defined.

6I

a f(x) = x 2, g(x) = ex

b f(x) = |x| g(x) = x2 1

c f(x) = 1 x2, g(x) = ex

d f ( x ) = x , g( x ) = sin ( x )

e f(x) = ex, g(x) = cos (x)

f f(x) = logex, g(x) = sin (x)

Trigonometric functions with an


increasing trend
Consider a trigonometric function where there is an increasing trend, for example, economic
growth cycles, tidal heights due to global warming or increasing seasonal populations. These
situations can be modelled by a function of the form
y = ax + b + m sin (nx)
where ax + b represents the increasing trend line and m sin (nx) represents the seasonal variation.

Worked Example 32

Consider a remote island where global warming has caused the temperature to increase by
0.1degree each month. The mean daily temperature is modelled by the function

T ( m) = 16 + 0.1 m + 6 cos m , where T is the temperature in degrees Celsius and m is the number
6
of months after January 2008.
a Sketch a graph of the function for a five year period from January 2008, using a CAS calculator
for assistance.
b Find the mean daily temperature for March 2009
c When will the mean daily temperature first reach 23 degrees?
Think

Write

a On a Graphs page, complete the function entry

line as:

f 1( x ) = 16 + 0.1x + 6 cos x
6
Then press ENTER .
Select an appropriate window:
XMin: 0
XMax: 60
YMin: 10
YMax: 30

Chapter 6 Circular (trigonometric) functions

311

March 2009 occurs when m = 14.


On a Calculator page, complete the entry
line as:
f1(14)
Then press ENTER .

The mean daily temperature in March 2009 would


be 20.4 degrees Celsius.
c


To solve 23 = 16 + 0.1m + 6 cos m ,
6
for m, return to the Graphs page.
Complete the function entry line as:
f2(x) = 23
Then press ENTER .

Press:
MENU b
7:Points & Lines 7
3:Intersection Point(s) 3
Select each function and then press
ENTER .
The points of intersection will appear.

m = 11.5631
Hence, the first time the temperature reaches
23 degrees Celsius will be during the 12th month
after January 2008. That is, during January 2009.
REMEMBER

Trigonometric functions with an increasing trend can be modelled by a function of


the form y = ax + b + m sin (nx) where ax + b represents the increasing trend line and
m sin (nx) represents the seasonal variation.

312

Maths Quest 12 Mathematical Methods CAS for the TI-Nspire

Exercise

6I

Trigonometric functions with an increasing trend


1 WE32 A fisherman finds himself stranded on an island, where the mean daily temperature is
increasing as a result of global warming. He finds that the temperature can be modelled by the

function T (m) = 12 + 0.2m + 5 cos m , where T is the temperature in degrees Celsius and
6
m is the number of months after January 2008.
a Sketch a graph of the function for a 2-year period beginning with January 2008.
b Find the mean daily temperature for December 2009.
c When will the temperature first reach 18 degrees?
2 In a region of country Victoria, a study shows that increased wheat production causes the

mouse population to increase according to the function M = 15 000 + 100t 4000 cos t ,
6
where M is the number of mice and t is the number of months after July 2008.
a How many mice are being added to the average population per month?
b Draw a graph of this situation for a 5-year period beginning with July 2008.
c How many mice would you expect to be in the region in December 2008?
d When would the mice population first reach 20 000?
3 The value of a particular stock on the market follows a trigonometric model, and inflation
causes the stocks value to have an overall upward trend. The value of the stock can be

represented by the equation V (t ) = 20 + 0.02t + 5 sin t , where V is the value of the stock
6
in dollars and t is the number of months after January 2006.
a What is the inflation rate per month?
b What was the initial value of the stock?
c What will be the stocks value after 6 months?
d When will its value first reach $25.50?

Chapter 6 Circular (trigonometric) functions

313

Summary
Revision of radians and the unit circle

1c = the size of the angle formed where the length of an arc is equal to the radius of the circle.
c = 180
Angles are in radians unless a degree symbol is shown.
180
To change radians to degrees, multiply by
.

To change degrees to radians, multiply by


.
180
Identities
1. sin2 () + cos2 () = 1
sin ( )
2. tan ( ) =
cos ( )

Symmetry and exact values

Exact values can be determined


by using the equilateral triangle
and the right isosceles triangle
shown at right.

3 30

45

45

60
1

1
3
2

0 (0)

30 6

45 4

60 3

90 2

180 ()

sin ()

1
2

2
2

3
2

cos ()

3
2

2
2

1
2

tan ()

3
3

Undef.

Undef.

360 (2)

270

The unit circle is symmetrical so that the magnitude of sine, cosine and tangent are the same in each
quadrant but the sign varies. All functions (sine, cosine and tangent) are positive in the 1st quadrant, sine is
positive in the 2nd quadrant, tangent is positive in the 3rd quadrant and cosine is positive in the 4th quadrant.
sin ( ) = sin ()

sin ( + ) = sin ()

sin (2 ) = sin ()

cos ( ) = cos ()

cos ( + ) = cos ()

cos (2 ) = cos ()

tan ( + ) = tan ()

tan (2 ) = tan ()

tan ( ) =

tan

()

Negative angles
sin () = sin (), cos () = cos (), tan () = tan ()

Complementary angles add to 90o or radians.


2
The sine of an angle is equal to the cosine of its complement.
The complement of the tangent of an angle is the cotangent.

( ) = cos ( )
cos ( ) = sin ( )
tan ( ) = cot ( )
sin

314

( + ) = cos ( )
cos ( + ) = sin ( )
tan ( + ) = cot ( )
sin

(
cos (
tan (
sin

Maths Quest 12 Mathematical Methods CAS for the TI-Nspire

3
2
3
2
3
2

)
) = sin ( )
) = cot ( )

= cos ( )

sin

3
2

(
tan (
cos

+ = cos ( )

3
2

3
2

+ = sin ( )

+ = cot ( )

Trigonometric equations

Given a general equation such as sin x = a, there can be an infinite number of solutions. The domain is
usually restricted and it is important to find all values for x within this domain.
If the domain is given in radians, then the solution(s) to x should be in radians. If the domain is given in
degrees, then the solution(s) to x should be in degrees.
Adjust the domain to match what has been done to the angle in the question.
Sine is positive in the 1st and 2nd quadrants, cosine is positive in the 1st and 4th quadrants and tangent is
positive in the 1st and 3rd quadrants.
If sin (x) = a then the general solution is x = sin 1(a) + 2np, n Z.
If cos (x) = a then the general solution is x = cos 1(a) + 2np, n Z.
-1
If tan (x) = a then the general solution is x = tan (a) + np, n Z.
Find all the solutions within the specified domain by substituting integer values for n into the
general solution.
If the equation is of the form sin (ax) = k cos (ax), divide both sides by cos (ax) to change the equation to
tan (ax) = k.
Trigonometric graphs

Sine and cosine graphs


Graphs of the form y = a sin [n(x b)] + c and y = a cos [n(x b)] + c are transformations of y = sin (x)
and y = cos (x).
The amplitude a is a dilation from the x-axis. If a is negative, the amplitude is still positive but the graph is a
reflection in the x-axis.
The vertical translation c is a translation parallel to the y-axis. If c is positive, the graph is translated c units
up, and if c is negative, the graph is translated c units down.
c represents the median value of the function.
The range is [c | a |, c + | a |].
2
The period is .
n
1
The factor n is the horizontal dilation where the graph has been dilated by a factor of from the y-axis.
n
The value b is the horizontal translation or a translation parallel to the x-axis. If b is positive, the graph is
translated b units to the right and if b is negative, the graph is translated b units to the left.
Tangent graphs

y
x = 32 x = 2
x = 2 x = 32
The graph of y = tan (x) has the following properties.
3
It has no amplitude.
2
The period is .

1
There are x-intercepts at x = ..., 2, , 0, , 2, ...
3 3
, , ,
,...
There are vertical asymptotes at x = . . .,
0 3
2 32 2
2 x
2
2
2 2 2 2
1
The range is R.
2
The graph of y = tan (nx) has the following properties.
It has no amplitude.
3

The period is .
n
k
There are x-intercepts at x = where k = 0, 1, 2, ...
n
(2k + 1)
There are vertical asymptotes at x =
where k = 0, 1, 2, ...
2n
The range is R.
Compared to the basic graph of y = tan (x), the graph of y = a tan [n(x b)] + c is:
dilated by the factor of a from the x-axis
1

dilated by the factor of from the y-axis (and hence has period of )
n
n

Chapter 6 Circular (trigonometric) functions

315

reflected in the x-axis if a < 0


translated b units horizontally (to the right if b > 0 and to the left if b < 0)
translated c units vertically (up if c > 0 and down if c < 0).
Further graphs

For the graph of the sum/difference function, dom(f(x) g(x)) = dom f(x) dom g(x). The graph of the sum/
difference function can be obtained by using the addition of ordinates method.
The graph of the modulus function y = |f(x)| can be obtained by sketching the graph of y = f(x) and then
reflecting all of the sections of the graph that are below the x-axis in the x-axis.
For the product function, dom(f(x)g(x)) = dom f(x) dom g(x). Some features of the graph of the product
function are as follows:
the x-intercepts of f(x)g(x) occur where either f(x) or g(x) have their x-intercepts
f(x)g(x) is above the x-axis where f(x) and g(x) are either both positive or both negative
f(x)g(x) is below the x-axis where one of the functions f(x) or g(x) is positive and the other is negative.
For the composite function f(g(x)) to be defined, the range of g must be a subset of the domain of f.
Furthermore, if f(g(x)) is defined, the domain of f(g(x)) equals the domain of g(x).
Trigonometric functions with an increasing trend

These situations can be modelled by a function of the form y = ax + b + msin (nx), where ax + b represents
the increasing trend line and msin (nx) is the seasonal variation.

316

Maths Quest 12 Mathematical Methods CAS for the TI-Nspire

chapter review
Short answer

1 Convert the following into radians.


a 60
b 30
d 90
e 360
g 150
h 300
j 120
k 210

c
f
i
l

45
270
225
315

2 Find the value of:


2
a cos
3
5
d sin
4
g cos (135)
j tan (225)

b sin
4

7
c tan
6

e tan (2)

f sin (120)

h tan (30)

i cos (315)

9 The depth, d metres, of water in a shallow bay at


t
thours after 9 am is given by d = 5 + 3 sin ,
6
for 0 t 24 hours.
a Sketch the graph of d for 0 t 24 hours.
b At what times will the depth of water in the
bay be 6.5 metres?
c Particular water sports cannot run in the bay
when the depth of water is less than 6.5 metres.
At what times of the day (not night) will the
water sports be able to run?

3 If sin (x) = 0.85 and x is in the first quadrant, find:


b sin ( + x)
a sin ( x)
c sin (2 + x)
d sin (4 x)
4 Find the general solution to the equation
2 cos ( x ) 2 = 0.
5 Find the general solution for 3 sin (2 x ) = cos(2 x )
and hence all solutions for x between 0 and 2.

6 Consider the graph of y = 3 sin x + 2.

4
a State the translations required to form this
graph from y = sin (x).
b State the amplitude and period of the
transformed trigonometric function.
c Sketch the graph of the transformed function
over the domain 0 x 2.
7 State the period and sketch the graph of
y = tan (2x) + 2.
8 The equation of the following graph is of the
form y = a sin (nx) + b. Find the values of
a, n and b, and hence find the equation of the
function.
y
3
2
1

1
2
3
4

10 Sketch the graph of y = |2 sin (2x)| over the domain


[0, 2].
11 On the same set of axes and over the domain
[0, 2], sketch the graphs of:
a y = 2x
b y = sin (x)
c y = sin (x) + 2x.
12 On the same set of axes and over the domain
[0, 2], sketch the graphs of:
a y = x
b y = sin (x)
c y = x sin (x).
13 For the pair of functions f(x) = sin (x) and
g(x) = x2 + 2:
a show that f(g(x)) is defined
b find f(g(x))
c state the domain and range of f(g(x)).
14 For the function

f : [ , ] R, f ( x ) = 5 cos (2( x + )):


3
a Write down the amplitude and period of the
function.

Chapter 6 Circular (trigonometric) functions

317

b Sketch the graph of the function f. Label


intercepts with their coordinates. Label end
points of the graph with their coordinates.
Exam tip Make sure that you use the period and

amplitude found in part a to help sketch the graph.


The graph should be a smooth symmetrical curve with all
intercepts (x and y) clearly labelled with coordinates.
[Assessment report 1 2006]

[ VCAA 2006]

Multiple choice

1 What is 320 expressed in radians?


A 2
B 5
C 8
D 4
9
18
9
9
c
13
2 What is
expressed in degrees?
6
A 390 B 420 C 150 D 120

E 16
9

E 330

5
3 The expression sin equals:
3
A

1
2

1
2

3
2

3
2

E 1

4 A trigonometric function is given by


f : R R, f(x) = 3 cos (2x + ) 1
The amplitude, period, and range of f are
respectively:
C 3, , [4, 2]
B 2, , R
A 3, , [ 4, 2]
2

E 3,
D 2, , [ 1, 3]
2

8 The domain and range of the function


f : [0, ] R, f ( x ) = 4 sin 3 x + 2 is:

B [0, ], [2, 6]
C R, R
A [0, ], R

D [ , 2 ] [ 4, 4] E [0, ], [2, 4]
3
9 Using addition of ordinates for the graph of the
difference function y = 2 cos (x) 3 sin (x), the value

of y at x = is:
3
1
1+ 3 3
2+3 3
A
B
C
3
2
2
D

1 3 3
2

6 The function f : [, ] R,
f (x) = sin (2x) + cos (2x) has the following
x-intercepts:
5 3 7
3
3
,
, ,
A
, B
, C
4
4 4 4
2 4
8 8
5 3 7

,
, ,
,
D
E
8
8 8 8
4 4
7 The solution of the equation 4 sin ( x ) = 2 3
3
between and
is:
2
A
B
C 4
6
3
3
7

D
E
3
3

318

23 3
2

x
10 The period of the graph of y = tan is:
3

B
C 2
D 3
A
3

E 6

11 The graph of the function f : [1.5, 1.5] R,


f(x) = a + b cos ( nx) is shown below. The values
of a, b, and n are respectively:
y

max = 3.5

3
2
1
1.5

1.0

0.5

0
1

0.5

1.0
1.5
min = 1.5

5 When f(x) = 0 the following function


f : [0, 2] R, f(x) = 2 sin 2 ( x + ) + 1 has:

A 0 solutions
B 1 solution
C 2 solutions
D 3 solutions
E 4 solutions

A 1,
D 1,

5
2
5
3

,3

B 1, 3 , 2

,3

1,

5
3

C 1, 5, 3

,2

12 If 0 c , the x-intercepts of the function


f : [0, 2] R, f (x) = b sin (x c) are:
B , 2
C c, c
A c, 2 c
D + c, 2 c
E c, + c
13 The position of a particle from a fixed point O is
given by the equation x = 2 2 sin ( t). If 0 t
2, the particle is at the point O when t equals:
E 6
A 0
B 2
C 0.5
E 6
14 The smallest positive value of x for which
tan (2x) = 1 is:

A 0
B
C
D
8
4
2

Maths Quest 12 Mathematical Methods CAS for the TI-Nspire

E p

[ VCAA 2006]

Extended response

1 At the South Pole in midsummer on the planet Marus, the red sun of its solar system does not set. It dips
towards the horizon until its lower rim just touches it, then rises until its lowest point is at an angle of D(t)
to the horizontal before sinking again. It continues in this pattern. The angle above the horizontal can be
modelled by the following relation:
D(t) = a b sin n(t + c) where t is the time in hours after midnight and a, b, c and n are positive constants.
The graph of D(t) for 24 hours is shown on the axes in the figure below.
D(t)
10
8
6
4
2
0

2 4 6 8 10 12 14 16 18 20 22 24

a State the values of a, b, c and n and hence write the rule for D(t).
b What would be the angle above the horizon at 6.00 am and at 9.00 pm? Give an exact answer where
possible, otherwise give your answer correct to 2 decimal places.
c Use your graph to find at what times the angle to the horizontal is 8. When does the rim of the sun reach
this angle again?
d By using an appropriate equation, check your answer and account for any difference in your two
solutions.
e If a spot on the surface of the sun is 5 above the horizon at midnight, what would be the relation G(t)
that models its path?
2 Nathan and Rachel are competing in the National Ballroom Dancing Championships. The judges are evenly
spaced around the circular dance floor, standing just outside the edge. As Nathan and Rachel waltz around the
circular floor, their distance (in metres) from judge Maya can be described by the function

d = 10.5 9 cos t , where t is time (in seconds) from the beginning of the dance.
30
a How far is the couple from judge Maya when they start dancing?
b What is the couples maximum distance from the judge?
c Assuming that, while dancing, Rachel and Nathan trace a perfect circle, what is its diameter?
d How long does it take for the couple to complete one full circle around the dance floor?
e What is the couples average speed (in m/s)? Give your answer i in exact form and ii correct to 2 decimal
places.

f If the duration of the waltz is 2.5 minutes, draw the graph of d = 10.5 9 cos t over the domain,
30
showing the full length of this dance.
g Judge Joseph is positioned further down the dance floor, so that Nathan and Rachel are closest to him
6seconds after the waltz begins. Write the equation describing the couples distance from judge Joseph
at any time, t, from the beginning of the dance.
h How far is the couple from judge Joseph when they finish the waltz?
3 Tasmania Jones is attempting to recover the lost Zambeji diamond. The diamond is buried at a point 4 km into
Death Gorge, which is infested with savage insects. In order to recover the diamond, Tasmania will need to
run into the gorge, dig up the diamond and return the same way that he came.
The concentration of insects in the gorge is a continuous function of time. The concentration, C, of
insects per square metre is given by

(t 8)
+ 2)2 1000, 8 t 16
1000 (cos
C (t ) =
2
m
0 t < 8 or 16 < t 24

where t is the number of hours after midnight and m is a real constant.

Chapter 6 Circular (trigonometric) functions

319

a What is the value of m?


b Sketch the graph of C for 0 t 24.
c What is the minimum concentration of insects and at what value(s) of t does that occur?
The insects infecting the gorge are known to be deadly if their concentration is more than 1250 insects
per square metre.
d At what time after midnight does the concentration of insects first stop being deadly?
e During a 24-hour period, what is the total length of time for which the concentration of insects is less
than 1250 insects per square metre?

eBook plus
Digital doc

Test Yourself

exam Tip

Be aware of the word continuous when working out what value(s) to substitute to find m.
When drawing the graph, clearly show minimum turning points and be aware of the domain
stated in the question.

Chapter 6

[Assessment report 2 2007]

[ Vcaa 2007]

320

maths Quest 12 mathematical methods CaS for the Ti-nspire

eBook plus

aCTiviTieS

chapter opener
Digital doc

10 Quick Questions: Warm up with ten quick


questions on circular functions. (page 258)
6a

Revision of radians and the unit circle

Digital docs

Spreadsheet 142: Investigate the unit circle.


(page 263)
SkillSHEET 6.1: Practise changing degrees to
radians. (page 263)
SkillSHEET 6.2 : Practise working with tangent
ratios. (page 263)
6b

Symmetry and exact values

Tutorials

We 3 int-0546: Watch a worked example on exact


values using degrees and radians. (page 265)
We 5 int-0547: Watch a worked example on
determining trigonometric ratios. (page 266)
We 8 int-0548: Watch a worked example on
complementary angle formulas. (page 270)
Digital docs

SkillSHEET 6.3: Practise rationalising the


denominator. (page 271)
Spreadsheet 142: Investigate the unit circle.
(page 272)
SkillSHEET 6.4: Practise problem solving using
trigonometry. (page 273)
6c

Trigonometric equations

Tutorial

We 13 int-0549: Watch a worked example on


determining the general solution to a trigonometric
equation. (page 277)
Digital docs

Spreadsheet 135: Practise solving trigonometric


equations. (page 281)
WorkSHEET 6.1: Use exact values, solve
trigonometric equations and application questions.
(page 282)
6D

Trigonometric graphs

Digital docs

SkillSHEET 6.5: Practise identifying the period and


amplitude of sine and cosine graphs. (page 288)
Spreadsheet 124: Investigate sine graphs. (page 288)
Spreadsheet 012: Investigate cosine graphs.
(page 289)
6E
Tutorial

Graphs of the tangent function

6F

Finding equations of trigonometric graphs

Interactivity int-0251

Finding equations of trigonometric graphs: Use the


interactivity to consolidate your understanding of
trigonometric graphs. (page 295)
Digital docs

Spreadsheet 124: Investigate sine graphs. (page 298)


Spreadsheet 012: Investigate cosine graphs.
(page 298)
WorkSHEET 6.2: Sketch trigonometric graphs over
given domains, including composite and absolute
value graphs, and determine the equations of
trigonometric graphs. (page 299)
6G

Trigonometric modelling

eLesson eles-0092

Trigonometric modelling: Learn how trigonometry


can be used to model sinusoidal patterns. (page 299)
Digital doc

Spreadsheet 041: Investigate graphs of functions.


(page 301)
6H

Further graphs

Tutorial

We 30 int-0551: Watch how to sketch a composite


function using a CAS calculator. (page 308)
Digital doc

SkillSHEET 6.6: Practise using addition of ordinates


to sketch graphs. (page 310)
chapter review
Digital doc

Test Yourself: Take the end-of-chapter test to test


your progress. (page 320)
To access eBookPLUS activities, log on to
www.jacplus.com.au

We 21 int-0550: Watch a worked example on


calculating the period of trigonometric functions.
(page 293)

Chapter 6

Circular (trigonometric) functions

321

EXAM PRACTICE 2
Short answer

30 minutes

1 Solve the equation log2 (2x - 3) - log2 (3) = 1 for x.



1 mark
2 Solve the equation 32x + 3 33 = 9 for x.


1 mark

[-2,0]

3 The graph of the function f :


R where
f(x) = ex is reflected in the y-axis, translated 2 units
to the left then translated 3 units up. (Note the
change to the domain.)
a Specify the rule of the transformed graph.
b Determine its exact range.

2 marks
4 The diagram below shows the graph of y = sin (x)
and a second graph formed by transforming
sin (x) by carrying out dilation(s) followed by
translation(s).
y
1

0 /6

y = sin (x)

a Write down the transformations (type, direction


and quantity) necessary to create the second
graph.
b Write down the equation of the second graph.
4 marks

5 Find the exact solutions of the equation


sin (3x) - cos (3x) = 0 for 0 x


2 marks

6 Specify the range of the function


f :[


) R, f ( x ) = 2 tan(2 x ) + 1.
4 8
4
,

2 marks

7 Solve the equation




e2x - ex

= 2 exactly for x.
2 marks

1,

8 For f : [
3] R, where f(x) = x sin (x):
a sketch f
b determine the minimum value of f(x)
c estimate the maximum value of f(x) to
1 decimal place.


322

Chapters 1 TO 6

Multiple choice

12 minutes

Each question is worth 1 mark.


1 If y = 2abx - 1 + 5, then x is equal to:
y5
y
log a ( 5) + 1
a 2 +1
2
B
A
b
b
y5
+1
y5
2
+b
C
D
2b
b
y5
log a (
) +1
2
E
b
2 The solution set of the equation e2x - 3ex = 4 over
R is:
A {1, 4}
B {4}
C {loge (4)}
D {loge (-1), loge (4)}
E {loge (1), loge (4)}
3 The graph of the function
f :[2, 5] R, f(x) = loge (x) is reflected in the
y-axis, translated 2 units to the left then translated
3 units up. The domain of the new graph is:
A [7, 1]
B [4, 1]
7, 4]
C [
D [loge (-7), loge (-4)]
E not defined
4 The function f : [1, ) R, where f(x) = x2 + 1 has
as its inverse function:
A f-1: R R, where f 1( x ) = x 1
B f-1: [2, ] R, where f 1( x ) = x 1
1
C f-1: [-1, ] R, where f 1 ( x ) = 2
x +1
D f1: R+ {0} R, where f 1( x ) = x 1
1
1
E f 1: [ , ) R, where f 1( x ) = 2
2
x +1
5 The equation a sin (3) - b = 0 has = 2 as one
solution. Which of the following could also be a
solution?
A 3 + 6
B + 2
2
C -2
D
+2
3
2
E
+6
3

4 marks

Maths Quest 12 Mathematical Methods CAS for the TI-Nspire

7 The graph of y = 3 cos (2x) 1 is shown below. If a


and b are two adjacent x-intercepts, b a is equal to:

6 The graph shown could be the graph of:


y
3a

0
a

b
x

2
cos
+a
T

T
+a
a 2a cos
2

2
c a (cos
+ 1)
T

D 2a sin (

2a

( )) + a
T
2


T
E a (sin
( ) + 1)
2
2

2
c 3a
E 2a

D 2 a

exTended reSponSe

20 minutes

Bird population

An extensive study has been made of the population of silver-banded lorikeets in an area proposed for a
wind farm. The variation in estimated numbers over a number of months is recorded in the graph below. It
is suggested that the relationship could be modelled by a sine function of the form P(t) = a sin (b(t + c)) + d,
where P is the bird population and t is the time in months since estimates commenced.
y
500
400
300
200
100
0

a
b
c
d
e
f
g

4 5 6
Months

9 x

What is the amplitude of the sine model (to the nearest 10 birds)?
1 mark
What is the period (to the nearest month)?
1 mark
What is the mean population (to the nearest 10 birds)?
1 mark
After how many months is the population a minimum?
1 mark
Determine the values of a, b, c and d in the model P(t) = a sin b(t + c) + d.
4 marks
What was the initial bird population when the observations commenced?
1 mark
A second researcher conducts observations over a 10-year period at the same time each year and records the
following estimates for the population.
Time (months)

Population

500

60

370

120

274

She believes that the population can be modelled by an exponential function of the form
Q(t) = Aekt, where t is the time in months since the first estimate was calculated. Determine the
values of A and k.
2 marks
h Explain how both researchers results are consistent with the model
eBook plus
P( x )Q( x )
Population =
.
3 marks
Digital doc
500
Solutions
Exam practice 2

exam practice 2

323

7a Review gradient and rates of change


7b Limits and differentiation from first
principles
7c The derivative of xn
7d The chain rule
7e The derivative of e x
7F The derivative of loge (x
(x)
7G The derivatives of sin ((x), cos ((x) and tan ((x)
7H The product rule
7I The quotient rule
7J Mixed problems on differentiation

Differentiation
AREAS OF STUDY

Graphical treatment of limits, continuity and


differentiation of functions of a single real
variable
Deducing the graph of a derivative function
from the graph of a function
Derivatives of xn, n Q, ex, ln (x), sin (x),
cos (x), tan (x)
Properties of derivatives
(af
af (x) bg(x)) = af (x) bg(x), a, b R

f ( x)
, and
g( x )
f (g(x)) where f and g are polynomial functions,
exponential, circular, logarithmic or power
functions (or combinations of these functions)

Derivatives of f (x) g(x), f (x) g(x),

such as x 5 + 1 x 2 , x sin (2 x ), e cos ( x )


2
and ln ( x + 4)
x

eBook
eBoo
k plus

7A

Review gradient and


rates of change

Digital doc

10 Quick Questions

Gradient is a measure of how one quantity changes with respect to another quantity; in other
words, the rate of change of one quantity with respect to another quantity. The gradient
(or slope) is the measure of how the vertical distance (the rise) changes with respect to the
horizontal distance (the run). Speed is the measure of how distance travelled changes with
respect to the time taken; another way of saying this is that speed is the rate of change of
distance with respect to time.

Constant rates
When the rate of change of one quantity with respect to another quantity
does not change, the rate is constant. If the rate of pay for babysitting is
fixed at $15 an hour, this is a constant rate. This means that a babysitter
would be paid $15 for 1 hour and $150 for 10 hours. A constant rate can be
represented graphically by a straight line.

324

Maths Quest 12 Mathematical Methods CAS for the TI-Nspire

y
(1, 15)

(0, 0) x

Average rates
An average rate of change is the rate of change over a period of time. A
car travelling at 60 km/h is probably not actually registering that speed at
every moment of the distance travelled; more likely, its speed varies above
and below 60 km/h. The average rate of change between two points can be
represented by the gradient of a straight line joining the two points.

(0, 0)

Instantaneous rates

(1, 0) x
(1_2, 33_4)

(2, 3)

If the police want to find how fast a car is travelling, they measure
the speed at a particular instant. This is known as instantaneous
(1, 0) (1, 0)
rate of change. Graphically, thisis found by drawing a tangent
x
0
to the curve at a particular point and finding the gradient of the
(3, 0)
tangent. If a section of the graph is increasing, the gradient of the
tangent is positive, and if a section of the graph is decreasing, the
(0.15, 3.08)
gradient of the tangent is negative.
For example, for the curve y = (x 1)(x + 1)(x + 3), the gradient is positive when x<2 and
when x > 0.15; the gradient is negative when 2 < x < 0.15; and the gradient is zero when x = 2
and x = 0.15.

Graphing the gradient function from the graph of a function


Differentiation is the process of calculating the gradient function from a given function. It can be
used to find the gradient of a curve at a particular point and to find the maximum and minimum
values of a function.
y
f(x)
Derivatives can be found at a given point if:
There is a break in the
1. the function is continuous at that point
graph at x = 1(discontinuous),
2. the function is smooth at that point.
so gradient does not exist
at x = 1. That is, gradient
A function is continuous if there are no
exists for R\{1}.
breaks, jumps or asymptotes on its graph.
x
0
1
The gradient does not exist where the function
is not continuous. An example is shown at right.
A function is smooth if there are no sharp
points on its graph.
The gradient does not exist where the function is
y
not smooth. An example is shown at right.
f (x) is not smooth at x = 2
so gradient does not exist
at x = 2. That is, gradient
exists for R\{2}.

2
f(x)

The gradient of a function exists wherever the function is


smooth and continuous. That is, the gradient of a function
exists at a point providing only one tangent can be drawn at
that point. An example is shown at right.
A function is not differentiable where there is a gap, hole
or asymptote, a sharp corner or an end point.

y
f(x)

2
2

The gradient of f(x)


exists for x R as
f(x) is smooth and
continuous.
x

Chapter 7 Differentiation

325

Gradient function of straight lines


In general, for straight lines in the form f (x) = mx + c (or y = mx + c), the rule for the gradient is
f (x) = m, that is, a horizontal straight line through y = m.
WORKED EXAMPLE 1

For the straight line function shown, sketch the graph


of its gradient function.

f(x)

2
THINK

WRITE/DRAW

Find the gradient of f (x) using m =

Sketch the graph of y =


gradient.

1
2

rise
.
run

m=

to represent the

1
2
y
1
2

f'(x)
x

Note: The domain of f (x) is R as f (x) is smooth and continuous.

Gradient function of quadratic functions


The gradient function of a polynomial function is also a polynomial function but the degree is
reduced by 1. That is, the gradient function of f (x) = ax2 + bx + c is of the form y = mx + c. So
the gradient of a quadratic function is a linear function.

WORKED EXAMPLE 2

Sketch the graph of the gradient function for the quadratic function shown and state its domain.
y

0
THINK

326

f(x)

WRITE/DRAW

Find when f (x) = 0.

f (x) = 0 when x = 1.

Find when f (x) > 0.

f (x) > 0 when x > 1.

Find when f (x) < 0.

f (x) < 0 when x < 1.

Maths Quest 12 Mathematical Methods CAS for the TI-Nspire

Sketch f (x).

f'(x)

Find where f (x) is smooth and continuous


and hence find the domain of f (x).

The domain is R.

Gradient function of cubic functions


The gradient function of a cubic is a quadratic function.
WORKED EXAMPLE 3

eBook plus

For the cubic function shown, sketch the


gradient function and state its domain.

Tutorial

f(x)

int-0552
Worked example 3

3
THINK

0
1

WRITE/DRAW

Find when f (x) = 0.

f (x) = 0 when x = 3 and x = 1.

Find when f (x) > 0.

f (x) > 0 when x < 3 and x > 1.

Find when f (x) < 0.

f (x) < 0 when 3 < x < 1.

Sketch the graph of the gradient function.

Find the domain by determining where f (x)


is smooth and continuous.

f'(x)

The domain is R.

WORKED EXAMPLE 4

For the function f (x) shown, state the domain of


the gradient function f (x).

1 0 2

x
f(x)

Chapter 7

Differentiation

327

THINK

WRITE

The domain = R \{1, 2}.

The function is smooth and continuous


everywhere except at x = 1 (discontinuous)
and x = 2 (not smooth).

REMEMBER

1.
2.
3.
4.
5.

6.
7.
8.
9.
10.

EXERCISE

7A
eBook plus
Digital doc

A function is smooth if there are no sharp points on its graph.


A function is continuous if the graph can be drawn without lifting pen from paper.
The gradient of a function exists where the function is smooth and continuous.
The graph of the gradient function is a graph of the gradients of all the points of the
original function.
The gradient function of a polynomial function is also a polynomial function but the
degree is reduced by 1. For example, the gradient function of f (x) = ax2 + bx + c is of
the form y = mx + c.
Wherever the gradient of a function is zero, its gradient function will have an
x-intercept.
Wherever the gradient of a function is positive (sloping /), the gradient function graph
is above the x-axis.
Wherever the gradient of a function is negative (sloping \), the gradient function graph
is below the x-axis.
The gradient of a horizontal line is 0.
The gradient of a vertical line is undefined.

Review gradient and rates of change


1 WE 1 For each straight line function shown below, sketch the graph of its gradient function.
b
c
a
y
y
y
f(x)

SkillSHEET 7.1

Gradient
positive,
negative and
zero

f(x)
2

1
f(x)

eBook plus
Digital doc

SkillSHEET 7.2

y
0

Gradient
function

f(x)
2

y
f(x)

x
3
0

328

Maths Quest 12 Mathematical Methods CAS for the TI-Nspire

0
1

2 MC a The gradient of the line in the graph at right is:


a 1

b 2

c 0

1
2

y
f(x)

0 1

b The graph of the gradient function in the graph above is represented by which of the
diagrams below?
b
c
a
y
y
y
f'(x)

f'(x)

0 1

y
1
2

y
x

f'(x)

f'(x)

f'(x)
x

3 WE 2
a

Sketch the graph of the gradient function for each quadratic function shown below.
b
c
y
y
y
g(x)

g(x)

g(x)

g(x)

y
2

0
1

g(x)

4 MC a The gradient of the function shown in the graph at


right is:
a always increasing
b always decreasing
c decreasing then increasing
d increasing then decreasing
e constant

x
f(x)

Chapter 7

Differentiation

329

b The gradient function for the graph in question 4a is shown by which of the graphs
shown below?
a

f'(x)

f'(x)

f'(x)

f'(x)

0
f'(x)

5 WE3
a

For each cubic function f(x) graphed below, sketch the gradient function.
y
b y
c
y
f(x)

f(x)

f (x)

y
Gradient = 0

f(x)

Gradient = 0

f(x)
0

x
3

x
f(x)

f(x)

y
Gradient = 0
2
0

6 MC a The figure at right has a positive gradient where:


A 1 < x < 2
B x < 1 only
C x > 2 only
D x < 1 and x > 2
E x > 0
b The figure above has a negative gradient where:
A x > 1
B x < 2
C 1 < x < 2
D x < 1 and x > 2
E x < 0
330

Maths Quest 12 Mathematical Methods CAS for the TI-Nspire

y
f(x)

c The graph of the gradient function for the figure on the previous page is:
b
c
a
y
y
y
f '(x)

f '(x)
1 0

2x

1 0

f '(x)

1 0

f '(x)

f '(x)
1 0

1 0

7 For the functions graphed below, state the domain (where applicable) where the gradient:
i is equal to zero
ii is positive
iii is negative
iv does not exist.
a

f(x)

g(x)

1 0

f(x)

g(x) y
0

1
0

1 0

f(x)

x
0

x
f(x)

g(x)

Gradient = 0
x

2 0

f(x)

g(x)

x
Gradient = 0

y
5

y
4
2
2 0

x
f(x)

8 Sketch the gradient function for each function in question 7.

Chapter 7 Differentiation

331

9 WE 4 For each function f (x) graphed below, state the domain of the gradient function f (x).
(Do not sketch the graph of f (x).)
b
c
a
y
y
y
f(x)

f(x)
2
0

8 x

0
f(x)

f(x)

f(x)
1

f(x)

f(x)

f(x)

f(x)
2

f(x)
5

7B

Limits and differentiation


from rst principles
The limit of a function is the value that the function approaches as x
approaches a given value.
If the function is continuous at the point in question then
the limit exists and can be found by direct substitution.
For example, the limit of f (x) = 3x + 1 as x approaches 1 is
denoted as
lim (3 x + 1)

eBook plus
eLesson

eles-0093
Limits and
differentiation
from first principles

f(x)

Continuous

x 1

In this example f (x) is a continuous function at x = 1. (In fact, it


is continuous for all values of x.)
Therefore, the limit is found by direct substitution, that is,
lim (3 x + 1) = 3(1) + 1 = 4

x 1

332

Maths Quest 12 Mathematical Methods CAS for the TI-Nspire

1
0

If the function is discontinuous at the point in question, then the limit exists if the function is
approaching the same value from the left as it is from the right.
For example:
For example:
y

Discontinuous
3

Discontinuous
at x = 2

f(x)

f(x)
5
3

2
0

The limit exists at x = 2, as the function is


approaching 3 from the left and from the right.
Therefore:
lim f ( x ) = 3
x2

The limit does not exist at x = 2, as the


function is approaching 3 from the left and 5
from the right. That is, if the left-hand limit is
not equal to the right-hand limit, then the limit
does not exist.

WORKED EXAMPLE 5

Evaluate the following limits.


a lim ( x 2 3 x )
x 5

eBook plus

x2 + 5 x + 6
x 0
x+3

Tutorial

b lim

int-0053

THINK
a

WRITE

Decide whether f (x) is continuous at x = 5. If


so, substitute x = 5 into f (x).

Evaluate.

1
2

Worked example 5

lim ( x 2 3 x ) = 52 3(5)

x5

= 10

Decide whether f (x) is continuous at x = 0. If


so, substitute x = 0 into f (x).

lim

+ 5 x + 6 0 2 + 5(0) + 6
=
x+3
0+3
6
=
3
=2

x2

x0

Evaluate.

If direct substitution makes the denominator zero, the limit of a rational expression can be
evaluated by first simplifying the expressions and then using direct substitution.
WORKED EXAMPLE 6

Evaluate

lim

x 3

x2 + 5 x + 6
.
x+3

THINK

WRITE

x 2 + 5x + 6
( x + 3) ( x + 2)
= lim
3
x

3
x+3
x+3

If the limit cant be found because the


denominator becomes zero, factorise the
numerator.

Simplify by cancelling.

= lim ( x + 2), x 3

Substitute x = 3.

= 3 + 2

Evaluate.

= 1

lim

x 3

Chapter 7

Differentiation

333

Differentiation using rst principles


The gradient function is the rule for the instantaneous rate of change of a given function at any
point. It also gives the gradient of the tangent drawn at any point of the given function.
Consider the chord (straight line) PQ to the curve below.
f(x)
Q

y
f(x + h)

f(x + h) f(x)
P

f(x)

x+h

rise
run
f ( x + h) f ( x )
=
h
As Q moves along the curve towards P, the value of h gets smaller and smaller. Or as Q gets
as close as possible to P, h 0, and PQ becomes a tangent at P. The gradient of the curve at a
point P is the gradient of the tangent at that point.
f ( x + h) f ( x )
That is, gradient at point P is f ( x ) = lim
, h 0.
h0
h
Finding the gradient this way is known as differentiation from first principles.
Differentiating f (x) gives f (x) or f (x) is the derivative of f (x).
dy
dy
Differentiating y gives , or
is the derivative of y with respect to x.
dx
dx
The gradient of PQ =

WORKED EXAMPLE 7

Find the gradient of the chord PQ drawn to the curve f (x) = x2 + 2 in the diagram.
y
Q [1 + h, f(1 + h)]

P [1, f(1)]
x

0
THINK

Find the gradient and simplify.


rise
Gr ient =
Grad
.
run

WRITE

f ( x + h) f ( x )
,h0
h
f (1 + h) f (1)
=
h
Gr ient =
Grad

(1 + h)2 + 2 (12 + 2)
h
3 + 2h + h 2 3
=
h
=

334

Maths Quest 12 Mathematical Methods CAS for the TI-Nspire

2h + h 2
h
h(2 + h)
=
h
=2+h
The gradient is 2 + h.
=

WORKED EXAMPLE 8

Find the gradient of f (x) = x2 1 at the point where x = 2 by:


a sketching a graph and finding the gradient of the tangent at x = 2
b differentiating using first principles.
THINK
a

1
2

WRITE

Sketch the graph of f (x) over a domain which


includes the given value of x.
Construct a tangent at the given point on the
curve. (It is difficult to be accurate.)

Find the gradient of this tangent by


evaluating rise .
run

Find f (x) using first principles.

y
8
7
(3, 6)
6
5
4
3
2
11
1
0 12345
2
3
4
5
(0, 5)
3

Gradient of tangent at x = 2 is
11
approximately = 3 23 .
3
f
(
x + h) f ( x )
b f ( x ) = lim
,h0
h0
h
( x + h)2 1 ( x 2 1)
h0
h

= lim

x 2 + 2 xxhh + h 2 1 x 2 + 1
h0
h

= lim

2 xh + h 2
h0
h
h(2 x + h)
= lim
h0
h
= lim (2 x + h)
= lim

h0

= 2x
2

Evaluate f (2) to find the gradient at x = 2.

f (2) = 2(2)
= 4

Note: Answer a (3 23 ) is very close to


answer b (4).

Chapter 7

Differentiation

335

WORKED EXAMPLE 9

Use first principles to differentiate g(x) = x2 x.


THINK

WRITE

Find g(x + h) and simplify.

Find g (x) using first principles.

g(x + h) = (x + h)2 (x + h)
= x2 + 2xh + h2 x h
g( x + h) g( x )
g ( x ) = lim
, h 0,
h0
h
x 2 + 2 xxhh + h 2 x h ( x 2 x )
h0
h
2
2
x + 2 xxhh + h x h x 2 + x
= lim
h0
h
2
2 xh + h h
= lim
h0
h
h(2 x + h 1)
= lim
h0
h
= lim (2 x + h 1)
= lim

h0

= 2x 1
REMEMBER

1. The limit of a function is the value that y approaches as x approaches a given value.
2. If a function is continuous at a point then a limit exists at that point and can be found
by direct substitution.
3. If a function is discontinuous at a point then the limit exists only if the function is
approaching the same value from both left and right. If there is a break in the curve and
the point lies within the break then the limit does not exist.
4. The limit of expressions with a denominator can be found.
(a) If direct substitution makes the denominator equal to zero then factorise the
numerator, cancel and then use direct substitution.
(b) If direct substitution does not make the denominator equal to zero then use direct
substitution.
f ( x + h) f ( x )
5. The gradient of a chord or secant is found using
.
h
f ( x + h) f ( x )
, h 0.
6. The gradient at a point P on a curve is f ( x ) = lim
h0
h
7. The gradient at a point P on a curve is the gradient of the tangent to the curve at that point.
EXERCISE

7B

Limits and differentiation from rst principles


1 WE 5
Evaluate the following limits.
a lim ( x + 4)
b lim (2 p 3)
x2

d lim ( x 2 5)

h lim

x3

336

p 2

lim (10 x + x 2 x 3 )

x 2

lim ( x 2 + 4 x 3)

x 2 + 5x + 6
x0
x+2

x 1

Maths Quest 12 Mathematical Methods CAS for the TI-Nspire

lim (8 3h)

h0

lim ( x 3 5 x + 2)

x3

x2 2x 3
x 1
x3
lim

2 WE 6 By first simplifying the rational expression, evaluate the following limits.


2x2 + 2x
x 2 + 3x
3x 2 3x
a lim
c lim
b lim
x 1 x +1
x0
x 1 x 1
x
2 4
2 + 3x + 2
x
x
x 2 5x 6
d lim
e lim
f lim
x2 x 2
x 1
x6
x +1
x6
h3 8
x 3 + 27
x2 + 4x 5
g lim
h lim
i lim
h2 h 2
x 3 x + 3
x 5
x+5

3 Evaluate the following.


a lim (3 x 4)
x3

x3 + 4
x 1 x + 2
x 2 + 3x
g lim
x3 x 1
d lim

x2 9
3 x+3

lim

h3 64
h4 h 4
x3 + 1
h lim
x 1 x 1
e lim

x2 + x 6
x2
x2
lim

lim ( x 3 + x 2 6)

x 2

x 2 + 7 x + 12
4
x+4

lim

Questions 4 and 5 refer to the following diagram. Consider the chord PQ drawn to the curve
f (x) as shown.
4 WE 7 Find the gradient of the chord PQ
drawn to the curve f (x) = x2 + 1 in the
diagram at right.

y
f(x) = x2+1
Q (2+h, f(2+h))

5 MC The gradient of the tangent at P in the


diagram is:
a h
b 4+h
c 4h
e 4
d 4 + h2

P (2, f(2))

1
0

6 a Find the gradient of the chord PQ to the


function f (x) = x(x + 2) if the x-coordinates
of P and Q are 1 and 1 + h respectively.
b Hence, find the gradient of the function at P.

7 If the gradient of the chord joining two points on the curve f (x) = x2 + 2x + 3 is
find the gradient of the curve at the point where x = 1.

f (1 + h) f (1)
,
h

8 WE 8 Find the gradient of g (x) = 4 x2 at the point where x = 2 by:


a sketching a graph and finding the gradient of the tangent at x = 2
b differentiating using first principles.
9 Find the gradient of h (x) = 2x2 6x at x = 1 by using:
a a sketch graph to find the gradient of the tangent at x = 1
b differentiation from first principles.
10 MC The gradient of a function f (x) at the point where x = 3 is:
f ( x + h) f ( x )
f (3 + h) f ( x )
f (3 + h) f (3)
a lim
b lim
c lim
h0
h0
h0
h
h
h
f (3 + h) f (3)
f ( x + h) f ( x )
d
e
h
h
11 WE9 Use first principles to differentiate f (x) if:
a f (x) = 3x + 5
b f (x) = x2 3
d f (x) = (x 4)(x + 2)
e f (x) = 8 3x2
dy
12 Use first principles to find
if:
dx
a y = 9 4x
b y = x2 + 3x
3
d y = x 4x
e y = 5x 2x3

c
f

f (x) = x2 + 6x
f (x) = x3 + 2

c
f

y = 3x2 + 8x 5
y = x2 2x

Chapter 7

Differentiation

337

The derivative of xn

7C

Instead of using the procedure of differentiating from first principles, rules can be applied to find
derivatives. These rules can be derived from first principles and have been looked at in detail in
Maths Quest 11 Mathematical Methods CAS.
If f (x) = axn then f (x) = naxn 1, where a and n are constants and n is rational.
If f (x) = c then f (x) = 0, where c is a constant. (This is because c = x0 and, using the rule, the
derivative of x0 is 0 x 1 or 0.)
dy
For example, if y = x7 then
= 7 x 6 . If f (x) = 5x4 then f (x) = 20x3.
dx
If f (x) = g(x) + h(x) then f (x) = g(x) + h(x), that is, differentiate each term of a function
separately.
If f (x) = a g(x), where a is a constant, then f (x) = a g(x).
WORKED EXAMPLE 10

Differentiate y = x 4

3 2
x + 7.
2

THINK

WRITE

y = x 4 32 x 2 + 7

Write the equation.

Differentiate each of the three terms


separately.

dy
= 4 x 4 1 32 (2) x 2 1 + 0
dx
= 4x3 3x

WORKED EXAMPLE 11

Find the derivative of:


1
1
a f ( x) = +
x
x

b f ( x) =

x+ x
.
x2

THINK
a

WRITE

Write the equation.

Rewrite

Differentiate each term.

1
1
and
using negative indices.
x
x

f ( x) =

f ( x ) = x 1 + x

Write the function in the form originally given.

Write the equation.

1
1
1 1 1
2
x
2
3
2
x

x 2

=
b

f ( x) =

1
1

x 2 2 x3

x+ x
x2
1

338

Rewrite x using indices.

Maths Quest 12 Mathematical Methods CAS for the TI-Nspire

1
2

f ( x ) = x
=

1 1
+
x
x

x + x2
=
x2

Separate the function into two terms expressed


in index form.

x x2
= 2+ 2
x
x

Simplify each term.

=x 1+x

Differentiate each term.

f ( x ) = 1x
= x

( x ) .
Simplify f (

=
=

3
2

1 1

x2

3 32 1
x
2

3 25
x
2
3
5

2x 2
3

x2 2 x5

WORKED EXAMPLE 12

If f ( x ) = x 3 2 x 2 +
a f '(x)

8
, use a CAS calculator to find:
x
b f '(2).

THINK
1

WRITE

On a Calculator page, press:


MENU b
4:Calculus 4
1:Derivative 1
Complete the entry lines as:
8
d 3
x 2x2 +

x
dx
d 3
8
x 2x2 + x = 2

x
dx
Press ENTER after each entry.
Alternatively, press / r to obtain the
expression template and choose the derivative
template.
Write the answers using the correct notation
for the derivative.

a f '( x ) = 3 x 2 4 x
b f '(2) = 2

8
x2

REMEMBER

1. Rules for differentiating axn


(a) If f (x) = axn then f '(x) = naxn 1, where a and n are constants.
(b) If f (x) = c then f '(x) = 0, where c is a constant.
(c) If f (x) = ag(x), where a is a constant then f '(x) = ag'(x).
2. If f (x) = g(x) + h(x) then f '(x) = g'(x) + h'( x).
Differentiate each term of a function separately.
3. To evaluate f (a) where a is a constant, replace every x in the f (x) equation with the
value of a. For example, if f (x) = 2x2 3x + 1, f (2) = 2 22 3 2 + 1 = 3.

Chapter 7

Differentiation

339

EXERCISE

7C

The derivative of xn
1

Find the derivative of each of the following.


a y = x6
b y = 3x2
c y = 5x4
f y = 5x
g y = 12 x 3
j y = 8x5
Differentiate each of the following.

e y = 4x3
i y = 10
2 WE 10

eBook plus
Digital doc

SkillSHEET 7.3
Index laws

d y = x20
x4
h y=
3

a f (x) = 4x3 + 5x

b g(x) = 5x2 + 6x + 1

c h( x ) = 9 +

d h(x) = 4 3x + 6x2 + x3

e g(x) = 7x11 + 6x5 8

g f (x) = 6x + 3x2 4x3

h g( x ) = 7 x 2 4 x + 23

x3
5

2x 5 x3
+ + 10
5
3
i h (x) = (x + 4)(x 1)
f ( x) =

j f (x) = (x2 + 2x)(3x 6)


3 WE 11
a

Find the derivative of each of the following.


1

2
x3

d 4x 4
g x

c x3

b 3 x

1
2

3
4x

1
3

x 2x2

x+3
x
2
k
5x 2

+ x3

i
l

1
+ x2
x
x 2 + x3
x
2

+ 3x 2
x

x3

4x +

x 3

x + x4
x

4 WE 12 If f (x) = 2x5 10x + 5 find:


a f (x)
b f (2).
5 MC If f (x) = x2 6x then f (4) is equal to:
a 8
b 12
c 12

d 2

16

6 MC The value of f (9) if f ( x ) = x 2 + x 2 10 x is:


b 18
d 8
e 0
c 12 12
1
7 Find g (2) if g( x ) = 2 + 3 x 8.
x
5
8 Find the gradient of the curve y = 4 at the point where a x = 2 and b x = 0.
x
1
9 Find the gradient of f ( x ) = 2 x 3 x 2 +
at the point where x equals:
x
a 1
b 4
c 9.
a

1
2

10 If g (x) =

x + 4 x , find:

a g (x)

b g(1)

c g (8)

d g (8).

11 Show that the derivative of y = k, where k is a constant, is zero.


12 For each of the following:
i expand the brackets
ii differentiate the expanded expression
iii factorise.
a (x + 1)2
b (x + 1)3
c (2x + 1)2
d (2x + 1)3
e (3x + 1)2
f (3x + 1)3
13 Using the results of question 12 give the derivative of (ax + b)n in factorised form. (a, b, n are
constants.)

340

Maths Quest 12 Mathematical Methods CAS for the TI-Nspire

The chain rule

7D

A function which can be expressed as a composition of two simpler functions is called a


composite function. For example, y = (x + 3)2 can be expressed as y = u2 where u = x + 3.
That is, to obtain y from x, the first function to be performed is to add 3 to x (u = x + 3), then
this function has to be squared ( y = u2).
Composite functions can be differentiated using the chain rule. For example, using the
previous function, y = (x + 3)2:
Let u = x + 3, so y = u2.
dy
du
= 2u.
Then
= 1 and
du
dx
dy
dy dy du
=
. This is known as the chain rule. It is known as the
But we require
and
dx
dx du dx
chain rule because u provides the link between y and x.
dy
Now
= 2u 1
dx
= 2(x + 3) 1 (replacing u with x + 3)

= 2(x + 3)
The chain rule is used when it is necessary to differentiate a function of a function as above.
WORKED EXAMPLE 13

eBook plus

If y = (3x
is expressed as y =
dy
dy
ddu
u
dy
dy
a
b
and hence c
.
du
du
dx
dx
dx
dx
2)3

un,

find:

Tutorial

int-0554
Worked example 13

THINK
a

WRITE

Write the equation.

a y = (3x 2)3

Express y as a function of u.

Differentiate y with respect to u.

Express u as a function of x.

Differentiate u with respect to x.

Let y = u3 where u = 3x 2
dy
= 3u 2
du
b u = 3x 2
du
=3
dx

Find

dy
using the chain rule.
dx

dy dy du
=

dx du dx
= 3u2 3
= 9u2

= 9(3x 2)2

Replace u as a function of x.

WORKED EXAMPLE 14

If f ( x ) =

1
2 x2

3x

, find f (x). Check your answer using a CAS calculator.

THINK
1

Write the equation.

WRITE

f ( x) =

1
2x2

3x

Chapter 7

Differentiation

341

Express f (x) in index form.

Express y as a function of u.

Differentiate y with respect to u.

Express u as a function of x.

Differentiate u with respect to x.

Find f (x) using the chain rule.

Replace u as a function of x and simplify.

y = (2 x 2 3 x )
1

Let y = u 2 where u = 2x2 3x

dy 1 32
= 2u
du
u = 2x2 3x
du
= 4x 3
dx
3
dy

= f ( x ) = 12 u 2 (4 x 3)
dx
=
=
=

1
2

1
(4 x 3) (2 x 2
2
(4 x 3)

3x)

3
2

2(2 x 2 3 x ) 2
3 4x
2 ( 2 x 2 3 x )3

On a Calculator page, press:


MENU b
4:Calculus 4
1:Derivative 1
Complete the entry line as:

d
1

2
dx 2 x 3 x
Press ENTER .

10

Note the answer from the CAS is not


expressed in the same format obtained
previously.

f '( x) =

(4 x 3)

2 x (2 x 3) x (2 x 3)

A quicker way to apply the chain rule when a function can be expressed in index form is as
follows.
If f (x) = [g(x)]n then f (x) = n[g(x)]n 1 g (x). That is, differentiate the bracket and then what
is inside the bracket; outside then inside.
WORKED EXAMPLE 15

Find the derivative of f (x) = (x2 2 x)3.


THINK

342

WRITE

Write the equation.

f (x) = (x2 2x)3

Let g(x) equal what is inside the bracket.

g(x) = x2 2x

Find g (x).

g(x) = 2x 2

Maths Quest 12 Mathematical Methods CAS for the TI-Nspire

Use the rule f (x) = n[g(x)]n 1 g(x) to


differentiate f (x).

f (x) = 3(x2 2x)3 1 (2x 2)


= 3(x2 2x)2 [2(x 1)]
= 6(x 1)(x2 2x)2

Simplify f (x) as far as possible.

= 6(x 1)[x(x 2)x(x 2)]


= 6x2(x 1)(x 2)2

REMEMBER

1. A composite function is a function composed of two (or more) functions.


2. Composite functions can be differentiated using the chain rule,
dy dy du
=
.
dx du dx
3. A short way of applying the chain rule is:
If f (x) = [g(x)]n then f (x) = n[g(x)]n 1 g(x).
EXERCISE

7D

The chain rule


1 If each of the following functions are expressed in the form y = un, state i u and ii n.
1
a y = (5x 4)3
b y = 3x + 1
c y=
(2 x + 3)4
4
1

d y = 7 4x
e y = (5x + 3) 6
f y = (4 3 x ) 3
2 MC If y = (x + 3)5 is expressed as y = u5 then:
a u = (x + 3)5
b u=x+3
d u=3
e u = x5

c u=x

3 WE 13
If each of the following composite functions are expressed as y = un, find:
du
dy
dy
i
ii
and hence iii
.
dx
du
dx
1
b y = (7 x)3
c y=
a y = (3x + 2)2
2x 5
3
1
d y=
e y = 5x + 2
f y=
4
(4 2 x )
3x 2
2

g y = 3(2x2 + 5x)5

h y = (4x 3x2)

y = x+

j y = 4(5 6x)

For questions 4, 5 and 6 below, y = x 2 3 x + 2 is expressed as y = un.


4 MC
a

dy
is equal to:
du

1
u
2

b u

1
2

1
2 u

3
1 2
u
2

1
2

1
1 2
u
2

du
5 MC
is equal to:
dx
a 2x 3

b x2 3x + 2

c x2 3x

x2 2 x + 1

Chapter 7

e x3

Differentiation

343

6 MC Using the chain rule, dy is equal to:


dx
2x 3
2x 3
b
a
2 x 2 3x + 2
u
d
7

1
(2 x
2

3)

1
(2 x
2

x 2 3x + 2
2 u

1
3)( x 2 3 x + 2) 2

Use the chain rule to find the derivative of the following.


a y = (8x + 3)4

b y = (2x 5)3

c f (x) = (4 3x)5

d y = 3x 2 4

f g(x) = (2x3 + x)

f ( x) = ( x 2 4 x) 3

g g( x ) = x

h y = (x2 3x)
1

8 WE 14 If f ( x ) =

4x + 7

, find f (x).

9 Use the chain rule to find the derivative of the following. (Hint: Simplify first using index
notation and the laws of indices.)
a y=

6x 5
6x 5

b f ( x) =

( x 2 + 2)2
x2 + 2

10 WE 15 Find the derivative of:


a f (x) = (x2 + 5x)8
b y = (x3 2x)2
c

f ( x ) = ( x 3 + 2 x 2 7) 5

d y = (2 x 4 3 x 2 + 1) 2

11 If f (x) = (2x 1)6, find f (3).

12 If g (x) = (x2 3x) 2, find g(2).


13 If f ( x ) = x 2 2 x + 1, find:
a f (3)
c f (3)
eBook plus
Digital doc

WorkSHEET 7.1

b f (x)
d f (x) when x = 2.

14 Find the gradient of the function h( x ) = 3 x 2 + 2 x at the point where x = 2.


3
15 Find the value of f (1) if f ( x ) =
.
5 4x
16 If f (x) = (2x 1)5 and f (x) = 10 (2x 1)n, find the value of n.
17 If f (x) = (3x + 2)7 and f (x) = a (3x + 2)6 3, find the value of a.
18 If f (x) = (5x 3)10 and f (x) = 10m (5x 3)9, find the value of m.

7E

The derivative of e x
If f (x) = ex then using first principles
f ( x + h) f ( x )
f ( x ) = lim
,h0
h0
h

344

ex + h ex
h0
h

= lim

= lim

e x eh e x
h0
h

Maths Quest 12 Mathematical Methods CAS for the TI-Nspire

e x (e h 1)
h0
h
h
e 1

= e x lim
h0
h
h
e 1
Note that lim
can be deduced by using a calculator and substituting values of h close to
h0
h
zero.

= lim

eh 1
h

0.01

1.0050

0.0001

1.000 05

0.000 001

1.000 000

eh 1
= 1.
h0
h
Therefore, f (x) = ex 1
= ex
That is, lim

If f (x) = ex then f (x) = ex.

WORKED EXAMPLE 16

Differentiate y = e

5x

THINK

WRITE

Write the equation.

Express u as a function of x and find

3
4
5

du
.
dx
dy
Express y as a function of u and find
.
du
dy
Find
using the chain rule.
dx
Replace u as a function of x.

5x

y=e

Let u = 5x so

du
= 5
dx

dy
= eu
du

y = eu so

dy u
= 5e
du
5x

= 5e

This example shows that if f (x) = ekx then f ' (x) = kekx.
WORKED EXAMPLE 17

Find the derivative of y = e2x + 1.


THINK
1

Write the equation.

Express u as a function of x and find

3
4

du
.
dx
dy
Express y as a function of u and find
.
du
dy
Find
using the chain rule.
dx
Replace u as a function of x.

WRITE

y = e2x + 1
Let u = 2x + 1 so
y = eu so

du
=2
dx

dy
= eu
du

dy
= eu 2
dx
= 2eu
= 2e2x + 1

Chapter 7

Differentiation

345

WORKED EXAMPLE 18

a f (x) = ex(ex 2)

Differentiate:

b f ( x) =

e2 x 2 e x
.
ex

THINK
a

WRITE

Write the equation.

Expand.

Differentiate.

Factorise in order to leave the answer in the


form it was given.

Write the equation.

Write each term in the numerator over each


term in the denominator.
Divide the numerator of each term by its
denominator using the laws of indices.

f (x) = ex(ex 2)

= e2x 2ex
f (x) = 2e2x 2ex
= 2ex(ex 1)
b

f ( x) =

e 2 x 2e x
ex

e 2 x 2e x
x
ex
e
x x

= e2x x 2e

= ex 2e 2x

Differentiate each term.

Write your answer in the form it was given.

f (x) = ex + 4e 2x
4
= ex + 2x
e

WORKED EXAMPLE 19
3 x

Find the derivative of y = ex

THINK

WRITE

Write the equation.

Express u as a function of x and find

3
4

y = ex

du
.
dx
dy
Express y as a function of u and find
.
du
dy
Find
using the chain rule.
dx

y = eu so

Replace u as a function of x.

Let u = x3 x so

du
= 3x 2 1
dx

dy
= eu
du

dy
= eu (3 x 2 1)
dx
= (3x2 1)eu
3 x

= (3x2 1)ex

This example shows that if f (x) = eg(x) then f (x) = g (x) e g (x).
REMEMBER

1.
2.
3.
4.

346

If f (x) = ex, f (x) = ex.


If f (x) = ekx, f (x) = ke kx.
If f (x) = aekx + c, f (x) = ake kx + c.
If f (x) = ae g (x), f (x) = g (x) ae g(x).

Maths Quest 12 Mathematical Methods CAS for the TI-Nspire

EXERCISE

7E

The derivative of ex
1 WE 16

Differentiate each of the following.


1x

a y = e10x
b y = e3
c y = e4
5x
2x

f y = 4e
g y = 6e
h y = 5e0.2x
2 WE 17 Find the derivative of each of the following.
a y = e6x 2
b y = e8 6x
7

2x
d y = 4e
e y = 3e8x + 1
6 9x
g y = 10e
h y = 5e3x + 4
j

x +1

y = 2e 2

k y = 3e

c y = 2e5x + 3
f y = 2e6 5x

i y = 6e 7x

2 x
3

3 MC
The derivative of y = e3x + 2 is equal to:
3x + 2
a 3e
b (3x + 2)e3x + 2
c 3e3x
d 3xe3x + 2
4 WE 18 Differentiate each of the following:
a f (x) = 2(ex + 1)
b f (x) = 3e2x(ex + 1)
x

d f (x) = (ex + 2)(e + 3)

f ( x) =

3e3 x

e2

4e5x

2x2

d f (x) = e2 5x

e f (x) = e6 3x + x

+ 2x)
2e x

2x

f g(x) = ex + 3x 2

i y = e(2x + 1)

f ( x) =

4e7 x

c y = ex 2x

h y = 5e1 2x 3x

j f (x) = e(4 x)

4x

b y = ex 3x + 1
2

e 3xe3x

a y = ex + 3x
g h(x) = 3e4x 7x

x +5

y = 4e 4

c f (x) = 5(e

e 6x

+
ex

g f (x) = +
h f (x) =
+
5 WE 19 Find the derivative of each of the following:
ex

e y = 2e3x

d y=e x

i y = 2e 11x

k g(x) = e(x + 2)

y=e

3x + 4

1
3

n h(x) = e(x2 + 3x)


m f ( x ) = e( x +1)
6 MC The derivative of 6ex3 5x is equal to:
a 3x2 5
b 6(3x2 5)ex3 5x
3 5x
3
x
d 6(x 5x)e
e 6(3x2 5)e3x2 5
9 4x
7 If f (x) = 5e
, find the exact value of f (2).

c (3x2 5)ex2 5x

8 If g (x) = 2ex2 3x + 2, give the exact value of g(0).


9 Find the exact value of h(1) if h(x) = 5ex3 + 2x.

7F

The derivative of loge (x)

The inverse of the function f (x) = ex is f 1(x) = loge (x).


If y = loge (x) then ey = x, as shown in chapter 4, Exponential and logarithmic equations.
Let
x = ey
dx
= ey
dy
dy
1
But
=
dx dx and ey = x
dy
dy 1
Therefore,
= .
dx e y
1

=
x

Chapter 7

Differentiation

347

That is, if f (x) = loge (x) then f'( x ) =

1
.
x

WORKED EXAMPLE 20

Differentiate y = loge (7x).


THINK
1
2
3
4

WRITE

Write the equation.


du
Express u as a function of x and find
.
dx
dy
Express y as a function of u and find
.
du
dy
Find
.
dx

y = loge (7x)
du
u = 7x, so
=7
dx
dy 1
y = loge (u), so
=
du u
dy 1
=
7
dx 7 x
1
=
x

If f (x) = loge (kx), where k is a constant then f ( x ) =

1
.
x

WORKED EXAMPLE 21

Find the derivative of y = 2 loge (3x 4).


THINK
1

WRITE

Write the equation.


Express u as a function of x.

Differentiate u with respect to x.

Express y as a function of u.

Differentiate y with respect to u.

Find

Replace u with 3x 4.

dy
using the chain rule.
dx

y = 2 loge (3x 4)
Let u = 3x 4.
du
=3
dx
y = 2 loge (u)
dy
1
= 2
du
u
2
=
u
dy 2
= 3
dx u
6
=
u
6
=
3x 4

WORKED EXAMPLE 22

Differentiate y = loge (x2 + 4x 1).


THINK

348

WRITE

Write the equation.

y = loge (x2 + 4x 1)

Let u equal the section in brackets.

Let u = x2 + 4x 1

Maths Quest 12 Mathematical Methods CAS for the TI-Nspire

Differentiate u with respect to x.

Express y as a function of u.

Differentiate y with respect to u.

Find

Replace u with what is in the brackets.

du
= 2x + 4
dx
y = loge (u)
dy 1
=
du u
dy 1
= (2 x + 4)
dx u
2x + 4
= 2
x + 4x 1

dy
using the chain rule.
dx

This example shows that if f (x) = loge [g(x)] then f ( x ) =

g ( x )
.
g( x )

WORKED EXAMPLE 23

Differentiate y = loge (x2 + 5x 2)2.


THINK

WRITE

Write the equation.

y = loge (x2 + 5x 2)2

Simplify the right side by using log laws.


du
Express u as a function of x and find
.
dx
dy
Express y as a function of u and find
.
du
dy
Use the chain rule to find
.
dx

y = 2 loge (x2 + 5x 2)
du
u = x2 + 5x 2, so
= 2x + 5
dx
dy 2
y = 2 loge (u), so
=
du u
dy dy du
dy 2(2 x + 5)
=

so
=
dx du dx
dx
u
2(2 x + 5)
= 2
x + 5x 2

3
4
5

REMEMBER

1
1. If f (x) = loge (x), then f ( x ) = .
x
1
2. If f (x) = loge (kx), then f ( x ) = .
x
g ( x )
.
3. If f (x) = loge [g(x)], then f ( x ) =
g( x )
EXERCISE

7F

The derivative of loge (x)


1 If y = loge (4x) is expressed as y = loge (u), then find:
du
a u
b
dx
c

dy
du

dy
using the chain rule.
dx
Chapter 7

Differentiation

349

2 WE 20

Differentiate each of the following.

a y = loge (10x)

b y = loge (5x)

c y = loge (x)

d y = loge (6x)

e y = 3 loge (4x)
x
h y = log e
3

f y = 6 loge (9x)
x
i y = 4 log e
5

x
g y = log e
2
j

2x
y = 5 log e
3

3 MC The derivative of loge (8x) is:

8
1
d
x
x
4 MC To differentiate y = loge (3x + 7) using the chain rule:
a u would be used to represent
d loge (3x)
a 3x + 7
b 3x
c loge (x)
b

a 8

1
8

e loge (8)

e x

dy
du
and
are respectively
du
dx

1
1
1
1
and 3x
c 3 and
b
and 3x + 7
d
and 3
3x
u
u
u
dy
c Hence
is equal to
dx
3
1
1
d
c
a 3
b
3x + 7
3x + 7
x
5 WE 21 Find the derivative of each of the following.
a y = loge (2x + 5)
b y = loge (6x + 1)
c y = loge (3x 4)
a

d y = loge (8x 1)

e y = loge (3 5x)

f y = loge (2 x)

g y = loge (4 7x)

h y = 6 loge (5x + 2)

i y = 8 loge (4x 2)

j y = 4 loge (12x + 5)

k y = 7 loge (8 9x)

6 WE 22

3
x

Differentiate the following.

a y = loge (3x4)

b y = loge (x2 + 3)

d y = loge (x2 2x3 + x4)


g y = log e (5 x +

1
2) 3

j f (x) = loge (3x 2)4

e y = log e

h f ( x ) = log
log e

2x + 1

c y = loge (x3 + 2x2 7x)

1
(2 3 x ) 5
(2
2

k f (x) = loge (5x + 8)

3 4x

a y = loge (x2 + 1)2

b y = loge (3 x2)2

c y = loge (x2 2x + 3)3

d y = loge (x2 + 4x + 4)3

y = log e

1
f ( x ) = log e
x + 3

2
f ( x ) = log e
4 + 3 x

8 MC Using the chain rule the derivative of f (x) = loge (x2 5x + 2) would be:

1
5
c 2x 5
a 2
b
2
x 5x + 2
x 5x + 2
1
2x 5
d
e
2
x (2 x 5)
x 5x + 2
9 Find the gradient of the function f (x) = 6 loge (4 3x) when x = 1.
Maths Quest 12 Mathematical Methods CAS for the TI-Nspire

7 WE 23 Differentiate each of the following.

350

e 1 and 3

10 If g(x) = 3 loge (3x + 5) find the value of g(0).


11 Find the exact value of f (2) if f (x) = 3x2 + 4 loge (x2 + x).
12 If y = eloge (x), find:
dy
a
dx
b the exact gradient when i x = 1
result?

ii x = 2

iii x = 4 iv x = 10. Can you explain this

13 If f (x) = eloge (x2), find:


a f ' (x)
b the exact value of
i f ' (1)
ii f ' (5)
iii f ' (2).

7G

The derivatives of sin (x), cos (x)


and tan (x)
The derivatives of sin (x), cos (x) and tan (x) can be found by differentiation from first principles
but are beyond the requirements of this course.
The derivatives of sin (x) and cos (x) can be shown by drawing the graphs of the gradient
functions. The domain of each of these functions is R, but we will use only part of that domain.
Consider the graph of f (x) = sin (x), domain [0, 2] shown below.
y

(
, 1)
2

( 3
, 1)
2

3
,
2 2

3
f '(x) < 0 when < x <
2
2
3

f '(x) > 0 when 0 < x < and


< x < 2
2
2
By sketching the graph of the gradient function, we can see
that it is y = cos (x).
f '(x) = 0 when x =

y
1
0
1

2 x

Similarly, by sketching the graph of the gradient function of y = cos (x), we can see that the
derivative of y = cos (x) is y = sin (x).
The derivative of tan (x) can be found using the quotient rule
y
(which appears later in this chapter).
If f (x) = sin (x) then f (x) = cos (x).
y = cos (x)
1
If f (x) = cos (x) then f (x) = sin (x).
y = sin (x)
1
0

3 2 x

If f (x) = tan (x) then f '( x ) =
, which can be
2
2
2 ( x)
1
2
cos
s
written as sec (x).

Chapter 7

Differentiation

351

WORKED EXAMPLE 24

Find the derivative of y = sin (5x).


THINK

WRITE

y = sin (5x)

Write the equation.

Express u as a function of x and find

du
.
dx

Let u = 5x so

du
=5
dx

Express y as a function of u and find

dy
.
du

y = sin (u) so

dy
= cos (u)
du

Find

Replace u with 5x.

It is important to note what happens when a


CAS calculator is used to answer this question.
On a calculator page, complete the entry line as:
d
(sin (5x))
dx
Press ENTER .
Note: The calculator gives an incorrect answer
as it is set in Degree mode.

dy
using the chain rule.
dx

dy
= 5cos (u)
dx
= 5 cos (5x)

In differentiating circular functions, the


calculator needs to be set in Radian mode as the
angles are given in radians.
Write the answer, using the correct notation for
the derivative.

dy
= 5 cos (5 x )
dx

This example shows that if f (x) = sin (ax), then f (x) = a cos (ax). Similarly, if f (x) = cos (ax),
then f (x) = a sin (ax).
352

Maths Quest 12 Mathematical Methods CAS for the TI-Nspire

WORKED EXAMPLE 25

Find the derivative of y = tan (3x).


THINK

WRITE

Write the equation.

Express u as a function of x and find

3
4

y = tan (3x)

du
.
dx
dy
Express y as a function of u and find
.
du
dy
Find
using the chain rule.
dx

du
=3
dx
dy
y = tan (u) so
= secc 2 (u)
du
dy
3
= 3 secc 2 (u) or
dx
coss2 (u)
3
= 3 sec 2 (3 x ) or
2
cos (3 x )
Let u = 3x so

This example shows that if f (x) = tan (ax), then f ' ( x ) =

a
or a sec2 (ax).
coss 2 ( ax )

WORKED EXAMPLE 26

eBook plus

Differentiate y = cos (x2 + 2x 3).

Tutorial

THINK
1

Write the equation.

Express u as a function of x and find

3
4
5

int-0555

WRITE

Worked example 26

y = cos (x2 + 2x 3)

du
.
dx
dy
Express y as a function of u and find
.
du
dy
Find
using the chain rule.
dx
Replace u with the part in brackets in the
rule and simplify.

Let u = x2 + 2x 3 so
y = cos (u) so

du
= 2x + 2
dx

dy
= sin(u)
du

dy
= sin u (2 x + 2)
dx
= 2(x + 1) sin (x2 + 2x 3)

This example shows that the chain rule can be applied as follows.
If f (x) = sin [g(x)] then f (x) = g(x) cos [g(x)].
If f (x) = cos [g(x)] then f (x) = g(x) sin [g(x)].
g'( x )
If f (x) = tan [g(x)] then f '( x ) =
cos 2 [ g( x ))]]

= g(x) sec2 [ g(x)]
REMEMBER

1. If f (x) = sin (x)


2. If f (x) = cos (x)
3. If f (x) = tan (x)
4. If f (x) = sin (ax)
5. If f (x) = cos (ax)

then f (x) = cos (x).


then f (x) = sin (x).
1
then f ( x ) =
= secc 2 ( x ).
cos2 ( x )
then f (x) = a cos (ax).
then f (x) = a sin (ax).

Chapter 7

Differentiation

353

6. If f (x) = tan (ax)

then f ( x ) =

7. If f (x) = sin [g(x)]


8. If f (x) = cos [g(x)]

then f (x) = g (x) cos [g(x)].


then f (x) = g (x) sin [g(x)].
g ( x )
= g ( x ) secc 2 [ g( x ))]].
then f ( x ) =
cos2 [ g( x)]
x

9. If f (x) = tan [g(x)]

= a sec 2 (aax ).

cos2 (ax )

EXERCISE

7G

The derivatives of sin (x), cos (x) and tan (x)


1 WE 24

Find the derivative of each of the following.

a y = sin (8x)

b y = sin (6x)

c y = sin (x)

x
d y = sin 3

x
e y = sin
2

2x
y = sin
3

Differentiate each of the following.


a y = cos (3x)

b y = cos (2x)

x
c y = cos
3

x
d y = cos
4

x
e y = cos
8

3 WE 25

2x
y = cos
5

Differentiate each of the following.

a y = tan (2x)

b y = tan (4x)

x
ta
c y = tan
5

3x
d y = tan
ta
4

4 MC a The derivative of sin (6x) is:


a 6 cos (6x)

b 6 cos (x)

d 6 cos (6x)

1
6

cos (6 x )

b The derivative of cos (4x) is:


a 4 sin (4x)
b 4 sin (x)
c The derivative of sin (4x) is:
a 4 cos (4x)
b 4 cos (x)
d The derivative of cos (8x) is:
a 8 cos (8x)
b 8 sin (8x)
e The derivative of tan (7x) is:
x
ta
a 7 tan
7

1
(7 x )

coss2

c 6 sin (x)

c 4 sin (x)

d 4 cos (4x)

sin (4x)

c 4 cos (4x)

d 4 cos (4x)

sin (4x)

c 8 sin (8x)

d 8 sin (x)

e 8 sin (x)

c 7 sec2 (7x)

d sec2 (7x)

5 WE26 If y = sin (4x + 3) is expressed as y = sin (u), find:


a

dy
du

du
dx

dy
using the chain rule.
dx

6 If y = cos (3x + 1) is expressed as y = cos (u), find:


a

354

dy
du

du
dx

Maths Quest 12 Mathematical Methods CAS for the TI-Nspire

dy
using the chain rule.
dx

1
7

sec ( x )

Differentiate each of the following.


a y = sin (2x + 3)
b y = sin (6 7x)

c y = sin (5x 4)

3x + 2
d y = sin
4

e y = 5 sin (2 x)

8 Differentiate each of the following.


a y = cos (3x 2)
b y = cos (4x + 7)
2x + 3
d y = cos
3

3x
y = 4 sin
8

c y = cos (6 5x)
f y = 6 cos (2x)

e y = 4 cos (10 x)

9 Differentiate each of the following.


a y = tan (2x + 1)
b y = tan (8 x)
d y = tan 2(x + 1)
e y = 3 tan (x)

c y = tan (5x 2)

10 Find the derivative of each of the following.


a cos (x2 4x + 3)

b sin (10 5x + x2)

c sin (ex)

d cos (x2 + 7x)

e tan (4x x2)

f tan (x2 + 3x)

g cos [loge (x)]

h sin (e4x)

1
cos
x

j sin [loge (2x 1)]

k cos (2e3x)

l 3 cos [loge (10x)]

m 4 tan (x3 + 2x2)

2 cos

x
4

8 tan
ta

3x
5

p cos (x2 + 2x) + sin (3x 9)

11 If f (x) = 3 sin (x2 + x), find f (1) (answer correct to 3 decimal places).
12 Find the gradient of the curve g (x) = 2 cos (x3 3x) at the point where x = 0.
13 For each of the following functions find:
i f (x) and

ii the exact value of f .


6
a f (x) = esin (x)

7H

b f (x) = ecos (x)

c f (x) = loge [sin (x)]

d f (x) = loge [cos (x)]

The product rule


Any function which is a product of two simpler functions, for example,
f (x) = (x + 2)(x 5)
or
f (x) = (x2 5x + 6) sin (3x + 5)
can be differentiated using the product rule of differentiation.
Although the first example can be expanded and then differentiated, the second example
cannot and therefore can be differentiated only using the product rule.

Product rule

dy
dy
dv
dv
ddu
u
= u
+v .
ddxx
dx
dx
dx
dx
Or if f (x) = u(x) v(x) then f (x) = u(x) v(x) + v(x) u(x).
If y = uv then

Chapter 7

Differentiation

355

WORKED EXAMPLE 27

If y = (3x 1)(x2 + 4x + 3) is expressed as y = uv, find:


du
du
dv
dv
dy
dy
dy
dy
dv
dv
du
du
=u +v .
a u and v
b
and
c
using
dx
dx
dx
dx
dx
dx
dx
dx
dx
dx
dx
dx
THINK
a

WRITE
a y = (3x 1)(x2 + 4x + 3)

Write the equation.

Identify u and v, two functions of x which are


multiplied together.

Differentiate u with respect to x.

Differentiate v with respect to x.

Apply the product rule to find

Let u = 3x 1

and

v = x2 + 4x + 3.

du
=3
dx
dv
= 2x + 4
dx
c dy = u dv + v du
dx
dx
dx

dy
.
dx

= (3x 1)(2x + 4) + (x2 + 4x + 3) 3


2

= 6x2 + 10x 4 + 3x2 + 12x + 9


= 9x2 + 22x + 5

Expand and simplify where possible.

WORKED EXAMPLE 28

Find the derivative of y = loge (4x) sin (3x 2).


THINK

WRITE

Write the equation.

y = loge (4x) sin (3x 2)

Identify u and v.
du
dv
Find
and
.
dx
dx
dy
Find
using the product rule.
dx

Let u = loge (4x) and let v = sin (3x 2).


du 1
dv
=
= 3 cos (3 x 2)
dx x
dx
dy
1
= log e (4 x ) 3 cos (3 x 2) + sin (3 x 2)
dx
x
1
= 3 log e (4 x ) cos (3 x 2) + sin (3 x 2)
x

3
4
5

Simplify wherever possible.

REMEMBER

dy
dv
du
=u +v .
dx
dx
dx
2. If f (x) = u(x) v(x) then f (x) = u(x) v(x) + v(x) u(x).
1. If y = u v then

EXERCISE

7H

The product rule


1 WE27 If y = (x + 3)(2x2 5x) is expressed as y = u v, find:
du
dv
dy
dy
dv
du
a u and v
b
and
c
using the product rule,
=u +v .
dx
dx
dx
dx
dx
dx

356

Maths Quest 12 Mathematical Methods CAS for the TI-Nspire

2 WE28
a y=

Find the derivative of:


4x3

loge (6x)

b g (x) = (3x 2) loge (2x).

3 MC
The derivative of f (x) =
a f (x) = 2x cos (2x)
c f (x) = 2x sin (2x) + x2 cos (2x)
e f (x) = 2x sin (x) + 2x2 cos (x)

x2

sin (2x) is:


b f (x) = 4x cos (2x)
d f (x) = 2x sin (2x) + 2x2 cos (2x)

4 Use the product rule to differentiate each of the following.


a y = x cos (x)
b y = 3x sin (x)
c y = (5x 2) ex
d y = e3x (2 11x)
e y = x5 cos (3x + 1)
f y = 2x3 loge (7x)
2x
g y = e loge (2x 5)
h y = 8 tan (5x) loge (5x)
4x
i y = 5 cos (2x) sin (x)
j y = sinn cos ( x )
3

4x 3
k f (x) = e
loge (6x)
l f (x) = 4e 5x sin (2 x)

1
n f ( x) = x e 3 x
cos (6 x )
m f ( x) =
x

o f (x) = 2x 3 sin (2x + 3)

q f (x) = (x2 + e3x)(4 e 3x)

p f (x) = e 2x loge (3x2 + 5)


r f (x) = (x2 6)(2 + 3x x2)

5 If f (x) = (2x + 1) loge (x + 3), find the exact value of f (1).


6 Find g(0) if g(x) = 5e2x cos (4x).
eBook plus

7 Find the value of f (2) if f (x) = (x2 + 2) sin (4 3x) (answer correct to 3 decimal places).

Digital doc

8 If g (x) = (6x + x2) ex 3, find the exact value of g(2).

WorkSHEET 7.2

9 Find the exact value of f ' () if f (x) = (3 x) tan (2x).

7I

The quotient rule


The quotient rule is used to differentiate functions which are rational expressions (that is, one
function divided by another). For example,
e3 x + 8
x2 6x + 3
or f ( x ) =
f ( x) =
coss ((6 x )
5x + 2

Quotient rule

du
du
dv
dv
u
u
dy v dx
dy
dx
dx .
dx
If y = then
=
v
ddxx
v2
u( x )
v ( x )u ( x ) u( x )v ( x )
Or if f ( x ) =
then f ( x ) =
.
v( x)
[v ( x)]
x 2
WORKED EXAMPLE 29

3 x
u
is expressed as y = , find:
v
x2 + 4 x
du
du
dv
dv
a u and v
b
and
dx
dx
dx
dx

If y =

dy
dy
.
ddxx

Chapter 7

Differentiation

357

THINK
a

WRITE

Write the equation.

Identify u and v.

Differentiate u with respect to x.

Differentiate v with respect to x.

Apply the quotient rule to obtain

3 x
x2 + 4x
Let u = 3 x and v = x2 + 4x.
du
b
= 1
dx
dv
= 2x + 4
dx
du
dv
v u
dy
dx
dx
c
=
dx
v2
( x 2 + 4 x ) 1 (3 x )(
)(2 x + 4)
=
2
2
( x + 4 x)
a y=

dy
.
dx

dy
where possible, factorising the
dx
final answer where appropriate.
Simplify

=
=

x2

4 x (12 + 2 x 2 x 2 )
( x 2 + 4 x )2

x2

4 x 12 2 x + 2 x 2
( x 2 + 4 x )2

x 2 6 x 12
( x 2 + 4 x )2

x 2 6 x 12
x 2 ( x + 4) 2

WORKED EXAMPLE 30

Find the derivative of f ( x ) =

eBook plus
3x

2e
.
cos ( 2 x 3)

THINK

Tutorial

int-0556

WRITE

2e 3 x
coss ((2
(2x
2xx 3)

Write the equation.

f ( x) =

Identify u(x) and v(x).

Let u(x) = 2e3x.


Let v(x) = cos (2x 3).

Differentiate u(x) and v(x) with respect to x.

Apply the quotient rule to obtain f (x).

u(x) = 6e3x
v(x) = 2 sin (2x 3)
v ( x )u ( x ) u( x )v ( x )
f ( x ) =
[v ( x)]
x 2
=

358

Simplify where possible.

Worked example 30

coss ((22x 3) 6e3 x 2e3 x [ 2 ssin


in (2 x 3))]]
[cos
cos (2
(2 x 3))]]2

6e3 x cos (2x 3) + 4 e3 x sin (2 x 3)


[cos
cos ((22xx 3))]]2
3
x
2e [3 cos (2 x 3) + 2 sin (2 x 3)]
=
cos2 (2 x 3)

Maths Quest 12 Mathematical Methods CAS for the TI-Nspire

REMEMBER

du
dv
v
u
u
dy
dx
dx .
1. If y = then
=
2
v
dx
v
u( x )
v ( x )u ( x ) u( x )v ( x )
2. If f ( x ) =
then f ( x ) =
.
v( x)
[v ( x)]
x 2

EXERCISE

7I

The quotient rule


1 WE 29 If y =

x+3
u
is expressed as y = , find:
x+7
v

a u and v
du
dv
b
and
dx
dx
du
dv
v
u
dy
dy
dx
dx .
c
using the quotient rule,
=
dx
dx
v2
x2 + 2x
u( x )
2 If f ( x ) =
is expressed as f ( x ) =
, find:
5x
v( x)
a u(x) and v(x)
b u (x) and v (x)
c f (x) using the quotient rule.
sin ( x )
3
f (x) = tan (x) can be written as f ( x ) =
. If u(x) = sin (x) and v(x) = cos (x), use the
cos ( x )
1
quotient rule to show that the derivative of tan (x) is
.
coss2 ( x )
4 WE 30
Find the derivative of each of the following.
a
d
g
j

x2

2x
4x

4x 7
10 x
sin (2 x )
cos (2 x )
x2

4x
+ 3x 2

3x

x2 + 7x + 6
3x + 2

e2 x
x

log e ( x + 1)
x2 + 2

2x3 + 7x
e5x

e
3x + 8

4 log e (8x
8x)
2
x 2x

2 cos (3 2 x )
x2

3e 2 7 x
x+3

5 MC If h( x ) =
9x2 8
x2
3x 2 8
d
x2
a

8 3x 2
then h(x) equals:
x
8 9x
9 2
b
x2
3x 2 + 8
e
x

cos ( x )
ex
3x 2
logg e (4 x )
e3 x + 2
cos (2 x )
x2 5
x
sin ( x )
x
2
x
e
2x 3

3x 2

+8

x2

Chapter 7

Differentiation

359

sinn (4 x )
is:
4x + 1
4(4 x + 1) cos ( x ) 4 sin (4 x )
a f ( x ) =
(4 x + 1)2
4(4 x + 1) cos (4x ) 4 sin (4 x )
c f ( x ) =
4x + 1
4 ssin (4x ) 4(4 x + 1) cos (4x )
e f ( x ) =
(4 x + 1)2
cos (33 x 2)
7 MC If g( x ) =
then g(x) is equal to:
ex
3e x sin (3 x 2) e x cos (3 x 2)
a
e2 x
e x cos (3 x 2) + 3e x cos (3 x 2)
c
e2 x
3e x sin (3 x 2) e x cos (3 x 2)
e
e2 x
6 MC The derivative of f ( x ) =

8 If y =

cos (2 x )
dy
find
when x = 0.
dx
e3 x

9 Find the gradient of the function f ( x ) =


10 Find the exact value of g(5) if g( x ) =

7J

(4 x + 1) cos (4 x ) 4 ssin
in (4 x )
(4 x + 1)2
4(4 x + 1) cos (4 x ) 4 sin (4 x )
d f ( x ) =
(4 x + 1)2
b f ( x ) =

ex

sin (3 x 2) e x cos (3 x 2)
e2 x

3e x sin (3 x 2) e x cos ((33x 2)


d
ex
b

2 x 3x 2
at the point where x = 1.
log e (3 x + 4)

4 log e (2 x )
.
3x

eBook plus
Interactivity

Mixed problems on
differentiation

int-0252
Mixed problems
on differentiation

Problems on differentiation may involve any combination of chain, product and quotient rules.
WORKED EXAMPLE 31

For each of the following decide which rule of differentiation, that is, chain, product or quotient rule,
would be useful to find the derivative.
logg e (x
( )
2
a
b (x2 5x)6
c (x2 + 2x 3) cos (2x)
d ex + 3x
sin ( x )
THINK
a

360

WRITE

Write the equation.

It is of the form

u
, that is, a rational function.
v
Write the equation.

It is a composite function of the form u6, where


u = x2 5x.

Write the equation.

It is of the form u v, that is, the product of two


functions.

Maths Quest 12 Mathematical Methods CAS for the TI-Nspire

a y=

log e ( x )
sin ( x )

Quotient rule
b y = (x2 5x)6

Chain rule
c y = (x2 + 2x 3) cos (2x)

Product rule

d y = ex2 + 3x

Write the equation.

It is a composite function of the form eu, where


u = x2 + 3x.

Chain rule

WORKED EXAMPLE 32

Find the derivative of y = 2x3 cos (x2 + x).


THINK

WRITE

Write the equation.

y = 2x3 cos (x2 + x)

Decide which rule to use and identify u and


v to apply the rule.

Use the product rule.


u = 2x3 and v = cos (x2 + x).

Differentiate u with respect to x.

du
= 6x2
dx

v is a composite function, so differentiate v


with respect to x using the chain rule.

Apply the product rule to find

v = cos (x2 + x)
Let w = x2 + x.
dw
= 2x + 1
dx
v = cos (w)
dv
= sin ( w)
dw
dv dv dw
=

dx dw dx
dv
So
= ( ssin
in ( w)))) (2 x + 1)
dx
= (2x + 1) sin (x2 + x)
dy
dv
du
=u +v
dx
dx
dx

dy
.
dx

= 2x3[(2x + 1)] sin (x2 + x) + cos (x2 + x) 6x2


6

= 2x3(2x + 1) sin (x2 + x) + 6x2 cos (x2 + x)


= 2x2[3 cos (x2 + x) x(2x + 1) sin (x2 + x)]

Simplify where possible.

Derivatives involving the absolute value function


x if x 0
Consider the absolute value function, f ( x ) = x =
.
x if x < 0
As discussed earlier, the graph of f (x) = |x| is continuous for all x
and has a cusp (sharp point) at x = 0.

1 if x > 0
( x ) =
The derivative of f (x) = x, f (
.
x
1 if x < 0
In order for the derivative to be defined at x = 0 for a function f (x), the limit as x approaches 0
from the left, written as lim f '( x ), and the limit as x approaches 0 from the right, written as
x 0

lim f '( x ), must be equal.

x 0+

Chapter 7

Differentiation

361

This is not the case, as lim f ' ( x ) = 1 and lim f ' ( x ) = 1. Thus,
x 0

x 0+

1
the function f '(x) = x is not differentiable at x = 0 and the graph of
y = f '(x) has open circles at x = 0 as shown right.
x
1
The chain rule is used to differentiate the composite function
f (x) = h(g(x)), to give f '(x) = g(x) h(g(x)).
1 if g( x ) > 0
Hence, for f (x) = h(g(x)), where h (x) = x, then f '( x ) = g'( x )
.
1 if g( x ) < 0
Note the derivative is a hybrid function and the domain is obtained by examining the graph of
the function y = g(x).
WORKED EXAMPLE 33

eBook plus

= x2

For the function: f (x)


4x:
a find the derivative
b sketch the graphs of y = f (x) and y = f (x) on the same set of axes.
THINK
a

int-0557
Worked example 33

WRITE

As f (x) = x2 4x is a composite
function, apply the chain rule to find the
derivative,
f (x) where g(x) = x2 4x and h(x) = x.

To determine the domain of the derivative,


consider the graph of the function
g(x) = x2 4x. From the graph, x2 4x > 0
if x < 0 or x > 4 and x2 4x < 0 if 0 < x < 4.

f (x) = h(g(x))
f (x) = g (x) h(g(x))
1
f '( x) = 2 x 4
1
2 x 4 if
f '( x) =
2 x + 4 if

x 2 4 x > 0.
x2 4x < 0

1 2 3 4 5x

2 x 4 if x < 0 or x > 4
f ' ( x ) = 2 x + 4 if 0 < x < 4

Write the derivative with the correct


domain.

To differentiate f (x) = x2 4x with a CAS


calculator, on a Calculator page, press:
MENU b
4:Calculus 4
1:Derivative 1
Alternatively press /r to obtain
the expression template and choose the
derivative template.
d
Complete the entry line as:
x2 4x
dx
Press ENTER .

if x 2 4 x > 0
if x 2 4 x < 0

y
4
3
2
1
0
1
1
2
3
4

Note: The answer from the CAS is not


expressed as a hybrid function with the
correct domain and hence cannot be used.

362

Tutorial

x.sign

(x 4) + sign (x). x 4 is not an


acceptable answer.

Maths Quest 12 Mathematical Methods CAS for the TI-Nspire

Sketch the graph of f (x) = x2 4x.

Sketch the graph of the derivative,


2 x 4 if x < 0 or x > 4
f '( x) =
2 x + 4 if 0 < x < 4
For the graph of y = f (x), put open
circles at x = 0 and x = 4 as the derivative
is not defined at these points.

f'(x) = 2x 4, x > 4

10
8
6
4
2
f(x) =x 4x 2
x
3 2 1 0 1 2 3 4 5 6 7
2
4
f '(x) = 2x + 4, 0 < x < 4
6
8
f '(x) = 2x 4, x < 0

To graph f (x) = x2 4x and y = f (x)


with a CAS calculator, on a Graphs
page, complete the function entry
lines as:
f1(x) = x2 4x
d
f 2(x) =
x2 4x
dx
and press ENTER after each entry.
Note: The open circles at x = 0 and x = 4
are not displayed.

WORKED EXAMPLE 34

For the function f (x) = sin (x) for x [0, 2]:


a find the derivative
b sketch the graphs of y = f (x) and y = f (x) on the same set of axes.
THINK
a

WRITE

As f (x) = |sin (x)| is a composite


function, apply the chain rule to find the
derivative, f (x) where g(x) = sin (x) and
h(x) = |x|.

f (x) = h(g(x))
f (x) = g (x) h(g(x))
sinn ( x ) > 0
1 if si
f ' ( x ) = cos
cos ( x )
sin ( x ) < 0
1 if sin
cos ( x ) if sin ( x ) > 0
f '( x) =
in ( x ) < 0
cos ( x ) if ssin

To determine the domain of the


derivative, consider the graph of the
function g(x) = sin (x) for x [0, 2].
From the graph, sin (x) > 0 if
0 < x < and sin (x) < 0 if
< x < 2.
Write the derivative with the correct
domain.

y
1
0.5
0
0.5
1

3
2

cos ( x ) if 0 < x <


f '( x) =
cos ( x ) if < x < 2

Chapter 7

Differentiation

363

Sketch the graph of f (x) = |sin(x)| for


x [0, 2].

y
1

Sketch the graph of the derivative,


cos ( x ) if 0 < x <
f '( x) =
cos ( x ) if < x < 2
For the graph of y = f (x), put open
circles at x = 0, x = and x = 2 as the
derivative is not defined at these points.

f(x) =sin (x)

0.5
0

0.5
1

3
2

f '(x) = cos (x), 0 < x < f '(x) = cos (x), < x < 2

REMEMBER

1. Chain rule:
dy dy du
(a)
=

dx du dx
(b) A short way to apply the chain rule is:
If f (x) = [g(x)]n then f (x) = n[g(x)n 1 g(x).
2. Product rule:
dy
dv
du
(a) If y = uv then
=u +v .
dx
dx
dx
(b) If f (x) = u(x) v(x) then f (x) = u(x) v(x) + v(x) u(x).
3. Quotient rule:
du
dv
v
u
u
dy
dx
dx.
(a) If y = then
=
v
dx
v2
u( x )
v ( x ) u ( x ) u( x ) v ( x )
(b) If f ( x ) =
then f ( x ) =
.
v( x)
[v ( x)]
x 2
EXERCISE

7J

Mixed problems on differentiation


1 WE 31 For each function given below, state which rule of differentiation would be used to find
the derivative, that is, chain (C), product (P) or quotient (Q).
3x + 7
a f (x) = loge (8x)
b f (x) = 3x sin (x)
c g( x ) =
4x2
d h( x ) =

4x
cos ( x )

e g(x) = e5x sin (x)

g h(x) = cos (x2 4x)

h f (x) = e x loge (5x)

j f (x) = sin2 (x)

k h( x ) =

m g(x) = ecos (x)

x2
ex
n f (x) = tan (x)

g( x ) =

x2 + 9x 8
log e ( x )

i g(x) = loge [sin (x)]


l

f ( x ) = log e ( x )

2 Using the appropriate rule find the derivative of each function in question 1.
3 WE 32 Find the derivative of each of the following. (Note that more than one rule will need to
be applied in some cases.)
x2

a y = e 5x cos (4x 7)
b y=
3x + 1
c y = loge (x + 1)3

364

Maths Quest 12 Mathematical Methods CAS for the TI-Nspire

d y = cos (x2 6x)

e f (x) = ex cos (2x)


1
sin ( x )

g f ( x) =

j f (x) = (x 1)(x2 + 5x + 3)

k g(x) = ex(x2 + 3)
4x

cos (4x 3)

s y=

p f ( x) =

cos (2 x )
sin (2 x )

log e

g( x ) =

(2 x + 3)5
x3 5

n y = cos2 (3x)

o y = loge [cos (3x)]


q f ( x) =

sin (2 x )
cos (2 x )

h y = loge [sin (3x)]

i y = 4e3x2 5x + 2

m f (x) = e

f ( x) =

sin ( x 4 )
x2

r f (x) = [loge (5x 1)]4

( x)

x + 3
y = sin
x 2
3

log e ( x 2 )
v g( x ) =
x2

x f (x) = 3 cos2 (x) + e 7x x3

u f (x) = 3x5 cos (2x + 1)


w y = ex sin (x)
y f (x) = 3 sin (6x) + loge

(5x2)

x
2
4e

z h (x) = cos3 (x)

4 WE 33 For the following functions:


a f (x) = x2 1
b f (x) = x2 + 2x
i find the derivative
ii sketch the graphs of y = f (x) and y = f (x) on the same set of axes.
5 WE 34 For the following functions:
a f (x) = sin (2x) for x [0, ]
b f (x) = cos (x) for x [0, 2]
i find the derivative
ii sketch the graphs of y = f (x) and y = f (x) on the same set of axes.

Chapter 7

Differentiation

365

SUMMARY
Review gradient and rates of change

The gradient of a function exists wherever the graph of the function is smooth and continuous.
If the gradient of a function, f (x), is zero at x = a, then the graph of its gradient function, f (x), will have an
x-intercept at x = a.
When the gradient of a function is positive, the graph of the gradient function is above the x-axis and when
the gradient of a function is negative, the graph of the gradient function is below the x-axis.
A polynomial function has a gradient function which is also a polynomial function, but its degree is reduced
by one.
Limits and differentiation from first principles

The gradient of a chord (secant) or the average rate of change is given by:
f ( x + h) f ( x ) .
h
A limit is the value that y approaches as x approaches a given value.
A limit exists if the function is approaching the same value from both left and right.
The gradient of the tangent to a curve at a point P is the gradient of the curve at P and is
f ( x + h) f ( x )
given by lim
.
h0
h
dy
For a function y = f (x), its derivative is expressed as either
or f (x).
dx
The derivative of xn

If f (x) = axn then f (x) = naxn 1, where a and n are constants.


If f (x) = c then f (x) = 0, where c is a constant.
If f (x) = ag(x) where a is a constant then f (x) = ag(x).
If f (x) = g(x) + h(x) then f (x) = g (x) + h (x). Differentiate each term separately.
The chain rule

The chain rule of differentiation is:


dy dy du
=

dx du dx
The derivative of ex

If f (x) = ex then f (x) = ex.


If f (x) = ekx then f (x) = kekx.
If f (x) = aekx + c then f (x) = akekx + c.
If f (x) = aeg(x) then f (x) = g (x) ae g(x).
The derivative of loge (x )

1
.
x
1
If f (x) = loge (kx) then f (x) = .
x
g ( x )
If f (x) = loge [g(x)] then f (x) =
.
g( x )

If f (x) = loge (x) then f (x) =

The derivatives of sin (x ), cos (x ) and tan (x )

If f (x) = sin (x) then f (x) = cos (x).


If f (x) = cos (x) then f (x) = sin (x).

366

Maths Quest 12 Mathematical Methods CAS for the TI-Nspire

1
= sec2 (x).
cos2 ( x )
If f (x) = sin (ax) then f (x) = a cos (ax).
If f (x) = cos (ax) then f (x) = a sin (ax).
a
If f (x) = tan (ax) then f (x) =
= a sec2 (ax).
2
cos (ax )
If f (x) = sin [g(x)] then f (x) = g (x) cos [g(x)].
If f (x) = cos [g(x)] then f (x) = g (x) sin [g(x)].
g ( x )
If f (x) = tan [g(x)] then f (x) =
= g (x) sec2 [g(x)].
cos2 [ g( x )]

If f (x) = tan (x) then f (x) =

The product rule

The product rule of differentiation states:


dy
dv
du
1. if y = u v, then
=u +v
dx
dx
dx
2. if f (x) = u(x) v (x) then
f (x) = u(x) v(x) + v(x) u(x).
The quotient rule

The quotient rule of differentiation states:


du
dv
v
u
u
dy
dx
dx .
1. if y = then
=
v
dx
v2
u( x )
2. if f ( x ) =
then
v( x)
v ( x )u ( x ) u( x )v ( x )
f ( x ) =
[v ( x)]
x 2
Summary of derivatives:
f (x)

f (x)

axn

naxn 1

[g(x)]n

ng(x)[g(x)]n 1

ex

ex

ekx

kekx

eg(x)

g(x)eg(x)

loge (x)

1
x
1
x
g ( x )
g( x )
a cos (ax)
a sin (ax)
a
cos2 (ax )

loge (kx)
loge [g(x)]
sin (ax)
cos (ax)
tan (ax)

(= a sec2 (ax))

Chapter 7

Differentiation

367

CHAPTER REVIEW
y

SHORT ANSWER

1 The graph of a cubic function is shown below.


y

1 0

1 2 3

Sketch the graph of its gradient function.


h 3 + 2h 2 + 4 h
Find lim
.
h0
h
a Find the derivative of f (x) = x3 + 2x using first
principles.
b Hence find the gradient at the point where
x = 1.
x3
a Find the gradient function if g( x ) =
4x .
3
b Find the gradient of g (x) when x = 3.
3x 4 x3
If h( x ) =
+ 3 x, find:
2
4
a h (x)
b i h(1)
ii h(2).
3
(4 x + 1) 2 ,

6 If y =
find the gradient when a x = 2
and b x = 1.
7 Find the derivative of x 2 + 4 .
8 Differentiate f (x) = e2x 1.
x2

9 a Find f (x) if f (x) = e .


b What is the value of x when f (x) = 0?
dy
if y = loge (2x3 4).
10 Find
dx
11 The tangent to the curve f (x) = loge (ax 1) when
x = 2, has a gradient of 1. Find the value of a.

a On the same set of axes, sketch the graph of the


derivative function.
b Write down the domain of the derivative
function.
EXAM TIP

a Common errors were drawing the cusp point at x = 0 as


a closed circle as well as the point at x = 3.
b In this question students often gave responses that were
inconsistent with the graph they had drawn in a. Union
and intersection were sometimes confused as was the
use of round, square or curly brackets.
[Assessment report 1 2007]

[ Vcaa 2007]

19 Find the derivative of y = f (x) where


f (x) = x2 2x .
20 Find the derivative of y = f (x) where f (x) = sin (x)

for x
, .
2 2
MULTIPLE CHOICE

1 The graph of f (x) is shown below.


y

13 Find f '(x) if f (x) = tan (5x).


14 Find f '(x) if f (x) = tan (2x2 3).
dy
15 If y = 3x2 loge (6x), find .
dx
cos ( x 2 )
16 Find f '(x) if f ( x ) =
.
x2
17 Differentiate esin (2x).

f(x)

12 Find f '(x) if f (x) = 3 sin (2x) and hence find f '(2).

The graph of its gradient function is:


y
b
a

18 The diagram (top, right) shows the graph of a


function with domain R.

368

f(x)

Maths Quest 12 Mathematical Methods CAS for the TI-Nspire

y
0

y
4

2
4x 9
6x

9 If y = 5e

then

a 30e 6x

c 5e 6x 1

e 5e 7x

2 For the function g(x) graphed below, the gradient


function g(x) is defined over the domain:
y
a R
b R \{1}
c R \{4}
d R \{1, 4}
x
e [1, 4]
0 1
4
x2 2x 8
is:
x4
x4
a undefined
b 0
c 1

d 4

5x
is:
x +1
a 22
b 22
c 6
d undefined
5 The derivative of f (x) = 4x3 x2 + 3x is:
a 12x2 2x + 3
b 4x2 2x + 3
c 12x2 2x
d 12x2 x + 3
e 4x2 2x
1
6 The derivative of g( x ) = 2 2 x is:
x
1
2
2
2
a

b 3
x
x
x
x
2
2
1
2
c
d

x
x3
x
x
2
1
3
e
x3
x2
4 lim

e 6

x3

7 The derivative of (2x + 5)6 is:


a 6(2x + 5)5
b 12x(2x + 5)5
5
d 12(2x + 5)
e 12(2x + 5)4
1
8 The derivative of
is:
4x 9
b

c 6x(2x + 5)5

(4 x 9)3

dy
is equal to:
dx

b 6e 6x

d 30e 6x 1
dy
is:
dx

a 4e4x + 6
c e4x + 7
e 4e3x + 7

b e4x + 6
d 4e4x + 7

11 The derivative of loge (3x 2) is:


1
1
a
b
3x 2
3x
1
3(3 x 2)

1
x

3
3x 2

12 The derivative of 2 loge (x2 + x) is:


2(2 x + 1)
c 2x + 1
x
x2 + x
4x
e
2
x +x
dy
13 If y = cos (8x) then is:
dx
a 8 sin (8x)
b sin (8x)
2(2 x + 1)
x2 + x
2x
d 2
x +x
a

x 3

a 2 4x 9

10 If y = e4x + 7 then

3 The value of lim

d 4 4x 9

(4 x 9) 2

c 8 sin (8x)
e

sin

d 8x sin (8x)

(8x)

14 If y = 2 sin (2x + 3) then


a 4x cos (2x + 3)
c 4 cos (2x + 3)
e 2 cos (2x + 3)

dy
is equal to:
dx
b 4 cos (2x + 3)
d 4 cos (2x)

15 If f (x) = tan (6 5x), then f (x) is equal to:


5
5
a
b
2 (6 5x
2
cos
s
5 )
coss (6 5 x )
2
2
c sec (6 5x)
d 5 cos (6 5x)
e 5 cos2 (6 5x)
16 If f (x) = x2 e2x then f (x) is equal to:
a 2xe2x + 2x2e2x
c 4xe2x
e 2xe2x + x2e2x

b 2xe2x
d 2xe2x 2x2e2x

Chapter 7

Differentiation

369

17 If g (x) = 2x loge (3x) then g(x) must be:


2

a 2 log e (3x
3 )+ 3

b 2 loge (3x) + 2

c 2 loge (3x) + 6x

3 ) 3
d 2 log e (3x

e 2 loge (3x) + 6x loge (3x)


2x + 1
18 The derivative of
is:
x2
4x 5
a
( x 2)2
4x 3
c
( x 2)2
5
e
( x 2)2
19 The derivative of

( x 2)2

d 4x 5

e4 x
is:
x2

( x 2)e 4 x
x3
2(2 x 1)e 4 x
c
x3
a

2(1 2 x )
x3

x 2 e 4 x 2e 4 x
x4

2e 4 x
x3
20 If g(x) = (x2 + 3x 7)5 then g(x) is equal to:
a 5(x2 + 3x 7)4
b (2x + 3)(x2 + 3x 7)4
4
c 5(2x + 3)
d 5(2x + 3)(x2 + 3x 7)4
2
4
e (x + 3x 7)
e

21 The derivative of sin (x) cos (x) is:


a 2 sin (x) cos (x)
b sin2 (x) cos2 (x)
2
2
c sin (x) + cos (x)
d cos2 (x) sin2 (x)

2
2
e sin (x) cos (x)
22 Which one of the following is not true about the
function f: R R, f (x) = |2x + 4|?
a The graph of f is continuous everywhere.
b The graph of f is continuous everywhere.
c f (x) 0 for all values of x.
d f (x) = 2 for all x > 0
e f (x) = 2 for all x < 2
[ Vcaa 2007]
23 If y = |cos (x)|, the rate of change of y with respect
to x at x = k, < k < 3 , is:
2
2
a sin (k)
b sin (k)
c cos (k)
d sin (1)
e sin (1)
[ Vcaa 2005]
24 Let f: R R be a differentiable function. For all
real values of x, the derivative of f (e2x) with respect
to x will be equal to:
a 2e2x f (x)
b e2x f (x)
c 2e2x f (e2x)
d 2f (e2x)
e f (e2x)
[ Vcaa 2005]

EXTENDED RESPONSE
1

1 A section of a roller-coaster ride follows part of the curve with the equation y = 200 ( x 3 + 30 x 2 ) as
shown below.
a For what values of x (domain) is the gradient:
y
i zero?
ii positive?
iii negative?
b Sketch the gradient function.
c Use the graph of the gradient function to find the value of x where the
0
28 20
gradient is steepest over the domain [25, 10].
dy
d Find .
dx
e Find the gradient where x equals:
i 25
ii 10
iii 10.
f Does this verify your answer to part c? Briey explain.
g What is the highest point reached by the roller-coaster? (Give your answer in metres.)
2 Consider the functions f ( x ) = 2 x and g(x) = x2 + 1.
a State the domain and range for each function.

370

Maths Quest 12 Mathematical Methods CAS for the TI-Nspire

12

b Find the composite functions:


i f (g(x))
ii g(f (x)).
c State the domain and range for f (g(x)) and g(f (x)).
d Find:
d
i
( f ( g(
g( x ))))
dx
d
ii
( g( f ( x ))))
dx
e Evaluate:
i f (g(2))
ii g (f (2))
4 x 2 , x 2

f
(
x
)
=
3 Consider the function
2 x 4, 2 < x < 5 .
x 1, x 5
a Sketch the graph of f (x).
b For what values of x is f (x) discontinuous?
c For what values of x is f (x) not differentiable?
d Find f (x).
e Sketch the graph of f (x).

eBook plus
Digital doc

Test Yourself
Chapter 7

Chapter 7

Differentiation

371

eBook plus

ACTIVITIES

chapter opener
Digital doc

10 Quick Questions: Warm up with ten quick


questions on differentiation. (page 324)
7a

Review gradient and rates of change

Tutorial

WE 3 int-0552: Watch how to sketch a gradient


function. (page 327)
Digital docs

SkillSHEET 7.1: Practise identifying positive


negative and zero gradients. (page 328)
SkillSHEET 7.2: Practise sketching the gradient
function given the original function. (page 328)
7b

Limits and differentiation from


first principles

eLesson eles-0093

Limits and differentiation from first principles.


Watch an eLesson on related rates of change
(page 332)
Tutorial

WE 5 int-0553: Watch how to evaluate limits.


(page 333)
7c

The derivative of x n

Digital doc

SkillSHEET 7.3: Practise using index laws.


(page 340)
7d

The chain rule

Tutorial

WE 13 int-0554: Watch a worked example on using


the chain rule. (page 341)
Digital doc

WorkSHEET 7.1: Sketch gradient functions,


identify where the derivative exists, evaluate limits,
apply first principles and differentiation rules to
determine derivatives. (page 344)
7G

The derivatives of sin (x), cos (x) and tan (x)

7H

Digital doc

WorkSHEET 7.2: Differentiation of mixed


expression involving the product, quotient and chain
rules. (page 357)
7I

The quotient rule

Tutorial

WE 30 int-0556: Watch a worked example on how to


use the quotient rule. (page 358)
7J

Mixed problems on differentiation

Interactivity int-0252

Differentiation: Consolidate your understanding of


differentiation. (page 360)
Tutorial

WE 33 int-0557: Watch a worked example on how


derivatives of an absolute value function. (page 362)
chapter review
Digital doc

Test Yourself: Take the end-of-chapter test to test


your progress. (page 371)
To access eBookPLUS activities, log on to
www.jacplus.com.au

Tutorial

WE 26 int-0555: Watch a worked example on


using the chain rule to differentiate trigonometric
functions. (page 353)

372

The product rule

Maths Quest 12 Mathematical Methods CAS for the TI-Nspire

8A Equations of tangents and normals


8B Sketching curves
8c Maximum and minimum problems
when the function is known
8D Maximum and minimum problems
when the function is unknown
8e Rates of change
8F Related rates
8G Linear approximation

Applications of
differentiation
AreAS oF STudy

Application of differentiation to curve sketching


and identification of:
key features of curves
intervals over which a function is constant,
stationary, strictly increasing or strictly
decreasing
the maximum rate of increase or decrease in
a given application context (consideration of
second derivative is not required)
tangents and normals to curves

Identification of:
local maximum/minimum values over an
interval and application to solving problems
interval end point maximum and minimum
values
Average and instantaneous rates of change,
including formulation of expressions and
solution and interpretation of problems
involving rates of change and simple cases of
related rates of change
The relationship f ( x + h) f ( x ) + hf
hf '( x ) for
a small value of h and its geometric interpretation
eBook plus

8A

Digital doc

equations of tangents
and normals
As we have seen, a tangent to a curve is a straight line
that touches the curve at a given point and whose
gradient represents the gradient of the curve at that
point. A normal to a curve is a straight line passing
through the point where the tangent touches the curve
and is perpendicular (at right angles) to the tangent at
that point.

10 Quick Questions

f(x)

Tangent

Point of tangency
Normal
x

1
If the gradient of the tangent to a curve is m, then the gradient of the normal is
(as the
m

product of the gradients of two perpendicular lines equals 1).

The equation of a straight line passing through the point (x1, y1) and having a gradient of m is:
y y1 = m(x x1)
The gradient of the tangent at x = a is f(a).

1
Therefore the gradient of the normal is
.
f (a)

Chapter 8

Applications of differentiation

373

The equation of the tangent at x = a is yf (a) = f(a) (x a)


and the equation of the normal is
1
y f (a)=
( x a).
f (a)

f(x)

f(a)
0

(a, f(a))
a

Worked exAmple 1

Find the equation of the tangent to y = x3 2x + 3 at the point (1, 2).


Think

WriTe

Write the equation.

Find

dy
.
dx

dy
where x = 1 to find the gradient of
dx
the tangent where x = 1.

y = x3 2x + 3
dy
= 3x2 2
dx

Evaluate

At x = 1,
dy
=32
dx
=1
So gradient of tangent is 1.

Substitute (1, 2) for (x1, y1) and m = 1 into the


rule for the equation of a straight line:
yy1=m(xx1).

Equation of tangent at the point (1, 2) is


y 2 = 1(x 1)
y2=x1

Rearrange the rule to a simple form.

y=x+1

Worked exAmple 2

Find the equation of:


a the tangent
b the normal to the curve with the equation
y = 3 loge (2x) at x = 1
c the tangent and the normal to the curve y = 3 loge (2x) at x = 1 using a
CAS calculator.
Think

Write the equation.

y = 3 loge (2x)

Evaluate y when x = 1.
dy
Find .
dx

At x = 1, y = 3 loge (2)
dy 3(2)
=
dx
2x
3
=
x
dy 3
a At x = 1,
=
dx 1
=3
So gradient of tangent is 3.

374

Tutorial

int-0558
Worked example 2

WriTe

eBook plus

dy
when x = 1 to obtain the
dx
gradient of the tangent at x = 1.

Evaluate

Determine the equation of the


tangent at (1, 3 loge (2)).

Equation of tangent is
y 3 loge (2) = 3(x 1)
= 3x 3
y = 3x 3 + 3 loge (2)

maths Quest 12 mathematical methods CAS for the Ti-nspire

Evaluate the gradient of the normal


which is

dy
dx

1
.
3

b Gradient of normal is

Determine the equation of the


normal at (1, 3 loge (2)).

On a Calculator page, press:


MENUb
4:Calculus4
Select either
9:Tangent Line 9 or
A:Normal Line A.
Complete the entry lines as:
tangentLine(3ln (2x),x,1)
normalLine(3ln (2x),x,1)
Press ENTER after each entry.

Note the answers from the CAS are


not written as equations.

Equation of normal is
1
y 3 loge (2) = 3 (x 1)
3y 9 loge (2) =1(x 1)
=x + 1

x + 3y = 1 + 9 loge (2)

y = 3 x + 3 (log e (2 1)) is the equation of the


tangent.
x
9 log e (2 + 1)
y=
is the equation
+
3
3
of the normal.

rememBer

1
1. If m is the gradient of the tangent, then
is the gradient of the normal. That is,

m
m1 m2 = 1.
2. The equation of a straight line passing through the point (x1, y1) with gradient m is
y y1 = m(x x1)

3. Parallel lines have the same gradient.


exerCiSe

8A

equations of tangents and normals


1 We1
Find the equation of the tangent to the curve y = x2 + x
at the point (2, 6).
2
3

Find the equations of the tangent to the curve y = x2 + 5x 6 at


the points where it crosses the x-axis.

eBook plus
Digital doc

Spreadsheet 129
Tangent and normal

Find the equation of the normal to the curve y = 3x2 5x + 4 at the point where x = 1.

Chapter 8

Applications of differentiation

375

Find the equation of the normal to the curve y = 12 x2 + 3x 7 at the point where it crosses
the y-axis.

5 We2 For each of the following functions, find the equation of:
i the tangent
ii the normal at the given value of x.
a y = x2 + 1, x = 1
b y = x36x, x =2
1
c y = , x = 2
d y = (x 1)(x2 + 2), x=1
x
e
g
i
k

y = x , x = 4
y = x(x + 2)(x 1), x =1
y = 2x3 + x2 6x + 2, x = 1
y = e3x + 2, x =1

y = 2 x + 3, x = 3
y = x3 3x2 + 4x, x = 0
y = e2x, x = 0
y = loge (x), x = 2

n y = sin (2x), x =
3

p y = sin 2 x + , x = 0

4
f
h
j
l

m y = loge (2x + 3), x = 0


x
o y = 3 cos , x =
2

6 mC If y = (2x + 3)4 then at the point (1, 1)


a the equation of the normal is:
A y + 4x 3 = 0
B 8y + x 7 = 0
D y + 2x + 8 = 0
e 2y x = 0

c y 4x + 5 = 0

b the value of x where the gradient of the tangent is parallel to the x-axis is:
A

2
3

1
3

3
2

1
3

2
3

7 Find the equation of the tangent to f (x) = x2 + 4x + 1 which is parallel to the line y = 2x + 3.
x2 + 1
at x = 0.
x2 1

9 Find the equation of the normal to y = x sin (x) at x = .


2
10 Find the equation of the normal to y = loge (x + 2) which is parallel to the line with equation
y + 3x 5 = 0.
8 Find the equation of the tangent to y =

11 Find the equations of the tangent and normal for each of the following
curves.
a f (x) = x2 + 1 at x = a
b f (x) = x at x = a

eBook plus
Digital doc

WorkSHEET 8.1

c f (x) = e x at x = 2a.
2

12 Find the equation of the tangent to the curve f (x) = e4x that is perpendicular to the line x + 8y = 16.
13 The graph of y = x has a normal with equation y = 8x + b, where b is a real constant. Find
the value of b.

8B

Sketching curves
When the graphs of polynomial functions are being sketched, four main characteristics should
be featured:
1. the basic shape (whenever possible)
2. the y-intercept
3. the x-intercept(s)
4. the stationary points.

376

maths Quest 12 mathematical methods CAS for the Ti-nspire

Stationary points
A stationary point is a point on a graph where the function momentarily stops rising or falling;
that is, it is a point where the gradient is zero.
y

or
x
Gradient = 0 where function
stops rising momentarily,
then continues to rise again
after this point

0
0

x
Function stops falling
and rises after this point

The stationary point (or turning point) of a quadratic function can be found by completing a
perfect square in the form y = (x + h)2 + k. In this case the stationary point is (h, k). For cubics,
quartics or higher-degree polynomials there is no similar procedure. Differentiation enables
stationary points to be found for any polynomial function where the rule is known.
The gradient function of a function f(x) is f(x).
Stationary points occur wherever the gradient is zero.
f(x) has stationary points when f(x) = 0
or
y has stationary points when

dy
= 0.
dx

The solution of f(x) = 0 gives the x-value or values where stationary points occur.
If f(a) = 0, a stationary point occurs when x = a and y = f(a). So the coordinate of the
stationary point is (a, f(a)).

Types of stationary points


There are four types of stationary points.
1. A local minimum turning point at x = a.
If x < a, then f(x) < 0 (immediately to the left of x = a,
the gradient is negative).
If x = a, then f(x) = 0 (at x = a the gradient is zero).
If x > a, then f(x) > 0 (immediately to the right of x=a,
the gradient is positive).

f(x)

f '(x) > 0
f '(x) < 0
0

2. A local maximum turning point at x = a.


If x < a, then f(x) > 0.
If x = a, then f(x) = 0.
If x > a, then f(x) < 0.
The two cases (1 and 2) can be called turning points because
the gradients each side of the stationary point are opposite in sign
(that is, the graph turns).
The term local turning point at x = a implies in the vicinity
ofx = a, as polynomials can have more than one stationary
point.

f '(x) = 0
x

y
0

f(x)

Chapter 8 Applications of differentiation

377

3. A positive stationary point of inflection at x = a.


If x < a, then f(x) > 0.
If x =a, then f(x) = 0.
If x > a, then f(x) > 0.
That is, the gradient is positive either side of the stationary
point.

Gradient = 0

f'(x)

4. A negative stationary point of inflection at x = a.


If x < a, then f(x) < 0.
If x =a, then f(x) = 0.
If x > a, then f(x) < 0.
In cases 3 and 4 above the word stationary implies that the
gradient is zero.
Not all points of inflection are stationary points.
y

f(x)

Gradient = 0

or
0

Gradient 0

When determining the nature of stationary points it is helpful to complete a gradient table,
which shows the sign of the gradient either side of any stationary points. This is known as the
rst derivative test.
Gradient tables are demonstrated in the examples that follow.

Worked exAmple 3

eBook plus

a Find the stationary points and their nature for the function
b
c
d
e

f (x) = x3 + 5x2 8x 12.


Show that the curve passes through (1, 0).
Find the coordinates of all other intercepts.
Hence, sketch the graph of f (x).
Use a CAS calculator to find the stationary points.

Think
a

378

Tutorial

int-0559
Worked example 3

WriTe/drAW
a f (x) = x3 + 5x2 8x 12

Write the rule for f (x).

Differentiate f (x) to find f(x).

f(x) = 3x2 + 10x 8

Find all values of x where f(x) = 0.

For stationary points 3x2 + 10x 8 = 0


(3x 2)(x + 4) = 0
x = 23 or x =4

maths Quest 12 mathematical methods CAS for the Ti-nspire

When x = 3 , f( 3 ) = ( 3 )3 + 5( 3 )2 8( 3 ) 12

Find the value of f(x) for each value


of x where f(x) = 0.

22

= 14 27
so ( 23 , 14 22
) is one stationary point.
27
When x = 4,

f(4) = (4)3 + 5(4)2 8(4) 12
= 36
so (-4, 36) is another stationary point.

Complete a gradient table to


determine the nature of the
stationary points.

To find the x-intercepts, factorise


f(x) by long division, or by another
appropriate method, knowing that
(x + 1) is a factor of f(x).

2
3

f(x)

Slope

Therefore (4, 36) is a local maximum stationary


point and ( 23 , 14 22
) is a local minimum stationary
27
point.

State each stationary point and its


nature.

Show that f(1) = 0.

Gradient table:

b f(1) = (1)3 + 5(1)2 8(1) 12


= 1 + 5 + 8 12

=0
Therefore f(x) passes through (1, 0).

c As f(1) = 0 then (x + 1) is a factor of f(x)

and f(x) = (x + 1)(x2 + 4x 12)



= (x + 1)(x + 6)(x 2)

Solve f(x) = 0.

x-intercepts:

(x + 1)(x + 6)(x 2) = 0

x = 1 or 6 or 2

State the coordinates of the


x-intercepts.

The x-intercepts are (1, 0), (6, 0) and (2, 0).

Evaluate f(0) to determine the


y-intercept.

State the coordinate of the


y-intercept.

The y-intercept is (0, 12).

Sketch the graph of f(x) showing all


intercepts and stationary points.

f(0) = (0)3 + 5(0)2 8(0) 12


= 12

d
(4, 36)

f(x)

(6, 0)
(1, 0)

(2, 0) x

(0, 12)
)
( 23 , 14 22
27

Chapter 8 Applications of differentiation

379

To find the stationary points with a


CAS calculator, on a Calculator page:
define the function f(x)
find f (x)
solve f (x) = 0
find the y-values of the stationary
points.
To do this, complete the entry lines as:
Define f(x) = x3 + 5x2 8x 12

d
( f ( x))
dx
solve(3x2 + 10x 8 = 0,x)

2
f 4,
3

Press ENTER after each entry.


2

The stationary points are (4, 36) and 23 ,

Write down the coordinates of the


stationary points.

400
27

Worked Example 4

Sketch the graph of g(x) = x2(4 x2), clearly indicating all stationary points and intercepts.
Think

Write/draw

Write the rule for g(x).

g(x) = x2(4 x2)

Expand g(x) to make it easier to


differentiate.

g(x) = 4x2 x4

Differentiate g(x).

g(x) = 8x 4x3

Solve g(x) = 0.

For stationary points,



g(x) = 0

8x 4x3 = 0

4x(2 x2) = 0
x = 0 or x2 = 2
x = 0 or x = 2 or 2

Find g(x) for each value of x


where g(x) = 0.

When x = 0, g(0) = 0
When x = 2 , g( 2 ) = 4( 2 )2 ( 2 )4

=4
When x = 2, g( 2) = 4( 2)2 ( 2)4

=4
Therefore the stationary points are ( 2 , 4), (0, 0) and ( 2, 4).

380

Complete a gradient table to


determine the nature of the
stationary points.

Gradient table:
x

g(x)

Slope

Maths Quest 12 Mathematical Methods CAS for the TI-Nspire

State the stationary points and


their nature.

Therefore ( 2 , 4) is a local maximum stationary point.


(0, 0) is a local minimum stationary point.
( 2, 4) is a local maximum stationary point.

Solve g(x) = 0 to determine the


x-intercepts.

x-intercepts: When g(x) = 0,



x2(4 x2) = 0

x2 = 0 or x2 = 4

x = 0 or x = 2 or 2
The x-intercepts are (2, 0), (0, 0) and (2, 0).

Find g(0) to determine the


y-intercept.

y-intercept: When x = 0,
g(0) = 02(4 02)
=0
The y-intercept is (0, 0).

10

Sketch the graph of g(x).

( 2, 4)

( 2, 4)

(2, 0)
(0, 0) 0

(2, 0)

x
g(x)

Worked Example 5

If f(x) = x3 + 4x2 3x 7:
a sketch the graph of f(x)
b state the values of x where f(x) is i increasing and i i decreasing.
Think
a

Write/draw
a f(x) = x3 + 4x2 3x 7

Write the rule for f(x).

Differentiate f(x) to find f(x).

f(x) = 3x2 + 8x 3

Solve f(x) = 0 to find the x-intercepts


of f(x).

x-intercepts: When f(x) = 0,



3x2 + 8x 3 = 0

(3x 1)(x + 3) = 0

x = 13 or 3
The x-intercepts of f(x) are ( 13 , 0)
and (3, 0).

Evaluate f(0) to find the y-intercept of


f(x).

Sketch the graph of f(x) (an upright


parabola).

y-intercept: When x = 0,
f(0) = 3
so the y-intercept of f(x) is (0, 3).
y

f '(x)

1
3

Chapter 8 Applications of differentiation

381

f(x) increases where f(x) > 0.


Byinspecting the graph of f(x) deduce
where f(x) is positive (that is, above
the x-axis).

i f(x) > 0 where x < 3 and x >

so f(x) is increasing where


x < 3 and x > 13 .

i i f(x) < 0 where 3 < x <

ii f(x) decreases where f(x) < 0.

Byinspecting the graph of f(x),


deduce where f(x) is negative (that is,
below the x-axis).

1
3

1
3

so f(x) is decreasing where 3 < x < 13 .

Worked Example 6

Consider the function f ( x ) = ( x + 3)( x a)2 where a is a positive real constant.


a Find f (x).
b Find the coordinates of the stationary points.
c Determine the nature of the stationary points.
dFind the value of a in exact form if the straight line with equation y = 5x + 15 intersects y = f(x) at
the maximum turning point.
Think
a

382

Write

Store the function to f(x). On a


Calculator page, press:
MENU b
1:Actions 1
1:Define 1
Complete the entry line as:
Define f(x) = (x + 3)(x a)2
Press ENTER .

On the same Calculator page,


select the derivative template:
Complete the entry line as:
d
( f ( x))
dx
Press ENTER .

Write the derivative with the


correct notation.

Solve f (x) = 0 to find the x-values


of each stationary point.
On the same Calculator page,
press:
MENU b
3:Algebra 3
1:Solve 1
Complete the entry line as:
solve((x + 3)(x a)2 = 0,x)
Press ENTER .

f (x) = (x a)(3x a + 6)
b

Maths Quest 12 Mathematical Methods CAS for the TI-Nspire

a 6 4(a + 3)3

and (a, 0) are the


,
27
3

Substitute each value of x into


f(x) to find the y-coordinates of
the stationary points. On the same
Calculator page, complete the
entry line as:

coordinates of the stationary points.

a 6
f
, a .
3

Press ENTER .
Write the stationary points in
coordinate form.
c

To determine the nature of the


stationary points, let a = 1 (as a is
given as a positive real constant).

c 1 6 4(1 + 3)3

3 ,

5 256
,
and (1, 0) are the stationary points.
3 27

Substitute a = 1 into the answer


found previously to obtain the
stationary points.
2

Complete a gradient table to


determine the nature of the
stationary points.

State the stationary points and


their nature.

The maximum turning point lies


on the straight line
y = 5x + 15. Substitute the
coordinates of the turning point
into y = 5x + 15 and solve for a.

On a Calculator page, press:


MENU b
3:Algebra 3
1:Solve 1
Complete the entry line as:

f(x)

Slope

(1, 0) is a local minimum turning point.


5 256
3 , 27 is a local maximum turning point.
(a, 0) is a local minimum turning point and

State the stationary points and


their nature in terms of a.

and (1, 0)

27

a 6 4(a + 3)3
is a local maximum turning point.
3 ,
27
d

4 ( a + 3)3 5(a 6)

solve
=
+ 15, a
27
3

Press ENTER .
3

Select the appropriate value of a


given that it is positive.

a=

52

Chapter 8 Applications of differentiation

383

REMEMBER

1. A stationary point (SP) occurs when f(x) = 0.


2. There are four types of stationary points:
(a) local maximum, where the gradient is positive on the left of the SP and negative on
the right
(b) local minimum, where the gradient is negative on the left of the SP and positive on
the right
(c) positive point of inflection, where the gradient is positive on both sides of the SP
(d) negative point of inflection, where the gradient is negative on both sides of the SP.
Exercise

8B

Sketching curves
1 WE3
Find the stationary points and their nature for each of the following functions.
a y = 8 x2
b f(x) = x3 3x
c g(x) = 2x2 8x
2
3
3
4
d f(x) = 4x 2x x
e g(x) = 4x 3x
f y = x2(x + 3)
2
3
g y = 5 6x + x
h f(x) = x + 8
i y = x2 x + 6
4
3
2
2
j y = 3x 8x + 6x + 5
k g(x) = x(x 27)
l y = x3 + 4x2 3x 2
3
3
m h(x) = 12 x
n g(x) = x (x 4)
2 WE3 Sketch the graph of each function in question 1 , clearly indicating all stationary points.
3 a  WE3 Find the stationary points of the function f(x) = x3 2x2 7x 4 and state their nature.
b Show that the graph passes through (4, 0).
c Give the coordinates of all other intercepts and hence sketch the graph of f(x).
4 a Find the stationary points, and their nature, for the curve y = x3 x2 16x + 16.
b
Show that the graph passes through (1, 0) and give the coordinates of all other
intercepts.
c Sketch the graph.
5 a WE3 Find the stationary points of the function g(x) = x4 4x2 and state their nature.
b Find the coordinates of all the intercepts.
c Sketch the graph of g(x).
6 a
b
c
d

If y = x4 6x2 + 8x 3, find each stationary point and its nature.


Show that the point (1, 0) lies on the curve.
Find all other intercepts.
Sketch the graph.

7 a If y = x4 + x3 5x2 6, find each stationary point and its nature.


b Find the y-intercept.
c Sketch the graph without finding the x-intercepts.
8 WE4 Sketch the graph of each of the following functions, clearly indicating all stationary
points and intercepts.
a f(x) = x4 x2
b f(x) = x3 3x2
3
4
c g(x) = x + 3x
d g(x) = x3 4x2 + 4x
3
2
e h(x) = x 4x 11x + 30
f h(x) = x(x + 3)(x 5)
g f(x) = x4 2x2 + 1
h f(x) = x(x2 + 1)
3
2
i g(x) = x + 9x + 24x + 20
j h(x) = (x2 1)3
9 MC If f(x) < 0 where x > 2 and f(x) > 0 where x < 2, then at x = 2, f(x) has a:
A local minimum
B local maximum
c point of inflection
D discontinuous point
e gradient of 2
384

Maths Quest 12 Mathematical Methods CAS for the TI-Nspire

10 mC The function f (x) = x3 + x2 8x 3 has stationary points when x is equal to:


A 2 and

4
3

c 2 only

B 3 only

D 3 and

4
3

e 0 and 2

11 mC The graph of y = + has:


A a local maximum where x = 0
B a local minimum where x = 0
x4

x3

3
4

c a local minimum where x =

3
4

D a local maximum where x =

4
3

e a local maximum where x =

12 mC A quadratic function has a turning point (2, 1) and a y-intercept of (0, 9).
The equation must be:
A y = (x 2)2 + 9
B y = (x 1)2 + 8
c y = (x 2)2 + 1
2
2
D y = 2(x 2) + 9
e y = 2(x 2) + 1
13 The graphs of f(x) are shown below. Find all values of x for which f (x) has stationary points
and state their nature.
y
y
y
a
b
c

f '(x)

f '(x)

f'(x)
y

f '(x)

y
f '(x)

0 1

f'(x)

14 Show that f (x) = x2 4x + 3 is decreasing for x < 2 and increasing for x > 2.
15 We5 For each of the following functions i sketch f(x) and, hence, state the values of x
where f (x) is ii increasing and iii decreasing.
a f (x) = 13 x3 + 2x2 + 2
b g(x) = x3 + 2x2 7x 5
c h(x) = x4 + 4x3 + 4x2
16 If y = f (x) has the following properties then sketch its graph.
f(x) = 0 if x =2 and x = 3 f(x) < 0 if 2 < x < 3 f(x) > 0 for all other x

17 If f (x) = x3 + ax2 + bx has stationary points at x = 2 and x = 43 , find:


a the value of a and b
b the nature of the stationary points.
eBook plus
Digital doc

SkillSHEET 8.1
Review of
derivatives
other than
polynomials

18 If f (x) = x4 + ax2 + b has a stationary point (1, 4), find:


a a and b
b the other stationary points
c the nature of each stationary point.
19 We6 Consider the function f ( x ) = (b x )( x + 2)2 where b is a positive real constant.
a Find f (x).
b Find the coordinates of the stationary points.
c State the nature of the stationary points.
d Find the equation of the tangent at x = 2.
e If the maximum turning point occurs at x = 4, find the value of b.

Chapter 8

Applications of differentiation

385

20 WE6 Consider the function f ( x ) = ( x + b)2 (2 x 1) where b is a positive real constant.


a Find f (x).
b Find the coordinates of the stationary points.
c Find the equation of the tangent to the curve when x = 2.
d Find the value of b in exact form if the straight line with equation y = 4x 2 intersects
y = f (x) at the minimum turning point.

8c

Maximum and minimum problems


when the function is known
In many practical situations the maximum
local
or minimum value of a quantity is desired.
y
maximum
maximum
For example it is important for manufacturers or
business operators to minimise the costs involved
in running their businesses. Equally, it is just as
important to maximise their profits.
local
A graph is always useful and helps us to find
minimum
x
0
approximately where a maximum or minimum
minimum
occurs.
As we have seen, local maximum and/or minimum stationary points occur where the
derivative is zero.
dP
If P = f(x) then a local maximum and/or minimum may exist where
= 0.
dx
To decide whether a solution is a maximum or a minimum the first derivative test must be
applied (by setting up a gradient table).
A graph of the function could be sketched if it is not difficult, to ensure that the maximum or
minimum value is applicable.
y
f(x)

1 2 3 4 5

The function f(x) has a limited domain of [1, 5].


The maximum value occurs at the point where x = 4.
The minimum value, however, is not at x = 2 but at the end point x = 5.
This example shows that the derivative test on its own is not always reliable for finding
maximum or minimum values and a graph is sometimes necessary.

Solving maximum and minimum problems


When finding the maximum/minimum value of f(x) the following steps are taken.
1. Find f(x) (to obtain the gradient function).
2. Solve for x where f(x) = 0 (to find the values of x where the maximum or minimum occur).
3. Apply the first derivative test as a check. Also, sketch the graph of f(x).
4. Check end points if domain is restricted.
5. Substitute the appropriate value of x into f(x) to obtain the maximum or minimum.

386

Maths Quest 12 Mathematical Methods CAS for the TI-Nspire

Worked exAmple 7

The population of a colony of birds at any time, t months,


after observation began can be modelled by the function:
t

P(t) =400 te 5 + 600,


where P is the number of birds.
Find:
a the initial population
b when the largest number of birds is reached
c the maximum number of birds.
Think
a

WriTe

Write the rule for P(t).

The initial value occurs when t = 0 and is P(0).

Find P(t) using the product rule.

a P(t) = 400te

P(0) =
+ 600
= 0 + 600
= 600
So the initial population of birds is 600.
b P(t) = 400te

=
Solve P(t) = 0.

+ 400t

( )

t
t
400 e 5 80te 5
t
80 e 5 (5 t)

t
5

1
5

t
5

+0

(5 t) = 0

t = 5 (as 80 e
Check that it is a maximum using the first
derivative test.

t
5

For maximum and/or minimum, P(t) =0


80 e

+ 600

400(0)e0

=
2

t
5

t
5

cannot equal 0)

Gradient table:
t

10

P(t)

Slope

Therefore the maximum population


occurs after 5 months.
c

Evaluate P(5) to find the maximum number of birds.

c P(5) = 400(5)e

5
5

+ 600

= 2000e 1 + 600
= 735.8 + 600
= 1335.8
Therefore the maximum number of birds
is approximately 1335 (as it does not
reach 1336).

Worked exAmple 8

eBook plus

The displacement of a particle moving in a straight line from the origin at any
time, t, is given by x(t) = 13 t3 4t2 + 12 t + 1, 0 t 7.
Find the maximum and minimum displacement.

Chapter 8

Tutorial

int-0560
Worked example 8

Applications of differentiation

387

Think

Write

t3
4t2 + 12t + 1
3

Write the rule.

x(t) =

Find x(t).

x(t) = t2 8t + 12

Solve x(t) = 0.

For maximum and/or minimum,


t2 8t + 12 = 0
(t 2)(t 6) = 0
t = 2 or t = 6

Test for maximum and minimum using the first


derivative test.

Gradient table:
x

x(t)

Slope

When t = 2 a local maximum occurs and when


t=6 a local minimum occurs.
5

Evaluate x(2) to find the local maximum.

x(2) = 13 (2)3 4(2)2 + 12(2) + 1


= 83 16 + 24 + 1

= 11 3

2
2

so a local maximum occurs at (2, 11 3 ).


6

Evaluate x(6) to find the local minimum.

x(6) = 13 (6)3 4(6)2 + 12(6) + 1



= 72 144 + 72 + 1

=1
so a local minimum occurs at (6, 1).

Sketch the graph of x(t) over the domain [0, 7] to


test that the maximum and minimum have been
found.

Evaluate x(0) and x(7) to see if the end points give


a smaller or larger value.

This graph indicates


that we have to find
the value of x at the
end points in case
they produce larger
or smaller values
than the stationary
points do.

x(t)

1 2 3 4 5 6 7 t

x(0) = 1 which is equal to the local minimum


above
x(7) = 13 (7)3 4(7)2 + 12(7) + 1

= 114 3 196 + 84 + 1
1

= 3 3 which is between the local maximum


and minimum above.
Therefore the maximum displacement is
2
11 3 units from the origin and the minimum
displacement is 1 unit from the origin.

388

Maths Quest 12 Mathematical Methods CAS for the TI-Nspire

REMEMBER

1. To find a maximum and/or minimum, let f(x) = 0.


2. If the function has a restricted domain, f(x) may not be useful for finding maximum
and/or minimum values.
3. You can prove your maximum and/or minimum value using a slope or gradient
diagram.
Exercise

8c

Maximum and minimum problems when


the function is known
1

For each of the following, write the expression for the derivative indicated.
a C = 5x 2x2 + 10,

dC

dx

dV

dh
dM
e M = k loge 3k,

dk
1

c V = 4 h4 3 h3,

b P = n3 + 2n2 5 n,
d h = t3 + 2et,
f L = t2 +

dP
dn

dh
dt

4
dL
2 cos (t),
t
dt

Questions 2 and 3 refer to the following information.


The function Q = n3 3n2 + 5, 0 n 5, is graphed at right.

2 MC The minimum value occurs where n equals:


A 2
B 0
c 1
d 2
e 5
3 MC The maximum value occurs where n is equal to:
A 0
b 2
c 5
d 1
e 1

4 WE7
The profit, $P per week, of a small manufacturing company is related to the
number of workers, n, by:
P = 12 n3 + 96n + 600
Find:
a the number of workers needed for maximum profit per week
b he maximum profit per week.
5

The cost, $C, of producing x-metre lengths of a certain tow rope is:
C = 15 x2 8x + 100 per rope, x > 0
What is the length of the cheapest tow rope that can be produced?

6 WE8 The number of people, P, visiting a certain beach on a particular day in January depends
on the number of hours, x, that the temperature is below 30C according to the rule
P = x3 12x2 + 21x + 105 where x 0.
Find the value of x for the maximum and minimum number of people who visit the beach.
7 The number of rabbits, N, which feed on a particular farmland on any night can be modelled

as N = 13 x3 + 5x2 + 75x + 500 where x is the average overnight temperature in C and x 5.


Find:
a the temperature for the minimum and maximum number of rabbits
b the minimum and maximum number of rabbits that will be found on the farmland.

Chapter 8 Applications of differentiation

389

8 The velocity, v cm/s, of an engines piston t seconds after the engine is started is approximated
by v = 0.8 sin (2t).
a Find the minimum velocity and the time it first occurs.
b Find the maximum velocity and the time it first occurs.
9 The length of a snake, L cm, at time t weeks after it is born is modelled as:
t
L = 12 + 6t + 2 sin , 0 t 20
4
Find:
a the length at i birth and ii 20 weeks
b R, the rate of growth, at any time, t
c the maximum and minimum growth rate.
10 The population of cheetahs, P, in a national park in Africa since 1 January 1986 can be
t

modelled as N = 100te 12 + 500 where t is the number of years.


a When does this model predict that the maximum population will be reached?
b What is the maximum population of cheetahs that will be reached?
c How many cheetahs will there be on 1 January in i 2010 and ii 2070?
11 In the same park as in question 10 the number of elephants, N, since 1 January 1986 is
t

modelled as N = 100 + 4t + 400 e 10 .


Find:
a the minimum number of elephants predicted
b when this minimum will occur.
12 The profit, $P, per item that a store makes by selling n items of a certain type each day is
P = 40 n + 25 200 2 n.
a Find the number of items that need to be sold to maximise the profit on each item.
b What is the maximum profit per item?
c Hence, find the total profit per day by selling this number of items.
13 The weight (in kg) of a bodybuilder t months after starting a training program is
W = 5t 20 loge (t + 1) + 90, 0 t 15
a Find the weight of the bodybuilder at the start of the program.
b Find the minimum weight obtained and how many months it takes to reach it.

8d

Maximum and minimum problems


when the function is unknown
When solving maximum and/or minimum problems when the function is not given directly, a
rule for the function must be obtained from the given information. This rule should have the
quantity being maximised or minimised in terms of one variable only. Sometimes a diagram will
assist in establishing the rule. Then solve the problem using differentiation.
These steps should be followed.
1. Draw a diagram if appropriate.
2. Identify the quantity to be maximised or minimised.
3. Express this quantity in terms of one variable only.
4. Solve f(x) = 0.
5. Verify it is a maximum or minimum using the first derivative test.
6. Sketch a graph to confirm the maximum or minimum found.
7. Answer the question that has been asked.

390

Maths Quest 12 Mathematical Methods CAS for the TI-Nspire

The following formulas may be useful in problem solving:


Area of circle
Curved surface area of a cylinder
Surface area of a sphere

A = r2
S = 2rh
S = 4r2

Volume of a sphere

V = 3 r3

Volume of a cylinder

V = r2h

Volume of a cone

V = 13 r 2 h

V = 3 Ah , where A is the area of the base

Volume of a right pyramid

d = ( x2 x1 )2 + ( y2 y1 )2

Distance between two points

Worked Example 9

The sum of two positive numbers is 10. Find the numbers if the sum of their squares is a minimum.
Think

Write

Let one number be x and the other number be y.


Form an equation.

Let x = 1st number and y = 2nd number.


x + y = 10

Express y in terms of x.

So y = 10 x

Write an expression for S(x), the sum of


thesquares of x and y, in terms of x only.

S(x) = x2 + y2

= x2 + (10 x)2

Simplify the equation.

Find S(x).

S(x) = 4x 20
Function has a stationary point when S(x) = 0

Find x for minimum S by solving S(x) = 0.

4x 20 = 0

4x = 20

x=5

Verify that it is a minimum by the first derivative


test.

Gradient table:

= x2 + 100 20x + x2
= 2x2 20x + 100

P(t)

Slope

So x = 5 gives a minimum for S.


8

Find y.

When x = 5,

y = 10 5

=5

State the two numbers.

Therefore, the two numbers which give a


minimum of their squares are both 5.

Note: The actual sum was not asked for in this example.

Chapter 8 Applications of differentiation

391

Worked exAmple 10

eBook plus

A cuboid container with a base length twice its width is to be made


with 48 m2 of metal.
8 2x
a Show that the height is given by the expression h = ,
x 3
where x is the width of the base.
b Express the volume, V, in terms of x.
c Find the maximum volume.
Think
a

Tutorial

int-0561
Worked example 10

WriTe

Draw a diagram of a cuboid.

2x
x

Let x be thewidth of the base and


hence express length in terms of x.

Let x = width and h = height


so length = 2x

Calculate the total surface area (TSA)


of the cuboid in terms of x and h only.

TSA = 2[2x(x) + 2x(h) + x(h)]


= 2(2x2 + 3xh)
= 4x2 + 6xh

Express h in terms of x.

As TSA = 48 m2
4x2 + 6xh = 48
6xh = 48 4x2
h=

48 4 x 2
6x

48 4 x 2

6x 6x

h=
b

Find the volume, V, in terms of x and h.

Express the volume in terms of x by


substituting for h.

b V = x(2x)h

8 2x
V(x) = 2x2
x 3
= 16x

Solve V(x) = 0.

Verify that x = 2 gives a maximum.

8 2x

x 3

4 x3
3

c V(x) = 16 4x2 = 0 for maximum or minimum

4x2 = 16
x2 = 4
x = 2 or 2 (reject 2 as width cannot be
negative)

Gradient table:
x

V(x)

Slope

The maximum volume is achieved when x = 2 m.


392

maths Quest 12 mathematical methods CAS for the Ti-nspire

Substitute x = 2 into the rule for V(x) to


obtain the maximum volume.

V(2) = 16(2)

4(2)3
3

32
3

= 32

= 32 10 23

= 21 13
1

Therefore the maximum volume is 21 3 m3.

Worked Example 11
y

Find the minimum distance from the straight line with equation
y = x 4 to the point (1, 1).

P (1, 1)

0 1
4
Think

y=x4
x

4
Q (x, y)

Write

The minimum distance between a straight


line and a point is a perpendicular line from
a point on the straight line, Q (x, y) to the
point P (1, 1).

Let Q be the point on the line with


coordinates(x, y).

From the given rule for the straight line, find y in


terms of x.

As Q is on the line y = x 4 then Q is (x, x 4).

Find the distance, d(x) between P and Q in terms


of x only using the formula for the distance
between 2 points.

d(x) = ( x2 x1 )2 + ( y2 y1 )2

Differentiate d(x) using the chain rule (or


differentiate the expression inside the square root
only, as the smallest value of this expression will
give the minimum distance when the square root
is taken).

= ( x 1)2 + ( x 4 1)2

= ( x 1)2 + ( x 5)2

= x 2 2 x + 1 + x 2 10 x + 25

= 2 x 2 12 x + 26

1
d
(2 x 2 12 x + 26) 2
dx

= 12 (4x 12)
=

1
1

(2 x 2 12 x + 26) 2

4 x 12
2 2 x 2 12 x + 26

OR
d
(2 x 2 12 x + 26)
dx
= 4x 12

Chapter 8 Applications of differentiation

393

Solve for x where the derivative equals zero.

For maximum or minimum,


4 x 12
=0
2 2 x 2 12 x + 26

4x 12 = 0

4x = 12

x=3

Verify that x = 3 gives a minimum.

Gradient table:
x

Derivative (4x 12)

Slope

So x = 3 gives the minimum distance.


7

Evaluate d(3) to obtain the minimum distance.


The exact answer is 8 and 2.828 is an
approximate answer.

d(3) = 2(3)2 12(3) + 26

d = 18 36 + 26

d= 8=2 2

or

2.828

Therefore the minimum distance is 2 2 units


or approximately 2.828 units.

REMEMBER

1. Express the quantity being maximised or minimised in terms of one variable only.
2. Find the derivative and let it equal zero to find the maximum or minimum.
3. Test for maximum and/or minimum using the first derivative test (gradient table).
4. Use exact answers where appropriate. If using an approximate answer show by use of
the symbol.
5. Ensure you have answered the question asked.
Exercise

8d

Maximum and minimum problems when


the function is unknown
1 WE9 The sum of two positive numbers is 10. Find the numbers if their product is
amaximum.
2 The sum of two positive numbers is 8. Find the numbers if the sum of the cube of one and
thesquare of the other is a minimum.
3 A rectangular frame is to be made from a piece of wire 120 cm long.
a If the width is x cm, show that the length is 60 x.
b Find an expression for the area of the rectangle in terms of x.
c Hence, find the dimensions for maximum area.
d Find the maximum area.
4 Find the area of the largest rectangular paddock that can be enclosed with 400 metres
offencing.

394

Maths Quest 12 Mathematical Methods CAS for the TI-Nspire

5 Cylindrical, cardboard postal tubes are made with the restriction


that the sum of the length and the circular circumference are
120cm. What should the dimensions be for maximum volume?

Length

Circumference

6 The frame of a container in the shape of a cuboid is shown at right.


If it is to be made with a total length of 18 metres of steel edging,
find:
a the value of L in terms of x
2x
b the expression for the volume in terms of x only
c the length of each edge for maximum volume
d the maximum volume.

7 WE10 A cuboid with a square base is to be made with 200 cm2 of material.
50 x
, where x is the side length of the base.
x 2
b Express the volume, V, in terms of x.
c Find the maximum volume (to the nearest unit).

a Show that the height, h =

8 A window frame is in the shape of a semicircle joined to a rectangle.


Find the maximum area of a window using 300 cm of framework.
9 A rectangular sheet of cardboard, 50 cm by 40 cm, is to have square
corners cut out so it can be folded into a rectangular tray.
Find the maximum volume possible for such a tray.

10 A bushwalker can walk at 5 km/h through clear land and


3 km/h through bushland. If she has to get from point A to
point B following a route indicated at right, find the value
of x so that the route is covered in a minimum time.
(Note: time =

50 cm
40 cm

B
Clear

Bush
3 km

distance
)
speed

11 The cost of running a train at a constant speed of v km/h is C = 50 +

2 km

v2
dollars/hour.
1000

a Find the time taken for an 800 km journey in terms of v.


b Hence, find an expression for the cost of an 800 km journey.
c Find the most economical speed for this journey.
12 Find the side length of the largest cube which can fit inside
a sphere of diameter 24cm.

24 cm

Chapter 8 Applications of differentiation

395

13 A cylinder of cheese is to be removed from a spherical piece


of cheese of radius 8 cm. What is the maximum volume of the
cylinder of cheese? (Express the answer to the nearest unit.)
8 cm

14 We11 Find the minimum distance from the line y = 2x +3


to the point (1, 0).

y = 2x + 3
3

Minimum distance
(1, 0)

0
y

eBook plus
Digital doc

WorkSHEET 8.2

1
y = x2

15 Find the minimum distance from the parabola y = to the point


(5, 0). (Express the answer to the nearest hundredth.)
x2

(5, 0)
5 x

8e

rates of change

eBook plus

P (x1, f (x1)) and Q (x2, f (x2)) are two points on the function with rule y = f (x)
as shown in the diagram.

Interactivity

int-0253
Rates of
change

y
y = f(x)
Q (x2, f(x2))

P (x1, f(x1))
0

x1

x2

The average rate of change of y with respect to x over the interval x [ x1, x2] is equal to the
gradient of the straight line (or chord) PQ.
Average rate of change =

change in f ( x )
change in x

f ( x2 ) f ( x1 )
x2 x1
The instantaneous rate of change is the rate of change at a specific point.
dy
The instantaneous rate of change of y with respect to x is given by the derivative .
dx
dy
If
> 0, then y is increasing as x increases (gradient is positive).
dx
dy
If
< 0, then y is decreasing as x increases (gradient is negative).
dx
=

396

maths Quest 12 mathematical methods CAS for the Ti-nspire

Note: Rates of change are often calculated with respect to time, but not always. If you are
required to find the rate of change with respect to some quantity other than time then the
quantity must be stated. If this quantity is not stated then the rate of change is taken as being
with respect to time.
Worked exAmple 12
a Find the rate of change of the surface area of a melting ice cube with respect to its side length (x).
b What is the rate of change when x = 2 cm? (Assume that the ice cube remains in the shape of

a cube.)

Think
a

WriTe

Express the surface area, S, of the


cube in terms of its side length, x.

Find

Place a negative sign in front of


the rate as the ice cube is melting
(that is, the rate is decreasing).
dS
Substitute x = 2 into .
dx

dS
.
dx

State the solution.

a S = 6x2 (total surface area of a cube)

dS
= 12x
dx
But

dS
= 12x because the surface area is decreasing.
dx

b When

x=2
dS
= 12(2)
dx
=24

Therefore, the rate of change of the surface area when


x =2 cm is 24 cm2/cm (it is decreasing at a rate of
24 cm2/cm).

Worked exAmple 13

eBook plus

The number of mosquitoes, N, around a dam on a certain night can be modelled


by the equation
N = 100 loge (2t + 1) +5t + 1000

Tutorial

int-0562
Worked example 13

where t equals hours after sunset. Find:


the initial number of mosquitoes
the average rate of change in the first 4 hours
the rate of change at any time, t
the rate of change when t = 4 hours.

a
b
c
d

Think
a

WriTe

Write the rule.

Find N when t = 0.

a N = 100 loge (2t + 1) + 5t + 1000

When t = 0,
N = 100 loge (2 0 + 1) + 5 0 + 1000
= 100 loge 1 + 1000
= 100 0 + 1000
= 1000
The initial number of mosquitoes
is 1000.

Chapter 8

Applications of differentiation

397

Find N when t = 4.

b When t = 4,

N = 100 loge (2 4 + 1) + 5 4 + 1000


= 100 loge (9) + 20 + 1000
= 1020 + 100 loge (9)
= 1020 + 219.7
= 1239.7
that is, 1239 mosquitoes (as the number
has not reached 1240).

Calculate the average rate between t = 0 and


t = 4 using

The average rate of change in the first


4 hours
1239 1000
=
40

f ( x2 ) f ( x1 )
.
x2 x1

239
4
= 59.75 mosquitoes per hour.
=

c Differentiate N with respect to t, to find

dN
.
dt

dN 100 2
=
+5
dt
2t + 1
=

d Find

dN
when t = 4.
dt

200
+5
2t + 1

d When t = 4 the rate of change is:

dN
200
=
+5
dt 2 4 1
=

200
9

+5

= 22 9 + 5
2

= 27 9 mosquitoes per hour.

rememBer

1. The average rate of change is the gradient of a straight line between two points.
f ( x2 ) f ( x1 )
.
x2 x1
3. The instantaneous rate of change at any point is given by the value of the derivative at
that point.
2. The average rate of change =

exerCiSe

8e

rates of change
In the following exercise, use a CAS calculator to assist with any graphing.
1 Express the following in simplest mathematical notation.
a The rate of change of volume, V, with respect to radius, r.
b The rate of change of surface area, S, with respect to height, h.
c The rate of change of area, A, with respect to time, t.

398

maths Quest 12 mathematical methods CAS for the Ti-nspire

eBook plus
Digital doc

SkillSHEET 8.2
Review of rates
of change

d The rate of change of cost, C, with respect to distance, x.


e The rate of change of intensity, I, with respect to pressure p.
f The rate of change of velocity.
2  WE12 a Find the rate of change of the area, A = r2, of an increasing circular oil spill with
respect to the radius, r.
b What is the rate of change when r = 10 metres?
3 a Find the rate of change of the volume, V = 43 r3, of a deflating spherical balloon with
respect to the radius, r.
b Hence, find the rate when r = 5 cm.
4 A sugar cube dissolves in a cup of tea. Find the rate of decrease of its surface area, S, when the
side length, x, is 0.5 cm.
5 The height of a projectile t seconds after being fired is h = 10 + 20t 5t2 metres.
a Find the rate of change of height after
i 1 second and ii 3 seconds.
b Explain the difference between these two answers.
6 The volume of water (in litres) which has flowed through a swimming pool filter t minutes
t4
1
(30t3 ) where 0 t 90.
after starting it is V = 100
4
a At what rate is water flowing through the filter at any time, t?
dV
over the domain [0, 90].
b Sketch a graph of
dt
c When is the rate of flow greatest?
7 A particle moves in a straight line so that its displacement from a point, O, at any time, t, is
x = 3t 2 + 4 .
Find:
a the velocity as a function of time
b the acceleration as a function of time
c the velocity and acceleration when t = 2.
8 If a particle is moving in a straight line so that its displacement from the origin at any time, t, is
x = t3 12t2 + 36t, find:
a the velocity
b the time and displacement when the velocity is zero
c the acceleration when the velocity is zero.
9 WE13 The number of people with the flu virus, N, in a particular town t days after a vaccine is
introduced is N = 3000 500 loge (8t + 1).
a How many people are infected in the town before the vaccine is introduced?
b Find the average rate of change over the first 5 days.
c Find the rate of change of the number of people in the town infected with flu.
d Find the rate of change after 5 days.
10 MC If V = t3 2t2 + 8 t , where V is in m3 and t is in hours
a the rate of change of V with respect to t when t = 4 is:
B 32 m3/h
c 28 m3/h
d 34 m3/h
e 20 m3/h
A 48 m3/h
b the average rate of change from t = 1 to t = 4 is:
d 10 13 m3/h e 0 m3/h
A 13 23 m3/h B 10 13 m3/h c 3 m3/h
11 MC Temperature, TC, is related to height, h metres, by T = h2 + 4eh.
a The rate of change, C/m, at h = 0 is:
A 0
b 4
c 4e
d 4e e 1
Chapter 8 Applications of differentiation

399

b The rate of change, C/m, at h = 4 is:


A 16 + 4e4
b 16 + e4
D 8 + e4
e 8 + 16e4

c 8 + 4e4

c The average rate of change between h = 0 and h = 4 in C/m is:


A 1 + e4
b e4
c 12 + 4e4
D 4 + e4
e 3 + e4
12 The height of water (in metres) at the entrance to a bay t hours after high tide is:
t
H = 10 + 2 cos
12

Find:
a the rate of change of H at any time, t
b the rate of change of H:
i 6 hours
ii 15 hours

iii 20 hours after high tide
c the minimum and maximum value of H and the time when they first occur after the initial
high tide.
13 The value of an antique ornament t years after being purchased is A = 2000 e 0.1 t dollars.
a Find the value of the antique

i when purchased and
ii 3 years after purchase.
b Hence, find the average rate of change of value in the 3 years since purchased.
c Find the rate of change of value 3 years after purchase.
20
14 The amount of chlorine in a jug of water t hours after it was filled from a tap is C =
,
t +1
where C is in millilitres. Find the rate of decrease of chlorine 9 hours after being
poured.
15 The graph below shows the price of the shares of a particular company over a period
of time.
Price ()
180
170
160
150
140
130
120
0

10 12 14 16 t (months)

Use the graph to estimate the answers to the following questions.


a Estimate the share price when t = 5.
b State the time interval over which the share price was decreasing.
c Find the average rate of change of the share prices over the entire period shown on the
graph.
d Estimate the rate of change of the share price at t = 11.
e What was the maximum share price over the period shown?
f When was the share price at its lowest value?
400

Maths Quest 12 Mathematical Methods CAS for the TI-Nspire

16 The graph below shows how the velocity of a car varies over a period of time.
v (m/s)
14
12
10
8
6
4
2
0

10 12 14 16

t (s)

Usethegraphtoestimatetheanswerstothefollowingquestions.
a Estimate the velocity of the car at t = 12.
b Estimate the acceleration of the car at t = 5.
c State the time intervals over which the car is accelerating.
d What was the average rate of change of velocity of the car over the interval [4, 8]?

8F

eBook plus

related rates

eLesson

eles-0094

When two variables are both functions of the third variable, we may need to
Related rates
use related rates to solve the problem. For example, we may need to
dx
dy
dx dx dy
find
when given . In such instances we would need to use the chain rule:
=
.
dt
dt
dt dy dt
When solving problems involving related rates, the following steps may be helpful:
1. Draw a diagram where appropriate. Sometimes two or more diagrams may be necessary.
2. Identify the variables.
3. Identify which rate is given and which rate is required.
4.Usethechainruletoconnecttherequiredandgivenrates.
5. Find an equation, or relationship, that connects the variables if not given.
6. Differentiate the equation.
7. Substitute into the chain rule and simplify.
8. Answer the question noting correct units.
Tools for finding relationships:
similartriangles
Pythagorastheorem
right-angledtriangletrigonometry
sineandcosinerules.
Worked exAmple 14

eBook plus

cm3

An ice cube is melting at a constant rate of 2


per hour. At what rate is
the side length of the cube changing when the side length is 1.2 cm?
Think
1

We are given the rate of change of volume


with respect to time. Write it using appropriate
notation. Since the volume is decreasing, the
rate is negative.

Tutorial

int-0563
Worked example 14

WriTe

dV
= 2
dt

Chapter 8

Applications of differentiation

401

dx
when x = 1.2
dt

State the rate of change that needs to be found.

Need to find

Use the chain rule to connect the rate that we


need to find with the one that is given.

dx dx dV
=

dt dV
dt

We do not know dx . To find it, we need the


dV
rule that connects x and V, that is, the rule that
connects the side length of the cube with its
volume. So write the formula for the volume
of the cube.

Differentiate.

Reciprocate both sides to obtain dx .


dV

Substitute 1 2 for dx and (2) for dv into the


dV
dt
3x
chain rule and simplify.

Vcube = x3, where x is the length of the side.

dV
= 3x2
dx
dx
1
= 2
dV 3 x
dx
1
= 2 (2)
dt 3 x

3x 2

We now have the formula that allows us to


find the rate of change of the side length for
any value of x. To find the rate of change when
length is 1.2 cm, simply substitute 1.2 for x
into this formula and evaluate.

So when x = 1.2,
2
dx
=
dt 3 (1.2 )2

State the answer, adding appropriate units.

When the side length of the cube is 1.2 cm, its

25
54

length is changing at a rate of 25 cm/h.


54

Worked Example 15

An empty inverted right circular cone has a radius of 4 cm and a height of 20 cm. Water is being
poured in at a constant rate of 0.1 cm3/second. Find the rate at which the depth of the water is
increasing at the instant the depth is 6 cm. Give your answer correct to 2 decimal places.
Think
1

Draw a diagram showing half the cross-section at any


depth, h cm.

Write
4 cm

r cm

20 cm
h cm

402

Maths Quest 12 Mathematical Methods CAS for the TI-Nspire

Identify the variables.

Identify the given rate.


Identify the required rate.

Use the chain rule to connect the rate that we need to


find with the one that is given.

Find a rule that connects volume, V with the depth of


water, h.
Use similar triangles to connect r and h.

Depth of the water, h cm


Radius, r cm, of surface of the water
Time, t seconds
Volume, V cm3
dV
= 0.1 cm3/second
dt
dh
Find
when h = 10 cm.
dt
dh dh dV
=

dt dV dt
1
V = r 2h
3
r
4
=
h 20
h
r=
5
2

Rewrite the formula with the two variables V and h.

1 h
V = h
3 5

3
h
75
dV 3 2
=
h
dh 75
dV 2
= h
dh 25
dh
25
=
dV h 2
dh dh dV
=

dt dV dt
dh
25
=
0.1
dt h 2
dh 2.5
=
dt h 2
dh
2.5
=
dt 62

V=
6

Differentiate V with respect to h.

dh
by taking reciprocals of both sides.
dV
Substitute into the Chain Rule and simplify.
Obtain

Substitute h = 6 cm to find the required rate of


change.
State the answer with appropriate units.

dh
= 0.02 cm/second (correct to
dt
2 decimal places)
The depth of water is increasing at a rate
of 0.02 cm/second when the depth is 6 cm.

REMEMBER

dx dx dy
1. To solve a problem involving related rates, use the chain rule, for example:
=
.
dt dy dt
2. Show appropriate units.

Chapter 8 Applications of differentiation

403

Exercise

8F

Related rates
1 WE14 A gym fitness ball is being inflated and its volume is increasing at a constant rate of
5cm3 per second. At what rate is the radius of the ball changing when the radius is 34 cm?
2 A particle moves along a path that can be described using the Cartesian equation y = 3x3 2x + 1.
If

dy
dx
= 3 when x = 5, find
at that moment.
dt
dt

3 The surface area of a cube is decreasing at a constant rate of 9 cm2/s. Find the rate at which the
sides of the cube are decreasing when the sides are 1.5 cm long.
4 A spherical balloon is being deflated and its radius, r cm, is decreasing at a constant rate of
5 cm/min. At what rate is its volume, V cm3, decreasing when the radius of the balloon is 4 cm?
5 A bowl is being filled with water at a rate of 12 cm3/s. The volume, V cm3, of water in the
5

bowl is given by V = 8h 2 where the depth of water in the bowl is h cm. Find the rate at which
the depth of water in the bowl is increasing when the depth is 9 cm.
6 The radius of a circular puddle of water is increasing at a rate of 2.5 cm/s. Find the exact rate at
which the area is increasing at the instant the radius is 12 cm.
7 MC A cylinder with radius of 4 m is being filled with water. If the rate of change of the depth
1
of the water is
m/s, the rate of change of volume, in m3/s, in the cylinder is:
8
1
1
1
A

B 2
c 2
d

e 2
2
2
108
8 WE15 An inverted right circular cone is filled with liquid. The cone has a radius of 3 m and
height of 7 m. The liquid flows from the apex of the cone at a constant rate of 0.6 m3/min.
Find the rate at which the depth of the liquid is dropping, correct to 2 decimal places, when the
depth of the liquid is 2 m.
9 The upper end of an 8 m ladder rests against a vertical wall with the lower end on the
horizontal ground. The lower end of the ladder on the ground slips away from the wall at a rate
of 6 m/s. Find the rate at which the upper end of the ladder is moving the instant the ladder is
4 m from the wall.
10 A stainless steel cylindrical tank of radius 6 m is being filled with milk at a constant rate of
1.5 m3/min. At what rate is the level of milk rising? Give your answer in terms of .
11 A sand timer consists of two cones joined at the apex. Each cone has height h, radius r and an
angle at the apex of 60.
a Express the radius of the top cone in terms of its height. Give your answer in exact form.
b Write the volume of the top cone as a function of its height.
c When the timer is turned over, the sand starts pouring from the top cone into the bottom
1
one at a constant rate of 32
cm3/s. Find the rate of change of the depth of the sand in the
top cone when the depth is 0.8 cm.

8G

Linear approximation
It is useful to be able to find how much a small change in the independent variable affects the
dependent variable. For example, how will a small increase in the radius of a circle affect the
area of a circle?

404

Maths Quest 12 Mathematical Methods CAS for the TI-Nspire

This concept can be illustrated graphically.


y

y = f(x)

B
y

y = f(x + h) f(x)

x
=h

Consider the function y = f (x) as shown with the points A (x, f (x)) and B (x + h, f (x + h)).
The gradient of the chord AB =

f ( x + h) f ( x )
. If h is very small, the gradient of the chord is
h

very close to the gradient of the tangent at the point A (x, f (x)).
f '( x )

f ( x + h) f ( x )
h

h f '( x ) f ( x + h) f ( x )
f ( x + h) f ( x ) + h f '( x )
The formula f ( x + h) f ( x ) + h f '( x ) is called the linear approximation formula.
Alternatively, if a small change in y, y, corresponds to a small change in x, x, then
y x

dy y
.

dx x

dy
where y f ( x + h) f ( x ).
dx

If y is the small change in a quantity, y, the percentage change (or error) in y is given by
h f '( x )
y
100% .
100% or
f ( x)
y

Worked Example 16

Use the approximation formula, f(x + h) f(x) + hf'(x), with f(x) = x2 and x = 1 to find an
approximate value of (1.01)2.
Think

Write

f(x) = x2, f'(x) = 2x, x = 1 and h = 0.01

Write f(x), f'(x), x and h.

Find x + h and hence f(x + h).

x + h = 1.01 and f(x + h) =


(x + h)2 = 1.012

Write the formula and hence find the value of 1.012.

f(x + h) f (x) + hf'(x)


1.012 12 + 0.01 2
1.012 1.02

Chapter 8 Applications of differentiation

405

Worked Example 17

A circular metal disc is being cooled. If the radius is decreased from 10 cm to 9.8 cm, find correct to
2 decimal places:
a the approximate change in the area b the percentage change in the area.
Think
a

Write

Write the relationship between area and radius.

A(r) = r2

Write A(r), r and h.

A(r) = 2r, r = 10 cm, h = 0.2

Write the approximation formula and substitute.

A(r + h) A(r ) + hA'(r )


A(9.8) A(10) 0.2 A'(10)

State the answer with correct units.

Write the change in area as fraction of the original


area, expressing as a percentage.

State the answer.

Change in A A(9.8) A(10)



0.2 20
Area decreases by 12.57 cm2 correct to
2 decimal places.

A 0.2 20
=
100%
A
10 2
= 4%
Area is decreased by approximately 4%

REMEMBER

1. Where h is a small change in x then


f ( x + h) f ( x ) + hf '( x ).
2. Alternatively
f ( x + h) f ( x ) hf '( x )
dy
or y x .
dx
Exercise

8G

Linear approximation
1 WE16 Use the approximation formula, f(x + h) f'(x)h + f (x), with each of the following.
a For f (x) = x3 and x = 1, find 1.013.
b For f (x) = x2 and x = 1, find 0.9992.
c For f (x) = 5x2 and x = 1, find 5 0.992.
d For f (x) = x and x = 1, find 1.001
2 If a spherical balloon has a radius of 5 cm, find the increase in volume of the balloon when the
radius expands by 0.02 cm.
3 A circular disc has a radius of 10 cm. Find the percentage increase in its area if the radius is
increased by 2%.
4 WE17 An isosceles triangle has its equal sides of length 10 cm

with an included angle of as shown in the diagram.


10 cm

406

Maths Quest 12 Mathematical Methods CAS for the TI-Nspire

10 cm

If changes from 60 to 61, find, correct to 2 decimal places:


a the approximate area of the triangle when = 61
b the approximate increase in the area, A, of the triangle.
5 MC If f (x) = x2 and x = 3, an approximate value of (2.9)2 is given by:
A f (3) + 0.1 f '(3)
B f (2.9) + 0.1 f '(2.9)
C f (3) f (2.9)
D f (2.9) 0.1 f '(2.9)
E f (3) 0.1 f '(3)
6 The volume, V cm3, of a gas is inversely proportional to the pressure, P atmospheres, of a gas,
k
that is, V = where k is a constant (that depends on the mass of the gas and the
P
temperature). If the pressure of the gas increases from 2 atmospheres to 2.25 atmospheres,
find the approximate change in the volume of the gas in terms of k. Hence find the
corresponding percentage change in the volume.
2
7 Find the approximate change in x as y decreases from 2 to 1.5 if f ( x ) =
.
x4
8 The length of a rectangle is four times its width. If the width increases by 5%, find the
corresponding percentage change in the perimeter and area of the rectangle.

Chapter 8 Applications of differentiation

407

Summary
Stationary points

A function f(x) has stationary points wherever its derivative f(x) = 0.


If f(a) = 0, then f(x) has a stationary point (a, f(a)) which is:
1. a local minimum turning point if:
for x < a, f(x) < 0
and x > a, f(x) > 0.
2. a local maximum turning point if:
for x < a, f(x) > 0
and x > a, f(x) < 0.
3. a positive stationary point of inflection if:
for x < a, f(x) > 0
and x > a, f(x) > 0.
4. a negative stationary point of inflection if:
for x < a, f(x) < 0
and x > a, f(x) < 0.
Equations of tangents and normals

The gradient of the tangent at x = a to the curve y = f(x) is f(a).


1
The gradient of the normal at x = a to the curve y = f(x) is
.
f'(a)
The equation of a straight line with gradient m and passing through the point (x1, y1) is:
y y1 = m(x x1)
Maximum and minimum problems

When solving maximum or minimum problems follow these steps.


1. Draw a diagram if appropriate.
2. Identify the quantity to be maximised or minimised (say, f(x)).
3. Express the quantity in terms of one variable only (say x).
4. Solve f(x) = 0.
5. Verify it is a maximum or minimum using the first derivative test.
6. Sketch a graph to confirm the maximum or minimum found.
7. Answer the question.
Rates of change

The instantaneous rate of change of y = f(x) at x = x1 is found by evaluating f(x1) (or finding
x = x1).
The average rate of change of y = f(x) between x = x1 and x = x2 is:
f ( x2 ) f ( x1 )
.
x2 x1

408

Maths Quest 12 Mathematical Methods CAS for the TI-Nspire

dy
where
dx

Related rates

To solve a problem involving related rates, use the chain rule, for example

dx dx dy
=
.
dt dy dt

Linear approximation

f ( x + h) f ( x ) + hf '( x )
dy
y x
dx
Percentage change: y 100%
y

Chapter 8 Applications of differentiation

409

chapter review
Short answer

1 Determine the stationary points and their nature for


the function f(x) = 2x3 + 3x2 36x + 5.

9 P is the point on the line 2x + y 10 = 0 such that


the length of OP, the line segment from the origin
O to P, is a minimum. Find the coordinates of P and
this minimum length.

2 Sketch the graph of y = x2(4 x2), clearly indicating


all stationary points and intercepts.8B

Exam tip Students failed to recognise this as a


maximum/minimum question. A common, incorrect
response was to find intercepts (0, 10) and (5, 0) on
the line then use the distance between two points to

3 Find the equation of the tangent to the curve


y = 6 3x + 2x2 x3 at the point where x = 1.
4 Find the equation of the tangent to the curve
y = 3 loge (x2) (x 0) which is parallel to the line
y 3x 7 = 0.

find the distance to be 5 5. Other common, incorrect


responses involved taking P as the midpoint of
the line, guessing and checking solutions without
justification, incorrectly differentiating the distance
equation, or using an incorrect diagram.

5 Find the equation of the normals to the curve


1
y = x + at the point where the normals are
x

[Assessment report 1 2007]

[ VCAA 2007]

parallel to the line 3y + 4x 10 = 0.C


6 The number of bees, N, in a hive can be modelled
by the function N = 2t(50 t) + 180 where t is the
number of days the hive has existed. What is the
maximum number of bees in the hive?

10 A normal to the graph of y = x has the equation


y= 4x + a, where a is a real constant. Find the
value of a.

7 The area of a certain triangular shape is:


A=

Exam tip Students found this question quite


challenging, and a significant number seemed to
have little or no idea of what was required. Many
were able to find the correct derivative but did not
know how to proceed, or equated the derivative to 4
1
instead of 4 . Students who got to this point without
error often did not solve correctly for x or y or both.
It was not uncommon to see two values found for x
and y and therefore two values for a, 18 and 14.

2x2
,x>6
3( x 6)

where x is the length of the base of the triangle.


a Find the value of x for minimum area.
b Hence, find the minimum area.
8 A wine glass is being filled with wine at a rate of
8cm3/s. The volume, V cm3, of wine in the glass
when the depth of wine in the glass is x cm is given
3
4x 2.

by V =
Find the rate at which the depth of
wine in the glass is increasing when the depth is
4cm.
Exam tip Some students had difficulty deciding
on how to label variables; x was sometimes
interchangeable with d or h and little or no
consistency was evident through the question. Most
students were able to correctly differentiate V with
respect to x but then either incorrectly used the chain
rule or could not manage the arithmetic required.
Common errors were to simply substitute 4 into V or
to substitute into the derivative.
[Assessment report 1 2007]

[ VCAA 2007]

410

[Assessment report 1 2006]

[ VCAA 2006]

Multiple choice

1 The graph of y = (x + 2)3 has:


A 1 turning point
B 2 turning points
C 1 point of inflection
D 2 points of inflection
E 0 stationary points
2 The graph of x3 + 2x2 + x 2 has:
A 2 points of inflection
B 1 point of inflection
C 1 turning point and 1 point of inflection
D 3 turning points
E 2 turning points

Maths Quest 12 Mathematical Methods CAS for the TI-Nspire

3 The graph of 13x3 4x2 9x + 5 has a local


maximum turning point at:
2

A (1, 9 3 ) B (9, 20) C (9, 18)


d (1, 10 23 )E (1, 7 13 )
4 If the graph of g(x) has the following properties:
i g'(x) = 0 if x = 3, 1 and 4
ii g'(x) < 0 if x < 3 and 1 < x < 4
iii g'(x) > 0 for all other x
then the graph of g(x) could be:
y

g(x)
3

0 1

01

4
g(x)

y
g(x)
0

g(x)
01

g(x)
3

01

8 The tangent to the curve y = 2 loge (3 2x) at the


point where x = 1 intercepts with the y-axis at the
point where y equals:
A 1
B 3
C 0
D 2
E 4
9 The graph of f(x)
y
shown at right
f '(x)
indicates that the
graph of f(x) has:
x
A a turning point at
0
4
2
x = 2 and x = 4
B a turning point at
x = 2 and point of
inflection at x = 4
C a turning point at x = 4 and point of inflection
at x = 2
D 3 stationary points
E 2 points of inflection at x = 4 and x = 2
Questions 10 and 11 refer to the following
information. The volume of liquid in a container
varies with time according to the rule V = 1 + t2et,
where t is in hours and V is in thousands of litres, and
0 t 40.
10 The minimum value of V occurs when t equals:
A 1 hour
B 2 hours
C 0 hours
d 3 hours
E 40 hours

7 The equation of the normal to the curve y = 3ex + 2


at the point where x = 2 is:
A 3y + x 7 = 0
B 3y x + 7 = 0
C y 3x 9 = 0
d y 3x 3 = 0
E y + 3x 9 = 0

11 The maximum volume, to the nearest tens of


litres,is:
A 1450
B 1540
C 1390
d 1360
E 1630
Questions 12 to 16 follow from the isosceles triangle
below which has a perimeter of 40 cm.

5 The equation of a line with a gradient of 2 and


passing through the point (1, 3) is:
A y = 2x + 5B y = 2x C y = 2x 1
d y = 2x 5E y = x + 1
6 The equation of the tangent to the curve
y = x (x + 2)(x 1) at the point where x = 1 is:
A y x + 1 = 0
B y x + 3 = 0
C y + x + 3 = 0
d y + x 1 = 0
E y x 1 = 0

8D

12 The value of y in terms of x is:


A 40 2x
B 20 x
C 40 x
d 20 2x
E x2

Chapter 8 Applications of differentiation

411

13 The height of the triangle in terms of x is:


A 400 40 x

B 20 40 x

400 40 x + 2 x 2

40x

400 40 x + x 2

14 The area in terms of x is:


A ( 20 x ) 400 40 x

B x 400 40 x + x 2

C 2 x 400 40 x + x 2

D x 400 40 x

E 2 x 400 40 x
15 The maximum area of the triangle is obtained if x
equals:
A 6 23 cm
B 10 cm
C 20 cm
d 5 cm

e 10 25 cm

16 Therefore, the maximum area possible is:


A 50 2 cm2
d 40 3
9

B 64 cm2
400 3
E
9

C 32 5 cm2

17 The average rate of change of the function


f(x) = x4 3x3 + 5x between x = 1 and x = 3 is:
A 15
B 6
C 12
d 4 12
E 16
18 An iceberg in the shape of a cube is slowly melting.
The rate of change of the surface area of the iceberg
in m3/m when the side length is 40 metres is:
A 480
B -240
C 720
d -480
E 240
19 The radius of a sphere is increasing at a rate of
3 cm/min. When the radius is 6 cm, the rate of
increase of the volume, in cm3/min, in the sphere is:
A 432
B 48
C 144
d 108
E 16

[ VCAA 2006]
20 Using the approximation formula
f ( x + h) f ( x ) + hf '( x ) , with f(x) = x3 and x = 2,
an approximate value of (1.8)3 is given by:
A f (2) + 0.2 f '(2)
B f (1.8) + 0.2 f '(1.8)
C f (2) f (1.8)
d f (2) 0.2 f '(2)
E f (1.8) 0.2 f '(1.8)

[ VCAA 2004]

Extended response

1 Consider the cone with a slant side of 10 cm shown at right.


10 cm

a Show that the height, h, is 100 r 2 .


h

b Show that the volume of the cone is V = 13 r2 100 r 2 .


c Hence, find the maximum volume.

r
2 The number of bacteria present in a lasagne, t minutes after it is placed in a

microwave to heat up, can be modelled by N = 3000e 0.5t, where t > 0.


a Find the initial number of bacteria.
b Find the rate of change of bacteria after being in the microwave for 10 minutes.
3 The height above the ground of a person on a ferris wheel at any time, t, seconds after the ride has
t
started is h=5.4 4 cos metres.
15
a Find the initial height of the person above ground level.
d (m)
b Find the height after 5 seconds.
700
c Hence, find the average rate of change of height during the
first 5 seconds.
600
d Find the rate of change of height at t = 5.
500
4 Lena is walking in the park. The graph at right shows her
400
displacement from the park entrance over a period of time.
a Estimate Lenas displacement from the park entrance at t = 20. 300
b Find Lenas average velocity over the interval [20, 30].
200
c What was Lenas maximum displacement from her starting
100
point?
d Estimate Lenas velocity at t = 15.
0

412

Maths Quest 12 Mathematical Methods CAS for the TI-Nspire

10 15 20 25 30 35 t(min)

5 A manufacturing company is required to produce cylindrical cans (for tuna) of volume 50 cm3. Tinused to
produce the cans costs 40 cents per 100cm2.
a Find the area of tin required, A, in terms of the radius, r.
b Find the radius of the can (to the nearest tenth) for minimum area.
c Hence, find the minimum area (to the nearest tenth).
d What is the cost of tin to produce 10 000 such cans?
6 A small manufacturing company needs to order new cardboard boxes for
packaging their product. Each box is to be in the shape of a prism with a square end
w
and is to have a volume of 27 000 cm3. To hold each box together, tape is used all
around the box as shown in the diagram at right.
w
l
a Express the length of the box (l) in terms of its width (w).
b Write the formula for the total area of cardboard (A) in terms of w.
c Find the dimensions of the box which uses the least amount of cardboard.
d Write the formula for the total length of the tape (L) in terms of w. (Ignore the width of the tape and any
overlaps.)
e Find the dimensions of the box that uses the least amount of tape.
f The cardboard used to make the boxes costs 0.01 cents per square centimetre, and the tape can be
purchased at $0.50 per metre. Write the formula for the total cost (in cents) of the package (that is, the
cost of the cardboard and tape) in terms of w.
g Find the dimensions of the box with the minimum total cost.
h Find the minimum total cost to the nearest cent.
7 The cross-section of a pencil head when it is first placed in a
sharpening device is shown at right.
The gradient of the tangent at point A is 4.
The equation of the pencil head is y = 4x2.
If the x- and y-axes are as indicated and all distances are in
centimetres, find:
a the coordinates of points A and B
b the equation of the tangent to the curve y = 4x2 at point A
c the coordinate of point C
d the minimum distance from the pencil head to point C
e the length of the pencil head if it starts at the point where the
normal at point A meets the y-axis.

y = 4x2

Pencil
head
B

Tangent
x

0
C

8 The population of rabbits on a particular island t weeks after a virus is introduced is modelled by

P= 1200e 0.1t, where P is the number of rabbits.


Find:
a the time taken for the population to halve (to the nearest week)
b the rate of decrease of the population after:
i 2 weeks and
ii 10 weeks.
After 15 weeks the virus has become ineffective and the population of rabbits starts to increase again
according to the model
P = P0 + 10(t 15) loge (2t 29)
where t is the number of weeks since the virus was first introduced.
Find:
c the value of P0
d the population after 30 weeks
e the rate of change of the population after i 20 weeks and ii 30 weeks
f how many weeks the population takes to get back to its original number.

Chapter 8 Applications of differentiation

413

9 After washing the kitchen floor, Alex put his favourite mop flat against the wall
and left it there. A few minutes later, the mop starts to slide down the wall. Let h be the
height of the top end of the mop above the floor and let y be the horizontal distance of
h
the bottom end of the mop from the wall at any time, t.
a If the mop is 1.2 m tall, express y in terms of x.
b If the top end of the mop slides down with a constant speed of 5 cm per second,
find the speed (in terms of h) with which the bottom end of the mop moves away
y
from the wall.
c Find the speed with which the bottom end of the mop moves away from the wall when the top of the
mop:
i is 0.8 m from the floor
ii has slid down by 20 cm.
d Find the speed with which the bottom end of the mop moves away from the wall 6 seconds after the top
of the mop started sliding down.
10 Consider the function f ( x ) = ( x 3)( x + 2a)2 where a is a positive real constant.
a Find f (x).
b Find the coordinates of the stationary points.
c Determine the nature of the stationary points.
d Find the equation of the tangent at x = 2.
e Find the x-intercept of the tangent.
11 Consider the function y = (x2 a)2 where a R.
dy
a Find
at x = 2.
dx
b Find the equation of the tangent to the curve at x = 2 in terms of a.
c Hence find the x-intercept of the tangent line in terms of a.
d A straight line with equation y = 2x + 1 passes through the x-intercept of the tangent line. Find the
value of a.
e Usingthevalueofa found previously, what is the equation of the tangent line?
eBook plus
Digital doc

Test Yourself
Chapter 8

414

maths Quest 12 mathematical methods CAS for the Ti-nspire

eBook plus

ACTiviTieS

chapter opener
Digital doc

10 Quick Questions: Warm up with ten quick


questions on applications of differentiation.
(page 373)
8A

equations of tangents and normals

Tutorial

We 2 int-0558: Watch a worked example on finding


equations of tangents and normals. (page 374)
Digital docs

Spreadsheet 129: Investigate tangents and normals.


(page 375)
WorkSHEET 8.1: Find stationary points and their
nature and equations of normals and tangents.
(page 376)
8B

Sketching curves

Tutorial

We 3 int-0559: Watch a worked example on


sketching curves and finding stationary points.
(page 378)
Digital doc

SkillSHEET 8.1: Practise differentiating expressions


other than polynomials. (page 385)
8c

Maximum and minimum problems when


the function is known

Tutorial

We 8 int-0560: Watch a worked example on


applications of maximum and minimum problems.
(page 387)
8D

Maximum and minimum problems when


the function is unknown

Tutorial

We 10 int-0561: Watch a worked example on


maximising volume. (page 392)
Digital doc

WorkSHEET 8.2: Apply differentiation skills to a


variety of problems. (page 396)
8e

Tutorial

We 13 int-0562: Watch a worked example on


application of rates of change and average rates of
change. (page 397)
Digital doc

SkillSHEET 8.2: Practise identifying rates of


change. (page 398)
8F

Related rates

eLesson eles-0094

Related rates: Learn about using differentiation for


related rates. (page 401)
Tutorial

We 14 int-0563: Watch a worked example on using


related rates to determine an instantaneous rate of
change. (page 401)
8G

Linear approximation

chapter review
Digital doc

Test Yourself: Take the end-of-chapter test to test


your progress. (page 414)
To access eBookPLUS activities, log on to
www.jacplus.com.au

Rates of change

Interactivity int-0253

Rates of change: Consolidate your understanding


of applying differentiation techniques by riding and
rolling. (page 396)

Chapter 8

Applications of differentiation

415

9A
9B
9C
9D
9E
9F
9G
9H
9I
9J

Antidifferentiation
Integration of ex, sin ((x) and cos ((x)
Integration by recognition
Approximating areas enclosed
by functions
The fundamental theorem of integral
calculus
Signed areas
Further areas
Areas between two curves
Average value of a function
Further applications of integration

Integration
areaS of STudy

Informal treatment of the fundamental theorem

Antiderivatives of polynomial functions and of


f (ax + b), where f is xn for n Q, ex, sin (x),
cos (x) or linear combinations of these
Definition of the definite integral as the limiting
b

f ( xi* ) xi,
x0

value of a sum f ( x ) dx = lim

i =1

where the interval [a, b] is partitioned into


n subintervals, with the ith subinterval of
length x
i and containing xi*, and
x = max{xi: i = 1, 2, n} and evaluation
of numerical approximations based on this
definition
Informal approximation using areas under
curves by left and right rectangles
Examples of the definite integral as a limiting
value of a sum involving quantities such as area
under a curve, distance travelled in a straight
line and cumulative effects of growth such as
inflation
Antidifferentiation by recognition that
F ( x ) = f ( x ) implies f ( x ) dx = F ( x ) + c

9a

of calculus, f ( x ) dx = F (b) F (a)


a

Properties of antiderivatives and definite


integrals:
( x ) bg( x )] dx = a f ( x ) ddxx b g( x ) dx
[af (x
b

f ( x ) dx +

b f ( x) dx = a f ( x) dx

f ( x ) dx =

f ( x ) dx

f ( x ) dx = 0

Application of integration to problems


involving calculation of the area of a region
under a curve and simple cases of areas
between curves, such as distance travelled in a
straight line; average value of a function; other
situations modelled by the use of the definite
integral as a limiting value of a sum over an
interval; and finding a function from a known
rate of change

antidifferentiation

eBoo
k plus
eBook
Digital doc

10 Quick Questions

As we have seen, the process of differentiation enables us to find the gradient of a function.
The reverse process, antidifferentiation (or integration), will find the function for a particular
gradient.
Integration has wider applications including calculation of areas, volumes, energy, probability
and many more quantities in science and business.
Note that d f ( x ) means differentiate f (x) with respect to x; that is, d f ( x ) = f ( x ).
dx
dx

416

maths Quest 12 mathematical methods CaS for the TI-nspire

So f(x) is the antiderivative of f'(x), denoted as f ( x ) = f '( x ) dx


where means antidifferentiate, or integrate, or find an indefinite integral and dx indicates that
the integration of the function is with respect to x.
Since

d
(ax + c) = a, where a and c are constants
dx

then

a dx = ax + c

Since
then

d ax n + 1
= ax n
dx n + 1

ax n dx =

ax n + 1
+ c, n 1
n +1

In the expression above, the term c is used to denote a constant. In the antiderivative of a
function, there are an infinite number of possibilities for c. However, when we are finding an
antiderivative, we set c to zero. That is, finding an antiderivative means let c = 0, or do not
add on the c.
For example, the antiderivative of 3x2 + 4x + 5 is x3 + 2x2 + 5x + c. An antiderivative of
2
3x + 4x + 5 is x3 + 2x2 + 5x.

Properties of integrals
Since d is a linear operator, so too is ..Therefore,
dx
[ f ( x) g( x)] dx = f ( x) dx g( x) dx
That is, each term can be integrated separately, and

k f ( x) dx = k f ( x) dx
That is, a constant factor of the function can be taken to the front of the integral.
So [af ( x) bg( x)]dx = a f ( x )dx b g( x )dx
Worked Example 1

Antidifferentiate each of the following, expressing answers with positive indices.

3
a 2x7 b 4x 3 c
x
Think
a

Write

Integrate by rule; that is, add 1 to the index and


divide by the new index.

Simplify.

Integrate by rule.

Simplify.

Express the answer with a positive index.

When a square root is involved, replace it with


a fractional index.

2 x 7 dx =

2 x8
+c
8

x8
+c
4

4x 2
dx = + c
2
=

4x

= 2x 2 + c
2
= 2
x
3
3
dx = 1 dx
x
x2

Chapter 9 Integration

417

= 3 x

Bring the x to the numerator and change the sign of


the index.

1
2 dx

3x 2

+c

Integrate by rule.

Simplify.

= 6x 2 + c

Write the answer in the form it was given.

=6 x +c

1
2
1

Worked Example 2

Find the following indefinite integral.


(x. - 1)(3x + 5) dx
Think

Write

( x 1)(3x + 5) dx = (3x 2 3x + 5x 5) dx
= (3 x 2 + 2 x 5) dx

Expand the expression.

Collect like terms.

Integrate each term separately.

Simplify each term.

= x3 + x 2 5x + c

3x3 2 x 2
+
5x + c
3
2

Integration of (ax + b)n where n 1


By applying the chain rule for differentiation:
d
(ax + b) n + 1 = a( n + 1)(ax + b) n
dx


so

a(n + 1)(ax + b)n dx = (ax + b)n + 1 + c

or

a( n + 1) (ax + b) n dx = (ax + b) n + 1 + c

or

(ax + b)n dx =

(ax + b) n + 1
+c
a( n + 1)

Worked Example 3
n+1

Antidifferentiate 4(5x 2)3 by using ( ax + b) n dx = ( ax + b)

a( n + 1)
Think

418

+ c.

Write

4(5x 2)3 dx = 4 (5x 2)3 dx

Express as an integral and take 4 out as a factor.

Apply the rule where a = 5 and n = 3.

Simplify the antiderivative by cancelling the


fraction.

= 5(5x 2)4 + c

Maths Quest 12 Mathematical Methods CAS for the TI-Nspire

4(5 x 2)4
+c
5(4)
1

Integration of x1
Since
then

1
d
log e ( x ) =
dx
x
1
x dx = loge ( x) + c, where x > 0

or

1
dx

= log e x + c.

Worked Example 4

Antidifferentiate 4 .
7x
Think
1

Write

=
2

7 x dx = 7 x dx

Take 47 out as a factor.

4
7

x dx

= 47 log e x + c

Integrate by rule.

-1

Integration of (ax + b)

This can be done by applying the chain rule for differentiation:


d
a
log e (ax + b) =
, where a and b are constants.
dx
ax + b
Multiplying both sides by 1 gives
a
1 d
a
1

log e (ax + b) =

a dx
ax + b a
1
ax + b
1
1 d
log e ax + b dx
dx =
ax + b
a dx
=


So

so

( ax + b)

dx =

1
log e ax + b + c
a

1
Note that the a in the fraction a is the derivative of the linear function ax + b.
Worked Example 5

Antidifferentiate

5 .
2x + 3

Think

Write

Method 1: Technology-free
1

Express as an integral and take 5 out as a


factor.

Integrate by rule where a = 2.

2 x + 3 dx = 5 2 x + 3 dx
=

5
log e 2 x + 3 + c
2

Chapter 9 Integration

419

Method 2: Technology-enabled
1

On a Calculator page, press:


MENU b
4:Calculus 4
3:Integral 3
Alternatively, select the indefinite integral
template from the Maths expression
template and fill in the required fields.
Complete the entry line as: 5 dx
2 x + 3
Press ENTER .

Write the answer and put in the constant of


integration, as the CAS calculator omits it.
Also note that the CAS uses ln instead of
loge.

5
5

dx = log e 2 x + 3 + c
2
2 x + 3

Worked Example 6

Find

6x + 5
dx .
x2

Think

Write

6x + 5
6x 5
dx = 2 + 2 dx
2
x
x
x

Express as separate fractions.

Simplify each fraction.

= (6 x

Integrate each term separately by rule.

= 6 log e x +

Simplify leaving the answer with positive indices.

= 6 log e x 5 x + c
5
= 6 log e x + c
x

+ 5 x 2 ) dx
5x

1
1

+c

Worked Example 7

Find the equation of the curve g(x) given that g(x) = 3 x + 2 and the curve passes through (1, 2).
Think

420

Write

Write the rule for g(x).

g(x) = 3 x + 2

Rewrite g(x) in index form.

g(x) = 3 x 2 + 2

Express g(x) in integral notation.

g(x) =

Antidifferentiate to obtain a general rule for g(x).

Maths Quest 12 Mathematical Methods CAS for the TI-Nspire

3x 2 + 2 dx
3

= 3x 2 3 + 2 x + c
2

Simplify.

3
2

= 3x 2 + 2 x + c
3
1

g(x) = 2 x 2 + 2 x + c

Substitute coordinates of the given point into g(x).

Find the constant of antidifferentiation, c.

State the rule for g(x) in the form that it is given.

As g(1) = 2, 2(1) 2 + 2(1) + c = 2


2+2+c=2
so c = 2
3

g(x) = 2 x 2 + 2 x 2

= 2 x3 + 2x 2

Worked Example 8

If a curve has a stationary point (2, 3), and a gradient of 2x k, where k is a constant, find:
a the value of k b y when x = 1.
Think
a

Write

The gradient is dy so write the rule for the


dx
gradient.

Let dy = 0 (as stationary points occur when


dx
the derivative is zero) and substitute the value
of x into this equation.

dy
= 2x k
dx
For stationary points,
dy
= 0, so 2x k = 0
dx
2(2) k = 0 as x = 2

Solve for k.

Rewrite the rule for the gradient function,


using the value of k found in a above.

Integrate to obtain the general rule for y.

4 k = 0 so k = 4
b

dy
= 2x 4
dx
y = (2 x 4)dx
= x2 4x + c

Substitute the coordinates of the given point


on the curve to find the value of c.

Since curve passes through (2, 3),


3 = 22 4(2) + c
3=48+c
c=7

State the rule for y.

So y = x2 4x + 7

Substitute the given value of x and calculate y.

When x = 1,

y = (1)2 4(1) + 7

=4

The relationship between the graph of an antiderivative


function and the graph of the original function
f(x) is the antiderivative of f(x) and is written as f ( x ) = f ( x )dx.
f(x) is the gradient function of the antiderivative function f(x).

Chapter 9 Integration

421

The graph of the antiderivative function f(x) can be derived from the graph of f(x) and results in
a family of curves. For example if f(x) = 1 then the antiderivative function is f(x) = x + c, where
c can take any real value.

Sketching the antiderivative function from the graph of the


original function
1. The general shape of the graph of the antiderivative function can be determined from the
graph of a polynomial function by increasing the degree by one. For example, if f(x) is a
quadratic function, then f(x) is a cubic function.
2. The x-intercepts of f(x) become the turning points on the graph of f(x).
3. When f(x) is above the x-axis, the gradient of f(x) is positive.
4. When f(x) is below the x-axis, the gradient of f(x) is negative.
Worked Example 9

Sketch the graph of the antiderivative function from the graph of the gradient
function f(x) shown.
f (x)

(1, 0)

(2, 0)
0

(0, 1)
Think

Write

State the shape of the antiderivative function.

The antiderivative function will be a positive


cubic function.

Find the x-intercepts of the gradient function,


f(x), and hence find the x-coordinates of the
turning points.

There are x-intercepts when x = 2 and when


x = 1, so f(x) has turning points when x = 2 and
x = 1.

Find when the given graph, f(x), is above the


x-axis and hence find when f(x) has a positive
gradient.

f(x) is above the x-axis when x < 2 and when


x > 1, so f(x) has a positive gradient when x < 2
and when x > 1.

Find when the given graph, f(x), is below the


x-axis and hence find when f(x) has a negative
gradient.

f(x) is below the x-axis when 2 < x < 1, so f(x)


has a negative gradient when 2 < x < 1.

Sketch the curve.

The graph could be any of the family of graphs


formed by vertical translations of the graph
shown.
f(x)

422

Maths Quest 12 Mathematical Methods CAS for the TI-Nspire

REMEMBER

1.

d
f ( x ) = f ( x )
dx

2. f ( x ) = f ( x ) dx
3. a dx = ax + c
4. ax n dx =

ax n + 1
+ c, n 1
n +1

5. [ f ( x ) g( x )] dx =

f ( x) dx g( x) dx

6. k f ( x ) dx = k f ( x ) dx
7. (ax + b) n dx =

(ax + b) n + 1
+ c, n 1
a( n + 1)

1
dx = log e x + c, x 0 or x 1 dx = log e x + c
x

8.

1
1
1
dx = log e ax + b + c or (ax + b)1 dx = log e ax + b + c
ax + b
a
a
10. Write the answer in the same form as the question unless otherwise stated.
11. To draw the graph of the antiderivative function f(x) from the graph of the gradient
function f(x), use the x-intercepts as the x-coordinates of the turning points, and
make the gradient positive when f(x) is above the x-axis and negative when f(x) is
below the x-axis.
12. For a polynomial function, the graph of f(x) is one degree greater than the graph of f(x).
9.

Exercise

9a

Antidifferentiation
1 WE1
a x

Antidifferentiate each of the following, giving answers with positive indices.


c x7
d 3x5
b x4

e 5x 2
x4
i

5

x3
j

2

m x 3
q

n 4 x 4

9

x2

2 WE2

2x4

10

x6

g 6x 4

x 4
k

3

h 2 x

o x

3
7

8
x

x
5
x3
6

(x x )

Find the following indefinite integrals.

a (2 x + 5) dx

b (3 x 2 + 4 x 10) dx

(10 x 4 + 6 x3 + 2) dx

d ( 4 x 5 + x 3 6 x 2 + 2 x ) dx

( x3 + 12 x 2 ) dx

( x + 3)( x 7) dx

g 5( x 2 + 2 x 1) dx

h ( x 2 + 4)( x 7) dx

x( x 1)( x + 4) dx
Chapter 9 Integration

423

3 MC

( x 2 + x + 2) dx is equal to:

A x 2 dx + x + 2

B x 2 dx + x dx + 2 dx

C (x 2 + x ) dx + 2

D x 2 + ( x + 2) dx

E x 2 + x + 2 dx
4 MC

x( x + 3) dx is equal to:

A x dx ( x + 3) dx

B x ( x + 3) dx

C ( x + 1) x dx

D x dx + ( x + 3) dx

( x 2 + 3x) dx

5 WE3 Antidifferentiate each of the following by using (ax + b) n dx =


a (x + 3)2
c 2(2x + 1)4
e (6x + 5)4
g (4 x)3
i 4(8 3x)4

k (2x + 3) 2

m 6(4x 7) 4

o (6 5x) 3
6 MC

b
d
f
h
j
l
n
p

(ax + b) n + 1
+ c.
a( n + 1)

(x 5)3
2(3x 4)5
3(4x 1)2
(7 x)4
3(8 9x)10

(6x + 5) 3

(3x 8) 6
10(7 5x)4

3( x + 2)4 dx is equal to:

A 3 + ( x + 2)4 dx

B 3 dx + ( x + 2)4 dx

C 3 ( x + 2)4 dx

D 3 dx ( x + 2)4 dx

E ( x + 2)4 3 dx
7 WE4,5 Antidifferentiate the following.

424

x dx

x dx

5x dx

3x dx

7x dx

x + 3 dx

x + 3 dx

x + 4 dx

x + 5 dx

3x + 2 dx

5x + 6 dx

2 x 5 dx

5 x dx

3 + 2x

3
6 11x dx

dx

n
q

6 + 7x dx
2

4 3x

dx

Maths Quest 12 Mathematical Methods CAS for the TI-Nspire

5 2x dx

x + 5 dx is equal to:

8 mC
A 6

1
dx
x+5

6dx

B
E

( x + 5) dx

9 We6 Find

6dx x + 5 dx

3(4 x + 1)

m
eBook plus
Digital doc

SkillSHEET 9.1
Substitution
and evaluation

x+

6dx + x + 5 dx

4(2 x 5)5 dx

( x 5)( x + 3)
dx
x3

x2 + x4
dx
x

( x + 5) dx

(2 x + 7)
dx.
x

10 For the following mixed sets, find:


1

a x4 + 2x +
b (3 x + 1)5 dx

x
5
3
d
e
dx
dx
2x + 1
6 10x
g

( x + 4) 2
dx
2x
1
3
k 5 x 2 2 x + 3 x 3 dx

dx

2
dx
3 x

x2 + 2x 1
x

dx

3x 2 + 2 x 1
dx
x2

10 x + 2 x 4
dx
x3

11 We7 Find the equation of the curve f (x) given that:


a f (x) = 4x + 1 and the curve passes through (0, 2)
b f (x) = 5 2x and the curve passes through (1, 1)

c f (x) = x 2 + 3 and the curve passes through (1, 4)


d f (x) = x + x and f (4) = 10
1

e f (x) = x 3 3 x 2 + 50 and f (8) = 100


1
f f (x) =
2 x and f (1) = 5
x
g f (x) = (x + 4)3 and the curve passes through (2, 5)

h f (x) = 8(1 2x) 5 and f (1) = 3


1
i f (x) = (x + 5) and the curve passes through (4, 2)
8
j f (x)
and f (3) = 7
( x) =
7 2x
12 We8 If a curve has a stationary point (1, 5), and a gradient of 8x + k, where k is a constant, find:
a the value of k
b y when x = 2.
13 A curve g(x) has g ( x ) = kx +2 x , where k is a constant, and a stationary point (1, 2). Find:
x
a the value of k
b g(x)
c g(4).
14 We9 Sketch the graph of the antiderivative functions from each of the following graphs.
a
b
c
f (x)
f(x)
f (x)

(1, 0)

(1, 0)
x

(2, 0)
0

(0, 2)

(0, 3)

(0, 1)

Chapter 9

Integration

425

f (x)

f (x)

f (x)

(0, 1)
(0, 0)

( 12 , 0)
0 (1, 0)

(0, 1)

f (x)

(2, 0) (0, 0)

9B

(1, 0)

Integration of e x, sin (x) and cos (x)


Integration of the exponential function e x
Since

d x
(e ) = e x
dx

then

e x ddxx = e x + c

and
or

Therefore,

d kx
(e ) = kke kx , where k is a constant
dx

kekx dx = ekx + c
k e kx dx = e kx + c
1
ekx ddx = k ekx + c.

Worked examPle 10

Antidifferentiate each of the following.

e 5x
4x
a 3e
b
c (e x 1)2
4
ThInk
a

426

WrITe

Integrate by rule where k = 4.

Simplify.

Rewrite the function to be integrated so that


the coefficient of the e term is clear.

Integrate by rule where k = 5.

maths Quest 12 mathematical methods CaS for the TI-nspire

3e 4 x
+c
4
3
= 4 e4 x + c

3e4 x ddxx =

5 x

dx =
=

4 e 5 x ddx
1
4

5 x

+c

1
= 4e

Simplify the antiderivative.

5x
5x

1
= 20
e

Expand the function to be integrated.

Integrate each term by the rule.

5 x

1
5

+c

+c

(e x 1)2 dx = (e2 x 2e x + 1) dx
= 12 e 2 x 2e x + x + c

Integration of trigonometric functions


Since

d
[sin (ax)] = a cos (ax)
dx

and

d
[cos (ax )] = a sin (ax )
dx

it follows that

sin ( ax) dx =

1
cos ( ax ) + c
a

cos ( ax) dx = a sin ( ax) + c


Worked examPle 11

Antidifferentiate the following.


a sin (6x)

b 8 cos (4x)

c 3 sin

x
2

ThInk

WrITe

Integrate by rule.

Integrate by rule.

Simplify the result.

Integrate by rule.

Simplify the result.

sin (6 x)dx = 6 cos (6 x) + c


8
8 cos (4 x)dx = 4 sin (4 x) + c
= 2 sin (4x) + c

x
dx =
2

3 sin

3
1
2

x
cos + c
2

x
= 6 cos + c
2

Worked examPle 12

Find [ 2 e 4 x 5 sin ( 2 x ) + 4 x ] dx.


ThInk

WrITe

Method 1: Technology-free
1

Integrate each term separately.

[2e4 x 5 sin (2 x) + 4 x]dx


= 42 e 4 x

Simplify each term where appropriate.

5
2

cos (2 x ) + 42 x 2 + c

= 12 e 4 x + 25 cos (2 x ) + 2 x 2 + c

Chapter 9

Integration

427

Method 2: Technology-enabled
1

On a Calculator page, press:


MENU b
4:Calculus 4
2:Integral 2
Complete the entry line as:

(2e4 x 5 sin (2 x) + 4 x) dx
Press ENTER .
Note: When integrating trigonometric
functions, ensure the CAS is set to radian
mode.

(2e 4 x 5 sin (2 x ) + 4 x ) dx

Write the answer and put in the constant of


integration.

e 4 x 5 cos (2 x )
+
+ 2x2 + c
2
2

REMEMBER

Exercise

9b

1.

e x dx = e x + c and ekx dx = k ekx + c

2.

sin (ax) dx =

cos (ax ) + c and cos (ax ) dx =

1
sin (ax ) + c
a

Integration of ex, sin (x) and cos (x)


1 WE10
a e2x

d e 3x
e6 x
g

2
j

8e2x
x

m 3e 2
e x e x
p
2

Antidifferentiate each of the following.


b e4x
e 5e5x
2e 3 x
h

3
x

x
3

3e6x
x

l 0.1e 4

k e 3
n 3e

c ex
f 7e4x

o e x + ex

2 Find an antiderivative of (1 + ex)2.


3 Find an antiderivative of (ex 1)3.
4 Find an antiderivative of x3 3x2 + 6e3x.
5 MC If f(x) = e2x + k and f(x) has a stationary point (0, 2), where k is a constant, then:
a k is equal to:
A e
B e2
C 1
D 1
E e
b f(1) is equal to:
A e2 1
B e 2 + 12
C 12 e 2 + 12
D e4
E 1 12

428

Maths Quest 12 Mathematical Methods CAS for the TI-Nspire

6 WE11
Antidifferentiate the following.
a sin (3x)
b sin (4x)
cos
(
2
x
)
d

e sin (2x)
3
4
sin
(6 x )
g
h 8 cos (4x)
3
x
j 2 cos (x)
k sin
3
x
x
m 3 sin
n 2 sin
5
4
x
p 6 cos
2
s

2 sin

5x

2

3 cos

sin (3x)
x
l cos
2
x
o 4 cos
4

3x
r 6 cos
4

7x

4

u 5 sin (x)
4x
s
x sin

b [sin (2x ) cos ( x )] dx

[cos (4 x) + sin (2 x)] dx


e [4 cos (4 x ) 13 sin (2 x)] dx
c

x
w 2 cos
3

7 WE12 Find:
a [sin ( x ) + cos ( x )] dx

3 sin ( 2x

f cos (3x)

2x
q 4 sin
3

x
v 3 cos
2

c cos (7x)

) + 2 cos ( 3x ) dx

) cos (2x) dx

sin ( 2x

[5x + 2 sin ( x)] dx

h [3e6 x 4 sin (8 x ) + 7] dx

8 Find the antiderivative of e4x + sin (2x) + x3.


9 Find an antiderivative of 3x2 2 cos (2x) + 6e3x.
10 Antidifferentiate each of the following.
1
a x 3
b x2 + 4 cos (2x) ex
+ e2 x
2x + 3
x
x
c sin + e 2 (3 x 1)4
3

x
x
x
e 3 sin 2 cos e 5
3
2

1
x
+ e 4 x + cos
5
3x 2
x
x + 2 x 2 sin + 5
3

11 In each of the following, find f(x) if:

a f(x) = cos (x) and f = 5


b f(x) = 4 sin (2x) and f(0) = 1
2
x
x
x
c f ( x ) = 3 cos and f ( ) = 9 2
d f ( x ) = cos sin and f(2) = 2.
2
4
4
dy
x
12 If
= sin + k, where k is a constant, and y has a stationary point (3, 4), find:
6
dx
a the value of k
b the equation of the curve
c y when x = 6.
13 A curve has a gradient function f(x) = 4 cos (2x) + kex, where k is a constant, and a stationary
point (0, 1). Find:
a the value of k
b the equation of the curve f(x)

c f correct to 2 decimal places.


6

Chapter 9 Integration

429

9C

Integration by recognition
As we have seen, if d [ f ( x )] = g( x )
dx
then

g( x) dx = f ( x) + c, where g( x) = f '( x).

This result can be used to determine integrals of functions that are too difficult to
antidifferentiate, via differentiation of a related function.

Worked examPle 13

eBook plus

a Find the derivative of the function y = (5x + 1)3.


b Use this result to deduce the antiderivative of 3(5x + 1)2.

Tutorial

int-0564

ThInk
a

WrITe

Write the function and recognise that the chain


rule can be used.

Let u equal the function inside the


brackets.
du
Find .
dx

3
4

Express y in terms of u.

Find

Write the chain rule.

Find

Replace u with the expression inside the


brackets and simplify where applicable.

Worked example 13

a y = (5x + 1)3

Let u = 5x + 1
du
=5
dx
y = u3

dy
.
du

dy
= 3u 2
du
dy dy du
=

dx du dx
dy
= 3u 2 5
dx

dy
using the chain rule.
dx

du
dx = y + c1, express the relationship
dx
in integral notation.
dy
Remove a factor from
so that it resembles
dx
the integral required.
Since

Divide both sides by the factor in order to


obtain the integral required.

= 15(5x + 1)2
b

15(5x + 1)2 ddxx = (5x + 1)3 + c1


5 3(5 x + 1)2 dx = (5 x + 1)3 + c1

3(5x + 1)2 dx = 15 (5x + 1)3 + c,


where c =

c1
5

Therefore, the antiderivative of


3(5x + 1)2 is 15 (5x
( 5x ++ 11))33 + cc..

Note that the shorter form of the chain rule below can be used to differentiate.
If f ( x ) = [ g( x ))]]n then f ( x ) = ng ( x )[ g( x ))]]n 1 .

430

maths Quest 12 mathematical methods CaS for the TI-nspire

Worked examPle 14
a Differentiate e x .
3

b Hence, antidifferentiate 6x2 e x .


3

ThInk
a

Write the equation and apply the


chain rule to differentiate y.

Let u equal the index of e.

Find

Express y in terms of u.

Find

WrITe
a

y = ex

Let u = x3

du
.
dx

du
= 3x 2
dx
y = eu

dy
.
du
dy
Find
using the chain rule and
dx
replace u.

dy
= eu
du
dy
= 3 x 2 eu
dx
= 3x 2 e x

dy
in integral notation.
dx

Express

Multiply both sides by a constant to


obtain the integral required.

3x 2 e x dx = e x
3

+ c1

2 3 x 2 e x dx = 2e
2e x + 2c1
3

2e x
6 x 2 e x dx = 2e
3

+ c,
c where c = 2c1.

Therefore, the antiderivative of 6 x 2 e x is 2e x + c.


c
3

Note that the shorter form of the chain rule below can be used to differentiate.
If y = e f ( x ) then

dy
= f ( x)e f ( x ) .
dx

Worked examPle 15
a Find the derivative of sin (2 x + 1) and use this result to deduce the antiderivative of 8 cos (2 x + 1).
b Differentiate loge(5x2 2) and hence antidifferentiate
ThInk
a

x
.
5 x2 2
WrITe
a f (x) = sin (2x + 1)

Define f (x).

Differentiate using f (x) = g(x) cos [g(x)]


where f (x) = sin [g(x)].

Express f (x) using integral notation.

Multiply both sides by whatever is necessary


for it to resemble the integral required.

4 2 cos (2 x + 1)
1)dx = 4 sin (2 x + 1) + c

Write the integral in the form in which the


question is asked.

8 cos (2 x + 1)dx = 4 ssinin (2 x + 1) + c

Define f (x).

f (x) = 2 cos (2x + 1)

2 cos (2 x + 1)dx = sin (2 x + 1) + c1

The antiderivative of 8 cos (2x + 1) is


4 sin (2x + 1) + c.
b f (x) = loge (5x2 2)

Chapter 9

Integration

431

Differentiate using f ( x ) =
where f (x) = loge [g(x)].

g ( x )
g( x )

Express f (x) using integral notation.

Take out a factor so that f (x) resembles the


integral required.

Divide both sides by the factor to obtain the


required integral.

10 x
5x 2 2

f ( x ) =

10
10x

5x 2 2 dx = loge
10

5 x 2 2 + c1

x
dx = log e 5 x 2 2 + c1
2

5x 2

5x 2 2 dx = 101 loge

5x 2 2 + c

The antiderivative of
x
is 1 log e 5 x 2 2 + c.
5 x 2 2 10

Worked examPle 16

Differentiate x cos (x) and hence find an antiderivative of x sin (x).


ThInk

WrITe

Let y = x cos (x)

Write the rule.

Apply the product rule to differentiate x cos (x).

Express the result in integral notation. (Do not


add c, as an antiderivative is required.)

[cos ( x ) x sin
sin ( x )] ddxx = x cos ( x )

Express the integral as two separate integrals.

Simplify by integrating. (Do not add c.)

Make the expression to be integrated the subject


of the equation.

cos ( x ) dx x ssin
in ( x ) ddxx = x cos ( x )
cos
sin ( x ) x ssin
in ( x ) ddxx = x cos ( x )
x sin ( x ) dx = x cos ( x ) sin ( x )

Simplify.

dy
= x [sin (x)] + [cos (x)](1)
dx
= x sin (x) + cos (x)
= cos (x) x sin (x)

x ) dx = ssin
in ( x ) x cos ( x )
x sin ( x)
Therefore, an antiderivative of x sin (x) is
sin (x) x cos (x).

Worked examPle 17
a Show that

5x + 1
4
= 5
.
x+1
x+1

b Hence, find

ThInk
a

432

Usealgebraiclongdivisiontodividethe
numerator into the denominator.

5x + 1
dx.
x+1
WrITe

a x + 1 5x + 1

maths Quest 12 mathematical methods CaS for the TI-nspire

5x + 5
4

So

5x + 1
4
= 5
x +1
x +1

5x + 1
4

dx = 5

x +1
x + 1

Write the expression using integral


notation.

Express as two separate integrals.

= 5 dx

Antidifferentiate each part.

= 5 x 4 log e x + 1 + c

4
dx
x +1

rememBer

1. To differentiate using the chain rule, use one of the following rules.
(a) If f (x) = [g(x)]n then f (x) = ng(x)[g(x)]n 1
(b) If y = e f (x),

dy
= f ( x )e f ( x )
dx

dy dy du
=

dx du dx
(d) f (x) = g(x) cos [g(x)] where f (x) = sin [g(x)]
f (x) = g(x) sin [g(x)] where f (x) = cos [g(x)]
(c)

(e) f ( x ) =

g ( x)
where f ( x ) = logg e g( x ) .
g( x )

2. To antidifferentiate, use g( x ) dx = f ( x ) + c where g(x) = f (x).

exerCISe

9C

Integration by recognition
1 We13
For each of the following find the derivative of the function in i and use this result
to deduce the antiderivative of the function in ii.
a i (3x 2)8
ii 12(3x 2)7
2
5
b i (x + 1)
ii 5x(x2 + 1)4
1
c i 2x 5
ii
2x 5
d i

4x + 3

+ 3x
1
f i 2
x 1
e i

(x2

ii
7)4

2
4x + 3

ii (2x + 3)(x2 + 3x 7)3


4x
ii
2
( x 1)2

2 mC The derivative of (x + 7)4 is 4(x + 7)3.


a Therefore, the antiderivative of 4(x + 7)3 is:
A (x + 7)4 + c
B 14 (x + 7)4 + c
D 3(x + 7)4 + c
E 12(x + 7)4 + c
3
b The antiderivative of (x + 7) is:
A (x + 7)4 + c
B 14 (x + 7)4 + c
D 3(x + 7)4 + c

C 4(x + 7)4 + c

C 4(x + 7)4 + c

E 12(x + 7)4 + c

Chapter 9

Integration

433

3 mC If the derivative of (2x 3)6 is 12(2x 3)5, then 6(2 x 3)5 dx is:
A 2(2x 3)6 + c
B 4(2x 3)6 + c
C (2x 3)6 + c
1
6
6
D 6(2x 3) + c
E 2 (2x 3) + c
4 We14

For each of the following differentiate i and hence antidifferentiate ii.

e4x 5

a i
c i ex

ii 2e4x 5
ii x e x

b i e6 5x
d i ex x

ii 10e6 5x
ii (1 2 x )e x x

5 We15
For each of the following find the derivative of the function in i and use this result
to deduce the antiderivative of the function in ii.
a i sin (x 5)
ii cos (x 5)
b i sin (3x + 2)
ii 6 cos (3x + 2)
c i cos (4x 7) ii sin (4x 7)
d i cos (6x 3)
ii 3 sin (6x 3)
e i sin (2 5x)
ii 10 cos (2 5x)
f i cos (3 4x)
ii 2 sin (3 4x)
12 x
20
g i loge (5x + 2) ii
h i loge (x2 + 3)
ii 2
5x + 2
x +3
i i loge (x2 4x) ii

x2
x2 4x

6 We16

Differentiate i and hence find an antiderivative of ii.


2[ x cos
cos ( x ) sin ( x )]
sin ( x )
a i x cos (x) + 2 sin (x)
ii x sin (x)
b i
ii
x
x2
c i e x sin (x)
ii 3e x [sin (x) + cos (x)]
d i x sin (x)
ii x cos (x)
e i x ex
ii x ex

7 For each of the following differentiate i and use this result to antidifferentiate ii.
a i (2x 3x2)6
ii 6x5(1 3x)(2 3x)5
x3 + 2x

b i
8 We17

a Show that

9 a Show that
10

x3 + 2x

3x 2
1
=3+
.
x 1
x 1

8x 7
5
=4+
.
2x 3
2x 3

6x 5
4
= 3+
.
3 2x
3 2x
2

b Hence, find

3x 2
dx.
x 1

b Hence, find

5x + 8
dx.
x+2

b Hence, find

8x 7
dx.
2x 3

b Hence, find

6x 5
dx.
3 2x

If y = loge[cos (x)]:
a find

dy
.
dx

13 Differentiate
14

3x 2 + 2

5x + 8
2
=5
.
x+2
x+2

a Show that

11 a Show that
12

ii

b Hence, find ta
tan ( x ) dx.
cos ( x )
1
and hence find an antiderivative of 2 .
sin ( x )
sinn ( x )

Differentiate loge (3x2 4) and hence find an antiderivative of

x
.
4

3x 2

15 Differentiate sin (ax + b) and hence find an antiderivative of cos (ax + b). (Here, a and b are
constants.)
16 Differentiate cos (ax + b) and hence find an antiderivative of sin (ax + b). (Here, a and b are
constants.)

434

maths Quest 12 mathematical methods CaS for the TI-nspire

17 Differentiate eax + b and hence find an antiderivative of eax + b. (Here, a and b are constants.)
18 Antidifferentiate each of the following.
a sin (3 x + 1)
b cos (1 4 x)
x

d sinn 2 +
e 3 cos
+ 5

9d

eBook plus

c e x + 3

Digital doc

f cos (x)esin (x)

WorkSHEET 9.1

approximating areas
enclosed by functions

eBook plus
Interactivity

int-0254
Approximating areas
enclosed by functions

There are several ways of finding an approximation to the area


between a graph and the x-axis. We will look at two methods:
1. the left rectangle method
2. the right rectangle method

The left rectangle method


Consider the area between the curve f (x) shown at right, the x-axis
y
f(x)
and the lines x = 1 and x = 5.
If the area is approximated by left rectangles of width 1 unit then
the top left corner of each rectangle touches the curve at one point.
So,
the height of rectangle R1 is f (1) units
and
the area of R1 = 1 f (1) square units (area of a
R1 R2 R3 R4
rectangle = height width).
Similarly,
the area of R2 = 1 f (2) square units,
0 1 2 3 4 5 x
the area of R3 = 1 f (3) square units,
the area of R4 = 1 f (4) square units.
Therefore, the approximate area under the graph between the curve f (x), the x-axis and the lines
x = 1 to x = 5 is 1[ f (1) + f (2) + f (3) + f (4)] square units, (the sum of the area of the four rectangles).
If the same area was approximated using rectangle widths of 0.5 there would be 8 rectangles
and the sum of their areas would be:
0.5[ f (1) + f (1.5) + f (2) + f (2.5) + f (3) + f (3.5) + f (4) + f (4.5)] square units.
From the diagram it can be seen that the left rectangle approximation is less than the actual
area under the curve.
Worked examPle 18

Find an approximation for the area between the


curve f (x) shown and the x-axis from x = 1 to
x = 3 using left rectangles of width 0.5 units.
The graph shown has the equation f (x) = 0.2x2 + 3.

eBook plus

f(x)

Tutorial

int-0565
Worked example 18

0 0.5 1 1.5 2 2.5 3 x


ThInk

WrITe

Method 1: Technology-free
1

Write the number of rectangles and their


width.

There are 4 rectangles of width 0.5 units.

Chapter 9

Integration

435

Find the height of each rectangle (left) by


substituting the appropriate x-value into the
f(x) equation.

h1 = f(1) = 0.2(1)2 + 3 = 3.2


h2 = f(1.5) = 0.2(1.5)2 + 3 = 3.45
h3 = f(2) = 0.2(2)2 + 3 = 3.8
h4 = f(2.5) = 0.2(2.5)2 + 3 = 4.25

Area equals the width multiplied by the


sum of the heights.

Area = width (sum of heights of 4 rectangles)



= 0.5(3.2 + 3.45 + 3.8 + 4.25)

= 0.5(14.7)

Calculate this area.

State the solution.

The approximate area is 7.35 square units.

= 7.35

Method 2: Technology-enabled
1

Store the function to f(x). On a Calculator


page, press:
MENU b
1:Actions 1
1:Define 1
Complete the entry line as:
Define f(x) = 0.2x2 + 3
Press ENTER .

To find the approximate area, complete the


entry line as:
0.5(f(1) + f(1.5) + f(2) + f(2.5))
Press ENTER .

Write the answer with the correct units for


area.

The approximate area is 7.35 square units.

The right rectangle method


y
f(x)
Consider the area between the curve f(x) shown at right, the x-axis and
the lines x = 1 and x = 5.
If the area is approximated by right rectangles of width 1 unit then
the top right corner of each rectangle touches the curve at one point.
So,
the height of R1 is f(2) units
and
the area of R1 is 1 f(2) square units.
R1 R2 R3 R4
Similarly,
the area of R2 = 1 f(3) square units,
0 1 2 3 4 5 x

the area of R3 = 1 f(4) square units

the area of R4 = 1 f(5) square units.
Therefore, the approximate area between the curve f(x), the x-axis
and the lines x=1 to x = 5 is (R1 + R2 + R3 + R4) = 1[ f(2) + f(3) + f(4) + f(5)] square units.
If the same area was approximated with upper rectangle widths of 0.5 units, the sum of their
areas would equal:
0.5[ f(1.5) + f(2) + f(2.5) + f(3) + f(3.5) + f(4) + f(4.5) + f(5)] square units.
From the diagram it can be seen that the right rectangle approximation is greater than the
actual area under the curve.

For an increasing function, left rectangle approximation actual area right rectangle
approximation.
For a decreasing function, left rectangle approximation actual area right rectangle
approximation.

436

Maths Quest 12 Mathematical Methods CAS for the TI-Nspire

Worked Example 19

Find an approximation for the area in the diagram in worked example 18 using right rectangles of
width 0.5 units. f(x) = 0.2x2 + 3
Think

Write

Find the number of rectangles and the height of


each one (from left to right).

There are 4 rectangles:


h1 = f(1.5) = 0.2(1.5)2 + 3 = 3.45
h2 = f(2) = 0.2(2)2 + 3 = 3.8
h3 = f(2.5) = 0.2(2.5)2 + 3 = 4.25
h4 = f(3) = 0.2(3)2 + 3 = 4.8

Area is the width of the interval multiplied by the


sum of the heights.

Area = 0.5(3.45 + 3.8 + 4.25 + 4.8)

Calculate the area.

State the solution.

The approximate area is 8.15 square units.

= 0.5(16.3)
= 8.15

It can be seen that the left rectangle approximation (7.35 units) is less than the right rectangle
approximation (8.15 units).
If the area is divided into narrower strips, the estimate of the area would be closer to the true
value.
Worked Example 20

With width intervals of 1 unit, calculate an approximation for the area between the graph
of f(x) = x2 + 2 and the x-axis from x = 2 to x = 3 using:
a left rectangles b right rectangles c averaging of the left and right rectangle areas.
Think

Write

Sketch the graph of f(x) over a domain which


exceeds the width of the required area

Draw the left and right rectangles.

y = x2 + 2

2
2 1

0 1 2 3 x
= Left rectangles
= Right rectangles

Calculate the height of the left rectangles


by substituting the appropriate values
of x into the equation for f(x). Note the
two rectangles to the right and left of
the origin have the same height and are
equal in area.
Find the area by multiplying the width
by the sum of the heights.

a Left rectangle heights:

f (2) = (2)2 + 2 = 6
f(1) = (1)2 + 2 = 3
f(0) = 02 + 2 = 2
f(1) = 12 + 2 = 3
f(2) = 22 + 2 = 6

Area = 1(6 + 3 + 2 + 3 + 6)

= 20
Using left rectangles, the approximate area is
20 square units.

Chapter 9 Integration

437

Calculate the height of the right


rectangles by substituting the appropriate
values of x into the equation for f (x).

b Right rectangle heights:

Find the area by multiplying the width


by the sum of the heights.

Area = 1(3 + 2 + 3 + 6 + 11)


= 25
Usingrightrectangles,theapproximateareais
25 square units.
20 + 25
c Average of the areas =
2
= 22.5
The approximate area is 22.5 square units when
averaging the left and right rectangle areas and
using widths of 1 unit.

c Find the average by adding the area of the

left rectangles and right rectangles and


dividing by 2.

f (1) = 3 (from above)


f (0) = 2
f (1) = 3
f (2) = 6
f (3) = 32 + 2 = 11

Note that this average is between the area of the left rectangles and the area of the right
rectangles and is closer to the actual area.
rememBer

1. An approximation to the area between a curve and the x-axis can be found by dividing
the area into a series of rectangles that are all the same width. The approximation is
found by finding the sum of all the areas of the rectangles.
2. For an increasing function, left rectangle approximation actual area right rectangle
approximation.
3. For a decreasing function, left rectangle approximation actual area right rectangle
approximation.
exerCISe

9d

approximating areas enclosed by functions


1 We18 Find an approximation for the area between the curve f (x)
at right and the x-axis from x = 1 to x = 5 using left rectangles of
width 2 units.
2 Find an approximation for the area between the curves below
and the x-axis, from x = 1 to x = 5, by calculating the area of the
shaded rectangles.
y
a
b
y
f(x)
(5, 4)

4
2

19

(1, 2)

12
0

438

f(x)

(4, 19)
(3, 12)

7 (2, 7)
4
(1, 4)
3
0 1 2 3 4 5x

maths Quest 12 mathematical methods CaS for the TI-nspire

f(x)

(3, 3)
3
(1, 2)
2
0 1

3 MC Consider the graph of y = x2 from x = 0 to x = 4 at right.


a The width of each rectangle is:
A 1 unit
B 2 units
C 3 units
D 4 units
E varying
b The height of the right-most rectangle is:
A 9 units
B 4 units
C 16 units
D 12 units
E 1 unit
c The area between the curve y = x2 and the x-axis from x = 0 to
x = 4 can be approximated by the area of the left rectangles as:
A 20 sq. units
B 14 sq. units
C 18 sq. units
D 15 sq. units
E 30 sq. units
4 WE19 a Find an approximation for the area in the diagram at right
using right rectangles of width 1 unit.
b A better approximation for the area under this curve can be found by
averaging the right and left rectangle areas. State this approximate
value.
5 Find an approximation for the area between the curves below and
the x-axis, from x=1 to x=5, by calculating the area of the shaded
rectangles.
b y
c
a y
(1, 8)

8
7

(1, 8) (3, 8)

5 f(x)

(5, 5)

6
4

0 1

1 2 3 4

y
11
10

1 2 3 4

(2, 11)

(3, 10)
(4, 7)

1 2 3 4 5

f y
4

f(x)

(1, 4)

f(x)

(2, 6)

(3, 2)

(1, 4)

x
0

f(x)

(4, 10)
(3, 9)

y = x2

x
5 f(x)

y
10
9

f(x)

(3, 3)

8
7

0 1 3

y = x2

2 3 4 5

y
7

(1, 7)
(2, 5)

5
4

(4, 7)

f(x)
x

6 WE20 With width intervals of 1 unit calculate an


approximation for the area between the graph of
f(x) = x2 + 4 and the x-axis from x = 1 to x = 4 using:
a left rectangles
b right rectangles
c averaging of the left and right rectangle areas.

y
y = x2 + 4

2 3 4

Chapter 9 Integration

439

7 Find the approximate area between the curves below and the x-axis, over the interval indicated,
by calculating the area of the shaded rectangles. Give exact answers.
y
a y

b
y = ex
y = x2 + 3x + 8

1 2 3 4
x = 1 to x = 4

c y

x
1 0 1 2
x = 1 to x = 2

y = loge(x)

d y

0 1 2 3 4 5 x
x = 1 to x = 5

e y

f(x) =

1 3
3x

0 1 2 3 4 5
x = 1 to x = 5

3x2 + 8x

y = (x 4)2

1 2 3 4 5 6
x = 2 to x = 6

f(x) = x2 4x

3 2.5 2 1.5 1

x = 3 to x = 1

x = 2 to x = 6
2 3 4 5 6
x

y = x3 6x2

8 Calculate an approximation for the area between the graph of y = x(4 x), the x-axis and the
lines x = 1 and x = 4, using interval widths of 1 unit and:
a left rectangles
b right rectangles
c averaging the left and right rectangle areas.
9 Calculate an approximation for the area under the graph of y = x2 4x + 5 to the x-axis
between x=0 and x = 3, using interval widths of 0.5 units and:
a left rectangles
b right rectangles
c averaging the left and right rectangle areas.
10 Find an approximation for the area under the graph of y = 2x between x= 0 and x = 3, using
interval widths of 1 unit, by averaging the left and right rectangle areas.

440

Maths Quest 12 Mathematical Methods CAS for the TI-Nspire

9e

The fundamental theorem


of integral calculus
Consider the region under the curve f (x) between x = a and x = b,
where f (x) 0 and is continuous for all x [a, b].
Let F (x) be the function that is the measure of the area under
the curve between a and x.
F (x + h) is the area under the curve between a and x + h
and F (x + h) F (x) is the area of the strip indicated on the
graph.
The area of the strip is between the areas of the left and right
rectangles; that is, f (x)h < F (x + h) F (x) < F (x + h)h
or f ( x ) <

F ( x + h) F
F(( x )
< f ( x + h), h 0 (dividing by h).
h

y = f(x)

a
x x+h b
F(x)
F(x + h) F(x)

As h 0, f (x + h) f (x)
F ( x + h) F ( x )
= f ( x)
h
that is, F (x) = f (x) (differentiation from first principles).
lim

or

h0

F ( x ) = f ( x ) dx

Therefore,
that is, F (x) is an antiderivative of f (x)

f ( x) dx = F ( x) + c

or
but when x = a,

f ( x) dx = F (a) + c
= 0 (as the area defined is zero at x = a)
c = F
F((a).

or
Therefore,

f ( x) dx = F ( x) F (a)

and when x = b,

f ( x) dx = F (b) F (a).

That is, the area under the graph of f (x) between x = a and x = b is F (b) F (a).

f ( x) dx is the indefinite integral, which represents the general antiderivative of the function

being integrated.
This is the fundamental theorem of integral calculus and it enables areas under graphs to be
calculated exactly. It applies only to functions that are smooth and continuous over the interval
[a, b].
b

It can be stated as area = a f ( x ) dx


= [ F ( x )]ba [do not add c as F (x) is an antiderivative of f (x)]
= F (b) F (a)
a and b are called the terminals of this definite integral and indicate the domain over which the
integral is taken.

Chapter 9

Integration

441

f ( x ) dx is called the definite integral because it can be expressed in terms of its terminals
a and b, which are usually real numbers. In this case the value of the definite integral is a real
number and not a function.
The function being integrated, f (x), is called the integrand.

Properties of denite integrals

eBook plus

Definite integrals have the following five properties.


1.

Digital doc

f ( x ) dx = 0

Investigation
Definite integrals
c

2.

f ( x ) dx = f ( x ) dx + f ( x ) dx, a < c < b

3.

a k f ( x) dx = k a

4.

a [ f ( x) + g( x)] dx = a

5.

f ( x ) dx

f ( x ) dx =

f ( x ) dx + g( x ) dx
a

f ( x ) dx

Worked examPle 21

eBook plus

Evaluate the following definite integrals.


3

1 (2 x + 1)3 dx

Tutorial

int-0566

( 3 x 2 + 4 x 1) dx

Worked example 21

ThInk

WrITe

Method 1: Technology-free
a

Antidifferentiate each term of the


integrand and write in the form
[ F ( x )]ba.

0 (3x 2 + 4 x 1) dx
= [ x 3 + 2 x 2 x ]30

Substitute values of a and b into


F (b) F (a).

= [33 + 2(3)2 3] [0 3 + 2(0)2 0]


=

Evaluate the integral.

= 42 0
= 42

Express the integrand with a


negative power.

1 (2 x + 1)3 dx = 1 4(2 x + 1) 3 dx
2

Antidifferentiate by rule.

4(2 x + 1) 2
=

2 2
1

= (2 x + 1) 2

442

Express the integral with a


positive power.

maths Quest 12 mathematical methods CaS for the TI-nspire

1
=
2
(2 x + 1) 1

1 1
= 2 2
5 3
1 1
=
+
25 9
16
=
225

Evaluate the definite integral.

1
=
2
(2 x + 1) 1

Substitute the values of a and b


into F(b) F(a) where a = 1 and
b = 2.

Method 2: Technology-enabled
1

On a Calculator page, press:


MENU b
4:Calculus 4
3:Integral 3
Alternatively, select the definite
integral template from the
expression template and fill in the
required fields.
Complete the entry lines as:
3

0 (3x 2 + 4 x 1) dx
2

1 (2 x + 1)3 dx
Press ENTER after each entry.
2

Write the answers.

0 (3x 2 + 4 x 1) dx = 42

1 (2 x + 1)3 dx = 225

16

Worked Example 22

Find the exact value of each of the following definite integrals.


a

x
sin dx
6

0 ( e 3 x e
1

3x

) dx

Think
a

Write

Antidifferentiate the integrand,


writing it in the form [ F ( x )]ba.

Substitute values of a and b into


F(b) F(a).

x
x
sin dx = 6 cos
6
6

2

= 6 cos 6 6 cos 6



= 6 cos 6 cos
3
6

Chapter 9 Integration

443

1 3
= 6 2 6 2

Evaluate the integral.

= [ 3] [ 3 3 ]
= 3+ 3 3
b

Antidifferentiate the integrand,


using [ F ( x )]ba.

Substitute values of a and b into


F(a) F(b).

Evaluate.

0 (e3 x e

3 x

) dx
1

= 13 e3 x + 13 e 3 x
0

= 13 e3 + 13 e 3 13 e 0 + 13 e 0

= 13 e3 + 13 e
= 13 ( e3 + e

23

2)

Worked Example 23

If

0 8 x dx = 36, find k.

Think

Write

Method 1: Technology-free
1

Antidifferentiate the integrand, using


[ F ( x )]ba.

Substitute the values of a and b into


F(a) F(b).
Simplify the integral.

Solve the equation.

0 8x dx = [4 x 2 ]0k
So [4 x 2 ]k0 = 36
[4k2] [4(0)2] = 36
4k2 0 = 36
4k2 = 36
k2 = 9
k= 9
k = 3 or 3

Method 2: Technology-enabled
1
On a Calculator page, press:
MENU b
3:Algebra 3
1:Solve 1
MENU b
4:Calculus 4
3:Integral 3
Complete the entry line as:
k
solve (8 x ) dx = 36, k
0

Press ENTER .
2

444

Write the solutions.

Solving (8 x ) dx = 36 for k implies k = 3 or k = 3.

Maths Quest 12 Mathematical Methods CAS for the TI-Nspire

Sigma notation

An alternative notation for the definite integral of f(x) for


x [a, b] is the sigma notation (meaning the sum of).
y = f(x)

(xi, f(xi))

Divide the interval [a, b] into n equal subintervals with the ith subinterval of width xi and
height f(xi), i [1, n]. The area of the rectangle formed by the ith subinterval is Ai = f(xi) xi.
n

As n , xi 0 and Ai f ( x ) dx .
a

So

f ( xi ) xi
x0

f ( x ) dx = lim

i =1

Worked Example 24

The interval [1, 3] is divided into n equal subintervals by the points x0, x1, ... xn 1, xn, where
1 = x0 < x1 < ... < xn 1 < xn = 3. Let x = xi xi 1 for i = 1, 2, ... n.
n

( 6 x i 2 x ) .
x 0

Rewrite as a definite integral and thus evaluate lim


Think
1

i=1

Write
n

Recognise the alternate notation for the


definite integral.

(6 xi 2 x) = 1 6 x 2 dx
x0

Evaluate.

1 6 x 2 dx = 2 x3 1

lim

i =1

= 2 27 2 1
= 54 2
= 52

REMEMBER

1. The fundamental theorem of calculus is



where F(x) is an antiderivative of f(x).
2. The expression

f ( x ) dx = [ F ( x )]ba
= F(b) F(a).

f ( x ) dx is called the definite integral where a and b are the

terminals and represent the upper and lower values of x.


3. Properties of definite integrals
a

(a) f ( x ) dx = 0
a

(b) f ( x ) dx = f ( x )dx + f ( x ) dx, a < c < b

Chapter 9 Integration

445

4.

exerCISe

9e

(c)

a k f ( x) dx = k a

(d)

a [ f ( x) + g( x)] dx = a

(e)

Digital doc

SkillSHEET 9.2
Subtracting
function values

f ( x )dx + g( x ) dx
a

f ( x ) dx

f ( x ) dx = lim

xi 0

f ( xi ) xi
i =1

The fundamental theorem of integral calculus


Evaluate the following definite integrals.

0 x 2 dx

2 x 2 dx

1 (4 x 2 + 2 x 6) dx

(6 2 x + x 2 ) dx

4 ( 2 3 x )3

3 2x3

dx

+ 5x 2
dx
x

8 3x

2 We22

446

f ( x ) dx =

1 We21
eBook plus

f ( x ) dx

1 (3e6 x + 5x) dx

32 sin ( x) dx

2 sin

( 3x ) dx

x
3 cos dx
6

0 2 + sin

( x 3 + 3 x 2 2 x ) dx

4 ( x3 + x 4) dx

0 2( x + 4)4 dx

5
dx
(2 x 7)3

1 5x dx

3 ( x 2 2 x) dx

1 (3x 2 + 2 x 2 ) dx

4 3

1 3(5x 2)4 dx

1 2 x 2 3x

x dx

1
2x 5

dx

dx

Find the exact value of each of the following definite integrals.

e 4 x dx

dx

0 x3 dx

( 4x ) dx

1 x + e 2 dx

2 3 sin (4 x) dx

cos (2 x) dx

e 3 dx
5

2 7 cos 2 dx

1 [ x 2 + 3 6 sin (3x)] dx

maths Quest 12 mathematical methods CaS for the TI-nspire

1 4e 2 x dx

3 (e2 x e 2 x ) dx

0 5 sin

2 8 cos (4 x) dx

( 4x ) dx

2 4

1 x + 3 cos

cos (3 x ) dx

( 2x ) dx

3 If

f ( x ) dx = 6, find the value of

4 MC Given that
a

1 f ( x) dx = 6,

1 [ f ( x) + 1] dx is equal to:
A 16

1 3 f ( x) dx.

B 10

C 11

D 19

E 22

C 5

D 6

E 0

5 f ( x) dx is equal to:
A 6

B 5

5 Evaluate the following.


3

1 t

(t 2 4t ) dt

10

sin x dx
5

4 3

dt

2
0

2 cos (3t ) dt

0 e 4 cos (2 x) dx

1 [3 sin (2 x) e

3x

1 4m 3 dm

3
( p 3) 2

dp

] dx

6 WE23 If (2 x + 3) dx = 4, find k.
0

7 If 3 x 2 dx = 8, find k.
0

8 If

k
1

2
dx = log e (9), find k.
x
x

9 If e 2 dx = 4, find the value of a.


0

10 If cos (2 x) dx =
k

3
, find the value of k given that 0 < k < .
2
4

11 Given that f ( x ) dx = 8, evaluate the following.


2

2 2 f ( x) dx

f ( x ) dx c

[3 f ( x ) 3] dx d

2 [ f ( x) + 2 x] dx

12 WE24 MC The interval [1, 3] is divided into n equal subintervals by the points x0, x1, ... xn 1,
xn, where 1 = x0 < x1 < ... < xn 1 < xn = 3. Let x = xi xi 1 for i = 1, 2, ... n.
n

Then lim

x0

( xi3 x) is equal to:


i =1

x 3 dx b

3
1

3
x4
dx c x 3 D 20 E 26
1
4

13 MC The interval [0, 3] is divided into n equal subintervals by the points x0, x1, ... xn 1, xn,
where 0 = x0 < x1 < ... < xn 1 < xn = 3. Let x = xi xi 1 for i = 1, 2, ... n.
n

( xi 2 x) is equal to:
x0

Then lim

i =1

A 9b

3
x3
dx c x 3 0 D
3

3 x 2 dx

E 27

Chapter 9 Integration

447

9f

Signed areas
When calculating areas between the graph of a function f(x) and the x-axis using the definite
integral

f ( x ) dx, the area is signed; that is, it is positive or negative. If f(x) > 0, the region is

above the x-axis; if f(x) < 0 it is below the axis. We shall now examine these two situations and
look at how we calculate the area of regions that include both.

Region above axis

If f(x) > 0, that is, the region is above the x-axis, then
b

y = f(x)

f ( x ) dx > 0, so the value of the definite integral is positive.


For example, if f(x) > 0, then the area =

f ( x ) dx.
0

Region below axis

If f(x) < 0, that is, the region is below the x-axis, then
b

f ( x ) dx < 0, so the value of the definite integral is negative.


b

For example, if f(x) < 0, then the area = f ( x ) dx

or

f ( x ) dx , as the region is below the x-axis or

y = f(x)
a

b
x

area = f ( x ) dx (reversing the terminals changes the sign).


b

Therefore, for areas below the x-axis, ensure that the area has a positive
value. (Areas cannot be negative.)

Combining regions
For regions that are combinations of areas above and below the
x-axis, each area has to be calculated by separate integrals one
for each area above and one for each area below the x-axis.
For example, from the diagram,
Area = A1 + A2
b

However, f ( x ) dx = A1 A2 , because the areas are signed.

y = f(x)

A1
a

0 A
2

To overcome this difficulty we find the correct area as:


b

Area = f ( x ) dx f ( x ) dx (= A1 A2 = A1 + A2 )
b

or = f ( x ) dx +
c

a f ( x) dx

or = f ( x ) dx + f ( x ) dx
Note: When calculating the area between a curve and the x-axis it is essential that the
x-intercepts are determined and a graph of the curve is sketched over the interval required. The
term |x| means that we should make the value of x positive even if it is negative.
448

Maths Quest 12 Mathematical Methods CAS for the TI-Nspire

Worked Example 25
a Express the shaded area as a definite integral.
bEvaluate the definite integral to find the shaded area, giving your

y = x1

answer as an exact value.

0
Think

Write
4

1
dx
x

Express the area in definite integral notation where


a = 1 and b = 4.

a Area =

Antidifferentiate the integrand.

b Area = log e x
1

Evaluate.

State the solution as an exact answer.

= [loge (4)] [loge (1)]


= loge (4) 0
= loge (4)
The area is loge (4) square units.

Worked Example 26

Calculate the shaded area.

y = x2 4x

Think
1

Express the area in definite integral


notation, with a negative sign in front
of the integral as the region is below the
x-axis.

Antidifferentiate the integrand.

Evaluate.

State the solution.

3 4

Write
3

Area = ( x 2 4 x ) dx
1

= [ 1 x 3 2 x 2 ]13
3

= [( 3 (3)3 2(3)2 ) ( 3 (1)3 2(1)2 )]

= [(9 18) ( 3 2)]


2
= [ 9 ( 1 3 )]

= [ 9 + 1 23 ]

= ( 7 13 )

= 7 13

The area is 7 3 square units.

Chapter 9 Integration

449

Worked examPle 27
y

a Express the shaded area as a definite integral.


b Calculate the area.
c Calculate the area using a CAS calculator.

2
ThInk

y = x3 4x
2

WrITe
0

2 ( x3 4 x ) dx 0 ( x3 4 x ) dx

Express the area above the x-axis as an


integral and the area below the x-axis as an
integral. For the area below the x-axis, take
the negative of the integral from 0 to 2.

a Area =

Antidifferentiate the integrands.

b = 14 x 4 2 x 2 14 x 4 2 x 2
2
0

Evaluate.

Simplify.

State the solution.


On a Calculator page, press:
MENU b
4:Calculus 4
3:Integral 3
Complete the entry line as:

= [0 (4 8)] [(4 8) 0]
= 4 (4)
=8
The area is 8 square units.

( 14 (0)4 2(0)2 ) ( 14 (2)4 2(2)2 )


( 14 (2)4 2(2)2 ) ( 14 (0)4 2(0)2 )

2 ( x3 4 x) dx 0 ( x3 4 x) dx
Press ENTER .
Alternatively, take the absolute value
for the area below the x-axis as
shown.

Write the area with the correct units.

The area is 8 square units.

Worked examPle 28

eBook plus

a Sketch the graph of y = ex 2 showing all intercepts and using exact values

for all key features.


b Find the area between the curve and the x-axis from x = 0 to x = 2.
ThInk
a

450

WrITe

Find the x-intercept by letting y = 0


and solving for x.

a When y = 0, ex 2 = 0

maths Quest 12 mathematical methods CaS for the TI-nspire

ex = 2

Tutorial

int-0567
Worked example 28

loge (ex) = loge (2)


x = loge (2) (or approximately 0.69)
so the x-intercept is loge (2).
When x = 0,
y = e0 2

=12

= 1
so the y-intercept is 1.

Take loge of both sides.

Find the y-intercept by letting


x=0.

Note the vertical translation and


hence sketch the graph showing the
appropriate horizontal asymptote
and intercept.
Shade the region required.

Express the area above the x-axis


as an integral and the area below
the x-axis as an integral. Subtract
the area below the x-axis from the
area above the x-axis.
Antidifferentiate the integrands.

Evaluate.
(Remember: eloge (a) = a)

Simplify.

State the solution.

0
1

y y = ex 2

b Area =

loge2

x
2
y=2

log e ( 2)

loge (2) (e x 2) dx - 0

= [e x 2 x ]2log

(e x 2) dx

log e ( 2)

e ( 2)

[e x 2 x ]0


= [e2 2(2)] [eloge (2) 2 loge (2)]

[[eloge (2) 2 (loge (2)] [e0 2(0)]]

= [e2 4] [2 2 loge (2)] [[2 2 loge (2)]

[1 0]]

= e2 4 2 + 2 loge (2) 2 + 2 loge (2) + 1

= e2 7 + 4 loge (2)
The area is [e2 7 + 4 loge (2)] or approximately
3.162 square units.

REMEMBER

1. If f(x) > 0, area =

f ( x ) dx .

2. If f(x) < 0, area = f ( x ) dx , as the region is below the x-axis or


a

= f ( x ) dx , (reversing the terminals changes the sign).

f ( x ) dx , or

3. Check if the required area lies above and below the x-axis.
4. Area =

f ( x ) dx f ( x ) dx (= A1 A2 = A1 + A2)

or= f ( x ) dx +
c

5. e

log e ( a )

y = f(x)

A1

a f ( x) dx

0 A
2

=a

Chapter 9 Integration

451

Exercise

9f

Signed areas
y

1 Find the area of the triangle at right.


a geometrically
b using integration

y=x

2 Find the area of the triangle at right.


a geometrically
b using integration

3
0

3
y=3x

3 WE25a Express the following shaded areas as definite integrals.


a y

b y
c
y = 2x

y = x2

01
y

y = 3x2

1 0
y

y = ex

x
y=4x

y = x3 9x2 + 20x

1 2
y

h y

y = x3 4x2 4x

y
2

y = 2 sin (2x)

2x

y=e

1
0

1
y

y = cos (3x)

1
0

4 WE25b Evaluate each of the definite integrals in question 3 to find the shaded area. Give your
answer as an exact value.
5 For each of the following, sketch a graph to illustrate the region for which the definite integral
gives the area.

452

0 4 x dx

1 (4 x 2 ) dx

2 loge

( 2x ) dx

1 (6 x) dx

x dx

2
3 sin (2 x ) dx
0

Maths Quest 12 Mathematical Methods CAS for the TI-Nspire

1 x 2 dx

3 2e x dx

6 WE26 Calculate each of the shaded areas below.


y
y
a

b

y=x2

y y = x2 4

1 0

y = 4 2x

2
y

d
2

y = x3

y
1

y = x3 + 2x2 x 2

0 1 x

y = 1 x2

1
2

y
y = sin (x)

x
0

y=

y = ex

e2x

y = 2 cos (2x )

7 MC aThe area between the graph, the x-axis and the lines
x = 2 and x = 1 is equal to:
2

1 f ( x) dx

f ( x ) dx

2 f ( x) dx

2 f ( x) dx

3
y = f(x)

E f ( x ) dx
2
b The area between the graph, the x-axis and the lines x = 2 and x = 3 is equal to:
3

0 f ( x) dx + 0

2 f ( x) dx + 1

2 f ( x) dx 0 f ( x) dx

f ( x) dx

f ( x) dx

2 f ( x) dx
2

f ( x ) dx

8 WE27a Express the following shaded areas as definite integrals which give the correct area.
b

c
a
y
y
y
g(x)
f(x)
0

0 1

0 2

x
h(x)

Chapter 9 Integration

453

5 4 2

f(x)

g(x)

3 2

2 3

9 mC Examine the graph.


y

y = x3 2x2 5x + 6

a The area between the curve and the x-axis from x = 2 and x = 1 is equal to:
1

B 154 sq. units

E 10 sq. units

A 1712 sq. units


D 154 sq. units

C 1712 sq. units

b The area between the curve and the x-axis from x = 1 and x = 3 is equal to:
A 6 2 sq. units
B 2 sq. units
C 513 sq. units
3

D 513 sq. units

E 6 2 sq. units
3

c The area between the curve and the x-axis from x = 2 and x = 3 is equal to:
5

A 1012 sq. units

B 114 sq. units

D 12 sq. units

E 2112 sq. units

C 2212 sq. units

10 We28 Sketch the graph of the curve y = x2 4, showing all intercepts and using exact values
for all key features. Find the area between the curve and the x-axis:
a from x = 0 to x = 2
b from x = 2 to x = 4
c from x = 0 to x = 4.
11 Sketch the graph of the curve y = x3 + x2 2x, showing all intercepts. Find the area between the
curve and the x-axis between the lines:
a x = 2 and x = 0
b x = 0 and x = 1
c x = 2 and x = 1.
12 Sketch the graph of the curve y = 1 + 3 cos (2x) over [0, ]. Find the exact area between the
curve and the x-axis from:

a x = 0 to x =
4
3
b x=
to x = .
4
eBook plus
Digital doc

WorkSHEET 9.2

454

13 Sketch the graph of f (x) = x 1 and find the area between the curve and the x-axis and the
lines x = 2 and x = 3. Give both an exact answer and an approximation to 3 decimal places.
14 Find the exact area between the curve y = 1 , the x-axis and the lines x = 12 and x = 2.
x
15 Find the exact area bounded by the curve g(x) = ex + 2, the x-axis and the lines x = 1 and x = 3.

maths Quest 12 mathematical methods CaS for the TI-nspire

9g

Further areas
Areas bound by a curve and the x-axis
For graphs with two or more x-intercepts, there is an enclosed region
(or regions) between the graph and the x-axis.
The area bound by the graph of f(x) and the x-axis is:

or

f ( x ) dx (negative because the area is below the x-axis)


b

y = f(x)

f ( x ) dx .
y

The area bound by the graph of g(x) and the x-axis is:
b

c g( x) dx a g( x) dx
or

a g( x) dx + c g( x) dx

That is, if the graph has two x-intercepts then one integrand is
y = g(x)
required.
If the graph has three x-intercepts then two integrands are required,
and so on.
Note: Wherever possible it is good practice to use sketch graphs to assist in any problems
involving the calculation of areas under curves.
Worked Example 29
a Sketch the graph of the function g(x) = (3 x)(2 + x).
b Find the area bound by the x-axis and the graph of the function.
Think
a

Write

Determine the type of graph by


looking at the number of brackets
and the sign of the x-terms.

Solve g(x) = 0 to find the


x-intercepts.

Sketch the graph.

Shade the region bound by g(x) and


the x-axis.

a g(x) = (3 x)(2 + x) is an inverted parabola.

For x-intercepts, g(x) = 0


(3 x)(2 + x) = 0
x = 3 and x = 2.
The x-intercepts are 2 and 3.
b

x
3
y = g(x)

2 0

2 (6 + x x 2 ) dx

Express the area as an integral.

Area =

Evaluate.

= [6 x + 12 x 2 13 x 3 ]3 2

= [6(3) + 2 (3)2 3 (3)3]

[6(2) + 2 (2)2 3 (2)3]

Chapter 9 Integration

455

= (18 + 2 9) (12 + 2 + 3 )

= 13 2 (7 3 )

= 13 2 + 7 3

= 20 6

1
1

5
5

The area bound by g(x) and the x-axis is 20 6 square


units.

State the solution.

Finding areas without sketch graphs


When finding areas under curves that involve functions whose graphs are not easily sketched,
the area can be calculated providing the x-intercepts can be determined.
Note: The symbol f(x)is known as the absolute value of f(x), which means that we must make
the f(x) function or value positive whenever it is negative.
For example,

5= 5

3= 3

2
3

=3

As areas cannot be negative, taking the absolute value of the integrands involved in a problem
will ensure that all areas are made positive.
y
For example,
The shaded area at right =

or

f ( x ) dx +
f ( x ) dx +

a f ( x) dx

y = f(x)
a

c 0

a f ( x) dx

Worked Example 30
a Find the x-intercepts of y = sin (2x) over the domain [0, 2].
b Calculate the area between the curve, the x-axis and x = 0 and x = .
Think
a

Write

To find the x-intercepts, let y = 0.

Solve for x over the given domain.

a For x-intercepts, y = 0

sin (2x) = 0

2x = 0, , 2, 3, 4, etc.

456

Pick the x-intercepts that are


between the given end points of the
area.

State the regions for which


it is necessary to calculate
the area.

b x=

x = 0,

, , , 2
2
2

is the only x-intercept between 0 and .


2

Area =

Maths Quest 12 Mathematical Methods CAS for the TI-Nspire

02 sin (2x) dx + sin (2x) dx


2

Evaluate the absolute value of the


integral for each region.

= [ 12 cos (2x )] 2 + [ 12 cos (2x )]

[ 12

cos

( ) ( 12

cos (0))]

+ [ 12 cos (2 ) ( 12 cos ( ))]

= [ 12 ( 12 )] + [ 12 ( 12 )]

= 1 + 1
=1+1
=2

Add the result to give the total area.

State the solution.

The area is 2 square units.

Worked Example 31
a Differentiate loge (x2 1).

x
.
1
x
c Find the area between the graph of 2
, the x-axis, x = 2 and x = 3, giving your answer correct
x 1
to 2 decimal places.
b Hence, find an antiderivative of

x2

Think
a

Write
a Let y = loge (x2 1)

Let y equal the expression to be


differentiated.

Express u as a function of x in order to


apply the chain rule for differentiation.
(Let u equal the function inside the
brackets.)

Find

Write y in terms of u.

Find

dy
.
du

dy 1
=
du u

Find

dy
using the chain rule.
dx

So

dy 1
= 2x
dx u

Let u = x2 1

du
.
dx

du
= 2x
dx

y = loge (u)

dy

dx dx = y + c, express the

Since

relationship in integral notation.


dy
Remove a factor from
so that it
dx
resembles the integral required.

2x

2x
1

x2

x 2 1 dx = loge
2

x2

x2 1 + c

x
dx = log e x 2 1 + c
1

Chapter 9 Integration

457

Divide both sides by the factor in order


to obtain the required integral.

x
dx = 12 log e x 2 1 + c
x2 1

An antiderivative of
c

Find the x-intercepts.


x
(For 2
= 0, the numerator = 0.)
x 1

If the x-intercepts are not between the


terminals of the area, find the area by
evaluating the integrand.

c For x-intercepts,

Area =

State the solution.

x
1
is 2 logx2 1.
1

x
=0
x2 1
x=0

2 x 2 1 dx
3

1
2
= 2 log e x 1 2

= 1 log e (32 1) 1 log e (22 1)


2
2

x2

1
2

log e (8) 12 log e (3)

()
= 12 log e ( 83 )
1
log e 83
2

The area is 12 log e


square units.

( 83 ) or approximately 0.49

REMEMBER

1. For graphs with two or more intercepts, there is an enclosed region (or regions)
between the graph and the x-axis.
2. The number of regions is one less than the number of intercepts.
3. Where possible, sketch graphs to make it easier to calculate the areas under curves.
4. As areas cannot be negative, take the absolute values of the integrals.
5. When graphs are not easily drawn, areas can be calculated by finding the x-intercepts
and determining whether they are within the bounds of the required area.
Exercise

9G

Further areas
In the following exercise give all answers correct to 2 decimal places where appropriate, unless
otherwise stated.
1 WE29 iSketch the graph of each of the following functions.
iiFind the area bound by the x-axis and the graph of each function.
a f(x) = x2 3x
b g(x) = (2 x)(4 + x)
2 Find the area bound by the x-axis and the graph of each of the following functions.
a h(x) = (x + 3)(5 x)
b h(x) = x2 + 5x 6
2
c g(x) = 8 x
d g(x) = x3 4x2
e f(x) = x(x 2)(x 3)
f f(x) = x3 4x2 4x + 16
3
2
g g(x) = x + 3x x 3
h h(x) = (x 1)(x + 2)(x + 5)
3 MC The area bound by the curve with equation y = x2 6x + 8 and the x-axis is equal to:
A 1 13 square units
B 6 23 square units
C 12 square units
D 3 square units

458

E 1 13 square units

Maths Quest 12 Mathematical Methods CAS for the TI-Nspire

4 MC The area between the curve at right, the x-axis and the
lines x = 3 and x = 4 is equal to:
A

3 f ( x) dx
3

3 f ( x) dx 2

f ( x ) dx

D
4

3 f ( x) dx + 2

f ( x) dx

0 2

4
y = f(x)

f ( x) dx f ( x ) dx
3

f ( x) dx

5 MC The area between the curve y = x2 x 6, the x-axis and the lines x = 2 and x = 4 is
equal to:
2
3

A 2 65 square units

C 5 square units

D 2 43 square units

E 4 12 square units

square units

6 For each of the following:


i sketch the graph of the curve over an appropriate domain, clearly labelling any
x-intercepts in the interval required.
ii find the area between the curve, the x-axis and the lines indicated below.
2
a y = 3 3x2, x = 0 and x = 2
b y = , x = 1 and x = 3
x
1
c y = , x = 1 and x = 2
d y = x3 4x, x = 2 and x = 1
x2

e y = e2x, x = 2 and x = 0

g y = 2 sin (x), x =

i y = sin (3x), x =

and x =
6
3

and x =
2
6

f y = ex, x = 0 and x = 2

x
h y = cos , x = and x = 2
2
2
j y = x x, x 0, x = 0 and x = 4

7 WE30 For each of the following functions:


i find the x-intercepts over the given domain
ii calculate the area between the curve, the x-axis and the given lines.
Use sketch graphs to assist your workings.

a y = x 4x 1, x 0, x = 1 and x = 3
b y = sin (x) cos (x), x [0, ], x = 0 and x =
c y = ex e, x = 0 and x = 3
1
d y = x 2 , x 0, x = 12 and x = 2
x
x

e y = e 2 , x = 2 and x = 2
f y = x4 3x2 4, x = 1 and x = 4
g y = (x 2)4, x = 1 and x = 3
8 Find the exact area of the region enclosed by the x-axis, y = e3x and the lines x = 1 and x = 2.

9 Find the exact area of the region enclosed by the x-axis, y = cos (x) and the lines x= and
3
5
x=
.
6
10 Find the area bound by y = (x 1)3, the x-axis and the y-axis.
1
11 a Sketch the graph of y =
showing all asymptotes and intercepts.
( x 3)2
b Find the area under the curve between x = 1 and x = 1.

Chapter 9 Integration

459

12 a Give the equation of the asymptotes for the function f(x) = (x + 2) 3.


b Find the area between the curve, the x-axis, x = 1 and x = 1.
13 Find the area bound by the curve y = 3 e2x, the x-axis, x = 2 and x = 0. (Find the x-intercepts
first.)

14 Find the area bound by the curve y = 4 sin (2x), the x-axis, x =
and x = . (Check the
2
x-intercepts first.)
15 WE31 a Differentiate x loge (x). (x > 0) b Hence, find an antiderivative of loge (x).
c Find the area bound by the graph of loge (x), the x-axis, x = 1 and x = 4 giving exact
answers.
x
16 a Differentiate loge (x2 + 2). b Hence, find an antiderivative of 2
.
x +2
x
c Find the area between
, the x-axis, x = 1 and x = 1.
2 +2
x
17 aFind the area between the graph of y = x2, the x-axis, x = 0 and x = 2.
y
b Use this result to calculate the area between the graph, the y-axis
y = e2x
and the line y=4.

0 2

18 Find the exact area of the shaded region on the graph y = e2x below.
y

y = x2
(2, 4)

19 Find the shaded area below. (Hint: It is easier if you use symmetry.)
y

y = 2 sin (x)

20

9H

a The area of the region bounded by the y-axis, the x-axis, the curve y = 2e-x and the
3
line x = k, where k is a positive real constant, is square units. Find k.
2
b The area of the region bounded by the y-axis, the x-axis, the curve y = sin (2x) and the
line x = k, where k is a positive real constant, is 1 square unit. Find k.

Areas between two curves


We shall now consider the area between two functions, f(x) and g(x), over an interval [a, b]. Our
approach depends on whether the curves intersect or do not intersect over this interval.

If the two curves f(x) and g(x) do not intersect


over the interval [a, b]
Here, we may look at three circumstances: when the region is above the x-axis, when it is below
the x-axis, and when it crosses the x-axis.

460

Maths Quest 12 Mathematical Methods CAS for the TI-Nspire

Region above x-axis


b

The brown shaded area =

f ( x ) dx g( x ) dx

g(x)

a [ f ( x) g( x)] dx.

f(x)

0 a

Note: The lower function is subtracted from the higher function to ensure a positive answer.
y

Region below x-axis


Again, the lower function is subtracted from the higher function to
ensure a positive answer.
Brown shaded area =

b
f(x)

a [ f ( x) g( x)] dx, as f(x) is above g(x)

over the interval [a, b].

a [ f ( x) g( x)] dx

g(x)

f(x)
g(x)

Region crosses x-axis


Shaded area =

Worked Example 32
y

a State the definite integral that describes the shaded area on the

y = 2x + 1

graph at right.

y=x

b Find the area.

Think
a

Write
a f(x) = 2x + 1 and g(x) = x

State the two functions f(x) and g(x).

Subtract the equation of the lower function from the


equation of the upper function and simplify.

f (x) g(x) = 2x + 1 x

=x+1

Write as a definite integral between the given


values of x.

Area =

0 [ f ( x) g( x)]
2

0 ( x + 1) dx

b Area = [ 12 x 2 + x ]20

Antidifferentiate.

Evaluate the integral.

State the area.

The area is 4 square units.

= [ 12 (2)2 + 2] [ 12 (0)2 + 0]
= (2 + 2) (0)
=4

Worked Example 33

a Find the values of x where the functions y = x and y = x2 2 intersect.


b Sketch the graphs on the same axes.

c Hence, find the area bound by the curves.


Think
a

Write/draw

State the two functions.

a y = x and y = x2 2

Chapter 9 Integration

461

Find where the


curves intersect.

Solve for x.

For points of intersection:



x = x2 2
2
x x2=0
(x 2)(x + 1) = 0

x = 2 or x = 1
b For y = x,

b Find the key points of each

y
when x = 0, y = 0
y = x2 2
when x = 2, y = 2
(2, 2)
when x = 1, y = 1
y=x
Line passes through (0, 0), (2, 2)
and (1, 1)
x
0
(1, 1)
For y = x2 2,
when x = 0, y = 2
Hence the y-intercept is 2.
Parabola also passes through (2, 2) and (1, 1).

function and sketch.

c Let f(x) = x and g(x) = x2 2

Define f(x) and g(x).

Write the area as


a definite integral
between the values
of x at the points of
intersection.

Area = [ f ( x ) g( x )] dx

= ( x x 2 + 2) dx

Antidifferentiate.

= [ 12 x 2 13 x 3 + 2 x ]21

Evaluate the integral.

= [ 12 (2)2 13 (2)3 + 2(2)] [ 12 ( 1)2 13 ( 1)3 + 2( 1)]

= (2 83 + 4) ( 12 +

= (3 13 ) (-1 16 )

= 3 13 + 1 16

= 4 12

State the area.

= [ x ( x 2 2)] dx
1

1
3

2)

The area is 4 12 square units.

If the two curves intersect over


the interval [a, b]

Where c1 and c2 are the values of x where f(x) and g(x) intersect
over the interval [a,b], the area is found by considering the
intervals [a, c1], [c1, c2] and [c2, b] separately. For each interval
care must be taken to make sure the integrand is the higher
function. Subtract the lower function.
So the shaded area equals:
c1

c2

a [ g( x) f ( x)] dx + c

a c1 0

g(x)

c2

x
f(x)

[ f ( x ) g( x )] dx + [ g( x ) f ( x )] dx
c2

Therefore, when finding areas between two curves over an interval, it must be determined
whether the curves intersect within that interval. If they do, the area is broken into sub-intervals
as shown above.
As with areas under curves, sketch graphs should be used to assist in finding areas between
curves.

462

Maths Quest 12 Mathematical Methods CAS for the TI-Nspire

If sketch graphs are not used, the absolute value of each integral, for each sub-interval, should
be taken to ensure the correct value is obtained.

Worked examPle 34

eBook plus

a Find the values of x where the graph of the functions f (x) =

4
and
x

Tutorial

int-0568
g(x) = x intersect.
Worked example 34
b Sketch the graphs on the same axes. Shade the region between the two
curves and x = 1 and x = 3.
c Find the exact area between f (x) and g(x) from x = 1 to x = 3 using a CAS calculator.

ThInk
a

WrITe/draW
a f (x) = 4 , g (x) = x

State the two functions.

Let f (x) = g(x) to find the values


of x where the graphs intersect.

For points of intersection, x =

Solve for x.

x2 = 4
4=0
(x 2)(x + 2) = 0
x = 2 and x = 2

4
x

x2

b Sketch f (x) and g(x) on the same axes and

shade the region between the two curves


from x = 1 to x = 3.

f(x) = x4
g(x) = x
01 2 3

State the area as the sum of two


integrals for the two sub-intervals.

On a Calculator page, press:


MENUb
4:Calculus4
3:Integral3
Complete the entry line as:
2

c Area =

1 ( x x
2

) dx + ( x 4x ) dx
3

( 4x x ) dx + ( x 4x ) dx
3

Press ENTER .

Write the area with the correct


units.

4
The area is 4 log e + 1 square units.
3

Chapter 9

Integration

463

rememBer

1. If two curves f (x) and g(x) do not intersect over the interval [a, b] and f (x) > g(x), then
the area enclosed by the two curves and the lines x = a and x = b is found by using the
formula:
b
b
b
x ) g( x)]
x dx
f ( x) dx g( x) dx = [ f ( x)
a

2. If two curves f (x) and g(x) intersect over the interval [a, b], it is necessary to find the
points of intersection and hence find the area of each section, because sometimes
f (x) > g(x) and sometimes g(x) > f (x).
3. If sketch graphs are not used to determine which is the upper curve, then it is necessary
to take the absolute value or positive value of each integral.
exerCISe

9h

areas between two curves


1 We32a State the definite integral that will find the shaded areas on each graph below.
y
y
y
a
b
c
2
y=x

y = 2x

y = 3x

y=x
y=x+1

y = x2

2 x
y = 8 x2

0 1

y=x
01

y = x3

y = 3x

x
1
y = 4 x2

y = ex

x
1
y = 9 x2

y = x2 5

1 0

x
y = 4

y=e

2 We32b Find each of the areas in question 1.

f(x)

3 mC Which one of the following does not equal the shaded area?
5

1 g( x) dx + 5 f ( x) dx

f ( x ) dx g( x ) dx

1 [g( x) f ( x)] dx

5 [ f ( x) g( x)] dx

g(x)

1 g( x) dx 1
5

f ( x ) dx

0 1

5
y

g(x)

4 mC The area bound by the curves f (x), g(x) and the lines
x = 3 and x = 1 at right is equal to:
A

464

[ f ( x ) g( x )] dx

f(x)

3 [ f ( x) + g( x)] dx

maths Quest 12 mathematical methods CaS for the TI-nspire

3 1

C
E

[ g( x ) f ( x )] dx

3 [ f ( x) g( x)] dx

1 [ f ( x) + g( x)] dx
y

5 MC The shaded area at right is equal to:


A
B

g(x)

0 [ f ( x) g( x)] dx
3

f(x)
4

0 [g( x) f ( x)] dx + 3 [ f ( x) g( x)] dx

0 [g( x) f ( x)] dx

0 [ f ( x) g( x)] dx

0 [ f ( x) g( x)] dx + 3 [g( x) f ( x)]

3 4

6 WE33 In each of the following:


i find the values of x where the functions intersect
ii sketch the graphs on the same axes
iii hence, find the area bound by the curves.
a y = 4x and y = x2
by = 2x and y = 3 x2
2
2
c y = x 1 and y = 1 x
dy = x2 4 and y = 4 x2
2
2
e y = (x + 1) and y = 1 x
fy = x and y = x2
7 WE34 iFind the values of x where the functions intersect.
ii Sketch the graphs on the same axes.
iii Find the area between f(x) and g(x) giving an exact answer.
a y = x3 and y = x
b y = 3x2 and y = x3 + 2x
8 Find the area between the pairs of curves below, over the given interval.
a y = x3, y = x2, x [1, 1]
b y = sin (x), y = cos (x), x [0, ]
c y = (x 1)2, y = (x + 1)2, x [1, 1]
d y = x3 5x, y = 6 2x2, x [0, 3]
1
e y = , y = 4x, x [ 14 , 1]
x
f y = ex, y = ex, x [0, 1]

g y = 2 cos (x), y = x , x [0, ]


2
2
x

h y = e , y = e , x [ 2, 1]
9 Find the area between the curve y = ex and the lines y = x, x = 1 and x = 3.
10 Find the area between the curve y = x2 and the lines y = x + 3, x = 1 and x = 3.
2
11 Calculate the area between the curves y = sin (2x) and y = cos (x) from x = 0 to x = .
2
12 Calculate the area between the curves y = 3 sin (2x) and y = sin (2x) from x = 0 to x= .
4
13 Find the exact area bound by the curves y = ex and y=3 2ex.
y
1
14 The graph at right shows the cross-section of a bricked archway.
(All measurements are in metres.)
a Find the x-intercepts of f(x).
b Find the x-intercepts of g(x).
c Find the cross-sectional area of the brickwork.

f(x) = 4 4 x2

x
0
g(x) = 3 13 x2

Chapter 9 Integration

465

15 The diagram below shows the outline of a window frame. If all measurements are in metres,
what is the area of glass which fits into the frame?
y

y = 2 12 2x2
y = 2x2

12

1
2

16 The diagram at right shows the side view of a concrete


bridge. (All measurements are in metres.) Find:
a the x-intercepts of the curve
b the length of the bridge
c the area of the side of the bridge
d the volume of concrete used to build the bridge if the
bridge is 9 metres wide.
17 The cross-section of a road tunnel entrance is shown at right.
(All measurements are in metres.) The shaded area is to be
concreted. Find:
a the exact area, above the entrance, which is to be concreted
b the exact volume of concrete required to build this tunnel if
it is 200 metres long.

9I

5
2

x
y = 4
100

x
f(x) = 5 sin (
30 )

Average value of a function


Consider the function y = f(x). The average value, yav, for the function y = f(x) over the interval
[a, b] is given by:
b
1

yav =
f ( x ) dx.
b a a
b

This can be rearranged to give yav (b a) = f ( x ) dx.


a

y = f(x)

yav
a

Geometrically, the average value of the function is the height, yav, of the rectangle of width
(b a) that has the same area as the area under the graph of y = f(x) for the interval [a, b].
Worked Example 35

Find the average value of f(x) = 2x2 for the interval [1, 4]. Find the value of x that corresponds
to the average value.
Think
1

466

Write the relationship for the average value.

Write

yav =

Maths Quest 12 Mathematical Methods CAS for the TI-Nspire

b
1
f ( x ) dx
b a a

4
1
2 x 2 dx

1
4 1

Identify f(x) = 2x2 and substitute in the values


for a and b.

yav =

Antidifferentiate.

1 2x3
yav =

3 3 1

Evaluate the definite integral and multiply


by 1 .
3

1 2 43 2 13
yav =

3 3 3

1 128 2


3 3
3

1 126

3
3
= 14
The average value is 14.

Solve f(x) = 14 to find x.

2 x2 = 14
x2 = 7
x= 7

x = 7 as x [1, 4].

Choose the value of x that satisfies the given


interval.

Worked Example 36

Find the average value of f(x) = loge (2x) for the interval [2, 4]. Give your answer in exact form.
Think
1

Write the relationship for the


average value and substitute the
values for a and b.

The integral of loge (2x) is


not covered in this course.
The average value can only be
evaluated using a CAS Calculator.
On a Calculator page, press:
MENU b
4:Calculus 4
3:Integral 3
Complete the entry line as:

Write

yav =

4
1
log e (2 x ) dx

2
42

1 4
( ln (2 x)) dx
2 2
Press ENTER .
3

Write the answer.

The average value f(x) = loge (2x) for the interval


[2, 4] is 4 loge (2) 1.

Chapter 9 Integration

467

rememBer

The average value, yav, for the function y = f (x) over the interval [a, b] is given by
yav =

b
1
f ( x ) dx .

ba a

exerCISe

9I

average value of a function


1

We35

Find the average value, in exact form, of the function for the given interval.

a y = x3 x, x [1, 3]


b y = sin (x), x 0,
6

c y = x, x [1, 4]

d y = e3x, x [0, 2]

We36 Find the average value of the function, in exact form, for the given interval.

a y = x loge (x), x [2, 5]


b y = tan (2x), x 0,
6

c y = xex, x [0, 4]

d y = x x + 1, x [1, 7]

3 For the function y = e 2x for x [1, 0], find:


a the average value of the function
b the corresponding x-value in exact form.
4

mC The average value of the function f (x) = loge (2x + 1) over the interval [0, 4] is:

A
5

B loge (9)

1
[ 9 log e (3) 4 ]
4

D 9 loge (3) 4

mC The average value of the function y = sin (2x) over the interval 0,

9J

log e (9)
4

B 2

1
2

is:
4

log e (3) + 4
4

further applications of integration


Differentiation can be applied to rates of change and related rates. If the rate of change of a
function is known, antidifferentiation allows the original function to be found. So integration has
many practical applications.

Worked examPle 37

eBook plus

The rate of change of position, velocity, of a particle travelling in a straight line is


dx

given by
= 40 10 e 0.4 t m/s, t 0, where x is measured in metres and t in
dt
seconds.
a Find the velocity:
i initially
ii after 10 seconds, correct to 2 decimal places.
b Find the time taken to reach a velocity of 35 m/s.
dx
c Sketch the graph of
against t.
dt
d Find the total distance travelled by the particle in the first 10 seconds.
468

maths Quest 12 mathematical methods CaS for the TI-nspire

Tutorial

int-0569
Worked example 37

Think
a

write
1

The initial velocity occurs when


t = 0.
Substitute t = 0 into

ii

dx
.
dt

Answer with the correct units.


dx
Substitute
= 35.
dt

dx

= 40 10 e 0.4 t
dt

= 40 10 e 0.4 0
= 40 10 e 0
= 40 10
= 30
Velocity is initially 30 m/s.
dx

= 40 10 e 0.4 10
dt

= 40 10 e 4
= 39.82

Answer with the correct units.


dx
Substitute t = 10 into .
dt

After 10 seconds, the velocity is 39.82 m/s.


dx

b
= 40 10 e 0.4 t
dt
35 = 40 10 e

Solve for t.

= 10 e

0.4 t

0.4 t

e 0.4 t = 0.5
0.4t = ln (0.5)
ln (0.5)
0.4
= 1.73 s

t=

Time taken is 1.73 seconds.

Answer the question correct to 2 decimal


places.
To graph dx = 40 10 e 0.4 t , on a
dt
Graphs page, complete the function
entry line as:

f1(x) = 40 10 e 0.4 x
Press ENTER .
Note the viewing window settings:
Xmin:0, Xmax:15, Ymin:0 and
Ymax:55

When drawing your graph, label the


axes with the given variables.
Note the horizontal asymptote
dx = 40 is not displayed. The
dt
asymptotic behaviour of the
function, however, is clearly
visible.

Distance travelled = area under the graph


State the distance as a definite integral.

d dx = 40 10 e 0.4 t

dt

x=

10

(40 10 e

0.4 t
)dt

Chapter 9 Integration

469

Antidifferntiate.

Evaluate the integral.

State the distance travelled with


correct units.

10

= 40t + 25e 0.4 t

= (40 10 + 25e 0.4 10 ) (40 0 + 25e

= (400 + 25e 4 ) (25e 0 )


4
= 400 + 25e 25

= 375 + 25e 4
= 375.46

0.4 0

The distance travelled in the first 10 seconds is


375.46 metres.

Worked Example 38

The rate of change of pressure, P atmospheres, of a given mass of gas with respect to its volume,
dP k
, k > 0.
=
dV V 2
a If d P = 5 when V = 10, find k.
dV
b Find the pressure, P, as a function of V given that when P = 10 atmospheres, V = 50 cm3.
c Find the volume when the pressure is 20 atmospheres.
V cm3, is given by

Think
a

Write

dP
Substitute the conditions
= 5 and V = 10
dV
into the given rate.

dP k
=
dV V 2
5

Simplify.

Solve for k.

Write the rate with the value of k found


previously.

Antidifferentiate.

(10)

k
100
k = 500
b dP = 500
2
dV
V
5

P=

500
V

dV

= 500 V

470

Substitute the given conditions P = 10 and


V = 50 to find c.

Write the relationship for P.

Maths Quest 12 Mathematical Methods CAS for the TI-Nspire

500
+c
V
500
P=
+c
V
500
10 =
+c
50
c=0
500
P=
V
=

dV

Substitute P = 20.

Solve for V.

State the answer with correct units.

Exercise

9J

500
V
500
20 =
V
500
V=
20
The volume is 25 cm3.
P=

Further applications of integration


1 If f(x) = (2 x)2 and the y-intercept of f(x) is 4 , find the rule for f(x).
3

dy
2 If
= 1 4 cos (2 x ) and the y-intercept is 2, find the exact value of y when x = .
12
dx
3 The rate of deflection from a horizontal position of a 3-metre diving board when an 80-kg
dy
person is x metres from its fixed end is given by
= 0.03( x + 1)2 + 0.03, where y is the
dx
deflection in metres.
y (Metres)
0
Board

(Metres)
x
Deflection

a What is the deflection when x = 0?


b Determine the equation that measures the deflection.
c Hence, find the maximum deflection.
4 On any day the cost per item for a machine producing n items is given by
dC
= 40 2e 0.01n, where n [0, 200] and C is the cost in dollars.
dn
a Use the rate to find the cost of producing the 100th item.
b Express C as a function of n.
c What is the total cost of producing the first 100 items?
d Find the average cost of production for:
i the first 100 items
ii the second 100 items.
5 WE37 The rate of change of position (velocity) of a racing car travelling down a straight
dx
stretch of road is given by
= t (16 t ), where x is measured in metres and t in seconds.
dt
a Find the velocity when:
i t = 0
ii t = 4.
b Determine:
i when the maximum velocity occurs
ii the maximum velocity.
dx
c Sketch the graph of
against t for 0 t 16.
dt

Chapter 9 Integration

471

d Find the area under the graph between t = 0 and t = 10.


e What does this area represent?

6 WE38 The rate at which water is pumped out of a dam, in L/min, t minutes after the pump is
dV
t
= 5 + cos .
started is
40
dt
a How much water is pumped out in the 40th minute?
b Find the volume of water pumped out at any time, t, after the pump is started.
c How much water is pumped out after 40 minutes?
d Find the average rate at which water is pumped in the first hour.
e How long would it take to fill a tank holding 1600 litres?
7 The rate of flow of water into a hot water system during a 12-hour period is thought to be
dV
t
= 10 + cos , where V is in litres and t is the number of hours after 8 am.
2
dt
dV
a Sketch the graph of
against t.
dt
b Find the length of time for which the rate is above 10.5 L/h.
c Find the volume of water that has flowed into the system between:
i 8 am and 2 pm
ii 3 pm and 8 pm.
8 The roof of a stadium has the shape given by the function
f : [25, 25] R, f(x) = 20 0.024x2.
The stadium is 75 metres long and its cross-section is
shown at right.
a Find the volume of the stadium.
b The stadium is to have several airconditioners
strategically placed around it. Each can service a
volume of 11 250 m3. How many airconditioners are
required?
9 The cross-section of a channel is parabolic. It is 3 metres wide
at the top and 2 metres deep. Find the depth of water, to the
nearest cm, when the channel is half full.
10 For any point P on the curve y = x3, prove that the area under
the curve is one quarter of the area of the rectangle.

472

Maths Quest 12 Mathematical Methods CAS for the TI-Nspire

y (metres)
20
5
25

(metres)
x
25

y = x3
P

11 The arch of a concrete bridge has the shape of a parabola.


It is 6 metres high and 8 metres long.
a Find the rule for the function corresponding to the arch
of the bridge.
b Find the area of the shaded region.
c If the bridge is 10 metres wide, find the volume of
concrete in the bridge.
12 In the figure at right f(x) and g(x) intersect at O and B.
a Show that the coordinate of B is (loge (2), 1).
b Find the exact area of the region bound by f(x)
andg(x).
c Show that the sum of the areas under f(x) and g(x),
from x = 0 to x = loge (2), is equal to the area of the
rectangle OABC.

y (metres)
7
6

4
y

4 5

(metres)
x

f(x) = ex 1
B
g(x) = 2ex + 2

Chapter 9 Integration

473

Summary
Antidifferentiation rules

The relationships between f(x) and f ( x ) dx are:


f(x)

f ( x) dx

ax + c

axn

ax n + 1
+c
n +1

(ax + b)n

(ax + b) n + 1
+c
a( n + 1)

1
x

loge |x| + c

1
ax + b

1
log |ax + b| + c
a

ex

ex + c

ekx

1 kx
e +c
k

sin (ax)
cos (ax)
[ f ( x ) g( x )] dx =

f ( x) dx g( x) dx

g( x) dx = f ( x) + c, where g(x) = f(x) dx

1
cos (ax) + c
a
1
sin (ax) + c
a

kf ( x) dx = k f ( x) dx

f ( x) dx is the indefinite integral

Definite integrals

The fundamental theorem of integral calculus:


b

f ( x ) dx = [ F ( x )]ba = F (b) F (a) where F(x) is an antiderivative of f(x).

a f ( x) dx = a f ( x) dx + c

a [ f ( x) g( x)] dx = a

f ( x ) dx is the definite integral

f ( x ) dx =

a kf ( x) dx = k a

f ( x ) dx

f ( x ) dx, a < c < b


b

f ( x ) dx g( x ) dx
a

f ( x ) dx

Graphs of the antiderivative function

474

For a polynomial function, the graph of f(x) is one degree higher than the graph of f(x).
The x-intercepts of f(x) are the x-coordinates of the turning points of f(x).
When f(x) is above the x-axis, the gradient of f(x) is positive.
When f(x) is below the x-axis, the gradient of f(x) is negative.

Maths Quest 12 Mathematical Methods CAS for the TI-Nspire

Approximating areas under curves

An approximation to the area between a curve and the x-axis can be found by dividing the area into a series
of rectangles that are all the same width. The approximation is found by finding the sum of all the areas of
the rectangles.
For an increasing function, left rectangle approximation actual area right rectangle approximation.
For a decreasing function, left rectangle approximation actual area right rectangle approximation.
Area under curves
b

Area = f ( x ) dx, if f ( x ) > 0 for x [a, b]


a

Area = f ( x ) dx, if f ( x ) < 0, or


a

a f ( x)dx , for x [a, b]


y

y = f(x)

y = f(x)
a

b
x

Area =

x
y

f ( x ) dx f ( x ) dx

y = f(x)

= f ( x ) dx +
c

a f ( x) dx , if

f ( x ) > 0 for x [c, b]

and f(x) < 0 for x [a, c]

A1
0 A
2

Area between curves


y

Area = [ f ( x ) g( x )] dx, if f ( x ) > g( x ) for x [a, b]

f(x)

g(x)
0 a

Area = [ g( x ) f ( x )] dx + [ f ( x ) g( x )] dx,

g(x)
f(x)

if g(x) > f(x) for x [a, c]


and f(x) > g(x) for x [c, b]
a 0

Limiting value of a sum

f ( xi ) xi
x0

f ( x )dx = lim

i =1

Average value of a function

yav =

1
ba

f ( x )dx

Chapter 9 Integration

475

chapter review
x

Short answer

1 Find the equation of the curve f(x) if it passes


3x3 2 x 2
through (1, 3) and f ( x ) =
.
x
dy
x
= cos + k, where
2 A particular curve has
4
dx
k is a constant, and it has a stationary point (2, 1).
Find:
a the value of k
b the equation of the curve
c the value of y when x = 6.
dy
3 If y = sin (x2 + 2x), find
and hence
dx
antidifferentiate (x + 1) cos (x2 + 2x).

12 The graph of f: R R, f ( x ) = e 2 + 1 is shown. The


normal to the graph of f where it crosses the y-axis
is also shown.
y

a Find the equation of the normal to the graph of


f where it crosses the y-axis.
b Find the exact area of the shaded region.

4 A curve has a gradient function f(x) = 2ex + k. It


has a stationary point at (0, 3). Find:
a the value of k
b the equation of the curve f(x).

Exam tip
a Students often had difficulty correctly finding
the y-intercept; (0, 1) was a common answer,
leading to the normal being y = 2x + 1. Quite a
few students differentiated but then did not find
the value of the derivative at x = 0 and simply
substituted the expression for the derivative into
the equation of the normal, which led to incorrect
attempts at algebraic manipulation and nonlinear
normals. Some students found a tangent instead.
b Most students correctly set up an area expression
but some were unable to proceed due to their
normal not being a linear expression. Many
had the correct terminals, although 2 instead of
1 was a popular incorrect value. Arithmetic errors
appeared frequently and subtraction mistakes in
the integrand were also common. Some students
also lost 1 from the equation of the curve. The
use of dx was surprisingly good. A few students
used the area under the curve minus the triangle
with some success.

5 Calculate the exact area between the curve


y = ex 4 and the lines y = x, x = 1 and x = 2.
(Hint: y = ex 4 and y = x do not intersect between
x = 1 and x = 2.)
6 Evaluate each of the following definite integrals.
0
9
a
dx
1 (2 x + 3) 4
b

2
3

cos (2 x ) dx

7 Given that

0 (4 x 5) dx = 2, find two

possible values for k.


1
.
x2
b Find the exact area between the graph of f(x),
the x-axis and the lines x = 3 and x = 6.

8 a Sketch the graph of the function f ( x ) =

9 Find the area bound by the x-axis and the curve


g(x) = (4 x)(6 + x).
10 Calculate the area between the curve y = 2 cos (x)

and the lines y = x, x = 0 and x = .


2
11 Use the method of left rectangles to approximate
the area under the curve y = x2 + 1, from x = 1 to
x = 4, using interval widths of 1 unit.
476

[Assessment report 1 2007]

[ VCAA 2007]

Multiple choice

1 The antiderivative of 4 x 3
A x4 loge (1 x) + c
B x4 + loge (1 x) + c
c 16x4 loge (1 x) + c
D 16x4 + loge (1 x) + c
1
+c
E x 4
(1 x )2

Maths Quest 12 Mathematical Methods CAS for the TI-Nspire

1
is:
1 x

2 The indefinite integral (5 x 4)4 dx is equal to:


A 25(5x 4)5 + c

B 5(5x 4)5 + c

C (5x 4)5 + c

1
(5 x 4) 5
25

1
(5 x 4) 5
5

+c

2
9

8(3x

(3x + 4)

3
3x

1
2

3x

(3x + 4)

+ 4)

4 The antiderivative of 6e

A 2e 3x + c

C 18e 3x + c
E

2
9

is:

B 3e 3x + c

D 2e 3x +1 + c

+c

x
5 The indefinite integral cos 3 sin ( 3 x ) dx

3
is equal to:
x
A sin
+ cos (3x) + c
3

()
x
B sin ( ) + 3 cos (3x) + c
3
x
C 3 sin ( ) + cos (3x) + c
3
x
D 3 sin ( ) + cos (3x) + c
3
x
E 3 sin ( ) cos (3x) + c
3
1
3

6 An antiderivative of x3 + sin (4x) + e4x is:


A 4[x4 cos (4x) + e4x]
x 4 + cos (4 x ) + 14 e 4 x

1
4

1 4
x
4

1
4

[x4 cos (x) + e4x]

1
4

[x4

4 cos (4x) + 14 e4x

3
3e x + 3 x

+3

+c

+3x

+ 1)e x + 3 x

is:

is 3( x 2 + 1)e x
(x2
B
D

1
3( x 2 + 1)
1 x3 +3 x
e
3

+c

+c

E undefined

x
2
10 If the derivative of loge (5 x2) is
then the
5 x2
x
antiderivative of
is:
5 x2
A 12 loge (5 x2) + c
B 2 loge (5 x2) + c

C 12 loge (5 x2) + c
D 2 loge (5 x2) + c
E undefined
11 The approximation for the y
(3.5, 9)
area under the graph at
(3,
6)
right from x = 2 to x = 4,
(2.5, 4)
using the lower
(2, 3)
rectangles is:
A 22 sq. units
B 14 sq. units
x
0
3
2
4
C 11 sq. units
D 10 sq. units
E 20 sq. units
12 The area under the graph
y
at right from x = 5 to
(5, 8)
x = 1 can be
(4, 6)
approximated by the area
(3, 5)
(2, 4)
of the upper rectangles
and is equal to:
A 20 sq. units
x
5
1 0
B 21 sq. units
C 23 sq. units
D 11 12 sq. units
E 10 sq. units
13 The interval [0, 4] is divided into n equal
subintervals by the points x0, x1, . . . xn 1, xn,
where 0 = x0 < x1 < . . . < xn 1 < xn = 4. Let
x = xi xi 1 for i = 1, 2, . . . n.
n

( xi x) is equal to:
x0

Then lim

7 If f (x) has a stationary point at (0, 3) and


f (x) = ex + k, where k is a constant, then f (x) is:
A ex 2x + 2
B ex x + 2
C ex x + 2
x
x
D e +x+2
E e + 2x + 1

8 If the derivative of (x x2)8 is 8(1 2x)(x x2)7


then an antiderivative of 24(1 2x)(x x2)7 is:
A 2(x x2)8
B 3(x x2)8
C 12 (x x2)8
1
2
8
2
8
D 3 (x x )
E 8(x x )

+3x

+c

e4x]

cos (4x) +

then the antiderivative of

C e3 x

+c

3 An antiderivative of 2(3x + 4) 4 is:

A 23 (3x + 4) 3
B 23 (3x + 4) 3 + 5
C

9 If the derivative of e x

i =1

4 x dx

x2
dx
2

C 0

D 4 E 8

exam TIP
The definition of the definite integral
as the limiting value of a sum, and its corresponding
representation in integral form, is a fundamental
property of integral calculus and must be understood.

[Assessment report 1 2003]

[ VCAA 2003]

Chapter 9

Integration

477

Questions 20 to 22 apply to the curve with equation


f(x) = ex 1.

14 The expression (3 x x ) dx is equal to:


0

A 2
B 8
C 2 12
D 20
E 16

20 The graph of f(x) is best represented by:


y
y
A
B
f(x)
f(x)
2

15 The exact value of the definite integral


2

A
e

B 2e4 e 4

4
C e 2e 4
4
4
D e + e

E e4 e 4
3e4

()

f(x)

1 8 The shaded area on the


graph at right is:
A 20 sq. units
B 20 sq. units
C 16 sq. units
D 16 sq. units
E 18 sq. units
19 The area bound by the
curve on the graph at
right and the x-axis is
equal to:
5
20 12
1

5
1012

sq. units

D 10 12 sq. units
7

E 20 12 sq. units

f(x)
0

f(x)
x

21 The area bound by the graph of f(x), the x-axis and


the line x = 2 is equal to:
A e2 1 B e2 2 C e2 + 1
D e2 + 2 E e2 3

y = (x 2)3

4 x

22 The area bound by the graph of f(x), the y-axis and


the line y = e2 1 is equal to:
A e2 5 B e2 3 C e2 + 2
D e2 + 1 E 5 e2
Use the graph below to answer questions 23 and 24.
y

y
0

y = 2x + 3 0

1
y = 1 3x2

y = x2

y = x(x + 2)(x 3)

2 3 The two graphs intersect where x is equal to:


A 1 and 3 B 1 and 3 C 1 and 2
D 1 and 2 E 1 and 3
24 The area bound by the two graphs is equal to:
2
1
A 10 3 sq. units
B 7 3 sq. units
1

C 7 3 sq. units

sq. units

B 21 12 sq. units

1 0

E 3 2
1 7 The shaded area on the
graph at right is equal to:
A 12 sq. units
B 16 sq. units
C 10 sq. units
D 4 sq. units
E 8 sq. units

1
y

D 3 3

478

x
16 The exact value of 2 cos
dx is:
0
3
A 3
B 3
C 3 3

2 (4e2 x 2e 2 x ) dx is:

D 11 3 sq. units

E 6 3 sq. units
25 The average value of the function y = cos (2x) over

the interval 0, is:
4
4
2

A
B
c

8
1
d
E 2
2

Maths Quest 12 Mathematical Methods CAS for the TI-Nspire

Extended response

1 From past records it has been found that the cost rate of maintaining a certain car is
dC
= 75t 2 + 50t + 800, where C is the accumulated cost in dollars and t is the time in years since the car
dt
was first used. Find:
a the initial maintenance cost
b C as a function of t
c the total maintenance cost during the first 5 years of use of the car
d the total maintenance cost from 3 to 5 years
e the maintenance cost for the second year.
2 Over a 24-hour period on a particular autumn day, starting
at 12 midnight, the rate of change of the temperature for
Melbourne was approximately
dT 5
t
cos , where T is the temperature
=
12
dt
12
in C and t is the number of hours since midnight
when the temperature was 10C. Find:
a the temperature at any time, t
b whether the temperature reaches 17C at any time
during the day
c the maximum temperature and the time at which it
occurs
d the minimum temperature and the time at which it
occurs
e the temperature at
i 2 am

ii 3 pm
f the time when the temperature first reaches 14.33C.

3 The diagram at right shows part of the curve with equation y = e 2 . Find:
a the coordinate of point A
b the equation of the normal to the curve at point A
c the coordinate of point B
d the coordinate of point C
e the area bound by the curve and the lines AB and BC.
4 a Find the derivative of x loge (x).
b Hence, find an antiderivative of loge (x).
The cross-section of a platform is shown at right. (All measurements
are in metres.)
c Find the height of the platform.
d Find the cross-sectional area of the platform.
e Find the volume of concrete required to build this platform if it is
20 metres long.

y
x

Normal

y = e2

B
A

C
0

y
1
0

e2

e x
1
f(x) = logex

Chapter 9 Integration

479

5 A thick metal pipe is filled with boiling water and is kept boiling. The temperature, T C,
of the metal in the pipe decreases relative to its distance, x cm, from the centre of the pipe.
dT 20
It is known that
and 4 x 8.
=
dx
x
a Find the rate of change of the temperature in the metal on the outside of the pipe.
b Express T as a function of x.
c Find the temperature of the metal, correct to 2 decimal places:
i when x = 6 cm
ii on the outside of the pipe.
eBook plus
Digital doc

Test Yourself
Chapter 9

480

maths Quest 12 mathematical methods CaS for the TI-nspire

eBook plus

aCTIvITIeS

Chapter opener
Digital doc

10 Quick Questions: Warm up with ten quick


questions on integration. (page 416)
9A

Antidifferentiation

Digital doc

SkillSHEET 9.1: Practise substitution and


evaluation. (page 425)
9C

Integration by recognition

Tutorial

We 13 int-0564: Watch a worked example on


performing integration by recognition. (page 430)
Digital doc

WorkSHEET 9.1: Determine functions using


antidifferentiation. (page 435)
9D

Approximating areas enclosed by functions

Interactivity int-0254

Approximating areas enclosed by functions:


Consolidate your understanding of varying
approximations to areas enclosed by functions.
(page 435)
Tutorial

We 18 int-0565: Watch a worked example on the


approximation of the area under a curve. (page 435)
9E

The fundamental theorem of integral


calculus

Tutorial

We 21 int-0566: Watch a worked example on


evaluating definite integrals. (page 442)

9H

Areas between two curves

Tutorial

We 34 int-0568: Watch a worked example on


calculating the area between two curves using CAS.
(page 463)
9J

Further applications of integration

Tutorial

We 37 int-0569: Watch a worked example on


applications of integration. (page 468)
Chapter review
Digital doc

Test Yourself: Take the end-of-chapter test to test


your progress. (page 480)
To access eBookPLUS activities, log on to
www.jacplus.com.au

Digital docs

Definite integrals: Investigate the properties of


definite integrals. (page 442)
SkillSHEET 9.2: Practise subtracting function
values. (page 446)
9F

Signed areas

Tutorial

We 28 int-0567: Watch a worked example on finding


the area bound by an exponential curve above and
below the x-axis. (page 450)
Digital doc

WorkSHEET 9.2: Approximate areas between


curves and calculate areas between curves using
integrals. (page 454)

Chapter 9

Integration

481

EXAM PRACTICE 3


Short answer

Chapters 1 TO 9

40 minutes

1 The graph of f(x): R R, where f(x) = 10xe-x is


graphed below.

y
4

2
1

2
2

a By selecting approximate values from the


graph, determine a left-rectangle approximation
to the area bounded by the graph, the x-axis and
the lines x = 1 and x = 5 by dividing the area into
four strips of equal width.
dy
b If y = (x + 1)ex, find .
dx
c Hence determine an exact value for the shaded
area.
3 marks

2 Write down an antiderivative of:


a y = 4x3 - 2 cos (x) + ex
3
b y = x +
x


4 marks

Multiple choice

8 minutes

Each question is worth 1 mark.


1 Which integral could be evaluated in order to
calculate the area enclosed between the graphs of
y = f(x) and y = g(x) and the vertical lines x = a and
x = b?
f
x=b

4 The area of the region bound by the curve with


x
equation y = a cos , the x-axis and the line
3
2 marks

5 A spa is being filled with water at the rate of


30 litres per minute. The volume (V litres) of water
in the spa is related to its depth (x cm) by
V = 250 loge (x). At what rate (cm per min) is
the depth of the spa increasing?
2 marks

6 The graph of f : R R, f(x) = (x - 1)(5 - x) is


shown. The tangent to the graph of f where it
crosses the x-axis is also shown.

482

a Find the equation of the tangent to the graph of


f where it crosses the x-axis.
b Find the exact area of the shaded region.

x=a

2 marks

with equation x = is 3. Find a.

3 A is (0, 2), a point on the y-axis. P is a point on the


parabola y = x2 such that the length of AP, the line
segment from the A to P is a minimum.
a Find the coordinates of P.
b Find the minimum length of AP.

2 marks

8x

g
x

A ( f ( x) g( x )) dx
b
b

C ( f ( x) g( x )) dx
a

B ( f ( x) + g( x )) dx
a
a

D ( f ( x) + g( x )) dx
b

(g( x) f ( x)) dx
b

2 The average value of the function y = 3 sin (2 x + )


3

over the interval [0, ] is:


3
3 3
A 0
B
4
1
3 3
C 2
D
4
2
E

Maths Quest 12 Mathematical Methods CAS for the TI-Nspire

27
4

3 Usingthefactthat f ( x ) dx = 5, what is the value


1

of (3 f ( x ) 1) dx?
2
16

B 14
E 16

A
D 14

C 12

4 A function with the rule y = f (x) is shown in the


first sketch below.
y

Which of these rules could correspond to the


second sketch?
A y = f (x)
B y = f 1(x)
C y = f (x)
D y = f (x)
1
E y=
f ( x)
5 If f (x) = (2x + 1)2 and f (0) = 1 then the
antiderivative of f (x) will be equal to:
(2 x + 1)3 2
+
3
3
1
C ((2 x + 1)3 + 5)
6
E 4x3 + 4x2 + x + 1

B 4x + 1

D 2(2x + 1) + 1

exTended reSPonSe

30 minutes

The volume of a cylindrical soft drink can is 250 mL. The volume of a cylinder can be
calculated using V = r2h, the area of a circle is given by A = r2 and its perimeter is given
by P = 2 r. Also 1 mL = 1 cm3.
a Create an expression for the total surface area of the can, S, in terms of r and h.

1 mark

b Usethefactthatthevolumeis250mLtoshowthatS can be expressed in terms of r


500
only as S = 2 r 2 +
.
1 mark
r
500
y
c The graphs at right are of y = 2 r2 and y =
for 0 r 10.Usethesegraphs
r
700
600
500
to create a sketch of S = 2 r 2 +
on the same axes.
500
r
1 mark
d Hence obtain an estimate, to 1 decimal place, of the radius corresponding to the
minimum total surface area.

1 mark

400
300
200

100

e Usingcalculusdetermineanexact value for the radius corresponding to the


0 2
minimum surface area.
f i Write down a decimal approximation to 2 decimal places for the radius corresponding to the
minimum total surface area.
ii Confirm using the sign of gradient test that the stationary point located is in fact a minimum.
iii Write down the value of the minimum surface area to the nearest cm2.

8 10 x

1 + 1 = 2 marks

g If the cost of the material for the bottom and top of the can is twice the cost for the sides determine the
radius of the can for minimum total cost.
h The actual radius for a can of soft drink is 3.0 cm. Determine to 2 decimal places the ratio
cost of material for top and bottom
which would be consistent with this can having the minimum
cost of material for the sides
cost of materials.

2 marks

2 marks

eBook plus
Digital doc

Solutions
Exam practice 3

exam practice 3

483

10

10A Probability revision


10B Discrete random variables
10C Measures of centre of discrete random
distributions
10D Measures of variability of discrete
random distributions

Discrete random
variables
areaS oF STuDy

 Random variables, including:


the concept of discrete and continuous
random variables
calculation and interpretation of the expected
value, variance and standard deviation of a
random variable
calculation and interpretation of central
measures (mean, median, mode)
the property that, for many random variables,
approximately 95% of their probability
distribution is within 2 standard deviations of
the mean

 Discrete random variables, including:


specification of probability distributions
for discrete random variables using graphs,
tables and probability functions
interpretation and use of mean (), median,
mode, variance ( 2) and standard deviation
of a discrete random variable
probabilities for specific values of a random
variable and intervals defined in terms of
a random variable, including conditional
probability

eBook plus
Digital doc

10a

Probability revision

10 Quick Questions

To introduce this chapter we shall revise important concepts and skills that were covered in
Mathematical Methods (CAS) Units 1 and 2.

Terminology
The circular spinner at right is divided into 8 equal sectors. When the
2
1
spinner is spun, the possible outcomes are 1, 2, 3, 4, 5, 6, 7, 8. These
outcomes may be listed as the elements of a set. The set of all possible
3
8
outcomes of an experiment is called the sample space (or the universal
set) and is denoted by , and each possible outcome is called a sample
7
4
point. Therefore, spinning the spinner gives = {1, 2, 3, 4, 5, 6, 7, 8}.
A subset of the sample space is known as an event. For the example
5
6
above, if event A is defined as an odd number when the spinner is spun,
then A = {1, 3, 5, 7}. If event B is defined as a number less than 5 when the spinner is spun,
then B = {1, 2, 3, 4}.
The union (symbol ) of the two events A and B above implies a combined event, that
is, either event A or event B or both occurring. Therefore the set A B = {1, 2, 3, 4, 5, 7}.
(Note: Common elements are written only once.)

484

maths Quest 12 mathematical methods CaS for the Ti-nspire

The intersection (symbol ) of the two events A and B above is represented by the common
sample points of the two events. Therefore the set A B = {1, 3}.

Venn diagrams
Venn diagrams involve drawing a rectangle that represents the sample space and a series of
circles that represent subsets of the sample space. They provide a visual representation of the
information at hand and clearly display the relationships between sets.
The Venn diagrams below illustrate an alternative way of presenting information regarding the
circular spinner shown on the previous page.

A
5
7

1
3

2
4

5
7

6
8

B
1
3

2
4

6
8

AB

AB

B
5

1
3

2
4

6
8

Note: Sample points not belonging to either set are placed outside the circles but remain
inside the rectangle.
Venn diagrams can also assist in determining whether or not two sets are equal, that is,
whether they contain the same elements. As the examples below show, the equality of two sets
may not be obvious from the set notation but is often easier to see in a diagram.
A

A B = (A B)

A B = (A B)

Probability
Probability deals with the likelihood or chance of some event occurring. The probability of a
specific event, say A, occurring is defined by the rule:
Pr(A) =

number of favourable outcomes


total number of possible outcomes

Its probability lies within the restricted interval 0 Pr(A) 1. A probability of zero implies
that the event cannot occur, while a probability of 1 implies that the event will most certainly
occur.
The individual probabilities of a particular experiment will sum to a value of 1 and can be
denoted as follows.

p( x) = 1
If Pr(A) is defined as the probability of an event occurring, then its complement, Pr(A), is
defined as the probability of an event not occurring.
Therefore, it can be stated that Pr(A) + Pr(A) = 1
which can be transposed to Pr(A) = 1 Pr(A).

Chapter 10 Discrete random variables

485

Worked Example 1

Two fair dice are rolled simultaneously and the sum of the two numbers appearing uppermost is
recorded as shown below.
(1, 1)

(1, 2)

(1, 3)

(1, 4)

(1, 5)

(1, 6)

(2, 1)

(2, 2)

(2, 3)

(2, 4)

(2, 5)

(2, 6)

(3, 1)

(3, 2)

(3, 3)

(3, 4)

(3, 5)

(3, 6)

(4, 1)

(4, 2)

(4, 3)

(4, 4)

(4, 5)

(4, 6)

(5, 1)

(5, 2)

(5, 3)

(5, 4)

(5, 5)

(5, 6)

(6, 1)

(6, 2)

(6, 3)

(6, 4)

(6, 5)

(6, 6)

Find the probability that the sum will be:


a 6 b 10 c a number less than 5 d at least 9 e an odd number.
Think
a

Write

List all the possible outcomes.

Define the event.

Substitute the values into the


probability rule.

List all the possible outcomes.

Define the event.

Substitute the values into the


probability rule.

Simplify.

List all the possible outcomes.

Define the event.

Substitute the values into the


probability rule.

Simplify.

List all the possible outcomes.

a (5, 1) (4, 2) (3, 3) (2, 4) (1, 5)

Let A = the sum of 6.


number of favourable outcomes
Pr(A) =
total number of possible outcomes
= 5
36
b (6, 4) (5, 5) (4, 6)

Let A = the sum of 10.


number of favourable outcomes
Pr(A) =
total number of possible outcomes
= 3
36
1
=
12
c

(1, 1) (2, 1) (3, 1) (1, 2) (2, 2) (1, 3)


Let A = a number less than 5.
number of favourable outcomes
Pr(A) =
total number of possible outcomes
6
=
36
1
=
6

d (6, 3) (5, 4) (6, 4) (4, 5) (5, 5)

(6, 5) (3, 6) (4, 6) (5, 6) (6, 6)

486

Define the event.

Substitute the values into the


probability rule.

Maths Quest 12 Mathematical Methods CAS for the TI-Nspire

Let A = at least 9.
number of favourable outcomes
Pr(A) =
total number of possible outcomes
10
=
36

Simplify.

List all the possible outcomes.

5
18

e (2, 1) (4, 1) (6, 1) (1, 2) (3, 2) (5, 2)

(2, 3) (4, 3) (6, 3) (1, 4) (3, 4) (5, 4)


(2, 5) (4, 5) (6, 5) (1, 6) (3, 6) (5, 6)
2

Define the event.

Substitute the values into the


probability rule.

Simplify.

Let A = an odd number.


number of favourable outcomes
Pr(A) =
total number of possible outcomes
18
=
36
1
=
2

Worked Example 2

A bag contains 15 marbles comprising 5 black, 3 red, 4 blue, 2 white and 1 green. One marble is
drawn randomly from the bag.
a Determine the probability of each of the coloured marbles being drawn:
i black ii red iii blue
iv white v green.
b Show that the probabilities sum to 1.
c What is the probability that the marble drawn is:
i not black? ii either black or white?
iii neither blue nor green?
Think
a

ii

iii

iv

Write
1

Define the event.

Substitute the values into the


probability rule.

Simplify.

Define the event.

Substitute the values into the


probability rule.

Simplify.

Define the event.

Substitute the values into the


probability rule.

Define the event.

i Let B = a black marble.

number of favourable outcomes

Pr(B) =

total number of possible outcomes


5
=
15
1
=
3

i i Let R = a red marble.

number of favourable outcomes

Pr(R) =

total number of possible outcomes


3
=
15
1
=
5
i i i Let Bl = a blue marble.
Pr(Bl) =

number of favourable outcomes

total number of possible outcomes


4
=
15
i v Let W = a white marble.

Chapter 10 Discrete random variables

487

Substitute the values into the


probability rule.

Define the event.

Substitute the values into the


probability rule.

Add each of the probabilities.

number of favourable outcomes

Pr(W) =

total number of possible outcomes


2
=
15

v Let G = a green marble.

number of favourable outcomes

Pr(G) =

total number of possible outcomes


1
=
15

b Sum of probabilities = 1 +
3

1
5

+ 15 + 15 + 15

=1
c

ii

iii

i Pr(B) = 1 Pr(B)

Write the appropriate rule:


Pr(A) = 1 Pr(A).

Substitute the known values into


the rule.

=13

Evaluate.

Add each of the probabilities


together.

Substitute the known values into


the rule.

= 3 + 15

Evaluate.

= 15

Write the appropriate rule.

Substitute the known values into


the rule.

= 1

Evaluate.

= 1 15

2
3

i i Pr(B W) = Pr(B) + Pr(W)


1

i i i Pr(Bl G) = 1 [Pr(Bl) + Pr(G)]

4
15

+ 15

10

= 15
4

Simplify.

=3

The addition rule of probability


The addition rule of probability states that Pr(A B) = Pr(A) + Pr(B) Pr(A B).

Worked Example 3

a If Pr(A) = 0.4, Pr(B) = 0.7 and Pr(A B) = 0.2, find Pr(A B).
b If Pr(A) = 0.6, Pr(B) = 0.8 and Pr(A B) = 0.9, find Pr(A B).
Think
a

488

Write
a Pr(A B) = Pr(A) + Pr(B) Pr(A B)

Write the addition rule.

Substitute the known values into the rule.

= 0.4 + 0.7 0.2

Evaluate.

= 1.1 0.2
= 0.9

Maths Quest 12 Mathematical Methods CAS for the TI-Nspire

b Pr(A B) = Pr(A) + Pr(B) Pr(A B)

Write the addition rule.

Substitute the known values into the rule.

Transpose the equation to make Pr(A B)


the subject.

Evaluate.

0.9 = 0.6 + 0.8 Pr(A B)


0.9 = 1.4 Pr(A B)
Pr(A B) = 1.4 0.9
= 0.5

Venn diagrams may also be used to display the probabilities rather than just the outcomes, as
shown in the diagram below.

Pr
Pr (AB) (AB) Pr (AB)

Note: Pr(A B) is represented by the shaded section.


The probabilities given and calculated in worked example 3(a) and 3(b) can be displayed as
follows.

B
0.2

0.2

B
0.1

0.5

0.5

0.3
0.1

0.1

Worked example 3(a)

Worked example 3(b)

Mutually exclusive events


If two or more events cannot occur simultaneously, they are said to be mutually exclusive
or disjoint; that is, they have nothing in common. In set notation this may be expressed
as Pr(A B) = { } or Pr(A B) = 0.
Therefore for mutually exclusive events the addition rule becomes:
Pr(A B) = Pr(A) + Pr(B)

Independent events
Two events A and B are independent if one event does not influence the other event from
occurring. The mathematical definition of independence is given by:
Pr(A B) = Pr(A) Pr(B)

Worked Example 4

Two fair dice are rolled with S representing the event of obtaining a number less than 4 on the first
die and T the event of obtaining a number greater than 4 on the second die. Find:
a Pr(S)
b Pr(T)
c if events S and T are mutually exclusive
d if events S and T are independent.

Chapter 10 Discrete random variables

489

Think
a

Write

Refer to the dice results recorded in the


question in worked example 1.

Define the event.

Determine the probability.

Simplify.

Define the event.

Let S = obtaining a number less than 4 on the


first die.
18
Pr(S) =
36
1
2

b Let T = obtaining a number greater than 4 on

the first die.

Determine the probability.

Simplify.

Pr(T) =
=

12
36
1
3

c Answer the question with reasoning.

d From the dice results recorded in the question

Answer the question using the dice results


in worked example 1.

Events S and T are not mutually exclusive


since they have common points; that is,
(1,5) (1, 6) (2, 5) (2, 6) (3, 5) (3, 6).
in worked example 1,
Pr(S T) = 6
=

36
1
6

Using the rule Pr(S T) = Pr(S) Pr(T)


1
=13

Check with answer obtained using the


rule.

=1
6
Since both methods give the same answer,
Sand T are independent events.

Worked Example 5

Two fair dice are rolled with U representing the event of obtaining a 5 on the first die and V the
event of the sum of numbers on the two dice exceeding 10. Find:
a Pr(U) b Pr(V) c if events U and V are independent.
Think
a

Write
a

Refer to the dice results recorded in the


question in worked example 1.

Define the event.

Let U = obtaining a 5 on the first die.

Determine the probability.

Pr(U) = 36

Simplify.

Define the event.

= 16
b Let V = the sum of the numbers on the two

dice exceeds 10.

490

Determine the probability.

Simplify.

Maths Quest 12 Mathematical Methods CAS for the TI-Nspire

Pr(V) = 36
1

= 12

Answer the question using the dice results.

Check with answer obtained using the


rule.

c From the dice results, Pr(U V) =

1
36

Using rule Pr(U V) = Pr(U) Pr(V)


1

= 6 12
1

= 72
Since the two methods do not give the same
answer, U and V are not independent events.

Karnaugh maps and probability tables


Karnaugh maps and probability tables summarise all combinations of two events (for example A
and B) and their complements (for example A and B).
B

AB

A B

A B

A B

Pr(A B)

Pr(A B)

Pr(A)

Pr(A B)

Pr(A B)

Pr(A)

Pr(B)

Pr(B)

Karnaugh map

Probability table

Worked Example 6

For the probability table shown, A is the event


Column 1 Column 2 Column 3
not more than 17 years of age and B is the
B
B
event has a learner permit.
a Complete the probability table at right.
Row 1
A
0.12
bWhat do the following probabilities
Row 2
A
0.5
represent?
Row 3
0.63
1
i Pr(AB) ii Pr(AB)
c What is the probability that:
ia person over the age of 17 does not have a learner permit?
ii a person has a learner permit and is older than 17?
iiia person over the age of 17 has a learner permit or a person at or under the age of 17 does not
have a learner permit?
Think
a

Write

Calculate the values of the cells in


row 1, column 3 and row 3, column 1.

a Pr(A) = 1 Pr(A)

= 1 0.5
= 0.5

Enter the values into the appropriate


cells.
Row 1

Row 2

Row 3
3

Calculate the value of the cell in


row 1, column 2.

Pr(B) = 1 Pr(B)
= 1 0.63
= 0.37
Column 1

Column 2

0.12

Column 3
0.5
0.5

0.37

0.63

Pr(A B) = Pr(A) Pr(A B)



= 0.5 0.12

= 0.38

Chapter 10 Discrete random variables

491

Enter the value into the appropriate


cell.
Row 1

Row 2

Row 3
5

Calculate the value of the cell in


row 2, column 1.

Enter the value into the appropriate


cell.

Calculate the value of the cell in


row 2, column 2.

Enter the value into the appropriate


cell.

Explain what Pr(A B) represents in


this example.

State the appropriate probability from


the table.

ii State the appropriate probability from

the table.
iii State the appropriate probabilities

from the table and evaluate.

0.12

0.38

0.5
0.5

0.63

Column 1

Column 2

Column 3

B
0.38

Row 1

0.12

Row 2

0.25

0.5
0.5

0.37

0.63

Column 1

Column 2

Column 3

Row 1

0.12

0.38

0.5

Row 2

0.25

0.25

0.5

0.37

0.63

i Pr(AB) represents the probability of a person

at or under the age of 17 having a learner permit.


In this case the probability of the given event
occurring is 0.12.
i i Pr(AB) represents the probability of a person

in this example.

Pr(AB) = Pr(B) Pr(AB)



= 0.63 0.38

= 0.25

ii Explain what Pr(AB) represents

Column 3

Pr(AB) = Pr(B) Pr(AB)



= 0.37 0.12

= 0.25

Row 3
b

Column 2

0.37

Row 3
7

Column 1

over the age of 17 not having a learner permit.


In this case the probability of the given event
occurring is 0.25.
c

i Pr(AB) = 0.25
i i Pr(AB) = 0.25
i i i Pr(AB) + Pr(AB) = 0.25 + 0.38

= 0.63

Conditional probability
Conditional probability deals with an event which has previously occurred and has an effect on
an event we are interested in. Due to the initial condition (or restriction) imposed, the number of
Pr ( A B)
possible events is reduced. Conditional probability is defined by the rule Pr ( A | B) =
,,
Pr(B)
where Pr(B) 0, and can be transposed to Pr(A B) = Pr(A | B) Pr(B). The latter is called the
multiplication rule. Pr(A | B) is read as the probability of A given B.
492

Maths Quest 12 Mathematical Methods CAS for the TI-Nspire

WorkeD examPle 7

eBook plus

If Pr(A) = 1 , Pr(B) = 1 and Pr(A B) = 1 , find:


5
10
20
a Pr(A B)
b Pr(A | B)
c Pr(B | A)
d if events A and B are mutually exclusive
e if events A and B are independent.
Think
a

int-0570
Worked example 7

WriTe
a Pr(A B) = Pr(A) + Pr(B) Pr(A B)

Write the addition rule.

Substitute the known values into the rule.

= 15 + 101

Evaluate.

5
20

Simplify.

1
4

Write the appropriate rule.

Substitute the known values into the rule.

Evaluate.

Simplify.

Write the appropriate rule.

Substitute the known values into the rule.

Evaluate.

Tutorial

Simplify.

State the answer, showing reasoning.

b Pr(A | B) =

Pr( A B)
Pr( B)
1
20
1
10

1
20
1
20

10
20

1
2

c Pr(B | A) =

1
20

101
101

Pr( B A)
Pr( A)

1
20
1
5

1
20

1
5

1
20

5
1

5
20

1
4

d Events A and B are not mutually exclusive

since they have common events, that is,


Pr(A B) =

Compare the given value with the answer


obtained using the rule.

e Pr(A B) =

1
.
20
1
.
20

Using rule Pr(A B) = Pr(A) Pr(B)


= 15 101
=

1
50

Since the two methods do not give the same


answer, A and B are not independent events.
From worked example 7(b) and 7(c) it can be seen that Pr(A | B) Pr(B | A).

Chapter 10

Discrete random variables

493

Tree diagrams
Tree diagrams are a useful tool in solving probability tasks as they display each of the possible
outcomes along with their respective probabilities.

Worked Example 8

Nadia knows that if her car starts, she has an 80%


chance of getting to work on time. However, if her car
doesnt start, her chance of arriving on time is 50%.
If Nadias car starts only 70% of the time, what is
the probability that:
a her car starts and she gets to work on time?
b she arrives at work late?
c she arrives at work on time?
d her car starts, given that she arrives at work on time?

Think
a

Write

Define the events.

a Let C = car starts

Assign probabilities to each event.

Let C = car doesnt start


Let O = Nadia arrives at work on time
Let L = Nadia arrives at work late

70
Pr(C ) = 100

= 107
Pr(C') = 103

80

If car starts, Pr(O) = 100


If car starts, Pr(L) =

4
5
1
5
50

If car doesnt start, Pr(O) = 100


1

=2
1

If car doesnt start, Pr(L) = 2


3

4
5

Draw a tree diagram with each branch


assigned the appropriate probability.

7
10

3
10

1
5

O
1
2

C
1
2

494

Calculate the required probability.

Simplify.

Maths Quest 12 Mathematical Methods CAS for the TI-Nspire

L
O

Pr(CO) =

7
10

28
50
14
25

Calculate the required probability.

Calculate the required probability.

b Pr(L) = Pr(CL) + Pr(CL)


7
10
7
50

14
100

29
100

1 +
+

5
3
20

3
10

15
100

c Pr(O) = Pr(CO) + Pr(CO)

7
4+ 3
10
5
10
28
3
+
50
20
56
+ 15
100
100
71
100

d Pr(C|O) =

Write the appropriate rule.


Note: Given implies conditional
probability.

Substitute the known values into the rule.

Evaluate and simplify.

Pr(CO)
Pr(O)
14
25
71
100
14
71

25 100

= 14 100
25
71
= 56

71

Worked Example 9

A fair coin is tossed three times. Find the probability of obtaining two heads given the first toss
resulted in a tail.
Think
1

Write

Draw a tree diagram and list all of the possible


outcomes.

H
H

H
T

H
T

H
T

H
T

HHH, HHT, HTH, HTT, THH, THT, TTH, TTT


2

Write the appropriate rule.

Calculate the probability of each event.

Pr(2H | tail first toss) =

Pr (2H tail first toss)


Pr ( tail first toss)

Pr(2H tail first toss) = 1


Pr(tail first toss) =

8
4
8
1
2

Chapter 10 Discrete random variables

495

Substitute the known values into the rule.

Evaluate.

Pr(2H | tail first toss) =

1
8
1
2
1

=82
=12
=

Simplify.

8
2
8

=1
4

Combinations
In mathematics, a combination deals with the number of ways items may be selected from a set
of elements where the order is not important. For example, in how many ways can 3 numbers be
selected from the set {1, 2, 3, 4}, taking into account that order is not important?
The following selections can be made:
1, 2, 3 1, 2, 4 2, 3, 4 3, 4, 1
Hence 4 selections could be made.
If order was important, there would be a greater number of possibilities since each of the above
selections could be arranged in 6 ways. For example, the selection (1, 2, 3) could be arranged as:
1, 2, 3 2, 1, 3 2, 3, 1 3, 1, 2 3, 2, 1 1, 3, 2.
A combination is also referred to as a selection or choice, and is defined by the rule nCr.
nC = the number of selections of n different objects taken r at a time
r
n!
=

n
( r )!r !
nC
r

()

may also be expressed as n and is read as n over (above) r.


r

WorkeD examPle 10

eBook plus

A drawer contains 7 T-shirts of which 3 are white and the rest are black.
If 2 T-shirts are randomly selected from the drawer simultaneously, find
the probability that they are:
a both black
b both white
c different colours
d the same colour.
Think
a

496

Tutorial

int-0571
Worked example 10

WriTe

Calculate the number of selections of


taking 2 black T-shirts from a total of 4.

Calculate the number of selections of


taking 2 T-shirts from a total of 7.

Calculate the probability using the rule.

Substitute the known values into the rule.

Simplify.

Calculate the number of selections of


taking 2 white T-shirts from a total of 3.

2 = 6; that is, there are 6 ways of selecting 2


black T-shirts from a total of 4.
4C

2 = 21; that is, there are 21 ways of selecting


2 T-shirts from a total of 7.
7C

Pr(both black) =

2 black T-shirts from 4


2 T-shirts from 7

6
=
21
2
=
7
b

maths Quest 12 mathematical methods CaS for the Ti-nspire

2 = 3; that is, there are 3 ways of selecting


2 white T-shirts from a total of 3.
3C

Calculate the number of selections of


taking 2 T-shirts from a total of 7.

7C = 21; that is, there are 21 ways of selecting


2
2 T-shirts from a total of 7.

Calculate the probability using the rule.

Pr(both white) =

Substitute the known values into the rule.

Simplify.

Calculate the number of selections of


taking 1 black T-shirt from a total of 4.

Calculate the number of selections of


taking 1 white T-shirt from a total of 3.

= 3; that is, there are 3 ways of selecting


1white T-shirt from a total of 3.

Calculate the number of selections of


taking 2 T-shirts from a total of 7.

Calculate the probability using the rule.

Substitute the known values into the rule.

Evaluate.

Simplify.

= 21; that is, there are 21 ways of selecting


2 T-shirts from a total of 7.
1 black 1 white
Pr(different colours) =
2 T-shirts from 7
43

=
21
12

=
21
4

=
7

Calculate the probability using the rule.

2 white T-shirts from 3


2 T-shirts from 7

3
21
1
=
7
=

= 4; that is, there are 4 ways of selecting


1black T-shirt from a total of 4.
4C
1
3C
1
7C
2

d Pr(same colours) = Pr(both black)

+ Pr(both white)

Substitute the known values into the rule.

Evaluate.

=2+1
7

3
7

REMEMBER

1. Outcomes are results of experiments.


2. The set of all possible outcomes of an experiment is called the sample space and is
denoted by , and each possible outcome is called a sample point.
3. A subset of the sample space is known as an event.
4. The union (symbol ) of two events A and B implies a combined event, that is, either
event A or event B or both occurring. Common elements are written only once.
5. The intersection (symbol ) of two events is represented by the common sample
points of the two events.
6. Venn diagrams involve drawing a rectangle that represents the sample space and a series
of circles that represent subsets of the sample space. They provide a visual representation
of the information at hand and clearly display the relationship between sets.
7. The probability of an event occurring is defined by the rule
Pr(A) =

number of favourable outcomes


total number of possible outcomes

Chapter 10 Discrete random variables

497

8. The probability of an event occurring lies within the restricted interval


0 Pr(A) 1
9. The individual probabilities of a particular experiment will sum to 1; that is,

p( x) = 1
10. The addition rule of probability is defined by the rule
Pr(A B) = Pr(A) + Pr(B) Pr(A B)
11. If two events, A and B, are mutually exclusive then Pr(A B) = 0, and therefore the
addition rule becomes Pr(A B) = Pr(A) + Pr(B).
12. If two events, A and B, are independent then Pr(A B) = Pr(A) Pr(B).
13. Karnaugh maps and probability tables summarise all combinations of two events (for
example A and B) and their complements (for example A and B).
14. Conditional probability is defined by the rule Pr(A|B) =

Pr(A B)
, where Pr(B) 0.
Pr ( B)

This can be transposed to Pr(A B) = Pr(A|B) Pr(B).


15. Tree diagrams are useful tools in solving probability tasks because they display each
of the possible outcomes along with their respective probabilities.
16. A combination is defined by nCr; that is, the number of selections of n different
objects taken r at a time.

Exercise

10A

Probability revision
1 WE1 Two fair dice are rolled simultaneously and the sum of the two numbers appearing
uppermost is recorded. Find the probability that the sum will be:
a 3
b 12
c 7
d greater than 4
e at least 7
f an even number
g a prime number.
2 WE2 A bag contains 12 marbles comprising 3 black, 5 red and 4 green. One marble is drawn
randomly from the bag.
a Determine the probability of each of the coloured marbles being drawn:
i black ii red iii green.
b Show that the probabilities sum to 1.
c What is the probability that the marble drawn is:

i not green?
ii either black or red?
iii neither red nor green?
iv either black, red or green?
3 A fair coin is tossed three times. Find the probability of obtaining:
a three heads
b two heads
c one head
d no heads
e at least two heads.
4 A circular spinner is divided into 8 equal sectors and numbered as
shown in the diagram at right. If the spinner is spun once, find the
probability of obtaining:
a a one b a two c a three or a four or a five
d a one or a two.

498

Maths Quest 12 Mathematical Methods CAS for the TI-Nspire

2
4

5 WE3a If Pr(A) = 0.3, Pr(B) = 0.6 and Pr(A B) = 0.2, find Pr(A B).

6 WE3b If Pr(A) = 0.5, Pr(B) = 0.4 and Pr(A B) = 0.8, find Pr(A B).
7 If Pr(A) = 4 Pr(B), Pr(A B) = 0.8 and Pr(A B) = 0.2, find: a Pr(B) b Pr(A).
8 Of the 200 students studying VCE at Merlynston Secondary College, 80 study Maths Methods,
while there are 65 Physics students. If there are 85 students who dont take either Maths
Methods or Physics, find the probability that a randomly selected student:
a studies Maths Methods
b studies Physics
c studies neither Maths Methods nor Physics
d studies Maths Methods and Physics
e studies Physics, given that the student studies Maths Methods.
9 WE4,5 Two fair dice are rolled, with F representing the event of obtaining a number greater
than 4 on the first die and G the event of obtaining an even number on the second. Find:
a Pr(F) b Pr(G)
c if events F and G are mutually exclusive
d if events F and G are independent.
10 For two events P and Q, Pr(P) = 0.72, Pr(Q) = 0.25 and Pr(P Q) = 0.91. Are P and Q
mutually exclusive events?
11 For two events X and Y, Pr(X) = 0.4, Pr(Y) = 0.5 and Pr(X Y) = 0.2. Are X and Y independent
events?

12 WE6 For the probability table shown, A is the event is unfit and B is the event is a smoker.
a Complete the probability table at right.

Row 1

Row 2

Column 1

Column 2

0.22
0.60

Row 3
b
c



0.68
1

What do the following probabilities represent? i Pr(A B) ii Pr(A B)


What is the probability that:
i a person is unfit and a non-smoker?
ii a person is a smoker and fit?
iii a person is unfit and a smoker or is unfit and a non-smoker?
iv a person is a non-smoker?

If Pr(A) = 12 , Pr(B) = 13 and Pr(A B) =


a Pr(A B)
b Pr(A | B)
d if events A and B are mutually exclusive
e if events A and B are independent.

13 WE7

14

Column 3

1
6

find:

If Pr(A) = 0.4, Pr(B) = 0.5 and Pr(A B) = 0.2 find:


a Pr(A B)
b Pr(A | B)

c Pr(B | A)

c Pr(B | A).

15 WE8 A recent study has shown that 60% of people who dont wear glasses get regular
headaches, while only 30% of people who wear glasses are headache sufferers. If 35% of
people wear glasses, find the probability that a randomly selected person:
a wears glasses and gets headaches
b does not wear glasses and suffers from headaches
c suffers from headaches
d wears glasses, given that the person suffers from headaches.

Chapter 10 Discrete random variables

499

16

Jemma knows that if her alarm goes off, she


has a 90% chance of getting to school on time.
However, if the alarm does not ring, her chance of
arriving on time is only 40%. If Jemmas alarm clock
works only 60% of the time, what is the probability that:
a she gets to school on time
b she arrives late to school
c her alarm rang, given that she arrived on time?

17

A bag contains 5 red marbles and 3 green marbles. A marble is selected at random, its
colour is observed and it is then replaced. A second selection is then made. Find the probability
that the two marbles chosen were:
a both red
b both green
c different colours
d the same colour.

18 MC Two fair dice are rolled. The probability of the numbers showing uppermost on both
dice being the same is:
A

1

36

1

18

C 1
6

D 3

1
2

19 MC If Pr(S) = 0.2, Pr(T) = 0.5 and Pr(S T) = 0.6, which one of the following is not true?
A Pr(S T) = 0.1 B Pr(S | T) = 0.2 C Pr(T | S) = 0.5
D S and T are mutually exclusive. E S and T are independent.
20 MC The probability of picking a red picture card from a standard pack of playing
cards is:
A 1
B 3
C 2
D 3
E 1
2

13

13

26

26

21 MC If Pr(M) = 0.3, Pr(N) = 0.4 and Pr(M | N) = 0.5 then Pr(M N) is equal to:
A 0.15
B 0.2
C 0.6
D 0.75
E 0.8
22 WE9 A fair coin is tossed three times. Find the probability of obtaining three tails, given that
the first toss resulted in a tail.
23

If Pr(A) = 0.6, Pr(B) = 0.5 and Pr(A B) = 0.36, find:


a Pr(A)
b Pr(B)
d Pr(A B)
e Pr(A B)
g Pr(B | A)
h Pr(A | B)

c Pr(A B)
f Pr(A | B)
i Pr(B | A).

24 WE10 A drawer contains 6 T-shirts, of which 2 are white and the rest are black. If 2 T-shirts
are randomly selected from the drawer simultaneously, find the probability that they are:
a both black
b both white
c different colours d the same colour.
25 A box contains one dozen chocolates, of which 4 are strawberry creams, 3 are orange creams
and 5 are peppermint creams. Two chocolates are selected at random. Find the probability that
they are both the same type if:
a the first chocolate is replaced before the second is drawn
b the first chocolate is not replaced before the second is drawn.
26 MC A fair die has its 4-spot changed to a 5-spot and its 2-spot changed to a 3-spot. The
probability of getting an even number when the altered die is rolled is:
A 1
B 1
C 1
D 2
E 5
6

25

25

20

27 MC A box contains 3 red balls and 2 green balls. Two balls are chosen simultaneously. The
probability that they are the same colour is:
A 8
B 2
C 13
D 3
E 13
28 A bag contains 5 red cubes and 3 black cubes. Three cubes are chosen at random. Find the
probability of at least 2 reds being chosen, given that the first cube was red:
a if the cubes are replaced after each draw
b if the cubes are not replaced after each draw.
500

Maths Quest 12 Mathematical Methods CAS for the TI-Nspire

29 Jo-anne knows that her chance of winning each tennis match she plays is 0.8. A knockout
tournament requires players to win five matches to win the championship. What is the
probability that Jo-anne:
a wins the tournament? Give your answer to 4 decimal places.
b wins the tournament given that she wins her first three matches?
30

In a particular suburb the chances of a woman owning her own home is 0.4, while the
probability of a woman owning her own home and being employed is 0.2. Find the probability
that a woman who owns her own home is also employed.

31

The probability of Vanessas car starting on a cold morning is 0.6, while on a normal
morning the chance of it starting is 0.9. The probability of any morning being a cold one is 0.3.
If Vanessas car starts tomorrow morning, find the probability that the morning is cold.

32 The Roosters know that they will win 80% of their home matches and 40% of their away
matches. This seasons fixture has the Roosters playing 55% of their games at home. Given
that the Roosters won their last game, what was the probability that it was played at home?
33 Tatiana is trying out for a place on the high jump team. In order to qualify she must clear three of the
four heights. She knows that she has a 70% chance of clearing the first height and a 65% chance of
clearing any subsequent height. What is the probability, to 4 decimal places, that Tatiana:
a clears the first, third and fourth heights only?
b clears three heights?
c clears three heights, given she did not clear the first height?

10B

Discrete random variables


A random variable is one whose value cannot be predicted but is determined by the outcome
of an experiment. For example, two dice are rolled simultaneously a number of times. The sum
of the numbers appearing uppermost is recorded. The possible outcomes we could expect are
{2, 3,4, 5, 6, 7, 8, 9, 10, 11, 12}. Since the possible outcomes may vary each time the dice are
rolled, the sum of the numbers appearing uppermost is a random variable.
Random variables are expressed as capital letters, usually from the end of the alphabet
(for example, X, Y, Z) and the value they can take on is represented by lowercase letters (for
example, x, y, z respectively).
The above situation illustrates an example of a discrete random variable since the possible
outcomes were able to be counted. Discrete random variables generally deal with number or size.
A random variable that can take on any value is defined as a continuous random variable.
Continuous random variables generally deal with quantities that can be measured, such as mass,
height or time.

Worked Example 11

Which of the following represent discrete random variables?


a The number of goals scored at a football match
b The height of students in a Maths Methods class
c Shoe sizes
d The number of girls in a five-child family
e The time taken, in minutes, to run a distance of 10 kilometres
Think

Write

Determine whether the variable can be countedor needs to be measured.


a Goals can be counted.
b Height must be measured.

a Discrete
b Continuous

Chapter 10 Discrete random variables

501

c The number of shoe sizes can be counted.

c Discrete

d The number of girls can be counted.

d Discrete

e Time must be measured.

e Continuous

Discrete probability distributions


When dealing with random variables, the probabilities associated with them are often required.
Worked Example 12

Let X represent the number of tails obtained in three tosses. Draw up a table that displays the values
the discrete random variable can assume and the corresponding probabilities.
Think
1

Write

Draw a tree diagram and list all of the


possible outcomes.

H
H

H
T

H
T

H
T

H
T

HHH, HHT, HTH, HTT, THH, THT, TTH, TTT


2
3

Draw a table with two columns: one labelled


number of tails, the other probability.

Number of tails (x)

Probability Pr(x)

1
8
3
8
3
8
1
8

Enter the information into the table.

1
2
3

The table above displays the probability distribution of the total number of tails obtained in
three tosses of a fair coin. Since the variable in this case is discrete, the table displays a discrete
probability distribution.
In worked example 12, X denoted the random variable and x the value that the random
variable could take. Thus the probability can be denoted by p(x) or Pr (X = x). Hence the table in
worked example 12 could be presented as shown below.
x

Pr(X = x)

1
8

3
8

3
8

1
8

Close inspection of this table shows important characteristics that satisfy all discrete probability
distributions.
1. Each probability lies in a restricted interval 0 Pr(X = x) 1.
2. The probabilities of a particular experiment sum to 1, that is,

Pr (x = x) = 1
If these two characteristics are not satisfied, then there is no discrete probability
distribution.

502

Maths Quest 12 Mathematical Methods CAS for the TI-Nspire

Worked Example 13

Draw a probability distribution graph of the outcomes in worked example 12.


Think
1

Write

Draw a set of axes in the first quadrant only.


Label the horizontal axis x and the vertical
axis Pr(X = x).

Pr(X = x)
3
8

Mark graduations evenly along the horizontal


and vertical axes, and label with appropriate
values.

2
8
1
8

Draw a straight line from each x-value to its


corresponding probability.

Note: The probability distribution graph may also be drawn as follows.


Pr(X = x)

Pr(X = x)
3
8

3
8

2
8

2
8

1
8

1
8

A column graph

A dot graph

Worked Example 14

Which of the following tables represent a discrete probability distribution?


a

b
x
0
1
2
3
x
0
2
4
c

Pr(X = x)

0.2

0.5

0.2

0.1

Pr(X = x)

0.2

0.1

0.3

0.3

Think
a

Pr(X = x)

0.5

0.3

0.1

0.1

0.2

0.3

0.5

0.4

Pr(X = x)

Write

Check whether each of the probabilities lie


within the restricted interval
0 Pr(X = x) 1.

a All probabilities lie between 0 and 1

inclusive.

Check that the probabilities sum to 1.

0.2 + 0.5 + 0.2 + 0.1 = 1

Answer the question.

Yes, this is a discrete probability distribution


since both requirements have been met.

Check whether each of the probabilities lie


within the restricted interval
0 Pr(X = x) 1.

b All probabilities lie between 0 and 1

inclusive.

Check that the probabilities sum to 1.

0.5 + 0.3 + 0.1 + 0.1 = 1

Answer the question.

Yes, this is a discrete probability distribution


since both requirements have been met.

Chapter 10 Discrete random variables

503

Check whether each of the probabilities


lie within the restricted interval
0 Pr(X = x) 1.

Check that the probabilities sum to 1.

0.2 + 0.1 + 0.3 + 0.3 1 (totals to 0.9)

Answer the question.

No, this is not a discrete probability


distribution since both requirements have
not been met.

Check whether each of the probabilities


lie within the restricted interval
0 Pr(X = x) 1.

Check that the probabilities sum to 1.

0.2

Answer the question.

No, this is not a discrete probability


distribution since both requirements have
not been met.

All probabilities lie between 0 and 1


inclusive.

The first probability is a negative value, so not


all probabilities lie between 0 and 1 inclusive.
+ 0.3 + 0.5 + 0.4 = 1

WorkeD examPle 15

Find the value of k for each of the following discrete probability distributions.
a
x
1
3
5
7
Pr(X = x)
b

0.2

0.2

0.3

0.1

Pr(X = x)

5k

6k

4k

3k

2k

Think
a

WriTe

Add up each of the given probabilities.


They should sum to 1.

Simplify.

Solve to find k.

Add up each of the given probabilities.


They should sum to 1.

Simplify.

Solve to find k.

Pr( X = x) = 1
0.2 + k + 0.2 + 0.3 + 0.1 = 1
0.8 + k = 1
k = 1 0.8
= 0.2

5k + 6k + 4k + 3k + 2k = 1

20k = 1
k=

WorkeD examPle 16

eBook plus
1
42 (5x

a Show that the function p(x) =


+ 3), where x = 0, 1, 2, 3 is a
probability function.
1
b Show that the function p(x) = 100 x2 (6 x), where x = 2, 3, 4, 5 is a
probability function.
Think
a

504

1
20

Tutorial

int-0572
Worked example 16

WriTe

Substitute each of the x-values into the


equation and obtain the corresponding
probability.

maths Quest 12 mathematical methods CaS for the Ti-nspire

When x = 0, p(x) =
=

3
42
1
14

When x = 1, p(x) =

Simplify where possible.

When x = 2, p(x) =
When x = 3, p(x) =
=

8
42
4
21
13
42
18
42
3
7

Check whether each of the probabilities lie


within the restricted interval 0 Pr(X = x) 1.

All probabilities lie between 0 and 1


inclusive.

Check whether the probabilities sum to 1.

1
4
+
14 21

State whether the function is a probability


function.

Yes, this is a probability function since both


requirements have been met.

Substitute each of the x-values into the


equation and obtain the corresponding
probability.

13

+ 42 + 7 = 1

b When x = 2, p(x) = 16
100

= 25
27

When x = 3, p(x) = 100

Simplify where possible.

32

When x = 4, p(x) = 100


= 25
25

When x = 5, p(x) = 100


=4

Check whether each of the probabilities lie


within the restricted interval 0 Pr(X = x) 1.

All probabilities lie between 0 and 1


inclusive.

Check whether the probabilities sum to 1.

4
25

State whether the function is a probability


function.

Yes, this is a probability function since both


requirements have been met.

27

+ 100 + 25 + 4 = 1

Worked Example 17

Three balls are selected from a box containing 6 blue balls and 4 yellow balls. If the ball chosen after
each selection is replaced before the next selection, find:
a the probability distribution for the number of blue balls drawn:
i 0 blue balls ii 1 blue ball iii 2 blue balls iv 3 blue balls
b the probability that 3 blue balls are chosen, given that at least one ball was blue.
Think
a

Write/draw
1

Draw a tree diagram and list all


the possible outcomes with their
respective probabilities.

10

10

10

10

10

Y
4

10

10

10

10

6
10
10
4

10

10

10

Outcomes
B
Y

BBB
BBY

B
Y

BYB
BYY

B
Y

YBB
YBY

B
Y

YYB
YYY

Probability
6

10
6

10
6

10
6

10
4

10
4

10
4

10
4

10

6
6
=

10 10
6
4

10 10
4
6

10 10
4
4

10 10
6
6

10 10
6
4

10 10
4
6

10 10
4
4

10 10

216

1000
= 144

1000
= 144

1000
96
=
1000
= 144

1000
96
=
1000
96
=
1000
64
=
1000

Chapter 10 Discrete random variables

505

ii

iii

iv

Obtain the probability required.

List the required probabilities


from the tree diagram obtained
in part i .
Note: Three outcomes
correspond to 1 blue ball.

Evaluate and simplify.

List the required probabilities


from the tree diagram obtained
in part i .
Note: Three outcomes
correspond to 2 blue balls.

Evaluate and simplify.

Obtain the probability required.

Place all of the information in


a table.

Pr(0 blue balls) =

i i Pr(1 blue ball) = Pr(BYY) + Pr(YBY)

+ Pr(YYB)

96
1000
288
=
(or 0.288)
1000
= 3

i i i Pr(2 blue balls) = Pr(BBY) + Pr(BYB)

+ Pr(YBB)

144
1000
432
=
(or 0.432)
1000
= 3

i v Pr(3 blue balls) =

x
Pr(X = x)

Check that the probabilities


sum to 1.

Define the rule.

Determine each of the probabilities.

Substitute values into the rule.

Evaluate and simplify.

64
(or 0.064)
1000

216
(or 0.216)
1000

0.064

0.288

0.432

0.216

Pr(X = x) = 0.064 + 0.288 + 0.432 + 0.216


=1
Pr ( X = 3 X > 1)
Pr ( X = 3 | X > 1) =
Pr ( X > 1)
Pr(X = 3 X > 1) = Pr(X = 3) = 0.216
Pr(X > 1) = 0.432 + 0.216
= 0.648
0.216
Pr(X = 3 | X > 1) =
0.648
1
=
3

REMEMBER

1. Discrete random variables generally deal with number or size and are able to be counted.
2. A discrete probability distribution exists only if the following two characteristics are
satisfied.
(a) Each probability lies in a restricted interval 0 Pr(X = x) 1.
(b) The probabilities of a particular experiment sum to 1, that is, Pr(X = x) = 1.

506

Maths Quest 12 Mathematical Methods CAS for the TI-Nspire

exerCiSe

10B

Discrete random variables


1 We11 Which of the following represent discrete random variables?
a The number of people at a tennis match
b The time taken to read this question
c The length of the left arms of students in your class
d The shoe sizes of twenty people
e The weights of babies at a maternity ward
f The number of grains in ten 250-gram packets of rice
g The height of jockeys competing in a certain race
h The number of books in Melbourne libraries

eBook plus
Digital doc

Spreadsheet 100
Probability
distribution

2 We12,13 a If X represents the number of heads obtained in two tosses of a coin, draw
up a table that displays the values that the discrete random variable can assume and the
corresponding probabilities.
b Draw a probability distribution graph of the outcomes in part a.
3 A fair coin is tossed three times and a note is taken of the number of tails.
a List the possible outcomes.
b List the possible values of the random variable X, representing the number of tails
obtained in the three tosses.
c Find the probability distribution of X.
d Find Pr(X 2).
4 Draw graphs for each of the following probability distributions.
a

x
Pr(X = x)

x
Pr(X = x)

x
Pr(X = x)

x
Pr(X = x)

0.05

0.2

0.5

0.2

0.05

10

15

20

0.5

0.3

0.15

0.05

10

0.1

0.2

0.4

0.2

0.1

0.1

0.2

0.3

0.4

5 We14 Which of the following tables represent a discrete probability distribution?


a

x
Pr(X = x)

x
Pr(X = x)

0.2

0.3

0.2

0.2

0.1

0.1

0.1

0.1

0.1

0.1

12

15

Pr(X = x)

0.3

0.2

0.4

0.2

0.1

Pr(X = x)

0.1

0.1

0.4

0.2

0.2

Chapter 10

Discrete random variables

507

6 WE15 Find the value of k for each of the following discrete probability distributions.
a x
Pr(X = x)
b x
Pr(X = x)
c x
Pr(X = x)
d x
Pr(X = x)
7

0.3

0.2

0.2

0.1

10

0.1

0.1

0.1

0.1

2k

3k

4k

0.2

3k

0.3

0.1

Find the value of k for the following discrete probability distribution.


a x
Pr(X = x)

1
3k
13

3
5k 4
13

k2
13

4
k2
13

5
7
13

b Explain why one of the values of k had to be discarded.


8

Two fair dice are rolled simultaneously, and X, the sum of the two numbers appearing
uppermost, is recorded.
a Draw up a table that displays the probability distribution of X, and find:
b Pr(X > 9)
c Pr(X < 6)
d Pr(4 X < 6)
e Pr(3 X 9)
f Pr(X < 12)
g Pr(6 X < 10).

9 A spinner is numbered from 1 to 5, with each number being equally likely to come up. If X is
the random variable representing the number showing on the spinner, find:
b the probability of getting an even number
a the probability distribution of X
c Pr(X > 2).
10 A fair die is rolled and X is the square of the number appearing uppermost.
a Draw up a table that displays the probability distribution of X, and find:
b Pr(X < 30)
c Pr(X > 10).
11

A fair die is altered so that the 1 is changed to a 5. If X is the random variable representing
the number uppermost on the die, find:
a the probability distribution of X
b the probability of a number bigger than 2 appearing uppermost
c Pr(X = 5 | X > 2).

12 WE16a
function.

Show that the function p(x) =

13 WE16b Show that the function p(x) =

1
160

1
90

(8x + 2), where x = 0, 1, 2, 3, 4 is a probability

x2 (x + 2), where x = 1, 2, 3, 4 is a probability function.

14 WE17 Three balls are selected from a box containing 4 red balls and 5 blue balls. If the ball
chosen after each selection is replaced before the next selection, find, correct to 4 decimal places:
a the probability distribution for the number of red balls drawn:
i 0 red balls
ii 1 red ball
iii 2 red balls
iv 3 red balls
b the probability that three reds are chosen, given that at least one ball is red.
508

Maths Quest 12 Mathematical Methods CAS for the TI-Nspire

15

A circular spinner, divided into five equal sectors numbered 1, 2, 3, 4, 5, is spun twice,
and the sum of the numbers the pointer lands on is recorded. The following events are then
defined.
A = an odd number on the first spin
B = an even number on the second spin
C = the sum of the two numbers is odd
D = the sum of the two numbers is at most 7
a List each of the possible outcomes.
b Find:
i Pr(A) ii Pr(B) iii Pr(C) iv Pr(D)
c Find:
i Pr(A | B)ii Pr(B | C)iii Pr(C | D)

16

A biased coin is tossed twice. If the probability of obtaining a head is 3:


5
a find the probability distribution of the number of heads in 2 tosses
b show that the sum of the probabilities is 1.

17

A discrete random variable has the following probability distribution:


x
Pr(X = x)

0.2

0.11

0.15

0.09

0.17

0.13

0.15

Find:
a Pr(X > 3)
d Pr(2 < X < 5)
g {x: Pr(X x) = 0.54}

b Pr(X 4)
e Pr(X < 3 | X < 5)

c Pr(3 X 6)
f {x: Pr(X < x) = 0.46}

18 MC Which one of the following random variables is not discrete?


A The price, in cents, of a loaf of bread at the local supermarket
B The number of runs scored by a batsman in each innings over a season
C The weight of a baby as he grows over a one-year period
D The number of houses sold by a real estate agent each month for a year
E The number of newspapers recycled by a family each month.
19 MC What is the value of k which will make this table a probability distribution table?
x

Pr(X = x)

2k

3k

4k

A 0

B 1

C 0.1

D 1

20 MC Examine the following probability distribution table.


x
Pr(X = x)

E 0.1

16

25

36

0.16

0.21

0.35

0.08

0.2

Pr(X 10) is equal to:


A 0.38
B 0.84

C 0.35

D 0.28

E 0.63

21 MC The following table represents a discrete probability distribution for a


random variable, Y.
x

10

13

Pr(X = x)

4d

5d

2k

The value of d is:


B
A 1
9

1

10

1

11

1

12

1
13

Chapter 10 Discrete random variables

509

22 mC A coin is biased so that the probability of obtaining a head is 3 . If the coin is tossed
7
3 times the probability of obtaining exactly 2 heads is:
27
108
144
135
A
B
C
D
E 64
343

343

343

343

343

23 mC Which of the following is a probability function?


1
A p(x) = 0.1, 0.3, 0.4, 0.2, 0.1, x = 0, 1, 2, 3, 4
B p(x) = 66 (3x + 7), x = 0, 1, 2, 3, 4
1

C p(x) = 40 (5x 1), x = 1, 2, 3, 4


E p(x) =
24

x2
20

D p(x) = 20 x2 (4 x), x = 1, 2, 3

(3x 1), x = 1, 2, 3

eBook plus

If the random variable X represents the number of boys in a


four-child family:
a write the values that X may take
b assuming that the Pr(boy) = 1 , find the probability distribution of X.

Digital doc

WorkSHEET 10.1

10C

measures of centre of discrete


random distributions
The expected value of a discrete random variable, X, is the average value of X. It is also referred
to as the mean of X or the expectation.
The expected value of a discrete random variable, X, is denoted by E(X) or the symbol
(mu). It is defined as the sum of each value of X multiplied by its respective probability;
that is,
E(X) = x1Pr(X = x1) + x2Pr(X = x2) + x3Pr(X = x3) + + xnPr(X = xn)
= x Pr(X = x)
all x

Note: The expected value will not always assume a discrete value.
WorkeD examPle 18

Find the expected value of a random variable that has the following probability distribution.
x

Pr(X = x)

2
5

1
10

3
10

1
10

1
10

Think
1

WriTe

E(X) = x Pr ( X = x )

Write the rule for the expected value.

all x

Substitute the values into the rule.

Evaluate.

E(X) = 1 5 + 2 10 + 3 10 + 4 10 + 5 10
2

= 5 + 10 + 10 + 10 + 10
2

= 25
WorkeD examPle 19

Find the unknown probability, a, and hence determine the expected value of a random variable that
has the following probability distribution.
x
Pr(X = x)

510

10

0.2

0.4

0.1

0.1

maths Quest 12 mathematical methods CaS for the Ti-nspire

Think

Write

Determine the unknown value of a using the


knowledge that the sum of the probabilities
must total 1.

0.2 + 0.4 + a + 0.1 + 0.1 = 1


0.8 + a = 1
a = 1 0.8
= 0.2

Write the rule for the expected value.

E(X) = x Pr ( X = x )
all x

Substitute the values into the rule.

E(X) = 2 0.2 + 4 0.4 + 6 0.2 + 8 0.1 + 10 0.1

Evaluate.

= 0.4 + 1.6 + 1.2 + 0.8 + 1


= 5

Worked Example 20

Find the values of a and b of the following probability distribution if E(X) = 4.29.
x
Pr(X = x)

0.1

0.1

0.3

0.2

0.2

Think

Write

Write an equation for the values of a and


b using the knowledge that the sum of the
probabilities must total 1. Call this equation [1].

0.1 + 0.1 + a + 0.3 + 0.2 + b + 0.2 = 1


0.9 + a + b = 1
a + b = 1 0.9
a + b = 0.1 [1]

Write the rule for the expected value.

E(X) = x Pr(X = x )
all x

Substitute the values into the rule.

4.29 = 1  0.1 + 2 0.1 + 3 a + 4 0.3 + 5 0.2


+ 6 b + 7 0.2

Evaluate and call this equation [2].

= 0.1 + 0.2 + 3a + 1.2 + 1 + 6b + 1.4


4.29 3.9 = 3a + 6b
3a + 6b = 0.39[2]

Solve the equations simultaneously.


Multiply equation [1] by 3 and call it equation
[3]. Subtract equation [3] from equation [2].
Solve for b. Substitute b = 0.03 into equation
[1]. Solve for a.

a + b = 0.1[1]
3a + 6b = 0.39[2]
3 (a + b = 0.1)
3a + 3b = 0.3[3]
[2] [3]: 3b = 0.09
b = 0.03
a + 0.03 = 0.1
a = 0.1 0.03
= 0.07

Answer the question.

a = 0.07 and b = 0.03

Worked Example 21

Niki and Melanie devise a gambling game based on tossing three coins simultaneously. If three heads
or three tails are obtained, the player wins $20. Otherwise the player loses $5. In order to make a
profit they charge each person $2 to play.
a What is the expected gain to the player?
b Do Niki and Melanie make a profit?
c Is this a fair game?

Chapter 10 Discrete random variables

511

Think
a

WriTe
a

List the possible outcomes and place


all of the information in a table.

Write the rule for the expected value.

S = {HHH, HHT, HTH, HTT, THH, THT, TTH,


TTT}
x

Pr(X = x)

1
8

3
8

3
8

1
8

Gain ($)

20

20

E(X) =

x Pr( X = x)

all x

Substitute the values into the rule.

= 20 1 + 5 3 + 5 3 + 20 1

Evaluate.

= 20 15 15 + 20

8
10
8

=
= $1.25
5

Answer the question.

The players expected gain per game is $1.25;


however, as each game incurs a cost of $2, the
player in fact loses 75c per game.

b Answer the question using the results from a .

The girls make a profit of 75c per game.

c Answer the question using the results from a .

No, this is not a fair game, since the cost to play


each game does not equal the expected gain of
each game.

Note: In a fair game E(X) = 0.

It is important to understand that the expected value signifies the average outcome of an
experiment and can be used to determine the feasibility of a situation. The previous worked
example illustrates that, in the long run, the player will lose on average 75 cents per game; it
does not mean the player will lose 75 cents each time the game is played.

expectation theorems
WorkeD examPle 22

eBook plus

A random variable has the following probability distribution.


x
Pr(X = x)
Find:

0.25

0.26

0.14

0.35

a E(X)
c E(2X 4)

int-0573
Worked example 22

b E(3X)
d E(X2).

Think
a

Tutorial

WriTe

Write the rule for the expected value.

E(X) = x Pr(X = x )
all x

512

Substitute the values into the rule.

Evaluate.

E(X) = 1 0.25 + 2 0.26 + 3 0.14


+ 4 0.35
= 0.25 + 0.52 + 0.42 + 1.4
= 2.59

maths Quest 12 mathematical methods CaS for the Ti-nspire

Write the rule for the expected value.

b E(3X) =

3x Pr(X = x)

all x

Substitute the values into the rule.

E(3X) = (3 1) 0.25 + (3 2) 0.26


+ (3 3) 0.14 + (3 4) 0.35

Evaluate.
Note: 1. The probability remains the same.

2. Each x-value is multiplied by 3
because of the new function, 3x.

= 3 0.25 + 6 0.26 + 9 0.14


+ 12 0.35
= 0.75 + 1.56 + 1.26 + 4.2
= 7.77

Write the rule for the expected value.

c E(2X 4) =

(2 x 4) Pr(X = x)

all x

Substitute the values into the rule.

= (2 1 4) 0.25 + (2 2 4)
0.26 + (2 3 4) 0.14
+ (2 4 4) 0.35

Evaluate.
Note: 1. The probability remains the same.

2. Each x-value is multiplied by
2 and then 4 is subtracted from
the result, because of the new
function, 2x 4.

=  2 0.25 + 0 0.26 + 2 0.14


+ 4 0.35
= 0.5 + 0 + 0.28 + 1.4
= 1.18

Write the rule for the expected value.

Substitute the values into the rule.

= (12) 0.25 + (22) 0.26 + (32) 0.14


+(42) 0.35

Evaluate.
Note: 1. The probability remains the same.

2. Each x-value is squared because
of the new function, x2.

= 1 0.25 + 4 0.26 + 9 0.14


+ 16 0.35
= 0.25 + 1.04 + 1.26 + 5.6
= 8.15

d E(X2) =

x 2 Pr(X = x)

all x

The above worked example displays some important points that shall be investigated.
For this example,
E(X) = 2.59
from part (b)
E(3X) = 7.77
note that
3E(X) = 3 2.59

= 7.77
from part (c)
E(2X 4) = 1.18
note that
2E(X) 4 = 2 2.59 4
= 1.18
Hence if X is a random variable and a is a constant, its expected value is defined by
E(aX) = aE(X). Furthermore, if X is a random variable where a and b are constants, then the
expected value of a linear function in the form f(X) = aX + b is defined by

E(aX + b) = aE(X) + b
If a = 0 then
E(aX + b) = aE(X) + b
becomes
E(0X + b) = 0E(X) + b
=b
These rules are called expectation theorems and are summarised below.
E(aX) = aE(X)
where X is a random variable and a is a constant.
E(aX + b) = aE(X) + b
where X is a random variable and a and b are constants.
E(b) = b
where b is a constant.
E(X + Y) = E(X) + E(Y)
where X and Y are both random variables.

Chapter 10 Discrete random variables

513

These theorems make it easier to calculate the expected values.


Finally, from part (d) of the above example it can be seen that
E(X2) [E(X)]2

Worked Example 23

Casey decides to apply for a job selling mobile phones. She receives a base salary of $200 per month
and $15 for every mobile phone sold. The following table shows the probability of a particular
number of mobile phones, x, being sold per month. What would be the expected salary Casey would
receive each month?
x
Pr(X = x)

50

100

150

200

250

0.48

0.32

0.1

0.06

0.04

Think

Write

Method 1
1

Define a random variable.

Let X = the number of mobile phones sold by


Casey in a month.

Write the rule for the expected salary.

E(15X + 200) =

(15x + 200)Pr(X = x)

all x

Substitute the values into the rule.

= (15 50 + 200) 0.48


+ (15 100 + 200) 0.32
+ (15 150 + 200) 0.1
+ (15 200 + 200) 0.06
+ (15 250 + 200) 0.04

Evaluate.

= 950 0.48 + 1700 0.32


+ 2450 0.1 + 3200 0.06
+ 3950 0.04
= 456 + 544 + 245 + 192 + 158
= 1595

Answer the question.

The expected salary Casey would receive each


month would be $1595.

Method 2
Using the expectation theorem:
1

Write the rule for the expected salary.

E(X) = x Pr (X = x )
all x

Substitute the values into the rule.

= 50 0.48 + 100 0.32 + 150 0.1


+ 200 0.06 + 250 0.04

Evaluate.

= 24 + 32 + 15 + 12 + 10
= 93

Using the fact that


E(aX + b) = aE(X) + b find E(15X + 200).

E(15X + 200) = 15E(X) + 200



= 15 93 + 200

= 1595

Note: Using the expectation theorem is quicker because it is easier to evaluate aE(X) + b than
E(aX + b).

514

Maths Quest 12 Mathematical Methods CAS for the TI-Nspire

Median and mode


The median is the middle value of the distribution. It is the value such that 50% of the
distribution lies to the left of this value, and 50% of the distribution lies to the right.
For a random variable, X, the mode is the most commonly occurring value, that is, it is the
variable with the highest probability.
Worked Example 24

For the following probability distributions, calculate:


i the median
ii the mode.

x
Pr(X = x)

0.25

0.4

0.15

0.2

11

14

0.18

0.32

0.2

0.3

b x

Pr(X = x)
Think
a

ii

Write the answer.

To calculate the mode, identify the highest


probability in the table.

i Pr(X 3) = 0.25, which is 0.5

Pr(X 5) = 0.25 + 0.4



= 0.65, which is 0.5
The median value is 5.

i i The highest probability is 0.4.

That is, Pr(X = 5) = 0.4.

Write the answer.

The mode is 5.
b

To calculate the median, work from the left


of the data. Add up the probabilities until
the total is 0.5 or greater.

i Pr(X 5) = 0.18, which is 0.5.

Pr(X 8) = 0.18 + 0.32



= 0.5
Similarly Pr(X 11) = 0.2 + 0.3

= 0.5
8 + 11
2
= 9.5

Calculate the median by calculating the


mean of 8 and 11.

Median =

Write the answer.

The median is 9.5.

To calculate the mode, identify the highest


probability in the table.

ii

To calculate the median, work from the left


of the data. Add up the probabilities until
the total is 0.5 or greater.

WRITE

i i The highest probability is 0.32.

That is, Pr(X = 8) = 0.32.

Write the answer.

The mode is 8.

REMEMBER

1. The expected value of a discrete random variable, X, is defined by the rule


E(X) =

x Pr(X = x). Also E(X2) = x2 Pr(X = x).

all x

all x

2. A game is considered fair if the cost to play the game is equal to the expected gain.
3. A fair game is one in which E(X) = 0.

Chapter 10 Discrete random variables

515

4. The expected value of a linear function can be calculated using the expectation
theorems:
E(aX) = aE(X)
E(aX + b) = aE(X) + b
E(b) = b
E(X + Y) = E(X) + E(Y)
5. E(X2) [E(X)]2
6. The median is the middle value of a distribution.
7. The mode is the variable with the highest probability.

Exercise

10c

Measures of centre of discrete random


distributions
1 WE18 Find the expected value of a random variable that has the following probability
distribution.
x
Pr(X = x)
2

12

0.21

0.08

0.19

0.17

0.35

Find the expected value of a random variable that has the following probability
distribution.
x

-2

-1

Pr(X = x)

1
18

1
3

1
18

2
9

1
6

1
18

1
9

3 WE19
Find the unknown probability, a, and hence determine the expected value of a
random variable that has the following probability distribution.
x
Pr(X = x)

11

0.11

0.3

0.15

0.25

0.1

4 Find the unknown probability, a, and hence determine the expected value of a random variable
that has the following probability distribution.
x

-2

10

13

Pr(X = x)

5
18

1
9

5
18

1
18

2
9

5 Find the unknown probability, b, and hence determine the expected value of a random variable
that has the following probability distribution.
x

Pr(X = x)

0.2

0.02

3b

0.1

0.08

6 Find the value of k, and hence determine the expected value of a random variable that has the
following probability distribution.

516

12

16

20

Pr(X = x)

6k

2k

3k

8k

If X represents the outcome of a fair die being rolled, find:


a the probability distribution of each outcome

Maths Quest 12 Mathematical Methods CAS for the TI-Nspire

b E(X).

8 Two fair dice are rolled simultaneously. If X represents the sum of the two numbers appearing
uppermost, find:
a the probability distribution of each outcome
b E(X).
9 A fair coin is tossed 4 times. If X represents the number of tails obtained, find:
a the probability distribution of each outcome
b E(X).
10 We20
x
Pr(X = x)
11

Find the values of a and b of the following distribution if E(X) = 1.91.


0

0.2

0.32

0.18

0.05

0.05

Find the values of a and b of the following distribution if E(X) = 2.41.


x
Pr(X = x)

0.2

0.23

0.15

0.12

12 We21 Lucas contemplates playing a new game which involves tossing three coins
simultaneously. He will receive $15 if he obtains 3 heads, $10 if he obtains 2 heads and $5 if
he obtains 1 head. However, if he obtains no heads he must pay $30. He must also pay $5 for
each game he plays.
a What is Lucass expected gain?
b Should he play the game? Why?
c Is this a fair game? Why?
13

Angie plays a game based on tossing three coins simultaneously. She will receive $10 if
she obtains 3 tails, $5 if she obtains 2 tails and $5 if she obtains 1 tail. However, if she obtains
no tails she must pay $40.
a What is Angies expected gain?
b Should she play the game? Why?
c Is this a fair game? Why?

14

X is a discrete random variable with the following probability distribution.


x
Pr(X = x)

0.3

0.2

0.4

0.1

Find the value of k if the mean is 5.3.


15 X is a discrete random variable with the following probability distribution.
x

10

14

Pr(X = x)

0.1

0.08

0.07

0.27

0.16

0.32

Find the value of k if the mean is 10.98.

eBook plus
Digital doc

SkillSHEET 10.1
Expected value
of a function
of a random
variable

16 A coin is biased such that the probability of obtaining a tail is 0.6. If X represents the number
of tails in three tosses of the coin, find:
a the probability distribution of X
b E(X)
c the mode.
17 We22
x
Pr(X = x)
Find:
a E(X)

A random variable has the following probability distribution.


1

2
15

7
15

1
3

1
15

b E(4X)

c E(2X + 1)

Chapter 10

d E(X2).

Discrete random variables

517

18 A random variable has the following probability distribution.


x
Pr(X = x)
Find:
a E(X)

0.33

0.25

0.27

0.15

c E(X 2 + 1)

b E(4X 6)

d E(3X 2).

19 We23 Christian decides to apply for a job selling mobile phones.


He receives a base salary of $180 per month and $12 for every
mobile phone sold. The following table shows the probability of a
particular number of mobile phones, x, being sold per month. What
would be the expected salary Christian would receive each month?
x
Pr(X = x)

50

100

150

200

250

0.32

0.38

0.2

0.06

0.04

20 We24
For the following probability distributions, calculate:
i the median
ii the mode.
eBook plus

Pr(X = x)

Digital doc

WorkSHEET 10.2

x
Pr(X = x)

x
Pr(X = x)

10D

0.25

0.15

0.1

0.1

0.4

13

17

0.06

0.36

0.17

0.29

0.12

1
4

1
16

3
16

1
8

1
8

1
16

3
16

measures of variability of
discrete random distributions

eBook plus
Interactivity

int-0255

Measures of
variability of discrete
random distributions

Variance

Variance is an important feature of probability distributions as it provides information about the


spread of the distribution with respect to the mean. If the variance is large, it implies that the
possible values are spread (or deviate) quite a distance from the mean. A small variance implies
that the possible values are close to the mean. Variance is also called a measure of spread or
dispersion.
The variance is written as Var(X) and denoted by the symbol 2 (sigma squared). It is defined
as the expected value (or average) of the squares of the spreads (deviations) from the mean.
The rule for variance is given by: Var(X) = E(X )2
= (X )2 Pr(X = x)
Although this rule clearly demonstrates how to obtain the variance, performing the calculation is
quite a lengthy process. Hence an alternative rule is used for calculating the variance:
Var(X) = E(X )2
= E(X2 2X + 2)
= E(X2) E(2X) + E(2)
= E(X2) 2E(X) + E(2)
= E(X2) 22 + 2
since E(X) = (the mean)
= E(X2) 2
= E(X2) [E(X)]2
518

maths Quest 12 mathematical methods CaS for the Ti-nspire

Worked Example 25

Find the expected value and variance of the following probability distribution table.
x
Pr(X = x)

0.15

0.12

0.24

0.37

0.12

Think

Write

Method 1: Using the rule


1

Write the rule for the expected value.

E(X) = x Pr(X = x )
all x

Substitute the values into the rule.

= 1 0.15 + 2 0.12 + 3 0.24


+ 4 0.37 + 5 0.12

Evaluate.

= 0.15 + 0.24 + 0.72 + 1.48 + 0.6


= 3.19

Calculate E(X2).

E(X2) =

x 2 Pr(X = x)

all x

= (12) 0.15 + (22) 0.12 + (32) 0.24 + (42)


0.37 + (52) 0.12
= 1 0.15 + 4 0.12 + 9 0.24 + 16 0.37
+25 0.12
= 0.15 + 0.48 + 2.16 + 5.92 + 3
= 11.71

Calculate [E(X)]2.

[E(X)]2 = 3.192

= 10.1761

Calculate Var(X) using the rule for variance.

Var(X) = E(X2) [E(X)]2



= 11.71 10.1761

= 1.5339

Method 2: Using a CAS calculator


1

Open a Lists & Spreadsheet page. Label


column A as x and column B as probx.
Enter the data into the relevant columns.
Press:
MENU b
4:Statistics 4
1:Stat Calculations 1
1:One-Variable Statistics 1
The number of lists is 1. The X1 List is x and
the frequency is probx.

The expected value and the variance (SSD)


can be read from the screen. You will need to
scroll down to get the value of the variance.

E(X) = 3.19
Var(X) = 1.5339

Chapter 10 Discrete random variables

519

WorkeD examPle 26

eBook plus

Find the variance of 2Y + 1 for the following probability distribution table.


y
Pr(Y = y)

0.25

0.35

0.2

0.2

Think

Tutorial

int-0574
Worked example 26

WriTe

E(2Y + 1) = (2 y + 1) Pr(Y = y )

Write the rule for the expected value.

Substitute the values into the rule.

= (2 0 + 1) 0.25 + (2 1 + 1) 0.35
+ (2 2 + 1) 0.2 + (2 3 + 1) 0.2

Evaluate.

= 1 0.25 + 3 0.35 + 5 0.2 + 7 0.2


= 0.25 + 1.05 + 1 + 1.4
= 3.7

Calculate [E(2Y + 1)]2.

Calculate E(2Y + 1)2.

Calculate Var(2Y + 1) using the rule.

all x

[E(2Y + 1)]2 = 3.72


= 13.69
E(2Y + 1)2 = 12 0.25 + 32 0.35 + 52 0.2 + 72 0.2
= 0.25 + 3.15 + 5 + 9.8
= 18.2
Var(2Y + 1) = E(2Y + 1)2 [E(2Y + 1)]2
= 18.2 13.69
= 4.51

The variance of a linear function can also be calculated by the following rule:
Var (aX + b) = a2Var (X)
For worked example 26, given that Var(Y) = 1.1275, Var(2Y + 1) can be determined using the
above rule:
Var(2Y + 1) = 22Var(Y)
= 4 1.1275
= 4.51 as before.
WorkeD examPle 27

X is a discrete random variable with the following probability distribution.


x
Pr(X = x)

0.15

0.3

0.45

0.1

Find the value of k, a positive integer, if the variance is 1.7475.


Think
1

Write the rule for the expected value.

WriTe

E(X) = x Pr(X = x )
all x

520

Substitute the values into the rule.

= 3 0.15 + 4 0.3 + 6 0.45 + k 0.1

Evaluate.

= 0.45 + 1.2 + 2.7 + 0.1k


= 4.35 + 0.1k

maths Quest 12 mathematical methods CaS for the Ti-nspire

E(X2) =

Calculate E(X2).

x 2 Pr(X = x)

all x

= (32) 0.15 + (42) 0.3 + (62) 0.45 + (k2) 0.1


= 9 0.15 + 16 0.3 + 36 0.45 + k2 0.1
= 1.35 + 4.8 + 16.2 + 0.1k2
= 22.35 + 0.1k2

Calculate [E(X)]2.

[E(X)]2 = (4.35 + 0.1k)2



= 0.01k2 + 0.87k + 18.9225

Calculate Var(X) using the rule and


equate it to the given value of the
variance; that is, 2 = 1.7475.

Var(X) = E(X2) [E(X)]2


1.7475 = 22.35 + 0.1k2 (0.01k2 + 0.87k + 18.9225)
1.7475 = 0.09k2 0.87k + 3.4275

Solve for k.

0.09k2 0.87k + 3.4275 1.7475 = 0


0.09k2 0.87k + 1.68 = 0
(9k 24)(k 7) = 0
k=

24
9

(or 2 3 ) or k = 7

k = 7. Reject the other value of k since the variable is


discrete.

Answer the question.

Standard deviation
Another important measure of spread is the standard deviation. It is written as SD(X) or denoted
by the symbol (sigma). The standard deviation is the positive square root of the variance. It is
defined by the rule:
SD(X) =

Var ( X )


= 2

=
Variation and standard deviation are used extensively in many real-life applications involving
statistics.
Analysis of data would be useless without any information about the spread of the data.
Worked Example 28

A random variable has the following probability distribution.


x

Pr(X = x)

1
4

3
8

1
8

1
4

Calculate the expected value, the variance and the standard deviation.
Think

Write

Method 1: Using the rule


1

Calculate the expected value.

Calculate [E(X)]2.

E(X) = 0 4 + 1 8 + 2 8 + 3 4
3

=0+8+8 +4

= 18

( )

[E(X)]2 = 1 83

57

= 164 (1.890625)

Chapter 10 Discrete random variables

521

E(X2) = 02 4 + 12 8 + 22 8 + 32 4

Calculate E(X2).

=0+8+8 +4

= 38

Var(X) = E(X2) [E(X)]2


1
57

= 38 164

Calculate Var(X).

15

= 164 ( 1.234375)

Calculate the standard deviation.

SD(X) = 1.234 375

Round the answer to 4 decimal places.

= 1.1110

Method 2: Using a CAS calculator


1

Open a Lists & Spreadsheet page. Label


column A as x and column B as probx.
Enter the data into the relevant columns.
Press:
MENU b
4:Statistics 4
1:Stat Calculations 1
1:One-Variable Statistics 1
The number of lists is 1. The X1 List is x and
the frequency is probx.

The expected value and the variance (SSD) and


the standard deviation can be read from the
screen. You will need to scroll down to get the
value of the variance.

E(X) = 1.375
Var(X) 1.2344
SD(x) 1.1110

Worked Example 29

In order to encourage car pooling, a new toll is to be introduced on the Eastgate Bridge. If the
car has no passengers, a toll of $2 applies. Cars with one passenger pay a $1.50 toll, cars with two
passengers pay a $1 toll and cars with 3 or more passengers pay no toll. Long-term statistics show
that the number of passengers (X) follows the probability distribution given below.
x (no. of passengers)
Pr(X = x)

0.4

0.35

0.2

0.05

a Construct a probability distribution of the toll paid.


b Find the mean toll paid per car.
c Find the standard deviation of tolls paid.
Think
a Construct a table of values of toll information.

Probabilities remain the same.

Write
a Let Y = toll to be paid.

y
Pr(Y = y)

522

Maths Quest 12 Mathematical Methods CAS for the TI-Nspire

1.5

0.4

0.35

0.2

0.05

b E(Y) =

Write the rule for the expected value.

y Pr(Y = y)

all y

Substitute the values into the rule.

= 2 0.4 + 1.5 0.35 + 1 0.2 + 0 0.05

Evaluate.

= 0.8 + 0.525 + 0.2 + 0


= 1.525

Round the answer to 2 decimal places.

= 1.53

Answer the question.

The mean toll is $1.53.

Calculate

E(Y2).

E(Y2) =

y 2 Pr (Y = y)

all y

= (22) 0.4 + (1.52) 0.35 + (12) 0.2


+ (02) 0.05
= 4 0.4 + 2.25 0.35 + 1 0.2 + 0
= 1.6 + 0.7875 + 0.2
= 2.5875

Calculate [E(Y)]2.

[E(Y)]2 = 1.5252

= 2.325 625

Calculate Var(Y).

Var(Y) = E(Y2) [E(Y)]2



= 2.5875 2.325 625

= 0.261 875

Calculate the standard deviation.

SD(Y) =

Round the answer to 2 decimal places.

Answer the question.

The standard deviation of tolls paid is $0.51.

0.261 875

= 0.51

Interpreting the standard deviation


A characteristic of many distributions is that approximately 95% of the spread or distribution
lies between 2 standard deviations of the mean; that is,
Pr( 2 X + 2) 0.95
It is important to note that when calculating the Pr( 2 X + 2) for a specific
distribution an exact value of 0.95 will not always be achieved, but should be close to it.
For many random variables, approximately 95% of the spread of the population lies
between 2 standard deviations of the mean, that is,
Pr( 2 X + 2) 0.95.
Worked Example 30

A probability distribution is shown below.


Given that = 3.38 and = 0.946, calculate Pr( - 2 X + 2).
x
Pr(X = x)

0.14

0.5

0.23

0.1

0.03

Think
1

Calculate - 2

Write

- 2 = 3.38 - 2 0.946

= 3.38 - 1.892

= 1.488

Chapter 10 Discrete random variables

523

Calculate + 2.

+ 2 = 3.38 + 2 0.946
= 3.38 + 1.892
= 5.272

Substitute the values obtained in steps 1 and 2


into the given interval.
Since X represents discrete values,
Pr(1.488 X 5.272) becomes Pr(2 X 5).

Pr( 2 X + 2) = Pr(1.488 X 5.272)


= Pr(2 X 5)
= 0.14 + 0.5 + 0.23 + 0.1
= 0.97
Note: In this example, 97% of the distribution lies
within 2 standard deviations of the mean, which is
close to the estimated 95%.

WorkeD examPle 31

The table below represents the probability distribution of the number of accidents
per week in a factory.
x
Pr(X = x)

0.02

0.22

0.18

0.16

0.14

0.07

0.13

0.03

0.05

Given that = 4.36 and = 2.105 find Pr( 2 X + 2).


Think

WriTe

Calculate 2.

2 = 4.36 2 2.105
= 4.36 4.21
= 0.15

Calculate + 2.

+ 2 = 4.36 + 2 2.105
= 4.36 + 4.21
= 8.57

Substitute the values obtained in steps 1 and 2


into the given interval.
Since X represents discrete values,
Pr(0.15 X 8.57) becomes Pr(1 X 8).
Note: In this example,
Pr(1 X 8) = 1 Pr(X = 9).

Pr( 2 X + 2) = Pr(0.15 X 8.57)


= Pr(1 X 8)
= Pr(X = 1) + Pr(X = 2)
+ Pr(X = 3) + Pr(X = 4)
+ Pr(X = 5) + Pr(X = 6)
+ Pr(X = 7) + Pr(X = 8)
= 1 Pr(X = 9)
= 1 0.05
= 0.95
Note: The answer is the estimated one of 95%.
In this case, 95% of the distribution lies within
2 standard deviations of the mean.

WorkeD examPle 32

The probability distribution of X is given by the formula:


x2
Pr(X = x) =
where x = 2, 3, 4, 5.
54
Find:
a the probability distribution of X as a table
b the expected value of X, correct to 4 decimal places
c the standard deviation of X, correct to 4 decimal places
d Pr( 2 X + 2), correct to 3 decimal places.
524

maths Quest 12 mathematical methods CaS for the Ti-nspire

eBook plus
Tutorial

int-0575
Worked example 32

Think
a

WriTe

Substitute each of the x-values


into the equation and obtain the
corresponding probability.

a When x = 2, p(x) =

=
When x = 3, p(x) =
=
When x = 4, p(x) =
=
When x = 5, p(x) =

Enter the information into a table.

Calculate the expected value.

Pr(X = x)

2
27

1
6

8
27

25
54

=
c

Round the answer to 4 decimal places.

Calculate [E(X)]2.

2
+31+
27
6
4
+ 3 + 32 + 125
27
6
27
54
4
4 27

b E(X) = 2

4
54
2
27
9
54
1
6
16
54
8
27
25
54

8
27

+5

25
54

= 4.1481
c [E(X)]2 = (4 4 )2
27
151

= 17 729 ( 17.2071)
2

Calculate E(X2).

25

E(X2) = 22 27 + 32 6 + 42 27 + 52 54
8

= 27 + 6 +

128
27

625
54

= 18 9 ( 18.1111)
3

Calculate Var(X).

Var(X) = E(X2) [E(X)]2


1

151

= 18 9 17 729
=

659
729

( 0.9040)

SD(X) = 0.9040
= 0.9507776039

Calculate the standard deviation.

Round the answer to 4 decimal places.

Calculate 2.

d 2 = 4.1481 2 0.9508

Calculate + 2.

Substitute the values obtained in


steps 1 and 2 into the given interval.
Since X represents discrete values,
Pr(2.2465 X 6.0497) becomes
Pr(3 X 6).

+ 2 = 4.1481 + 2 0.9508
= 4.1481 + 1.9016
= 6.0497
Pr( 2 X + 2)
= Pr(2.2465 X 6.0497)
= Pr(3 X 6)
= 1 Pr(X = 2)
=1 2

= 0.9508
= 4.1481 1.9016
= 2.2465

27

= 25
27

Chapter 10

Discrete random variables

525

= 0.926

Round the answer to 3 decimal places.

Note: In this example, 92.6% of the distribution lies


within 2 standard deviations of the mean, which is
close to the estimated value of 95%.
rememBer

1. The variance, Var(X) or 2 is defined by the rule:


Var(X) = E(X2) [E(X)]2
2. The variance of a linear function can also be calculated by the following rule:
Var (aX + b) = a2Var(X).
3. The standard deviation, SD(X) or , is defined by the rule:
SD(X) = Var ( X )
= 2
4. Approximately 95% of the spread of the population in many distributions lies between
2 standard deviations of the mean; that is, Pr ( 2 x + 2) 0.95.
exerCiSe

10D
eBook plus
Digital doc

Spreadsheet 100
Probability
distribution

measures of variability of discrete random


distributions
1 We25 Find the expected value and variance of the following probability distribution table.
x
Pr(X = x)

0.2

0.4

0.3

0.1

2 A random variable has the following probability distribution.


x

Pr(X = x)

1
8

3
16

9
16

1
8

Find:
a the expected value, E(X)

b the variance of X, Var(X).

3 The cost of a loaf of bread is known to vary on any day according to the following probability
distribution.
x

$1.20

$1.25

$1.30

$1.35

$1.60

Pr(Y = y)

0.05

0.2

0.1

0.25

0.4

Find:
a the expected cost of a loaf of bread

b the variance of the cost.

4 We26 Find the variance of 2Y 1 for the following probability distribution table.
x
Pr(Y = y)

0.3

0.2

0.3

0.2

5 A random variable has the following probability distribution.


x
Pr(X = x)
Find:
a Var(X)
526

0.15

0.3

0.42

0.13

b Var(2X)

maths Quest 12 mathematical methods CaS for the Ti-nspire

c Var(3X + 1)

d Var(5X + 7).

6 A random variable has the following probability distribution.


x
Pr(X = x)

0.27

0.15

0.13

0.1

0.35

Find:
a Var(X)
7 WE27

d Var(5X 2).

Let X be a discrete random variable with the following probability distribution.

x
Pr(X = x)

c Var(10X 5)

b Var(3X)
2

0.3

0.1

0.5

0.1

Find the value of k, a positive integer, if the variance is 5.8.


8 Let X be a discrete random variable with the following probability distribution.
x
Pr(X = x)

10

0.1

0.2

0.3

0.4

Find the value of k, a positive integer, if the variance is 7.96.


9 WE28 A random variable has the following probability distribution.
x

Pr(X = x)

1
4

1
3

1
4

1
6

Calculate the expected value, the variance and the standard deviation.
10 A random variable has the following probability distribution.
x
Pr(X = x)

10

12

0.3

0.3

0.2

0.2

Find:
a the expected value, E(X)
c the standard deviation of X, SD(X), to 2 decimal places.

b the variance of X, Var(X)

11

For a random variable, X, E(X) = 12 and


E(X2) = 340. Find the standard deviation of X.
12 For a random variable, X, E(X) = 20 and E(X2) = 529. Find
the standard deviation of X, to 2 decimal places.
13 WE29 In order to encourage car pooling, a new toll is
to be introduced on the International Gateway. If the car
has no passengers, a toll of $2 applies. Cars with one
passenger pay a $1.50 toll, cars with two passengers pay
a $1 toll and cars with 3 or more passengers pay no toll.
Long-term statistics show that the number of passengers
follows the probability distribution given below.
x
Pr(X = x)

0.5

0.3

0.15

0.05

a Construct a probability distribution of the toll paid.


b Find the mean toll paid per car.
c Find the standard deviation of tolls paid.
14 WE31 The table below represents the probability
distribution of the number of accidents per week in afactory.
x
Pr(X = x)

0.03

0.21

0.16

0.18

0.14

0.07

0.15

0.01

0.05

Given that = 4.35 and = 2.08, find Pr( 2 X + 2).

Chapter 10 Discrete random variables

527

x2
15 WE32 The probability distribution of X is given by the formula, Pr(X = x) =
where
35
x = 1,3, 5. Find:
a the probability distribution of X as a table
b the expected value of X
c the standard deviation of X, to 4 decimal places
d Pr( 2 X + 2).

16 The probability distribution of X is given by the formula, Pr(X = x) =


Find:
a the probability distribution of X as a table
c the standard deviation of X

x2 1
where x = 2, 3, 4, 5.
50

b the expected value of X


d Pr( 2 X + 2).

17 A random variable has the following probability distribution.


x
Pr(X = x)

0.4

0.2

0.2

Find:
a the value of the constant k
c E(X), the mean of X
e SD(X), the standard deviation of X, to 4 decimal places

b the most likely value of X


d Var(X), the variance of X
f Pr( 2 X + 2).

18 Calculate the values between which 95% of the distribution would be expected to lie where:
a = 4, = 2
b = 10, = 3
c = 35, = 7
1
1
d = 21.6, = 5.2
e = 9.7, = 0.7
f = 17 2 , = 23.

19 Two fair dice are rolled and the outcomes are noted. If X represents the sum of the two

numbers showing, find:


a the expected value of X
b the variance of X, to 2 decimal places
c Pr( 2 X + 2), to 2 decimal places.

20 For the spinner shown, X represents the number obtained. Find:


a the probability distribution of X
b the expected value of X
c the standard deviation of X, to 2 decimal places
d the probability that the number is 4, given that it is not 1.

21 MC The following table represents a discrete probability

2
1

Pr(X = x)

2k

3k

4k

distribution for a random variable, X.


x

2
2

The standard deviation of x is:


A 1.0
B 1.2
c 1.8
d 2.0
e 2.2
Questions 22 and 23 refer to the following information. The probability distribution of X is
given in the table below.
x
Pr(X = x)

0.4

0.3

0.1

0.2

22 MC The variance and standard deviation of X, respectively, are:


A 3.9, 15.21

B 26.1, 3.3

23 MC Var (6X 3) is equal to:


A 91.26

528

B 541.56

c 26.1, 3.9

d 26.1, 5.11

e 11.61, 3.41

c 417.96

d 939.6

e 140.4

Maths Quest 12 Mathematical Methods CAS for the TI-Nspire

Summary
Probability revision

Outcomes are results of experiments.


The set of all possible outcomes of an experiment is called the sample space and is denoted by , and each
possible outcome is called a sample point.
A subset of the sample space is known as an event.
The union (symbol ) of two events A and B implies a combined event, that is, either event A or event B or
both occurring. Common elements are written only once.
The intersection (symbol ) of two events A and B is represented by the common sample points of the two
events.
Venn diagrams involve drawing a rectangle that represents the sample space and a series of circles that
represent subsets of the sample space. They provide a visual representation of the information at hand and
clearly display the relationships between sets.
The probability of an event occurring is defined by the rule
Pr(A) =

number of favourable outcomes


total number of possible outcomes

The probability of an event occurring lies within the restricted interval 0 Pr(A) 1.
The individual probabilities of a particular experiment will sum to 1; that is, p(x) = 1.
The addition rule of probability is defined by the rule Pr(A B) = Pr(A) + Pr(B) Pr (A B).
If two events A and B are mutually exclusive, then Pr(A B) = 0 and therefore the addition rule becomes
Pr(A B) = Pr(A) + Pr(B).
If two events A and B are independent, then Pr (A B) = Pr(A) Pr(B).
Karnaugh maps and probability tables summarise all combinations of two events (for example A and B) and
their complements (for example A and B).
Pr ( A B)
, where Pr(B) 0. This can be transposed
Conditional probability is defined by the rule Pr(A|B) =
Pr ( B)
to Pr(A B) = Pr (A|B) Pr(B).

Tree diagrams are useful tools in solving probability tasks as they display each of the possible outcomes
along with their respective probabilities.
A combination is defined by nCr, that is, the number of selections of n different objects taken r at a time.
Discrete random variables

A random variable is one whose value is determined by the outcome of an experiment.


Discrete random variables generally deal with number or size and are able to be counted.
Two important characteristics satisfy all discrete probability distributions:
1. Each probability lies in a restricted interval 0 Pr(X = x) 1.
2. The probabilities of a particular experiment sum to 1; that is,

Pr( X = x) = 1
If these two characteristics are not satisfied, then there is no discrete probability distribution.
Measures of centre of discrete random distributions

The expected value of a discrete random variable, X, is denoted by E(X) or the symbol (mu). It is defined
by the rule:
E(X) =

x Pr (X = x)

all x

A fair game is one in which E(X) = 0.

Chapter 10 Discrete random variables

529

The expected value of a linear function can be calculated using the expectation theorems:
E(aX) = aE(X)
E(aX + b) = aE(X) + b
E(b) = b
E(X + Y) = E(X) + E(Y)
Note: E(X2) [E(X)]2
The median is the middle value of a distribution.
The mode is the variable with the highest probability.
Measures of variability of discrete random distributions

The variance is denoted by Var(X) or the symbol 2 (sigma squared).


It is defined by the rule:
Var(X) = E(X2) [E(X)]2.
The variance of a linear function can also be calculated by the following rule:
Var(aX + b) = a2Var(X)
The standard deviation is written as SD(X) or denoted by the symbol .
It is defined by the rule:
SD(X) = Var( X )

= 2
Approximately 95% of the spread of the population in many distributions lies between 2 standard deviations
of the mean, that is,
Pr( 2 X + 2) 0.95

530

Maths Quest 12 Mathematical Methods CAS for the TI-Nspire

chapter review
Short answer

1 The Santaroos have 3 soccer teams, A, B of and


C, entered in the interschool championships.
Each team is entered in a separate division. The
probability of each team winning their particular
final is given as follows:
1
1
Pr(Team A wins) = 2 Pr(Team B wins) = 3
3

Pr(Team C wins) = 4 .
Find the probability that:
a none of the Santaroos teams win
b one of the Santaroos teams wins
c two of the Santaroos teams win
d each of the Santaroos teams win their particular
final.
2 The probability of event A occurring is 0.25 and the
probability of event B occurring is 0.5.
a Calculate Pr(A B) when Pr(A B) = 0.1.
b Calculate Pr(A B) when events A and B are
mutually exclusive.
Exam tip Many students confuse independent

events with mutually exclusive events.




[Assessment report 1 2007]


[ VCAA 2007]
3 Thirty students were required to complete a logic
puzzle. The time taken to complete the puzzle was
recorded in the table below.
Time taken
(whole
number of
minutes)

Number of
students

10

For the information given:


a what proportion of students completed the
puzzle in less than 5 minutes?
b what proportion of students took more than
5minutes to complete the puzzle?
4 The probability distribution for the service time at a
bakery is given below.
Service
time (whole
number of
minutes)
Probability

0.2

0.3

0.2

0.2

0.1

a What is the probability that the service time for


a customer is:

i 2 minutes?
ii 2 minutes or less?
iii more than 2 minutes?
iv not more than 4 minutes, given it is more
than 1 minute?
b What is the expected value for the service
time?
c If 50 customers were served at the bakery in a
morning, how many of these would you expect
to take 4 minutes to be served?
5 A LUCKYDIPZ is a toy that is packaged inside
an egg-shaped chocolate. A certain manufacturer
produces 4 different types of LUCKYDIPZ toy
car, ship, plane and ring, in the proportions given in
the table below.
Car

4k2 + 4k

Ship

5k2 + 2k

Plane

k2 + k

Ring

2k

a Show that k must be a solution of the quadratic


equation 10k2 + 9k 1 = 0.
b Find the exact value of k.

[ VCAA 2004]
6 The probability distribution of X is given by the
x2
formula, Pr(X = x) = , where x = 1, 2, 3, 4. Find:
30
a the probability distribution of X as a table
b the expected value of X.
7 A player rolls a fair die. If the player gets a 1 on the
first roll, she rolls again and her score is the sum of
the two results, otherwise her score is the result of
the first roll. The die cannot be thrown more than
twice. Find:
a the probability distribution
b the expected score
c Pr(X < ).
8 Tayah and Sandy are playing a game where a
biased die is rolled. The probabilities of rolling
each number are Pr(1) = 0.2, Pr(2) = Pr(3) =
Pr(5) = 0.1, Pr(4) = 0.3 and Pr(6) = 0.2.
They have to pay $1.00 to play. If a 2, 3 or 5 is
rolled, they win $3. If a 1 or 6 is rolled, they get

Chapter 10 Discrete random variables

531

their money back, and if a 4 is rolled, they do not


receive any money. Is this a fair game to play?
9 A game of three-up is played where three coins
are tossed simultaneously. A player must pay $2 to
play the game. If three heads come up, the player
collects $6. If two heads come up, the player
collects $3. Is it a fair game?
10 Kylie is about to
2 points
30 cm
compete in her
clubs archery
5 points
finals. If she is
10 cm
equally likely to
11
points
hit any point on
the board and
20 cm
never misses the
target, find:
a her expected
score from

i 1 shot at the target shown
ii 5 shots at the target shown
b her probability of getting five points on every
one of her 5 shots.

0.1

0.45

0.09

0.26

0.1

11

Pr(X = x)

1
5

1
5

1
10

2
5

1
10

Pr(X = x)
b

12 At Fast Eddys Drive-In Theatre the cost is $5 per


car, plus $1 per occupant. The variable X represents
the number of people in any car and is known to
follow the probability distribution below.
x
Pr(X = x)

0.4

0.2

0.3

0.1

Find:
a the expected cost per car
b Fast Eddys expected profit if 100 cars enter
and costs for wages, electricity, etc. are $300
c the mode.
13 Let X be a discrete random variable with the
following probability distribution.
x
Pr(X = x)

1
0.1

3
0.25

5
0.35

Find the value of n if the mean is 4.7.

532

15 A raffle is to be drawn from 500 tickets. Each ticket


was purchased for $1, with first prize being $200,
second prize $150 and third prize $100. Find:
a the expected loss per ticket
b the profit made by organisers, that is, the house
c the house percentage.
x ( x + 1)
for 0 x n
40
a Find the value of n.
b Find the probability distribution for X as a
table.
c Find the expected value of X.

16 Pr(X = x) =

Multiple choice

11 For the following probability distributions,


calculate:
i the median
ii the mode.
a

14 Five thousand scratch-and-match tickets are to


be sold for $2 each. The tickets offer the following
prizes:
1 prize of $5000 2 prizes of $500
20 prizes of $50 100 prizes of $10.
Find:
a the expected loss per ticket
b the profit made by organisers, that is, the house
c the house percentage.

1 If Pr(A) = 0.65, Pr(B) = 0.37 and Pr (A B) = 0.28,


then Pr(A B) is equal to:
A 0.93
B 0.56
C 0.09
D 0.74
E 1.02
2 If Pr(A) = 0.47, Pr(B) = 0.27 and Pr (A B) = 0.19,
then Pr(A|B) is equal to:
A 0.57
B 0.40
C 0.70
D 0.43
E 0.30
3 The probability that Fiona attends an aerobics
class is 0.60, and the probability that Kath attends
an aerobics class is 0.85. If these two events are
independent, the probability of one of these two
people attending an aerobics class is:
A 0.60
B 0.85
C 0.51
D 0.34
E 0.43
4 A bag contains 3 white balls and 7 yellow balls.
Three balls are drawn one at a time from the bag
without replacement. The probability that they are
all yellow is:
A 3
B 27
c 21
D

500
7

24

1000
243
1000

100

Exam tip Students need to be reminded that


material from Units 1 and 2 of the study can be drawn
on for the end of year exams.

n
0.3

[Assessment report 1 2006]

[ VCAA 2006]

Maths Quest 12 Mathematical Methods CAS for the TI-Nspire

5_61_60255_MQ12MM_CAS_10.indd 532

7/15/10 9:24:36 AM

5 Which of the following random variables is


discrete?
A The number of runs scored by Sir Donald
Bradman in his cricketing career
B The weight of people in an elevator
C The life span of a fly
D The volume, in litres, of water in the Yarra
River
E The time, in hours, for a student to complete a
Mathematical Methods test
6 Which of the following does not represent a
probability distribution?
A x

10

0.2

0.3

0.2

0.2

0.1

0.1

0.24

0.03

0.56

0.07

12

15

0.36

0.12

0.4

0.02

0.1

0.1

0.2

0.4

0.2

0.3

0.09

0.12

0.41

0.18

0.2

Pr(X = x)
B

x
Pr(X = x)

C x
Pr(X = x)
D x
Pr(X = x)
E

x
Pr(X = x)

7 X is a discrete random variable with the following


probability distribution.
x
Pr(X = x)

0.1

0.2

0.3

0.1

0.2

0.1

The probability that the variable is an odd number,


given that it is less than 4 is:
A 1
B 7
c 4
2

10

d 3

1
3

8 The value of k for the following probability


distribution is:
x
Pr(X = x)
A 0.15
D 0.4

0.3

5k

0.2

3k

0.1

B 0.05
e 1

c 0.25

9 A die is biased so that Pr(X = 1) = Pr(X = 2) = 0.1


and Pr(X = 3) = Pr(X = 4) = Pr(X = 5) = Pr(X = 6)
= 0.2. A game is played where a player rolls the die.
The player receives $5 if a number greater than 4
is obtained but must pay $2 if a number less than

or equal to 4 comes up. The expected result for the


player for each roll is:
A a loss of 80c
B a loss of $1.40
C a win of 80c
d a loss of $1.40
E a win of $1.20
10 Gertrudes Gambling House offers patrons a card
game which uses a deck comprising four aces,
three kings, two queens and one jack. A player
draws a card at random. If a jack is drawn the
player wins $5, while a queen results in a win of
$2. However, if the player draws an ace, a loss of
$1 is incurred. On average, Gertrudes Gambling
House wins 40c each time the game is played. If a
king is drawn, a player must pay:
A $1.00
B $3.00
c $1.50
D $2.00
e $2.50
11 Sams chance of getting a bullseye while playing
darts is 0.1 and his chance of missing the board
altogether is 0.2. Sam collects $2 for a bullseye or
20c for hitting any other part of the board. If the
game is to be fair, missing the board altogether
means Sam must pay:
A $1.70
B $2.70
c $2.20
D $1.50
e $1.85
12 The toll for a new freeway is $2 per car and
50cperoccupant. Long-term surveys show that X, the
number of occupants per car, is distributed as follows.
x
Pr(X = x)

0.3

0.3

0.2

0.1

0.1

The expected toll for each car is:


A $3.20
B $3
c $3.50
D $2.95
e $3.65
Questions 13 and 14 refer to the following
probability distribution.
x
Pr(X = x)

0.3

0.3

0.2

0.1

0.1

13 E(X) is equal to:


A 2.5
B 3.1
c 5.2
D 2.4
e 2.6
14 E(5X 8) is equal to:
A 18
B 4.5
c 4
D 7.5
e 5
Questions 15, 16 and 17 refer to the following
probability distribution.
x

Pr(X = x)

0.1

0.25

0.2

0.15

0.3

15 E(X) is equal to:


A 1.2
B 0.5
D 1.5
e 0.3

c 1.4

Chapter 10 Discrete random variables

533

16 Var(X) is equal to:


A 2
B 1.7
D 0.61
e 2.11

c 1.91

17 Var(3X + 2) is equal to:


A 8
B 17.19
D 15.3
e 2.7

c 7.73

C = 7.6 and = 3.3


D = 7.5 and = 3.3
E = 6 and = 10.89
19 Let X be a random variable with the following
probability distribution.
x

18 Let X be a discrete random variable with the


following probability distribution.
x
Pr(X = x)

Pr(X = x)

12

0.21

0.35

0.17

0.27

The mean and standard deviation are:


A = 6 and = 3.3
B = 7.5 and = 10.89

0.2

0.2

0.4

0.2

The values between which 95% of the distribution


lie for the discrete random variable X are:
A (1, 3)
B (1, 4)
c (2, 4)
D (1, 2)
e (2, 3)

Extended Response

Probability

Jam

$14.40

1
4

Iced

$15.60

1
4

Cinnamon

$12.00

3
10

Iced jam

$18.00

1
5

12

35

3 26 0 32 15 1

4
21

5 17 34 6 27 1
22
33

Find:
a the mean price per box
b the standard deviation per box
c the average price per doughnut.
The canteen wants to make a 14% profit on costs. Find:
d the cost of a doughnut at the canteen, if all doughnuts are to be sold
for the same amount
e the average profit per box.

Cost per box (2 dozen)

1 9 22 18 29

Doughnut type

28

1 Bobs Bakery makes four different types of doughnut, each at a different price, depending on the ingredients
used. A school canteen buys all its doughnuts from Bob and is currently estimating budgets for the upcoming
financial year. Types of doughnut and their prices are listed below, along with their popularity (expressed as a
probability).

14 3

20

3
11

23 10 5 24 1
63
0 8
3

534

Maths Quest 12 Mathematical Methods CAS for the TI-Nspire

2 Amina plays roulette, a game where a wheel containing 37 slots numbered


036 is spun and the winning number is the one in which a ball lodges
when the wheel stops spinning. Amina plays three different games:
a First she bets $20 on her favourite number coming up at Casinonominated odds of 35:1 against.

i How much would Amina collect if her number came up?
ii Find her expected win or loss for the game.
iii Is this game fair?
b In the second game Amina bets $20 on an even number coming up at Casino-nominated odds of 1:1
(even money chance).

i How much would Amina collect if an even number came up?
ii Find her expected win or loss for the game.
iii Is this game fair?

c In game number three, Amina bets $20 on a line of 12; that is, if numbers 112 come up, she wins. The
Casino-nominated odds for this game are 2:1 against.
i How much would Amina collect if one of her numbers came up?
ii Find her expected win or loss for the game.
iii Is this game fair?
d What is the house percentage for these games?
3 A door-to-door telecommunications representative has recorded her day-by-day sales figures over a period of
time. She knows that her probability of selling X packages on any one day follows the probability distribution
shown in the table.
x
Pr(X = y)

>5

2t2

3t

2t2

2t

4t2 + t

a
b
c
d
e
f
g

Find the value of t.


Find the probability that she sells at least 2 packages on any one day.
Find the probability that she sells at most 4 packages on any one day.
Find the number of packages she can expect to sell each day.
Calculate the Var(X) and standard deviation of X, correct to 4 decimal places.
Find Pr( 2 X + 2).
If the representative receives a commission of $25 per package sold and a bonus of $200 if she sells 4 or
more packages in one day, find her expected daily earnings from commissions and bonuses.
h Given that the representative will sell at least two packages tomorrow, find the probability that she will
get her $200 bonus.
eBook plus
Digital doc

Test Yourself
Chapter 10

Chapter 10

Discrete random variables

535

eBook plus

aCTiViTieS

Chapter opener
Digital doc

10 Quick Questions: Warm up with ten quick


questions on applications of discrete random
variables. (page 484)
10A

Probability revision

Tutorials

We 7 int-0570: Watch a worked example on


calculating conditional probability. (page 493)
We 10 int-0571: Watch a worked example on
considering combinations when conducting
probability experiments. (page 496)
10 B

Discrete random variables

Tutorial

We 16 int-0572: Watch a worked example on


verifying probability functions. (page 504)
Digital docs

Spreadsheet 100: Investigate probability


distributions. (page 507)
WorkSHEET 10.1: Determine probabilities given
discrete data and distributions including conditional
probability and graphs. (page 510)
10C

Measures of centre of discrete random


distributions

Measures of variability of discrete random


distributions

Interactivity int-0255

Measures of variability of discrete random


distributions: Consolidate your understanding of
measures of centre and variability. (page 518)
Tutorials

We 26 int-0574: Watch a worked example on


calculating variance. (page 520)
We 32 int-0575: Watching a worked example
on constructing a probability distribution table.
(page 524)
Digital doc

Tutorial

Spreadsheet 100: Investigate probability


distributions. (page 526)

Digital docs

Digital doc

We 22 int-0573: Watch a worked example on


calculating measures of centre. (page 512)
SkillSHEET 10.1: Practise finding the expected
value of a function of a random variable. (page 517)
WorkSHEET 10.2: Calculate probabilities, expected
values and variance for discrete probability
distributions. (page 518)

536

10D

Chapter review

Test Yourself: Take the end-of-chapter test to test


your progress. (page 535)
To access eBookPLUS activities, log on to
www.jacplus.com.au

maths Quest 12 mathematical methods CaS for the Ti-nspire

11

11A The binomial distribution


11B Problems involving the binomial
distribution for multiple probabilities
11C Markov chains and transition matrices
11D Expected value, variance and standard
deviation of the binomial distribution

The binomial
distribution
areaS oF STudy

Random variables, including:


calculation and interpretation of the expected
value, variance and standard deviation of a
random variable
Bernoulli trials and two-state Markov
chains, including the length of run in a
sequence, steady values for a Markov
chain (familiarity with the use of transition
matrices to compute values of a Markov
chain will be assumed)
Discrete random variables, including:
specification of probability distributions
for discrete random variables using graphs,
tables and probability functions

interpretation and use of mean (), median,


mode, variance ( 2) and standard deviation
of a discrete random variable
the binomial distribution, Bi(n, p), as an
example of a probability distribution for
a discrete random variable (students are
expected to be familiar with the binomial
theorem and related binomial expansions)
the effect of variation in the value(s) of defining
parameters on the graph of a given probability
function for a discrete random variable
probabilities for specific values of a random
variable and intervals defined in terms of
a random variable, including conditional
probability
eBook plus

11a

The binomial distribution

Digital doc

10 Quick Questions

The binomial distribution is an example of a particular type of discrete probability distribution.


It has relevance and importance in many real-life everyday applications. This particular branch
of mathematics moves away from the textbook and the classroom and into the areas of medical
research, simulation activities and business applications such as quality control.
The binomial distribution may be referred to as a Bernoulli distribution, and the
trials conducted are known as Bernoulli trials. They were named in honour of the Swiss
mathematician Jakob Bernoulli (16541705).

Bernoulli trials and sequences


A Bernoulli trial is an experiment in which the outcome is either a success or a failure.
A Bernoulli sequence is a sequence of Bernoulli trials in which:
1. the probability of each possible outcome is independent of the results of the previous trial
2. the probability of each possible outcome is the same for each trial.
Note: In a Bernoulli sequence, the number of successes follows the binomial distribution.

Chapter 11

The binomial distribution

537

Worked Example 1

Determine which of the following sequences can be defined as Bernoulli sequences.


a Rolling an 8-sided die numbered 1 to 8 forty times and recording the number of 6s obtained
b Drawing a card from a fair deck with replacement and recording the number of aces
c Rolling a die 60 times and recording the number that is obtained
Think

Write

a Check that all the characteristics have been

Yes, this is an example of a Bernoulli


sequence, as there are two possible
outcomes for each trial (success is
obtaining a 6 and failure is not
obtaining a 6), the outcome of each trial
is independent of the outcome of previous
trials, and the probability of success is the
same for each trial.

Yes, this is an example of a Bernoulli


sequence, as there are two possible outcomes
for each trial (success is obtaining an
ace and failure is not obtaining an ace),
the outcome of each trial is independent
of the outcome of previous trials, and the
probability of success is the same for each
trial (as each time a card is drawn it is
replaced).

This is not an example of a Bernoulli


sequence, since success has not been
defined.

satisfied for a Bernoulli sequence.

b Check that all the characteristics have been

satisfied for a Bernoulli sequence.

c Check that all the characteristics have been

satisfied for a Bernoulli sequence.

If X represents a random variable that has a binomial distribution, then it can be expressed as:
X Bi(n, p) or X B(n, p)
Translated into words, X Bi(n, p) means that X follows a binomial distribution with
parameters n (the number of trials) and p (the probability of success).
Consider the experiment where a fair die is rolled four times. If X represents the number
of times a 3 appears uppermost, then X is a binomial variable. Obtaining a 3 will represent a
success and all other values will represent a failure. The die is rolled four times so the number
of trials, n, equals 4 and the probability, p, of obtaining a 3 is equal to 16 . Using the shorthand
notation, X Bi(n, p) becomes X ~ Bi(4, 16 ).
We will now determine the probability of a 3 appearing uppermost 0, 1, 2, 3 and 4 times.
Obtaining 3 is defined as a success and is denoted by S. All other numbers are defined as a
failure and are denoted by F. The possible outcomes are listed in the table below.

538

Occurrence of 3

Possible outcomes

FFFF

SFFF FSFF FFSF


FFFS

Maths Quest 12 Mathematical Methods CAS for the TI-Nspire

Probability
1
4

( 65 )

625
= 1296

500
( 65 ) ( 16 ) = 1296
3

q4
4q3p

Occurrence of 3

Possible outcomes

Probability

SSFF SFSF SFFS


FSSF FSFS FFSS

( 65 ) ( 16 )

SSSF SSFS SFSS FSSS

( 65 )( 16 )

SSSS

( 16 )

150
= 1296

20
= 1296

1
= 1296

6q2p2
4qp3
p4

It is interesting to note that the binomial probability distribution is closely related to the
binomial theorem (see far right-hand column). Furthermore, if we examine the coefficients of
the terms that is, 1, 4, 6, 4, 1 it is evident that they are the entries of Pascals triangle.
This procedure for determining the individual probabilities can become tedious, particularly
once the number of trials increases. Hence if X is a binomial random variable, its probability is
defined as follows.
Pr(X = x) = nCx pxqn x where x = 0, 1, 2, . . . n. That is:
x = the occurrence of the successful outcome.
The formula may also be written as
Pr(X = x) = nCx px(1 p)n x where x = 0, 1, 2, . . . n
Here, the probability of failure, q, is replaced by 1 p.
nC represents the number of ways that x different outcomes can be obtained from n trials. It can
x
n
n!
also be written as and has the formula nCx =
.
x !( n x )!
x
Since this is a probability distribution, we would expect that the sum of the probabilities is 1.
Therefore, for the previous example:
Pr(X = x) = Pr(X = 0) + Pr(X = 1) + Pr(X = 2) + Pr(X = 3) + Pr(X = 4)
625
500
150
20
1
= 1296
+ 1296
+ 1296
+ 1296
+ 1296

=1
Worked example 2

eBook plus

A binomial variable, X, has the probability function Pr(X = x) = 6Cx(0.4)x(0.6)6 x


where x = 0, 1, . . . 6. Find:
a n, the number of trials
b p, the probability of success
c the probability distribution for x as a table.
Think

Tutorial

int-0576
Worked example 2

WriTe

Method 1: Using the rule


a Obtain the relevant information from

the given function. The number of


trials, n, is the value of the number
located at the top left-hand corner of C.
b Obtain the relevant information

from the given function. The


probability of success, p, is the
value in the first bracket.

a n=6

b p = 0.4

Chapter 11

The binomial distribution

539

Pr(X = x) = 6Cx(0.4)x(0.6)6 x

Write the rule of the given


probability function.

Substitute x = 0 into the rule.

Pr(X = 0) = 6C0(0.4)0(0.6)6

Evaluate.

Substitute x = 1 into the rule.

Pr(X = 1) = 6C1(0.4)(0.6)5

Evaluate.

Substitute x = 2 into the rule.

Pr(X = 2) = 6C2(0.4)2(0.6)4

Evaluate.

Substitute x = 3 into the rule.

Pr(X = 3) = 6C3(0.4)3(0.6)3

Evaluate.

10

Substitute x = 4 into the rule.

Pr(X = 4) = 6C4(0.4)4(0.6)2

11

Evaluate.

12

Substitute x = 5 into the rule.

Pr(X = 5) = 6C5(0.4)5(0.6)

13

Evaluate.

14

Substitute x = 6 into the rule.

Pr(X = 6) = 6C6(0.4)6(0.6)0

15

Evaluate.

16

Place values in a table.


Note: The table at right clearly
displays the probability
distribution of the given
function. It also shows that the
individual probabilities sum to 1.

= 1 1 0.046 656
= 0.046 656
= 6 0.4 0.077 76
= 0.186 624
= 15 0.16 0.1296
= 0.311 04
= 20 0.064 0.216
= 0.276 48
= 15 0.0256 0.36
= 0.138 24
= 6 0.010 24 0.6
= 0.036 864
= 1 0.004 096 1
= 0.004 096
x

540

Pr(X = x) 0.0467 0.1866 0.3110 0.2765 0.1382 0.0369 0.0041

Method 2: Using a CAS Calculator


1

A CAS calculator
can also be used to
find the probability distribution.
n=6
p = 0.4
X ~ Bi(6, 0.4)
On a Calculator page, press:
MENU b
5:Probability 5
5:Distributions 5
D:BinomialPdf D
Enter values of n and p, using
Tab e to move between fields.

Maths Quest 12 Mathematical Methods CAS for the TI-Nspire

Press ENTER .
Scroll across to see all the
probabilities.

Write the distribution in a table.

Pr(X = x)

Pr(X = 0)

0.046 656

Pr(X = 1)

0.186 624

Pr(X = 2)

0.311 04

Pr(X = 3)

0.276 48

Pr(X = 4)

0.138 24

Pr(X = 5)

0.036 864

Pr(X = 6)

0.004 096

Worked Example 3

A fair die is rolled five times. Find the probability of obtaining:


a exactly four 5s
b exactly two even numbers
c all results greater than 3
d a 5 on the first roll only
e a 5 on the second and third roll only.
Think
a

Write

Check that all the characteristics have


been satisfied for a binomial distribution.

a Binomial distribution n independent trials

and two outcomes, fixed p and q.

Write the rule for the binomial probability


distribution.

Pr(X = x) = nCxpxqn x

Define and assign values to variables. The


number of 5s obtained is exactly four.

n=5
Let X = the number of 5s obtained.
That is, x = 4
p = probability of a 5 = 16
q = 65

Substitute the values into the rule.

Pr( X = 4) = 5C4

Evaluate.

( 16 ) ( 65 )
4

1
= 5 1296

5
6

25
7776

Chapter 11 The binomial distribution

541

Define and assign values to variables. Two


even numbers means we have x = 2.

b n = 5

Let X = the number of even numbers.


That is, x = 2
p = probability of an even number = 12
q=

Substitute the values into the rule.

Evaluate.

1
2

Pr( X = 2) = 5C2

= 10

Simplify.

Define and assign values to variables.


We require 5 occasions when results are
greater than 3.

Evaluate.

1
8

= 165

1
2

Pr( X > 3) = Pr( X = 5) = 5C5


=1
=

1
32

( 12 ) ( 12 )
5

1
32

d Pr(5 on the first roll only)

binomial rule is not required.

= Pr(SFFFF)

= 16 65 65 65
=

e Since a specific order is required here, the

Let X = values greater than 3.


That is, x = 5
Three outcomes 4, 5, 6 out of six are
greater than 3.
p = probability of number greater than
3 = 12


d Since a specific order is required here, the

1
4

c n = 5

q=
Substitute the values into the rule.

= 10
32

( 12 ) ( 12 )

5
6

625
7776

e Pr(5 on the second and third roll only)

binomial rule is not required.

= Pr(FSSFF)

= 65 16 16 65
=

5
6

125
7776

Note: If the rule for the binomial probability distribution were to be used in part d, it would
625
provide an answer of 5 7776
= 3125
. This answer gives the probability of obtaining a 5 once on
7776
any of the five trials, not necessarily on the first roll only.
Hence, if a specific order is required the rule for the binomial probability
distribution should not be used.
Similarly in part e the rule would produce an answer of 10
of obtaining a 5 twice on any of the five trials.

542

Maths Quest 12 Mathematical Methods CAS for the TI-Nspire

125
7776

= 1250
, giving the probability
7776

Worked Example 4

A new drug for hay fever is known to be successful in 40% of cases. Ten hay fever sufferers take part
in the testing of the drug. Find the probability, correct to 4 decimal places, that:
a four people are cured
b no people are cured
c all 10 are cured.
a

Think

Write

a This is a binomial distribution with n independent


Check that all the characteristics have
been satisfied for a binomial distribution.
trials and two outcomes, p and q.

2
3

Write the rule for the binomial


probability distribution.
Define and assign values to variables.

Substitute the values into the rule.

Evaluate.

Round the answer to 4 decimal places.

Note: A CAS calculator can also be


used to calculate the probability for a
particular x-value.
Find the number of trials, n, the
probability of success, p, and the x-value
you want to find the probability of.
On a Calculator page, press:
MENU b
5:Probability 5
5:Distributions 5
D:Binomial Pdf D
Enter values of n, p and x, using
Tab e to move between fields.

Pr(X = x) = nCxpxqn x
n = 10
Let X = the number of people cured, therefore x = 4
p = 0.4
q = 0.6
Pr(X = 4) = 10C4(0.4)4 (0.6)6


= 210 0.0256 0.046 656


= 0.250 822 656
0.2508

n = 10
p = 0.4
X ~ Bi(10, 0.4)
X = the number of people cured; therefore x = 4.

Press ENTER .

Chapter 11 The binomial distribution

543

9
10

Write the solution.

Pr(X = 4) = binomPdf(10, 0.4, 4)


= 0.250 823

Answer the question and round to


4 decimal places.

The probability that 4 people are cured is 0.2508.


b X ~ Bi(10, 0.4)

Define and assign values to


variables.

As X = the number of people cured, therefore x = 0.

Repeat as above using the CAS


calculator to find Pr(X = 0).

Pr(X = 0) = binomPdf(10, 0.4, 0)


= 0.006 046 6

Answer the question and round to


4 decimal places.

The probability that no people are cured is 0.0060.

Define and assign values to


variables.

c X ~ Bi(10, 0.4)

As X = the number of people cured, therefore x = 10.

Repeat as above using the CAS


calculator to find Pr(X = 10).

Pr(X = 10) = binomPdf(10, 0.4, 10)


= 0.000 104 86

Answer the question and round to


4 decimal places.

The probability that no people are cured is 0.0001.

Worked example 5

eBook plus

Grant is a keen darts player and knows that his chance of scoring a bullseye on any
Tutorial
one throw is 0.3.
int-0577
a If Grant takes 6 shots at the target, find the probability, correct to 4 decimal Worked example 5
places, that he:
i misses the bullseye each time
ii hits the bullseye at least once.
b Find the number of throws Grant would need to ensure a probability of more than 0.8 of scoring
at least one bullseye.
Think
a

2
3

544

WriTe

Check that all the characteristics


have been satisfied for a binomial
distribution.
Write the rule for the binomial
probability distribution.
Define and assign values to
variables.

i This is a binomial distribution with n independent

trials and two outcomes, p and q.


Pr(X = x) = nCx pxqn x

Substitute the values into the rule.

n=6
Let X = the number of bullseyes, therefore
x = 0, 1, 2, 3, 4, 5, 6
p = 0.3
q = 0.7
Pr(X = x) = nCx pxqn x

Evaluate and round answer to


4 decimal places.
Note: Check the answer using
binomPdf(6,0.3,0).

Pr(X = 0) = 6C0(0.3)0(0.7)6
= 1 1 0.117 649
= 0.117 649
0.1176

Answer the question.

The probability that Grant misses the bullseye each


time is 0.1176.

maths Quest 12 mathematical methods CaS for the Ti-nspire

ii

Define and assign values to variables.


Note: Pr(X 1) would involve adding
probabilities from Pr(X = 1) to
Pr(X = 6). Using the fact that
Pr(X 1) = 1 Pr(X = 0) allows us to
solve the problem using fewer terms.
Substitute the values into
the rule.

ii n = 6
Let X = the number of bullseyes, therefore
x = 0, 1, 2, 3, 4, 5, 6
p = 0.3
q = 0.7
Pr(X 1) = Pr(X = 1) + Pr(X = 2)
+ . . . + Pr(X = 6)

= 1 Pr(X = 0)

= 1 6C0(0.3)0(0.7)6
= 1 0.117 649
= 0.882 351
0.8824

Evaluate and round answer to


4 decimal places.
Note: Check the answer using
1 binomPdf(6,0.3,0).

Answer the question.

The probability that Grant hits the bullseye at least


once is 0.8824.

Define and assign values to


variables.

Write the rule as required.

Substitute the values into the rule.

1 nC0(0.3)0(0.7)n > 0.8

Evaluate by solving for n.

1 0.7n > 0.8


1 0.8 > 0.7n
0.2 > 0.7n
loge (0.2) > loge (0.7n)
loge (0.2) > n loge (0.7)
log e (0.2)
<n
log e (0.7)
n > 4.512 338 026

Interpret the results and answer


the question.

Grant would need to take 5 shots to ensure a


probability of 0.8 of scoring at least one bullseye.

Alternatively, a CAS calculator can


be used to solve the inequation.
On a Calculator page, press:
MENU b
3:Algebra 3
1:Solve 1
Complete the entry line as:
solve(1 0.7n > 0.8,n)
Then press ENTER .

n=?
Let X = the number of bullseyes, therefore
x = 0, 1, 2, 3, 4, 5, 6
p = 0.3
q = 0.7
Pr(X 1) > 0.8
Pr(X 1) = Pr(X = 1) + Pr(X = 2) + ... + Pr(X = 6)
= 1 Pr(X = 0)
1 Pr(X = 0) > 0.8

Chapter 11 The binomial distribution

545

Write the solution.

Solving 1 0.7n > 0.8 for n implies


n > 4.51234

Interpret the results and answer


the question.

Grant would need to take 5 shots to ensure a


probability of 0.8 of scoring at least one bullseye.

Graphs of the binomial distribution


We will now consider the graph of a binomial distribution. If we refer to the example of obtaining a
3 when rolling a die four times (see the table on pages 5389) we note that X ~ Bi(4, 16 ). The
probability distribution of the random variable, X, is given in the table and graph over the page.
Pr(X = x)
0.5

Pr(X = x)

0.4823

0.3858

0.3

0.1157

0.2

0.0154

0.1

0.0008

0.4

0.0

01234

The effect of changing n and p on binomial


distribution graphs
The effect of increasing p can be seen in the series of graphs below.
Pr(X = x)
0.4

Pr(X = x)
0.4
0.3

X ~ Bi(8, 0.2)

Pr(X = x)
0.4
X ~ Bi(8, 0.5)

0.3

0.3 X ~ Bi(8, 0.8)

0.2

0.2

0.2

0.1

0.1

0.1

0.0

0.0

01234 56 78 x

The graph is positively skewed.

01234 56 78 x

The graph is symmetrical. It is also


called a normal distribution curve.

0.0

01234 56 78 x

The graph is negatively skewed.

We will now keep p fixed and vary n.


Pr(X = x)
0.34
0.32
0.30
0.28
0.26
0.24
0.22
0.20
0.18
0.16
0.14
0.12
0.10
0.08
0.06
0.04
0.02
0.00

546

X ~ Bi(5, 0.5)

012 34 5

Pr(X = x)
0.34
0.32
0.30
0.28
0.26
0.24
0.22
0.20
0.18
0.16
0.14
0.12
0.10
0.08
0.06
0.04
0.02
0.00

Maths Quest 12 Mathematical Methods CAS for the TI-Nspire

X ~ Bi(15, 0.5)

0 1 2 3 4 5 6 7 8 9 101112131415

Pr(X = x)
0.34
0.32
0.30
0.28
0.26
0.24
0.22
0.20
0.18
0.16
0.14
0.12
0.10
0.08
0.06
0.04
0.02
0.00

X ~ Bi(25, 0.5)

0 1 2 3 4 5 6 7 8 9 10111213141516171819202122232425

From the preceding graphs it can be seen that:


1. when p = 0.5, the graph is symmetrical
2. as n increases, and the interval between the vertical columns decreases, the graph
approximates a smooth hump or bell shape.
The effects that the parameters n and p have on the binomial probability distribution curve can
be summarised in the following way.
If p < 0.5, the graph is
positively skewed.
Pr(X = x)

If p = 0.5, the graph is


symmetrical.

If p > 0.5, the graph is


negatively skewed.

Pr(X = x)

Pr(X = x)

When n is large and p = 0.5, the vertical columns are closer together and the line graph
becomes a bell-shaped curve or a normal distribution curve.

REMEMBER

1. The binomial distribution is an example of a discrete probability distribution.


2. The binomial distribution may be referred to as a Bernoulli distribution, and the trials
conducted are known as Bernoulli trials.
3. For a sequence to be defined as a Bernoulli sequence, each of the following
characteristics must be satisfied.
(a) Each trial in the sequence must be a Bernoulli trial, that is, there must be only two
possible outcomes for each trial (success and failure).
(b) The probability of success, p, is the same for each trial, and is independent of the
outcome of previous trials. (The probability of failure, q, is equal to 1 p.)
4. X ~ Bi(n, p) denotes that X follows a binomial distribution with parameters n (the
number of independent trials) and p (the probability of success).
5. If X is a binomial random variable, its probability is defined as
Pr(X = x) = nCx pxqn x where x = 0, 1, 2, ... n.
Note: x represents the occurrence of the successful outcomes.

Chapter 11 The binomial distribution

547

6. The parameters n and p affect the binomial probability distribution curve as follows:
(a) If p < 0.5, the graph is positively skewed.
(b) If p = 0.5, the graph is symmetrical or is a normal distribution curve.
(c) If p > 0.5, the graph is negatively skewed.
(d) When n is large and p = 0.5, the interval between the vertical columns decreases
and the graph approximates a smooth hump or bell shape.
exerCiSe

11a

eBook plus
Digital doc

Spreadsheet 003
Binomial
distribution

The binomial distribution


1 We1 Determine which of the following sequences can be defined as a Bernoulli sequence:
a Rolling a die 10 times and recording the number that comes up
b Rolling a die 10 times and recording the number of 3s that come up
c Spinning a spinner numbered 1 to 10 and recording the number that is obtained
d Tossing a coin 15 times and recording the number of tails obtained
e Drawing a card from a fair deck, without replacement, and recording the number of
picture cards
f Drawing a card from a fair deck, with replacement, and recording the number of black cards
g Selecting three marbles from a jar containing three yellow marbles and two black
marbles, without replacement.
2 Evalute the following, correct to 4 decimal places:
b 9C3 (0.1)3 (0.9)6
a 7C2 (0.4)2 (0.6)5
d 8C5 (0.2)5 (0.8)3

9C

( ) ( 23 )

1
7 3

10C (0.5)5 (0.5)5


5
10C (0.15)0 (0.85)10.
0
x) = 5Cx (0.3)x (0.7)5 x

3 We2 A binomial variable, X, has the probability function Pr(X =


where x = 0, 1, . . ., 5. Find:
a n, the number of trials
b p, the probability of success
c the probability distribution for x as a table.
4

Twenty per cent of items made by a


certain machine are defective. The items
are packed and sold in boxes of 5. What is
the probability of 4 items being defective
in a box?

exam Tip
nC
n1

Students should know that nCn = 1 and


= n. Similarly nC0 = 1 and nC1 = n.

[Assessment report 1 2007 VCAA]

Alex lives so close to where she works that she only has a 0.1 chance of being late. What
is the probability that she is late on 3 out of 4 days?

Ange has four chances to knock an empty can off a stand by throwing a ball. On each
1
throw, the probability of success is . Find the probability that she will knock the empty can
3
off the stand:
a once

b twice

c at least once.

7 We3
A fair coin is tossed four times. Find the probability of obtaining:
a heads on the first two tosses and tails on the second two
b heads on every roll
c two heads and two tails.
8

548

Peter is quite poor at doing crossword puzzles and the probability of him completing one
is 0.2. Find the probability that:
a of the next three crossword puzzles that he attempts, he is successful in completing two
b he successfully completes the first three crossword puzzles that he tries, but has no luck
on the next one

maths Quest 12 mathematical methods CaS for the Ti-nspire

c he successfully completes the first three crossword puzzles that he tries


d he successfully completes the first three puzzles that he tries given that he was successful
in completing his first two.
9 A weighted coin is biased such that a tail comes up 60% of the time. The coin is tossed five
times. Find the probability of obtaining:
a tails on the first four tosses only
b four tails.
10 WE4 Fifty-five per cent of the local municipality support the local council. If eight people are
selected at random, find the probability, correct to 4 decimal places, that:
a half support the council
b all eight support the council
c five support the council
d three oppose the council.
11 The probability of Colin beating Maria at golf is 0.4. If they play once a week throughout the
entire year and the outcome of each game is independent of any other, find the probability that
they will have won the same number of matches, correct to 4 decimal places.
12 It is known that 5 out of every 8 people eat Superflakes for breakfast. Find the probability that
half of a random sample of 20 people surveyed eat Superflakes, correct to 4 decimal places.
13 On a certain evening, during a ratings
period, two television stations put their
best shows on against each other. The
ratings showed that 39% of people
watched Channel6, while only 30%
of people watched Channel 8. The rest
watched other channels. A random sample
of 10 people were surveyed the next day.
Find the probability, correct to 4 decimal
places, that exactly:
a six watched Channel 6
b four watched Channel 8.
14 Three per cent of items produced by a certain machine are defective. A random sample of
10 items is taken. Find the probability that exactly 10% are defective, correct to 4 decimal places.
15 A new drug being trialled gives 8% of the patients a violent reaction. If 200 patients trial
the drug, find the probability that 12 patients have a violent reaction to the drug, correct to
4 decimal places.
16 A pen company produces 30 000 pens per week. However, of these 30 000 pens, 600 are
defective. Pens are sold in boxes of 20. Find the probability, correct to 4 decimal places, that:
a two defective pens are found in one box
b a box is free from defective pens.
17 A box contains 5 red marbles, 3 blue marbles and 2 yellow marbles. A marble is chosen at
random and replaced. This selection process is completed eight times. Find the probability,
correct to 4 decimal places, that:
a exactly 4 reds are selected
b exactly 2 blues are selected
c no yellows are selected.
18 MC Claires position in the netball team is goal shooter. The probability of her shooting a
goal is 78%. If Claire has 10 attempts at scoring, the probability she will score fewer than
3 goals is:
A 10C3 (0.78)3(0.22)7
B 10C3 (0.78)3(0.22)7 + 10C4 (0.78)4(0.22)6 + ... + (0.78)10
C 10C2 (0.78)2(0.22)8 + 10C1 (0.78)1(0.22)9 + (0.22)10
D 10C2 (0.78)2(0.22)8 + 10C1 (0.78)1(0.22)9 + (0.78)10
E 10C2 (0.22)2(0.78)8 + 10C1 (0.22)1(0.78)9 + (0.78)10

Chapter 11 The binomial distribution

549

19 mC The probability that the temperature in Melbourne will rise above 25 C on any given
summer day, independent of any other summer day, is 0.6. The probability that four days in
a week reach in excess of 25 C is:
A 0.64 0.43
B 7 0.64 0.43
C 47 0.43
D 0.64
E 35 0.64 0.43
20 mC Rachel sits a multiple-choice test containing 20 questions, each with four possible
answers. If she guesses every answer, the probability of Rachel getting 11 questions correct is:
A
D

( ) ( 43 )
9
11
1
3
20 C
11 ( 4 ) ( 4 )
1
20 C
11 4

20

B
E

( ) ( 43 )
9
11
1
3
20 C
10 ( 4 ) ( 4 )
1
20 C
11 4

11

( ) ( 43 )

1
20 C
10 4

11

21 mC A smoke detector has a probability of failing 2% of the time. If a shopping complex


has installed 40 of these smoke detectors, the probability that at least one fails is given by:
A 1 40C1 (0.98)1(0.02)39
B 1 40C1 (0.02)1(0.98)39
C 1 (0.02)40
D 1 (0.98)40
E (0.98)40
22 mC It is found that 3 out of every 10 cars are unroadworthy. Ten cars are selected at
random. The probability that 3 are unroadworthy is:
A 0.009
B 0.2601
C 0.2668
D 0.5
E 1
23 We5 Grant is a keen darts player and
knows that his chance of scoring a
bullseye on any one throw is 0.6.
a If Grant takes 5 shots at the target find
the probability that he:
i misses the bullseye each time
ii hits the bullseye at least once.
b Find the number of throws Grant
would need to ensure a probability of
more than 0.7 of scoring at least one
bullseye.
eBook plus
Digital doc

SkillSHEET 11.1
Solving indicial
equations

24 The chance of winning a major prize in a


raffle is 0.05. Find the number of tickets
required to ensure a probability of more
than 0.6 of winning a major prize at least
once.
25 A darts player knows that her chance of
scoring a bullseye on any one throw is 0.1.
Find the number of turns she would need
to ensure a probability of 0.9 of scoring at
least one bullseye.
26 In Tattslotto, your chance of winning first
division is 1 . Find:
8 145 060

a the number of games you would need to play if you wanted to ensure a more than 50%
chance of winning first division at least once
b the number of tickets you would need to buy for part a if there are 16 games on each
ticket
c the cost of buying these tickets, if they cost $4.10 each.
550

maths Quest 12 mathematical methods CaS for the Ti-nspire

27 The following probability distribution is for p = 0.2 and n = 10.


Pr(X = x)
0.3
0.2
0.1
0.0

0 1 2 3 4 5 6 7 8 9 10

a Find the most likely outcome for x.


b Describe the plot.
eBook plus
Digital doc

Spreadsheet 003

28 From the following binomial distribution tables:


i draw a graph of the probability distribution
ii describe the skewness of each graph.
a X Bi(10, 0.3)
b X Bi(10, 0.5)

Binomial
distribution

c X Bi(10, 0.8)

Pr(X = x)

Pr(X = x)

Pr(X = x)

0.028 25

0.000 98

1 107

0.121 06

0.009 77

4.1 106

0.233 47

0.043 95

0.000 07

0.266 83

0.117 19

0.000 79

0.200 12

0.205 08

0.005 51

0.102 92

0.246 09

0.026 42

0.036 76

0.205 08

0.088 08

0.009 00

0.117 19

0.201 33

0.001 45

0.043 95

0.301 99

0.000 01

0.009 77

0.268 44

10

5.9 106

10

0.000 98

10

0.107 37

29

a Describe the plots of the following binomial probability distributions, without drawing
the graphs.
i n = 25, p = 0.1
ii n = 50, p = 0.5
iii n = 30, p = 0.9
b What effect does p have on the graph of a binomial probability distribution?

30

a Describe the plot of the binomial probability distribution, X Bi(60, 0.5), without
drawing the graph.
b Suggest how the graph might look for a binomial probability distribution with the same p,
but double the value of n.

31

a Describe the plot of the binomial probability distribution, X Bi(100, 0.4), without
drawing the graph.
b Suggest how the graph might look for a binomial probability distribution with the same n,
but double the value of p.

Chapter 11

The binomial distribution

551

32 Describe the skewness of the graphs of the following binomial probability distributions.
a Pr(X = x)

eBook plus
Digital doc

WorkSHEET 11.1

Pr(X = x)

0.4

0.4

0.2

0.2

0.0

0.0

1 2 3 4 5

d Pr(X = x)

Pr(X = x)
0.4

0.4

0.2

0.2

0.0

5 10 15 20 x

0.0

2 4 6 8 10

1 2 3 4

problems involving the binomial


distribution for multiple probabilities

11B

We shall now work with problems involving the binomial distribution for multiple probabilities.

Worked example 6

The binomial variable, X, has the following probability table.


x
Pr(X = x)

0.2311

0.3147

0.3321

0.1061

0.0112

0.0048

a Pr(X > 3)

b Pr(X 4).

Think
a

552

WriTe

Pr (X > 3) means Pr(X = 4) or Pr(X = 5). Add


these probabilities.

Evaluate.

Pr(X 4) would involve adding the


probabilities from Pr(X = 0) to Pr(X = 4).
Using the fact that Pr(X 4) = 1 Pr(X > 4)
allows us to solve the problem using fewer
terms.

Substitute the value into the rule.

= 1 0.0048

Evaluate.

= 0.9952

maths Quest 12 mathematical methods CaS for the Ti-nspire

Pr(X > 3) = Pr(X = 4) + Pr(X = 5)


= 0.0112 + 0.0048
= 0.0160

Pr(X 4) = Pr(X = 0) + . . . + Pr(X = 4)


= 1 Pr(X = 5)

Worked example 7

Find Pr(X 3) if X has a binomial distribution with the probability of success, p, and the number of
trials, n, given by p = 0.3, n = 5.
Think

WriTe

State the probability distribution.

X i(n, p)
X i(5, 0.3)

Write what is required.

Pr(X 3) = Pr(X = 3) + Pr(X = 4) + Pr(X = 5)

Substitute the values for n, p and q into the rule


Pr(X = x) = nCx pxqn x.

= 5C3(0.3)3(0.7)2 + 5C4(0.3)4(0.7)
+ 5C5(0.3)5(0.7)0

Evaluate.

= 10 0.027 0.49 + 5 0.0081 0.7


+ 1 0.002 43 1
= 0.1323 + 0.028 35 + 0.002 43
= 0.163 08

Worked example 8

eBook plus

So Jung has a bag containing 4 red and 3 blue marbles. She selects a marble at
random and then replaces it. She does this 7 times. Find the probability,
correct to 4 decimal places, that:
a at least 5 marbles are red
b greater than 3 are red
c no more than 2 are red.
Think
a

Tutorial

int-0578
Worked example 8

WriTe

State the probability distribution


and define and assign values to
variables.

a Let X = number of red marbles selected

n=7
p = 47

X ~ Bi(7, 47 )
As X = number of red marbles selected, therefore
x = 5.
We want at least 5 red marbles, Pr(X 5)
2

On a Calculator page, press:


MENUb
5:Probability5
5:Distributions5
E:BinomialCdfE
Enter values of n and p, using Tab e
to move between fields.
Enter lower bound as 5 and upper
bound as 7, as the number of trials is 7.

Chapter 11

The binomial distribution

553

Press ENTER .

Pr(X 5) = binomCdf(7, 47 , 5, 7)

Write the solution.

Answer the question and round to


4 decimal places.

Repeat as above, using the CAS


calculator to find Pr(X > 3).
Note: Only inclusive values can be
entered into the CAS calculator,
Pr(X > 3) = Pr(X 4)
Remember that only discrete
values are possible for a binomial
distribution.

Answer the question and round to


4 decimal places.

Repeat as above, using the CAS


calculator to find Pr(X 2).
Note: this time the upper bound will be
2 and the lower bound will be 0.

Write the solution.

The probability that at least 5 red marbles are chosen


is 0.3593.
b Pr(X > 3) = binomCdf(7, 4 , 4, 7)
7

554

Answer the question and round to


4 decimal places.

= 0.65310008

The probability that greater than 3 marbles are chosen


is 0.6531.
c

Pr(X 2) = binomCdf(7, 47 , 0, 2)

= 0.359345

= 0.126584

The probability that no more than 2 marbles are


chosen is 0.1266.

Maths Quest 12 Mathematical Methods CAS for the TI-Nspire

Worked Example 9

X follows a binomial distribution with n = 9, p = 0.4. Find, correct to 4 decimal places:


a Pr(X 7)
b the probability that X is greater than 7 given it is greater than 5; that is,
Pr(X > 7X > 5).
Think
a

write

State the probability


distribution.

X ~ Bi(9, 0.4)

Write what is required.

Pr(X 7) = Pr(X = 7) + Pr(X = 8) + Pr(X = 9)

Substitute the values for n, p


and q into the rule
Pr(X = x) = nCx pxqn x.

Pr(X 7) = 9C7(0.4)7(0.6)2 + 9C8(0.4)8(0.6)


+ 9C9(0.4)9(0.6)0

Evaluate and round the answer


to 4 decimal places.

= 36 1.6384 103 0.36 + 9 6.5536


104 0.6 + 1 2.621 44 104 1

= 0.021 23 + 0.003 54 + 2.621 44 104
= 0.025 034 752

0.0250
Pr(X 7) = binomCdf(9, 0.4, 7, 9)

= 0.0250

Part a can also be solved with


a CAS calculator. (Refer to
worked example 8).
b

a X ~ Bi(n, p)

Write what is required.

Evaluate each probability


individually.
Pr(X > 7) can be obtained from
results in a.

Evaluate

Pr(X > 7) = Pr(X = 8) + Pr(X = 9)



= 3.801 088 103
Pr(X > 5) = Pr(X = 6) + Pr(X = 7) + Pr(X = 8) + Pr(X = 9)

= 9C6(0.4)6(0.6)3 + 0.025 034 752

= 84 4.096 103 0.216 + 0.025 034 752

= 0.993 525 76
Pr( X > 7) 3.801 088 10 3
=
Pr( X > 5)
0.993 525 76

Pr( X > 7)
.
Pr( X > 5)

Round the answer to 4 decimal


places.
Part b can also be solved with
a CAS calculator.

Pr(( X > 7) ( X > 5))


Pr( X > 5)
Pr( X > 7)
=
Pr( X > 5)

b Pr(X > 7 | X > 5) =

= 0.038 258 75

0.0383

Pr(X > 7 | X > 5) =

Pr( X > 7)
Pr( X > 5)
binomCdf(9, 0.4, 7, 9)
binomCdf(9, 0.4, 7, 9 )

= 0.0383

Chapter 11 The binomial distribution

555

Worked example 10

eBook plus

Seventy per cent of all scheduled trains through Westbourne station arrive on time.
Tutorial
If 10 trains go through the station every day, find, correct to 4 decimal places:
int-0579
a the probability that at least 8 trains are on time
Worked example 10
b the probability that at least 8 trains are on time for 9 out of the next 10 days.
Think
a

1
2
3

WriTe

State the probability


distribution.
Write what is required.
Substitute the values of n, p
and q into the rule
Pr(X = x) = nCx pxqn x.
Evaluate and round the
answer to 4 decimal places.

Part a can also be solved with


a CAS calculator. (Refer to
worked example 8).
5 Answer the question.
b 1 State the probability
distribution.
2 Write what is required.
3

Substitute the values of n, p


and q into the rule
Pr(X = x) = nCx pxqn x.

Evaluate and round the


answer to 4 decimal places.
Part b can also be solved with
a CAS calculator.

Answer the question.

a X i(n, p)

X i(10, 0.7)

Pr(X 8) = Pr(X = 8) + Pr(X = 9) + Pr(X = 10)


= 10C8(0.7)8(0.3)2
+ 10C9(0.7)9(0.3)
+ 10C10(0.7)10(0.3)0
= 45 0.057 648 01 0.09
+ 10 0.040 353 607 0.3
+ 1 0.028 247 524 9 1
= 0.233 474 440 5
+ 0.121 060 821
+ 0.028 247 524 9
= 0.382 782 786 4
0.3828
Pr(X 8) = binomCdf(10, 0.7, 8, 10)
= 0.3828
The probability that at least 8 trains are on time is 0.3828.
b X i(n, p)

X i(10, 0.3828)
Pr(X = 9) = 10C9(0.3828)9(0.6172)

= 10 0.000 176 501 08 0.6172


= 0.001 089 364 7
0.0011
Pr(X = 9) = binomPdf(10, 0.3828, 9)
= 0.0011
The probability that at least 8 trains are on time for 9 out
of the next 10 days is 0.0011.

rememBer

When solving problems dealing with the binomial distribution for multiple probabilities
always:
1. define the distribution
2. write what is required
3. write the rule for the binomial probability distribution
4. substitute the values into the given rule and evaluate.

556

maths Quest 12 mathematical methods CaS for the Ti-nspire

exerCiSe

11B

problems involving the binomial distribution


for multiple probabilities
1 Find:
a 4C3(0.4)3 (0.6) + 4C4(0.4)4 (0.6)0
b 5C3(0.6)3 (0.4)2 + 5C4(0.6)4 (0.4) + 5C5(0.6)5 (0.4)0.
2 We6

The binomial variable, X, has the following probability table.

x
Pr(X = x)
Find:
a Pr(X 4)

eBook plus
Digital doc

SkillSHEET 11.2
Multiple
probabilities

0.15

0.3

0.1

0.22

0.15

0.08

b Pr(X > 0)

c Pr(X 4)

3 A binomial variable, X, has the probability function Pr(X = x) =


the probability of success and x = 0, 1, . . ., 5.
Find:
a Pr(X 2)
b Pr(X < 4).

d Pr(X < 2).


5C (0.3)x
x

(0.7)5 x, where x is

4 We7 Find Pr(X 4), correct to 4 decimal places, if X has a binomial distribution with the
probability of success, p, and the number of trials, n, given by:
a p = 0.6, n = 5
b p = 0.5, n = 6
c p = 0.2, n = 7.
5

A fair coin is tossed 5 times. Calculate the probability of obtaining:


a at least one tail
b greater than three tails
c greater than three tails, given that at least one tail was obtained.

1
Marco has a faulty alarm clock and the probability that it sounds in the morning is .
3
Calculate the probability, for the next 4 mornings, that his alarm clock:
a works at least 3 times
b works fewer than 2 times
c works at least 3 times, given that it works at least once.
Generally, 10% of people who enter a modelling contest are male. For a particular
competition, three winners were chosen. What is the probability that less than two females
were chosen?

8 We8 A bag contains 4 red and 2 blue marbles. A marble is selected at random and replaced.
The experiment is repeated 6 times. Find the probability that:
a all 6 selections are red
b at least 2 are red
c not more than 1 is red.
9 It is known that 40% of Victorians play sport regularly. Ten people are selected at random.
Calculate the probability, correct to 4 decimal places, that:
a at least half play sport regularly
b at least nine dont play sport regularly.
10 Surveys have shown that 8 out of 10 VCE students study every night. Six VCE students are
selected at random. Find the probability, correct to 4 decimal places, that, on any one day:
a at least 50% of these students study every night
b less than 3 students study every night.
11 A die is weighted such that Pr ( X = 6) = 12 , Pr ( X = 2) = Pr ( X = 4) = 16 and Pr (X = 1) =
1
Pr ( X = 3) = Pr ( X = 5) = 18
. The die is rolled five times. Calculate the probability, correct to
4 decimal places, of obtaining:
a at least three 6s
b at least two even numbers
c a maximum of two odd numbers.

Chapter 11

The binomial distribution

557

12 WE9 If X is binomially distributed with n = 8 and p = 0.7, find, correct to 4 decimal places:
a Pr(X 7)
b Pr(X > 7 | X > 5).
13 A survey shows that 49% of the public support the current government. Twelve people are
selected at random. Calculate, correct to 4 decimal places:
a the probability that at least 8 support the government
b the probability that at least 10 support the government, given that at least 8 do.
14 MC When Graeme kicks for goal, the probability of his kicking a goal is 0.7. If he has five
kicks at goal, the probability that he will score fewer than two goals is:
A 5C1(0.7)1 (0.3)4 + (0.3)5
B 5C2(0.7)2 (0.3)3
C 5C2(0.7)2 (0.3)3 + 5C1(0.7)1 (0.3)4
D 5C2(0.7)2 (0.3)3 + 5C1(0.7)1 (0.3)4 + (0.3)5
E 1 5C2(0.7)2 (0.3)3
15 MC The proportion of patients who suffer a violent reaction from a new drug being trialled
is p. If 80 patients trial the drug, the probability that one-quarter of the patients have a
violent reaction is:
A 80C25 (p)25(1 p)55
B 80C20 (1 p)20(p)60
C 80C25 (1 p)25(p)55
D 80C20 (p)20(1 p)60
E 80C20 (p)20
16 MC If X is a random variable, binomially distributed with n = 10 and p = k, Pr(X 1) is:
A 1 (1 k)10
B (1 k)10
C 10(k)(1 k)9
D (k)10
E 1 (k)10
17 MC Three per cent of items made by a certain machine are defective. The items are packed
and sold in boxes of 10. If 3 or more are defective, the box can be returned and money
refunded. The chance of being eligible for a refund is:
A 0
B 0.0002
C 0.0036
D 0.0028
E 0.9972
18 MC Long-term statistics show that Silvana wins 60% of her tennis matches. The
probability that she will win at least 80% of her next 10 matches is:
A 0.0061
B 0.0464
C 0.1673
D 0.8327
E 0.9536
19 MC Nineteen out of every 20 cricketers prefer Boundary cricket gear. A squad of
12 cricketers train together. The probability that at least 11 use Boundary gear, given that at
least 10 use it, is:
A 0.5404
B 0.6129
C 0.8816
D 0.8992
E 0.9804
20 A school council, comprising 15 members of the school community, requires a minimum twothirds majority to pass a motion. It is known that 50% of the school community favour a new
uniform. Calculate the probability that the school council will pass a motion in favour of a new
uniform, correct to 4 decimal places.
21 A car insurance salesman knows that he has a good chance of finding customers in the age
group from 18 to 20, as people often buy their first car at this age. Five per cent of all people
in this age group are looking to purchase a car. The salesman questions 30 people in this age
group. Calculate the probability, correct to 4 decimal places, that he will get:
a no more than 3 sales
b at least 3 sales.
22 Police radar camera tests have shown that 1% of all cars drive at over 30km/h above the speed
limit, 2% between 10 km/h and 30 km/h above the limit and 4% below 10km/h over the
limit. In one particular hour, a radar camera tests 50 cars. Caculate the probability, correct to
4 decimal places, that:
a at most, one car is over 30 km/h above the limit
b at most, two cars are between 10 km/h and 30 km/h above the limit
c at most, two cars are below 10 km/h above the limit
d at most, three cars are above the limit.
558

Maths Quest 12 Mathematical Methods CAS for the TI-Nspire

23 An Australian cricketer scores 50 or more in one-third


of all his test match innings. The Australian selectors
are aiming to predict his next 10 innings. Calculate,
correct to 4 decimal places, the probability that he will
score 50 or more on:
a no occasions
b exactly four occasions
c at least two occasions.
24 Two dice are rolled simultaneously and their difference
is recorded. Find the probability, correct to 4 decimal
places, that in 5 rolls:
a a difference of zero occurs at least once
b a difference of 1 occurs at least twice
c a difference of 5 occurs at least once.
25 We10 Eighty per cent of all scheduled trains
through Westbourne station arrive on time. If
10 trains go through the station every day, find:
a the probability that at least 8 trains are on time
b the probability that at least 8 trains are on time for
9 out of the next 10 days.
26 Seventy-five per cent of all scheduled trains
through Westbourne station arrive on time. If
15 trains go through the station every day, find, correct to 4 decimal places:
a the probability that at least 10 trains are on time
b the probability that at least 10 trains are on time for 8 out of the next 10 days.
27 An experiment involves rolling a die 6 times. Calculate, correct to 4 decimal places:
a the probability of obtaining at least four prime numbers
b the probability of obtaining at least four prime numbers on 5 occasions if the experiment
is repeated 8 times.
28 Tennis balls are packed in cans of 6. Five per cent of all balls are made too flat (that is, they
dont bounce high enough). The cans are then packed in boxes of two dozen. Calculate the
probability, correct to 4 decimal places, that:
a a can contains, at most, one flat ball
b a box contains at least 22 cans with a maximum of one flat ball.
eBook plus

11C

markov chains and transition


matrices

Interactivity

int-0256
Markov chains and
transition matrices

Andrei Markov was a Russian mathematician whose name is given to a technique that calculates
probability associated with the state of various transitions. It answers questions such as, What is
the probability that James will be late to work today given that he was late yesterday? or What can be
said about the long-term behaviour of James punctuality?.
A Markov chain is a sequence of repetitions of an experiment in which:
1. The probability of a particular outcome in an experiment is conditional only on the outcome
of the experiment immediately before it.
2. The conditional probabilities of each outcome in a particular experiment are the same every
single time.
A two-state Markov chain is one in which there are only two possible outcomes for each
experiment.

Chapter 11

The binomial distribution

559

Consider a leisure centre that offers aerobics classes and has a gym. Records show that 20%
of the members who use the gym on a particular day will participate in an aerobics class the next
day and 70% of the members who participate in an aerobics class on a particular day will use the
gym the next day. It is also known that 200 members use the leisure centre each day and they all
participate in aerobics classes or use the gym, but not both. On a particular day 150 members
use the gym and 50 members attend an aerobics class. The possible outcomes may be illustrated
on a tree diagram as shown below.
Day 1

Day 2
0.8

Gym

0.2

Aerobics

0.7

Gym

0.3

Aerobics

Gym



Aerobics

Outcome Interpretation
GG The member uses the gym on day 2 given the
member used the gym on day 1.
GA The member attends an aerobics class on day 2 given
the member used the gym on day 1.
AG The member uses the gym on day 2 given the
member attended an aerobics class on day 1.
AA The member attends an aerobics class on day 2 given
the member attended an aerobics class on day 1.

The tree diagram can also be used to calculate how many members use the gym or attend an
aerobics class. From the tree diagram below, it can be seen that on the second day 155 members
use the gym and 45 attend an aerobics class.
Day 1

Gym


Aerobics

Day 2
0.8

Gym

0.2

Aerobics

0.7

Gym

0.3

Aerobics

Outcome Number of members


GG
150 0.8 = 120
GA
150 0.2 = 30
AG 50 0.7 = 35
AA 50 0.3 = 15

The tree diagram may be extended to display the possible outcomes and their respective
probabilities for the third day.
Day 1

Day 2
0.8

Gym

0.2

Aerobics

0.7

Gym

0.3

Aerobics

Gym

Aerobics

Day 3
0.8

Gym

0.2

Aerobics

0.7

Gym

0.3

Aerobics

0.8

Gym

0.2

Aerobics

0.7

Gym

0.3

Aerobics

Outcome
GGG
GGA

Probability
0.8 0.8 = 0.64
0.8 0.2 = 0.16

GAG
GAA

0.2 0.7 = 0.14


0.2 0.3 = 0.06

AGG
AGA

0.7 0.8 = 0.56


0.7 0.2 = 0.14

AAG
AAA

0.3 0.7 = 0.21


0.3 0.3 = 0.09

GGG represents the member using the gym on all three days; Pr(GGG) = 0.64.
If you want to find out the probability of the member using the gym on two out of three days,
three outcomes need to be considered: GGA, GAG and AGG.
Pr(gym 2 out of 3 days) = Pr(GGA) + Pr(GAG) + Pr(AGG)

= 0.16 + 0.14 + 0.56

= 0.86
The tree diagram may be further extended to display the possible outcomes and their
respective probabilities on the fourth day.

560

Maths Quest 12 Mathematical Methods CAS for the TI-Nspire

1
Day





Gym








Aerobics



Day 2

0.8

0.2

Gym

Aerobics

Day 3
0.8
0.2

Aerobics

0.7

Gym

0.3

0.7

0.2
0.7
0.3
0.8
0.2

Gym
Aerobics

0.7
Aerobics
0.3

0.8

Gym

0.2

Aerobics

Gym

0.8

Gym

0.8

Gym

0.2

Aerobics
Gym

0.7
0.3
0.8

0.3

Aerobics

0.7
0.3

Gym
Aerobics

0.2
0.7
0.3

Day 4
Gym
Aerobics
Gym
Aerobics
Gym
Aerobics

Aerobics
Gym
Aerobics
Gym
Aerobics

Outcome
GGGG
GGGA
GGAG
GGAA
GAGG
GAGA
GAAG
GAAA
AGGG
AGGA
AGAG
AGAA
AAGG
AAGA
AAAG
AAAA

Probability
0.8 0.8 0.8 = 0.512
0.8 0.8 0.2 = 0.128
0.8 0.2 0.7 = 0.112
0.8 0.2 0.3 = 0.048
0.2 0.7 0.8 = 0.112
0.2 0.7 0.2 = 0.028
0.2 0.3 0.7 = 0.042
0.2 0.3 0.3 = 0.018
0.7 0.8 0.8 = 0.448
0.7 0.8 0.2 = 0.112
0.7 0.2 0.7 = 0.098
0.7 0.2 0.3 = 0.042
0.3 0.7 0.8 = 0.168
0.3 0.7 0.2 = 0.042
0.3 0.3 0.7 = 0.063
0.3 0.3 0.3 = 0.027

As we extend the analysis to cover a greater number of


days, setting up the tree diagram becomes very messy and time
consuming.
The original information can be set up as a pair of recurrence
relationships. That is, if 20% of the members using the gym
attend an aerobics class the next day, 80% will use the gym
the next day. Furthermore, if 70% of the members attending
an aerobics class use the gym the next day, 30% will attend an
aerobics class the next day.
Let gi = the number of members who use the gym on day i.
Let ai = the number of members who attend an aerobics class on
day i.
gi + 1 = 0.8gi + 0.7ai
and
ai + 1 = 0.2gi + 0.3ai
Number of gym users

Number of aerobics participants

Day i

Day i

gi = 150

ai = 50

Day i + 1

Day i + 1

gi + 1 = 0.8gi + 0.7ai

ai + 1 = 0.2gi + 0.3ai

= 0.8 150 + 0.7 50

= 0.2 150 + 0.3 50

= 155

= 45

Day i + 2

Day i + 2

gi + 2 = 0.8gi + 1 + 0.7ai + 1

ai + 2 = 0.2gi + 1 + 0.3ai + 1

= 0.8 155 + 0.7 45

= 0.2 155 + 0.3 45

= 155.5

= 44.5

Day i + 3

Day i + 3

gi + 3 = 0.8gi + 2 + 0.7ai + 2

ai + 3 = 0.2gi + 2 + 0.3ai + 2

= 0.8 155.5 + 0.7 44.5

= 0.2 155.5 + 0.3 44.5

= 155.55

= 44.45

Chapter 11 The binomial distribution

561

Day i + 4

Day i + 4

gi + 4 = 0.8gi + 3 + 0.7ai + 3

ai + 4 = 0.2gi + 3 + 0.3ai + 3

= 0.8 155.55 + 0.7 44.45

= 0.2 155.55 + 0.3 44.45

= 155.555

= 44.445

This method allows us to clearly see how many members (when rounded to integer values)
are using the gym or attending an aerobics class each day.
Often we are interested in the long-term behaviour (or the steady state, as it is often called)
of a particular situation, in this case how many members will use the gym or attend an aerobics
class. We can determine this by using the following information.
Let g = the number of members who use the gym.
Let a = the number of members who attend an aerobics class.
Total number of members = 200
This gives the equation
g + a = 200
which when rearranged is equal to
a = 200 g
[1]
Also
g = 0.8g + 0.7a
[2]
and
a = 0.2g + 0.3a
[3]
Rearranging equation [2]
g 0.8g = 0.7a

0.2g = 0.7a


Substituting [4] into [2] and solving gives


0.2 g 0.7a
=
0.2
0.2
g = 3.5a
a = 200 3.5a
a + 3.5a = 200
4.5a = 200
400
a=
9

[4]


a = 44.4444 (correct to 4 decimal places)
Substituting a = 44.4444 gives g = 155.5556.
In the long term, 156 members will use the gym and 44 members will attend an aerobics
class.

Worked Example 11

The Nee Islands are very wet. If it is raining on a particular day, the chance that it will rain
the next day is 60%. If it is not raining on a particular day, the chance that it will rain on the
following day is 45%.
a If it is raining on Tuesday, draw a tree diagram to represent the next two days.
b Extending the tree diagram, calculate the probability that, if it is raining on Tuesday, it will also
be raining on Friday of the same week.
Think
a Draw a tree diagram labelling each

Write
Wednesday

branch and place the appropriate


probability along the relevant branch.

0.60

Rain

0.40

Dry

Tuesday

Thursday
0.60

Rain

0.40

Dry

0.45

Rain

0.55

562

Maths Quest 12 Mathematical Methods CAS for the TI-Nspire

Dry

Draw a tree diagram labelling


each branch and place the
appropriate probability along the
relevant branch.
Note: The tree diagram will start
at Tuesday and extend to Friday.

Wednesday
0.60

Rain

Tuesday
0.40

Dry

Thursday
Rain
0.60

0.40

Dry

0.45

Rain

0.55

Dry

0.45
0.55
0.45

0.60
0.40
0.60
0.40

0.55

Friday
Rain
Dry
Rain
Dry
Rain
Dry
Rain
Dry

List each of the outcomes and


calculate the probability for each
individual outcome.

Outcome
RRR
RRD
RDR
RDD
DRR
DRD
DDR
DDD

Probability
0.6 0.6 0.6 = 0.216
0.6 0.6 0.4 = 0.144
0.6 0.4 0.45 = 0.108
0.6 0.4 0.55 = 0.132
0.4 0.45 0.6 = 0.108
0.4 0.45 0.4 = 0.072
0.4 0.55 0.45 = 0.099
0.4 0.55 0.45 = 0.121

Add the probabilities of the


required outcomes.

0.216 + 0.108 + 0.108 + 0.099 = 0.531

Interpret the answer.

The probability that it will be raining on Friday if


it is raining on Tuesday is 0.531. That is, there is a
53.1% chance of raining on Friday given it will rain on
Tuesday.

Worked example 12

eBook plus

Commuters travelling into the centre of Trenchtown use either the bus or the train.
Tutorial
Research has shown that each month 20% of those using the bus switch to train
int-0580
travel and 30% of those using the train switch to bus travel.
Worked example 12
If, at the beginning of January, 4800 people were using the bus and 3600 were
using the train to get into the city, calculate:
a the number of people using the train at the beginning of May
b the number of people using the bus and train in the long term.
Think
a

WriTe

Interpret the given information.

a Each month 20% of bus commuters switch to train

travel and 80% will not switch.


Each month 30% of train commuters switch to bus
travel and 70% will not switch.
2

Define the variables.

Let ti = the number of people using the train at the


beginning of month i.
Let bi = the number of people using the bus at the
beginning of month i.

Set up a pair of recurrence


relationships.

ti + 1 = 0.7ti + 0.2bi and bi + 1 = 0.3ti + 0.8bi

Substitute the given values of


bus and train commuters into the
recurrence relationships.

Chapter 11

The binomial distribution

563

Train travellers
January (month i)
ti = 3600

Bus travellers
January (month i)
bi = 4800

February (month i + 1)
ti + 1 = 0.7ti + 0.2bi

= 0.7 3600 + 0.2 4800

= 3480

February (month i + 1)
bi + 1 = 0.3ti + 0.8bi

= 0.3 3600 + 0.8 4800

= 4920

March (month i + 2)
ti + 2 = 0.7ti + 1 + 0.2bi + 1

= 0.7 3480 + 0.2 4920

= 3420

March (month i + 2)
bi + 2 = 0.3ti + 1 + 0.8bi + 1

= 0.3 3480 + 0.8 4920

= 4980

April (month i + 3)
ti + 3 = 0.7ti + 2 + 0.2bi + 2

= 0.7 3420 + 0.2 4980

= 3390

April (month i + 3)
bi + 3 = 0.3ti + 2 + 0.8bi + 2

= 0.3 3420 + 0.8 4980

= 5010

May (month i + 4)
ti + 4 = 0.7ti + 3 + 0.2bi + 3

= 0.7 3390 + 0.2 5010

= 3375

May (month i + 4)
bi + 4 = 0.3ti + 3 + 0.8bi + 3

= 0.3 3390 + 0.8 5010

= 5025

Evaluate the number of bus and


train commuters at the beginning of
February.

Use the values obtained in step 5 to


calculate the number of bus and train
commuters at the beginning of March.

Repeat the processes involved in


step 5 and 6 until the number of
bus and train commuters at the
beginning of May is obtained.

Answer the question.

Set up and number equations that


relate to:
(i) the number of train commuters
(ii) the number of bus commuters
(iii) the total number of commuters.
Note: Maintain the variables defined
in part a .

564

Rearrange equation [1] so that t is


the subject.
Note: Either variable may be the
subject.
Rearrange equation [3] so that b
can be expressed in terms of t.
Note: Again either variable may be
transposed.

At the beginning of May, the number of people using


the bus and train respectively is 5025 and 3375.
b Let t = number of people using the train.

Let b = number of people using the bus.


Total = 3600 + 4800
t + b = 8400
[1]
t = 0.7t + 0.2b
[2]
b = 0.3t + 0.8b
[3]
t + b = 8400
t = 8400 b

[4]


b = 0.3t + 0.8b
b 0.8b = 0.3t
0.2b = 0.3t
0.2b 0.3t
=

0.2
0.2

b = 1.5t

[5]

Maths Quest 12 Mathematical Methods CAS for the TI-Nspire


t = 8400 b

t = 8400 1.5t
t + 1.5t = 8400

2.5t = 8400
2.5t 8400

=
2.5
2.5

Substitute equation [5] into [4] and


solve for t.

t = 3360

Substitute t into equation [5] and


solve for b.

b = 1.5t
b = 1.5 3360
= 5040

Answer the question.

In the long term, 3360 commuters will travel by train


and 5040 will travel by bus.

Transition matrices
Transition matrices are another technique for solving some Markov chain problems. Transition
matrices are specifically used for problems where you are only interested in the probability of
the final outcome (for example, the probability that a person goes to the gym on the seventh
day). Tree diagrams must still be used to solve problems that take into account the number of
different ways of reaching the final outcome (e.g. the probability that a person goes to the gym
4out of the next 7 days).
Consider again the previous scenario, from page 560, of the probability of using the gym or
attending an aerobics class.
Day 1
Gym

Aerobics

Day 2
0.8

Gym

0.2

Aerobics

0.7

Gym

0.3

Aerobics

This can also be described in a transition probability table:


Gym today

Aerobics
today

Gym
tomorrow

0.8

0.7

Aerobics
tomorrow

0.2

0.3

or

Aerobics
today

Gym today

Aerobics
tomorrow

0.3

0.2

Gym
tomorrow

0.7

0.8

Note: Each column should add up to one, as do the pairs of branches in the given tree diagram.
A transition matrix can be used to simplify calculations involving Markov chains. As
its name suggests, the matrix assists in the calculation of the transition from one state to
the next.
We can convert the above transition probability tables into a transition matrix, T, as shown
below.
A1 G1

G1 A1
G 0.8 0.7
A 0.3 0.2
T = 2
or
T = 2

A 2 0.2 0.3
G 2 0.7 0.8

Chapter 11 The binomial distribution

565

The proportion of the population that use the gym or attend the aerobics class is called the
state of the system, S. It can either relate to specific populations, or the probabilities associated
with the different outcomes.
For our example, the initial system, S0 is given by:
n( using the gym on day 1) 150
n(doing aerobics on day 1) 50

=
or
=

n(doing aerobics onn day 1) 50


n( using the gym onn day 1) 150
n(G 0 ) n( A 0 )
At any time, the state is represented by the column matrix
or
, where n(G) is
n(A 0 ) n(G 0 )
the number of gym users and n(A) is the number of aerobics participants.
There are two different transition and initial state matrices for this problem, so which one
should we choose to use? It does not matter which one we choose, as long as the correct
pairing of T and S0 is selected.
150
0.8 0.7
For example, if we choose T =
.
, S0 must be
50
0.2 0.3
The element in the 1st row 1st column of S0 must represent the initial value for the element in
the 1st row, 1st column of the transition matrix.

These elements must match (i.e.


the probability about gym users
matches with the initial number of
gym users).

Similarly, these elements must


match (i.e. the probability about
aerobics participants matches with
the initial number of aerobics
participants).

0.8 0.7 150


i.e. when multiplying T and S:

0.2 0.3 50

0.8 150 + 0.7 50


=

0.2 150 + 0.3 50

155
=

45

They are then multiplied


together as part of the 1st
element for S0.

n(G1 ) 0.8 0.7 150


Hence, the state after 1 day, S1 is
=

n( A1 ) 0.2 0.3 50

0.8 150 + 0.7 50


=

0.2 150 + 0.3 50

155
=

45

Note: these are the same values as we calculated earlier.


From the above calculation, we can see that S1 = T S0.
If the conditional probabilities remain the same, then a similar equation will express the
transition from any particular state to the next.

566

Maths Quest 12 Mathematical Methods CAS for the TI-Nspire

Therefore, in general, Sn + 1 = T Sn
Also, as, S1 = T S0 and, S2 = T S1,
then, S2 = T T S0
= T2 S0
and, S3 = T T T S0
= T3 S0
In general, Sn = Tn S0
To find the long-term behaviour, or steady state, choose n to be a large number, for example
n= 50, and find S50.
T can be written from the generalised transition probability table below:
Event A1

Event A1

Event A2

Pr(A2 | A1)

Pr(A2 | A1)

Event A2

Pr(A2 | A1)

Pr(A2 | A1)

n( A1 )0 Pr( A1 )0
Pr( A 2 | A1 ) Pr( A 2 | A1 )
T=
and S0 =
or
.
Pr( A 2 | A1 ) Pr( A 2 | A1 )
n( A 2 )0 Pr( A 2 )0

Worked Example 13

Using the above data for attending the gym or aerobics class, find:
a the proportion of people attending the gym and aerobics class on the 5th day
b the number of people attending the gym or aerobics class in the long term.
Think
a

Write

Write down the transition matrix.

Write down a suitable initial state


matrix. In this case, it is the initial
numbers of people attending the
gym and aerobics class.

Identify which state matrix is


required.

On a Calculator page, complete the


entry lines as:

0.8 0.7

0.2 0.3

a T=

150
S0 =

50

As S0 corresponds to day 1, therefore day 5


corresponds to the state matrix S4.

0.8 0.7

t
0.2 0.3
150

s0
50
Press ENTER after each entry.

Chapter 11 The binomial distribution

567

Calculate the proportion of people


attending the gym or aerobics on
the 5th day.
That is, S4 = T4 S0.

Write the solution.

S4 = T4 S0
4

0.8 0.7
150
=

0
.
2
0
.
3

50
155.555
=

44.445

Answer the question (rounding to


the nearest whole number).

Repeat as above using the CAS


calculator to find the long-term
proportion. Choose n = 50.
S50 = T50 S0

Write the solution.

568

Answer the question (rounding to


the nearest whole number).

On the 5th day, there will be 156 people


at the gym and 44 people attending the
aerobics class.
b

S50 = T50 S0

0.8 0.7
=

0.2 0.3

155.555
=

44.445

50

150

50

In the long-term, 156 people attend the gym


and 44 people go to aerobics class.

Maths Quest 12 Mathematical Methods CAS for the TI-Nspire

Worked Example 14

The chance of Jos netball team winning a given game depends on how the team performs in the
previous game. If her team wins, then the chance that it will win the next game is 0.75. If her team
loses, there is only a 0.4 chance that they will win the next match. Given the team wins their first
match, find the probability, that:
a they win two out of their first three games
b they win the 7th game they play, correct to 4 decimal places.
Think
a

Write

Draw a tree diagram labelling each


branch and place the appropriate
probability along the relevant
branch. List the outcomes.

Game 2

Won
game 1

Won

0.75

Outcomes
WWW

0.4

Lost
Won

WWL
WLW

0.6

Lost

WLL

0.25
0.25

Game 3
Won

0.75

Lost

Select the appropriate outcomes


for the required probability.

Pr(win 2 out of 3 games) = Pr(WWL) + Pr(WLW)

Calculate the probabilities by


multiplying along the branches.

Pr(win 2 out of 3 games) = 0.75 0.25 + 0.25 0.4



= 0.1875 + 0.1

= 0.2875

Write down the transition matrix, T.

W1 L1
W2 0.75 0.4
T=

L 2 0.25 0.6

0.75 0.4
T=

0.25 0.6
2

Write down a suitable initial state


matrix, S0.
As the value of the 1st row, 1st
column of T is the probability
associated with winning
consecutive games, the first
element of S0 will be the
probability that the team has won.
We know for certain that Jos
team won the first match, so the
probability must be 1. Therefore,
the probability of the second
element must be 0.
Identify which state matrix is
required.

Pr( Jos team wins 1st game)


S0 =

Pr( Jos team loses 1st game)


1
S0 =
0

As S0 corresponds to game 1, therefore game 7


corresponds to the state matrix S6.

Chapter 11 The binomial distribution

569

On a Calculator page, complete the


entry lines as:
0.75 0.4

t
0.25 0.6
1
s0
0
t 6 s0
Press ENTER after each entry.

Pr(winning the 7th game) = T6 S0

Write the solution.

0.75 0.4
1
=

0.25 0.6
0
0.616 092
=

0.383 908
6

As the answer required is the


probability of winning the 7th
game, we need the first element of
the state matrix. Round the answer
to 4 decimal places.

Pr(winning the 7th game) = 0.6161

REMEMBER

1. A Markov chain is a sequence of repetitions of an experiment in which:


(a) the probability of a particular outcome in an experiment is conditional only on the
result of the outcome immediately before it
(b) the conditional probabilities of each outcome in a particular experiment are the
same every single time.
2. A two-state Markov chain is where there are only two possible outcomes for each
experiment.
3. Problems involving two-state Markov chains can be solved using:
(a) tree diagrams where the probabilities are multiplied along the branches
(b) recurrence relationships
Pr( A 2 | A1 ) Pr( A 2 | A1 )
(c) transition matrices where T =
,
Pr( A 2 | A1 ) Pr( A 2 | A1 )

S0 = the initial state, n( A1 )0


n( A 2 ) 0

Pr ( A )
1 0
or
Pr ( A 2 )0

Sn = Tn S0
To find the long-term proportion or steady state, let n = a large number, relative to the
problem, and solve for Sn.

570

Maths Quest 12 Mathematical Methods CAS for the TI-Nspire

Exercise

11C

Markov chains and transition matrices


1 WE11 Kelly has developed a method for predicting whether or not the surf will be good on a
particular day. If it is good today, there is a 65% chance it will be good tomorrow. If it is poor
today, there is a 45% chance it will be poor tomorrow.
a If the surf is good on Thursday, draw a tree diagram to represent the next two days.
b Extending the tree diagram, calculate the probability that, if the surf is good on Thursday,
it will also be good on Sunday.
2 Mio Custors films have either been hugely successful or have failed miserably at the box
office. It is known that there is a 25% chance of Mios next film being successful if his
previous film was a success. It is also known there is a 62% chance of his next film failing if
the previous film was a failure. He is currently filming a trilogy that will be released over a
period of 3 years.
a What is the probability that the second instalment of the trilogy will be a failure if his
latest film before the trilogy was a success?
b What is the probability that the final instalment of the trilogy will be a failure if his latest
film before the trilogy was a success? Answer to 4 decimal places.
3 In the local cricket competition, teams can use either of two types of ball Kingfisher or Best
Match. At the end of each season, clubs sometimes decide to change the ball they use.
Research suggests that 80% of those using Kingfisher stay with that ball for the next season.
An incomplete tree diagram representing this situation is
0.80
shown at right. Complete the diagram and then answer the
0.20
following questions.
a If a club chooses Best Match one season, what is the
0.70
probability it will choose Best Match the following
0.30
season?
b If a club chooses Best Match one season, what is the probability it will be using
Kingfisher in 3 years time?
4

The probability that Alicia is on time to school in the morning is dependent upon if she is
asleep by 10 pm. If she is asleep before 10 pm, the probability of her being on time to school
is 0.8. If she stays up until after 10 pm the night before, the probability of her being on time
to school is only 0.4. The probability that
Exam tip Multiplication of decimals has not
she is asleep before 10 pm is 0.6.
been
well answered by students on exam questions.
a Calculate the probability that she is on
Conditional probability is also an area that needs
time to school on any given day.
more attention.
b Given that she was on time to school,
[Assessment report 1 2007 VCAA]
find the probability that Alicia went to
bed later than 10 pm.

The Lo Schiavo family take a holiday every summer. They choose between a resort in
Cairns and visiting relatives in Tasmania. If they fly to Cairns one year, the probability of
returning to Cairns the next year is 0.4. If they decide to visit their relatives one summer,
there is only a 0.3 chance of a repeat visit the following year. In a particular year, the
Lo Schiavos visited their relatives in Tasmania. What is the probability that they were
holidaying in Cairns two years later?

Every Sunday night, Priya gets takeaway. She only selects from Chinese takeaway or
fish and chips. If she eats Chinese takeaway one week, the probability of her eating fish
and chips the week after is 0.7. If she ate fish and chips one Sunday, the probability
that she eats Chinese the next Sunday is 0.5. Given that she eats Chinese on a particular
Sunday, calculate the probability that she eats Chinese takeaway on only one of the next
three Sundays.

Chapter 11 The binomial distribution

571

7 WE12 Commuters travelling into the centre of Trenchtown use either the bus or train.
Research has shown that each month 30% of those using the bus switch to train travel and 60%
of those using the train revert to bus travel.
If, at the beginning of January, 5600 people were using the bus and 4900 were using the
train to get into the city, calculate:
a the number of people using the train at the beginning of May
b the number of people using the bus and train in the long term.
8 WE13 Residents of Trenchtown purchase their petrol from either Petes Premium Petrol or
Slick Sams Servo. Research has shown that each month 10% of Petes customers switch to
Slick Sams Servo and 20% of Sams customers switch to Petes Premium Petrol.
If, at the beginning of June, 2800 customers purchased their petrol from Pete and
3100 customers purchased their petrol from Sam, calculate:
a the number of people purchasing petrol from Pete at the beginning of October
b the number of people purchasing petrol from Pete and Sam in the long term.
9 A leisure centre offers aerobics classes and has a
gym. Records show that 10% of the members who
use the gym on a particular day will participate
in an aerobics class the next day, and 70% of the
members who participate in an aerobics class on a
particular day will use the gym the next day. It is
also known that 300 members use the leisure centre
each day and they all participate in aerobics classes
or use the gym, but not both. On a particular day
200 members use the gym and 100 members attend
an aerobics class.
a In the long term, how many people will use the gym?
b In the long term, how many people will attend an aerobics class?
10 MC Records show that if a local football team make the top eight in a particular year,
the chance that they make the top eight in the following year is 70%. If they dont make
the top eight in a particular year, the chance that they make the top eight in the following
year is 40%.
The tree diagram that best represents this situation is:
A

B
0.3 Make
0.7 Make
Make top 8

0.7
,
Don t
make top 8

C
Make top 8

0.4
0.6
0.7
0.3

,
Don t
make top 8

E
Make top 8

0.4
0.6
0.6
0.4

,
Don t
make top 8

572

0.3
0.7

top 8
,
Don t make
top 8

Make top 8

Make
top 8
,
Don t make
top 8

,
Don t
make top 8

Make

top 8
,
Don t make
top 8
Make
top 8
,
Don t make
top 8
Make
top 8
,
Don t make
top 8
Make
top 8
,
Don t make
top 8

Maths Quest 12 Mathematical Methods CAS for the TI-Nspire

0.3

D
Make top 8

0.6
0.4
0.4
0.6

,
Don t
make top 8

0.7
0.3

top 8
,
Don t make
top 8

Make
top 8
,
Don t make
top 8
Make
top 8
,
Don t make
top 8
Make
top 8
,
Don t make
top 8

11 Miya prefers to shop at either Southland or Chadstone each weekend. The place she shops at
depends only on where she shopped the previous time. If she visited Southland one weekend,
the next weekend she goes shopping, the probability of her returning to Southland is 1 . The
4

transition matrix for the probabilities of Miya visiting either Chadstone or Southland given
1 2
the shopping centre she visited the weekend before is 4 5 .
3
3
4 5
a If Miya shops at Chadstone one weekend, what is the probability she shops at Southland
the following weekend?
b Miya does visit Chadstone on a particular weekend. What is the probability, correct to
3 decimal places, that she will be at Chadstone again in four weekends time?
c In the long term, what proportion of weekends, correct to 3 decimal places, does Miya
spend at Chadstone?
eBook plus
Digital doc

WorkSHEET 11.2

11d

12 We 14 The chance of Paul hitting a bullseye in darts is dependent on the success of his
previous throw. If he hits a bullseye, then the chance that his next throw will also be a
bullseye is 0.65. If he misses, though, there is only a 0.15 chance that he will get a bullseye
on his next throw. Given that Pauls first throw is a bullseye, find the probability, correct to
4 decimal places, that:
a he hits the bullseye on his next two throws, but misses on the third
b on his tenth throw, he gets a bullseye.

expected value, variance and


standard deviation of the binomial
distribution
When working with the binomial probability distribution, (like other distributions) it is very
useful to know the expected value (mean), variance and the standard deviation.
The random variable, X, is such that X Bi(8, 0.3) and has the following probability distribution.

x
Pr(X = x)

0.057 65

0.197 65

0.296 48

0.254 12

0.136 14

0.046 68

0.010 00

0.001 22

0.000 07

In chapter 10 we saw that the expected value, E(X) was defined as E( X ) = x Pr( X = x ).
Hence, the expected value for the above table is:
E( X ) = x Pr( X = x ).
= 0 0.057 65 + 1 0.197 65 + 2 0.296 48 + 3 0.254 12 + 4 0.136 14 + 5 0.046 68
+ 6 0.010 00 + 7 0.001 22 + 8 0.000 07
= 0 + 0.197 65 + 0.592 96 + 0.762 36 + 0.544 56 + 0.233 400 + 0.060 00 + 0.008 54
+ 0.000 56
= 2.4
The variance was defined by the rule Var(X) = E(X2) [E(X)]2. Hence, the variance for the
above table is:
Var(X) = E(X2) [E(X)]2
= 02 0.057 65 + 12 0.197 65 + 22 0.296 48 + 32 0.254 12 + 42 0.136 14
+ 52 0.046 68 + 62 0.010 00 + 72 0.001 22 + 82 0.000 07 (2.4)2
= 0 + 0.197 65 + 1.185 92 + 2.287 08 + 2.178 24 + 1.167 00 + 0.360 00 + 0.059 78
+ 0.004 48 (2.4)2
= 7.440 15 (2.4)2
= 1.68

Chapter 11

The binomial distribution

573

The standard deviation was defined by the rule SD( X ) = Var ( X ). Hence the standard
deviation for the above table is:
SD( X ) = Var ( X ) = 1.68 = 1.30
Since the above method for obtaining the expected value, variance and the standard deviation
is tedious and time consuming, a quicker method has been developed to calculate these terms. It
can be shown that if X Bi(n, p) then:
E ( X ) = np

Var ( X ) = npq

SD( X ) = npq

To check that these agree with the previous example, we will substitute the values into the
given rules. When X Bi(8, 0.3), we obtain the following.
The expected value:
E(X) = np

= 8 0.3

= 2.4
The variance:
Var(X) = npq

= 8 0.3 0.7

= 1.68
The standard deviation:

SD( X ) = npq

= 1.68
= 1.30
As can be seen, these values correspond to those obtained earlier. A great deal of time is saved
using these rules and the margin for making mistakes is reduced.

Hence, if X is a random variable and X Bi(n, p) then:



E(X) = np
Var(X) = npq
Note: The distribution must be binomial for these rules to apply.

SD( X ) = npq

Worked Example 15

The random variable X follows a binomial distribution such that X Bi(40, 0.25). Determine the:
a expected value b variance and standard deviation.
Think
a

574

Write
a E(X) = np

Write the rule for the expected value.

List the values for n and p.

n = 40, p = 0.25

Substitute the values into the rule.

E(X) = 40 0.25

Evaluate.

Write the rule for the variance.

Write the values for n, p and q.

n = 10, p = 0.25, q = 0.75

Substitute the values into the rule.

Var(X) = 40 0.25 0.75

Evaluate.

Write the rule for the standard


deviation.

SD(X) = npq

Substitute the value obtained for the


variance and take the square root.

= 7.5

Evaluate.

= 2.74

= 10

b Var(X) = npq

Maths Quest 12 Mathematical Methods CAS for the TI-Nspire

= 7.5

Worked Example 16

A fair die is rolled 15 times. Find:


a the expected number of 3s rolled
b the probability of obtaining more than the expected number of 3s.
Think
a

Write
a E(X) = np

Write the rule for the


expected value.

List the values for n and p.

n = 15, p =

Substitute the values into the


rule.

E(X) = 15 1
6

Evaluate.

State the probability


distribution.

1
6

= 2.5

b X ~ Bi(n, p)

X ~ Bi 15,

Write what is required.


Note: Since X represents the
number of 3s, X can have
only whole number values.

Pr(X > 2.5) = Pr(X 3)


= 1 Pr(X < 3)
= 1 [Pr(X = 0) + Pr(X = 1) + Pr(X = 2)]

Write the rule for the


binomial probability
distribution. Then substitute
the values of n, p and q into
the rule.

14
2
13
0
15
1

1 5
1 5
1 5
= 1 15 C0 + 15C1 + 15C2
6 6
6 6
6 6

Evaluate and round the


answer to 4 decimal places.

= 1 (0.064 905 471 5 + 0.194 716 414 6


+ 0.272 602 980 4)
= 1 0.532 224 866 5
= 0.467 775 133 5
0.4678

Part b can also be solved with a


CAS calculator.

Pr(X > 2.5) = Pr(X 3)


= binomCdf(15, 1 , 3, 15)
= 0.4678

Worked Example 17

A binomial random variable has an expected value of 14.4 and a variance of 8.64. Find:
a the probability of success, p b the number of trials, n.
Think
a

Write

Write what is known and what is


required.

a E(X) = 14.4

Substitute the value of np into the


variance equation.

npq = 8.64
14.4q = 8.64

Var(X) = 8.64
p=?

so np = 14.4
so npq = 8.64

Chapter 11 The binomial distribution

575

8.64
14.4

Transpose the equation to make q


the subject.

q=

Evaluate.

q = 0.6

Solve for p using the relationship


q = 1 p.

Write what is known and what is


required.

Substitute the values into the


equation.

Transpose the equation to make n


the subject.

q=1p
0.6 = 1 p
p = 0.4
b np = 14.4

n=?

where p = 0.4

n 0.4 = 14.4
14.4
0.4
= 36

n=

Worked example 18

eBook plus

A new test designed to assess the reading ability of students entering high school
Tutorial
showed that 10% of the students displayed a reading level that was inadequate to
int-0581
cope with high school.
Worked example 18
a If 400 students are selected at random, find the expected number of students
whose reading level is inadequate to cope with high school.
b Determine the standard deviation of students whose reading level is inadequate to cope with high
school and hence calculate 2.
c Discuss the results obtained in part b.
Think
a

576

WriTe
a E(X) = np

Write the rule for the expected value.

Write the values for n and p.

n = 400, p = 0.1

Substitute the values into the rule.

E(X) = 400 0.1

Evaluate.

Interpret the answer.

Write the rule for the variance.

Write the values for n, p and q.

n = 400, p = 0.1, q = 0.9

Substitute the values into the rule.

Var(X) = 400 0.1 0.9

Evaluate.

Write the rule for the standard


deviation.

Substitute the value obtained for the


variance and take the square root.

= 36

Evaluate.

=6

Calculate 2.

= 40
It is expected that 40 students will have a reading
level that is inadequate to cope with high school.
b Var(X) = npq

= 36
SD(X) = npq

2 = 40 2 6
= 28

maths Quest 12 mathematical methods CaS for the Ti-nspire

Calculate + 2.

Interpret the results obtained in part b.


Note: Recall that for many random
variables, approximately 0.95 of the
probability distribution lies within two
standard deviations of the mean.

+ 2 = 40 + 2 6
= 52
There is a probability of 0.95 that between 28 to
52 students (inclusive) will have a reading level that
is inadequate to cope with high school.

rememBer

If X is a random variable and X Bi(n, p) then:


E(X) = np
Var(X) = npq where q = 1 p
SD(X) = npq
exerCiSe

11d

expected value, variance and standard deviation


of the binomial distribution
In questions 1, 2 and 3, assume we have a binomial distribution with
number of trials, n and probability of success, p, as given.
1 We15a
Determine the mean if:
a n = 10 and p = 0.6
c n = 100 and p = 0.5
2 We15b
Determine the variance if:
a n = 20 and p = 0.6
c n = 25 and p = 0.4

eBook plus
Digital doc

Spreadsheet 003

b n = 8 and p = 0.2
d n = 50 and p =

Binomial distribution

3
4

b n = 15 and p = 0.9
d n = 20 and p =

1
4

3 We15b Determine the standard deviation if:


a n = 10 and p = 0.2
b n = 30 and p = 0.5
c n = 50 and p = 0.7

d n = 72 and p =

2
5

4 A fair coin is tossed 10 times. Find:


a the expected number of heads
b the variance for the number of heads
c the standard deviation for the number of heads.
5 A card is selected at random from a standard playing
pack of 52 and then replaced. This procedure is
completed 20 times. Find:
a the expected number of picture cards
b the variance for the number of picture cards
c the standard deviation for the number of picture
cards.
6 Six out of every 10 cars manufactured are white.
Twenty cars are randomly selected. Find:
a the expected number of white cars
b the variance for the number of white cars
c the standard deviation for the number of white cars.

Chapter 11

The binomial distribution

577

7 WE16 A fair die is rolled 10 times. Find:


a the expected number of 2s rolled
b the probability of obtaining more than the expected number of 2s.
8 A fair die is rolled 15 times. Find:
a the expected number of prime values
b the probability of obtaining more than the expected number of prime values.
9 A mathematics exam contains 27 multiple-choice questions, each with 5 possible answers.
Find:
a the expected number of correct answers if a student guessed each question
b the probability of obtaining more than the expected number of correct answers.
10 A new drug that is being trialled has a success rate of 35%. Of the next 80 patients who trial
the drug, find:
a the expected number of patients who will be cured
b the probability that more than the expected number of patients will be cured.
11

Eighty per cent of rabbits that contract a certain disease will die. If a group of 120 rabbits
contract the disease, how many would you expect to:
a die?
b live?

12 WE17
A binomial random variable has a mean of 10 and a variance of 5. Find:
a the probability of success, p
b the number of trials, n.
13

A binomial random variable has a mean of 12 and a variance of 3. Find:


a the probability of success, p
b the number of trials, n.

14 X is a binomial random variable with a mean of 9 and a variance of 6. Find:


a the probability of success, p
b the number of trials, n
c Pr(X = 10).
15 If X is a binomial random variable with E(X) = 3 and Var(X) = 2.4, find:
a the probability of success, p
b the number of trials, n
c Pr(X = 10)
d Pr(X 2).
16 MC The expected number of heads in 20 tosses of a fair coin is:
A

1
2

B 5

C 10

D 15

E 20

17 MC Jenny is a billiards player who knows from


experience that 7 out of every 10 shots she plays will result
in a ball being potted. If she has 40 shots, the number of
balls she expects to pot is:
A 7
B 14
c 21
d 25
e 28
18 MC The variance of the number of balls that Jenny pots
from her 40 shots in question 17 is:
A 2.898
B 7.3
c 8.4
d 22.2

e 28

19 MC Eighty per cent of children are immunised against a certain disease. A sample of
200 children is taken. The mean and variance of the number of immunised children is:
A 80 and 5.66 respectively
B 80 and 32 respectively
C 100 and 50 respectively
d 160 and 5.66 respectively
E 160 and 32 respectively
20 MC A binomial random variable has a mean of 10 and a variance of 6. The values of n and
p respectively are:
A 5 and 25

578

B 5 and 35

C 20 and 35

Maths Quest 12 Mathematical Methods CAS for the TI-Nspire

D 25 and 25

E 25 and

3
5

21 MC A Bernoulli trial has a probability of success, p. If 5 trials are conducted, the


probability of three successes is:
A p3q2
B p2q3
C 10p2q3
D 10p3(1 p)2
E p3
22 A binomial experiment is completed 20 times, with the expected number of successes being
16. Find:
a the probability of success, p
b the variance
c the standard deviation.
23 A multiple-choice test has 20 questions with five different choices of answer for each question.
If the answers to each question are guessed, find:
a the probability of getting 50% of the questions correct
b the probability of getting at least three correct.
24 Four per cent of pens made at a certain factory do not work. If pens are sold in boxes of
25, find the probability that a box contains more than the expected number of faulty pens.
25 A biased coin is tossed 10 times. Let X be the random variable representing the number of tails
obtained. If X has an expectation of 3, find:
a the probability of obtaining exactly two tails
b the probability of obtaining no more than two tails.
26

Eighty per cent of Melbourne households have DVD players. A random sample of
500 households is taken. How many would you expect to have DVD players?

27 The executive committee of a certain company contains 15 members. Find the probability that
more females than males will hold positions if:
a males and females are equally likely to fill any position
b females have a 55% chance of holding any position
c females have a 45% chance of holding any position.
28

A statistician estimates the probability that a spectator at a Carlton versus Collingwood


AFL match barracks for Carlton is 12 . At an AFL grand final between these two teams there
are 10000 spectators. Find:
a the expected number of Carlton supporters
b the variance of the number of Carlton supporters
c the standard deviation of the number of Carlton supporters.

29

Thirty children are given five different yoghurts to try. The yoghurts are marked A to E,
and each child has to select his or her preferred yoghurt. Each child is equally likely to select
any brand. The company running the tests manufactures yoghurt B.
a How many children would the company expect to pick yoghurt B?
b The tests showed that half of the children selected yoghurt B as their favourite. What does
this tell the company manufacturing this product?

30 The proportion of defective fuses made by a certain company is 0.02. A sample of 30 fuses is
taken for quality control inspection.
a Find the probability that there are no defective fuses in the sample.
b Find the probability that there is only one defective fuse in the sample.
c How many defective fuses would you expect in the sample?
d The hardware chain that sells the fuses will accept the latest batch for sale only if, upon
inspection, there is at most one defective fuse in the sample of 30. What is the probability
that they accept the batch?
e Ten quality control inspections are conducted monthly for the hardware chain. Find the
probability that all of these inspections will result in acceptable batches.

Chapter 11 The binomial distribution

579

eBook plus
Digital doc

Investigation
Winning at
racquetball!

580

31 We18
A new test designed to assess the reading ability of students entering high school
showed that 10% of the students displayed a reading level that was inadequate to cope with
high school.
a If 1600 students are selected at random, find the
expected number of students whose reading level is
inadequate to cope with high school.
b Determine the standard deviation of students whose
reading level is inadequate to cope with high school,
and hence calculate 2.
c Discuss the results obtained in part b.
32 The success rate of a new drug that is being trialled is 70%.
a If 1800 patients are selected at random, find the
expected number of patients cured.
b Determine the standard deviation of patients cured, and
hence calculate 2.
c Discuss the results obtained in part b.

maths Quest 12 mathematical methods CaS for the Ti-nspire

Summary
The binomial distribution

The binomial distribution is an example of a discrete probability distribution.


The binomial distribution may be referred to as a Bernoulli distribution, and the trials conducted are known
as Bernoulli trials.
For a sequence to be defined as a Bernoulli sequence, each of the following characteristics must be satisfied.
1. n independent trials must be conducted.
2. Only two possible outcomes must exist for each trial, that is, success (p) and failure (q).
3. The probability of success, p, is fixed for each trial.
If X represents a random variable that has a binomial distribution, then it can be expressed as X ~ Bi(n, p) or
X ~ B(n, p). This means that X follows a binomial distribution with parameters n (the number of trials) and p
(the probability of success).
If X is a binomial random variable its probability is defined as:
Pr(X = x) = nCxpxqn x where x = 0, 1, 2, ... n
The effects of n and p on binomial distribution graphs

The parameters n and p affect the binomial probability distribution curve as follows.
1. If p < 0.5, the graph is positively skewed.
2. If p = 0.5, the graph is symmetrical or is a normal distribution curve.
3. If p > 0.5, the graph is negatively skewed.
4. When n is large and p = 0.5, the interval between the vertical columns decreases and the graph
approximates a smooth hump or bell shape.
Problems involving the binomial distribution for multiple probabilities

When solving problems dealing with the binomial distribution for multiple probabilities, always:
1. define the distribution
2. write what is required
3. write the rule for the binomial probability distribution
4. substitute the values into the given rule and evaluate.
Markov chains and transition matrices

A Markov chain is a sequence of repetitions of an experiment in which:


1. The probability of a particular outcome in an experiment is conditional only on the result of the outcome
immediately before it.
2. The conditional probabilities of each outcome in a particular experiment are the same every single time.
A two-state Markov chain is where there are only two possible outcomes for each experiment.
Problems involving two-state Markov chains can be solved using:
1. tree diagrams where the probabilities are multiplied along the branches
2. recurrence relationships
Pr( A 2 | A1 ) Pr( A 2 | A1 )
3. transition matrices where T =
,
Pr( A 2 | A1 ) Pr( A 2 | A1 )
n( A )
1 0
S0 = the initial state,
n( A 2 ) 0

Pr ( A )
1 0
or
Pr ( A 2 )0

Sn = Tn S0
To find the long-term proportion or steady state, let n = a large number, relative to the problem, and solve
forSn.

Chapter 11 The binomial distribution

581

Expected value, variance and standard deviation of the binomial distribution

If X is a random variable and X Bi(n, p) then:


E(X) = np
Var(X) = npq
SD( X ) = npq
This applies only to a binomial distribution.

582

Maths Quest 12 Mathematical Methods CAS for the TI-Nspire

chapter review
Short answer

1 Zoe tosses a fair coin 5 times. Calculate:


a the probability of obtaining 3 heads
b the probability of obtaining at least 1 head
c the probability of obtaining 4 heads then a tail.
2 Matt is a dedicated football player and knows that
his chance of scoring a goal on any one kick is 0.9.
If he has three shots at goal in a row, calculate the
probability that:
a he scores two goals
b all of his shots miss
c after missing the first, he scores a goal on each
of his next two shots.
3 Nisha is pulling marbles out of a bag containing
3 green and 9 red ones. After each selection she
replaces the marble. In total she selects four
marbles. What is the probability she selects:
a two red marbles?
b two red marbles, given that at least one red
marble was selected?
4 One out of three people read the Bugle newspaper,
while one out of five read the Headline. If four
people are sampled, what is the probability that:
a one of them reads the Headline?
b at least three of them read the Bugle?
5 A barrel contains 100 balls, some of which are
rainbow-coloured. Four balls are randomly selected
from the barrel, with replacement (that is, a ball
is selected, its colour noted, and the ball replaced
before the next ball is selected).
Let p be the proportion of rainbow-coloured
balls in the barrel.
a Write down an expression for the probability
that exactly one of the four balls selected is
rainbow-coloured.
b Use calculus to find the exact value of p for
which this probability will be a maximum.
Exam tip Only 44% of students scored the mark
for part a, and 75% of students received 0/2 for part
b. Students who used q in part a failed to define it. In
part b, sometimes the instruction to use calculus was
not followed.
[Assessment report Mathematical Methods 1 2004]

[ VCAA 2004]

6 Statistics gathered at a local bookstore have found


that 50% of customers purchase crime novels. If

four customers enter the bookstore, what is the


probability that less than two of these customers
buy a crime novel?
Exam tip This was not well answered. Some
students put their probability expressions equal to
one (for example) and tried to solve for p. A
method mark was awarded for indicating that
the derivative of the probability expression was to
be found, set to zero, and solved for p. Some
students simply wrote down an answer (correct or
otherwise) and failed to obtain full marks, as the
question required working to be shown.
[Assessment report 1 2004 VCAA]

7 A cafe prepares its macchiatos with either skinny


or regular milk. It is known that 20% of customers
who have a regular macchiato on a particular day
will have a skinny macchiato the next day. Also,
70% of customers who have a skinny macchiato
on a particular day will have a regular macchiato
the next day. Given a particular customer has
a skinny macchiato on Monday, calculate the
probability:
a they had a regular macchiato on Tuesday and a
skinny macchiato on Wednesday
b they had a skinny macchiato on Wednesday.
8 Hilary loves going to the movies every Friday
night, and prefers either comedy or action films.
If she watches a comedy one week, the probability
that she sees a comedy the following week is 0.1.
If she sees an action movie on a particular Friday,
there is a probability of 0.4 that she will see an
action movie the following Friday. Given that
Hilary saw a comedy on the first Friday in March,
calculate the probability:
a she sees an action film on the third Friday of
the month
b she saw a comedy on the 2nd Friday, given that
she saw an action film on the third Friday of
the month.
9 Maria and Patrick love to play a game of tennis
each week. The probability of Maria beating
Patrick if she wins the previous week is 0.7;
however, if she loses a match, the chance of her
beating him the following week is only 0.2. Given
that Maria beats Patrick in their first match of
the year, calculate the probability that she will
beat him exactly one more time in the next three
weeks.

Chapter 11 The binomial distribution

583

10 Calculate:
i the mean ii the variance
for the binomial random variables with n and p
given by:
a n = 100 and p = 0.5 b n = 50 and p = 0.8.
11 A binomial random variable has a mean of 10 and
variance of 8. Calculate:
a the probability of success, p
b the number of trials, n.
12 A mathematics exam contains 40 multiple-choice
questions, each with 5 possible answers. Calculate:
a the expected number of correct answers if a
student guessed each question
b the variance of the number of correct answers
if a student guessed each question.
Multiple choice

1 Three out of every 7 students ride bikes to school.


Twenty students are randomly selected. The
probability that 8 of these rode to school today is:
A 0.0951
B 0.1426
C 0.1738
D 0.3178
E 0.4916
2 An unbiased 8-sided die is rolled 12 times. The
probability of obtaining three results greater than
5 is:
A 0.1135
B 0.1688
C 0.2188
D 0.2279
E 0.2824
3 Which of the following does not represent a
Bernoulli sequence?
A Rolling a die 10 times and recording the
number of 5s
B Rolling a die 50 times and recording the results
C Spinning a 7-sided spinner and recording the
number of 4s obtained
D Surveying 100 people and asking them if they
eat Superflakes cereal
E Drawing a card with replacement and recording
the number of red cards obtained
4 The probability that the temperature in Melbourne
will rise above 25C on any given summer day,
independent of any other summer day, is 0.65. The
probability that 3 days in a week reach in excess of
25C is:
A 0.653
B 37 0.354
C 7 0.653 0.44
E 0.653 0.354

D 35 0.653 0.354

5 A fair die is rolled 10 times. The probability of


obtaining 4 even numbers is:
A 0.0004
B 0.2051
C 0.0009
D 0.7949
E 0.2461
584

6 If the same die from question 5 is rolled 10 times, the


probability of obtaining at least 8 even numbers is:
A 0.0107
B 0.0390
C 0.0547
D 0.9453
E 0.9893
7 The probability of Sam beating Abby in a game of
cards is 0.36. Abby and Sam decide to play a game
every day for n days. What is the fewest number of
games they need to play to ensure the probability of
Sam winning at least once is greater than 0.85?
A 3
B 4
C 5
D 6
E 7
8 One in every 50 apples sold at Grubby Grannys
Greengrocers has worms in it. If I buy a box of
100 apples, the probability of at least three apples
containing worms is:
A 0.3233
B 0.3781
C 0.5
D 0.6219
E 0.6867
9 X is a random variable, binomially distributed with
n = 20 and p = 37 . Pr(X 1) is:
A 1
D

( 37 )

( 47 )
20

20

B 1
E

( 37 )

( 47 )

20

C 20

( 37 )( 47 )

19

20

10 Claires position in the netball team is goal shooter.


The probability of her shooting a goal is 67%. If
she has 15 attempts at scoring, the probability she
will score at least 7 goals is:
A 15C7 (0.67)7(0.33)8
B 15C7 (0.33)7(0.67)8 + 15C8 (0.33)8(0.67)7 +
+ (0.33)15
C 15C8 (0.67)8(0.33)7 + 15C9 (0.67)9(0.33)6 +
+ (0.67)15
D 15C7 (0.67)7(0.33)8 + 15C8 (0.67)8(0.33)7 +
+ (0.67)15
E (0.33)15 + 15C1 (0.67)(0.33)14 +
+ 15C7 (0.67)7(0.33)8
11 The proportion of patients that suffer a violent
reaction from a new drug being trialled is k. If 60
patients trial the drug, the probability that one-fifth
of the patients have a violent reaction is:
A 60C5 (1 k)5(k)55
B 60C12 (k)12
C 60C5 (k)5(1 k)55
D 60C12 (1 k)12(k)48
E 60C12 (k)12(1 k)48
12 Theo passes three sets of traffic lights on his
way to school each morning. The lights at each
intersection operate independently of each other.
The probability of him having to stop for a red
light is 23 and the probability of passing through
the intersection without stopping is 13 . If Theo

Maths Quest 12 Mathematical Methods CAS for the TI-Nspire

encountered a red light at least once during a


particular trip to school, what is the probability that
he had to stop at exactly two intersections?
A

2

13

6

19

C 3
5

6

13

2
9

13 Consider a Markov chain where the nth state is


given by Sn = Tn S0. Which one of the following
could represent the transition matrix, T, and the
initial state matrix, S0, respectively?
0.1 0.9
1
A
and
0.6 0.4
0
0.6 0.1
0.3
B
and
0.4 0.9
0.7
15
0.6 0.1
C and

30
0.4 0.9
1
0.25 0.75
D and

0
0.4 0.6
0.25 0.45
E
and
0.85 0.65

10

20

14 Kelly has developed a method for predicting


whether or not the surf will be good on a particular
day. If it is good today, there is an 83% chance it
will be good tomorrow. If it is poor today, there
is a 65% chance it will be poor tomorrow. The
probability that the surf will be poor on Thursday
given it was poor on Tuesday is:

A 0.4225
D 0.6620

B 0.4820
E 0.6500

C 0.0595

15 Two fair coins are tossed simultaneously 60 times.


The number of times that they both show heads is
expected to be:
A 10 B 15 C 20 d 30 E 45
16 The variance in the number of heads obtained from
50 tosses of a fair coin is:
A

1
50

2
25

1
4

25
2

E 3.53

17 The expected number of heads in 50 tosses of a fair


coin is:
A

1
2

B 6.25 C 10 d 2.5E 25

18 A variance of 1.35 occurs when:


A n = 20 and p = 0.6 B n = 15 and p = 0.9
C n = 25 and p = 0.4 D n = 20 and p = 14
E n = 50 and p = 0.3
19 The number of defective computer parts in a box of
computer parts ready for sale is a random variable
with a binomial distribution with mean 10 and
standard deviation 3. If a computer part is drawn
at random from the box, the probability that it is
defective is:
A 0.1 B 0.3 C 0.5 D 0.7 E 0.9
Exam tip Only 40% of students answered this
question correctly.
[Assessment report 1 2002]

[ VCAA 2002]

Extended response

1 Speedy Saverios Pizza House claims to cook and deliver 90%


of pizzas within 15minutes of the order being placed. If your
pizza is not delivered within this time, it is free. On one busy
Saturday night, Saverio has to make 150 deliveries.
a How many deliveries are expected to be made within
15 minutes of placing the order?
b What is the probability of receiving a free pizza on this
night?
c If Saverio loses an average of $4 for every late delivery, how
much would he expect to lose on late deliveries this night?
2 Ten per cent of all Olympic athletes are tested for drugs at the conclusion of their event. One per cent of all
athletes use performance enhancing drugs. Of the 1000 Olympic wrestlers competing from all over the world,
Australia sends 10. Find:
a the expected number of Australian wrestlers who are tested for drugs
b the probability that half the Australian wrestlers are tested for drugs
c the probability that at least two Australian wrestlers are tested for drugs
d the expected number of drug users among all wrestlers.

Chapter 11 The binomial distribution

585

3 Five per cent of watches made at a certain factory are defective. Watches are sold to retailers in boxes of 20. Find:
a the expected number of defective watches in each box
b the probability that a box contains more than the expected number of defective watches per box
c the probability of a bad batch, if a bad batch entails more than a quarter of the box being defective.
4 Aiko is a keen basketball player and knows that her chance of scoring a goal on any one throw is 0.65.
a If Aiko takes 6 shots for a goal, find the probability, correct to 4 decimal places, that she:

i misses each time
ii scores a goal at least three times
iii scores a goal five times, given that she scored a goal at least three times.
b Find the number of throws Aiko would need to ensure a probability of more than 0.9 of scoring at least
one goal.
5 Sixty-eight percent of all scheduled trains through Westbourne station arrive on time. If 20 trains go through
the station every day, calculate the probability, correct to 4 decimal places, that:
a no more than 10 trains are on time
b at least 12 trains are on time
c at least 12 trains are on time for 9 out of the next 10 days.
6 The success rate of a new drug being trialled is 60%.
a If 2400 patients are selected at random, find the expected number of patients cured.
b Determine the standard deviation of patients cured and hence calculate 2.
c Interpret the results found in part b.
7 Phil is running a stall at the local Primary School Fair involving lucky dips. It costs $2 to have a go, and
contained in a large box are 80 lucky dips from which to choose. Phil claims that one in 5 lucky dips contains a
prize. By the end of the day, all 80 have been sold. Calculate the probability, correct to 4 decimal places, that:
a the first four people to select a lucky dip dont win a prize, but the next two do
b there are at least 10 winners
c there are no more than 18 prize winners, given that at least 10 people won a prize.
8 Sharelle is the goal shooter for her netball team. During her matches, she has many attempts at scoring a goal.
Assume each attempt at scoring a goal is independent of any other attempt. In the long term, her scoring
rate has been shown to be 80% (that is, 8 out of 10 attempts to score a goal are successful).
a i What is the probability, correct to 4 decimal places, that her first 8 attempts at scoring a goal in a
match are successful?
ii What is the probability, correct to 4 decimal places, that exactly 6 of her first 8 attempts at scoring a
goal in a match are successful?
iii What is the probability, correct to 3 decimal places, that her first 4 attempts at scoring a goal are
successful, given that exactly 6 of her first 8 attempts at scoring a goal in a match are successful?
Assume instead that the success of an attempt to score a goal depends only on the success or otherwise of
her previous attempt at scoring a goal.
If an attempt at scoring a goal in a match is successful, then the probability that her next attempt at
scoring a goal in the match is successful is 0.84. However, if an attempt at scoring a goal in a match is
unsuccessful, then the probability that her next attempt at scoring a goal in the match is successful is 0.64.
Her first attempt at scoring a goal in a match is successful.
b i What is the probability, correct to 4 decimal places, that her next 7 attempts at scoring a goal in the
match will be successful?
ii What is the probability, correct to 4 decimal places, that exactly 2 of her next 3 attempts at scoring a
goal in the match will be successful?
[ VCAA 2008]
9 Every afternoon Anna either goes for a run or a walk. If she goes for a walk one afternoon, the probability that
she goes for a run the next is 0.45, and if she decides to run one afternoon, then the probability of her going
for a walk the next afternoon is 0.8. On Wednesday, Anna decides to go for a walk around the park.
a What is the probability that she goes for a run on each of the next three afternoons?
b What is the probability, correct to 4 decimal places, that over the next three afternoons, she goes for a
run at least once?

586

Maths Quest 12 Mathematical Methods CAS for the TI-Nspire

c What is the probability, correct to 4 decimal places, that on the following Wednesday Anna will decide
to go for a run?
d In the long term, on what proportion of afternoons will she choose to go for a run?
10 A small town has two restaurants Kazs Kitchen and Als Fine Dining. Records show that 30% of
customers who eat at Kazs Kitchen one week will visit Als Fine Dining the next. Also, 40% of customers
who eat at Als Fine Dining during the week will dine at Kazs Kitchen the following week. There are
400 members of the town who regularly dine out once a week. During the first week of July, 300 people visit
Kazs Kitchen and Al has 100 customers at his restaurant.
a How many customers will visit each restaurant in the first week of August?
b In the long run, how many customers will Kaz and Al each get per week?
11 Two companies are competing for the mobile phone market. At the end of January, market research revealed
the following patterns in the subscriptions of mobile phone users.
Of the 400 Tellya customers who were interviewed, 340 were staying with Tellya and 60 were changing
to Yodacall.
Of the 100 Yodacall customers who were interviewed, 90 were staying with Yodacall and 10 were
changing to Tellya.
a Set up a pair of recurrence relationships that describes the given patterns.
b What is the original state of the companies in terms of market share?
c What is the state of each company at the end of the next month (February)?
d What is the state, in terms of market share, of each company 4 months later (May)?
e What is the state, in terms of market share, of each company 7 months later (August)?
f What is the state, in terms of market share, of each company at the end of the following January?
g A company will fail to be viable if its market share falls below 25%. Which, if either, of these companies
will not achieve this market share in the long run?
eBook plus
Digital doc

Test Yourself
Chapter 11

Chapter 11

The binomial distribution

587

eBook plus

aCTiviTieS

Chapter opener
Digital doc

10 Quick Questions: Warm up with ten quick


questions on applications of the binomial
distribution. (page 537)
11A

The binomial distribution

Tutorials

We 2 int-0576: Watch a worked example on


indentifying the number of trials in an experiment.
(page 539)
We 5 int-0577: Watch a worked example on
constructing a probability distribution table.
(page 544)
Digital docs

Spreadsheet 003: Investigate the binomial


distribution. (pages 548 and 551)
SkillSHEET 11.1: Practise solving indicial
equations. (page 550)
WorkSHEET 11.1: Recognise Bernoulli sequences
and calculate cumulative and non-cumulative
probabilities. (page 552)
11B

Problems involving the binomial


distribution for multiple probabilities

Tutorials

We 8 int-0578: Watch a worked example on


calculating probabilities using a CAS calculator.
(page 553)
We 10 int-0579: Watch a worked example on
calculating probabilities using the cumulative
binomial distribution. (page 556)
Digital doc

SkillSHEET 11.2: Practise multiple probabilities.


(page 557)
11C

Markov chains and transition matrices

Interactivity int-0256

Tutorial

We 12 int-0580: Watch a worked example on


calculating long-term probabilities. (page 563)
Digital doc

WorkSHEET 11.2: Identify and perform appropriate


techniques to calculate probabilities. (page 573)
11D
Tutorial

We 18 int-0581: Watch a worked example on


calculating the mean, median and mode of a
probability density function. (page 576)
Digital docs

Spreadsheet 003: Investigate the binomial


distribution. (page 577)
Investigation: Winning at racquetball (page 580)
Chapter review
Digital doc

Test Yourself: Take the end-of-chapter test to test


your progress. (page 587)
To access eBookPLUS activities, log on to
www.jacplus.com.au

The binomial distribution: Consolidate your


understanding of the binomial distribution.
(page 559)

588

Expected value, variance and standard


deviation of the binomial distribution

maths Quest 12 mathematical methods CaS for the Ti-nspire

12

12A Continuous random variables


12B Using a probability density function
to find probabilities of continuous
random variables
12C Measures of central tendency
and spread
12D Applications to problem solving
12E The normal distribution
12F The standard normal distribution
12G The inverse cumulative normal
distribution

Continuous
distributions
AREAS OF STUDY

Random variables, including:


the concept of discrete and continuous
random variables
calculation and interpretation of the expected
value, variance and standard deviation of a
random variable (for discrete and continuous
random variables, including consideration of
the connection between these)
calculation and interpretation of central
measures (mean, median, mode)
the property that, for many random variables,
approximately 95% of their probability
distribution is within 2 standard deviations of
the mean
Continuous random variables, including:
the construction of probability density
functions from non-negative continuous
functions of a real variable
the specification of probability distributions
for continuous random variables using
probability density functions

calculation using technology, interpretation


and use of mean (), median, mode,
variance ( 2) and standard deviation of a
continuous random variable
standard normal distribution, N(0, 1), and
transformed normal distributions, N(, 2),
as examples of a probability distribution
for a continuous random variable (use as an
approximation to the binomial distribution is
not required)
the effect of variation in the value(s) of
defining parameters on the graph of a given
probability density function for a continuous
random variable
probabilities for intervals defined in terms
of a random variable, including conditional
probability (students should be familiar with
the use of definite integrals, evaluated by
hand or using technology, for a probability
density function to calculate probabilities;
teachers may also choose to relate this to the
notion of a cumulative distribution function,
although this is not required)
eBook plus

12A

Digital doc

Continuous random
variables

10 Quick Questions

In previous chapters we worked with discrete random variables and the probability distributions
associated with them. We will now focus on continuous random variables and an important
distribution associated with them the normal distribution.
Continuous random variables represent quantities that can be measured and thus may assume
any value in a given range. They include such variables as time, height and weight.

Chapter 12

Continuous distributions

589

Weight (kg)
x

Frequencies
f

40 < 50

50 < 60

10

60 < 70

19

70 < 80

15

80 < 90

90 < 100

Number of students ((ff)


f)

The weights of 50 Year 12 students are displayed in the table below, alongside a histogram.

Total = 50

f
20
18
16
14
12
10
8
6
4
2
0

40 50 60 70 80 90 100 x
Weight (kg)

The frequency of individual weights cannot be determined because the weights have been
grouped into class intervals. This limits the information we are able to extract from the
histogram. For example, to determine the number of students weighing less than 60 kg we
simply add the frequency of the 40 < 50 and 50 < 60 class intervals, that is, 10 + 2 = 12
students. However, we would not be able to determine the weight of students below 75 kg as this
value lies within the class interval rather than being an end point.
Since the value that a continuous random variable can assume is measured in some way, the
exact value cannot be obtained. Hence a weight of 60 kg, if we measure in whole numbers, is
actually between 59.5 and 60.5 kg. Therefore, the probability of a continuous random variable
assuming an exact value is zero. In order to determine the probability of continuous random
variables, a new method must be employed.
If we were to increase the size of the sample of students weights and make the class intervals
very small, then the histogram would become a smooth frequency curve as shown below.
A special scaled version of a smooth frequency curve is the
f x)
f(
probability density function or pdf. The scale is such that the
probability of the random variable lying between certain
values is given by the area between the pdf and the horizontal
axis. Hence, sections such as the shaded region between x = a
and x = b, as shown below right may be regarded as
x
0
probabilities.
The curve is positioned above the x-axis since it represents
a probability distribution in which individual probabilities
f x)
f(
assume a value between 0 and 1.
Hence, the probability of X, a continuous variable, falling
between x = a and x = b is represented by the shaded area
between a probability density function y = f (x), the x-axis and
the lines x = a and x = b. The area is determined by integrating
x
0
a b
f (x) from x = a to x = b. Using mathematical notation, this may
be summarised as
b

Pr(a < x < b) = f ( x ) dx.


a

Properties of a probability density function


1. f (x) 0 for all x, (that is, we never have negative probabilities) and

2. f ( x ) dx = 1 (that is, the sum of all probabilities is equal to one).

590

Maths Quest 12 Mathematical Methods CAS for the TI-Nspire

Continuous distributions
Probability density functions
The domain of a probability density function (pdf) is usually R, that is, the variable is
continuous and can assume any real value (at least in theory).
If we need our function to be non-zero over a particular interval only, say, over (a, b), then we
need to specify that the function is equal to zero everywhere else. In such case, the area under
b

the curve between x = a and x = b must be equal to one. That is, f ( x ) dx = 1.


a

WORKED EXAMPLE 1

eBoo
k plus
eBook

Sketch the graph of each of the following and state whether each function may be
a probability density function (pdf).

Tutorial

int-0582
Worked example 1

4 x, 1 x 2
0, elsewhere

a f (x
(x) =

10 x 4, 0 x 1
0, eelsewhere

b f (x
(x) =

0 . 5, 1 x 1
0, elsewhere

c f (x
(x) =
THINK
a

WRITE

The graph of f (x) is a straight line with


gradient 4 over the domain [1, 2]. When x = 1,
y = 4 1 = 4, and when x = 2, y = 4 2 = 8.
So the end points of the line are (1, 4) and
(2, 8). Everywhere else, the function is zero.

y
(2, 8)

8
4

(1, 4)

For a function to be a pdf, it has to be positive


or zero over the entire domain. State whether
this condition is satisfied by inspecting the
graph of f (x).

f (x) 0 for all x.

The total area under the graph of a pdf must


be equal to one. Check whether this condition
is observed by finding the area under f (x), and
comment on the result. (This can be done by
integration.)

A = 4 xdx
xd

4x2
=

2 1
= 2 x 2

= 2 (2)2 2 (1)2
=6
Area under the curve 1
4

Although the first condition for the function


to be a pdf is observed, the second one is not.
State your conclusion.

f (x) could not be a probability density


function.

Chapter 12

Continuous distributions

591

The graph of f (x) is a straight line over


the domain [0, 1]. Its gradient is 10 and its
y-intercept is at y = 4. When x = 0, y = 4,
and when x = 1, y = 10 1 4 = 6. So the
end points of the line are (0, 4) and (1, 6).
Everywhere else, the function is zero.

y
(1, 6)

(0, 4)

For a function to be a pdf, it has to be positive or


zero over the entire domain. State whether this
condition is satisfied by inspecting the graph of f (x).

f (x) is not positive or zero for all x, as part


of the graph is below the x-axis.

As the first condition is not observed, there


is no need to check the area under the curve.
State your conclusion.

f (x) does not represent a probability


density function.

The graph of f (x) is a horizontal line through


y = 0.5 over the domain [1, 1]. Everywhere
else, the function is zero.

y
(0, 0.5)
0

For a function to be a pdf, it has to be positive or


zero over the entire domain. State whether this
condition is satisfied by inspecting the graph of f (x).

f (x) 0 for all x.

The total area under the graph of the pdf


must equal 1. Check whether this condition is
observed by finding the area under f (x), and
comment on the result.

A = 0.5dx
5

Both conditions for the function to be a pdf are


satisfied. State your conclusion.

= 0.5x
5 1
= 0.5 (1) 0.5 (1)
=1
Area under the curve = 1
f (x) could be a probability density
function.

WORKED EXAMPLE 2

Find the value of a so that the given function may be a probability density function (pdf).
f [2, 4] R, f (x
f:
(x) = a(12 3x
3 )
THINK
1

592

For the function to be a pdf, the area


under its graph must be 1. Area is
found by evaluating the definite integral
with terminals being the end points
of the domain. Write the appropriate
statement, equating area to 1.

WRITE
4

2 a(12 3x)dx = 1

Maths Quest 12 Mathematical Methods CAS for the TI-Nspire

Take the coefficient, a, out of the


integral.

a (12 3 x )dx = 1

Antidifferentiate.

3x 2
a 12 x
=1
2 2

Substitute the terminals in and


simplify the left-hand side of the
equation.

3 42
3 22
a 12 4
12 2
=1

2
2

a[(48 24) (24 6)] = 1


a(24 18) = 1
6a = 1
a=

Solve for a.

Sketch the graph of the resultant


function and check whether f (x) 0
for all x.

1
6

1
(12 3x)
6
x
=2 ,2x4
2

f (x) =

y
(2, 1)

1
0

f (x) 0 for all x

REMEMBER

1. If X is a continuous random variable, then the probability of it falling between certain


values is given by the area under a frequency curve, known as a probability density
function or pdf.
2. A pdf must be greater than or equal to zero for all values of x, and the total area under
the curve must be equal to one. That is, f (x) 0 for all x, and

f ( x)dx = 1.

3. The domain of a pdf is usually R, that is, the variable is continuous and can assume
any real value (at least in theory).
4. If we need our function to be non-zero over a particular interval only, say, over
(a, b), then we need to specify that the function is equal to zero everywhere else.
For example,
2
x, 1 x 2
f ( x) = 3
0, elsewhere
er
ere
5. If a pdf is only non-zero over a particular interval (a, b), the area under the curve
b

between x = a and x = b must be equal to one. That is, f ( x )dx = 1.


a

Chapter 12

Continuous distributions

593

EXERCISE

12A

Continuous random variables


1 WE1
Sketch the graph of each of the following functions and state whether each function
may be a probability density function.
1
1

4 x 2, 0 x
2, 0 x
a f (x) =
b f (x) =
2
2
0, elsewhere
0, elsewhere
er
ere
1 x, 1 x 1
c f (x) =
0, elsewhere

3x 2
, 1 x 1

d f (x) = 2
0, elsewhere

x
cos (22 x ),
e f (x) =
4
4
0, elsewhere
er
ere

2 sin ((xx ), 2 x
f f (x) =
2
0, elsewhere
er
ere

1
, 1 x e
g f (x) = x
0, elsewhere
er
ere

2 x 1, 1 x 2
h f (x) =
er
ere
0, elsewhere

2 WE2
For each of the following functions, find the value of a so that the given function
may be a pdf.
a ff: [1, 2] R, f (x) = a(2x
(2 1)
(2x
b ff: [1, 1] R, f (x) = a(10 3x2)
c ff: [0, ] R, f (x) =

a
sin (x)
2

ax 3
4

4, 1] R, f (x) = ax 2 + a

e ff: [0, 2] R, f (x) =

d ff: [2, 3] R, f (x) = aex 2


f ff: [7, 2] R, f (x) = a 2 x
h ff: [3, 4] R, f (x) = ax(x 1)

Find the value of a, such that the function may be a probability density function. Sketch
the graph of the resultant function.
a, 0 x 1

f (x) = ax, 1 < x 2


0, elsewhere
er
ere

4 Find the value of a, such that the function may be a probability density function. Sketch the
graph of the resultant function.
a( x + 2),
), 1 x 0

1
f (x) = a x + 2 , 0 < x 2

2
0, elsewher
erre

5 Find the value of m, such that the function may be a probability density function. Sketch the
graph of the resultant function.
mx 2 , 1 x 3

f (x) = m(12 x ), 3 x 4
0, elsewhere
er
ere

594

Maths Quest 12 Mathematical Methods CAS for the TI-Nspire

x , 1 x k
2
f (x) =
0, elsewhere

7
8
9
10

The diagram at right shows the graph of the function


(_1 , _12)
2

1_
2

(where k > 0).


_1
2

Find the value of k such that the function may be a probability density function. Give your
answer in exact form.
x2
The function ff: [1, n] R, f (x
(x) =
is a probability density function. Find the value of n.
4
(Give the answer in surd form.)
), 0 x 0 . 5
k sin (2 xx),
The function f ( x ) =
is a probability density function. Find
0
,
elsewhere
er
ere

the value of k.
ke x , 0 x llog
og e (3)
The function f ( x ) =
is a probability density function. Find the
0, elsewhere
value of k.
The diagram at right is a graph of rectangular or uniform distribution.
y
a Specify the probability density function of this distribution
in terms of a and b.
b Hence, or otherwise, find the value of p if the probability
density function is defined as
0

1
, p x 30
f (x) = 25
.
0, elsewhere

12B

(k, k)

Using a probability density function


to find probabilities of continuous
random variables
If X is a continuous random variable, then the probability of it falling between x = a and x = b is
given by the area bounded by the probability density function,
the x-axis and the lines x = a and x = b. This area can be
y
easily calculated by evaluating the definite integral
between a and b. That is,
Pr(a x b)
b

Pr(a X b) = f ( x ) dx .
a

As mentioned in chapter 9,
Pr(X > a) = Pr(X a)

f ( x ) dx = 0, so Pr(X = a) = 0.

WORKED EXAMPLE 3

If X is a continuous random variable with the probability density function defined as


1
x 4, 8 x 10
f (x
(x) = 2
0, elsewhere
find:
a Pr(X > 9)

b Pr(8.5 < X < 9)

c Pr(X > 9 X > 8.5).

Chapter 12

Continuous distributions

595

THINK
a

WRITE

Draw the graph of f (x) and shade the


required region. (It is the area under
the curve to the right of x = 9.)

To find the required probability (that


is, to find the shaded area under the
curve), integrate f (x) between x = 9
and x = 10.

Antidifferentiate and simplify.

(10, 1)

(8, 0)

10

10 1

Pr(X > 9) = x 4 ddxx


9 2

10

1 x2

4x
=
2 2
9
10

x2

= 4x
4
9
4

10 2
92

=
4 10 4 9
4
4

Substitute the terminals in and


evaluate to obtain the required
probability.

Show the required region on the


diagram. (It is the area under the
curve between x = 8.5 and x = 9.)

= (25 40) (20.25 36)


= 15 (15.75)
= 0.75
b

y
(10, 1)

(8, 0) 8.5 9

To find the required probability, integrate


f (x) between x = 8.5 and x = 9.

Pr(8.5 < X < 9) =

Antidifferentiate. (Use the result


from part a.)

x2

= 4x
4
8.5

Substitute the terminals in and


evaluate to obtain the required
probability.

1
8.5 2

10

x 4 dx

596

The rule for conditional probability


Pr ( A B)
is Pr(
Pr(A | B) =
. Write the
Pr ( B)
appropriate statement.

92
8.52

= 4 9
4 8.5
4
4

= (20.25 36) (18.0625 34)


= 15.75 (15.9375)
= 0.1875
c Pr(X > 9 | X > 8.5)

Pr[( X > 9) ( X > 8.5))]


Pr(( X > 8.5)
Pr ( X > 9)
=
Pr ( X > 8.5)
=

Maths Quest 12 Mathematical Methods CAS for the TI-Nspire

The expression in the denominator


can be found by adding the
probability of X being between 8.5
and 9 and the probability of X being
greater than 9.

Pr ( X > 9)
Pr (8.5 < X < 9) + Pr(
Pr ( X > 9)

The required probabilities are


already found in parts a and b.
Substitute the numbers in and
evaluate.

0.775
0.1875 + 0.775

0.775
0.9375
= 0.8
=

The required probabilities can also be obtained with the aid of a CAS calculator. Where exact
answers are not required, it is appropriate to use the CAS.
WORKED EXAMPLE 4

logg e ( 0 . 5 x )
, 2 x 2e

A random variable, X, has its frequency curve defined as f ( x ) =


2
0, elsewhere
Calculate the probability, correct to 4 decimal places, that X is:
a less than 4
b between 2.5 and 3.5.
THINK
a

WRITE

The required probability can be


obtained by evaluating the definite
integral of f (x) over the interval
[2, 4].
Note: Remember Pr(X < 4) =
Pr(X 4) because Pr(X = 4) = 0
On a Calculator page, press:
MENU b
4:Calculus 4
3:Integral 3
Complete the entry line as:
4

ln((0.5 x )
dx
2

Press ENTER .
2

Write the solution, rounding to


4 decimal places.

Pr(X < 4) = Pr(2 X < 4)


logg e (0.5 x )
dx
2
= 0.3863
=

Find the required probability as shown


above, this time using the interval
[2.5, 3.5].
Write the solution, rounding to 4 decimal
places.

b Pr(2.5 X 3.5) =

3.5 log
g e (0.5 x )

2.5

dx

= 0.2004

Chapter 12

Continuous distributions

597

In many instances we need to integrate over intervals


involving infinity. In such cases, the limits are evaluated
as shown at right.

Interval

Limit
lim

k f ( x)dx

(, )

(, a)

k k

(a, )

k a

lim

lim

f ( x )dx
f ( x )dx

When evaluating limits involving infinity, it is useful to remember that as a number tends to
infinity, its reciprocal tends to zero. For example, lim 1 = 0 and lim 1 = 0. This concept is
x x
x e x
illustrated in the worked example that follows.
WORKED EXAMPLE 5

eBoo
k plus
eBook
1

1
x
2 , x>0

A random variable, X, has its frequency curve defined as f (x


(x) = 2 e
.
a Draw the graph of f (x
(x).
0, elsewhere

b Show that f (x
(x) is a probability density function.
c Find the probability, correct to 4 decimal places, that X is:
i smaller than 3
ii greater than 2.5
iii greater than 2.5, given that it is smaller than 3.
THINK

WRITE/DRAW

a Draw the graph of f (x).


). It is a decreasing

function with a starting point (0, 1 ) and a


2
horizontal asymptote y = 0.

y
1

(0, 2)

0
b

A pdf must be greater than or equal


to 0 for all values of x. Check
whether this condition is observed by
inspecting the graph of f (x).
Find the total area under the curve by
evaluating the definite integral of f (x).
Note that the interval over which the
integral needs to be evaluated is
[0, ). So, in this case, evaluate
k
lim
f ( x )dx .

k 0

b f (x) 0 for all x

A = lim

k 0

1
e
2

1
= lim e
k 2

= lim [ e
k

1
x
2 d
dx
x

x k
]0
k

1
= lim 1
k
x
e 2 0
3

598

Substitute the terminals in and


evaluate the limit. Remember that

lim 1 = 0.
k k
e2

1 1

= klim
k
e 0
e2
1
=0+
1
=1

Maths Quest 12 Mathematical Methods CAS for the TI-Nspire

Tutorial

int-0583
Worked example 5

Both conditions required for the


function to be a pdf are observed.
State your conclusion.
i The required probability can be

obtained by evaluating the definite


integral of f (x) over the interval
[0, 3]. Use the expression for the
antiderivative found in part b to
speed up your calculations. Give your
answer correct to 4 decimal places.

ii

To find Pr(X > 2.5), evaluate


the integral of f (x) over the
interval [2.5, ]. Alternatively,
find Pr(X < 2.5) and subtract
it from 1. This method avoids
finding limits, as the interval
over which the integral needs
to be evaluated is [0, 2.5], that
is, it does not involve .

Since f (x) 0 for all x and the total area under the
curve is 1, f (x) is a pdf.

c i Pr(X < 3) =

31

0 2

1
x
2

ddxx

1
= x
e 0
1
2


1 1
= 3 0
e
e2
= 0.7769
ii Pr(X > 2.5) = 1 Pr(X < 2.5)

=1

2.5 1

1
x
2

ddxx

2.5

1
= 1 1x
2
e 0


1
1
= 1 2.5 0
2 e
e
= 1 0.7135
= 0.2865

The required probability can


also be obtained by using the
CAS over the interval [2.5, )
On a Calculator page, press:
MENU b
4:Calculus 4
3:Integral 3
Complete the entry line as:

2.5 2 e 2

ddxx

Press ENTER
3

iii

Write the solution, rounding to


4 decimal places.
Note: The answer is the same
as above.
Write the appropriate
statement for the conditional
probability.

Pr(X > 2.5) =

2.5 12 e

1
2

ddxx

= 0.2865
iii Pr(( X > 2.5X < 3) =

Pr[( X > 2.5) ( X < 3))]


Pr(( X < 3)
Pr((2.5 < X < 3)
Pr(( X < 3)

Chapter 12

Continuous distributions

599

Using the CAS as shown


above, find Pr(2.5 < X < 3),
with your interval as [2.5, 3].

Fill in the required values into


the conditional probability
statement. Answer to 4 decimal
places.

Pr(2.5 < X < 3) = 0.06337

Pr(( X > 2.5X < 3) =

Pr((2.5 < X < 3)


Pr(( X < 3)

0.06337
0.7769
= 0.0816
=

WORKED EXAMPLE 6

If X is a continuous random variable with a probability density function given by


4 2 x, 1 x 2
f (x
(x) =
, find the value of a such that Pr(X a) = 0.75, where 1 a 2.
0, elsewhere
THINK
1

WRITE

Pr(X a) = 0.75 means that the area under the


curve of f (x) to the left of a is 0.75, where
1 a 2. Draw a diagram to illustrate the given
information.

(1, 2)

0.75

(2, 0)
x

Copy down the probability statement as given in


the question.

Pr(X a) = 0.75

The required probability can be found by


integrating f (x) over the interval [1, a]. So replace
Pr(X a) with the appropriate integral.

1 ( 4 2 x ) ddxx = 0.775
a

Antidifferentiate and simplify.

2x2
4 x
= 0.775
2 1

4 x x 2 = 0.75
1
5

Substitute the terminals in.

(4a a2) (4 1) = 0.75

The result is a quadratic equation. Use the


quadratic formula to solve for a.

4a a2 3 = 0.75
a2 4a + 3.75 = 0
4 4 2 4 1 3.775
2 1
4 1
=
2
5
3
a = or a =
2
2
a=

600

Maths Quest 12 Mathematical Methods CAS for the TI-Nspire

Since a must be within the interval [1, 2], discard


the first solution.

Since 1 a 2, a = 3 .
2

Quantiles and percentiles


Quantiles and percentiles both define the value of the random variable X, below which a given
proportion of the distribution falls. The only difference is that a quantile is a probability value
expressed as a decimal, while a percentile is expressed as a percentage. For example,
if Pr(X < k) = 0.6, then k is the 0.6 quantile, or the 60th percentile.
Finding a particular quantile or percentile involves solving an equation similar to the one
used in worked example 6. For example, if we want to find the 60th percentile of a continuous
random variable, X, with a probability density function given by f (x) and a X b, solve for k
k

the equation of the form f ( x )dx = 0.6.


a

Note that in worked example 6, we found a such that Pr(X < a) = 0.75. In effect, we were
finding the 0.75 quantile, or the 75th percentile.

REMEMBER

1. If X is a continuous random variable, then the


probability of it falling between x = a and x = b
is given by the area bounded by the probability
density function, the x-axis and the lines x = a
and x = b. That is,
b
Pr(a X b) = f ( x )dx .

y
Pr(a x b)
0

2. Pr(X = a) = 0
Pr(X > a) = Pr(X a)
3. The required probabilities can be calculated by using a CAS calculator and selecting
the integral function from the maths expression template, where f (x) is the function
that needs to be integrated and a and b are the terminals.
4. To integrate over intervals involving infinity, the limits are evaluated as shown.
Interval
(, )
(, a)

Limit
k

f ( x)dx
k k
lim

k k
lim

(a, )

k a
lim

f ( x )dx
f ( x )dx

5. As a number tends to infinity, its reciprocal tends to zero. For example,


1
1
lim = 0 and lim x = 0.
x e
x x
6. Quantiles and percentiles both define the value of the random variable, X, below which
a given proportion of the distribution falls.
7. A quantile is a probability value expressed as a decimal, while a percentile is a
probability value expressed as a percentage.

Chapter 12

Continuous distributions

601

EXERCISE

12B

Using a probability density function to find


probabilities of continuous random variables
1 WE3 If X is a continuous random variable with the probability density function defined as
1 (2 x + 1), 1 x 2
f (x)= 4
, find, correct to 4 decimal places:
er
ere
0, elsewhere
a Pr(X > 1.5)

b Pr(1.2 < X < 1.75)

c Pr(X > 1.5 | X > 1.2).

2 If X is a continuous random variable with the probability density function defined as


1 (4 x )2 , 1 x 4
f (x) = 9
, find, correct to 4 decimal places:
er
ere
0, elsewhere
a Pr(X < 3)
d Pr(2 < X < 3 | X < 3)

b Pr(X > 2)
e Pr(X > 2 | X < 3).

c Pr(2 < X < 3)

ax ( x 2), 0 x 2
3 A random variable, X, has a pdf given by f (x
(x) =
.
er
ere
0, elsewhere
a Find the value of a.
b Find the probability, correct to 4 decimal places, that X is:
i greater than 1.5
ii less than 0.4
iii greater than 0.4 but less than 1.5
iv greater than 0.4, given that it is less than 1.5.
4 WE4 A continuous random variable, X, has probability density function defined by

1
x co s ( x 2 ), 2 x 3
f (x) = 5
. Calculate, correct to 4 decimal places:
er
ere
0, elsewhere
a Pr(X < 8)
b Pr(X < 8 | X < 8.5)

0 . 25
2 5e 0.25 x , x 0
5 WE5 A random variable, X, has its frequency curve defined as f (x
(x) =
.
er
ere
0, elsewhere
a Draw the graph of f (x).
b Show that it is a probability density function.
c Find the probability that X is:
i smaller than 2
ii greater than 1
iii greater than 1, given that it is smaller than 2.
1
, x 1
6
A random variable, X, has its frequency curve defined as f (x
(x) = x 2
.

0,
elsewhere
er
ere
a Draw the graph of f (x).

b Show that it is a probability density function.


c Find:
i Pr(X > 2)
ii Pr(X < 3)
iii Pr(2 < X < 3)
iv Pr(2 < X < 3 | X > 2)
v Pr(X < 3 | X > 2).

7 WE6 If X is a continuous random variable with a probability density function given by


1
1
x+ 2, 1 x 2
f (x) = 3
, find the value of a such that Pr(X a) = 0.36, where 1 a 2.
0, elsewhere
er
ere
602

Maths Quest 12 Mathematical Methods CAS for the TI-Nspire

If X is a continuous random variable with a probability density function given by


4
, 2x4
1
f (x) = x 2
, and Pr(X b) = 3, find the value of b, where 2 b 4.
0, elsewhere
er
ere

9 If X is a random variable with a probability density function given by


2
(3 x ), 0 x 1
f (x) = 5
, find:
0, elsewhere
er
ere
a the 0.4 quantile
b the 0.85 quantile
c the 70th percentile.


2 cos ( ),
10
If f () =
2
2 is the probability density function of the
0, elsewhere
er
ere

random variable :
a sketch the graph of the probability density function and shade the region corresponding to
the equation Pr(a a) = 12
b find the exact value of a.

The random variable, X, has a probability density function defined as


ax, 1 x 1
2
2
f (x) =
er
ere
0, elsewhere
a Sketch the probability density function.
b Hence, or otherwise, find the value of a.
c Calculate:
i Pr(0.5 < X < 0)
ii Pr(X 0.5)
Pr(0.5 < X < 0 | X 0.25)

The random variable, X, has


a probability density function defined as
1 ( x 2)),, 2 x 6
16
f (x) = 1 (10 x ),
), 6 < x 110
16

er
ere
0, elsewhere
a Sketch the probability density function.
b Calculate:
i Pr(X < 4)
ii Pr(3 < X 8)

EXAM TIP
On previous exams, students have
chosen a y-scale that was too large, drawn curved
graphs where straight lines were required, drawn an
unsymmetrical graph, or omitted the horizontal lines.
Greater care needs to be taken.
[Authors' advice]

13 The random variable, X, has a probability density function defined as

3 x + 1 , 10 2 x 0
8
256
3
f (x) = 1
.
4 x, 0 x 4
16
0, elsewhere
ere
ere

a Draw the graph of f (x).


b Show that the total area under the graph is 1.

Chapter 12

Continuous distributions

603

c Find:
i Pr(X < 1)
iv Pr(X < 3)

ii Pr(X > 2)
v Pr(2 < X < 3)

iii Pr(X > 5)


vi Pr(2 < X < 1 | X < 3).

A random variable, X, has a probability density function given by

x
coss ((222xx ),
f (x) =
4
4.
0, elsewhere
er
ere

a Draw the graph of the probability density function.


b Show that the area under the graph is 1.
c Calculate each of the following (giving answers in exact form).

i Pr(X < )
12

ii Pr(X <
)
6

iii Pr(X <


| X < 0)
6
A random variable, X, has its frequency function given by

x
a sinn , 0 x
2
f (x) =
.
0, elsewhere
er
ere

a Find the value of a such that f (x) is a probability density function.


b Draw the graph of the probability density function.
c Find, in exact form, the probability that X is:

i smaller than
2

ii greater than
3

2
iii between and
3
3
2

iv less than , given that it is greater than .


3
2
1
a
for x R and a > 0.
2
a + x2
a Sketch the graph of f (x) for a = 1, 2 and 3 on the same set of axes. Comment on the effect
of varying a in the shape of the graph.
b For a = 1, use a CAS calculator to find the following probabilities to 4 decimal places.
i Pr(X < 1)
ii Pr(1 < X < 1)
iii Pr(1 < X < 1 | X > 1)

16 A Cauchy distribution is defined as f (x


(x) =

17 a Sketch the graph of g(x


( )=
(x

e x , x R.

2
b Find the area bounded by the curve and the x-axis.
c Use your answers to a and b to define the probability density function f (x) whose rule is
the same as the rule of g(x).
d If X is a random variable whose probability density function is given by f (x), find, correct
to 4 decimal places:
i Pr(X > 0.5)
ii Pr(X < 1)
iii Pr(X > 0.32)
iv Pr(0.5 < X < 1.09).

604

Maths Quest 12 Mathematical Methods CAS for the TI-Nspire

12C

Measures of central tendency


and spread
In this section we discuss how to find measures of central tendency (mean, median and mode)
and measures of spread (variance and standard deviation) for a continuous random variable.

Measures of central tendency


Mean
If X is a continuous random variable with a probability distribution function f (x) over the

domain R, the mean of X can be found using the formula = E(x) = xf ( x ) ddx .

However, if f (x) has a certain rule for a X b and is zero elsewhere, then the mean can be
b

obtained by using the formula = E(x) = xf ( x )ddx . In the worked examples below and the
a

exercise that follows, we will mostly deal with the latter case. In general, E[g(x)] = g( x ) f ( x ) dx.
a

Median
The median value, m, of the continuous random variable, X, is a value such that Pr(X m) = 1 .
2
(It follows that Pr(X m) = 1 as well.) In other words, m is a value of X such that there is a 50%
2
chance that X will be less than or equal to m and a 50% chance that it will be greater than
m

or equal to m. Since Pr(X m) is given by f ( x )dx, to find the median, solve the equation
a
m
1
a f ( x)dx = 2 . The solution of equations of this type was discussed in detail in the previous
section of this chapter.
Mode
The mode is a value of X for which f (x) has its maximum. To find the mode of the continuous
random variable, it is best to sketch the graph of its probability density function first.
If the probability density function has a maximum turning point in the interval [a, b], the
mode is given by the x-coordinate of the turning point. It can be easily obtained by finding the
derivative of f (x), making it equal to zero, and solving for x.
If the probability density function continuously increases or decreases over the interval [a, b],
or if it has a minimum turning point, the mode is then given by the end point of the interval.
That is, the mode is either a or b (whichever one corresponds to the maximum value of f (x)).
Note that it is possible to have more than one mode.
f x)
f(

f x)
f(

Mode

f x)
f(

Mode x

Mode 0

WORKED EXAMPLE 7

eBoo
k plus
eBook

Find the mean, median and mode for the following probability density function.
2 x, 0 x 1
f (x
(x) =
0, elsewhere
THINK
1

Write the formula for finding the mean of the


continuous random variable.

Tutorial

int-0584
Worked example 7

WRITE
b

= E(x) = x f ( x )ddx
a

Chapter 12

Continuous distributions

605

Substitute 0 for a and 1 for b and replace f (x)


with 22x. Simplify the integrand.

0 x (2 x)dx

= 2 x 2 ddxx
0

Antidifferentiate.

2x3
=

3 0

Substitute the terminals in and evaluate.

2(1)3 2(0)3

3
3

= 23
5

State the value of the mean.

The mean is 23 .

Write the formula for finding the median, m.

Substitute 0 for a and replace f (x)) with 22x.

f ( x )dx =
m

2 x ddx =

1
2
1
2

2x2

=
2 0

Antidifferentiate and simplify.

Substitute in the terminals and simplify.

10

Evaluate the median and rationalise the


denominator. (Note that only the positive square
root is required, as 0 x 1.)

11

State the value of the median.

12

Sketch the graph of the pdf.

1
2

x 2 =
0

1
2

(m)2 (0)2 =

1
2

m2 =

1
2

m=

2
.
2

The median is
y

(1, 2)

0
13

The mode is the value of X for which f (x)


has its maximum. As seen from the graph,
the largest value of f (x) is at the right-handside end point of the domain. So this point
represents the mode.

2
2

The mode is 1.

Note: In the worked example above, the value of the mode was obtained directly from the graph.
In other cases differentiation may be required.

606

Maths Quest 12 Mathematical Methods CAS for the TI-Nspire

Consider the random variable, X, with the probability density


function given by

y
(0, 1)

1 ( x 2 2 x 3), 0 x 3
f (x) = 9
.
er
ere
0, elsewhere

(3, 0)
1

As can be seen from the graph, the probability density function


has a maximum turning point. The x-coordinate of this point represents the mode of X. To find
the x-coordinate of the turning point, we need to differentiate, make the derivative equal to zero,
and solve for x.
f (x) =

1
9

(2 2)
(2x

f (x) = 0
1
9

(2 2) = 0
(2x

2 2=0
2x
x=1
So the mode of X is 1.

Measures of spread
Variance
If X is a continuous random variable with the probability distribution function f (x) over the
domain R, the variance of X can be found using the formula
Var(X)
X = E(x )2 =
X)

( x )2 f ( x) ddxx .

However, if f (x) has a certain rule for a X b and is zero elsewhere, then the variance can be
obtained by using the formula Var(X)
X = E(x )2 =
X)

a ( x )2 f ( x) ddxx .

This formula may prove to be rather hard to use when f (x) is complicated. The alternative
formula for variance is Var(X)
X = E(X
X)
X2) 2. This can also be expressed as
Var(X)
X =
X)

a ( x)2 f ( x) ddxx a x f ( x) ddxx

Note that both formulas require us to find the mean of X first.


Standard deviation
The standard deviation of X can be easily found by taking a positive square root from the
Va X ) .
variance of X. That is, SD(X)
X = Var(
X)
Note: Standard deviation is often denoted as and variance as 2.
The calculations of both variance and standard deviation are shown in detail in the worked
example below.
WORKED EXAMPLE 8

Find the variance and standard deviation for the following probability density function.
x 1 , 1 x 2
2
f (x
(x) =
.
0, elsewhere
THINK
1

Write down the formula for finding the variance.

WRITE

Var(X)
X = E(X
X)
X2) 2

Chapter 12

Continuous distributions

607

Find the value of the mean. Write down the


appropriate formula.

= E(x) = x f ( x ) ddx

Substitute 1 and 2 for a and b respectively, and

= x x

replace f (x) with x

1
.
2

Simplify the integrand.

( ) dx

= x2
1

1
2

x
dx
2
2

Antidifferentiate.

Substitute the terminals in and evaluate .

x3 x 2
=
4 1
3

23 22 13 12
=
4 3 4
3
8 1 1 5 1
= 1 =
3 3 4 3 12
= 19
12

Find the value of E(


E(X
X2). Write the appropriate
formula.

E(x2) = x 2 f ( x ) ddx

Make all necessary substitutions and simplify


the integrand.

E(x2) = x 2 x

( ) dx

1
2

2
x2
= x 3 dx
1
2
2

Antidifferentiate.

x 4 x3
=
6 1
4

Substitute the terminals in and evaluate E(X


X2).

24 23 14 13
=
6 4 6
4
4 1 1

= 4

3 4 6
=

8 1

3 12

31
= 12

10

Substitute the values of E(X


X2) and into the
variance formula and evaluate.

31

( )

31

361

Var(X)
X = 12
X)

19
12

= 12 144
=

608

11
144

11

State the value of the variance.

11
Variance = 144

12

Write the formula for the standard deviation.

SD(X)
X = Var(
X)
Va X )

Maths Quest 12 Mathematical Methods CAS for the TI-Nspire

13

Substitute the value of the variance into the


formula and evaluate.

11
144

11
12

=
14

Standard deviation =

State the value of the standard deviation.

WORKED EXAMPLE 9

eBoo
k plus
eBook

logg ( x ), 1 < x < e


.
The probability density function for X is given by f ( x ) = e
0, elsewhere
Calculate, correct to 3 decimal places:
a the mean
c the standard deviation

b the median
d Pr( 2 X + 2).

THINK
a

1
2

11
12

Tutorial

int-0585
Worked example 9

WRITE

Write the formula for finding the mean


of the continuous random variable.
We need to use the CAS
calculator to integrate and find the
probability.
On a Calculator page, complete
the entry line as:

a = E( X ) =

0 x loge ( x) ddxx

= E( X ) =

1 x loge ( x) ddxx

n( x )) dx
1 ( x lln(
Then press ENTER .

Write the solution and round to


3 decimal places.
Write the formula for finding the
median.
On a Calculator page, press:
MENU b
3:Algebra 3
1:Solve 1
Complete the entry line as:
m
1
solvee ( lln( x ) ) dx = , m
2
1
Then press ENTER .

= 2.097
b

logg e ( x ) ddxx =

1
2

Chapter 12

Continuous distributions

609

Write the solution.

Solving logg e ( x ) ddxx =


1

1
2

for m implies

m = 0.1866 or m = 2.1555.

As 1 m e, so m must
be the bigger of the two
possible solutions. Answer
the question and round to
3 decimal places.

We first need to find the variance.


On a Calculator page, complete
the entry line as:

The median is 2.156.

c Var(X)
X =
X)

x dx 2 .
1 x 2 loge ( x)

1 ( x 2 ln( x)) dx (2.09726)2


e

Then press ENTER .

Calculate the standard deviation,


correct to 3 decimal places.

Find the intervals 2 and


+ 2.

SD(X)
X = Var(
X)
Va X )
= 0.176047371282
= 0.420
d 2 = 2.097 2 0.420

= 1.2581
+ 2 = 2.097 + 2 0.420
= 2.9364

State the interval


2 X + 2.

2 X + 2
= 1.2581 X 2.9364
= 1.2581 X e, since 2.9364 > e (the upper
value).

Calculate Pr( 2 X + 2)
using the CAS calculator.

Pr( 2 X + 2) =

1.2581

logg e ( x ) ddxx

= 0.969
Note that in this example, 96.9% of the data lies
within 2 standard deviations of the mean, which is
close to the estimated value of 95%.

Interquartile range
The interquartile range (IQR) is the middle 50% of the distribution. In the previous section we
discussed percentiles and quantiles.
IQR = 75th percentile 25th percentile.
= Q3 Q1

610

Maths Quest 12 Mathematical Methods CAS for the TI-Nspire

REMEMBER

1. If X is a continuous random variable with a probability distribution function f (x)


over the domain R, the mean of X can be found using the formula

= E(x) = xf ( x )ddx.

2. If f (x) has a certain rule for a X b and is zero elsewhere, then the mean can be
b

obtained by using the formula = E(x) = xf ( x )ddx .


a

In general, E[g(x)] = g( x ) f ( x )dx.


a

3. The median value, m, of the continuous random variable, X, is a value such that
Pr(X m) = 12 .

4. To find the median of X we need to solve for m the equation f ( x )dx = 12 .


a
5. The mode is a value of X for which f (x) has its maximum.
6. To find the mode of the continuous random variable, it is best to sketch the graph of
its probability density function first.
7. If the probability density function of X has a maximum turning point in the interval
[a, b], the mode is given by the x-coordinate of the turning point. It can be obtained
by finding the derivative of f (x), making it equal to zero, and solving for x.
8. If the probability density function continuously increases or decreases over the
interval [a, b], or if it has a minimum turning point, the mode is then given by the end
point of the interval (a or b) that corresponds to the maximum value of f (x).
9. It is possible to have more than one mode.
10. If X is a continuous random variable with the probability distribution function f (x)
over the domain R, the variance of X can be found using the formula

Var(X)
X = E(x )2 = ( x )2 f ( x )ddxx .
X)

11. If f (x) has a certain rule for a X b and is zero elsewhere, then the variance can be
b

obtained by using the formula Var(X)


X = E(x )2 = ( x )2 f ( x )ddxx .
X)
a

12. The alternative formula for variance is given by Var(X)


X = E(X
X2) 2. This can also be
b

expressed as Var(X)
X = ( x )2 f ( x )ddxx ( xf ((xx )dx )2 .
X)
a
a

13. The standard deviation of X can be found by taking the positive square root of the
variance of X. That is, SD(X)
X = Var(
X)
Va X ) .
14. The interquartile range (IQR) is the middle 50% of the distribution.
IQR = 75th percentile 25th percentile.
= Q3 Q1
EXERCISE

12C

Measures of central tendency and spread


1 WE7
Find the mean, median and mode for each of the following probability density
functions.
1 x + 1, 2 x 0
2, 2 x 1 . 5
a f (x) =
b f (x) = 2
0, elsewhere
0, elsewhere
2
3 x , 0 x 1
c f (x) =
er
ere
0, elsewhere

Chapter 12

Continuous distributions

611

2 Find the mean, median and mode for each of the following probability density functions.
1 x, 11 x 1
a f (x) = 5
0, elsewhere

3 (2 x )2 , 2 x 4
b f (x) = 8
er
ere
0, elsewhere

6
x ( x 1), 1 x 2
c f (x) = 5
er
ere
0, elsewhere

3 WE8
Find the variance and standard deviation for each of the following probability
density functions.
1 , 2 x 6
a f (x) = 4
er
ere
0, elsewhere

1 (3 x ), 0 x 2
b f (x) = 4
er
ere
0, elsewhere

4 ( x x )3 , 0 x 1
c f (x) =
er
ere
0, elsewhere

4 Find the variance and standard deviation for each of the following probability density
functions.
3
2( x 2), 2 x 3
( x + 1)2 , 0 x 1
a f (x) =
b f (x)= 7
er
ere
er
ere
0, elsewhere
0, elsewhere
1 sin ( x ), 2 x
c f (x) = 2
er
ere
0, elsewhere
5 A continuous random variable, X, has a probability density function defined by
3x
0 . 3e 0.3x
, x0
f (x) =
. Calculate:
er
ere
0, elsewhere
a the mean of X
b the median of X
c the mode of X
d the variance of X
e the standard deviation of X.
a
, 2x3

2
6 WE9 For f (x
(x) = ( x 1)
, find:

er
ere
0, elsewhere
a the value of a, such that f (x) would represent the probability density function of a random
variable, X
b the mean of X, correct to 4 decimal places.
c the standard deviation of X, correct to 4 decimal places.
d Pr( 2 X + 2), that is, the probability that X is within 2 standard deviations
from the mean, correct to 4 decimal places.
7 X is a continuous random variable with a probability density function given by

a co s x , 0 x

2
f (x) =
.
0, elsewhere
er
ere

a Calculate the value of a.


b Sketch the graph of the probability density function.
c Calculate the exact mean and the variance, correct to 4 decimal places.

d Find Pr X .
e Find Pr X X .

2
2
4

x
a sin 2 x + ,
For f (x
(x) =

4 4
4:
0, elsewhere
er
ere

612

Maths Quest 12 Mathematical Methods CAS for the TI-Nspire

a calculate the value of a such that f (x) could be a probability density function
b sketch the graph of the probability density function
c find the mean, median and mode
explain the result obtained in part c.
9

The probability density function of a random variable, X, is given by


k ( x + 2)3 , 2 x 0
f (x) =
.
er
ere
0, elsewhere
a Find the value of k.
b Calculate the median.
c Calculate the 0.25 quantile.
d Calculate the 0.75 quantile.
e Use the answers to parts c and d to find the IQR (interquartile range).

10 The probability density function of a random variable, X, is given by


cos ( x ) , wh er
eree x [0, 1]
1
f (x) = 2
er
ere
0, elsewhere
a Sketch the probability distribution function.
Calculate:
b the mean
c the median
d the mode.
11 The probability density function of a random variable, X, is given by
5
mx 2 + nx, 1 x 0
f (x) =
. If the mean of X is 7 , find the values of m and n.
0, elsewhere
(Hint: Use simultaneous equations.)

12D

Applications to problem solving


As was shown in the previous sections of this chapter, if the probability density function
of a continuous variable is known, we can use it for finding the probability of that variable
falling between particular values. So far, we have been practising finding these probabilities
for variables with various probability density functions without concerning ourselves as to
what these variables actually represent. In this section, we will look at continuous random
variables that represent something of practical interest, for instance, waiting time for a dental
appointment, or the weight of fish caught on a trout farm.

WORKED EXAMPLE 10

The local milk bar sells a particular brand of chocolate bar. The number sold per day is given by the
probability distribution function
x , 0 x 10

f ( x ) = 50
0, elsewhere
Calculate:
a the maximum number of chocolate bars that are sold each day
b the expected number of chocolate bars sold each day
c the probability that fewer than 4 chocolate bars are sold each day
d the probability that more than 8 chocolate bars are sold each day.
THINK
a The maximum number of chocolate bars that could

be sold is given by the domain of the function.

WRITE
a The maximum number of chocolate bars

that could be sold is 10.

Chapter 12

Continuous distributions

613

Write the formula for finding the expected


number of chocolate bars sold. Substitute for
f (x) and the appropriate interval [0, 10] for a
and b.

b = E( X ) =

a x f ( x)ddx

10

x x dx
50

10

x 2 dx
50

10

x3
=

150 0

Antidifferentiate and substitute in the


terminals. Evaluate.

State the expected value.

1000
0
150

20
3

The expected number of chocolate bars


sold each day is

Sketch the graph of the pdf curve. Shade


the required section on the graph. State the
probability that needs to be found.

20
.
3

y
(10, 1_5 )

10

Pr(X < 4) = Pr(0 < X < 4)


2

bh
2
The base of the triangle = 4.
The height of the triangle is f (4).
2
4
f (4) =
=
50 25
A=

As the shaded region forms a triangle, it is


more efficient to find the area by a geometric
method, rather than integrating.

A=
=
3

State the final answer.

4 25
2
4
25

The probability that less than 4 chocolate


4

bars are sold is 25.


d

Shade the required section on the graph. State


the probability that needs to be found.

y
(10, 1_5 )

10

Pr(X > 8) = Pr(8 < X < 10)

614

Maths Quest 12 Mathematical Methods CAS for the TI-Nspire

Pr(X > 8) =

This time, we will integrate to find the area


under the curve. State the integral that needs to
be evaluated to find the probability.
Note: Instead of integrating, the area of the
trapezium could be found.

10

= x

x
dx
50
10

100 8
2

= 100 64

State the final answer.

100 100
36
100

9
25

The probability that more than 8


9
chocolate bars are sold is 25.

WORKED EXAMPLE 11

The length of time (in minutes) between the successive incoming calls to a certain national helpline is
a continuous random variable with a probability density function given by
0.6 e 0.6 x , x 0
f (x
(x) =
.
0, elsewhere
Find, correct to 4 decimal places, the probability that the next call will come:
a within 30 seconds of the previous call
b between 2 and 3 minutes after the previous call
c more than 7 minutes after the last call.

THINK
a

WRITE

Sketch the graph of the pdf curve. If X


represents time (in minutes) between the
successive calls, then getting a call within
30 seconds from the previous one would
mean that X must be between 0 and 0.5 (as
30 seconds = 0.5 minutes). Shade the required
section on the graph. State the probability that
needs to be found and the integral that needs to
be evaluated in order to find that probability.

y
(0, 0.6)

0.5 1

Pr(0 X 0.5) =

0.5

0.6e

0.6 x

ddxx

0.5

0.6e 0.6 x
=

0.6 0

Evaluate by first antidifferentiating and


then substituting terminals in for x. Give
the answer correct to 4 decimal places as
required.

= e

0.6 x

0.5

= ( e 0.6 0.5 ) ( e
= e 0.3 + e 0
= 0.7408 + 1
= 0.2592

0.6 0 )

X is anywhere between 2 and 3. Show the


required section on the graph. State the
probability that needs to be found and the
integral that needs to be evaluated in order
to find that probability.

y
(0, 0.6)

2
3

6e
Pr(2 X 3) = 0.6e
2

Chapter 12

3
0.6 x

ddxx

Continuous distributions

615

0.6e 0.6 x
=

0.6 2

Evaluate.

= e

0.6 x

= ( e 0.6 3 ) ( e 0.6 2 )

= e 1.8 + e 1.2
= 0.1653 + 0.3012
= 0.1359

The probability that X will exceed 7 can be


obtained by finding the area under the pdf
curve to the right of 7. This is equivalent
to finding the area to the left of 7 and
subtracting it from the total area under the
curve (that is, from 1).

y
(0, 0.6)

0 1 2 3 4 5 6 7 8 9 10

Pr(X > 7) = 1 Pr(X 7)


7

6e
= 1 0.6e

0.6 x

ddxx
7

0.6e 0.6 x
=1

0.6 0

Evaluate.

= 1 e

0.6 x

= 1 [( e 0.6 7 ) ( e
= 1 ( e 4.2 + e 0 )
= 1 (0.0150 + 1)
= 0.0150

0.6
.6 0 )]

REMEMBER

1. Always sketch diagrams to represent the required probability.


2. Find probabilities by integrating the probability density function between the required
values.
EXERCISE

12D

Applications to problem solving


1

WE10 The time, in hours, that Kathryn spends watching television each day is a continuous
random variable with probability density function given by
3

(t 2 + 1), 0 t 2
f ( x ) = 14
0, elsewhere
Calculate:
a the maximum number of hours that Kathryn could watch television each day
b the mean number of hours of television she watches each day, correct to 2 decimal places.
c the probability that she watches more than 1.5 hours of television
d the probability that she watches up to 1 hour of television each day, correct to 4 decimal
places.

616

Maths Quest 12 Mathematical Methods CAS for the TI-Nspire

2 The lifespan (in days) of freshly cut roses is a random variable X with the probability density
function given by
1 x 1 , 2 x 4,
2
4
1
f ( x ) = x + 3 , 4 x 6,
2
4
0
,
e
lsewhere
ree.

a What is the probability that a freshly cut rose will last for:
i over 5 days?
ii between 3 and 5 days?
iii less than 4 days and 6 hours, correct to 3 decimal places?
b Leos girlfriend Elly likes white roses. For their three-month anniversary Leo bought her
a bunch. If the florist said that the freshly cut flowers were delivered to the shop 1.5 days
ago, what is the probability that the roses will last for at least 3 more days?
3

WE11 The length (in minutes) of waiting time at the dentist is a continuous random variable

0 . 15
1 5e 0.15 x , x 0
with probability density function given by f (x) =
. Find, correct to
er
ere
0, elsewhere

4 decimal places, the probability that:


a a patient will be admitted within 10 minutes of arrival
b a patient will have to wait anywhere between 10 and 20 minutes from arrival
c the waiting time will exceed half an hour.
4 The weight (in grams) of rainbow trout caught on a fish farm is a random variable, X, with
ke kx , x 0
probability density function given by f (x) =
, where k = 10 9000 .
0, elsewhere

a Find, correct to 4 decimal places, the probability that a randomly caught trout will weigh:
i less than 800 g
ii between 1 kg and 2 kg
iii over 2 kg.
b Calculate the mean weight of trout on this farm. Give your answer to the nearest gram.
c What proportion of trout on this farm weighs below the mean weight?
5 The life (in hours) of a particular brand of batteries is a random variable with probability
x
1

1000 , x 0
density function given by f (x) = 1000 e
.
0, elsewhere

a Find, correct to 4 decimal places, the probability that a randomly selected battery of this
brand will last:
i less than 100 hours
ii no more than 180 hours iii more than 200 hours.
b If after 180 hours of operation a battery is still working, what is the probability, correct to
4 decimal places, that it will last at least another 50 hours?
c A manufacturer of the batteries claims that 90% of their batteries will work for at least n
hours. Find the largest possible value of n.
A group of students is given a maths quiz. The time (in minutes) taken by a student to
complete the task is a continuous random variable with probability density function given by
0 . 1, 30 x 40
f (x) =
.
er
ere
0, elsewhere

Chapter 12

Continuous distributions

617

a State the time limits within which the quiz must be done.
b Find the probability that a student will:
i complete the task in under 33 minutes
ii take between 36 and 38 minutes to finish.
c Calculate the mean time taken to complete the quiz.
d The fastest 10% of the class will receive a bonus question and thus will have an
opportunity to earn extra marks. Find the longest time a student can take to finish the quiz
and still qualify for a bonus question.
7 Rachel is buying silk fabric for her formal dress. The dressmaker estimates that she will need
3.9 metres of silk. To be on the safe side, Rachel asks a salesperson to cut off 4 metres from
the selected roll. The difference (in centimetres) between the ordered length (4 metres) and the
actual length of material cut by the salesperson is a continuous random variable with
0.1
probability density function given by f (x) =
.
(0.001 + x 2 )
a Find the probability, correct to 4 decimal places, that the actual length of the material will be:
i within 2 centimetres of the ordered length
ii within 5 centimetres of the ordered length.
b Find the probability that the actual length of the material is at least 4.07 metres, correct to
4 decimal places.
c What is the probability that the purchased material will not be long enough to make a
dress (that is, will be shorter than 3.9 metres)? Answer to 4 decimal places.
8 An Australian Chamber Orchestra concert is to be broadcasted live on Classic FM. It is
scheduled to begin at 7 pm sharp. Although every effort is made to ensure the concert will start
on time (due to the live broadcast), it may still start anywhere between 6.55 pm and 7.05 pm.
The difference (in minutes) between the advertised starting time and the actual starting time is
a continuous random variable with probability density function given by

x
cos , 5 x 5

20
f (x) = 20 2
.
0, elsewhere
er
ere

a What is the probability that the concert will start within 30 seconds of the scheduled
time? Answer correct to 4 decimal places.
b Maya is listening to the concert at home. She turns on her radio at 6.58 pm. What is the
probability that Maya will miss the beginning, correct to 4 decimal places?
c Patrons who arrive after the concert has started will not be admitted until the interval.
Lena and Alex are caught in a traffic jam and estimate that they will arrive at the concert
hall at 7.03 pm. Assuming that the couple will indeed arrive at their estimated time, what
is the probability that they will be admitted, correct to 4 decimal places?
9 In a certain bank, the time (in minutes) the customer has to wait to be served is a random
variable with a probability density function defined as
kx, 0 x 10

f (x) = k ( x 20)),, 10 < x 20, fo r k > 0.


0, else
wh ere
er
elsewh
a Sketch the graph of the probability density function.
b Hence, or otherwise, find the value of k.
c Find the probability that a customer had to wait more than 10 minutes, if it is known that
he was waiting for at least 7 minutes, correct to 4 decimal places.
eBoo
k plus
eBook
d Nathan parked his car in a 15-minute parking zone just outside
the bank. He needs to deposit a cheque, which usually takes
Digital doc
3 minutes. What is the probability that Nathan will return to his
WorkSHEET 12.1
Work
car on time (that is, within 15 minutes)?

618

Maths Quest 12 Mathematical Methods CAS for the TI-Nspire

12E

The normal distribution

eBoo
k plus
eBook

Interactivity
The normal distribution is an important tool when dealing with the
int-0257
probability distribution of a continuous random variable. The frequency
The normal distribution
curve of the normal distribution is characterised by the symmetrical bell
shape called the normal distribution curve or normal curve. The normal
curve fairly realistically models many observed frequency distributions such as heights and weights
of infants, Mathematical Methods examination results, the intelligence quotient of children in a
particular age group, the lengths of battery lives, the diameters of steel cans, etc.
If X is a continuous random variable that follows a normal distribution with mean, , and
variance, 2, it is written as X N(
N , 2).

f x)
f(

Properties of the normal distribution

1
2

1. The normal probability distribution is characterised by a bellshaped curve that is symmetrical about the mean.
2. The equation of the normal curve is given by the probability
distribution function (pdf)
f ( x) =

where x R.
3. From the graph, a maximum value of

Mean
Mode
Median

1 x 2

is obtained when x = .

4. For a normal distribution:


(a) the mean, mode and median are the same
(b) many of the frequencies are situated near the mean
(c) the graph extends indefinitely to the right and left of the mean, but never touches or goes
below the x-axis
(d) the area between the normal curve and the x-axis is equal to 1 square unit, that is,

(e) Pr(a < x < b) = a f ( x ) dx .


b

1
2

1 x2

=1

Effect of and on the graph of the normal distribution function


The equation of the normal distribution function is given by: f ( x ) =

1 x 2

.
2
The standard deviation, , and the mean, , represent the transformations of the graph.
The most basic form of the normal distribution function is when the mean is 0 and the standard
1
x
1
deviation is 1. In this case the equation would be: f ( x ) =
e 2 .
2
The effect of the mean and the standard deviation on the normal distribution curve is:
dilation factor of 1 from the x-axis
dilation factor of from the y-axis
translation of units in the positive x direction (for > 0).
2

Common probabilites associated with a normal distribution


1. Approximately 68% of the observations lie within one
standard deviation of the mean. This may be written as
Pr( X + ) 68%.

68%

Chapter 12

Continuous distributions

619

2. Approximately 95% of the observations lie within two


standard deviations of the mean. This may be written as
Pr( 2 X + 2) 95%. We can also say that a value of x
will most probably lie within two standard deviations of the mean.
3. Approximately 99.7% of the observations lie within three standard
deviations of the mean. This may be written as Pr( 3 X
+ 3) 99.7%. We can also say that a value of x will almost
certainly lie within three standard deviations of the mean.

95%

+ 2 x

99.7%

+ 3
The mean, , and the standard deviation, , are used when
dealing with a population and are thus called population
parameters. If these values are unknown then the sample mean, x, and sample standard
deviation, s, are used.
WORKED EXAMPLE 12

Draw a curve of the normal distribution, showing an appropriate scale, for = 25 and = 5.
THINK

WRITE

Calculate .

Calculate 2.

Calculate 3.

Rule up a set of axes.

Label the axes, class intervals, frequency values and


the , , 2, 3 values.

Draw the normal distribution curve.

= 25 5 = 20
+ = 25 + 5 = 30
2 = 25 2 5 = 15
+ 2 = 25 + 2 5 = 35
3 = 25 3 5 = 10
+ 3 = 25 + 3 5 = 40
(25, 5 12 )

10 15 20 25 30 35 40

1
Remember to add the maximum point: ,
2

WORKED EXAMPLE 13

Scores from a certain test are normally distributed with mean, = 84 and
standard deviation = 4. Find the percentage of scores which are:
a between 80 and 88
b between 72 and 96
c above 88
d above 96.
THINK
a

620

eBoo
k plus
eBook
Tutorial

int-0586
Worked example 13

WRITE

As the values 80 and 88 are symmetrical about


the mean, determine the difference between
the mean and the extreme value.
Divide the difference by the standard deviation.
Note: This gives us the number of standard
deviations by which the extreme value
differs from the mean.

Maths Quest 12 Mathematical Methods CAS for the TI-Nspire

Difference = 88 84
=4
Difference 4
=

4
=1

Match the number of standard deviations with


the appropriate percentage.

68%

Answer the question.

Approximately 68% of the observations


lie between the scores of 80 and 88.

Determine the difference between the mean


and the extreme value.

Divide the difference by the standard


deviation.

Difference = 96 84
= 12
Difference 12
=

4
=3

Match the number of standard deviations with


the appropriate percentage.

3 99.7%

Answer the question.

Approximately 99.7% of the observations


lie between the scores of 72 and 96.

Relate the score with the corresponding mean


and standard deviation difference.
Use part a to assist.

Draw a diagram representing the situation.

+ 88

16%

16%
68%
80

84

88

Relate the percentage with the corresponding


mean and standard deviation difference.

68%. Therefore 32% of the scores


lie outside of the values.

Answer the question.


Note: Since we are looking at values beyond
the extreme, we halve the percentage.

16% of the scores will be greater than 88.

Relate the score with the corresponding mean


and standard deviation difference.
Use part b to assist.

Draw a diagram representing the situation.

+ 3 96

0.15%

0.15%
99.7%
96 x

72
3

Relate the percentage with the corresponding


mean and standard deviation difference.

3 99.7%. Therefore 0.3% of


the scores lie outside of the 3
values.

Answer the question.


Note: Since we are looking at values beyond
the extreme, we halve the percentage.

0.15% of the scores will be greater


than 96.

In parts a and b of the above worked example, each end point was symmetric about the
mean. Hence, when determining the difference between the mean and extreme value, only
one value was required.

Chapter 12

Continuous distributions

621

WORKED EXAMPLE 14

For a normally distributed variable with mean = 70 and standard deviation = 18, find:
a the range between which 68% of the values lie
b the range between which 95% of the values lie
c the value which has 2.5% of all values below it.
THINK
a

WRITE
a

68%

Relate the given percentage with the


appropriate and .

Calculate .

= 70 18 = 52

Calculate + .

+ = 70 + 18 = 88

Answer the question.

68% of the observations lie between 52 and 88.

Relate the given percentage with the


appropriate and .

Calculate 2.

2 = 70 2 18 = 34

Calculate + 2.

+ 2 = 70 + 2 18 = 106

Answer the question.

95% of the observations lie between 34 and 106.

Draw the normal distribution curve


and relate the given percentage with
the appropriate mean and standard
deviation difference.

2 95%

2.5%
95%
34

2.5%

70

106

2 95%. Therefore, 5% of the observations


lie outside of this interval. Furthermore, 2.5% lie
above the range and 2.5% lie below the range.
2.5% of the observed values will lie below 34.

Answer the question.

REMEMBER

1. If X is a continuous random variable that follows a normal distribution with mean, ,


and variance, 2, it is written as X N(, 2)
2. The equation of the normal curve is given by the probability distribution function (pdf)
f ( x) =

1 x 2

where x R.
2
3. For a normal distribution, the mean, median and mode are the same.
1
4. The maximum value of the graph is
when x = .
2
e

f x)
f(

1
2

622

Mean
Mode
Median

Maths Quest 12 Mathematical Methods CAS for the TI-Nspire

5. The area between the normal curve and the x-axis is equal to 1 square unit, that is,

1
2

1 x 2

ddxx = 1

6. Pr(a < x < b) = a f ( x ) dx


7. The effect of the mean and the standard deviation on the normal distribution curve is:
dilation factor of 1 from the x-axis
dilation factor of from the y-axis
translation of units in the positive x direction (for > 0).
8. The common probabilities associated with the normal distribution are:
(a) Approximately 68% of the observations lie within one standard deviation of the mean.
(b) Approximately 95% of the observations lie within two standard deviations of
the mean, or we say that a value of x will most probably lie within two standard
deviations of the mean.
(c) Approximately 99.7% of the observations lie within three standard deviations of
the mean, or we say that a value of x will almost certainly lie within three standard
deviations of the mean.
EXERCISE

12E

The normal distribution


1 WE12
Draw a curve of the following normal distributions, showing an appropriate scale
for each.
a = 10 and = 2
b = 20 and = 5
c = 0 and = 1
The three curves on the scale below show three normal
distributions.

eBoo
k plus
eBook
Digital doc

Spreadsheet 079

ii

Normal curves

0
a
b
c
d

iii

10 15 20 25 30 35 40 45 50 55 60 x

State the mean for each curve.


Estimate the standard deviation for each curve.
Which two curves have the same mean?
Which two curves have the same standard deviation?
a Draw normal curves for the following on the one graph.

i = 60 and = 10
ii = 60 and = 20
iii = 80 and = 10
b How does an increase in affect the graph?
c How does an increase in affect the graph?
4

Draw graphs of the following distributions.


a X N(0, 1)
b X N(20, 25)
c X N(50, 100)

EXAM TIP
Axes and a scale
must be clearly shown as part of
the sketch.

Chapter 12

[Authors' advice]

Continuous distributions

623

5 A large number of azalea bushes are planted, with heights following a normal distribution.
Their heights are recorded after 3 months, 6 months and 9 months. These records are shown on
the three curves below.
a Which curve represents the
i
3-month readings?
ii
b What can be said about
iii
the mean heights as time
progresses?
c What can be said about the
x
standard deviation as time
progresses?
1 x 65

1
6
The pdf of a normal random variable is given by f ( x ) =
e 2 5 .
5 2
a State the mean and the standard deviation of X.
b Sketch the graph of the normal distribution.
2

a If a particular normally distributed variable has a mean of 20 and a standard deviation


of 2, state what effect (in terms of transformations) the mean and standard deviation has on the
graph of the distribution.
b A normal distribution is given by X ~ N(9, 0.52). State what effect (in terms of
transformations), the mean and standard deviation have on the graph of the distribution.
8 WE13
Scores on a certain test are normally distributed with mean = 65 and standard
deviation
10. Find the percentage of scores which are:
a between 55 and 75
b between 45 and 85
c between 35 and 95
d above 75
e below 45
f above 95.
9

The random variable, X, is normally distributed with mean = 30 and standard deviation
= 5. Find the percentage of values which are:
a above 35
b above 40
c above 45.
If X N(42, 169), find the percentage of values which are:
between 29 and 55
b between 16 and 68
d below 29
e above 55

c between 3 and 81
f below 3.

11 WE14
A normally distributed variable has = 40 and = 12. Find:
a the range between which 68% of the values lie
b the range between which 95% of the values lie
c the range between which 99.7% of the values lie.
A normally distributed variable has = 27.2 and = 1.4. Find:

12
a
b
c
d
e

a
b
c
d
e
f
624

the range between which 68% of the values lie


the range between which 95% of the values lie
the range between which 99.7% of the values lie
the value which has 16% of all values above it
the value which has 2.5% of all values above it
the value which has 0.15% of all values above it.
A normally distributed variable has = 16.6 and = 0.6. Find:
the range between which 68% of the values lie
the range between which 95% of the values lie
the range between which 99.7% of the values lie
the value which has 16% of all values below it
the value which has 2.5% of all values below it
the value which has 0.15% of all values below it.

Maths Quest 12 Mathematical Methods CAS for the TI-Nspire

a A normally distributed variable has = 102.3 and = 21.4. Draw a normal curve
marking the positions of , , 2, 3.
Find the percentage of values between:
102.3 and 123.7
ii 59.5 and 102.3
iii 102.3 and 166.5 iv 80.9 and 145.1.
A normally distributed variable has = 24 and = 3. Find the percentage of values
between:
a 18 and 27
b 30 and 33.
16

Weights of babies are normally distributed with a mean of 3 kg and a variance of 0.36 kg.
Find the weight range over which:
a 68% of babies weights lie
b 95% of babies weights lie
c 99.7% of babies weights lie.

17 MC Scores from a certain test are known to be normally distributed. Approximately 95% of
the scores attained were between 36 and 60. The mean and standard deviation respectively are:
A 48 and 4
B 48 and 6
C 48 and 12
D 50 and 5
E 60 and 12
18 MC Which of the following is not true for all normal distributions?
A The median is equal to the mean.
B 95% of the values lie within 2 standard deviations of the mean.
C Pr(X > + ) 0.16.
D The mean is greater than the standard deviation.
E 68% of the values lie within 1 standard deviation of the mean.
19 MC For the normal distribution below, the shaded area is approximately equal to 0.997.

19

Which of the following is true?


A = 19, = 7
B = 19, = 40
D = 40, = 19
E = 40, = 21

61 x

C = 40, = 7

20 MC If X N(9, 9), then approximately 95% of the values would lie in the range:
A 9 X 27
B 0 X 18
C 3 X 15
D 5.5 X 12.5
E 9 X 18
21 MC The heights of a class of 7-year-old girls are normally distributed with a mean of
100 cm and a standard deviation of 9 cm. In this class, 95% of the girls would be between
approximately:
A 73 cm and 127 cm
B 82 cm and 118 cm
C 91 cm and 109 cm
D 94 cm and 106 cm
E 97 cm and 103 cm
22 MC Which one of the following would represent a continuous random variable?
A The number of people watching a netball match
B The time taken to read this question
C The number of 6s obtained in ten rolls of a die
D The cost of a new car
E The number of heads obtained in twenty tosses of a coin
23 If X is normally distributed with a mean of 12 and a variance of 2, find the range, correct to
2 decimal places, between which:
a 68% of the values lie
b 95% of the values lie
c 99.7% of the values lie.

Chapter 12

Continuous distributions

625

12F

The standard normal distribution


As seen earlier, the position and shape of the normal curve depend on the parameters and
, respectively. Thus, the area below the normal curve and hence the probability for a given
interval would vary for each different value of and . Integrating the equation for the
normal distribution each time a value is required can become tedious. This problem can be
quickly rectified by introducing a standard normal distribution where = 0 and = 1. The
standard normal variable is denoted by the letter z in order to distinguish it from the normal
variable, x.
The equation of the normal distribution curve f ( x ) =
1

1 x 2

when converted to the

x
where z =
and g( z ) = f ( x ) .
2

The standard normal distribution is written as Z N(0, 12).


The graph of the standard normal distribution is shown below.
standard normal curve becomes g( z ) =

z2

y
1
2

3 2 1

To convert a normal distribution into a standard normal distribution, the mean, , is subtracted
from the observed value, x, and the result is divided by the standard deviation, .

Calculating probabilities
A CAS calculator can be used to calculate the probabilities associated with the
normal distribution for any value of or , using normCdf (x1, x2, , ). x1 and x2 are
the lower and upper limits, respectively, of the interval for which you want to find the
probability.

WORKED EXAMPLE 15

Calculate the value of the following probabilities, correct to 4 decimal places.


a Pr(Z < 2)
b Pr(Z 0.728)
c Pr(2.02 < Z < 1.59)
THINK
a

WRITE

Draw a diagram and shade the


region required.

626

Maths Quest 12 Mathematical Methods CAS for the TI-Nspire

On a Calculator page, press:


MENU b
5:Probability 5
5:Distributions 5
2:Normal Cdf 2
Enter the values as shown.
Note: As we are dealing with a Z
variable, = 0 and = 1.
The lower limit is as there is
no end point for the function.

Press ENTER .

Write the solution and round the


answer to 4 decimal places.

Draw a diagram and shade the


region required.

Pr(Z < 2) = normCdf (, 2, 0, 1)


= 0.9772.
b

0.728 0
2

For Pr(Z 0.728), repeat as


above on the CAS calculator, this
time the lower limit is 0.728 and
the upper limit is .
Remember: Pr(X = a) = 0
Pr(X > a) = Pr(X a)

Chapter 12

Continuous distributions

627

Write the solution and round the


answer to 4 decimal places.

Draw a diagram and shade the


region required.

Pr(Z 0.728) = normCdf (0.728, , 0, 1)


= 0.7667.
c

2.02
2

For Pr(2.02 < Z < 1.59), repeat


as above on the CAS calculator.
This time the lower limit is 2.02
and the upper limit is 1.59.

Write the solution and round the


answer to 4 decimal places.

0 1.59

Pr(2.02 < Z < 1.59) = normCdf (2.02, 1.59, 0, 1)


= 0.9224.

WORKED EXAMPLE 16

If X is normally distributed with = 50 and = 8, calculate, correct to 4 decimal places:


a Pr(X > 55)
b Pr(28 < X < 65)
c Pr(X < 40 | X < 70).
THINK
a

WRITE

Draw a diagram and shade the


region required.

50 55
2

628

For Pr(X > 55) repeat as shown in


worked example 15.
Note: As we are dealing with an
X variable, the mean and standard
deviation have changed and are
now 50 and 8, respectively.

Maths Quest 12 Mathematical Methods CAS for the TI-Nspire

Press ENTER .

Write the solution and round the


answer to 4 decimal places.

Draw a diagram and shade the


region required.

Pr(X > 55) = normCdf (55, , 50, 8)


= 0.2660
b

28

For Pr(28 < X < 65), repeat as


above on the CAS. This time the
lower limit is 28 and the upper
limit is 65.
Write the solution and round the
answer to 4 decimal places.

Write the rule for conditional


probability.
Note: Pr(X < 40 | X < 70) =
Pr(X < 40). This is given by the
overlapping region in the diagram
below.

50

65

Pr(28 < X < 65) = normCdf (28, 65, 50, 8)


= 0.9666.

Pr(( X < 40 | X < 70) =

Pr[( X < 40) ( X < 70)]


Pr(( X < 70)

Pr(( X < 40)


Pr(( X < 70)

Pr(( X < 40 | X < 70) =

Pr(( X < 40)


Pr(( X < 70)

Pr (X < 40)
Pr (X < 70)
Region required

40 50 70
2

Find the individual probabilities of


the fraction using the CAS, as seen
previously in the example.

nor df ( , 40, 50,8


normC
, )
nor df ( , 70, 50,8
normC
, )

0.105650
= 0.993790

Chapter 12

Continuous distributions

629

Pr(( X < 40 | X < 70) =

Write the solution and round the


answer to 4 decimal places.

Pr(( X < 40)


Pr(( X < 70)
nor df ( , 40, 50,8
normC
, )

nor df ( , 70, 50,8


normC
, )

= 0.1063

Symmetry properties
The symmetrical nature of the normal distribution curve can sometimes be used to work out
probabilities.

As an example, Pr(Z > z) is shown above on the left. Because of the symmetry of the curve,
Pr(Z > z) = Pr(Z < z) as shown in the graph on the right.
Comparing the unshaded regions above reveals another property: Pr(Z > z) = Pr(Z < z). As
the area under the curve adds to 1, Pr(Z > z) + Pr(Z < z) = 1. Therefore, Pr(Z < z) = 1 Pr(Z > z),
as demonstrated by the graph below.

It is also important to be able to find the probability of z falling between two values, say a and b.
A diagram is essential as it allows us to see the situation clearly and hence solve the problem.
Consider the equation Pr(a < Z < b) = Pr(Z < b) Pr(Z < a).
The figure below clearly demonstrates the above situation.
y

=
a

0 b

0 b

y
Common region
is cancelled out
=
a

0 b

=
a

630

0 b

Maths Quest 12 Mathematical Methods CAS for the TI-Nspire

WORKED EXAMPLE 17
a If Pr(Z < p) = 0.30, calculate:

i Pr(Z > p)
ii Pr(Z > p)
b If Pr(Z < b) = 0.8 and Pr(Z a) = 0.25, calculate:
i Pr(a < Z < b)
ii Pr(Z < a | Z < b)
THINK
a

WRITE

To calculate Pr(Z
Z > p), draw a diagram
and shade the region required.

a i

p0
2

ii

1
2

A property of normal distribution


curves is: Pr(Z < p) + Pr(Z > p) = 1
Remember: Pr(Z > p) = Pr(Z p) as
Pr(Z = p) = 0
Write the answer.
By symmetry:
Pr(Z < p) = Pr(Z > p)
Write the answer.
To calculate Pr(a < Z < b), draw a
diagram and shade the region required.

Pr(Z > p) = 1 Pr(Z < p)


= 1 0.30
= 0.70
Pr(Z
Z > p) = 0.70
ii

Pr(Z > p) = 0.30


b i

a0b
2

ii

Pr(a < Z < b) = Pr(Z < b) Pr(Z < a).


= 0.8 0.25
= 0.55
Pr(a < Z < b) = 0.55

State the probability you want to find


and the rule you will use.
Pr(a < Z < b) = Pr(Z < b) Pr(Z < a).
State the answer.
State the probability you want to find and
the rule you will use.

ii

Pr(Z < a | Z < b) =


=

Substitute in the appropriate


probabilities and simplify.

Pr(Z < a | Z < b) =

Pr(( Z < a) ( Z < b))


Pr(( Z < b)
Pr(( Z < a)
Pr(( Z < b)
Pr(( Z < a)
Pr(( Z < b)

0.225
0.8
25
=
80
5
=
16
5
Pr(Z < a | Z < b) =
16
=

Write the solution.

Chapter 12

Continuous distributions

631

WORKED EXAMPLE 18

X is a normal random variable with mean 16 and standard deviation 2, and Z is the standard normal
variable.
a Find m if Pr(X > 18) = Pr(Z > m).
b Find m if Pr(X < 11) = Pr(Z > m).
THINK
a

WRITE
a

18 16
=
2
2
=
2
=1
Pr(X > 18) = Pr(Z > 1)
m=1

As the probability of the X variable,


Pr(X > 18), is in the same form as the
probability of the Z variable,
Pr(Z > m), m must directly relate to 18.
To convert a normal variable to a
standard normal variable, we use the
x
rule z =
.

Answer the question.

This time the probability of the X


variable, Pr(X < 11), is not in the
same form as the probability of the Z
variable, Pr(Z > m); however, we can
x
.
still link the two via the rule z =

Use the symmetry properties of the


normal distribution to find m.

Pr(Z > z) = Pr(Z < z).


Pr(Z < 2.5) = Pr(Z > 2.5)
Pr(X < 11) = Pr(Z > 2.5)

Answer the question.

m = 2.5

z=

x
.

11 16
=
2

5
=
2
= 2.5
Pr(X < 11) = Pr(Z < 2.5)
z=

WORKED EXAMPLE 19

The lengths of matches made at a certain factory are normally distributed with mean 4.1 cm and
standard deviation 0.05 cm. Find the probability, correct to 4 decimal places, that a randomly
selected match is:
a greater than 4.1 cm
b less than 4.13 cm
c between 4.08 cm and 4.14 cm.
THINK
a

WRITE

Draw a diagram and shade the


region required.

4.1
2

632

= 4.1, Pr(X > 4.1) = Pr(X > )

Pr(X > 4.1) = 0.5000

Maths Quest 12 Mathematical Methods CAS for the TI-Nspire

Draw a diagram and shade the


region required.

4.14.13

Pr(X < 4.13) can be found using


the CAS as shown in previous
examples. Write the solution and
round to 4 decimal places.

Draw a diagram and shade the


region required.

Pr(X < 4.13) = normCdf (, 4.13, 4.1, 0.05)


= 0.7257

4.08 4.14.14

Pr(4.08 < X < 4.14) can be


found using the CAS as shown
in previous examples. Write the
solution and round to 4 decimal
places.

Pr(4.08 < X < 4.14) = normCdf (4.08, 4.14, 4.1, 0.05)


= 0.4436

REMEMBER

1. A CAS calculator can be used to calculate the probabilities associated with the normal
distribution for any value of or using normCdf (x1, x2, , ). x1 and x2 are the
lower and upper limits, respectively, of the interval for which you want to find the
probability.
2. Using the symmetrical nature of the standard normal curve, it can be seen that:
Pr(Z > z) = 1 Pr(Z < z)
Pr(Z > z) = Pr(Z < z).
3. To convert a given normal variable, x, to the standard normal variable, z, use
x
the rule z =
.

4. With all normal distribution problems, > is equivalent to and < is equivalent to ,
since Pr(Z = z) = 0.
EXERCISE

12F

The standard normal distribution


1 WE15 Find, correct to 4 decimal places:
a Pr(Z 1)
b Pr(Z 1.5)
d Pr(Z > 2.71)
e Pr(0.42 < Z < 1.513)
2

c Pr(Z < 1.75)


f Pr(1.6 Z 1.4)

Standardise the following X


X-values to Z-values:
Z
40 if X N(25, 25)
b X = 12 if X N(17, 9)
15 if X N(12, 6.25)

Chapter 12

Continuous distributions

633

3 The variable X is normally distributed with mean = 9 and standard deviation = 3.


Standardise the following X-values:
X
a X = 10
b X = 7.5
c X = 12.4
4 WE16 If X is normally distributed with = 40 and = 7, find, correct to 4 decimal places:
a Pr(X > 42)
b Pr(X 30)
c Pr(X < 45)
d Pr(X 27)
e Pr(25 X 45)
5 If X N(20, 25), find, correct to 4 decimal places:
a Pr(X
( > 27)
(X
b Pr(X
( 18)
(X
d Pr(7 X 12)
e Pr(X
( < 17 | X 25)
(X
6 WE17a
a Pr(

If Pr(Z
Z > m) = 0.25, calculate:
b Pr(Z
Z > m)

c Pr(X
( 8)
(X
f Pr(X
( < 17 | X < )
(X
c Pr(Z
Z < m)

7 WE17b
If Pr(Z
Z b) = 0.62 and Pr(Z
Z < a) = 0.16, calculate:
a Pr(a < Z < b)
b Pr(Z
Z < a | Z < b)
If Pr(Z
Z b) = 0.58 and Pr(a < Z < b) = 0.44, find Pr(Z < a).
b) = 0.12 and Pr(a < Z < b) = 0.60, find Pr(Z > a).
X is a normal random variable with mean 36 and standard deviation 4, and Z is the
standard normal variable.
a Find m if Pr(X
X < 30) = Pr(Z
Z < m).
b Find m if Pr(X
X > 43) = Pr(Z
Z < m).
11 WE19 Light bulbs have a mean life of 125 hours and a standard deviation of 11 hours. Find
the probability, correct to 4 decimal places, that a randomly selected light bulb lasts:
a longer than 140 hours
b less than 100 hours
c between 100 and 140 hours.
12 The heights jumped by Year 9 high-jump contestants follow a normal distribution with a mean
jump height of 152 cm and a variance of 49 cm. Find the probability, correct to 4 decimal
places, that a competitor jumps:
a at least 159 cm
b less than 150 cm
c between 145 cm and 159 cm
d between 140 cm and 160 cm
e between 145 cm and 150 cm, given that she jumped over 140 cm.
13 MC If Z has a standard normal distribution, then Pr(Z > 1.251) is:
A 0.1054
B 0.3945
C 0.6055
D 0.6623
14 MC If Z N(0, 1), then Pr(Z <
A 0.0987
B 0.4013

E 0.8945

0.25)

is:
C 0.5987

D 0.7124

E 0.9013

15 MC The variable X is normally distributed with mean = 20 and standard deviation = 6.


The standardised value of Z
Z, for X = 29 is:
A 1.5
B 1.15
C 0.483
D 1.15
E 1.5
16 MC Tennis balls are dropped from a height of 2 metres. The rebound height of the balls is
normally distributed with a mean of 1.4 metres and a standard deviation of 0.1 metres. The
probability that a ball rebounds more than 1.25 metres is:
A 0.0668
B 0.2826
C 0.4332
D 0.7174
E 0.9332
17 MC The variable X is normally distributed with mean 70 and variance 12. The probability
that X is greater than 77 is:
A 0.0217
B 0.0429
C 0.0909
D 0.9091
E 0.9783
634

Maths Quest 12 Mathematical Methods CAS for the TI-Nspire

18 MC The life span of dogs is normally distributed with a mean of 12 years and a standard
deviation of 2 years. The probability that a dog lives for less than 9 years is:
A 0.0668
B 0.2826
C 0.2926
D 0.4332
E 0.9332
19 MC If X N(16, 4), then Pr(
Pr(X > 11.5) equals:
A 0.0122
B 0.0836
C 0.7217

D 0.9164

E 0.9878

20 The volume of milk in a 1-litre carton is normally distributed with a mean of 1.000 litres and
a standard deviation of 0.006 litres. A randomly selected carton is known to have more than
1.004 litres. Find the probability, correct to 4 decimal places, that it has less than 1.011 litres.
21 Eye fillet steaks are cut with a mean weight of 82 grams and a standard deviation of 5 grams.
Steaks are sold at different prices according to their weights, as shown in the table below.
Weight (g)

< 70

7080

8090

> 90

Cost ($)

1.40

1.60

1.80

2.00

a Find the probability that a randomly selected steak weighs between 80 grams and
90 grams. Give your answer correct to 4 decimal places.
b Find the probability that a randomly selected steak costs $2, correct to 4 decimal places.
c Copy and complete the following table.
Cost ($)

1.40

1.60

1.80

2.00

Probability
d Using your answers from part c, find the average price of an eye fillet steak.
22 The length of 6-cm nails is actually normally distributed with a mean length of 6 cm and
a standard deviation of 0.03 cm. Only nails which are between 5.93 cm and 6.07 cm are
acceptable and packaged accordingly. Find:
a the probability of a randomly selected nail being an acceptable length, correct to
4 decimal places
b the expected number of acceptable nails in a batch of 1000.
23 The length of fish caught in a certain river follows a normal distribution, with mean 32 cm and
standard deviation 4 cm. Fish which are less than 27 cm are considered to be undersized and
must be returned to the river. Find:
a the probability that a fish is undersize, correct to 4 decimal places
b the expected number of fish that a fisherman could take home if he catches 20 fish in one
afternoon and follows the rules for undersize fish.
24 The heights of students in Year 12 class were found to be normally distributed with mean of
171.2 cm and standard deviation of 5.2 cm.
a What is the probability, correct to 4 decimal places, that a student has a height of:
i greater than 168 cm?
ii between 164 cm and 174 cm?
iii greater than 179 cm, given that they are taller than 168 cm?
b If a random sample of 10 students was chosen, what is the probability, correct to
4 decimal places, that 6 of the students would have a height greater than 168 cm?
25 Amy is a keen tennis player. The amount of time, in hours, she spends training each week is
normally distributed with a mean of 8.4 hours and standard deviation of 1.8 hours.
a Find, correct to 4 decimal places, the probability that:
i she spends more than 10 hours training per week
ii she spends more than 5 hours training per week, given that she spent less than
10.5 hours training per week.
b What is the probability, correct to 4 decimal places, that of the next 5 weeks, Amy will
spend at least 1 of them training more than 10 hours a week?

Chapter 12

Continuous distributions

635

12G

The inverse cumulative normal


distribution
Up to now, we have focused on finding probabilities associated with either the normal
distribution or standard normal distribution. Sometimes, however, we are given the probability
and must determine the value of the variable. A CAS calculator, via the function invNorm, can
be used to calculate the value associated with a given probability for any value of or , by
invNorm( , , ). A must be the area to the left of the required value, that is, the less-than
invNorm(A
probability (e.g. Pr(X
X < c) = 0.47). In this instance A would be 0.47.

WORKED EXAMPLE 20

eBoo
k plus
eBook

Find the value of c, correct to 3 decimal places, in the following.


a Pr(Z < c) = 0.57
b Pr(Z c) = 0.91
THINK
a

Tutorial

int-0587
Worked example 20

WRITE

To calculate Pr(Z < c) = 0.57, draw a


diagram and shade the region required.

a
57%

0 c
2

As the shaded area is to the left of the


unknown value, we can use invNorm
straight away.
On a Calculator page, press:
MENU b
5:Probability 5
5:Distributions 5
3:Inverse Normal 3
Enter the values as shown.
Note: As we are dealing with a Z variable,
= 0 and = 1.
Press ENTER .

636

Write the solution and round to 3 decimal


places.

Maths Quest 12 Mathematical Methods CAS for the TI-Nspire

c = invNorm(0.57, 0, 1)
= 0.176

To calculate Pr(Z c) = 0.91, draw a


diagram and shade the region required.

b
91%

c
2

For this example, the shaded area is to


the right of the unknown value, so we
subtract the given area from 1.

Find the value of c using the CAS as


shown above.

Write the solution and round to 3 decimal


places.

Pr(Z < c) = 1 Pr(Z > c)


= 1 0.91
= 0.09

c = invNorm(0.09, 0, 1)
= 1.341

WORKED EXAMPLE 21

Find the value of c in Pr(c < Z < c) = 0.9544. Give your answer correct to 3 decimal places.
THINK
1

Draw a diagram of the situation. Label the unknown


values as c1 and c2.
Note: c1 = c2.

WRITE

0.0228

0.0228
0.9544

c1

c2

Determine the probability of each of the unshaded


areas by subtracting the given probability from 1 and
dividing the result by 2.

Find Pr(Z
Z < c2) by adding the probabilities.

Therefore Pr(Z < c2) = 0.9544 + 0.0228


= 0.9772

Find the value of c2 using the CAS as shown


previously.

c2 = invNorm(0.9772, 0, 1)
= 1.999
Pr(1.999 < Z < 1.999) = 0.9554

Answer the question.

c = 1.999

1 0.
0 9544
2
0.0456
=
2
= 0.0228

Unshaded area =

Chapter 12

Continuous distributions

637

Percentiles and quantiles


Remember that a percentile is a probability value expressed as a percentage while a quantile is
a probability value expressed as a decimal; for example, Pr(Z < c) = 0.7 means that c is the 70th
percentile or the 0.70 quantile.
WORKED EXAMPLE 22

If Z N(0, 12), find, correct to 3 decimal places:


a the 0.30 quantile
b the 80th percentile.
THINK
a

WRITE
a

Draw a diagram illustrating the


information.

0.30

Find the value of c using the CAS as


shown previously. Write the solution.

c = invNorm(0.30, 0, 1)
= 0.5244

Answer the question, correct to 3 decimal


places.

c = 0.524

Draw a diagram of the situation.

b The 80th percentile is the 0.80 quantile.

0.80

Find the value of c using the CAS as


shown previously. Write the solution.

c = invNorm(0.80, 0, 1)
= 0.84162

Answer the question, correct to 3 decimal places.

c = 0.842

WORKED EXAMPLE 23

X is normally distributed with a mean of 10 and a standard deviation of 2. Calculate x1, correct to
3 decimal places, if:
a Pr(X x1) = 0.65
b Pr(X > x1) = 0.85.
THINK
a

WRITE

Draw a diagram and shade the


region required.

0.65

10 x1

638

Maths Quest 12 Mathematical Methods CAS for the TI-Nspire

As the shaded area is to the left


of the unknown value, we can use
invNorm straight away.
Note: This time we are dealing
with an X variable, so the mean and
standard deviation have changed
from 0 and 1, respectively. From
the information,
= 10 and = 2.
Enter the values as shown.

Press ENTER .

Write the solution and round to


3 decimal places.

Draw a diagram and shade the


region required.

x1 = invNorm(0.65, 10, 2)
= 10.771
b

0.85

x1

10

For this example, the shaded area


is to the right of the unknown
value, so we subtract the given
area from 1.

Pr(Z < x1) = 1 Pr(Z > x1)


= 1 0.85
= 0.15

Find the value of c using the CAS


as shown previously. Write the
solution and round to 3 decimal
places.

x1 = invNorm(0.15, 10, 2)
= 7.927

WORKED EXAMPLE 24

If X is normally distributed with a mean of 52.3 and Pr(X < 48) = 0.229, calculate the standard
deviation, correct to 2 decimal places.

Chapter 12

Continuous distributions

639

THINK

WRITE

We need to find the standardised Z


Z-value for the
X-value of 48. As we know and for Z, we can use
X
the invNorm function to find the standardised value.
Note: Even though z is rounded to 3 decimal places
on the page, be sure to use the maximum number of
decimal places in your subsequent working.

Pr(X < 48) = Pr(Z < z) = 0.229


z = invNorm(0.229, 0, 1)
= 0.742

Now we link the z-value to the x-value by


x
.
the rule z =

z=

0.742

Write the solution and round to 2 decimal places.

The standard deviation is 5.79.

48 52.3

0.742 = 4.3

4.3
=
0.742
= 5.794
=

REMEMBER

1. A CAS calculator can be used to calculate the value associated with a given probability
for any value of or , by invNorm(
invNorm(A, , ). A must be the area to the left of the
required value.
2. A percentile is a probability value expressed as a percentage while a quantile is a
probability value expressed as a decimal.
3. When the standard deviation or mean is unknown, the standardised value, z, must be
x
found. The mean or standard deviation is then calculated using the rule z =
.

EXERCISE

12G

The inverse cumulative normal distribution


1 WE20 Find the value of c in the following, correct to 3 decimal places.
a Pr(Z < c) = 0.9
b Pr(Z < c) = 0.2
c Pr(Z > c) = 0.54
d Pr(Z c) = 0.45
2 WE21 Find the value of c in the following.
a Pr(c < Z < c) = 0.6826
c Pr(c < Z < c) = 0.2

b Pr(c Z c) = 0.5
d Pr(c Z c) = 0.38

3 WE22 If Z N(0, 1), find:


a the 0.25 quantile
c the 0.72 quantile

b the 40th percentile


d the 0.995 quantile.

4 WE23 X is normally distributed with a mean of 10 and a standard deviation of 2. Find x1 if:
a Pr(X
( x1) = 0.72
(X
b Pr(X
( < x1) = 0.4
(X
c Pr(X
( > x1) = 0.63
(X
d Pr(X
( x1) = 0.2.
(X
5 X is normally distributed with a mean of 34 and a standard deviation of 16. Find c if:
a Pr(X
( > c) = 0.31
(X
b Pr(X
( c) = 0.75
(X
c Pr(X
( < c) = 0.21
(X
d Pr(X
( c) = 0.55.
(X
640

Maths Quest 12 Mathematical Methods CAS for the TI-Nspire

6 Let X N(22, 25). Find k if:


a Pr(22 k X 22 + k) = 0.7
b Pr(22 k < X < 22 + k) = 0.24
c Pr(X
( < k | X < 23) = 0.32.
(X
7 MC If Pr(Z c) = 0.8, then c equals:
A 0.842
B 0.253
C 0.253

D 0.524

E 0.842

8 MC If Pr(Z > c) = 0.7, then c equals:


A 0.524
B 0.496
C 0.496

D 0.524

E 0.553

< Z < k) = 0.4, then k equals:


B 0.253
C 0.038

D 0.253

E 0.885

10 MC If Z N(0, 1), then the 0.35 quantile is:


A 0.675
B 0.385
C 0.350

D 0.385

E 0.675

11 MC If Pr(Z < k | Z < 0.5) = 0.6, then k equals:


A 0.215
B 0.253
C 0

D 0.253

E 0.215

Pr(1.2

9 MC If
A 0.885

12 MC If X is normally distributed with a mean of 20 and a standard deviation of 4 and


Pr( < k) = 0.6, then k equals:
Pr(X
A 18.99
B 19.49
C 20.51
D 21.01
E 21.79
13 MC If X N(12, 4) and Pr(
Pr(X > k) = 0.82, then k equals:
A 8.34
B 10.17
C 13.83
D 14.27

E 15.66

14 The height of Year 9 students is known to be normally distributed with a mean of 160 cm and a
standard deviation of 8 cm. Answer the following; correct to 2 decimal places.
a How tall is Theo if he is taller than 95% of Year 9 students?
b How tall is Luisa if she is shorter than 80% of Year 9 students?
15 The length of nails manufactured as 45 mm are actually normally distributed with a mean
of 45 mm and a standard deviation of 0.5 mm. The shortest 20% and longest 20% of nails
manufactured are discarded before packaging and sold as seconds. Find correct to 2 decimal
places:
a the minimum length of packaged nails
b the maximum length of packaged nails.
16 At a qualifying meeting, the time taken for runners to complete 400 metres follows a normal
distribution, with a mean time of 50 seconds and a standard deviation of 2 seconds. If the
fastest 25% of runners qualify for the next meeting, how fast would you need to run to qualify?
Give your answer correct to 2 decimal places.
17 WE24 If X N(20, 2) and Pr(
Pr(X 19) = 0.7, find the standard deviation, .
18 X is normally distributed with a mean of 50 and a standard deviation of . Forty per cent of
X-values are less than 48. Find .
X
19 Weights of packaged rice are normally distributed with a mean of 500 grams. Ten per cent of
packages are under 485 grams. Find the standard deviation.
20 If X N(, 16) and Pr(
Pr(X
X 17) = 0.99, find the mean, .
21 X is normally distributed with a mean of and a standard deviation of 3. If 35% of X-values
are at least 27, find the mean.
22 The time taken for grade 4 students to complete a small jigsaw puzzle follows a normal
distribution with a standard deviation of 30 seconds. If 70% of grade 4 students complete
the puzzle in 4 minutes or less, find the mean completion time for grade 4 students correct to
2 decimal places.
23 Bridget owns a fruit and vegetable shop. Being a keen mathematician as well, she knows that
the weights of the watermelon she sells are normally distributed with a mean weight of 1.5 kg

Chapter 12

Continuous distributions

641

and a standard deviation of 0.2 kg. She classifies her watermelons as either large, medium or
small. The heaviest 15% are classified as large, the lightest 15% are classified as small and the
rest as medium. Find the range of weights, correct to 1 decimal place, for which:
a a large watermelon is classified
b a medium watermelon is classified.
eBoo
k plus
eBook
Digital doc

WorkSHEET 12.2

24 Mr Lim, a physics teacher, sets a particularly hard test for his students. He finds that the
average mark is 54 and the standard deviation is 8. He decides to award the top 10% an A, and
fail the bottom 10%. Find, correct to the nearest whole number:
eBoo
k plus
eBook
a the lowest mark required to achieve an A
b the range of marks that a student who fails the test could have achieved. Digital doc
Investigation
Sunflower stems

642

Maths Quest 12 Mathematical Methods CAS for the TI-Nspire

SUMMARY
Continuous random variables

Continuous random variables represent quantities that can be measured and thus may assume any value in a
given range.
Probability density functions

If X is a continuous random variable, then the probability of it falling between certain values is given by the
area under a frequency curve known as the probability density function or pdf.
The probability density function must be greater than or equal to zero for all values of x, and the total area

under the curve must be 1. That is, f(


f x) 0 for all x and f ( x )dx = 1.

The domain of a probability density function is usually R, that is, the variable is continuous and can assume
any real value (at least in theory).
If we need our function to be non-zero over a particular interval only, say, over (a, b), then we need to
specify that the function is equal to zero everywhere else.
If a probability density function is only non-zero over a particular interval (a, b), the area under the curve
b

between x = a and x = b must be 1. That is, a f ( x )dx = 1.


Finding probabilities using a probability density function
y

If X is a continuous random variable, then the probability of it falling


between x = a and x = b is given by the area bounded by the probability
density function, the x-axis and the lines x = a and x = b. That is,
Pr (a X b) =

Pr(a <
b)
x<

a f ( x)dx .

Pr(x = a) = 0
Pr(x > a) = Pr(x a)
To integrate over intervals involving infinity, the limits are evaluated as follows.
Interval

Limit
lim

(, )
(, a)

k f ( x) dx

lim

lim

k k

(a, )

k a

f ( x ) dx
f ( x ) dx

1
1
= 0 and lim x = 0.
x
x
e
Quantiles and percentiles both define the value of the random variable, X, below which a given proportion of
the distribution falls.
A quantile is a probability value expressed as a decimal, while a percentile is a probability value expressed
as a percentage.
As a number tends to infinity, its reciprocal tends to zero. For example, lim

Measures of central tendency

If X is a continuous random variable with a probability distribution function f (x) over the domain R, the
mean of X can be found using the formula

= E (x) =

xf ( x)ddx .

Chapter 12

Continuous distributions

643

In general, E[g(x)] =

a g( x) f ( x)dx.

If f (x) has a certain rule for a X b and is zero elsewhere, then the mean can be obtained by
b

using the formula = E(x) = xf ( x )ddx .


a
1
The median value, m, of the continuous random variable, X, is a value such that Pr(X m) = 2 .
To find the median of X, solve for m the equation

f ( x )dx = 2 .

The mode is a value of X for which f (x) has its maximum. It is possible to have more than one mode.
To find the mode of a continuous random variable:
sketch the graph of the probability density function first
if the probability density function of X has a maximum turning point in the interval [a, b], the mode is
given by the x-coordinate of the turning point. It can be obtained by finding the derivative of f (x), making
it equal to zero, and solving for x.
if the probability density function continuously increases or decreases over the interval [a, b], or if it has a
minimum turning point, the mode is then given by the end point of the interval (a or b) that corresponds to
the maximum value of f (x).
Measures of spread

If X is a continuous random variable with a probability distribution function f(


f x) over the domain R, the
variance of X can be found using the formula

Var(X)
X = E(x )2 = ( x )2 f ( x )ddxx .
X)

If f (x) has a certain rule for a X b and is zero elsewhere, then the variance can be obtained
by using the formula Var(X)
X = E(x )2 =
X)

a ( x )2 f ( x)ddxx .

The alternative formula for variance is Var(X)


X = E(X
X2) ()2. This can also be expressed as
2

a ( x)2 f ( x)ddxx a xf ( x)ddx .


The standard deviation of X can be found by taking the positive square root of the variance of
Var(X)
X =
X)

Va X ) .
X, that is, SD(X)
X = Var(
X)
The interquartile range (IQR) is the middle 50% of the distribution. IQR = 75th percentile 25th percentile.
Applications to problem solving

Always sketch diagrams to represent the required probability.


Find probabilities by integrating the probability density function between the required values. This can be
done manually or by using a CAS calculator.
The normal distribution

The normal distribution is the most important distribution associated with continuous random variables.
The normal probability distribution is characterised by a bell-shaped curve which is symmetrical about the mean.
The equation of the normal curve is given by the probability distribution function f ( x ) =
1

when x = .
2
For the normal distribution, the same rules apply as for all other
probability density functions:
1. f (x) 0 for all x,
where x R. It has a maximum value of

2.

x
2

f x)
f(

f ( x)dx = 1, and

3. Pr(a < X < b) =

644

f ( x )dx .

Maths Quest 12 Mathematical Methods CAS for the TI-Nspire

a b

The normal distribution with mean, , and variance, 2, is written as X N(


N( , 2).
The common probabilities associated with the normal distribution are as follows:
1. Approximately 68% of the observations lie within one standard
deviation of the mean.
68%
x

2. Approximately 95% of the observations lie within two standard


deviations of the mean, or we say that a value of x will most probably
lie within two standard deviations of the mean.

95%

3. Approximately 99.7% of the observations lie within three standard


deviations of the mean, or we say that a value of x will almost certainly
lie within three standard deviations of the mean.

+ 2 x

99.7%

The parameters and have an effect on the normal distribution


3

+ 3 x
curve as follows:
The mean, , controls the position of the curve.
The standard deviation, , controls the width of the curve.
Specifically, the effect of the mean and the standard deviation on the normal distribution curve is:
1. dilation factor of 1 from the x-axis
2. dilation factor of from the y-axis
3. translation of units in the positive x direction (for > 0).

The standard normal distribution

The standard normal distribution is written as Z N(0, 12).


x
To convert a given normal variable, x, to the standard normal variable, z, use the rule z =
.

A CAS calculator can be used to calculate the probabilities associated with the normal distribution for any
value of or using normCdf(x1, x2, , ). x1 and x2 are the lower and upper
limits, respectively, of the interval for which you want to find the probability.
Using the symmetrical nature of the standard normal curve, it can be seen that:
Pr(Z > z) = 1 Pr(Z < z)
0

Pr(Z > z) = Pr(Z < z)

With all normal distribution problems, > is equivalent to and < is equivalent to , since Pr(Z = z) = 0.
The inverse cumulative normal distribution

A CAS calculator can be used to calculate the value associated with a given probability for any value of
or , using invNorm(
invNorm(A, , ). A must be the area to the left of the required value.
A percentile is a probability value expressed as a percentage while a quantile is a probability value expressed
as a decimal.
When the standard deviation or mean is unknown, the standardised value, z, must be found. The mean or
x
.
standard deviation is then calculated using the rule z =

Chapter 12

Continuous distributions

645

CHAPTER REVIEW
SHORT ANSWER

2 ( 2 4 x ) , 0 x 1
2.
1 a Sketch the graph of f (x
( )=
er
ere
0, elsewhere
b State whether or not f (x) could be a probability
density function.
x , 0 x 4
2 a Sketch the graph of f (x
( )=
er
ere
0, elsewhere
and explain why it is not a probability density
function.
b Adjust the function so that it could be a
probability density function by adding a
dilation coefficient, that is, by changing the
rule to f (x) = a x and retaining the given
domain.
c Now adjust the left-hand-side end point of
the domain so that f (x) could be a probability
density function. That is, keep the rule
unchanged, but replace 0 with an appropriate
number in 0 x 4.
3 If X is a random variable with a probability density
sin (2 x ), 0 < x < 2
function given by f (x
( )=
.
er
ere
0, elsewhere
Calculate:

a Pr(X < 6 )

b Pr(X < X < 3 ).


6

4 Consider the random variable, X, with probability


density function given by
x 1,
1 x 2

f (x) = x + 3, 2 < x 3.
0, elsewhere

Calculate:
a Pr(1.5 < X < 2)
b Pr(X
X < 2.5)
c the mode
d the median.
5 If X is a random variable with a probability density
function given by
kx, 0 < x < 4
f (x
( ) =
.
er
ere
0, elsewhere

646

Calculate:
a the value of k
c the median

b the mean
d the mode.

6 The cooking time (in minutes) of a single portion


of rice in a particular microwave is a random
variable, X, with a probability density function
given by
1
, 4 x 6
f x) = 2
f(
.
er
ere
0, elsewhere

a Lenas cookbook recommends to cook one


portion of rice for 5.5 minutes. If Lena follows
the instructions in her cookbook, what is the
probability that the rice will not be ready?
b What is the probability that it will take
anywhere between 4 minutes and 36 seconds
and 5 minutes and 15 seconds for the rice to
cook?
c If Lena wants to be 90% sure that her rice will
be ready, what is the minimum cooking time
she has to set on the microwave?
7 Draw a curve of the following normal distributions,
showing an appropriate scale for each.
a = 8 and = 1
b = 25 and = 8
c = 0 and = 3
8 Draw graphs of the following distributions.
a X N(30, 64)
b X N(8, 4)
c X N(100, 100)
9 A normal distribution is given by X ~ N(22, 9).
State what effect (in terms of transformations) the
mean and standard deviation have on the graph of
the distribution.
10 Weights of wooden sleepers are normally
distributed with mean = 60 kg and standard
deviation = 9 kg. Find the percentage of sleepers
that are:
a between 51 kg and 69 kg
b between 42 kg and 78 kg
c between 33 kg and 87 kg.
11 A normally distributed variable has = 19.6 and
= 3.1. Find:
a the range between which 68% of the values lie
b the range between which 95% of the values lie

Maths Quest 12 Mathematical Methods CAS for the TI-Nspire

c
d
e
f

the range between which 99.7% of the values lie


the value which has 16% of all values above it
the value which has 2.5% of all values above it
the value which has 0.15% of all values
above it.

12 Standardise the following X


X-values to Z-values.
Z
a X = 26 if X N(22, 36)
b X = 18 if X N(16, 16)
c X = 56 if X N(50.9, 100)

14 a If Pr(Z b) = 0.81 and Pr(a < Z < b) = 0.44,


find Pr(Z < a).
b If Pr(Z
Z > b) = 0.36 and Pr(a < Z < b) = 0.24,
find Pr(Z < a).
15 X is a normal random variable with mean 27 and
standard deviation 3, and Z is the standard normal
variable.
a Find m if Pr(X
X < 33) = Pr(Z
Z < m).
b Find m if Pr(X
X > 22.5) = Pr(Z
Z < m).
16 Let X be a normally distributed random variable
with a mean of 72 and a standard deviation of 8.
Let Z be the standard normal random variable. Use
the result that Pr(Z
Z < 1) = 0.84, correct to 2 decimal
places, to find:
a the probability that X is greater than 80
b the probability that 64 < X < 72
c the probability that X < 64 given that X < 72.
EXAM TIP
Some students did not clearly
understand the relationship between the random
variables X and Z. In part c, 47% of students scored
0/2. Most realised that conditional probability was
required but were not able to apply it correctly.
[Assessment report 1 2006]

[ VCAA 2006]

MULTIPLE CHOICE

1 Which of the following could be a probability


density function?

2, 1 x 0
2
ii f (x) =
0, elsewhere
2 x, 1 x 2
iii f (x) =
0, elsewhere

B II only
E I, II and III

C III only

2 X is a random variable with probability density


2 sinn ((4 x ), a x b
function defined as f (x) =
.
er
ere
0, elsewhere
The values of a and b respectively are:

4
3
,
2 4

A 0,
D

13 If Pr(Z
Z > p) = 0.72, calculate:
a Pr(Z
Z < p)
b Pr(Z
Z > p).

1
3, 0 x 3
i f (x) =
er
ere
0, elsewhere

A I only
D I and III


,
4 2

C 0,

E 0,

a
, 1 x 2
3 The value of a such that f (x
( ) = x2
0, elsewhere
er
ere

could be a probability density function is:


1
2

B 1

D 3

E 4

C 2

4 If X is a random variable with a probability density


2
(4 x )),, 1 x 2
function defined by f (x) = 5
,
er
ere
0, elsewhere
then Pr(X
X > 1.2) can be obtained by evaluating:
A

1.2

C 1
E

2
(4
5

1.2
2
(4
5 1

2
2
(4
5 1

x )ddxx

1.2
2
(4
5 2

D 1

x )ddxx

2
(4
1.2 5

x )ddxx

x )ddxx

x )ddxx

5 If X is a random variable with a probability density


1 , 0 x 4
function defined by f(
f x) = 4
and
er
ere
0, elsewhere
Pr (X
X < a) = 0.625, the value of a is:
A 1.5
B 2
C 2.5
D 3
E 3.5
Questions 6 to 9 refer to the random variable, X,
with the probability density function given by
4 x 2 , 1 x 4
f (x) = 3
.
er
ere
0, elsewhere

6 The mean of X correct to 4 decimal places is:


A 1.8484
B 2.0000
C 2.5000
D 2.8484
E 3.8484
7 The median of X is:
A
D

4
3
15
8

8
5
8
15

1
12

5
8

8 The mode of X is:


A

1
12

D 1

and

4
3

4
3

E 4

Chapter 12

Continuous distributions

647

9 The variance of X can be found by evaluating:


A

1 3x dx ( 5 )

1 3x dx ( 8 )

4
dx
3x 2
4

4
D 3 x
1

4
E 3 x
1

4
3

4
3

X2 ~ N( 2, 22)

()

O
2

logg e (4)
logg e (4)

Questions 10 to 11 refer to the following


information.
The length of time (in minutes) of a gentlemans
haircut at the hairdressers is a continuous
random variable with a probability density
function given
3x , x 0
0 . 3e 0.3x
by f (x) =
.
er
ere
0, elsewhere

10 The probability that Alexs haircut will take longer


than a quarter of an hour is:
A 0.0521
B 0.0498
C 0.9502
D 0.0111
E 0.9889
11 The probability that Alexs haircut will take longer
than a quarter of an hour, given that it has already
taken 10 minutes is:
A 0.2231
B 0.0117
C 0.0524
D 0.0112
E 0.0504
12 Scores on a certain test are known to be normally
distributed. Approximately 95% of the scores
attained are between 26 and 74. The mean and
standard deviation respectively are:
A 48 and 6
B 48 and 8
C 48 and 12
D 50 and 6
E 50 and 12
13 If X N(27, 16), then approximately 95% of the
values would lie in the range:
A 5 X 59
B 9 X 39
C 0 X 54
D 19 X 35
E 23 X 31
14 The diagram below shows the graphs of two
normal distribution curves with means 1
and 2 and standard deviations 1 and 2
respectively.

648

X1 ~ N( 1, 12)

8
5

A
B
C
D
E

1 > 2 and 1 = 2.
1 > 2 and 1 > 2
1 = 2 and 1 > 2
1 = 2 and 1 < 2
1 < 2 and 1 = 2

[ VCAA 2003]

15 The random variable X has a normal distribution


with mean 20 and standard deviation 4. If the
random variable Z has the standard normal
distribution, then the probability that X is more than
24 is equal to:
A Pr(Z
Z < 1)
B 1 Pr(Z
Z > 1)
C Pr(Z
Z < 1)
D Pr(Z
Z > 1)
E 1 Pr(Z
Z < 1)
16 If Z has a standard normal distribution, then
Pr(Z > 1.111) is:
A 0.1333
B 0.1335
C 0.1357
D 0.8665
E 0.8667
17 X is normally distributed with mean = 16 and
standard deviation = 5. The standardised value
for X = 22 is:
A 1.2
B 1.0625
C 1.0625
D 1.2
E 1.5
18 The standing long jump distances of NBL
basketballers is known to follow a normal
distribution with a mean of 1.4 metres and a
standard deviation of 0.25 metres. The probability
that an NBL player has a standing long jump of at
least 1.6 metres is:
A 0.0013
B 0.0228
C 0.2119
D 0.3446
E 0.4207
19 If X N(8, 4), then Pr(
Pr(X < 9.25) equals:
A 0.5311
B 0.6228
C 0.7340
D 0.9223
E 0.9991
20 If X is normally distributed with a mean of 13 and a
standard deviation of 4 and Pr(
Pr(x < k) = 0.71, then k
equals:
A 9.18
B 10.79
C 15.21
D 16.81
E 18.11

Maths Quest 12 Mathematical Methods CAS for the TI-Nspire

EXTENDED RESPONSE

1 Maya lives in Melbourne, and her sister Rosa lives in Perth. Every Saturday evening they talk on the phone.
The length of their conversations (in minutes) is a continuous random variable, X, with a probability density
function given by

0 . 09
0 9e 0 . 0 9 x , x 0
f x) =
f(
.
er
ere
0, elsewhere
a What is the probability, correct to 4 decimal places, that next Saturday the two sisters will talk for:
i more than 10 minutes?
EXAM TIP
Students should take care in writing an
ii more than 20 minutes?
integral that will yield the correct answer as part of
iii more than half an hour?
their working. Students often do not gain full marks
on questions if they do not show any working.
[Assessment report 2 2005 VCAA]

b What is the probability that the sisters will talk for less than three-quarters of an hour, given that they
have been talking for at least 20 minutes? Give your answer correct to 4 decimal places.
c Mayas son Michael lives in London. He always calls Maya every Saturday at 7 pm. What is the latest
time that Maya should call her sister Rosa to be at least 95% certain that they will have enough time to
chat (i.e. that they will finish by 7 pm)? Give your answer to the nearest minute.
d Mayas phone company charges 60 cents per call for all calls to Perth under 10 minutes duration, $1.40 per
call for calls over 10 minutes and under 20 minutes duration, and $2 per call for calls between 20 minutes
and 30 minutes duration. All calls with duration of over 30 minutes are capped at $2.50. (That is, a call of
any length above 30 minutes will cost $2.50.) Find the expected cost of Mayas call to the nearest cent.
2 Shoelaces sold as 50 cm are manufactured at a certain factory. However, the lengths actually follow a normal
distribution with a mean of 50 cm. Quality control managers at the shoelace factory check the lengths of all
laces manufactured and follow the rules below:
i If a lace is greater than 53 cm, it is classed as oversize and sold as a 55 cm lace.
ii If a lace is smaller than 47 cm, it is classed as undersize and sold as a 45 cm lace.
iii If a lace is between 47 cm and 53 cm, it is classed as adequate and sold as a 50 cm lace.
If 90% of laces are adequate, find:
a the standard deviation, correct to 2 decimal places
b the percentage of oversize laces
c the percentage of undersize laces
d the probability of a lace being adequate, given that it is not undersize, correct to 4 decimal places.
3 The time taken for Year 7 students to read a certain
page of writing follows a normal distribution
with standard deviation of 20 seconds. If 80% of
students read the page in less than 3 minutes, find:
a the mean time taken to read the page, correct
to 1 decimal place
b the probability that a student will take longer
than 3.5 minutes to read the page, correct to
4 decimal places
c the probability that a student will take between
2.5 and 3.5 minutes to read the page, correct to
4 decimal places
d the speed with which 90% of students exceed
in reading the page, correct to 2 decimal places.
4 A jeweller knows that the diameter of wedding rings follows a normal distribution, with a mean of 18 mm and
a standard deviation of 1 mm. Find the probability that a customer requires a ring with a diameter that is:
a greater than 20.5 mm
b less than 19 mm
c greater than 19 mm, given that it is less than 20.5 mm.

Chapter 12

Continuous distributions

649

5 The lengths of fish caught in a particular river are normally distributed with a mean of 28 cm. Ninety per cent
of fish caught are longer than 25 cm. Find:
a the standard deviation for the length of fish caught in the river
b the probability of catching a fish that is longer than 30 cm from this river
c the maximum length of fish that must be thrown back, if the shortest 30% must be returned to the river.
6 The amount of time, in hours, that Emily spends training for the Ironwoman Championships is a continuous
random variable, with probability density function given by
4
( x 1)3 , 1 < x < 4
f ( x ) = 81
.
0, elsewhere
a What is the probability that she spends less than 2.5 hours training per week?
b What is the mean number of hours she trains each week?
c What is the median time, correct to the nearest minute, she spends training each week?
7 Each night Kim goes to the gym or the pool.
a When she goes to the pool, the time she spends swimming is normally distributed with a mean of
60 minutes. Twenty per cent of the time she swims for less than 50 minutes. Find the standard deviation
in minutes, correct to 1 decimal place, of the time she spends swimming.
When Kim goes to the gym, the time, T hours, that she spends working out is a continuous random
variable with probability density function given by
4t 3 2 4t 2 + 4 4t 2 4, if 1 t 2
f (t ) =
.
er
ere
0, elsewhere
b Sketch the graph of y = f (t). Label any stationary points with their coordinates, correct to 2 decimal
places.
c What is the probability, correct to 3 decimal places, that she spends less than 75 minutes working out
when she goes to the gym?
d What is the probability, correct to 2 decimal places, that she spends more than 75 minutes working out
on 4 out of the 5 next times she goes to the gym?
e Find the median time, to the nearest minute, that she spends working out in the gym.
[[ VCAA 2006]

eBoo
k plus
eBook
Digital doc

Test Yourself
Chapter 12

650

Maths Quest 12 Mathematical Methods CAS for the TI-Nspire

eBook plus

ACTIVITIES

Chapter opener
Digital doc

10 Quick Questions: Warm up with ten quick


questions on continuous distributions. (page 589)
12A

Continuous random variables

Tutorial

WE1 int-0582: Watch a worked example on verifying


density functions. (page 591)
12B

Using a probability density function to


find probabilities of continuous random
variables

Tutorial

WE 5 int-0583: Watch a worked example on


sketching density functions and calculating
probabilities. (page 598)
12C

Measures of central tendency and spread

Tutorials

WE 7 int-0584: Watch a worked example on


calculating the mean, median and mode of a
probability density function. (page 605)
WE 9 int-0585: Watch a worked example on finding
the mean median and standard deviation of a
probability distribution using a CAS calculator.
(page 609)
12D

Applications to problem solving

Digital doc

WorkSHEET 12.1: Using the cumulative normal


distribution table. (page 618)
12E

The normal distribution

Interactivity int-0257

Digital doc

Spreadsheet 079: Investigate normal curves using a


spreadsheet. (page 623)
12G

The inverse cumulative normal distribution

Tutorial

WE 20 int-0587: Watch a worked example on


inverse normal calculations using a CAS calculator.
(page 636)
Digital docs

WorkSHEET 12.2: For normal distributions


calculate standard deviations, means and
probabilities. (page 642)
Investigation: Sunflower stems (page 642)
Chapter review
Digital doc

Test Yourself: Take the end-of-chapter test to test


your progress. (page 650)
To access eBookPLUS activities, log on to
www.jacplus.com.au

The normal distribution: Consolidate your


understanding of the normal distribution. (page 619)
Tutorial

WE 13 int-0586: Watch a worked example on using


confidence intervals to determine probabilities.
(page 620)

Chapter 12

Continuous distributions

651

EXAM PRACTICE 4
SHORT ANSWER

25 minutes

1 Student scores in an exam are standardised so that


the mean is 30 and the standard deviation is 7.
Students are awarded a C if their standardised score
lies between 30 and 37. Assuming that the student
results are normally distributed, estimate to the
nearest 5% the percentage of students who would
receive a C.
1 mark

2 Two events A and B from a given event space have


1
1
probabilities Pr(A) = and Pr(B) = .
6
4
1
a Calculate Pr(A B ) if Pr(A B) = .
10
1
b Calculate Pr(A | B ) if Pr(A B) = .
10
c Calculate Pr(A B ) if A and B are mutually
exclusive events.
d Calculate Pr(A B ) if A and B are
independent events.
4 marks

3 The probability of Karen Gouch shooting a basket


from the free throw line is 0.8, given her team
is winning the game, and 0.6, given her team is
losing. The team wins 7 out of 10 games.
a What is Karen Gouchs overall percentage of
baskets from the free throw line?
b If Karen throws a basket from the free throw
line, what is the probability that her team is
winning?
2 marks

4 Claude has a coffee shop. He sells coffee and


biscotti. He realises that if a person buys a coffee
on a particular day, there is a 75% probability
that the person will return and buy coffee the
next day and a 20% probability that they will buy
biscotti the next day. In addition, if a person buys
biscotti one day, there is a 60% probability that
they will purchase biscotti the next day and a 30%
probability that they will buy coffee the next day.
On Monday, 90% of Claudes patrons bought
coffee and 40% bought biscotti.
a Determine a transition matrix, T, that models
this situation.
b Determine the initial state matrix, S.
652

CHAPTERS 1 TO 12

c What is the probability that a patron will


purchase a coffee on Tuesday?
d Write down a matrix expression that could be
used to calculate the probability that a patron
will purchase a coffee on Friday.
4 marks

MULTIPLE CHOICE

10 minutes

Each question is worth 1 mark.


1 One normally distributed variable has a variance
of 4. Another has a variance of 0.25. Which of the
graphs below would best represent these variables?
A
0

10

10

10

D
4

10

E
4

10

2 Paddy Beam averages one ace in every 5 serves,


but each serve is independent of any other. In one
particular game Paddy serves 7 times. What is the
probability that 3 of the serves are aces?
1
A
5

7 1 4
B
3 5 5
3

7 3 4
C
3 7 7

Maths Quest 12 Mathematical Methods CAS for the TI-Nspire

5 7
3 1
12
4

7 5
3 1
12
4

3 A discrete random variable X has the following


probability distribution.
x

Pr(X = x)

0.1

0.1

0.4

0.3

0.1

The standard deviation of X correct to 2 decimal


places is:
A 0.80
B 1.04
C 1.08
D 1.16
E 1.35
4 If a random variable X has a probability density
ax 2 3 if x [ 1,3
, ]
function f ( x ) =
0, elsewhere
then a is:

39
1
C
28
10
1
18
D
E
8
26
5 A random variable X has a normal distribution with
mean 12 and standard deviation 0.25. If the random
variable Z has the standard normal distribution, then
the probability that X is less than 12.5 is equal to:
A Pr(Z > 2)
B 1 Pr(Z < 2)
C Pr(Z > 2)
D 1 Pr(Z > 2)
E Pr(Z < 2)
A 1.29

EXTENDED RESPONSE

25 minutes

The graph of the probability density function for the standard normal distribution (mean = 0, variance = 1) is
plotted below.
y
x
1
e 2
It has the rule f: R R, where f ( x ) =
.
0.5
2
a Evaluate:
2.5 2 1.5 1 0.5 0
0.5
1
1.5
2
2.5
i f (2)
0.5
ii f (1)
to an accuracy of 3 decimal places.
2

1 + 1 = 2 marks

b Plot and label the corresponding coordinates on the graph above.

1 mark

c Exact integration of f is not possible. However, it is possible to obtain an approximate value by using a
rectangular approximation. By using four intervals of equal width, determine a rectangular approximation to
the area bound by the graph of f, the y-axis, the x-axis and the line x = 1.

2 marks

d The area under a probability density function represents the probability. Use your answer to part c to
determine an approximate value for the probability that a value of x lies within 1 standard deviation from the
mean (that is, that x lies in the interval [1, 1]).

1 mark

e Use your CAS calculator to determine an approximation for the area defined in part c to an accuracy of
3 decimal places.

1 mark

f The infinite series approximation to ex is e x 1 + x +

x2

x3

x4

+ + K
2
6 24
Use this approximation to show that f (x) can be approximated by g(x) where
g( x )

1
2

(1

x 2 x 4 x 6 x8
+

+
)
2
8 48 384 .

g Sketch the graph of g for 2 x 2 onto the graph of f provided at the start of this question.

3 marks
2 marks

h Determine an approximation for


1

g( x) dx.

eBook plus

f ( x) dx by using calculus to evaluate

2 marks

Digital doc

Solutions
Exam practice 4

Exam practice 4

653

Answers
CHAPTER 1

Graphs and polynomials


Exercise 1A The binomial theorem
1 a x2 + 6x + 9
b x5 + 20x4 + 160x3 + 640x2 + 1280x + 1024
c x8 8x7 + 28x6 56x5 + 70x4 56x3 + 28x2 8x + 1
d 16x4 + 96x3 + 216x2 + 216x + 81
e 2401 1372x + 294x2 28x3 + x4
f 32 240x + 720x2 1080x3 + 810x4 243x5
3 1
2 a x3 + 3x + + 3
x x
b 2187x7 10 206x5 + 20 412x3 22 680x
15 120 6048 1344 128
+ x x 3 + x 5 x 7
+ 729
c x12 + 18x9 + 135x6 + 540x3 + 1215 + 1458
x3
x6
243
810
1080
720
2
5
+ 240 x 32 x
d 10 7 + 4
x
x
x
x
3 a i 21
ii 1
iii 0
b i 40
ii 80
iii 80
c i 0
ii 810
iii 0
d i 0
ii 540
iii 0
e i 0
ii 302 526
iii 0
4 A
5 C
6 D
7 D
8 540x3y3
19 683 x 2
9
4
10 4860
11 160
12 54
13 105
14 6
Exercise 1B Polynomials
1 ii, iii, vi
2 a x5 2x4 2x2 3x + 7
b x5 3x4 8x3 11x2 + 4x 1
c 3x4 16x3 8x2 x + 24
d 17 18x + 29x2 + 24x3 + 5x4 x5
3 a i 6
ii 0
iii 124
b i 7
ii 8
iii 280
c i 6
ii 3
iii 331
d i 4
ii 7
iii 55
4 B
5 a = 4, b = 8
6 a = 3, b = 5
7 a = 0, b = 7
8 a = 2, b = 5
9 a C
b D
Exercise 1C Division of polynomials
1 a 50
b 171
c 465
2 a

b
3 a

654

i 50
iii 465
The result is the same.
No
b Yes

Answers

4
0

14
7

19

2
3
4
5

6

7

8

a
a
D
a
c
a
d
a
d
a

y 3x + 5 = 0
3x + y + 13 = 0

b 2x + y + 5 = 0
b y - x + 2 = 0

i 1
2x + y = 0
iii
vi
[2, )
[2, 3]

b y 3x + 7 = 0

ii 8
b
e
b
e

( 12, 15) y

v
i
(5, )
R
b


5 x

i (, 5) ii (, 0)

d Yes

iv
ii
[2, 3)
(, 6)

2
8 x
(12, 3)

i [12, 12] ii [3, 15]


y
c
)
(5, 14

0
2

c
f
c
f

(2, 23 )

3 20
27

1
2

iv
iv
iv
iv

ii 171
iv 3 20
27
c No

4 a C
b B
5 a (x 2)(x + 3)2
b (3x 1)(x + 2)(x 6)
c (x + 2)(x 2)(x + 3)(x 1)
d 4x(x + 1)(x + 4)(x 2)
6 a x = 3, 0 or 2
b x = 4, 3, 0 or 2
c x = 2, 32 or 2
d x = 1
7 a = 2
8 a = 5
9 a = 7
10 a = 10, b = 26
11 a = 9, b = 11
Exercise 1D Linear graphs
y
y
1 a
b

3
2

i [2, 5) ii [ 23 , 143 )

9 a 2x + y 9 = 0
b x + 3y 17 = 0
10 a y 2x 7 = 0
b x + 2y + 1 = 0
11 E
12 B
Exercise 1E Quadratic graphs
1 a 0
b 2
c 0
d 2
e 2
f 1

2 a

1
2

1 13
5 x

c (1 12 , 12 14 )

4



5


6

a
b
c
d
a
b
c
a

i
i
i
i
i
i
i

(0, 2)
(6, 0)
(2, 0)
(3, 6)
y = x2 2x 1
y = x2 4x + 1
y = 8 + 2x x2
y

ii
ii
ii
ii
ii
ii
ii

1 12

y 2
y0
y 0
y 6
[2, )
[3, )
[16, 9]
x
2 12

2 4 6 x

i [10, 6]

ii [4 14 , 128]
y

(1, 8)

x
5 4 3 2 1 0 1 2 3
(3, 4)
50
( 5, 100)

100
f(x)  5 6x 3x2


12 a
13 a
c
14 a
d
e

15

3
4]

y
100

f(x)  x2 3x 2
10 8 6 4 2 0

iii
iii
iii
iii
iii
iii
iii

112

ii (,

1
1
50 (12 , 4 4 )
(6, 16)

y
1

y = x2 x 1, x R

( 25 , 49 )
1
1
(12
, 12 24
)

R
R
R
R
R
[1, )
[4, 4)
b

4 x

12

3 a (3, 1)

i R+
c ( 10, 128)

10

1 3
4 (2 , 4 )
6

4
0

i [5, 3)
ii [-100, 8]
8 m3
b 80 m3
48 m
b 6 s
Domain = [0, 6], range = [0, 48]
12 m
b 6 s
c 48 m
Domain = [0, 11], range = [12, 48]
h
48 (0, 48)

(11, 37)

36

24
12
1 0.09

b C
y

( 4, 9)

11 t

Exercise 1F Cubic graphs


1 a y = x(x + 6)(x 5)
b y = (x + 2)2(x 1)
2 a v
b iv
c ii
d i
e vi
f viii
g vii
h iii
y
y
3 a
b

7 a B
8 D
9 A
10 a

(6, 12)

2.91 x

1.5

12

1 0

2 1 0

x
1 0

4
7

(2, 4)

y
3

11 a
( 2, 10)

1 0

18
3

24

x
3

1 0

(1, 8)

y
10

10

4 a B
5 C
6 A
10 a

2
5 y = x 2x 2
(2, 2)
2 (1, 1)

2 1 0

i [2, 2]

7 B

b E
8 D

2 3

9 E

ii [1, 10]

i [2, )

1 2

Answers 1a 1f

3
( 2, 1)

ii [0, )

Answers

655

y
4

( 1, 0)
1 0

2
3

( 2, 32)

i [2, 1]
y
c
0

ii [32, 0]
4
3

(1, 8)

2
y
36

3 x

36 (0, 36)

32 0

2 a

2
3

ii (36, 8]

i (0, 1]
y
d

(1.15, 2.08)

( 1, 4)
x

(0, 0)

(2, 0)

(3, 0) x

(2, 14)

i [1, 2)
y
e

ii (14, 4]

(2.59, 1.62)

(3, 33)

b
( 1, 0)

( 1, 3)

(1, 0)
x

( 2, 12)

i [2, 1) (0, 3]
y
f

(0, 1)

ii [12, 3) (0, 33]

( 1, 3)
0

(1, 3)
(2, 18)

y
( 0.22, 3.23) (0, 3)

(3, 57)

(1,

i
1) [2, 3)
ii (57, 18] (3, 3)
24
11 a = 6, b =
12 a = 7, b = 4
13 a a = 3, b = 3, c = 3 and f (x) = 3(x 3)3 + 3
b g(x) = 3(x + 3)3 + 3, domain = [4, 2], range = [0, 6]
c 7 cm
14 a a = 14 , b = 5
(5 t )
, domain = [0, 5]
b d (t ) = t 2
4
c d (km) d(t) = t2
5 t

( 3, 0)
( 1, 0)

0 (1, 0)

( 2.28, 9.91)
y
(0.25, 8.54)

5
4
3
(2, 3)
2
1
0 1 2 3 4 5 t (h)

( 2, 0)

d Max. d 4.6 km when t = 3 h and 20 min


Exercise 1G Quartic graphs
y
1 a
b

656

Answers

32

(1, 0)

24

3 1 0

2 0 1 2

3


4

a
c
e
a
b

E
b B
A
d D
D
f C
y = (x + 2)(x 1)(x 3)(x + 1)

y = 12 (x 2)2(x + 1)(x 4)

5
2

5 a

3 2

200

i [2, 3] ii

[30,

100

2 1

(3, 30)

y
400
300

( 1, 36)

24

( 2, 400)

0] i

0 1

(2, 1]

0
1
4

ii [36, 400)
y

Multiple choice
1 E
2
5 C
6
9 E
10
13 E
14
17 A
18
21 C

3
7
11
15
19

B
C
D
B
E

4
8
12
16
20

A
C
A
C
D

Extended response
1 a y = 2.24 (x + 2.5)2 + 4
b Domain [5, 0], range [4, 18]
c (1.83, 5)
d f : [3.17, 1.83] R, f(x) = 5
2 a 0 km
b d (1000 km)

(1, 4)

( 3, 45)

(2, 16)
0

i (, 2] ii (, 16]i [3, 2] ii [45, 0]


6 a = 4, b = 19
7 a = 1, b = 7
8 a = 3, b = 1
Chapter review
Short answer
1 a 32y5 240y4x + 720y3x2 1080y2x3 + 810yx4 243x5
x8
x6 7 x 4
224 448

+
14 x 2 + 70 2 + 4
b
256
8
4
x
x
512 256
+ 8

x6
x
2 a = 15, b = 2
3 a (x 9)(x + 2)(x 5) b (2x 3)(x + 1)(x + 6)(x 2)
4 a 7x + 6y 1 = 0
b x + 2y 9 = 0
y
5
( 1, 9)

Domain = R, range = (, 9]
y
6
0

1
3

)
( 43 , 25
3

c d = t3 6t2 + 9t
d The satellite by 40 000 km
e 10 days
f Domain = [0, 10], range = [0, 490]
a y = 3 0.75x2
b y = 1.3125, so cannot fit.
c Reduce the height by 0.4 m.
x (m)

Finishing
line

(2.1, 8.8)

10

(3.8, 5.24)

5
(0, 2.4) (0.3, 2.6)
0

(5.1,0)
4

2 3

Liney: Starts 2.4 m in front of start line moving


forwards at a constant 0.75 m/min. Passed by Cubric
after 0.3 min and 2.6 m from the start line, meets
Cubric coming back towards Liney at 3.8 min and 5.24
m from the start line.
Quadder: Starts at start line, travelling 1.3 m the wrong
way for 2.55 min, stopping momentarily then moving
forward with increasing speed. Meets Cubric at the start
line after 5.1 min.
Cubric: Starts at start line moving very fast towards the
finish passing Liney at (0.3, 2.6), then slowing, stopping
momentarily at (2.1, 8.8), then moving back towards the
start. Meets Liney at (3.8, 5.24) and Quadder at the start
line at 5.1 min. Slows down, stopping at (7.37, 5.8) and
then speeding for the finish.
Q

10.1

y
2

10
9.9

16

10 t (min)

(7.37, 5.8)

18

1 0

(2.55, 1.3)

x (m)
10.2

L
Q

(10.1, 10.1)
(10.07, 10)

(10.092, 10)
(10.0899, 9.97)

(10.04, 9.9)
10

L
(10.13, 10)

Answers 1g 1G

Range = [ 25
, 0]
3
7 a a = 9, b = 2
y
b
3

(3, 0)
t (days)

10

D
B
D
B
A

10.15 t (min)

Answers

657

The Finish: Quadder overtakes Limey at (10.04, 9.9).


Cubric overtakes Limey at (10.0899, 9.97).
Quadder finishes the race in 10.07 min, Cubric in
10.09 min, Liney in 10.13 min.
Quadder wins by 1.2 s, Cubric second by 2.4 s to Liney.
5 a C is (3, 0) and D is (2.25, 8.54)
b y = x4 3x3
c 2 km
d Yes, because a straight route from O to D to C is
approximately 17.4 km and the river course is longer
than this.
6 a y = 14 (x + 3)(x + 1)(x 1)(x 3)





7



8

b (4, 26 14 ) and (4, 26 14 )


c Domain [4, 4]
d (2.236, 4) and (2.236, 4); range [4, 26 14 ]
e y = 14 (x 2.236)2(x + 2.236)2
f Domain [4, 4], range = [0, 30 14 ]
Teacher to check the model
2(
x x 75)
a y =
6250
b 46.72 m
c Seems to be extremely low 2 km from touchdown.
Not very accurate.
3(
x 4)2 ( x + 4)2
a y =
256

b y =

3(

x 4)2 ( x + 2)
32

x2
3
4
( x 4)2
ii y =
; the gradients are 1 and 2,
2
respectively, so not smooth.
iii Teacher to check
d Cubic is the closest, 1.6875 m.
c

CHAPTER 2

Functions and transformations


Exercise 2A Transformations and the parabola
1 a Dilation by a factor of 2 from the x-axis
b Dilation by a factor of 13 from the x-axis
c Dilation by a factor of 3 from the x-axis, reflection in
the x-axis
d Translation 6 units down
e Dilation by a factor of 12 from the x-axis, reflection in
the x-axis, translation of 1 unit up
f Translation of 2 units to the right
g Reflection in the x-axis, translation of 3 units to the left
h Dilation by a factor of 2 from the x-axis, translation of
3 units to the right
i Translation of 2 units to the left, translation of 1 unit
down
j Translation of 0.5 unit to the right, translation of
2 units up
k Dilation by a factor of 2, reflection in the x-axis,
translation of 3 units to the left, translation of
1 unit up
l Dilation by a factor of 12 from the x-axis, translation
of 1.5 units to the right, translation of 0.25 units down
2 D
3 a (ii)
b (v)
c (i)
d (iv)
e (iii)

658

Answers

4 a
c
5 E
6 a
c
e
7 a
b
c

y = 12 (x 2)2 + 2
y = 3(x 1)2 + 3

b y = 2(x + 1)2 2
d y = (x + 2)2 4

y = 12 x2
y = (x 2)2 1
y = (x + 3)2
y = (x 3)2 4
y = 2(x + 1)2 + 1
y = 13 (x + 3)2 4

b y = x2
d y = 3x2 2

y = 12 (x 2)2 + 2
y = 3(x 1)2 + 6
y = 4(x + 2)2 + 8
y = 2x2
y = 2x2
y = 2(x + 1)2
y = 2(x + 1)2 + 3
(x, y)
(x, y)
(x, 3y)
(2x, y)
x
( , y)
3
f (x + 2, y)
g (x, y 1)
10 a z = 3 or z = 15
2
b y = 2(x 3)2 8 or y = 25
(x 15)2 8
11 a 3

b y = 13 (x + 4)2 + 3
c x = 7, x = 1
12 1. f(x + 2) 3, 4 x 0
2. f(x 2) 3, 0 x 4
3. f(x + 4), 6 x 2
4. f(x 4), 2 x 6
5. f(x + 4) + 6, 6 x 2
6. f(x) + 6, 2 x 2
7. f(x 4) + 6, 2 x 6
8. f(x + 2) + 9, 4 x 0
9. f(x 2) + 9, 0 x 4
Exercise 2B The cubic function in power form
1 a Dilation from the x-axis by the factor of 7
b Dilation from the x-axis by the factor of 23 , reflection
in the x-axis
c Translation by 4 units up
d Reflection in the x-axis, translation by 6 units up
e Translation by 1 unit to the right
f Reflection in the x-axis, translation by 3 units to the
left
g Dilation from the x-axis by the factor of 4, reflection
in the y-axis, translation by 2 units to the right
h Dilation from the x-axis by the factor of 6, reflection
in the x-axis, reflection in the y-axis, translation by
7 units to the right
i Dilation from the x-axis by the factor of 3, translation
by 3 units to the left, translation by 2units down
j Dilation from the x-axis by the factor of 12 , reflection
in the x-axis, translation by 1 unit to the right,
translation by 6 units up
k Dilation from the x-axis by the factor of 2, translation
by 25 units to the left
l Dilation from the x-axis by the factor of 14 , reflection
in the x-axis, translation by 8 units to the left,
translation by 3 units up
2 a i, iv
b iii, v
c ii
d i, ii, iv
e ii, v
f iii, iv



8



9



d
e
f
a
b
c
d
a
b
c
d
e

3 a i (0, 0)
ii (0, 0)
iii There are no translations, but there is a dilation of
3
4.
b i (0, 1)
ii (0.8, 0), (0, 1)
iii There is a reflection in the x-axis, a translation of
1 unit up, and a dilation of 2.
c i (0, 6)
ii (2.08, 0), (0, 6)
iii There is a translation of 6 down and a dilation
of 23 .
d i (4, 0)
ii (4, 0), (0, 128)
iii There is a translation of 4 right and a dilation of 2.
e i (2, 0)
ii (2, 0), (0, 4)
iii There is a reflection in the x-axis, a translation of
2 right, and a dilation of 12 .
f i (1, 0)
ii (1, 0), (0, 4)
iii There is a reflection in the y-axis, a translation of
1 right, and a dilation of 4.
g i (1, 2)
ii (0.3, 0), (0, 1)
iii There is a translation of 1 right and 2 up.
h i (2, 3)
ii (0.6, 0), (0, 5)
iii There is a reflection in the x-axis and a translation
of 2 left and 3 up.
i i (1, 6)
ii (0.4, 0), (0, 4)
iii There is a translation of 1 left and 6 down, and a
dilation of 2.
y
y
3 a iv

b iv
1

0.8
x

c iv

d iv

y
4

2.08

128

e iv

f iv

y
4

g iv

h iv

0.6

i iv

0.4
4
6

2
5

c
e
a
c

1
2

x3

5 C

y = (x 3)3 1
y = (x + 1)3 1
y = 2x3
y = 2(x 2)3

9 a y =

1
2

d y = 2x3 + 3
b y = 2x3
d y = 2(x 2)3 1

x3 + 4

b y = 2(x 1)3 + 2

c y = 3(x + 1)3 + 1
e y = 4(x + 1)3 12
10 E

12 a y = 12 (2 x)3 + 1

13 a ( 3, 1) or (1, 27)
y
b
3
y  (x 3) 1

6 B
b y = (x + 5)3

d y =

1
3

(x 3)3

11 y = 2(x + 1)3 4
b Positive cubic

28
27

y  (x 1)3 27
4 3 1

Exercise 2CThe power function (the hyperbola)


1 a Dilation from the x-axis by the factor of 2
b Dilation from the x-axis by the factor of 3, reflection
in the x-axis
c Translation by 6 units to the right
d Dilation from the x-axis by the factor of 2, translation
by 4 units to the left
e Translation by 7 units up
f Dilation from the x-axis by the factor of 2, translation
by 5 units down
g Translation by 4 units to the left, translation by 3units
down
h Dilation from the x-axis by the factor of 2, translation
by 3 units to the right, translation by 6 units up
i Dilation from the x-axis by the factor of 4, reflection
in the x-axis, translation by 1 unit to the right,
translation by 4 units down
2 a v
b iii
c i
d v, iii
e v, ii, iii
f i, iii
g v, i, iv
h ii, iv
3 a i x = 0, y = 0
ii Domain: R \ {0}
iii Range: R \ {0}
b i x = 6, y = 0
ii Domain: R \ {6}
iii Range: R \ {0}
c i x = 2, y = 0
ii Domain: R \ {2}
iii Range: R \ {0}
d i x = 3, y = 0
ii Domain: R \ {3}
iii Range: R \ {0}
e i x = 0, y = 4
ii Domain: R \ {0}
iii Range: R \ {4}
f i x = 0, y = 5
ii Domain: R \ {0}
iii Range: R \ {5}
g i x = 6, y = 2
ii Domain: R \ {6}
iii Range: R \ {2}
h i x = 2, y = 1
ii Domain: R \ {2}
iii Range: R \ {1}
i i x = n, y = m
ii Domain: R \ {n}
iii Range: R \ {m}
4 a i x = 4, y = 0
ii Domain: R \ {4}
iii Range: R \ {0}
b i x = 0, y = 2
ii Domain: R \ {0}
iii Range: R \ {2}
c i x = 3, y = 2
ii Domain: R \ {3}
iii Range: R \ {2}

Answers

Answers 2A 2C

2
1
0.3

4 E
7 a y =

659

d i
iii
e i
iii
f i
iii
5

x = 1, y = 1
Range: R \ {1}
x = m, y = n
Range: R \ {n}
x = b, y = a
Range: R \ {a}
y

ii Domain: R \ {1}
ii Domain: R \ {m}

y4

ii Domain: R \ {b}

34 x

7 E

x
x

4 x
3

6 a

1
3

21

b y =

1 5

43

2 21

3
2
x
1 121

11 1

f
2
5

2
5

1
(0, 1
)
2

y1

x
0

31

y 1

23

Answers

x 2
1 3
4 4

11

Domain: R \ {0}, range: R \ {3}

3
y
10
8
6
4
2

1
3

y

x 3
x 2

10 8 6 4 2 0 2 4 6 8 10 x
4
(0, 3
)
2
6
x 2
8
10

660

(3, 0)

4 31

185

y
4

1
2

1
3

21

52

1
7 2

1
2

3
+1
x

3 43

4
1
x
6
f y =
1
x +1
y
b
d y =

2 1

8 C

2
9 a y =

x2

c y = 3
x+4
2
e y =
+ 2
x4
y
10 a

2 x
3
2
3

(2, 11)

0 2 4
6 4 2
(0, 3)
x1

y
10
8
6
4
2

Exercise 2DThe power function (the truncus)


1 a Dilation from the x-axis by a factor of 2
b Dilation from the x-axis by a factor of 3, reflection in
the x-axis
c Translation by 2 units to the left
d Dilation from the x-axis by a factor of 2, translation
by 3 units to the right

e Dilation from the x-axis by a factor of 5, reflection in


the x-axis, translation by 4 units to the left
f Dilation from the x-axis by a factor of 2, translation
by 6 units up
g Reflection in the x-axis, translation by 3 units up
h Dilation from the x-axis by a factor of 4, translation
by 3 units to the right, translation by 1 unit up
i Reflection in the x-axis, translation by 2 units to the
left, translation by 5 units up
2 D 3 C
4 a i x = 0, y = 0
ii Domain R \ {0}
iii Range: y > 0
b i x = 0, y = 0
ii Domain R \ {0}
iii Range: y < 0
c i x = 2, y = 0
ii Domain R \ {2}
iii Range: y > 0
d i x = 1, y = 0
ii Domain R \ {1}
iii Range: y > 0
e i x = 4, y = 0
ii Domain R \ {4}
iii Range: y < 0
f i x = 0, y = 3
ii Domain R \ {0}
iii Range: y > 3
ii Domain R \ {0}
g i x = 0, y = 45

iii Range: y >

4
5

h i x = 0, y = 12

iii Range: y <

ii Domain R \ {0}

i i x = 1, y = 4
ii Domain R \ {1}
iii Range: y > 4
5 C
6 B
7 B
y
y
8 a
b
1
9

1
8

1
2

0.4

0.4

2
1

3.7 3 2.3

11

12
2
3

2.7

1
41

1
2

0.7

3
2

1 2 3

1
+ n
( x m) 2
1

c y =
( x r )2

b y = ( x + q)2 + p
d y =

1
+t
x2

1
b
( x + a)2
1
g y = 2 k
x
2
10 a y = 2 2
x

f y =

1
e
( x g)2

9 a y =

e y =

c y =
e y =

9
+ 1
( x + 2)2

8
3
( x 4)2

3
3
( x + 2)2

1
l
x2

3
b y =
( x 2)2
3
d y =
+4
( x + 1)2

h y =

5
2
( x 1)2
3
12 y =
+2
( x 1)2

f y =

Exercise 2EThe square root function in power


form
1 a Dilated from the x-axis by a factor of 2
b Dilated from the x-axis by a factor
of 13 , reflected in the x-axis
c Dilated from the x-axis by a factor of 3, translated
1 unit to the right
d Dilated from the x-axis by a factor of 2, reflected in the
x-axis, translated 4 units to the left
e Translated 1 unit down
f Dilated from the x-axis by a factor of 3, reflected in
the x-axis, translated 2 units up
g Translated 4 units to the right, translated 3 units up
h Dilated from the x-axis by a factor of 2, reflected in
the x-axis, translated 3 units to the left, translated
6 units up
i Dilated from the x-axis by a factor
of 12 , reflected in the x-axis, reflected in the y-axis,
translated 2 units to the right and 23 units up
2 a (0, 0)
b (0, 0)
c (1, 0)
d (4, 0)
e (0, 1)
f (0, 2)
g (4, 3)
h (3, 6)
i (2, 23 )
3 E
4 D
5 a Domain: x 1, range: y 0
b Domain: x 3, range: y 0
c Domain: x 0, range: y 3
d Domain: x 0, range: y 4
e Domain: x 0, range: y 5
f Domain: x 1, range: y 3
g Domain: x 2, range: y 1

h Domain: x 12 , range: y 4



i
j
k
l

Domain: x 43 , range: y 2
Domain: x 3, range: y 7
Domain: x 2, range: y 6
Domain: x 2, range: y 1

Answers

Answers 2D 2E

4
3 97

1
3 4

11 y =

1
2

661

6 D
8 a

7 D
b

1.4

(6, 1)

3 43

1 21

y
4.4
2

3
2

2
3





4

C
a
b
c
d
e
f
a

Domain: R, range: y 0
Domain: R, range: y 1
Domain: R, range: y 4
Domain: R, range: y 2
Domain: R \ {1}, range: y > 1
Domain: R \ {0}, range: y 0
y

b
x

1
2

3.7
2

11

y = 3 4x + 3

( 1, 1)

b y = 4 x + 1 + 8
d x 1
y
f

12 a p = 8
c x = 3
e y 8

2.7

Exercise 2F The absolute value function


1 a

b
y

41

1
4

6
6

x
1 3 5

2 x

35 63

( 1, 6)

3 x 1, x 13
5 a f ( x ) =
1
3 x + 1, x < 3

5
4

Answers

2
5

3.6
3

1
x

1
2

0.7

3 31
3

4
3

43

( 1, 8)

y
2 1

34

(1, 1)
x

2
(4, 3)

1
2

y
2

b y = 2 x 1 4

y
7

9 E
10 a m = 1

5 1 1

11

1 2

662

y
0.4

2 1

2
4

7
1

b f(0) =3 0 + 1 = 1, f(2) = 3(2) 1 = 5

99

y
7
6
5
4
3
2
1

f(x)  |3x 1|

3 2 1

x 2 3 x + 2, x 0 x 3
6 a f ( x ) = 2
x + 3 x + 2, 0 < x < 3
b f(1) = (-1)2 3 1 + 2 = 6,
f(2) = 22 + 3 2 + 2 = 4
c
y
2
f(x)  |x 3x| 2

7
6
5
4
3
2
1
0

7 a y =

3
2

x , 2 x 2

b Yellow: y = 6

3
2

x , 2 x 2;

green: y =

3
2

x 6, 2 x 2;

blue: y =

3
2

x , 2 x 2

c Teacher to check
Exercise 2G Transformations with matrices
1 i Reflection in the y-axis, dilation by a factor of 2 from
the x-axis
ii Dilation by a factor of 12 from the y-axis, dilation by a
factor of 4 from the x-axis, reflection in the x-axis
iii Reflection in the y-axis, dilation by a factor of 3 from
the x-axis, dilation by a factor of 2 from the y-axis
iv Dilation by a factor of 12 from the x-axis, reflection in
the x-axis

2 i (3, 10) ii ( 32 , 20 )
iii (6, 15)

2
x2
1
ii y =
x2

b i y = 2 x 3 10

iv (3,

5
)
2

ii y = x 2 2
6 a i y = x 3 + 2
b i y = x 2 9 x + 20
ii y = x 2 7 x + 8
7 C
8 a Reflection in the y-axis, dilation by a factor of 2 from
the x-axis, translation horizontally by +6, translation
vertically by +1

2
+2
f ( x ) =
( x + 6)
b Dilation by a factor of 2 from the y-axis, dilation by
a factor of 12 from the x-axis, reflection in the x-axis,
horizontal translation by +3, vertical translation by 1
1
y =
1
x3
c Horizontal translation by +1, vertical translation by 2,
13 dilation from the y-axis, reflection in the x-axis

1
+2
y =
3x + 1
0 x 1
+
y 3
10 a y = 2 x 3 + 8 x 1
b (2, 1); point lies on the curve
11 Dilation by a factor of 3 from the x-axis, reflection in the
x-axis, translation by 2 units in the positive direction of
the x-axis, translation by 1 unit in the positive direction of
the y-axis
1
x
9 T = 4
y 0

x 1
T =
y 0

0 x

2
+
1

3 y

12 a, b g(2( x + 1)) + 1 = 2( x + 1) + 1
13 a, b f(x) = 2x2 10x + 6
14 h(x) = x3 5
Exercise 2H Sum, difference and product
functions
1 a f(x) = x2 + x
y

3 a i y =

ii y =

32 x 3

c i y = 2

+ 20

2
1
3

0
1

dom f: R
x3
+ x
b f ( x ) =
2

Answers 2F 2H

ii y = 4 2 x
4 i Translation of 3 units in the positive direction of the
x-axis, translation of 2 units in the positive direction
of the y-axis.
ii Translation of 2 units in the positive direction of the
x-axis, translation of 2 units in the negative direction
of the y-axis.
iii Translation of 15 units in the negative direction of the
x-axis.
5 i (4, 0)
ii (3, 4)
iii ( 45 , 2)

y
3
2
1
3

0
1

dom f: [0, )

Answers

663

c f ( x ) = 3 x 2 + x

(2, 20.5)

y
1

1 1 0
2
3
( 2, 4) 4

2
1
1.5 1 0.5 0

0.5

1.5

6 a

( 2, 8)

6 4 2 2

2 a g( x ) h( x ) =

6 4 2

x +1 x

b
c
d
e
f
g

0
2

( 2, 2)

6 4 2 0
( 1, 2) 2

(4, 4.5)

664

1.5 1 0.5 0
2

0.5

(2, 2 )
1

(2, 6)
(2, 8)

g(x)
0.5

1.5

f (x)

g(x)
0.5
f (x)

1.5

f (x)g(x)

8 h( x ) = 8 yx + x + 5 , x [ 5, 8]
5
4
3
2
1

(2, 4)

a=0
f(0) = 0, g(0) = 0, h(0) = 0
f(1) = 1, g(1) = 1, h(1) = 0
f(2) = 4, g(2) = 2 , h(2) = 4 + 2

Answers

y
4
2

6
1

( 2, 4)

b
c
d
e

f ran h: [ 2, )
g

(2, 2)

c f(0) = 3, g(0) = 0, h(0) = 0


d f(1) = 2, g(1) = 1, h(1) = 2

e f(2) = 1, g(2) = 2, h(2) = 2

b R
d R

0
1
1
2
3
4

y
4

1.5 1 0.5 0
2

5 a

1 20

y
4

( 4, 4.5)

10
8
6
4
2

7 a dom h: [0, )
b

(2, 8)

4
6
8

y
( 1, 3)
4

( 2, 10)
( 2, 8)

b g( x ) h( x ) = x x + 1 + 2


3 a (, 3]
c [0, 1]
e [2, )

f(2) = 8, g(2) = 2, h(2) = 10


f(0) = 0, g(0) = 0, h(0) = 0
f(1) = 1, g(1) = 1, h(1) = 0
f(2) = 8, g(2) = 2, h(2) = 6
ran h: [6, 10]

0
2
4
8

y
4
( 1, 1)

(2, 2)

dom f: R\{0}

(2, 4)

3 2 1

6
4
2

(2, 20.5 4)

y
8

( 2, 2)

1
x
y

dom f: R

d f ( x ) = x + 2 +

6 5 4 3 2 10 1 2 3 4 5 6 7 8 9 x

y
h(x)
( 2, 2)
( 2.5, 1.38)

(2.5, 5.38)

6 (0, 6)
4

8 dom w: ( 3, ), ran w: (0, ), dom v: R+, ran v: (2, )


w v(x) = x + 1 , where x R+; v w(x) = x + 3 2,

f(x)

5 4 3 2 1 0
2
4

g(x)

Exercise 2I Composite functions and functional


equations
1 a ii f(g(x)) = 2 x + 3 1, domain [3, )
1
b ii f(g(x)) =
, domain R
|x|+3
c ii f(g(x)) = 3(x2 2)3, domain R
d ii f(g(x)) = x 3 , domain R

50
40
30
20
10

50
40
30
20
10
0
10

1 2 3 4 5

0
10

5 10 15 20 25 x

c f =

340

c a = 2, b = 3.2, y = 2x2 3.2


5 y = 14 x3 12
6 a

f
1000
800
600
400
200
0

2 4 6 8 10 L

f
1000
800
600
400
200
1
0 0.5 1 1.5 2 2.5 3 3.5
L

7 a

b I =

I
250
200
150
100
50
0

270
d2

4d

Answers 2i 2J

e ii f(g(x)) = (x2 + 1)(x2 + 3), domain R


2 Teacher to check
3 Teacher to check
4 a i f(x y) f(x) f(y)
ii f(x y) f(x) f(y)
iii f(x) + f(y) (x2 + y2) f(xy)
x
f ( x)
iv f( )) =
y
f ( y)
v f(xy) = f(x) f(y)
b i f(x y) f(x) f(y)
ii f(x y) f(x) f(y)
iii f(x) + f(y) (x2 + y2) f(xy)
x
f ( x)
iv f( )) =
y
f ( y)
v f(xy) = f(x) f(y)
c i f(x y) f(x ) f(y)
ii f(x y) f(x) f(y)
iii f(x) + f(y) (x2 + y2) f(xy)
x
f ( x)
iv f( )) =
y
f ( y)
v f(xy) = f(x) f(y)
d i f(x y) f(x) f(y)
ii f(x y ) f(x) f(y)
iii f(x) + f(y) = (x2 + y2)f(xy)
x
f ( x)
iv f( )) =
y
f ( y)
v f(xy) = f(x)f(y)
e i f(x y) f(x) f(y)
ii f(x y) f(x) f(y)
iii f(x) + f(y) (x2 + y2) f(xy)
x
f ( x)
iv f( )) =
y
f ( y)
v f(xy) = f(x)f(y)
f i f(x y) f(x) f(y)
ii f(x y) = f(x) f(y)
iii f(x) + f(y) (x2 + y2)f(xy)
f ( x)
x
iv f( ))
f ( y)
y
v f(xy) f(x) f(y)
5 a > 0
1
6 f g does not exist; g f(x) =
+2
x 2 +1
7 Restricted domain of g(x) = R\(1, 1)

where x (3, )
9 Teacher to check
10 Teacher to check
11 x = 4.4305 or 0.902832, maximum 4.23 when x = 4.43
12 Reflection in the y-axis, translated +1 parallel to the x-axis
Exercise 2J Modelling
1 a iii
b ii
c iv
d i
e v
2 a y = ax3, a = 0.3
b y = ax2, a = 6
a
d y = a x , a = 1.6
c y = 2 , a = 2
x
a
f y = ax3, a = 1.5
e y = , a = 5
x
3 D
y
y
4 a
b

8 y = 3 x + 4
9 a
$
11
10
9
8
7
6
5
4
3
2
1
0

Price

b p = 2 m + 4

8 10 Month

c 10.63, 10.93

Answers

665

12 a a = 8, b = 1

Chapter review
Short answer
1 a (3, 4)
b Domain: R, range: y 4
c
y
14

1 x
+2
8
(1 x )3 1 x
ii y =
+
+2
512
8

x
y
4 x2 2x
,y =
b y = 2
a
2
a
a
2
c a = 2, y = x - x
14 g:( 2, 1) R, g( x ) = x 3 + 2 x 2 + 1

b i y = 2

13 a x =

3
1.6

4.4

2 a b = 2, c = 1
b m = 7
3
c y =
+3
x4
7
3 f ( x ) = 2
3x + 1
Dilation 13 from the y-axis, dilation from the x-axis by a
factor of 7, reflection in the x-axis, translation

1
3

1
3

6 a Reflection in the x-axis, translation by 2 units to the


left and 1 unit down, dilation by the factor of 2 along
the y-axis
2
b y =
1
( x + 2)2
1
7 y =
2
( x 1)2
8 4 units to the right and 3 units up; y = 3
y
9 a

4x

g(x)
( 2, 1)
2

2.7 2

b Range [0, 20]


11 a = 3, b = 3

666

Answers

( x )+1
f

1
2

0
1

2
g

1
fg

16 a Yes
b No
c Yes
17 c 100
Multiple choice
1 D 2 E
3 C 4 D
5 C 6 E
7 D 8 D
9 E
10 C
11 A
12 C
13 E
14 B
15 C
16 E
17 C
18 D
19 E
20 E
Extended response
y
1 a i
D(4, 6)

d No

f(x)

B C(2, 3)
A 2
2
A' B'
2
C'(2, 3)

f(x)

b Domain: R \ {2}, range: (, 2]


10 a
y

f(x)

1
2

1.3

(1, 7)
( f g)(x)
(1, 4)
(1, 3)

15 g( x) = x + 1, ( fg)( x ) = x
y

( 1, 2)
( 3, 2)

y
7
6
5
4
3
2
1

parallel

to the x-axis in the negative direction, translation


+
2 parallel to the y-axis
4 y = 1 3(x 1)3
5 a x = 2, y = 1
b Domain: R \ {2}, range: R \ {1}

4
c y =
1
x+2
y
d
6

( f + g)( x ) = 2 x 3 + 4 x 2 + 1

D'(4, 6)

A(2, 0), B(0, 2), C (2, 3), D(4, 6)


y
ii
f(x)
f(x)
D'( 4, 6) D(4, 6)

x
C'( 2, 3)
A
2

C(2, 3)
A'
x
2

A(2, 0), B(0, 2), C(2, 3), D(4, 6)

4 a 55 m
b Maximum height is 30.01 m above ground and
maximum depth is 5.38 m.
c i Minimum depths and maximum heights would be
greater.
ii No change
iii Would be greater
5 a y = a(x b)2 + c
b b = 9
c Straight line (negative gradient)
d a = 0.55, c = 275
e y = 0.55(x 9)2 + 275
f No, the prices started going down.
g $266 000, $261 000
h About 4 months
6 a a = 40
b i The eagle is 48 m above the ground.
ii The eagle is 42 m above the ground.
c It takes 24.2 s to reach the ground.
d The speed is slow to start but increases rapidly as the
eagle approaches the ground.
e h

iii

f(x 2)
D (4, 6)
C(2, 3)
B2

A
2

f(x)
D'(6, 6)

C'(4, 3)
B'(2, 2)
x

A'

A(0, 0), B(2, 2), C(4, 3), D(6, 6)


iv
y
D'(4, 9)

A'( 2, 3)

C' (2, 6)
B'
5
B

f(x) 3 A
2

D(4, 6)

C(2, 3)
x

f(x)

A(2, 3), B(0, 5), C(2, 6), D(4, 9)


v
2f(x)
y
D'(4, 12)

D(4, 6)

B'4
A 2B
A' 2

50 (0, 48.4)
(5, 48)
40

f(x)

C'(2, 6)

C(2, 3)

30

20

A(2, 0), B(0, 4), C(2, 6), D(4, 12)


f(x)
y
vi
B
2

A'(3, 1)
A

D'(3, 5)


1),
C(1, 2), D(3, 5)
b Add multiples of 2, for example, f (x) + 2, f (x) + 4,
f (x) + 6, f (x) 2 etc. and keep the domain fixed at
[3, 7].
b p > 1, p <

25

f
x

y
30
20

1
0

2
3

4

10
1

1.5 1 0.5 0
10

0.5 1 1.5 2 2.5 3 3.5

4 4.5 x

Answers

Answers 2j 2j

3
2

1
e y = 2 + 1
4x

20

c ( 12 , 2), (1, 1.25),( 12 , 2), (1, 1.25)

15

x 3 or x 3
a Minimum value of -1
3
b x = 2
aLet P(x) = x3 - 7x - 6. P(-2) = -8 +14 - 6 = 0
so x - (-2) is a factor. Hence x + 2 is a factor.
b Q(x) = x2 - 2x - 4
1
5 c < 4

1
b 2 0 , 0

0 1 1

10

Exam practice 1
Short answer
y
1
f g

25
27

c i Dilation of k from the y-axis, translation of 1 in the


negative direction of the y-axis
ii x = k, x = 2k
d h [1, 2)
3 a Dilation from the y-axis by a factor of 12 , vertical
translation of +1

f a = 2.5, c = 10
40

, 0 t 24
50 +
g f (t ) =
t 25

2.5(t 24)2 + 10, 24 t 26

B(1, 1),

2 a a = 1, b = 35

(24.2, 0)
0

C(2, 3)
C'(1, 2)

B'(1, 1)

A(3,

10

D(4, 6)

1 f(x + 1)

(20, 42)

667

Multiple choice
1 B
2 C
3 E
4 B
5 D
6 C
Extended response
1 a v = 100t - 100
b 2:11 pm
c 3:42 pm
d It predicts that the wind speed will increase
indefinitely. It also predicts a negative speed at noon.
These problems could be avoided, for example,
simply by restricting the domain to be [1, 3.69].
e iIf the wind peaked at 5 pm, then (5, 256) would be
a turning point. In turning point form the rule for
the quadratic model would be v(t) = a(t - 5)2 + 256.
256
Since v(1) = 0, 0 = a(1 - 5)2 + 256 a =
= 16
16
so the model is v(t) = -16(t - 5)2 + 256. Using this
model, v(3) = -16 (-2)2 + 256 = 192 km/hour.
This is reasonably close to the recorded value of
200 km/hour.
iiThe predicted value is v(12) = -16 72 + 256
= -528 km/hour. Clearly the model is inadequate.
f i v2(t) = -8(x - 1)(x - 13)
iiCheck at t = 3. v2(3) = -8(3 - 1)(3 - 13) =
8 2 -10 = 160 km/hour. Compared with the
actual value of 200 km/hour, this model is not
adequate. Maximum wind speed predicted at
t = 7 is v2(7) = -8(7 - 1)(7 - 13) = -8 6 -6 =
288 km/hour, which is later and faster than the
actual data.
g v3(t) = x3 - 27x2 + 195x - 169
h i x = 250
1 875 000
ii v (t ) =
( x 250)2
625 000
iii P (t ) =
+ 1020
( x 250)2

CHAPTER 3

Exponential and logarithmic equations


Exercise 3A The index laws
1 a x7

b x5

e x

f 5x4y3

2 a 9

1
125
8
27

9
16

3 a x6
d 37

5 a

y4

b
b

1
y4

Answers

e log2

f logx (27) = 3

2
c 243 x
y2

3
8y 8

b 36n + 1
d 23
b 1
2x
c

1 x2

( 12 ) = 1

4 a 1
d 4
5 a

1
2

b 1
e 1

x2
c log2
y
c 2

b log3 (x)

log2 (x)

c log2 (10)
c
f
c
f
h

0.301
3.727
0
5 log10 (x)
log4 (x 2)

Exercise 3C Exponential equations


1 a 4
b 3
c 5
d 2
e 15
2 a 4
b 5
c 1
d 3
3 a 3
b 1
c 2
d 1
4 a 2, 0
b 2, 1
c 1, 0
d 3
e 1
5 a 0
b 1, 2
6 a 2.322
b 0.912
c 3.325 d 1.365

e 0.827 f 0.672
7 a x > 1.465
b x 1.404
c x < 0.683
d x 0.356
e x < 1.756
8 D
9 E

v 2

b i 81

c 5
f 3

6 a 1
b 1
d 1
e 9
7 a 4
b 3
d 2
e 12
8 a 0.477
b 0.861
d 0.322
e 2.161
9 a 0
b log2 (x)
d 0
e 3 log10 (x)
g log5 (x + 1)3 or 3 log5 (x + 1)
10 C
11 E
12 31.623

6 a 26n 1
c 2n 33n + 1
7 a 22n 32n + 1
1 + x2
( x + 1)2
b

8 a
x
x4
9 B

668

b log3 (81) = 4
d log5 (125) = x

Exercise 3D Logarithmic equations using any base


1 a i 25
ii 18
iii 100
iv 26

1
x6

c 216

5 5
6x 4 y 3

20 x

d 1
c 5x = 125

3 a log2 (8) = 3
c log4 (x) = 3

b 8
27
e 64

3
4 a 11 2
x y
x6

c x10
10 x 4 y 2
g
3
c 3

Exercise 3B Logarithm laws


1 a 0
b 0
c 1
2 a 24 = 16
b x2 = 25

d 35 = x
e 5 1 = 15

3
a 9b3
15 7
2a 2 b 2

1
d
x+y

v 0

2 a i
b i
3 a i
v
b i
v
4 a i
v
b i
v
5 a i
v
b i
v
6 B

3
2
3
1
2
2
5
4
2
128
5
3
2
3

vi

1
32

vii 624

ii

1
16
1
9

iii 16

vi

vii

iv 127

5
2

ii 125
ii 625
ii 1

iii 2
iii 8
iii 12

ii 2

iii

iv 0

ii 6

iii 10

iv 8

ii 9

iii

ii 6

iii 1

iv 2

ii 8

iii 5

iv 4

7 D

1
3

2
5

iv 6
iv 4
iv 0

iv 500

1
8 a 10, 100

b 1000,

e 3, 27
i 5, 3
9 16
Exercise 3E
1 a 7.389
e 1.284
i 0.405
2 a 1

f 25, 5
j 2, 5

1
10

1
4

c 16,

d 2,

g 1, 16

Exercise 3H Literal equations


a
1
1 x = 2 log e , a > 0
5

1
16

h 1, 27

2 D = Z 3cy
log e (3k ) n
3 x =
m
9 p
4 q =
5
5 Proof:

Exponential equations (base e )


b 54.598 c 1.649
d 1.396
f 1.221
g 1.386
h 1.609
j 1.281

b 2
c 6
d 12

e 2

f 0

11
6

h 1 14

3 a 0.693
b 1.609
c 0.693
e 0.262
f 0.956
g 1.099
4 a 0
b 0.693, 0.549 c
d 0
e 1
f
g 0.405, 1.099 h 0.251, 0.693
5 a 0.693
b 1.609
c
d 0.405, 1.099 e 0.288, 0.693 f
6 a x > 0
b x < 1
c
d x 0.693
e x 0.792
f
g x < 0.288
7 20.00, 0.01
8 0.04
Exercise 3F Equations with natural
logarithms
1
1 a e
b e2
c 2
e
2 a 3.695
b 0.906
c e
e 1.221
f 1.350
g 1.368
3 a 2
b 5
c 3
e 10
f 2.207
4 a 6
b 23
c 2.5
e 1
5 B
7 ex2
9 2
10 a 2.5

f 1.5

b e

kt

d 1.386
h 1.386
0.405, 0.693
0.693, 1.609

xy
xy
= log10 5 log10 5
z
z
= log10 (1)
= 0
4 9a
6 a x =
2b
7
b x =
3
log e 20 + 2b log e 2
7 x =
4 log e 2

(base e )
1
e
d e
h 0.432
d 4
d

d 6

8 x =

6 A
8 y = x
=

1
2

c t =

x
1 a y = loge
2

ln (2)
d 139 days
k

b y = loge (x) 1
1 + log e ( x )
d y =
2
2 log e ( x )
f y =
3

c y = loge (x) + 1
e y = 2 loge (x)

d y = loge (x 2) 1

d f ( x ) =

( )

f y = loge 2 x 1
3

b f(x) = e x 1

e +1
e f(x) = 2 ex
2
x

4 a y = ex 2

b y = e2 x

d f(x) = e2 x + 1 e y
5 C

x3
= 1+ e 2

6 E

c f(x) = e x + 1
f f ( x ) =
c y =

2e
3

x2
e 3

f f ( x ) = e
7 D

1
y m
log e
R
b

10 a =

16b
1
or a =
5
5 16b

11 x =

3 + 9 + 4a
1
log e

k
2

a2
e2
a x = 1.512 or x = 0.691
b 0.443
c 1.058
a x = 1.557
b x = 2.159 and x = 5.146
c x = 0.191 and x = 1.433
log ( p )
x = 7 log ( 4 )

1 x
3

14


15

16

Exercise 3I Exponential and logarithmic modelling


1 a 200
b y = 200e 0.6t c 110 g
1
d 2 4 hours
2 a 10 g

d 13 days

b y = 10e

0.18t

Answers 3a 3i

c y = loge 1 x
2
e

2
log e
3

9 x =

13 b =

b y = loge (2 x)

( )
e y = 2 + log ( 3 x )
2

a log e ( 3) + log e ( 2 )

12 x = 9 y 6

2 a y = loge (x 2)

3 a f ( x ) =

z2x5
x2y4
y
= log10 3 5 log10 3 4 + log10 4
z x
z
y z x
x2y4
z2x5y
= log10 3 5 log10 7 4
y z x
z x

1.099
0, 0.693
x < 0.693
x 1.303

Exercise 3G Inverses

x
e2

z2x5
x2y4
y
log10 3 5 log10 3 4 log10 4
z x
z
y z x

c 8 g

Answers

669

3 a 3, 2

b y = 3 + 2 loge (x)
c 5.197
d 10
4 a 1000, 0.05
b A = 1000e0.05t c 1051, 1649
d 14
5 a 500
b 0.03
c P = 500e0.03t
d 2240
e 1896

6 a 15 C, 90 C, 0.05
b T = 15 + 75e 0.05t
c 60 C
d 22
e 17
7 a 50, 0.09
b D = 50e0.09t
c 78 cm
d 8 years

8 a 100
b 0.002
c M = 100e 0.002t
d 90 g
e 347 years
9 a 0.23
b N = N0e0.23t
c 1000
d N = 1000e0.23t e 6000

10 a 0.1386
b I = I0e 0.1386d c 25%
d 16.6 metres
Chapter review
Short answer
1

33
x5

Exponential and logarithmic graphs


Exercise 4A Graphs of exponential functions
with any base
1 a Asymptote y = 0,
b Asymptote y = 0,
domain R, range R+
domain R, range R+
y

(0, 1)

(0, 1)
01 2 x

3 2 1

f (x) =

(1, 2)

3 3

4 1 23

5 8

7 4

6 2 5 ln (2)

( )

1
2

f (x) = 10x

c Asymptote y = 0,
domain R, range R+

d Asymptote y = 0,
domain R, range R+

y
(0, 3)

2 (0, 2)

( 1, 121) 2

f (x) = 2 10x
e Asymptote y = 0,
domain R, range R+

0 1

f (x) = 3

2x

(2, 1.2)
0 1

2 1

f (x) = 0.3

2
(0, 21 )

y
8

f (x) = 0.5

y
(1, 8)

8
6
4
2

(0, 1)
0
1

(12 , 10)

(0, 1)
0 0.5

0.5

Asymptote y = 0,
domain R, range R+
y
6

(0, 5)

4 (0, 4)

3 2 1 0 1 2 3

f (x) = 5

f (x) = 4

k Asymptote y = 0,
domain R, range R+

y
4

x
10 2

23x

Asymptote y = 0,
domain R, range R+
y
6

(3, 4)

2 (0, 2)

f (x) = 2

01 2 3

3 2 1

32x

3 2 1 0 1 2 3

f (x) = 102x

f (x) = 23x
Asymptote y = 0,
domain R, range R+
y
6

10x

h Asymptote y = 0,
domain R, range R+

0 1

2 1

2x

g Asymptote y = 0,
domain R, range R+

(1, 5)

(0, 0.3)

2 x

Asymptote y = 0,
domain R, range R+

k R+
8

b
13
,a 0
a
14 z 2y, x, z R+
Multiple choice
1 E
2 A
3 D
4 B
5 A
6 D
7 B
8 C
9 B
10 E
11 B
12 D
13 B
14 C
15 E
16 B
17 E
Extended response
1 0.41
x 4 x 5 x6
2 a
+ +
4! 5! 6!
1 1 1
1
1
b e = 1 + 1 + + +
+
+
2 6 24 120 720
3 4.75
4 7.8

5 a 2.5
b e kt = 0.5
ln (2)

d 139 days
c t =
k
6 a 30, 0.4055
b N = 30e0.4055t
c 228
d 9 years

e 1000, 0.1054
f E = 1000e 0.1054t
g 590
h 7 years
i 485
1
7 x = ln 2 + 4 + k
k

Answers

2 1

k
11

6
b + ln (2k )
12
, a R \{0}
a
bR

01 2

3 2 1

1
ln
3

01 2 x

3 2 1

2x

( 1, 0.2)

loge x + a
3
9 y = 1 e(x 3)
2e 2 5
10 x =
3

670

2 0.849

14
y 15

8 y =

CHAPTER 4

2 (0, 2)
x

3 2 1 0 1 2 3

f (x) = 2

x
23

2 a y-intercept is 2, no x-intercepts, horizontal asymptote


y = 1, domain R, range (1, )


f (x) = 2x + 1
y
6

h y-intercept is 18 , no x-intercepts, horizontal asymptote



b y-intercept is 3, no x-intercepts, horizontal asymptote


y = 2, domain R, range (2, )

y
6
4
2

f (x) =

3x

+2

-6,

i y-intercept is
x-intercept is 2, horizontal asymptote
y = 8, domain R, range is (8, )


f (x) = 2x + 1 8
(2, 0)
3 2 1 0 1 2 3 x
5

c y-intercept is 2, x-intercept is 1, horizontal asymptote


y = 3, domain R, range (3, )


f (x) = 3x 3
y
4 (1, 0)
2

(0, 6)

4
2

(2, 0)
3 2 1 0 1 2 3 x
2
(0, 3) y  4
4

asymptote y = 1, domain R, range (1, )



f (x) = 10x 2 + 1
y
2 (0, 1.01)

y1
0

y
6

4 (0, 4)

1,

3 a y-intercept is
no x-intercepts, horizontal asymptote
y = 0, domain R, range (, 0)


f (x) = 2x
y
x

b y-intercept is 1, no x-intercepts, horizontal asymptote


y = 0, domain R, range (, 0)

f (x) = 10x

y
0
1
(0, 1)
2

4 x

0
2
(0, 1)
2

e y-intercept is 4, no x-intercepts, horizontal asymptote


y = 0, domain R, range R+


f (x) = 2x + 2

y  8

1
j y-intercept is 100
, no x-intercepts, horizontal

1 2 3 x
(0, 2) y  3

d y-intercept is 3, x-intercept is 2, horizontal asymptote


y = 4, domain R, range (4, )

f (x) = 2x 4

y

( 2, 1)

3 2 1 0

f (x) = 2x 3

(3, 1)

0 1

(1, 5)
(0, 3) y  2
01 2 3

3 2 1

3
2
1 (0, 1 )
8

(1, 3)
2 (0, 2) y  1

3 2 1 0 1 2 3

y = 0, domain R, range R+


3 2 1 0 1 2 3 x

f y-intercept is 10, no x-intercepts, horizontal asymptote


y = 0, domain R, range R+


f (x) = 10x + 1
y
(0, 10)

5
01 2 3 x

1
g y-intercept is 81
, no x-intercepts, horizontal asymptote

y = 0, domain R, range R+
y
1

(4, 1)

f (x) =

3x 4

2
1
(1,
10 )
(0, 1)
2 1 0 1 2

d y-intercept is 1, no x-intercepts, horizontal asymptote


y = 0, domain R, range (0, )



f (x) = 2 x
y

0.5
(0,

81 )

0 1

(1, 2)
2

2 1

(0, 1)
0 1 2

Answers 4A 4A

( 1, 1)
3 2 1

c y-intercept is 1, no x-intercepts, horizontal asymptote


y = 0, domain R, range (0, )



f (x) = 10 x

Answers

671

e y-intercept is 0, x-intercept is 0, horizontal asymptote


y = 1, domain R, range (, 1)


f (x) = 1 3x
y

(1, 2)

10

y  10

(0, 9)

g y-intercept is 3, no x-intercepts, horizontal asymptote


y = 2, domain R, range (2, )



f (x) = 2 + 10 x
y

0 1

y2
x

h y-intercept is 2, no x-intercepts, horizontal asymptote


y = 1, domain R, range (1, )



f (x) = 1 + 2 x
(0, 2)
1

(1, 12 )

0 1

2 1

y1
x

i y-intercept is 0, x-intercept is 0, horizontal asymptote


y = 2, domain is R, range (, 2)

f (x) = 2 21 x

y

y2

2
2 1

(1, 1)
0 1 2

j y-intercept is 8, x-intercept is 2, horizontal asymptote


y = 1, domain is R, range (, 1)
y

(2, 0)
0 1

y1
3

f (x) = 1 32 x

5
(0, 8)

f (x) = 2

31 x

4 x

y-intercept is 6, no x-intercept.
Horizontal asymptote is the x-axis, domain is R, range R+.

672

Answers

(0.8, 0)
2 3

4 x

(0, 7)

y-intercept is 7, x-intercept is 0.8.


Horizontal asymptote is y = 5, domain is R, range is (, 5).
7 a Dilation by factor 13 from the y-axis
b Dilation by factor

1
4

from the y-axis

c Dilation by factor 2 from the x-axis


d Dilation by factor 3 from the x-axis
e Reflection in x-axis
f Reflection in y-axis
g Translation of 1 unit up
h Translation of 3 units down
i Translation of 1 unit right
j Translation of 5 units left
8 a 4 units up
b 2 units down
c 2 units to the left
d 3 units to the right
e 3 units down
f 2 units up
g 1 unit to the right
h 2 units to the left
i 5 units up and 1 unit to the left
j 2 units up and 4 units to the right
k 3 units to the right and 4 units down
l 2 units to the left and 3 units down
9 a R, R+, y = 0
b R, (1, ), y = 1
c R, (3, ), y = 3
d R, (, 1), y = 1
e R, (2, ), y = 2
f R, (, 1), y = 1
10 f (x) = 1 2x
11 f (x) = 1 3 2x + 1
12 E
13 B
14 C
15 a 50
b 400
c
Asymptote y = 0
p
(3, 400)

200
2 1

(1, 2)
0

400

y
6 (0, 6)

5
(1, 0)
0 1 2 x

1
(1, 2
10 )

(1, 3)

y-intercept is 1 12 , no x-intercepts.
Horizontal asymptote is the x-axis, domain is R, range is
(, 0).
f (x) = 5 4 31 x
6 y
y5 
0

(0, 3)
2

0 1 2 x
(0, 112)

f y-intercept is 9, x-intercept is 1, horizontal asymptote


y = 10, domain R, range (, 10)


f (x) = 10 10x
y

y
4

2 1

2 1

f (x) = 3 21 x

y1

1 0

2 1

(0, 50)
0 1 2 3 t

Exercise 4B Logarithmic graphs to any base


1 a y-axis
b y-axis
c y-axis
d y-axis
e y-axis
f y-axis
g y-axis
h y-axis

2 a

y
2

8 10 x

(6, 1)

(10, 2)
8 10 x

(2, 5)

y
2

( 23 , 1)
2 x

0 ( 1 , 0) 1
3

2 3

4 5 x

Asymptote x = 0
f (x) = log10 (4x)
R+, R

y
(5, 3)

2
( 12 , 0)

2 3

Asymptote x = 0
f (x) = 3 log10 (2x)
R+, R
3


a
d
g
j

4 a

b
(0, 1)

x3
(4, 0)
(5, 1)
2

f (x) = log2 (x 3)
Domain (3, ),
range R

0
1

1
0

f (x) = log2 (x 4)
Domain (4, ),
range R

Asymptote x = 0
R, R, in y-axis
y
2
(0.1, 0)

(1, 1)
6

(10, 0)
x
8 10

Asymptote x = 0
R, R, in y-axis

y
2

5 4 3 2 1

y
1
(
, 0)
100

Asymptote x = 0
R+, R, no reflection

2
x

Asymptote x = 0
R+, R, in x-axis

(6, 1)
(5, 0)
2

Asymptote x = 0
R, R, in y-axis

x4

y
2
( 1, 0)
5 4 3 2 1

1
x

1
( 1, 0)
0 x
5 4 3 2 1
1

f (x) = log2 (x + 1)
Domain (1, ),
range R
d

0 x

( 100, 0)

( 10, 1)1

100 80 60 40 20 0

Asymptote x = 0
R, R in x- and y-axis

Answers

Answers 4B 4B

f (x) = log2 (x + 2)
Domain (2, ),
range R

Asymptote x = 0
R+, R, in x-axis

(1, 1)
x

1 2 3 4
(3, 0)

Asymptote x = 0
R+, R, in x-axis

1 0 1
1

2 1 0 1 2 3 4 x

1
0

x  1
y
1

y
x  1
1 0
1

(1, 0)
0 0.5 1 1.5 2 2.5 3 3.5x

c 3 right
f 3 right, 2 up
i 3 left, 2 down

( 78 , 0)

2
(1, 0)
01 2 3 4

0 1 2 3 4 x

f (x) = log2 (x + 1) 2
Domain (1, ),
range R

Asymptote x = 0
f (x) = 2 log2 (3x)
R+, R

x  2 y

2 (0, 2)

2)

( 13 , 0)

2 left
b 1 left
4 right
e 5 right, 1 up
1 left, 2 up
h 1 left, 3 up
1 left, 2 down

( 1, 0)

5 a
( 23 ,

f (x) = 3 + log2 (x + 1)
Domain (1, ),
range R
j

x  3 y

(0, 3)

f (x) = log2 (x + 3) 2
Domain (3, ),
range R

y
2

4 5 x

(1, 0)
x
0
1 2 3 4
3 2 1
2

Asymptote x = 0
f (x) = log2 (3x)
R+, R

2
( 14 , 0)

y
x  1
4

f (x) = 2 + log2 (x + 1)
Domain (1, ),
range R

(2 12 , 1)

2 (0, 2)
1

Asymptote x = 0
f (x) = 5 log2 (x)
R+, R

(3.25, 0)
2

f (x) = log2 (x 3) + 2
Domain (3, ),
range R

(1, 3)

Asymptote x = 0
f (x) = 2 log10 (x)
R+, R

(4, 2)

y
x  13

x3

(1, 0)

(5.5, 0)
6 8 x

y
2

f (x) = 1 + log2 (x 5)
Domain (5, ),
range R
g

y
6
4
2

x5

0
1

Asymptote x = 0
f (x) = log10 (x)
R+, R

(1, 0)
2 4 6

(1, 0)
2 4 6

Asymptote x = 0
f (x) = log2 (x)
R+, R

(10, 1)

(2, 1)
(1, 0)
0 1 2 3 4 5 x

y
1

673

y
2
1

h 2, dilation by factor 4 from the x-axis, translation


2 units to the right and 1 unit down
i 2, dilation by factor 2 from the x-axis, and by factor
13 from the y-axis, translation 1 unit up
j 2, dilation by factor 4 from the x-axis and by factor 12
from the y-axis, translation of 3 units up and reflection
in the x-axis
y
9
x2

x  2
(1.5, 1)

1 0 1
2
3

2 3
(1, 0)

Domain ( 12, ), range R, dilation by factor 12 from the


y-axis, horizontal translation of 12 unit to the right
7 a

y
(1, 3)

13

(2 2, 0)

y
2 x1
(0, 0)

y
2
0 1 2
(2, 1)

x  1
y
2
(3.6, 0)
0 1

674

x2

0 1 2

(0.24, 0)

3
4
(3, 1)

0 2

2 (0, 2)

Asymptote x = 0
11 B

x-axis, translation of 1 unit up


i Dilation by factor 3 from the x-axis, translation of
2 units to the left and 1 unit down
2 a
b
y f(x)  ex
y f(x)  ex
4

4 6
(5, 1)

Asymptote x = 0
Asymptote x = 0
R+, R
R+, R
a 2, dilation by factor 2 from the x-axis, translation of
3 units up
b 10, dilation by factor 3 from the x-axis, translation of
1 unit up
c 10, reflection in the y-axis, translation of 1 unit to the
right
d 10, reflection in the y-axis, translation of 3 units to the
right
e 2, reflection in the x-axis, dilation by factor 2 parallel
to the y-axis, translation of 1 unit up
f 10, reflection in the x-axis, dilation by factor 5 from
the x-axis, translation 2 units up
g 10, dilation by factor 3 from the x-axis, translation
1 unit to the left, translation 2 units down

Answers

h Dilation by factor 13 from the y-axis, reflection in the

(2.8, 0)

8 10

g Dilation by factor 12 from the y-axis, reflection in the


x-axis, translation of 3 units up

y
4
2

(2, ), R

( 2 , 3)
2

(3.2, 0)

(0, 2)

b Dilation by factor 12 from the y-axis


c Dilation by factor 4 from the x-axis and by factor
2 from the y-axis
d Dilation by factor 2 from the x-axis and by factor
3 from the y-axis
e Translation of 2 units to the right and 1 unit up
f Translation of 5 units to the left and 2 units up

Asymptote x = 0
R+, R

(1, ), R

(1, 2)
2
(105 , 0)

y
5

Exercise 4C Graphs of exponential functions


with base e
1 a Dilation by factor 13 from the y-axis

0 1

Asymptote x = 0
R+, R

y
5

x3

(, 3), R

(10, 1.1)
(3.6, 0)
0 2 4

4
x

(1.2, 0)
0
1

( 2, 0)

y
2

(, 2), R
10 y

(2, 0)
0 0.5 1 1.5 2 2.5 3 x

(, 1), R

Asymptote x = 0
R+, R

1 12 3

(10 3 , 0)

(0, log103)

3 2 1 0 1 2

(1, 1)

Asymptote x = 0
R+, R

1
0

4 (0, 4)

y
2

f(x)  ex 1

f(x)  ex 3
2
(0, 1) (0, e 3)
(3, 1)
x
3 2 1 0 1 2 3

2
(0, 1)

(1, 1)
(0, 1e )
x
3 2 1 0 1 2 3

Asymptote y = 0
Translated 1 unit to
the right, R, R+
c

y f(x)  2ex
4
f(x)  ex

(0, 2)
(0, 1)
x
3 2 1 0 1 2 3

Asymptote y = 0
Dilated by factor 2
from the x-axis,
R, R+

Asymptote y = 0
Translated 3 units
to the right, R, R+
y f(x)  3ex
4
f(x)  ex
(0, 3)
2
(0, 1)
x
3 2 1 0 1 2 3

Asymptote y = 0
Dilated by factor 3
from the x-axis,
R, R+

f(x)  1 ex
f(x)  ex

4
(0, 2)

y
f(x) = e2 x
8 (0, e2) f(x)  ex

(0, 1)
0 1

2 1

y
4

f(x) 

(0, e)
0 1

2 1

y3

y
(0.5, e1.5)

y
4

Asymptote y = 0
Dilation by factor
from the y-axis,
R, R+

1 0.5

1
3

y
4 (0, 4)

Asymptote y = 0
Dilation by factor
from the y-axis,
R, R+

y
6
4

2 (0, 2)

(0, 3.37)

y3

2
x
1.5 1 0.5 0 0.5 1 1.5

x
1.5 1 0.5 0 0.5 1 1.5

y
4
2
1

(0, 2 )
x
3 2 1 0 1 2 3

Asymptote y = 0
Dilation by factor 2
from the x-axis,
R, R+

y
2

01 2 3

Translation of 1 unit
to the right and
3 units up, R, (3, )

y
4
2

y=2

Translation of 2 units
up, R, (2, )
y

8
6
4
2

(0, 8.39)

y
8
6
4
2
3 2 1

y5
(0, 5.14)
01 2 3 4 x

Translation of 2 units
to the right and 5 units
up, R, (5, )

y
(0, 8.39)
6

1
1
(0, 4 )
1

Answers 4C 4C

Asymptote y = 0
Dilation by factor 4
from the x-axis,
R, R+

3 2 1

Translation of 2 units
to the left and 1 unit
up, R, (1, )

Translation of 1 unit to
the left and 2 units up,
R, (2, )
g

y=1
x
3 2 1 0 1 2 3

x
3 2 1 0 1 2 3

1
2

x
3 2 1 0 1 2 3

y2

y
4

(0, 4.72)

(0.5, e)
(0, 1)
0 0.5

(3, 1)

(0, 3)

(0, 0.05)

y=1

Translation of 1 unit
up, R, (1, )
4

1 0.5

y
2

Asymptote y = 0
Translation of 3 units
to the right, R, R+

x
3 2 1 0 1 2 3

2
(0, 1)
0 0.5 1

0 1 x

2 (0, 2)

Translated 3 units up,


reflected in the x-axis,
R, (, 3)
3 a

Asymptote y = 0
Translation of 3 units
to the left, R, R+

0 1 2 x
f(x)  3 ex

2 1

Asymptote y = 0
Dilation by factor 3
from the x-axis
and by factor 14
from the y-axis,
R, R+

10
3 2 1

1.5 1 0.5

y
20 (0, 20.09)

ex

(0, 2)

4 a

Asymptote y = 0
Reflected in the y-axis,
translated 1 unit to the
right, R, R+

Asymptote y = 0
Reflected in the y-axis,
translated 2 units to the
right, R, R+

(0, 1)

f(x)  ex

Asymptote y = 0
Dilation by factor 2
from the x-axis
and by factor 13
from the y-axis,
R, R+

6
4

y  2

y
f(x)  e1 x 8

(0, 1)

y
4
2

1 2 3

Translated 2 units
down, R, (2, )

y
2 (0, 2)

(0, 3)

(0, 1) 2

6
2

3 2 1 0

Translated up 1 unit,
R, (1, )
g

f(x)  ex
f(x)  ex 2

2
(0, 1)

y1
(0, 1)
01 2 3 x

3 2 1

y
4

4
0

Asymptote y = 0
Dilation by factor 12
from the x-axis
And by factor 14 from

Asymptote y = 0
Dilation by factor 14
from the x-axis
and by factor 12 from

the y-axis, R, R+

the y-axis, R, R+

2
( 2, 0)
3 2 1

y  1

Translation of 2 units
to the left and 1 unit
down, R, (1, )

Answers

675

5 a

y
4
( 1, e) 2

(0, 1)
0 1

2 1

Reflection in the
x-axis, translation of
1 unit up, R, (, 1)

( 1, 1 e) 4
(0, 2)

0 1

Reflection in the
y-axis, translation of
1 unit up, R, (1, )

y
2

1 0

y5

Reflection in the
y-axis, reflection in the
x-axis, translation of
3 units up, R, (, 3)
y

2 (0, 2)

Asymptote y = 0
Reflection in the
y-axis, reflection in
the x-axis, dilation
by factor 2 from the
x-axis, R, R
y

(12 , 0)
0 1 2

2 1
1

x
y  1

(0, 0.95)

R, (1, )
y
7
(2, 0)
0 1 2

1
1

(0, 0.86)

[0, ), [0.86, )

676

Answers

(0, 1)

x
y  1

(0.69, 0)
x
1

y
2

(0, e)

1
(0, e 2)
0 1

y
2 (0, 2)
1

y1

( ln5, 0)
1

Reflection in the
y-axis, reflection in the
x-axis, translation of
5 units up, R, (, 5)

Asymptote y = 0

1 (0, 1)
x

y2

y
2

(0, 2)
( ln3, 0)

Asymptote y = 0

y1

Asymptote y = 0

4 (0, 4)

x
3 2 1 0 1 2 3

y3

y
4

y1

Translation of 1 unit
up, R, (1, )

y
4

(0, 2 )

Asymptote y = 0
e

y  2

x
3 2 1 0 1 2 3

y2

(0, 2)
0

y
4

2 1
(0, 1)

2 (0, 2)

Reflection in the
y-axis, translation of
2 units up, R, (2, )

y1

(0.69, 0)
0 1 2 x

y1

y
4

4
(0, 3)
( 1, 2 e)
2

1
(1, 1 e)

0 1

2 1

(1, e)

2 (0, 2)

y1

8 a

Asymptote y = 0
Reflection in the
x-axis, R, R

y
2

Asymptote y = 0
Reflection in the
y-axis, R, R+

1
(0, 1)
2

Asymptote y = 0
9 D
10 E
11 a 1, 3
b y = e x
Exercise 4D Logarithmic graphs to base e
1 a Dilation by factor 5 from the x-axis and by factor 1
2
from the y-axis
b Dilation by factor 2 from the x-axis and by factor 14
from the y-axis
c Translation of 1 unit to the left and 3 units down
d Translation of 2 units to the right and 1 unit up
e Reflection in the x-axis and in the y-axis
f Reflection in the x-axis and dilation by factor 1 from
2
the y-axis
g Translation of 2 units to the right and 1 unit up,
reflection in the x-axis
h Translation of 3 units to the left and 2 units up,
reflection in the x-axis
i Translation of 4 units to the left and 1 unit down,
dilation by factor 3 from the x-axis
j Translation of 4 units to the right and 1 unit up,
reflection in the x-axis
k Translation of 1 unit to the right and 3 units up,
dilation by factor 2 from the x-axis, reflection in the
y-axis
l Translation of 2 units to the right and 1 unit down,
dilation by factor 3 from the x-axis, reflection in the
y-axis

y
1

2 a

y
1

Asymptote x = 0
R+, R
y

(3, 0)

0.5 0.5 1 1.5 2 2.5 3


1

Asymptote x = 0
R+, R
y

x1

Asymptote x = 0
R+, R

x  2

y
2

( 1, 0)
2

(0, loge2)
0
2

y
4

x  3 y

0
2

x
3 2 1 0 1 2 3 4 5
1 (0, 0.9)

y
x  1
1
2

( 1, 0)
x
0
2 1.5 1 0.5 0.5
2

Asymptote x = 0
(, 0), R

0.5 1 1.5 2 2.5 3

y
5
0 0.5

1.5

y
2

2
(1, 0)
0 1 2

Asymptote x = 0
R+, R

(7.39, 0)
x

1 2 3 4 5 6 7
(1, 2)

Asymptote x = 0
R+, R
y

(3, 0)
0 0.5 1 1.5 2 2.5 3 x
2
4

(0, 2)

y
0

x3

b f(x) = ln (x) 2
x
d f(x) = ln
3
f f(x) = ln (x)
x
h f(x) = ln + 1
4

Asymptote x = 0
R+, R

Asymptote x = 0
R+, R
y
2

y
2

(2, 0)
x
3 2 1 0 1 2 3
(0, 1.10)
2

x1

(1, 0)
(6.4, 0)
01 2 3 4 5 6 7 x

(, 2), R

(, 3), R

(2,0)

4 a

(1, ), R
6

(1, 0)
0 0.5 1 1.5 2 2.5 3 x
2

Asymptote x = 0
R+, R

y
2
(1, 0)
x
0
2.5 2 1.5 1 0.5 0.5
2
4

Answers 4D 4d

(4.4, 0)

y
2
0

(1.4, 0)

x2

2
8

y
2

y
4 x1

2
(3.1, 0)

1), R

e f(x) = ln (x)

6 a

(0, loge3)

i f(x) = 2 ln (x) 3

2
2

x3

(,

x2

(0, 0.69) (1, 0)


3 2 1 0 1 2

0 1 2 3 x

x  3 y
2

y
2

x1

g f(x) = ln (x + 3)

( 2, 0)

Asymptote x = 0
R, R

c f(x) = 5 ln (x)

0 12 3 4 5 6 7x

(, 2), R
5 a f(x) = ln (x 1)

x2

y
5
1
( 4 , 0)
0
1
5

(1, 0)

(3, 0)

Asymptote x = 0
R, R

2 1 0
(0, 0.69)
2

Asymptote x = 0

(2, 0)
1

3 2 1

Asymptote x = 0
R, R

y
5

0
0.5 0.5 1 1.5 2 2.5 3

Asymptote x = 0

(e 2, 0)

( 2, 0)

(e 1, 0)
x
0
0.5 0.5 1 1.5 2 2.5 3

(0.5, 0)
x
0
0.5 0.5 1 1.5 2 2.5 3

3 a

y
1

Asymptote x = 0
R+, R

(1, 0)
x
1.5 1 0.5 00.5 1 1.5 2
1

Asymptote x = 0
R+, R

2
1
( 2 , 0)

(1, 0)
0 1 2

Asymptote x = 0
R+, R

Asymptote x = 0
R+, R

y
2

(1, 0)
x
1.5 1 0.5 00.5 1 1.5 2
1

( 3 , 0)
x
1.5 1 0.5 00.5 1 1.5 2
1

(0.5, 0)
x
1.5 1 0.5 00.5 1 1.5 2
1

y
1

Asymptote x = 0
R, R

Answers

677

x  3

2
x

(3, ), R
y
2

b
(1.47, 0)

( 2, 0)
2 1 0
(0, 1.10) 2

y
2
0

0.5 1 1.5 2 2.5 3

R+, R

(0.22, 0)

x
0
2 0.5 1 1.5 2 2.5 3

y1  e x 8
6
4
( 2, 5.4) y  x 2
2

x  2 y

( 2, 2)

0
2

2 4 6
( 1.45, 0)

(1, 0)
0

1
2

1
2,

( 2, 4.14)
( 2, 4)
( 2, 0.14)

(0, 2.08)

1, f(x) =

1
2

678

Asymptote y = 0

Answers

y2
0

(2, 4)
x

Asymptote y = 0
f ( 2, 11.4) y
10

(3, 9.05)
f(x) (3, 9)

y2  x2

( 2, 7.4)
( 2, 4)

y1  e x (3, e 3)

2 1

0 1 2 3 x

Asymptote y = 0
4 a
y
2

(1, 1)
0

0.5

1.5
(1, 0)

h(x)
g(x)
f(x)
x

Asymptote x = 0
Domain R+, range R
h(x)
b
y
2

(1, 1)

g(x)

x
0.5 0 0.5 1 1.5
2
f(x)  3 loge(x)

Asymptote x = 0
Domain R+, range R
c
y
5

f(x)
( 1, 5.4)
(1, 5.4)
4 (0, 4)
y1  2ex
y2  2e x 2
(1, 0.74)
( 1, 0.74)
(0, 2)
0 0.5 1 x
1 0.5

(2, 11.4)
f(x)
(2, e2)
y1

2x + 1

6 E
7 2, 3
8 4, 1
9 2.9, 2.0, 2.0, y = 2.9 loge (x + 2.0) + 2.0
10 2.0959, 3
11 1.5, 2, 3
Exercise 4F Addition of ordinates
1 a R\{0}
b [0, )
c [0, )
d [2, )
e R
f R\{3}
g R\{1}
h (, 1]
i R\{0}
j [1, 3]
2 C
( 1, 6.2) y
(1, 6.2)
3 a

y
10

1 2 0

(2, 2.14)
(2, 2)
(2, 0.14)
x
2

y
(2, 9.4)
f(x) (2, 7.4)
8
y1  ex
6
4 (0, 1) y  x
2
( 2, 0.14) 2
(2, 2)
( 2, 1.9)
x
1 2 0 1
2
( 2, 2)

9 D
10 B
11 C
12 a f: (2, ), g(x) = a b loge (x 2)
b Domain is (2, ) and range is R.
c h(x) = 4a 4b loge (x 2) or 4a + 4b loge (x + 2)
Exercise 4E Finding equations for graphs of
exponential and logarithmic functions
1 0.58, 2.42, f(x) = 0.58ex + 2.42, y = 2.42
2 0.84, 0.33
3 1.60, 0.92
32
4 A = 15
, B = 19
15

f(x)

Asymptote y = 0
d

2 (0, 2.58)
2

f(x) (1, 9.3)


8 (0, 6) (1, 8.2)
6
y1  3ex
y2  3e x 4
(0, 3)
2
(1, 1.1)
( 1, 1.1)
x
1 0.5 0 0.5 1

Asymptote y = 0
c
( 2, 7.4) y

Asymptote x = 0
R+, R
7 Dilation by factor 2 from the x-axis, translation of 2 units
to the left and 1 unit down

y
( 1, 9.3)
( 1, 8.2)

(1, 2)

0 0.5 1 1.5 x
1
5 f(x)  log (x)
e

Asymptote x = 0
Domain R+, range R

h(x)
g(x)  2x

y
2

0 0.5 1 1.5
(1, 0)

1
2

h(x)
g(x)  21 x
f(x)  loge(x)
x

(1, 2 )

y
y = x2 5x 6

Asymptote x = 0
Domain R+, range R
e
y

0.5 0 0.5 1 1.5


2
(1, 0)

f(x)  loge(x)

0 0.5 1 1.5

y = x3 x2 1

3 2

h(x)

2
g(x)
y

10 a

f(x)

y x 2 x

2
y 2 x

y x

(2, 3.76)

1
y

2
0

y x 3
x

x
y  2x x

Domain [2, 2], range [1, 3.76]


6 E
y
7 a
h(x)

1
4

y x

f(x)

y  x2

y  x 3 x2

g(x)
x

y  2x

(0, 1)
2

f(x)

Asymptote x = 0
Domain R+, range (0.6, )

y
( 2, 3.76) 4

g(x)

(1, 1)

0.5
2

h(x)

Asymptote x = 0
Domain R+, range [1, )
f
y
2

f(x)

6 5 4 3 2 1 0 1 2
g(x)

(1, 1)
x
g(x)

8
7
6
5
4
3
2
1

y 5 x

2 5 y x 5 5 x
y x 5

11 E
x
f(x)
g(x)
h(x)

12
y

d
x

h(x)
y

y  loge(x)

g(x)

y x

h(x)

Domain x > 0

Answers 4e 4f

f(x)

y  loge(x) x

f(x)
g(x)

y
4
3

13
f

h(x)

f(x)
g(x)
x

y
f(x)

2
1
x
f(x)
g(x)
h(x)

1
1
2
3
4

g(x)
1

h(x)

Answers

679

14 a

y
0.3
0.2
0.1
4

Domain R, range [0, )


Asymptote x = 0
b
y

0.6

0.2

1 2 3 4 5 6 7 8

6 (0, 3loge 2 3)
5
4
3
2
1
1 0 1
2
2
1
(2 e , 0)
x2

Domain R, range [0, )


Asymptote x = 0

1
Domain (0, ), range ,
e
x-intercept (1, 0)

Exercise 4G Exponential and logarithmic


functions with absolute values
1 a
y

0.4

1 0

( 1 e 12 , 0)

3 2

2
1

( 1 e 12 , 0)

1 1 0
(0, 2)
3
x = 1

(3.1, 0)

(2.9, 0)
0

0
x

( 1 , 0)
3
1

Domain R\{3}, range R


2 a y

1
Domain (0, ), range ,
2e
x-intercept (1, 0)
y

x3

Domain R\{1}, range R


c
y (0, log (3) 1)

Domain R, range [1, )


y-intercept (0, 1)

10

15 a

x
3
4
1
(2 e , 0)

Domain R\{2}, range R


y
b

2 (3, 0) 4
x2

Domain (2, ), Range [0, )


b
y
4
3

680

Domain (0, ), range ,

18e

x-intercept 1 , 0
3

Answers

(0, 2)
1
1

1
(1 e 12 , 0)
x=1

Domain (, 1), range [0, )

y
5
4
3
(0, 2)
1

3
2
1

( 2, 1)
3 2
x  3

(0, log10 (3) 1)

2 1 1 0 1

y
10
9
8
7
(0, e1 4) 65
4
3
2
1

(15, 92)

80

(10, 75)
60
(0, 50) (2, 54)
40

(1, 5)

20
0

Domain R, range [2, )


Exercise 4H Exponential and logarithmic
modelling using graphs
1 a 50 koalas
b 54 koalas, 75 koalas
c
d 92 koalas
K

Domain (3, ), range [1, )

3 a

y3

0 4

12 16

2 a 150 wallabies
b 162 wallabies, 220 wallabies
c
W

Domain R, range [5, )


y
b
(0, e4 4)

500
400
300
200
x

(4, 2)

(4 ln (3), 0)

(4 ln (3), 0)

Domain R, range [2, )


c
y
( 1, 1)

( ln (2) 1, 0)
3

2
1
1
(0, 2 e1)
2
3
4

(ln (2) 1, 0)


0

800
(0, 500)

(1, 167)

y2

1 1 0 1
2
2 (ln (6), 0)
3

Domain R, range

(,

4]

0 2 4 6 8 10121416 t

d
e
f
a
c
e

7 days
No, because the graph approaches the line E = 0.
4 days

T = 80e kt + 18
b 0.277

d 9 minutes
T = 80e 0.277t + 18
f 19C
T
80
60

(5, 38)
(9, 25)

40
20
0

T  18
4

12 16

Answers 4g 4H

4
(0, 3)
2
1

(5, 81)
(10, 33)

Domain R, range [0, )


b

8 t

b 167, 81

100
1
2
(ln (3), 0)

200

(1, 530.9)

400

e $810
4 a 0.18
c
E

(0, 2)

(5, 675)

600

0 2

y3

e 9 years
b $530.90

200

d 475 wallabies
a $500
c $174.95
d
A

Domain R, range (, 1]

4 a

(5, 220)
(1, 162)
(0, 150)
0 4
9 15

Answers

681

5 a f(x) = 2 log10 (x)



y

g The egg will never reach 18C because the line


T = 18 is an asymptote.
6 a A = 1242.67, B = 10 000
b 7139
c 279 weeks
d
P
10 000
6300
5000


7
8

(1, 0)
0
2

(1, 10 000)
(5, 8000)
(10, 7139)

e 6300
C
a C = 20n
c R 2000
d
y

2 4

6 8 10

2
(1, 0)
( 1, 0)
3 2 1 0 1 2 3
20

6 120 e
7 a

g(x)  2e x 1 4

R  2000
(55, 1992)

(0, 2e 4)

(20, 1729)

( 1 loge 2, 0)

P(n)
(20, 1329)

(55, 892)
y  4

(20, 400)

e
f
g
h
i

20

55

Teacher to check.

P = 2000 (1 e 0.1n) 20n


23 jackets, profit $1339
100
No. Not enough jackets are made before profits begin
to decrease.

(1, 0)
x
2
6 4 2 0
2 (0, 2)
y  4
4

y
4

Domain = (4, ), range = R


y

(5, 3)

2
0

Domain R, range (4, )


3 f(x) = 3ex 4
(f g) (x)
g(x)
f(x)
x

0 (0.33, 0)

x + 4

b g 1(x) = loge
1
2

y = h(x)

Chapter review
Short answer
1
y

4 a

(55, 1100)

b Teacher to check

2000

12 16 18 20 t

(10, 2)

1
( 2

, 0)
2

Domain R+, range R

Domain = R+, range = R


9 Dilation by a factor of 2 from the x-axis, reflection
through the y-axis, a translation of 3 units right and a
translation of 5 units up
10 a On 1 January 2006, there are 50 tigers and
400 elephants.
b On 1 April 2007, there are 3200 tigers and
50 elephants.
c The numbers of tigers and elephants is equal on
1 June 2006 (there are 100 of each).
d The elephant population is at risk from 1 September
2007, as the model predicts their population will
reach 25 on this date.
11
y
( 2, 0)

(f g) (x)

1 0
2

g(x)
f(x)

2 3 4 5

(0, loge (3))

4
x  3
0

682

Answers

Domain (3, ), range (, 0]

12 a R\{3}

b x-intercepts: x = 3 + e
1
2

( 3 + e

y-intercept:

(0, 2 log e (3) + 1)

, 0),

( 3 e

1
2

, x = 3 e

1
2

, 0)

y = 2 log e (3) + 1

6 a a = 1 because it is where the horizontal asymptote cuts


the y-axis. (0, 0) satisfies the equation, so 0 = 1 + be0
b = 1.

b Range of h: y [0, 1 e 2]
1
2

c h : [0, 1 e ] R, where h 1(x) = loge (1 x)


y
7 a

y
x  3

4
3
2
1

Multiple choice
1 D 2
4 C 5
7 C 8
10 D
11
13 C
14
16 A
17
19 B
20
22 C
23
Extended response
1
y

1
2,

2 x

0)

B 3 A
C 6 D
B 9 E
D
12 D
A
15 B
B
18 B
A
21 D
B

1 2 3 4 5 6 x

2 1 10
2
3

(0, 2loge (3) 1)

6 5 4 3 2 1 1 0 1
1
2
( 3 e 2, 0)
3
4
( 3 e

(0, loge (5) 1)

3
2
1


8




9

10
11



f 1 ( x) =

(5 e , 0)

x 5

5 ex 1

b
a $5000
b $5256.35
c $6420.15
d $6410.20
e 13.95 years
f The first investment, because the interest is
compounded continuously.
g $9.95
h About $20
a 1
b 0.52, 516 metres
c 16 metres
d 3.6 kilometres

a P = 101.3e 0.125x
b 76.7 kPa
a b = 5, c = 6
b x = loge (2) and loge (3)
c (0.92, 0.25)
d x = (loge (5), 6)
e g: R R, g(x) = e2x + 5ex 6
2x

f h: R R, h(x) = e 5e x + 6
g
(log (2), 0) y
e

(loge(3), 0)

0
2

3 2 1

(0, 2)

(0, 1.74)
y1
0 1 2 3 x

y  6

Domain R, range (1, )


2
y
x2

Domain
3 y

(,

(1, 0.3)
0

2), range R

Inverse functions

f(x)
y  log 2x

Exercise 5A Relations and their inverses


y
y
b
1 a

y  log x
x
3 4

3 2 1 10
2

5
4
3
2
1
0 123

Domain R+ range R
4 a 10 000
5
y
(0, 4)
3
2
1

CHAPTER 5

b 15 000
c

y 3

x
8

{0, 1, 3}, {1, 2, 5}


y

y  2x 5
0

1 2

(1, 1)
0 1

{8, 5, 2, 1}, {1, 1, 2, 7}

yx
1

2
1
2 0 1
1

5
2

Answers 5a 5A

1 2
(1, 0)

k(x) = e 2x + 5e x 6, domain R, range (, 0.25]

h y = e 2x + 4 5 x + 2 + 7 or y = e2(2 x) 5e(2 x) + 7,
domain R, range [0.75, )

4
2
( 0.7, 0) (0, 0.3)
x
3 2 1 0 1 2 3

(loge (5), 0)

Domain R, range (, 4]

R, R

R, R

Answers

683

f
2x 4y  8

2
0

5
2

y  x2 4x
x

d y = 12 x +

c y=x

5
2

(1, 1)
x

( 2, 4)

R, R

yx 1

R, [1, )

y  (x 1)2

1 0

R, [0, )

2 x

y2

h y = 1 x

g y = x +1

y
1

y  3
x

0
1

[1, ), R
i

x2 + y2 = 4

y=

3
x
y

2
(3, 1)

{4}, R

y 2x3

( 3, 1)

(1, 2)
x

0 1

[2,

2], [2, 2]

R\{0}, R\{0}

k x=2

y = 4

R, R
2 a {(1, 0), (2, 1), (2, 3), (5, 3)}

0
x

3
2
1
0 12345

R, {4}

{2}, R
x

{1, 2, 5}, {0, 1, 3}


b {(7, 8), (2, 5), (1, 2), (1, 1)}

m y=

1
1 0 1 2 3 4 5 6 7 x
2

{1, 1, 2, 7}, {8, 5, 2, 1}

(2, 1)

1
0

5
8

x
2
y

Answers

2
2

684

[0, ), R

x  4

R\{0}, R\{0}

R, {2}

( 4, 2)

[4, ), R

2
0

(1, 3)

1 0

y= x+42
y

( 1, 3) 3

[2, 2], [2, 2]

R, R

+4
y

x2 y2  4

2x

y
3

R, R

R, R

R, [4, )

R, R
3 A
4 C

5 a

x  2

yx

3
2
2
y

y3

y  2

yx

y
(4, 4)

(3, 1)
4

4 ( 2, 0) 0
2
( 4, 2)

yx

f
(1, 1)
6

(0, 3)

yx
y

h
2

(1, 1)
x

yx

1
2

yx

yx

( 1, 1)

yx

(1, 4)

yx

(1, 4)

( 4, 16)

yx

(1, 1 )

1
2

0 1

yx

2
4

2
4

2
0 2

4 2 0
2
4

R, R+

R, R

yx
1

y
2

2
3

x
0 1
2

R+, R

Answers 5B 5B

3
x

R, [16, )

R\{0}, R\{0}
yx

2
0 2

(3, 3)

1 0
3

R, [0, )

3
0

R, [9, )

( 3, 3)

3 x

9
x

yx

5
2

R, R

(1, 1)
0

(4, 0)

R, R

y
5

2 3

{4, 2, 0, 2, 3}, {2, 0, 1, 4, 6}


b
c
y

(2, 4)

(0, 1)

0 2

yx

yx

(3, 6)

2
x

(1, 3)

Exercise 5B Functions and their inverses


1 a Function
b Function
c Function
d Function
e Not a function
f Function
g Function
h Function
2 a
y

x3

[2, 2], [0, 2]

Answers

685

3 a

3
2
2

0 (1, 0) 4
(0, 2)

y

y
(0, 4)

5
2

y  25 21 x

4 a i

2
1
0

ii R, R+
iii R+, R
y
d i

yx

x
y o x 2
[0, c), R

ii R, [4, )
iii [4, ), R
e i
y

ii R, R+
iii R+, R
i

4
y  x

y1
x

ii [3, 3], [0, 3]


iii [0, 3], [3, 3]
y
g i

(4, 1)

y
yx

yx

y= x 2
[0, ), R

0
1

0
1

ii R, R
iii R, R
y
c i

y= x+9
[9, ), R

yx

yx
y o x 9
0 x [ 9, c), R

x1

ii (1, ), R
iii R, (1, )
h i

yx
3

( 4, 1)

R\{0}, R\{0}

ii R, R
iii R, R
i

y = o x 16 4
[ 16, c), R

y
5

yx

8
5

y = x + 16 4
[16, ), R

Answers

0
5

5 x

ii
iii
ii
iii

yx

1
0 1

( 16, 4)

686

y o 4 x2
[0, 2], [ 2, 2]

y = 4 x2
[0, 2], [2, 2]
b i
y

R, R+

y
2

4
( 4, 1)

R+, R

y  12 ex

(0, 1 )

R, R

2x

R, R

(3, 0) x

y  loge ( 2 )

j
y

4x 3y  12

R, R

{2, 0, 1, 4, 6}, {4, 2, 0, 2, 3}


b

0 1

( 2, 4)
4

{( 2, 4), (0, 2),


(1, 0), (4, 2), (6, 3)}

( 1 , 1)
1

(4, 2)

(6, 3)

R, [1, )
[1, ), R
[5, 5], [5, 0]
[5, 0], [5, 5]

(1, 4)

ii [2, 6], [2, 0]


iii [2, 0], [2, 6]

y
6

yx

2
2

6 x

iii
iv
v
c i
d i
e i
f i
ii

(0, 0)
R, R
R, R

f 1 does not exist

f 1 does not exist

f 1 does not exist

one-to-one, f 1 exists
y

y  (x 1)3

k i

ii
), R
iii R, (3, )

yx

x  3

(0, 1)
( 1, 0)

2
3 2

yx

(3,

y

x 1

(1, 0)
(0, 1)

2
3

(3, 4)

4
3
2
3 2

ii [2, 3), [0, 4)


iii [0, 4), [2, 3)

yx
(4, 3)

1 2 3 4 5

iii
iv
v
g i
ii

(2.3, 2.3)
R, R
R, R

one-to-one, f 1 exists
y

yx

5
6
7
8
9

D
A
E
B

a f 1 : [3, ) R, f 1 (x) =

b f

: (2, ) R, f

( x + 3) + 2

( )

= loge x 2 + 1
3

Exercise 5C Inverse functions

1 a i one-to-one, f 1 exists
y
ii
y  4x 1
yx
1

(0, 1)
1
4

( , 0)

(1, 0)

iii
iv
v
h i
i i
j i
ii

None
R\{0}, R\{0}
R\{0}, R\{0}

f 1 does not exist

f 1 does not exist

one-to-one, f 1 exists
x  2 y

y  e4x 2
yx

1
4

y
x
4
(0, 0.17)

1
4

(0, )

( 1, 0)
x

(0.17, 0)
(0, 1)
1, 1
3 3)

iii (

(1, 6)

(0.2, 0.2)
R, (2, )
(2, ), R

one-to-one, f 1 exists
y

yx

x1

Answers 5c 5c

iii
iv
v
k i
ii

iv R, R
v R, R

b i one-to-one, f 1 exists
y  6x
y
ii

y  2

y  e0.5x 1
yx

(0, 2)
(6, 1)
(0, 0)
y

y1
x

(2, 0)

1

x
6

y  2 loge (x 1)

Answers

687

iii None
iv (1, ), R
v R, (1, )
2 i a, e, g, i, j, k, l, m, n, o

ii a f 1(x) = x
4

m f 1(x) =

x
e2

n f (x) =

o f 1(x) = 12 (e3 x 3)

Function
Domain
a
b
c
d
e
f
g
h

R
[1, )
[3, 3]
R
R
R
[5, 5]
R+

f(x)
(0, 3)

Range

Domain

R
R
[0, 3]
R+
[10, )
(0, )
[0, 8]
R

R
R
[0, 3]
R+
[10, )
(0, )
[0, 8]
R
b
b
b
b
d
f
h

b f

c f

d f

e f

: (1, ) R, f

(x) =

f f

: R\{0} R, f

(x) =

g f

: [5, 0] R, f

(x) = 25 x 2

(x) =

0
(0, 1)

: [2, ) R, f

(x) = x + 2 3

: (3, ) R, f

(x) =

1
x+3
1 +4
x 1
1
+2
x

(3, 0)

yx

f(x)

0
f 1(x)

yx
y

3
f 1(x)
0

yx
y

(0, 1) y  x
(1, 0)
( 1, 0)

x +1
3

: [1, ) R, f

(x) =

f 1(x)

x3

Answers

R
[1, )
[3, 3]
R
R
R
[5, 5]
R+

functions
b [0, ) or (, 0]
d (0, ), or (, 0)
f R\{2}
h [1, 0] or [0, 1]
j (, 1]
l (5, )

( 1, 0)

Range

E
D
C
[3, )
R+
[9, 0] or [0, 9]
R or R+

2 a f

: [3, ) R, f

f(x)

Inverse of function

4 a B
5 a C
6 a D
7 a (, 0] or [0, )
c [1, )
e R
g [1, 5] or [5, 9]
i (, 5) or (5, )
Exercise 5D Restricting
1 a [0, ) or (, 0]
c [3, )
e (, 4) or (4, )
g [5, 0] or [0, 5]
i [4, )
k R

f 1(x)

yx

+4

( x 1)
e 2

6
5

( x + 2)
k f 1(x) = loge

3
x
e

l f 1(x) = 3

l f 1 : R R, f 1 (x) = e 2 + 5
3 a B
b C
c D
4 a
y

( )

j f 1 : [4, ) R, f 1 (x) = 1 + x 4

k f 1 : R+ R, f 1 (x) = 2 + loge (x)

e f 1(x) = 3 2 x

g f 1(x) = x2 + 2, where x [0, )

i f 1(x) = loge x 1
2

j f 1(x) = 2 + loge (5 x)

688

h f 1 : [0, 1] R, f 1 (x) = 1 x 2

i f 1 : [0, ) R, f 1 (x) = x2 + 4

0
f 1(x)

(0, 1)

y
f 1(x)
( 5, 2)
5

2
0

f(x)

yx
(2, 5)

d A

g
y4

4 ( x 3)2 or

4 x

f(x)

yx

g 1 : [3, 5] R, g 1 (x) = 4 ( x 3)2


Chapter review
Short answer
y
y

b
1 a

f 1(x)

10 g 1 : [3, 5] R, g 1 (x) =

x4

(4, 4)

2 a

(2, 5)

5
3

f 1(x)

(5, 2)

y
6

6 x

0
6

0 1

[6, 6], [6, 0] R, R+


3 a

b
y

f(x)

yx

5 a [1, )

b f 1 : [3, ) R, f 1 (x) = 1 + (x 3)2


y

y  2
y
8

f 1(x)

0
1

4
( 4, 1)

1 x

0
1
2

1
0

( 4, 4)

6 a 4

b f 1 : [5, 31] R, f 1 (x) = 4 x + 5


y

( 2, 31)
f(x)

yx

10

4
x

(4,

4 a
)
b (, 2] or [2, )
5 a i One-to-one
x +1
iii y =
2
iv
y
6

f 1(x)
0

10

7 a f

: R R, f

b x
8 a

c x

(x) = 12
x

7 6 5 4 3 2 1 0
2

yx

iii y =

1(x)

1
2

d Teacher to check

iv

x 1 +1
2
y
2

y 

((x 2 1))(13) 1
y 2 (x 1)3 1

f(x)
0

9 a 0

b g 1 : [55, 9] R, g 1 (x) = 9 x
c [0, 8]

0.4 0.2 0

0.2 0.4 0.6 0.8

1.2 1.4 1.6 1.8

Answers 5d 5d

f(x)

1
2

b i One-to-one
0

y  x 1
2

d Teacher to check
b (, 0] or [0, )

1
2

y 2x 1

x
(31, 2)

c R or R+

Answers

689

c i Many-to-one
ii x 0

6 a

b f 1(x) = ex 1 + 2
c m = 1

x2

iii y = x + 1 1, x 0
iv

y
y |(x

1.5

2 2.4

0
2 x

)|

(2, ), R
y
d

1
0.5

R, (2, )
yx

y x 1 1

0.8 0.6 0.4 0.2 0

0.2 0.4 0.6 0.8

1.2 1.4 1.6 1.8 x

(1, 3)

2.4

0.5

2.4

d i One-to-one
iii y = -x2 - 6x - 7, [-3, )
y
iv
e (3, 1)

yx
0.5 1 1.5 2 x

3 2.5 2 1.5 1 0.5 0


2
y  x2 6x 7
4

2.4

y (2 x)(.5) 3

(3, 1)
0

2.4

( )

iii y = log e x 1
3

e i One-to-one
iv
x

y 3e 1

y
4
3

5
2

8 a b = 1
c

h 1 :[0,

2
1

y ln

x 1
3

f i One-to-one
2 +2
iii y =
3( x + 1)
iv
y

9 a f
b

1
2

(x 3)

4
1
1
2

3 2 1 0
2

690

Answers

R, f 1 ( x ) = 12 log e ( x + 1)

y
y x

0
1 2 3 4 5 6 7 x
2

y  (3(x 2)) 1

ii Restrict domain to (-3, ).

g i Many-to-one
iii y = 1 3
x 1
y
iv
y 

1
:( 1, )

8
6
2
y (3(x 1)) 2 4
2

0
5 4 3 2 1
2
4
6
8

h 

y 

1 2 3 4 x

3 2 1 10
2
3

1 12 ] R,
e

b Range h( x ) = [0, 1 12 ]
e
log e (1 x )

h 1 ( x) =

y 

1
(.5)

(x 1)

1 2 3 4

3

( 1, 1)

c f ( f 1 (2 x )) =
Multiple choice
1 B
2
5 D
6
9 C
10
13 B
14
17 E
Extended response

2x
2x + 1
E
A
D
A

3
7
11
15

C
C
D
D

4
8
12
16

C
A
C
A

852 000 x + 3 079 777 1127


426
b The inverse is not a function because it is a one-tomany relation (does not pass the vertical line test).

1 a y =

c a = 2.65

d Domain f 1 ( x ), [ 3.61, ), range f 1 ( x ), [ 2.65, )


e x = 2.87

2
2 a f(x) = 3 x + 4
2
4 4
b ( 3 , 3 )
c Domain = [2, 43 ], range = [2, 4]
y

yx

A ( 4 , 4 )
3 3

4 x

( 2, 2)

e Heart-shaped
4

f i [2, 3 )
1

ii f : [ 2,
g i

[ 43 ,

4
3)

R, where f (x) =

2( x 4)

2 a
36
c
13
d

8


3



4



5


6

g
j
a
d
g
j
a
d
g
j
a
d
g
a
b

8
15

4]

ii Inverse of f(x) is y =
3 a (2, 0.54) and (0, 0)
b R, [0, )
c
y

2
c
3

e 1.12c

f 1.38c

4
f(x)  exx2

3
4

9 D

10 B

11

13.5C, 15.8C

Exercise 6B Symmetry and exact values


3

2

1 a

y0

x
(0.54, 2)

f 1

x0

[0, ), R
(0.57, 0.57)
(, 2]
[0, 0.54]

2

2

2

2

2 a

2

2

3

2

y
10
8 y  6 ln (\x 3\)
6
4
2

2

2
1

2

1

2

3
3

3
3

f
i 1

2

2

3

2

2

2

1
2

1

2

1

2

b a = 3
c x = 5.50
d a = 4.97

2

2

1

2

3
3

2.4

CHAPTER 6

6 x

Circular (trigonometric) functions


Exercise 6A Revision of radians and
the unit circle
1 a 171.89
b 286.48
c 275.02
d 146.68
e 63
f 54
g 150
h 225

3 a 0.3
91

10

g 0.3
j

91

10

m 0.3

b 0.5
e

3

2

h 0.5

3
3
1

f 2.4
i

2.4

3

2

l 2.4

n 0.5

o 2.4

Answers 6a 6B

2 1 20
4

( 2, 0.54)

d
e
f
g
a

4.10c
h 4.54c
i 5.41c
6.11c
0.389
b 0.717
c 0.170

0.129
e 0.246
f 0.916
0.966
h 0.927
i 0.940

0.996
k 11.430
l 1.732
0
b 0
c 1

1
e Undefined
f Undefined
1
h 0
i 1
1
k Undefined
l 0
1
b 1
c 1
1
e 1
f 1
2
h 5
sin (240), sin (150), sin (35), sin (120), sin (70)
cos (3.34), cos (1.5), cos (5.3), cos (0.2), cos (6.3)

2( x 4)
3

yx

b
12

Answers

691

7 7
49 576 625
24
+ =
+
=
=1
25
24 24
625 625 625

5 a

3
3
,

2
3

, 3

1
2

c
6 a

1

2

or

2
,1
2

3 1
,
2
2

2

2

3

2

3

2

f 1

3

2

2

2

3

2

2

2

3

2

3

2

o 1

3

2

3

2

m
7 2

8 a

2

2

1

2

1

2

3

2

2

2

1

2

5
3
,

b ,
6 6
2 2
4 5
7
c
,

d ,
3 3
4 4
6 a 0, 180, 360
b 105, 165, 285, 345
c 45, 75, 165, 195, 285, 315
d 20, 40, 140, 160, 260, 280
e 62.40, 117.60, 182.40, 237.60, 302.40, 357.60
f 39.44, 140.56, 219.44, 320.56
g 26.39, 333.61
h 101.22, 258.78

7 a

b 0, 2
c 2.7184
2
5 13 17 25 29
, ,
,
,
,
d 0.9772
e
18 18 18 18 18 18
7 9 15 17 23
,
,
,
,
,
f
12 12 12 12 12 12
5 7 17 19
g
,
,
,

h 1.2579, 5.0253
12 12 12 12
3
7

b
8 a
2
2
16
11
c

d
3
3
e 19.7766
f 6.9266
9 a The particle is 3 metres from O.
b It takes the particle 1.99 seconds to reach O for the
first time.
11 11
10 a x =
,
, ,
6 6 6 6

5 a

3
3

3
3

b x =

2
1
1
1 1
9
+
= + =1
2 2
2
2

10 a 0.7
b 0.3
d 0.3
e 0.7
g 0.7
h 0.3
j 0.3
k 0.7
11 C
12 D
13 a 10 cm/s
b 11 cm/s
14 a 0.9 m
b 0.3 m

c x =
c 2.5
f 2.5
i 2.5
l 2.5
c 12 cm/s
c 0.7 m

Exercise 6C Trigonometric equations


1 a
d
2

3



4


692

a
d
a
b
c
d
a
b
c
d

3
5 7
,

b
,

4 4
2 2
3
5 7

e
,
2
6 6
90
b 60, 300
180
e 45, 135
2.2904, 3.9928, 8.5736, 10.2760
1.1442, 1.9973, 7.4274, 8.2805
1.0701, 5.2130, 7.3533, 11.4962
3.5217, 5.9031, 9.8049, 12.1863
15.58, 164.42
137.91, 222.09
212.90, 327.10
78.83, 281.17

Answers

7
c ,
4 4

c 60, 120

11 x =
12 x =

11 5 7
,
, ,
6
6 6 6
7 5

3
, ,
4 4

11 7 5
,
, ,
12
12 12 12

5 7
,
,
9 9 9

13 a x =

7
,
6 6

b x =

5 9 13
, , ,
8 8 8 8

5 11 17 25 29 35
,
,
,
,
,
18 18 18 18 18 18
4 x = 2.0944, 1.5116, 0.1799, 1.0472
1
c x =

Exercise 6D Trigonometric graphs


1 a 2, 1
d 2, 13
g 4, 3
2
j
,4
3

b 2, 1
2
e
, 2
3
h 6, 2

c 2, 4
f , 3
i ,

1
3

6 a 1, 2, 0 to 2
y

2
2
2
, 2,
to
3
3 3

2 a

y
0.8
0.6
0.4
0.2

1
0
1
P

P
2

3P

b 1, 2, 2 to 0
y

2P Q

0.5

b 4, 2, 4 to 4
y

y
3
2
1

2
P
2

3P

2
, 2 to 2
3

P
4

3P

P
2

0
1
2
3
4

PQ

5
4
3
2
1

2
1
0
1

P
6

P
3

P
2

2P Q

1
2

2 1
, to
3 2

1
2

y
0.6
0.4
0.2
0
0.2
0.4
0.6

P
6

P
3

P
2

1
3

to

P
4

P
2

3P

P Q

0.4

2
P 2P

3P

0.5
1
1.5
2
2.5

4
2
3 , 4

to 4

4
2
3P
P
2

2P Q

P
3

2P x

P
2

2.5
2
1.5
1
0.5

y
3
2
1

0.5
P 2P 3P 4P 5P 6P Q

0 P

P
2

3P

2
, 5 to 1
3

g 3,

y
1

4 2, , 2 to 2

1
2
3
4
5

y
2
1

P
2

P
6

0 P
P P
2 1
2
2

5 a
to the left, up 3
b to the right, up 1
3
2

to the right, down 2 d to the left, down 1


c
4
3

P x

1
,
2

0 P
6

P
3

P
2

Answers 6c 6d

3 4,

f 1, , 0 to 2
y

h 3, 6, 3 to 3

0
1
2
3

4P Q

2P x

3P

y
0.5

1
3

y
0.4

0
0.2

2P Q

0
2
4

1
3,

0.2

g 4, 4, 4 to 4

y

0
2

P
2

2
, 2 to 0
3

e 1,

2P x

d 2, 2, 1 to 5
y

2P x

c 2, 2, 4 to 0
y

0
1
2
3

0
0.5
1
1.5
2

c 3, , 3 to 3

d 2,

2P x

3P

0
0.2
0.4
0.6
0.8

0
2

P
2

2P x

, 2.5 to 3.5

y
3
2
1
0

P
4

P
2

3P

P x

Answers

693

2
g
,1
3 y

i 3, 4, 1 to 7
y

6
4

2.5
2
1.5
1

2
0

2P

4P x

3P

1 0
1.5
2
2.5

j 2, 6, 3 to 1

y
0 P 2P 3P 4P 5P 6P x
1
2
3

1
2
3
4

0 P P 3P P
5P
3P
7P 2P Q
0.5 4 2
4
4 2 4
1
1.5

1
2
3

b 2, 1
y

P
2

3P

2.5
2
1.5
1

2P Q

1 0
1.5
2
2.5

c 2, 3
y

3
2
1

4P
5P P
7P
4P
9P
5P11
P P P
P 2P
6 3 2 6 6
6 3 6 3 6

d 2, 2
y

3
2
1

P
2

3P

2P Q

P
2

3P 2P Q

2
,3
3
y

694

Answers

g
a
d
g

b 3, 4, 3 to 3

d 1, 6, 1 to 1

f 5, , 7 to 3
2
1, 2, 1 to 1
h 1, 2, 1 to 1
1, 1
b 1, 1
c 3, 3
2, 2
e 2, 2
f 3, 3

5, 3
h 1, 3
i 1, 5

j 5, 3
10 a 2, 1, 1 to 1

y

P
2

k 2 12 , 1 12

3P

2P x

3P

2P x

b 2, 1, 1 to 1

y
1.5
1
0.5
0
0.5
1
1.5

P
2

c 2, 2, 2 to 2
y

3
2
1

8 a 4, , 4 to 4
2
c 2,
, 2 to 2
3
e 3, , 2 to 4

0.5
1
1.5

3
2
1

0
1
2
3

2P Q

1.5
1
0.5

e , 2
y

0 P P
3P P
5P
3P
7P 2P Q
4 2 4
4 2 4

j 4, 1
y

1.5
1
0.5
0
0.5
1
1.5

0
1
2
3

0 P P 3P P 5P 3P 7P 2P Q

4 2 4
4 2 4

i 2, 2
y

1.5
1
0.5

2P Q

7 a 2, 1

y

1
2
3

3P

h , 2
y

0
1
2
3

P
2

P
2

3P

2P Q

2
1

0
1
2

P
2

3P

2P x

3 2 , 4 1
3
3

d 2, 3, 3 to 3

y

Exercise 6E Graphs of the tangent function


1 a Period =
y

3
2
1
1
2
3

4
2
0

P
2

3P

2P x

b Period =

e , 3, 3 to 3
y

3
2
1

0 P P 3P P
5P
3P
7P 2P x
1 4 2
4
4 2 4
2
3

c Period =

2
, 1, 1 to 1
3
y

d Period =

P
2

3P

2P

e Period = 4

g 2, 4, 3 to 5

y
f Period = 3

0
2

P
2

3P

2P x

P
2

3P

2P x

g Period = 2

2 1 1
i
, , to 1 12
3 2 2

y
h Period =

1.5
1
0.5
0
0.5

2P P
P
3 3

4P
5P 2P x

3 3

j 2, 1, 1 to 3
y

3P

) + 1
12 y = 2 cos 3 (x ) + 1
3
4
2
)2
13 y = 3 sin (x
3
4 a 1
1
b 12
c 3 metres
d 1 metre
e 3.00 am and 3.00 pm
x
f 9.00 am and 9.00 pm g y = sin + 2
6
11 y = sin (x +

0
1
2

j Period = 2

P x

y
2
1
8

y
2
1
2P

4P x

3P

3P

y
2
1
x

y
3
2
1
P

0
1
2
3

2P x

y
5

P
4

P
2

P
8

P x
4

2P x

y
2
1
0
1
2

2P x

Answers 6e 6e

P
2

y
2
1

i Period =
4

3
2
1

P x

0
1
2

h , 2, 4 to 0
y

0
1
2
3
4

0
1
2

0
1
2

4
2

P
2

y
2
1

0
1
2

0
2
4

y
1
1
3

13

Answers

695


3
ii Dilated by factor of 2 from the x-axis and by factor
of 13 from the y-axis

b ii Period =
4
ii Dilated by factor of 14 from the y-axis; reflected in
the x-axis; translated 1 unit up

c i Period =
2
ii Dilated by factor of 3 from the x-axis and by factor
of 12 from the y-axis; translated 4 units down
d i Period = 2
ii Dilated by factor of 2 from the y-axis; reflected in
the x-axis; translated 2 units down
e i Period =
ii Dilated by factor of 5 from the x-axis; reflected in

the x-axis; translated units to the left


2

f i Period =
2
ii Dilated by factor of 12 from the y-axis; translated

units to the right


4

g i Period =
3
ii Dilated by factor of 14 from the x-axis and by factor

of 13 from the y-axis; translated units to the left


6
h i Period = 3
1
ii Dilated by factor of 2 from the x-axis and by
factor of 3 from the y-axis; reflected in the x-axis;
translated 6 units up

i i Period =
4

ii Dilated by factor of 14 from the y-axis; translated


12
units to the right and 13 units down
j i Period =

ii Dilated by factor of 2 from the x-axis; translated


3
units to the left and 5 units up
y
3 a Period =
2 a i Period =

2
1
0
1
2

b Period =

Answers

P
4

P
2

3
P
4

P x

P
2

P
3
4

P x

y
2
1
0
1
2

696

P x

y
2
1
0
1
2
3

c Period =

d Period =

y
2
1
P
2

0
1
2

e Period = 4

x =P

y
1
0

f Period = 6

P
2

P x

y
1
0

g Period = 2

P
2

2.78 P x

y
1
0
1

h Period =

i Period =

j Period = 2

P
2

y
4
3
2
1

2.21 P x
x P

0
P P 5P 3
P 7P P x
1 P8 P4 3
8 2
8 4
8
2
3
4
y
3
2
1
0 P
1 24
2
3
y
3
2
1
0
1
2
3

P
7
24

13
P
24

19
P P
24

P x

Exercise 6F Finding equations of trigonometric


graphs

1 a 3, 2, y = 3 sin (2x)
b 2, , y = 2 sin ( x)
6
6

2 a 2, 4, y = 2 cos (4x)
b 2, , y = 2 cos ( x)
3
3
3 a 12 , 1, 1, y = 12 sin (x) + 1

b 3, , 2, y = 3 sin ( x) 2
4
4

4 a 4, 2, , y = 4 cos 2(x )
2
2

b 5, 2, , y = 5 cos 2(x )
4
4

5 a 2, 1, , 1, y = 2 sin (x + ) 1
3
3

b 3, 1, , 2, y = 3 sin (x + ) + 2
2
2

6 a 1, , 3, y = 3 cos ( x)
6
6

b 1, , 2, y = 2 cos ( x)
4
4
7 D
8 E
Exercise 6G Trigonometric modelling
1 a 8.5 m, 3.00 pm b 12 hours
c 3.5 m
d d(t)
8
6
4
2

Exercise 6H Further graphs


1 a
y
1.5
1
0.5
0
0.5
1
1.5

0 2 4 6 8 10 12 14 16 18 20 22 t

2
1.5
1
0.5

0 0.5 1 1.5 2 2.5 3 3.5 4 4.5 t


0.5

5 a 2, 1

b h(t) = 18 16 cos (
c 5, 34 m
e 3 minutes

2
t)
3
d 2 m
f 26 m

0
1
2
y
2
1
1
2
y
2
1

P
2

3P

2P x

P
2

3P

2P x

0 P
2

3P

0 P
2

3P 2P x

y
2
1
1
2

0 0.5 1 1.5 2 2.5 3 3.5 4 4.5 5 5.5 x

T = 20 10 cos (t + 21)
12

2 a

0 P
2

h 13C

2P x

3P 2P x
P
2

y
2P
P
0

or T = 20 + 10 cos (t 15)
12

3P
P
2

Answers 6f 6h

c 20
e 3.00 am, 3.00 pm

f T = 20 10 cos (t 3) or
12

0 P

y
2
1
1
2

b 24

or T = 20 + 10 cos (t + 9)
12

g 10C, 20C, 30C, 20C

1
2

30
25
20
15
10
5

3P

P
2

2
1

g h(t) = 18 16 cos
(t + 1)
3

h
y

7 a 10
d 20

y
2
1

1
2

b 8 m, 8

2
6 a 18, 16,
3

2P x

0.5
1
1.5

2 4 to
6 85.00
10 12 14am
16 18
20 22
d 1.00 am
Saturday,
1.00 pm to 5.00 pm
Saturday, 1.00 am to 5.00 am Sunday morning
e 3.00 am to 5.00 am Sunday
3 a 18C, 14.5C
b 25C, January, December
c 21.5C
d 8 months
4 a 0.9, , 0.9, h = 0.9 sin ( t) + 0.9
b
c 1.3 seconds
h

3P

1.5
1
0.5

e 1.00 pm and 5.00 pm, 1.00 am and 5.00 am


2 a 10 m, 4 m
b d

c 3.00 am
10
8
6
4
2

P
2

2P x

y
1
0
1
2
3
4
5

P
2

P
3
2

2P

Answers

697

y
0
2
4
6
8
10
12

y
10
8
6
4
2

P
2

2P x

P
3
2

P
2

2P

P
3
2

0
5
15
20
25
30
35

P
2

2P x

3
P
2

2P

2P

P
3
2

P
3
2

2P x

P
2

P
3
2

2P x

P
2

P
3
2

2P x

y
3
2
1
0

P
2

y
6
4
2
0

y
5
4
3
2
1
0 P P 3P P 5P 3P 7P 2P x
4 2
4

y
5

4 a

0
5
10
15
20

P
2

3
P
2

2P

10
2
3
4
5

3 a

P
2

P
3
2

2P x

P
3
2

2P x

P
3
2

2P x

698

Answers

2P

P
2

P
3
2

2P x

(2.03, 0.91)
P
2

P
3
2

2P x

b Domain: R
P
2

P 2P x
3
2

y
3
2
1
0
1

3P
2

(4.91, 2.41)

0
1
2

P
2

y
3
2
1

3
2
1
P
2

2P x

5 a Domain: R

y

y
2
1
0

P
2

P
3
2

y
3
2
1

0
1

P
2

0
1

y
3
2
1
0

y
3
2
1

4 2 4

P
2

P
3
2

2P x

y
5
4
3
2 (2.03, 0.91)
1
0
1

P
2

P
3
2

2P x

(3.52, 2.34)

c Domain: R+

y
3
2
1
0
1
2
3
4
5

(2.13, 1.92)

P
2

0 (1, 0)

2P x

P
3
2

x0

(4.84, 4.69)

d Domain: R+ {0}

y
3
2
1
0
1
2
3
4
5

( 1, 0)
0 (1, 0)
x
( 0.58, 0.38) (0.58, 0.38)

(1.65, 1.28)

P
2

2P x

P
3
2

y
( 1.7, 2.7)

(3.29, 3.59)

( 2, 0)

e Domain: R
y

4
2
0
2
4

P
2

2P x

P
3
2

y
(0, 1)

f Domain: R

y

(5.57, 7.07)
8
6
4 (.61, 2.61)
(2.67, 1.08)
2

0
2
4
6

P
2

2P x

P
3
2

1
(0.5, 0.5)

(4.05, 3.98)
(1, 0) x

g Domain: R

y
3
2 (.18, 1.02)
1
0
1
2

(1, 0) x

P
2

(2P, 3.14)

2P x

P
3
2

(1, 0)
0 (0.61, 0.18) x

(3.32, 1.8)

h Domain: R (, 2]

y

2P 3P P P2 10
2
2
( 3.5, 2.3)

2 x

0
1

P
2

P
3
2

2P x

b ii f (g(x)) = sin (2 x ); domain f (g(x)) = R+ {0}


iii y

2 (0.62, 1)
1

( 2, 0)
0
( 113 , 1.09)

(0, 2)

(2, 0) x
(1, 2.7)

0
1
2

P
2

P
3
2

2P x

(5.55, 1)

c ii f (g(x)) = 2 sin (

x2
); domain f (g(x)) = R
4

Answers

Answers 6h 6h

7 a ii f (g(x)) = cos (loge (x)); domain f (g(x)) = R+


iii y (1, 1)

1 (1.27, .51)

6 a

699

y
2 (2.51, 2)
1

iii

0
1
2

P
2

b i
ii


iii

(5.6, 2)
2P x

P
3
2

(4.34, 2)

Both f (g(x)) and g( f (x)) are defined.


f (g(x)) = x 2 1 g( f(x)) = x 2 1
Domain is R Domain is R

Range is [0, ) Range is [ 1, )


y

f(g(x))

d ii f (g(x)) = 2 cos ( x ) + 2 ; domain f (g(x)) = R


iii y
2
1
0
1

(0, 1)

P
2

2P x

P
3
2

( 1, 0) 0

(1, 0)

g(f(x))

e ii f (g(x)) = (sin (x))2; domain f (g(x)) = R


iii y
2
1

0
1

P
2

2P x

P
3
2

( 1, 0) 0

f ii f (g(x)) =
iii y

2cos (2x);

domain f (g(x)) = R

2
1

0
1

P
2

2P x

P
3
2

(1, 0)
(0, 1)

c i
ii


iii

Both f (g(x)) and g( f (x)) are defined.


f (g(x)) = 1 e2x g( f (x)) = e(1 x
Domain is R Domain is R
Range is ( , 1) Range is (0, e)
y

y1

x2

(0, 0) x

g ii f (g(x)) = cos ( ); domain f (g(x)) = R


8
iii y
2
1

0
1

P
2

P
3
2

2P x

f(g(x))

(5.01, 1)

y
(0, 2.7)

h ii f (g(x)) = 2 sin ( x 3) + 1;
domain f (g(x)) = R+ {0}
iii y

g(f(x))

2
1
0
1

P
2

(2.04, 1)

P
3
2

2P x

8 a i Both f(g(x)) and g( f (x)) are defined.


ii f(g(x)) = ex 2 g( f (x)) = ex 2
Domain is R
Domain is R
Range is (2, ) Range is (0, )
iii
y
f(g(x))

(0.69, 0)
0

(0, 1)

y  2

d i Only g( f (x)) is defined.


ii g( f (x)) = sin ( x )
Domain is [0, ), Range is [1, 1]
iii y
(2.47, 1) g(f(x))
( 2, 0)

e i Both f (g(x)) and g( f (x)) are defined.


g( f (x)) = cos (ex)
ii f (g(x)) = ecos (x)
Domain is R
Domain is R
Range is [ 1, e] Range is [1, 1]
e
y
iii

g(f(x))

700

Answers

( , 1 )
e
2


(0, 0.14)
0

(0, e) (2 , e)
f(g(x))
( , 1 )
e
x
0
2

y
1

g(f(x))

0
1

b Amplitude = 3, period = 2
c
y

f i Only g( f(x)) is defined.


ii g( f (x)) = sin (loge (x))
Domain is (0, ), Range is [1, 1]
iii y (4.8, 1)
1

0
1

g(f(x))

5 10 15 20

2
1

Exercise 6I Trigonometric functions with an


increasing trend
1 a T(m)
P
20
18
16
14
12
10
8
6

10

20

1
2

30

40

2
x
8 a = 3, n = 12 , c = 1; y = 3 sin 1
2
9 a y

50

60 t

b $20
d April 2008

5
2
0

e 2

5

3
7
k

6
h

2

2

2

2

e 0

2

2

j 1
3 a 0.85
c 0.85
(8n + 1) (8n 1)
4
,
4
4
5

3

3

i
l

4
3
2
5
4
7
4

11

3P
2

2P

y
12
y  2x

10

3
2

2
2

6 a y = sin (x) is dilated 3 units from the x-axis, and

translated 4 units to the right and 2 units up.

24

2
1

3
3

(6 n + 1)
7 13 19
,x=, ,
,
,
12
12 12 12 12

18

b 10 am, 2 pm, 10 pm, 2 am


c Water sports may run between 10 am and 2 pm.
10 y

b 0.85
d 0.85

12

y  sin (x) 2x

4
2
0
2

12

y  sin (x)
P

2P

Answers 6i 6i

Period =

Chapter review
Short answer

1 a
3

d
2
5
g

6
2
j

3

2P

18964
December 2009
2%
$20.12

0 P P 3P

4 2 4

c December 2008

25000
20000
15000
10000
5000

2 a

P
3
2

y
4

2 4 6 8 10 12 14 16 18 20 22 24 m

b 20.9C
2 a 100
b
M

c
d
a
c

0
1

T(m) 12 0.2m 5 cos (m)


6

y
6
4

yx

2
0
2
4
6

y  sin x
P

2P

y  x sin x

Answers

701

13 a
b
c
14 a
b

Teacher to check. Show that range g domain f .


f (g(x)) = sin (x2 + 2)
Domain f(g(x)) = R, range f(g(x)) = [1, 1]
amp = 5 period = p
y
5
4
3
2
1
5 4 3 2 1 0
1
2
3
4
5

Multiple choice
1 E
3 D
5 B
7 C
9 E
11 A
13 C

1 2 3 4 5 x

2
4
6
8
10
12
14

A
C
D
B
D
E
B

Extended response

1 a 5, 5, 6 and , D(t) = 5 5 sin (t + 6)


12
12

b 5, 1.46
c 8.30 am and 3.30 pm. 8.30 am the next day.
d 8.27 am or 8.46 hours after midnight and 3.32 pm or
15.54 hours after midnight.
Differences occur due to inaccuracy of graphical
methods.

e D(t) = 10 5 sin (t + 6)
12
2 a 1.5 m
b 19.5 m
c 18 m
d 60 s
3
e i
m/s
ii 0.94 m/s
10
f f (m)
19.5
10.5
1.5

0
1
2

30

60

Exam practice 2
Short answer
1 x = 4 12
2 x = -2
3 a f(x) = e-x - 2 + 3
1
b ( 2 + 3, 4)
e
1
4 a Dilation of 2 vertically and 2 horizontally, translation

of left
3

1
b y = sin 2( x + )
2
3
5
9 3
or
5 x = ,
12 12
12 4
6 [3, )
7 x = loge (2)
8 a
y
f(x) xsin (x)

y sin (x)

0.8 0.4 0
1

0.4 0.8 1.2 1.6

Differentiation
Exercise 7A Review gradient and rates of
change
y
y
1 a
b

f '(x)

y
0

f '(x)

6000

702

Answers

2 4 6 8 10 12 14 16 18 20 22 24 t hours

c 0 at t = 10 and t = 14
d 9:20 am
e 2h 40min

CHAPTER 7

8000

2000

2.4 2.8

b 0
c 1.8
Multiple choice
1 E 2 C 3 C 4 B 5 D 6 B 7 E
Extended response
a 200 birds b 12 months c 300 birds

d 7 months e a = 200, b = , c = 2, d = 300


6
f 473g A = 500, k = 0.005018
P ( x )Q( x )
h Consider the graph of
.
500

10 000

4000

90 120 150 t (sec)

g d = 10.5 9 cos ( (t 6))


30
h 17.78 m
3 a m = 8000
b
C

y x

f '(x)
0

f '(x)

y
f '(x)
5
2

2 a B
3 a

g'(x)

b C
b

i i n/a
j i n/a
8 a

g'(x)

g'(x)

g'(x)

ii ( , 0) iii (0, )
ii ( , 0) iii (0, )
y
b

y
1

f '(x)

e
x

b A
b

f '(x)

f '(x)

f '(x)

g'(x)

1 0

y
f '(x)

f '(x)

ii
iv
iii
iii
iii
iii
ii
iv
iii
ii

c E
R
n/a
R
iv
(1, ) iv
(, 4)
iv
R\{0}
iv
(2, 3)
n/a
n/a
iv
( , 1) (2,

n/a
n/a
n/a
n/a
n/a
)

a
d
g
j

y
1

R\{2}
R\{1, 2}
R\{2}
R\{5, 4}

b R\{0}
e R\{3}
h R\{2}

0
1

x
f '(x)

c R
f R\{0}
i R\{5}

Exercise 7B Limits and differentiation from


firstprinciples
1 a 6
b 7
c 8
d 4
e 6
f 14
g 24
h 3
i 2

2 a 3
b 3
c 2
d 4
e 1
f 7
g 12
h 27
i 6

3 a 5
b 6
c 5
d 35
e 48
f 10
g 9
h 0
i 1
4 4 + h
5 E
6 a h b 0
7 4

Answers

Answers 7a 7B

D
b C
i Not applicable (n/a)
iii n/a
i n/a
ii n/a
i x = 1 ii ( , 1)
i x = 4
ii (4, )
i x = 0
ii n/a
i x = 2 and x = 3
iii ( , 2) (3, )
i x = 2
ii R\{2}
i x = 1 and x = 2
iii (1, 2) iv n/a

f '(x)

f '(x)

f '(x)

6 a
7 a

b
c
d
e
f

g
h

y
0

f '(x)

g'(x)

g'(x)
y

f '(x)

f'(x)

2 0

g'(x)

4 a B
5 a

g'(x)

iv x = 0
iv x = 0

703

8 a
9 a
10 C
11 a
d
12 a
d

Approx. 4
Approx. 10

b 4
b 10
b
e
b
e

3
2x 2
4
3x2 4

c 2x + 6
f 3x2
c 6x + 8
f 2x 2

2x
6x
2x + 3
5 6x2

Exercise 7C The derivative of x n


1 a
c
e
g
i
2 a
c

dy
= 6x5
dx
dy
= 20 x 3
dx
dy
= 12 x 2
dx
dy 3 2
= x
dx 2
dy
=0
dx
f( x) = 12x2 + 5
h( x) = 35 x2

b
d
f
h
j
b
d

e g( x) = 77x10 + 30x4
g f( x) = 6 + 6x 12x2
i h( x) = 2x + 3

6
3 a

x4
1

2
3x 3

e
g

2 x
x

2
+ x 3
3

i 1 + 2 x

4

5x3

m x 2 4 3 x
10x4

4 a
5 D

10

8 a

5
8

9 a

1
3 2

g

h

10 a g ( x) =

b Undefined
b
1
2

15
8716

+ 4

53

c 467 54
b

1
43

1
412

3x 3
c

1
412

704

i
iii
i
iii
i
iii

x2 + 2x + 1
2(x + 1)
x3 + 3x2 + 3x + 1
3(x + 1)2
4x2 + 4x + 1
4(2x + 1)

Answers

4

u5

3
2u 2

1
2
4

6
4
3

iii

ii 5

iii

ii 3

iii

2
(2 x 5)2

8

(4 2 x )5
5
2 5x + 2

9
3

2(3 x 2) 2

ii 4x + 5
i 15u4
iii 15(4x + 5)(2x2 + 5x)4

i 2u 3

ii 4 6x

iii 4(2 3x)(4x 3x2) 3


x2 1
i i 6u5
ii
x2
5
1

6( x 2 1) x +
x

iii
2
x

j i 16u 5
ii 6
iii 96(5 6x) 5
4 C
5 A
6 B
7 a 32(8x + 3)3
b 6(2x 5)2
3x
c 15(4 3x)4
d
3x 2 4
2
2

e ( x 2)( x 2 4 x ) 3 f 2(6x2 + 1)(2x3 + x) 3


3
5

1
1

g 6 1 + 2 x

x
x
8

h (2x 3)(x2 3x)

ii 3x2 + 6x + 3
ii 8x + 4

ii 2x + 2

81x2 + 54x + 9

1
3

18x + 6

iii

2 u

24x2 + 24x + 6

ii 2

(4 x + 7)3

3
9 a
3
(6 x 5) 2
10 a 8x7(2x + 5)(x + 5)7

11 Teacher to check
12 a

b

c

e i
f

7 3 4

d i

1
5x 4

1
+ 2x
f
x2
3
h x 2
3
j 4 x 2
1
6
3
l
3
x
x2
1
1
5
n
2 x
4x 4
b 150
6 C

4x

f f( x) = 2x4 + x2
h g( x) = 14x 4
j f( x) = 9x2 12
3
b
2 x
d

dy
= 6x
dx
dy
= 20 x19
dx
dy
= 5
dx
dy 4 3
= x
dx 3
dy
= 40 x 4
dx
g( x) = 10x + 6
h( x) = 3 + 12x + 3x2

d i 8x3 + 12x2 + 6x + 1 ii
iii 6(2x + 1)2
e i 9x2 + 6x + 1
ii
iii 6(3x + 1)
f i 27x3 + 27x2 + 9x + 1 ii
iii 9(3x + 1)2
13 na(ax + b)n 1
Exercise 7D The chain rule
1 a i 5x 4
ii
b i 3x + 1

ii
c i 2x + 3

ii
d i 7 4x
ii
e i 5x + 3
ii
f i 4 3x
ii
2 B
3 a i 2u
ii
iii 6(3x + 2)
b i 3u2
ii
iii 3(7 x)2

1
c i 2
ii
u

3x 2
5( x 3

+ 4x

2x2

4
7) 5

b 3 x x 2 + 2
b 2x(3x2 2)(x2 2)
d 3x(4x2 3) 2 x 4 3 x 2 + 1

11 37 500

7
500

12

13 a 2

c 1
7
14 4

d 1

15 9

16 4

17 7

or 0.014
x 1

x2 2x + 1

18 5

Exercise 7E The derivative of e x


1


2

a
d
g
a
d
g


3
4


5

j
A
a
d
g
a

10e10x

b
e
h
b
e
h

ex
2x

12e
6e6x 2
8e7 2x
90e6 9x
x

e2

+1

x
1 3
3e
6e3x

c
f
i 22e
c 10e5x + 3
f 10e6 5x

i 42e 7x

e0.2x
6e8 6x
24e8x + 1
15e3x + 4
e

x
3

x
+5
e 4

2 x

e (2 x 3)e6 3 x + x
g 3(8 x 7)e 4 x

7 x

5e 2 5 x
2

+3 x 2

h 10(1 + 3 x )e1 2 x 3 x

i 6(2 x + 1)2 e( 2 x +1)


3

2( x + 2) 3 e( x + 2) 2

1
3

4 ( 4 x )3 e ( 4 x )

3e

3x + 4

2 3x + 4

2
3

8

8x 1

log e (x)

1
x
1
d
x
1
h
x

3x 2 + 4 x 7
x3 + 2x 2 7x

4 x3 6x 2 + 2x
1
e

2x + 1
x 4 2 x3 + x 2

2
4x 3

5

3(5 x + 2)

12

3x 2

7 a

4x

x2 + 1

4x
x2 3

6( x 1)

( x 3)( x + 1)

6
x+2

18
7

10

5x + 8

c C
3
3x 4

5
5
or
3 5x 5 x 3

1
1
or
x2
2 x

7
7
or

7x 4
4 7x

16
i

2x 1

48
j

12 x + 5

1
x+3

3
4 + 3x

1
3

x
cos
3

2
3

2x
cos
3

+ 3 x )2

2 a 3 sin (3x)
d

x
4 sin 4 e

a
d
a
a
a

1
8

x
sin
8

30
5x + 2

63
k
8 9x

1
3

x
sin
3

2
5

2x
sin
5

b 4 sec2 (4x)

3 a 2 sec2 (2x)

4

5
6
7

x
cos
2

b 2 sin (2x)

x
sec2
5
A
B
cos (u)
sin (u)
2 cos (2x + 3)

1
2

1
5

b
e
b
b

3x
d 43 sec2
4
E
c D
C
4
c 4 cos (4x + 3)
3
c 3 sin (3x + 1)
b 7 cos (6 7x)
3x + 2
cos
4

c 5 cos (5x 4)

3
4

e 102 cos (2x)

3
2

8 a 3 sin (3x 2)

b 4 sin (4x + 7)

c 5 sin (6 5x)

10 95
11 15 13
1 log ( x )
12 a e e or 1
x
b i 1
ii 1
iii 1
iv 1
The gradient is always 1 since e loge ( x ) = x.
2 log ( x )
13 a e e or 2x
x
b i 2
ii 10
iii 4

3

5(3 x 2)

e 402 sin (10x)

3x
cos
8

2x + 3
sin
3

f 12 sin (2x)
2
3

Answers

Answers 7c 7g

Exercise 7F The derivative of


1
1 a 4x
b 4
c
u
1
1
1
2 a
b
c
x
x
x

3
1
6
e

f

g
x
x
x

4
5
i

j
x
x
3 D
4 a A
b D
2
6
5 a

b

2x + 5
6x + 1

x2

Exercise 7G The derivatives of sin (x), cos (x)


and tan (x)
1 a 8 cos (8x)
b 6 cos (6x) c cos (x)

e( x +1)

n 2x(2x + 3)(x + 3)e( x
3( x + 1)

6 B 7 20e 8 6e2 9 25e 3


m

2x

+3

f 3( x 2 + 1)e x

4

x

8 E

2ex
b 9e3x + 6e2x
c 20e 4x + 10

3ex 2e x
e 6e2x 7e 7x
f 36e9x 2ex
ex
h 20e5x + 4x
(2 x + 3)e x +3 x
b (2 x 3)e x 3 x 1

c 2( x 1)e x

x
1 4
4e
20e5x
11x

6 a

705

2

cos (2 x + 1)
5
c

cos2 (5 x 2)
3
e
2
cos ( x )
9 a

cos (8 x )
2
d
cos2 ( x + 1)
2

2(2 x) sin (x2 4x + 3)


(2x 5) cos (10 5x + x2)
ex cos (ex)
(2x + 7) sin (x2 + 7x)
4 2x
e
cos2 (4 x x 2 )
2x + 3
f
cos 2 ( x 2 + 3 x )

10 a
b
c
d

n
o
p
q
r
5

3
4

3x

3 xe 3 x
2 x

4
6x sin (2x + 3) + 4x 3 cos (2x + 3)

6 xe 2 x
2x
2e loge (3x2 + 5) + 2
3x + 5
3x
3x
8x + 12e 2xe + 3 x 2 e 3 x
3
4x + 9x2 + 16x 18

+ 2 loge (4)

Exercise 7I The quotient rule

11 3.745
2 0
1
13 a i cos (x) esin (x)

ii

b i sin (x) ecos (x)

ii

1
3e
3 2
e or
2
2

1
e
2

3
2

cos ( x )
or cot (x)
ii 3
sin ( x )
1
sin ( x )
d i
or tan (x) ii
cos ( x )
3
Exercise 7H The product rule
1 a u = x + 3, v = 2x2 5x
du
dv
b
=1,
= 4x 5
dx
dx
c 6x2 + 2x 15
2 a 4x2 + 12x2 loge (6x) or 4x2 (1 + 3 loge (6x))
3x 2
+ 3 loge (2x)
b
x
3 D
4 a cos (x) x sin (x)
b 3 sin (x) + 3x cos (x)
c 3ex + 5xex

Answers

4x
4x
cos cos (x) sin sin (x)
3
3

6 10
7 17.279
26
8
e
9 6 2

p 2(x + 1) sin (x2 + 2x) + 3 cos (3x 9)

706

4
3

e4 x 3
x

l 20e 5x sin (2 x) 4e 5x cos (2 x)


cos (6 x )
6 sin (6x )

m
x
2 x3

x
sin
4

c i

k 4e4x 3 loge (6x) +

1
2

d
e
f
g

1
sin (loge (x))
x
h 4e4x cos (e4x)
1
1
i
sin
x
x2
2
j
cos (loge (2x 1))
2x 1
3x
k 6e sin (2e3x)
3
l sin (loge (10x))
x
4 x (3 x + 4)
m
cos2 ( x 3 + 2 x 2 )
24
n
3x
5 cos2
5

5e3x

33xe3x
5x cos (3x + 1) 3x5 sin (3x + 1)
6x2 loge (7x) + 2x2

2e 2 x
2x
2e loge (2x 5) +
2x 5
40 log e (5 x ) 8 tan (5 x )
h
+
x
cos2 (5 x )
i 5 cos (2x) cos (x) 10 sin (2x) sin (x)

du
dv
= 1,
=1
1 a u = x + 3, v = x + 7
b
dx
dx
dy
4
c
=
dx ( x + 7)2
2 a u(x) = x2 + 2x, v(x) = 5 x
b u(x) = 2x + 2, v(x) = 1
2 10
x + x + 10
c f ( x ) =
(5 x )2
2
3 sec (x)
2
3x 2 + 4 x 4

b
4 a
2
( x 4)
(3 x + 2)2
c

(sin

( x ) + cos ( x ))

ex

e 2 x (2 x 1)

x2
2
g
cos2 (2 x )
e

33
(10 x )2

3 x (2 log e (4 x ) 1)
(log e (4 x ))2

x 2 + 2 2 x ( x + 1) log e ( x + 1)
( x + 1)( x 2 + 2)2

e3 x + 2 (3 cos (2 x ) + 2 sin (2 x ))
cos 2 (2 x )

j
k
l
m
n

5x

3 a e

4( x 2

+ 2)
( x 2 + 3 x 2)2
10 x 3

+ 6 x 2 35 x + 7
e5 x

3x 2 + 5

d
e

2 x3
9( x + 3)e 3 x

(3 x

+ 8)2

4[ x 2 2( x 1) log e (8 x )]
x 2 ( x 2)2

x cos ( x ) 2 sin ( x )
2x2
4[ x sin (3 2 x) cos (3 2 x )]
p
x3
3e 2 7 x (7 x + 22)
q
( x + 3)2
4( x 2)e 2 x
r
(2 x 3)2
o

5 D
7 E

6 D
8 3

9 Does not exist as f ( x) =


10

4 (1 log e (10))
75

15
0

Exercise 7J Mixed problems on differentiation


1 a C
b P
c Q
d Q
e P
f Q
g C
h P
i C
j C
k Q
l C
m C
n Q
1
2 a
x
b 3 (sin (x) + x cos (x))
3 x 14
c
4 x3
4 (cos ( x ) + x sin ( x ))
d
cos2 ( x )
e e5x (5 sin (x) + cos (x))

j 2 sin (x) cos (x)

3x
ex

g
h

2 (3 x + 1)3
3
x +1
2(3 x) sin (x2 6x)
ex (cos (2x) 2 sin (2x))
2
cos2 (2 x )
cos ( x )
sin 2 ( x )
3 cos (3 x )
sin (3 x )

i 4(6 x 5) e3 x 5 x + 2
j 3x2 + 8x 2
k ex (x2 + 2x + 3)
(4 x 3 9 x 2 50)(2 x + 3)4
l
( x 3 5)2
4x
m 4e [cos (4x 3) + sin (4x 3)]
n 6 cos (3x) sin (3x)

3 sin (3 x )
o
cos (3 x )
2

2(2 x 4 cos ( x 4 ) sin ( x 4 ))


x3

2
q
sin 2 (2 x )
p

20 [log e (5 x 1)]3
5x 1

1 2 log e ( x )
2x2
5
x+3
t
cos
x2
( x 2)2
u 3x4 [5 cos (2x + 1) 2x sin (2x + 1)]
s

( )

3 4 log e x 32
2x3
w (sin (x) + x cos (x)) ex sin (x)

x 6 cos (x) sin (x) 7e 7x 3x2

2 x, x < 1 or x > 1
4 a i

2 x, 1 < x < 1

ii
f(x) {x2 1{

2x log e ( x )

m sin (x) ecos (x)


1 or sec2 (x)
n
cos2 ( x )

2
2e 2
x
z 3 sin (x) cos2 (x)
y 18 cos (6x ) +

6
5
4
3
2
1

5 4 3 2 1 10
2
3
4

f '(x)  2x, x < 1

f '(x)  2x, x > 1

f '(x)  2x, 1< x < 1

Answers

Answers 7H 7j

x (2 x + 9) log e ( x ) x 2 9 x + 8
x (log e ( x ))2
g 2(2 x) sin (x2 4x)
1
h e x ( loge (5x))
x
cos ( x )
i
or cot (x)
sin ( x )
f

[5 cos (4x 7) + 4 sin (4x 7)]


3x + 8

707

2 x + 2, x < 2, x > 0
b i
2 < x < 0
2 x 2,
ii

7
6
f '(x) = 2x 2,
5
f (x) {x2 2x{
x>0
4
3
2
1
x
1 2 3
5 4 3 2 1 10
2 f '(x)  2x 2,
3 2 < x < 0
4
5
f '(x) 2x 2, x < 2

2 cos (2 x ), 0 < x < 2


5 a i
2 cos (2 x ), < x <

ii

4x
or 4x sec2 (2x2 3)
cos2 (2 x 2 3)
15 3x + 6x loge (6x)
2 x 2 sin ( x 2 ) cos ( x 2 )
16
x2
17 2 cos (2x) esin (2x)
y
18 a
14 f (x) =

y
2
1
0
1
2

8 2e2x 1

9 a 2 xe x
b x = 0
2
3x
10 3
x 2
11 a = 1
12 f (x) = 6 cos (2x), f (2) = 6
5
or 5 sec2 (5x)
13 f (x) =
cos2 (5 x )

f(x) {sin (2x){


3
P
4

P
2

3P
4

P x

f '(x) 2 cos (2x), 0 x  P


2
f '(x)  2 cos (2x), P x P
2

b R\{0, 3}

sin ( x ), 0 < x < 2 , 2 < x <


i

3
sin ( x ),
<x<

2
2
ii

P
2

f '(x)  sin (x), 0 < x < P


2

Chapter review
Short answer
y
1
1 0

2 x 2, x < 0, x > 2
19 f ( x ) =
2 x + 2, 0 < x < 2

f '(x)  sin (x), 3P < x 2P


2
f(x) {cos (x){

3P
2

2P

f '(x)  sin (x), P < x 3P


2
2

cos ( x ), 0 < x < 2


20 f ( x ) =
cos ( x ), < x < 0

2
Multiple choice
1 B
2 D
5 A
6 C
9 A
10 D
13 C
14 C
17 B
18 E
21 D
22 B
Extended response
1 a i x = 20 and x = 0
ii (28, 20) (0, 12)
iii (20, 0)
y
b

3
7
11
15
19
23

E
D
E
A
C
B

4
8
12
16
20
24

C
B
A
A
D
C

dy

dx

f '(x)

2
3
4
5

4
a 3x2 + 2
a x2 4
a 6x3 + 43 x2 3

25

28 20

b 5
b 5

b i 8 14
ii 48
6 a 18
x
7
x2 + 4

708

Answers

b 6 5

0 1012 x

dy
3x
=
( x + 20)
dx 200
e i 1.875 ii 1.5
iii 4.5
f Yes, the largest absolute value of the gradient is 4.5,
that is, the steepest section.
g 30.24 m (at x = 12)
2 a Domain f(x) = [0, ), range f(x) = [0, )
Domain g(x) = R, range g(x) = [1, )
c x = 10

b i 2 x 2 + 1

ii 4x + 1

c Domain f(g(x)) = R, range f(g(x)) = [2, )


Domain g(f(x)) = [0, ), range g(f(x)) = [1, )
2x
d i

ii 4
x2 + 1
4
e i

ii 4
5
y
3 a
7
6
5
4
3
2
1
0
3 2 1
1

1 2 3 4 5 6

b x = 5

c x = 2 and x = 5

2 x, x < 2

2< x<5
d f ( x ) = 2,
x>5
1,
e
y

3x + y = 3 + loge (3)
x
+ a2 + 32
11 a ytangent = 2ax + 1, ynormal =
2a
x
a
+
b ytangent =
, ynormal = 2 ax + a (2a + 1)
2 a 2
10

a2

c ytangent = 4 ae 4 a x + e 4 a (1 8a 2 ),

x
1 4 a2
2
+ e4 a
2 + e
2
4 ae 4 a
12 y = 8x + 2(1 loge (2))
13 b = 124
Exercise 8B Sketching curves
1 a (0, 8) a local max.
b (1, 2) a local max., (1, 2) a local min.
c (2, 8) a local min.

ynormal =

d (2, 8) a local min., ( 23 , 1 13


27 ) a local max.



e
f
g
h

(0, 0) a positive point of inflection, (1, 1) a local max.


(2, 4) a local max., (0, 0) a local min.
(3, 4) a local min.
(0, 8) a positive point of inflection

7
6
5
4
3
2
1
0
3 2 1
1

9 x + y =

i ( 12 , 6 14 ) a local max.
j (0, 5) a local min., (1, 6) a positive point of inflection
k (3, 54) a local max., (3, 54) a local min.
l (3, 16) a local max., ( 13 , 214
27 ) a local min.
1 2 3 4 5 6

2
3
4

m (0, 12) a negative point of inflection


n (0, 0) a negative point of inflection, (3, 27) a local min.
y
y
2 a
b
f(x)
(8, 0)

( 1, 2)

CHAPTER 8

Applications of differentiation

y  8 x2

(1, 2)

y
)
( 23 , 1 13
27

g(x)
0

( 2, 8)

f(x)

0
(2, 8)

y
( 2, 4)

(1, 1)
(0, 0)

y  x2 (x 3)

0 (0, 0)

g(x)

y  5 6x x2

f(x)

(0, 8)
x

Answers 8a 8b

Exercise 8A Equations of tangents and normals


1 y = 5x 4
2 At (6, 0) y + 7x + 42 = 0 and at (1, 0) y = 7x 7
3 x + y = 3
4 x + 3y + 21 = 0
5 a i y = 2x
ii x + 2y = 5
b i y = 6x + 16
ii x + 6y = 22
c i x + 4y = 4
ii 2y = 8x 15
d i y = 7x + 1
ii x + 7y + 43 = 0
e i 4y = x + 4
ii 4x + y = 18
f i 3y = x + 6
ii y + 3x = 12
g i x + y = 1
ii y = x + 3
h i y = 4x
ii x + 4y = 0
i i y = 2x 3
ii x + 2y + 1 = 0
j i y = 2x + 1
ii x + 2y = 2

k i y = 3e 1x + 4e 1
ii ex + 3y = 3e 1 e
l i 2y = x 2 + 2 loge (2) ii y + 2x = 4 + loge (2)
m i 3y = 2x + 3 loge (3) ii 3x + 2y = 2 loge (3)

3
3

ii y = x +

n i x + y = +
3
3
2
2
o i 3x + 2y = 3
ii 3y = 2x 2
2
p i y = 2 x +

ii x + 2 y = 1
2
6 a B
b C
7 y = 2x
8 y = 1

1 0

(3, 4)

Answers

709

( 12, 6 14)

y y  3x4 8x3 6x2 5


+ve point
of
inflection
(1, 6)

g(x)

(0, 5)
Minimum

y  x2 x 6

5 a ( 2 , 4) a local min., (0, 0) a local max., ( 2 , 4)


alocal min.
b (2, 0), (0, 0) and (2, 0)
c
y

(0, 0)
x

g(x)

2 x

( 2, 4)

( 2, 4)

( 3, 54)

a (2, 27) a local min., (1, 0) a positive point of inflection

6
c (1, 0), (3, 0) and (0, 3)
d
y
y  x4 6x2 8x 3

(3, 54)
y

(1, 0)

0 3

3
y  x3 4x2 3x 2

( 3, 16)

( 2, 27)
x

0
)
( 13 , 2 14
27

7 a (2, 18) a local min., (0, 6) a local max., (1 14 , 9 107


256 )
a local min.
b (0, 6) c
y
y  x4 x3 5x2 6

(0, 0)

( 1 , 14)

( 1 , 14)

y
( 1, 0)
1 0

( 14,

f(x)
4 x

(0, 0)
0

y

x3

x2

)
(3 23 , 14 22
27

( 2, 36) y

16x 16

f(x)

1 (0, 1)

16
4

1
4

)
(2 23, 14 22
27

710

Answers

g(x)

f
h(x)

4 a (2, 36) a local max. and (2 23 , 14 22


27 ) a local min.
b (4, 0), (1, 0), (4, 0) and (0, 16)
c

y
( 1, 36)

)
(2 13 , 18 14
27

y
5
( 23 , 1
)
27

1

)
256

g(x)

13

( 1, 0) (1, 0)

(2, 4)

3 a (1, 0) a local max., (2 13 , 1814


27 ) a local min.

c ( 1, 0), (0, 4)

f(x)

1 x

(3, 27)

y
(0, 0)

1
0

y
f(x)

g(x)
(0, 0)

)
(1 14 , 9107
256

( 2, 18)

8 a

h(x)

0 (0, 6)

(0, 12)

(2, 0)

y
( 123 ,

)
1422
27

h(x)

(3, 36)

No stationary points

f(x)

(1, 2)

Not a zero
gradient
0 1

ii 2 < x < 1 and x > 0


iii x < 2 and 1 < x < 0
y
16
f(x)
x

( 1, 2) 2

g(x)

h(x)

20

( 4, 4)
5

( 1, 0)
0

( 2, 0)

(1, 0)

(0, 1) 1

9 B 10 A 11 C 12 E
13 a x = 3 a local min., x = 0 a local max.
b x = 2 a local max., x = 1 a local min., x = 4 a local
max.
c x = 2 a negative point of inflection, x = 3 a local min.
d x = 5 a local min., x = 2 a positive point of inflection
e x = 3 a local max., x = 0 a local min., x = 2 a local
max.
f x = 1 a local max., x = 5 a local min.
y
14
f '(x)  2x 4
0

An example of
the answer.
17 a a = 1 and b = 8

b At x = 43 a local max. and at x = 2 a local min.


18 a a = 2 and b = 5
b (1, 4) and (0, 5)
c (1, 4) a local min., (0, 5) a local max. and (1, 4)
alocal min.
19 a (x + 2)(3x 2b + 2)
2(b 1) 4(b + 2)3
b
,
and (2, 0)
27
3
c (2, 0) is a minimum turning point.

2(b 1) 4(b + 2)3


3 ,
is a maximum turning point.
3

d y = 8(b 4)x + 32
e b = 7
20 a f (x) = 2(x + b)(3x + b 1)

1 b (2b + 1)3

b
,
and ( b, 0)
27
3

c y = 2(b + 2)(b + 5)x (b + 2)(b + 14)

f ( x) < 0 if x < 2 and f ( x) > 0 if x > 2


15 a i
y
f '(x)  x2 4x

ii x < 4 and x > 0


iii 4 < x < 0
y
b i
g'(x)  3x2 4x 7

ii x < 2 13 and x > 1


iii 2 13 < x < 1
c i
y
h'(x)  4x3 12x2 8x

2 1

dV
= h 3 2h 2
dh

dh

= 3t 2 2e t
dt

dM
= 1 + log e (3k )
dk

dL
4
= 2t 2 + 2 sin ( t )
dt
t

2 D
3 C
4 a 8 workers
b $1112
5 20 metres
6 x = 1 hour for max. and x = 7 hours for min.
7 a x = 5C for min. and x = 15C for max.
b Minimum is N = 291 rabbits (round down) and
maximum is N = 1625 rabbits.
3
8 a Minimum v = 0.8 cm/s when t =
4

b Maximum v = 0.8 cm/s when t =


4
9 a i 12 cm
ii 132 cm
t

b R = 6 + 2 cos 4
c Max. =

+12
12
, min. =
2
2

Answers

Answers 8c 8c

2 13

3 2 1
as b > 0
2
Exercise 8C Maximum and minimum problems
when the function is known
dC
dP
5
= 5 4x
1 a
b
= 3n 2 + 4 n
dx
dn
2 n

d b =

711

10 a
b
c
11 a
b
12 a
13 a
b

1 January 1998
941 cheetahs
i 824
ii 507 (round down)
232 elephants
Approx. 1 January 2009
75
b $50
c $3750
90 kg
Approx. 77.3 kg after 3 months

Exercise 8D Maximum and minimum problems


when the function is unknown
1 Both numbers are 5.
2 2 and 6
3 b A = 60x x2
c Length and width are each 30 cm.
d Max. area is 900 cm2.
4 10 000 m2
40
5 Radius =
cm (or 12.73 cm), length = 40 cm,

circumference = 80 cm
6 a L = 4.5 3x
b V = 9x2 6x3
c Edges are 1 m, 1.5 m and 2 m, for max.
d V = 3 m3
7 b V = 50x 12 x3

c 192 cm3

8 6301.1 cm2
9 Approx. 6564 cm3
10 x = 1.5 km
800
hours
v
c 223.61 km/h
12 Approx. 13.86 cm
13 1239 cm3

b C = 40 000 v 1 + 0.8 v

11 a

c t = 60 min

712

Answers

60

90

3t

1
(3t 2 + 4) 2

b a =

3t 2 + 4

3t
3t 2 + 4
9t 2

( 3t 2 + 4 )3

12

(3t 2 + 4) 2

c v = 1.5, a = 163
8


9

a
b
c
a
b

c
d
10 a
11 a
12 a

v = 3t2 24t + 36
t = 6 x = 0 and t = 2 x = 32
a = 12 and 12
3000

371.4 people/day
dN 4000
=
dt
8t + 1

97.56 people/day
D
b A
B
b C
dH
t
=
sin
dt
6
12

c E

3
m/h ii
m/h iii
m/h
12
6
12
c Min. H = 8 m at t = 12 hours and max. H = 12 m at
t = 24 hours
13 a i $2000
ii $2378.22
b 126.07 $/year
c 68.65 $/year
14 0.2 mL/h
15 a 150 cents
b (8, 13)
b i

2

d 15 cents/month
3
e 175 cents
f t = 13
16 a 7 m/s
b 0.4 m/s2
c (0, 6) and (9, 16)
d 0.2 m/s2
Exercise 8F Related rates
5
cm/s
1
4624
3
2
223
3 0.5 cm/s
4 320 cm3/min
1
5 45
cm/s
6 60 cm2/s
7 B
8 0.26 m/min

4 5 units
1
15 4.06 units
Exercise 8E Rates of change
dV
dS
dA

b

c
1 a
dr
dh
dt
dC
dI
dv
d

e

f
dx
dp
dt
2 a dA = 2 r
b 20 m2/m
dr
b 100 cm3/cm
3 a dV = 4 r 2
dr
4 6 cm2/cm
5 a i 10 m/s
ii 10 m/s
b When t = 1 the projectile is rising but when t = 3 it is
falling.
dv
1
6 a
=
(90t2 t3)
dt 100
b V

7 a v =

9 2 3 m/s
1
m/min
10
24
h
11 a r =

3

b V =

h3

9

Exercise 8G Linear approximation


1 a 1.03
b 0.998
c 4.9
d 1.0005
2 2 cm3 or about 6 cm3
3 4%
4 a 43.74 cm2
b 0.44 cm2

75
cm
/s
cm/s
512

7


8




5 E
k
; 12.5% decrease in volume
6
16
7 h

1
4

8 5% increase in perimeter, 10% increase in area


Chapter review
Short answer
1 (3, 86) a local max. and (2, 39) a local min.
2
y
( 2, 4)

(0, 0) 0

b
c
d
10 a

2 x

y = x2 (4 x2)

3
4
5
6
7

2x + y = 6
y = 3x 6 + 3 loge (4)
8x + 6y + 31 = 0 and 8x + 6y = 31
1430 bees
a x = 12 units
b 16 sq. units

2
3

cm/min

9 P(4, 2); 2 5
10 18
Multiple choice
1 C
2
5 D
6
9 C
10
13 A
14
17 B
18

3
7
11
15
19

E
D
C
D
D

A
A
B
A
A

4
8
12
16
20

A is (0.5, 1) and B is (0.5, 1)


y = 4x 1
c (0, 1)
1 cm
e 1.125 cm
7 weeks
i 98.25 rabbits/week ii 44.15 rabbits/week
P0 = 267 (round down)
782 rabbits
i 33 rabbits/week
ii 44 rabbits/week
39 weeks

9 a y = 14 400 h 2

( 2, 4)

a
b
d
a
b
c
d
e
f

D
E
B
E
D

5h
dy
=
cm/s
14 400 h 2
dt
i 4.47 cm/s
ii 7.54 cm/s
5.67 cm/s
f (x) = (x + 2a)(3x + 2a 6)
2(a 3) 4(2a + 3)3

and ( 2a, 0)
3
27

4 500
c (2, 0) is a local minimum turning point. ,
3 27
is a local maximum turning point. Therefore (2a, 0)
is a local minimum turning point and
2(a 3) 4(a + 3)3
,

3
27

is a local maximum turning point.


d y = 4(a 6)(a 1)x + 4(a 1)(3a + 7)
3a + 7
e
,0 , a 1
a 6
dy
= 8(4 a)
dx
a +12

c x =
8

b y = 8(4 a)x (4 a)( a + 12)

11 a

d a = 8

e y = 96x 48

Extended response
1 a 10 2 r 2 = h b

1
r2
3

100 r 2 c

2000 3 3
cm
27

Integration
Exercise 9A Antidifferentiation
1 a 12 x2 + c
b 15 x5 + c
d

1 6
2x

+ c

2
+ c
x3

1 4
8x

+ c

m 35 x 3 + c
5
+ c
p
2x2

s 16 x + c

2 a
+ 5x + c
b x3 + 2x2 10x + c
x2

+ c

1 8
8x

2 5
5x

+c
+c

+c

4
3

x 2 + c

1 5
25 x

1
+ c
9x3

2
3

7
16 4
7 x
9

x
12
x

+ c

+ c

7
4

x2 + c
4

x7 + c
2
r 5 + c
x
o

+c

c 2x5 + 32 x4 + 2x + c
d

2 6
3x

1 4
4x

+ 14 x4 2x3 + x2 + c

Answers 8D 9a

2 a 3000
b Approx. 10.11 bacteria/min
3 a 1.4 m
b 3.4 m
c 0.4 m/s
2 3
d
m/s
15
4 a 300 m
b 40 m/min
c 700 m
d 20 m/min
100
5 a A = 2r 2 +

b r = 2 cm
r
2
c A = 75.1 cm
d $3005
27000
6 a l =
w2
108000
b A = 2w2 +
w
c w = l = 30 cm (that is, a cube with side length of
30 cm)
54 000
d L = 10w +
w2
e w = 22.10 cm; l = 55.26 cm
1080
27000
f C = 0.02w2 +
+ 5w +
w
w2
g w = 23.74 cm; l = 47.91 cm
h 223 cents

CHAPTER 9

+ 12x 13 x3 + c

Answers

713

1 x3
3

2x2 21x + c

5 3
3x

+ 5x2 5x + c

1 4
4x

73 x3 + 2x2 28x + c

i 14 x4 + x3 2x2 + c
3 B
1
3

(x + 3)3 + c

1
5

(2x + 1)5 + c

1
5
30 (6x + 5) + c
1
4
4 (4 x) + c
4
5
15 (8 3x) + c
1
1
2 (2x + 3) + c
1
3
2 (4x 7) + c
2
1
10 (6 5x) + c

i
k
m
o

f
h
j
l
n
p

6 C
7 a 3 loge |x| + c

1
4
4 (x 5) + c
1
6
9 (3x 4) +

loge |x| + c

f loge |x + 3| + c

8
5 loge |5x
5
2 loge |3

m
o

4
3

+ 6| + c

+ 2x| + c

3
2 loge |2x 5| + c
2
7 loge |6 + 7x| + c
3
11 loge |6 11x| +

(0, 0)

loge |3x + 2| + c

1 5
5x

+ x2 + loge |x| + c b
1

c 3x + 2 loge |x| + x + c d
e

1
2

3
8

loge |6 10x| + c f

(4x + 1) 2 + c h

2
i loge |x| + 2x 1 + 15
2 x + c j
2

k 2 x 2 x2 + 92 x 3 + c l
5

1
18

loge |2x + 1| + c

1
3

(2x 5)6 + c

1 2
2x

+ 4x + 8 loge |x| + c
2 loge |3 x| + c
+ 14 x4 + c

m 25 x 2 + 43 x 2 2 x 2 + c n 5x 2 + x 1 + x2 + c
11 a f(x) = 2x2 + x + 2 b f(x) = 5x x2 5
3
1
d f(x) = 12 x2 + 23 x 2 103
c f(x) = 3x + 2
x

1 2x
2e
x

e + c

e e5x + c

1 e6x + c
12
1 6x
2 e + c
x
3e 3 + c

x
6e 2

g
i

7 4x
4e

+c

2 3x
9e

+c

+c

x
0.4 e 4
x

+c

n 9e 3 + c
e x + e x
+c
p
2

3 13 e3x 32 e2x + 3ex x

1
7

2 cos ( 2x)
2
9 cos (6x)

Answers

+c

2 x + 2ex + 12 e2x

h f(x) = (1 2x) + 2
i f(x) = loge |x + 5| + 2
j f(x) = 4 loge |7 2x| + 7

1 3x
3e

j 4e

o ex ex + c

f f(x) = 2 x x 2 6
(x + 4)4 + 1

2x

+ c

1
3

1
4

b 14 e4x + c

+ c

6 a

714

1 a

4 14 x4 x3 + 2e3x
5 a D

g f(x) =

(0, 0) (1.44, 0)
x
1 2

Exercise 9B Integration of e x , sin (x) and cos (x)

e f(x) = 43 x 3 x3 + 50x

f (x)

2 1

3
2

3
2 2
3x

(1, 1)
(0, 0)

(3x + 1)6 + c

1 2
4x

f (x)

( 2.77, 0)

q 23 loge |4 3x| + c
r 4 loge |5 2x| + c
8 A 9 2x + 7 loge |x| + c
10 a

f (x)

loge |x| + c

h 2 loge |x + 4| + c

loge |5 x| + c

1
2

(1, 3)

5x) 3 + c

4
7

f (x)

8) 5 + c

loge |x + 5| + c

+ 5) 2 + c

loge |x| + c

g 3 loge |x + 3| + c

f (x)

9x)11 + c

6
5

b 8 loge |x| + c
7
3

1
3
4 (4x 1) + c
1
5
5 (7 x) + c
1
33 (8
1
12 (6x
1
15 (3x
2
3 (7

f (x)

4 E

5 a

12 a k = 8
b y = 41
13 a k = 1
2
b g( x) = 4
loge |x|
x
c 3 loge (4)
f (x)
b
14 a

b C

cos (3x) + c

sin (7x) + c

1
4

1
6 sin (2x) +
1
sin (3x)
3

cos (4x) + c
c

+ c

+c

+ c

h 2 sin (4x) + c

j 2 sin (x) + c
l 2 sin x + c
2

i 2 cos (3x) + c
k 3 cos x + c
3
x
m 12 cos + c
4

n 10 cos x + c
5
x
p 12 sin
+c
2

o 16 sin x + c
4
q
s
u

6
4
5

2x
cos + c
3

3x
r 8 sin + c
4

cos 5 x + c
2

cos ( x) + c

sin 7 x + c
4

12
7

x
6
sin + c
2

sin x + c
x
cos 4x + c

4

3
7 a sin (x) cos (x) + c
b 12 cos (2x) sin (x) + c

1
4

1
2

sin (4x) cos (2x) + c

g 6 sin x cos x + c
2
3

1 4x
4e

cos (2x) +

1 4
4x

1 4
1
4x 2
1 3
3x + 2

+c

sin (2x) + e + c
x

c
d
12 a
b
c

x 2 + x2 +

+ 1)5 + c

d i

4x + 3

+c

ii

2x 5 + c

ii

4x + 3 + c

e i 4(2x + 3)(x2 + 3x 7)3 ii

2x

( x 2 1)2

ii

2 a A
3 E
4 a i 4e4x 5

b B

6 5x

ii

1 (x2
4

+ 3x 7)4 + c

2
+c
x2 1

1 4x 5
2e

+c

6 5x

ii 2e

1 x2
2e

+c

+c

c i 2 xe x

ii

d i (1 2 x )e x x
5 a i cos (x 5)
b i 3 cos (3x + 2)

ii e x x + c
ii sin (x 5) + c
ii 2 sin (3x + 2) + c

c i 4 sin (4x 7)

ii

i
6 a
b
c
d
e
7 a
b
8 a
9 a
10 a
11 a

i 4 sin (3 4x)
5

i
5x + 2
2x

i
x2 + 3
2x 4
i

x2 4 x
i 3 cos (x) x sin (x)
x cos ( x ) sin ( x )

i
x2
i [ex (sin (x) + cos (x)]
i sin (x) + x cos (x)
i ex + xex

1
4
1
2

cos (4x 7) + c

1
2

cos (3 4x) + c

ii
cos (6x 3) + c
ii 2 sin (2 5x) + c
ii

ii 4 loge (5x + 2) + c
ii 6 loge (x2 + 3) + c
ii

1
2

loge (x2 4x) + c

ii sin (x) x cos (x)


2 sin ( x )
ii
x
ii 3ex sin (x)
ii x sin (x) + cos (x)
ii xex ex
x6
i 12x5(1 3x)(2 3x)5 ii
(2 3x)6 + c
2
3x 2 + 2
i

ii 2 x 3 + 2 x + c
2 x3 + 2x
1

b 3x + loge |x 1| + c
3+
x 1
2

b 5x 2 loge |x + 2| + c
5
x2
5
4+

b 4x + 25 loge |2x 3| + c
2x 3
4
3+

b 3x 2 loge |3 2x| + c
3 2x

Answers

Answers 9b 9c

1 a
1
b

x
6
cos + 5x + c
3

f(x) = 4 + sin (x)


f(x) = 1 2 cos (2x)
x
f(x) = 3 2 + 12 sin
4
x
x
f(x) = 4 sin + 2 cos 4
4
2
k = 1
6
x
y=
cos x + 7
6

6
y=1+

2
3

1 2
2 (x

1
(3x 1)5 + c
c 3 cos x + 2e 2 15
3
d 13 loge |3x 2| + 14 e4x + 5 sin x + c
5

ii

x
x
e 6 cos 6 sin + 5e
2
3

b i 10x(x2 + 1)4
1

c i
2x 5

loge |2x + 3| + 12 e2x + c

2)8 + c

d i 6 sin (6x 3)
e i 5 cos (2 5x)

9 x3 sin (2x) + 2e3x


10 a

1
2 (3x

+ 12 cos (8x) + 7x + c

1
2

ii

2 cos (x) + c

1 6x
2e

1 a i 24(3x 2)7

e sin (4x) + 16 cos (2x) + c

Exercise 9C Integration by recognition

b i 5e

x
d 2 cos 12 sin (2x) + c
2
5 2
2x

c f = 2.02
6

13 a k = 4
b f(x) = 2 sin (2x) 4ex + 3

715

dy sin ( x )
=
= tan ( x )
dx
cos ( x )
b loge |cos (x)| + c
cos ( x )
13
sin ( x )

12 a

14

1
6

e 5 loge (4) + 2e2 2e 2


f

15 1 sin (ax + b)
a
1
16
cos (ax + b)
a
1
17 e ax + b
a
1
18 a
cos (3 x + 1) + c
3
1
sin (1 4 x) + c
b
4

p (2 2 2 + 4) 23.4

c 33 12

6 a 26

b 41

7 a 28

b e 1 + 1 + e c loge (24) d 6

e 23 13
8 a 10
9 a 6.875
10 10.5

f 7.25
b 7
b 5.375

g 100
c 8.5
c 6.125

Exercise 9E The fundamental theorem of


integral calculus

2 a

1 8
4 (e
2

20 14

716

Answers

(e e )

c 12

d 40

1 ( x3 9 x 2 + 20 x) dx f 2 ( x3 4 x 2 4 x) dx

1 e x dx

02 2 sin (2 x) dx

h 74
k 6313.2

i 38
l 5949

0 (4 x) dx

3 3x 2 dx

4 a 8

5 a

1 13

b 8

g e e
y

7
3

1 e 2 x dx

0 2 cos 3 dx

d 26


1
2

e 22

(e e ) i 2

y  4x

j 3

12

1
2

f 2 loge (5)
3

1 x 2 dx

f 56

4
3

1
3

e 8

b 3(1 e

c 3

1 2 x dx

q 35 loge (5) or 0.966


s 3.65

c 2(e e2)

4e 4

3 a

5 13

1)

2
3

Exercise 9F Signed areas


The answers to all area questions are in square units.
1 a 8
b 8
2 a 4.5
b 4.5

m 1360
n 60
49
o 24.8 3 loge (4) or 20.64
p 37 13
r 2

g 3
h
6 k = 1 or 4
7 k = 2
8 k = 3
9 a = 2 loge (3)
10 k =
6
11 a 16
b 8
12 D
13 A

Exercise 9D Approximating areas enclosed by


functions
All answers for areas in questions 1 to 15 are in square units.
1 10
2 a 8
b 42
3 a A
b A
c B
4 a 30
b 22
5 a 32
b 32
c 39
d 16
e 29
f 12
g 26

4
3

6
x

sin
+ 5 + c
2

g 12 23
j 1

n 0

d 10

f esin (x) + c

m 9 3 9

5 a 9

1 x + 3
e
+c

3
x

cos 2 + + c
d

1 a

h 0
j 6
l 0

q 14 23 + 2 cos (9c) 2 cos (3c)


r 6 + log () 6 sin (0.5c)
e
3 18
4 a B
b A

1
3
1
3

(e 2 + e2 e 6 e6)

g 1
i 20 10 2
k 0

loge |3x2 4|

1
2

x
0 1 2
y6 x

y  x2

3)

(e12 + 15 e6)

01
y  4 x2

0 1

y  loge ( 2x )
2

y  3 sin (2x)

3
x

0
3

6 a 2

c 5 13

b 1

g e e

i 1
7 a E

P
2

d
1

f 2 23

e 4

4
3
1
2

8 a

0 f ( x) dx 2 f ( x) dx

1 g( x) dx 3 g( x) dx

1 h( x) dx 3 h( x) dx

f 49 13

j 4
b A

5 8 + e3 e 1
1
Exercise 9G Further areas
All answers for areas are exact and in square units unless
stated otherwise.
1 a 4 12
b 36
64 2
1
c
d 21 13
e 312
2 a 85 13 b 57 16
3

y  2ex

y x

3 A
6 a i

1 0

(e 1 e 2)
b i

9 a B b D
y
10

c E

d i

y  x3 4x

ii

a 2 23

ii 1 e

1
c 312

y  1 3 cos (2x)

a
13

632

y  2 sin (x)

3P P

3
+
4 2

0
1

0 P P P P

Answers 9d 9g

3 1

ii

2
1

4
1
0 P

g i

y  e x
0

5
12

(1 e 4)

2
y  x3 x2 2x

1
2

y  e2x

e i

10 23

12

ii 5 43

a 5 13

2 0

1
2

1
y 
x2

2 0 1 2

b
c 16
y
11

ii

12

y  x2 4

0
2

ii 2 loge (3)

y  2x

c i

0 1

2 g( x) dx 2 g( x) dx + 3 g( x) dx

5 C

ii 6

y  3 3x2

4 f ( x) dx 5 f ( x) dx

y
2

4 D

y
3

h 40 12

g 8

h i
b

+
4

y  cos ( x)
2
2P

3
2

ii 4 2

0 P P

y x 1
1

4 2
Exact area is 2 3 1
or approx. 0.578
3
14 2 loge (2)

i
P

P P
2

y  sin (3x)
0 P

ii

2
3

Answers

717

y
yx x
8
6
4
2
1
x
0 1234

ii 12 45

2 a 16

d i x = 1

ii 1 85

()
3
4

(or approx. 2.15)

ii 2 2

h 43
3 C 4 D 5 E
y
6 a i x = 0 and 4
ii
2

ii

b i x = 3 and 1

ii

iii 10 23
y  2x
0 1

c i x = 1 and 1

ii

x
y  3 x2

y  x2 1

y  1 x2

1
y 
(x 3)2

d i x = 2 and 2

ii

11 + e
13 x-intercept is loge (3) (or 0.55), area is
2
14 No x-intercepts, area is 6 + 1.
15 a 1 + loge (x)
b x loge (x) x
c 4 loge (4) 3 (or approx. 2.55)
2x
16 a 2
x +2
1
2

e i x = 0 and 1

0 (4 x 2 3x) dx

0
2

i x = 0 and 1

ii

Answers

1 ( x 1) dx

2 (8 2 x 2 ) dx

dx h

b i x = 0, 1 and 2

ii

y  x2
y x

1
1

(9 x 2 e x ) dx
(1

x2 )

dx

iii

1
3

iii

1
2

iii

1
2

1
y

y  x3
yx
0

y  3x2

1
3

y  x3 2x

(3 x x 3 ) dx f

(x + ex )

2
Exercise 9H Areas between two curves
All answers are in square units unless stated otherwise.

iii

y  1 x2

7 a i x = 1, 0 and 1 ii

20 a loge (4) b

y y  (x 1)2

(3e 4 + 1)
(or approx. 82.40)
2
9 2 4 (or approx. 2.28)
1

iii 21 13

18

0 ( x x 2 ) dx

c loge (1.5) (or approx. 0.41)


17 a 2 23
b 5 13

ii

loge (x2 + 2)

1 a

y  x2 4

y  4 x2

4
9

1
2

1
4

12 a x = 2 and y = 0 b

iii 2 23

0 1

x3

1 0 1

1
4

iii 10 23

2
5

3 3
9
2

718

y  4x

(e6 e3)

11 a

e 2 14

ii e3 + 1 3e (or approx. 12.93)

10

d 21 13

yx

e i No x-intercepts ii 2e 2e 1 (or approx. 4.7)


f i x = 2 and 2
ii 139.2
1
3

c 2 16

g 1 12 + e2 e (or approx. 6.17)


ii 1 4 loge

1
2
1

f 17 13 e + e (approx. 14.98)

7 a i x = 2 and 2

b i x =
4
c i x = 1

g i x = 2

8 a

2
3

1
d 2312

b 2 2
e 1 18

c 4

f e e 1 2 (or approx. 1.09)


2
(or approx. 3.23)
g 2 +
8

h 2e 2e 2 (or approx. 5.17)


3
9 e e 4 (or approx. 13.37)
10 3 12

11

3
12
(or approx. 0.45)
12
14 a x = 4 and 4
c 9 13

6 a 4 L
b V = 5t +

c 200 L
d 4.79 L/min
e 5 h 20 min
7 a dv
dt

1
2

11
10
9

13 3 loge (2) 2
b x = 3 and 3

5 2 16 m2
1
16 a x = 20 and 20
c 93 13 m2

40
t
sin
40

12

Hours

b 4 h
c i 60 L
ii 50.6 L
8 a 56 250 m3
b 5
9 1.26 m

11 a y = 83 x2 + 6 b 38 m2

b 40 m
d 840 m3

300
) m2

60 000
b (30 000
) m3

17 a (150

12 b 3 loge (2) 2

Exercise 9I Average value of a function

Chapter review
Short answer
1 f(x) = x3 x2 3
2 a k = 0
4
4
x
b
sin + 1
4

3(2 3 )
c 149

1
2 a 12
[50 loge (5) 8 loge (2) 21]
3 log e (2)
b

c 14 (3e4 + 1)
1 a 8

302 2
45

3 a

1
2

1
2

e6 1
6

(e2 1)
e 2 1
log e
2

dt

64

8 16

Seconds

d 466 23 m
e The distance travelled in 10 seconds.

dy
= 2(x + 1) cos (x2 + 2x) and antiderivative is
dx
12 sin (x2 + 2x)
4 a 2
b 2ex 2x + 1
5 (5.5 e2 + e) sq. units
3
6 a 1 49
b
2
7 k = 12 or 2
3

8 a

1
y 
x 2

b loge (4)
9 166 23 sq. units
2

+ 2 sq. units (or approx. 3.23)


10
8

1 17 sq. units
1
12 a y = 2x + 2
b (2 e 2) sq. units
Multiple choice
1 B
2
5 D
6
9 D
10
13 E
14
17 D
18
21 E
22
25 B

E
E
A
B
A
D

3
7
11
15
19
23

C
C
C
E
B
B

4
8
12
16
20
24

A
B
C
C
C
A

Answers

Answers 9h 9j

4 C
5 E
Exercise 9J Further applications of integration

( 2 x )3
1 f ( x ) =
+4
3

2 1 +
12
3 a 0 m
b y = 0.01(x + 1)3 + 0.03x + 0.01 or y = 0.01x3 0.03x2
c 54 cm
4 a $34.56
b C = 40n 200e0.01n + 200
c $3656.34
d i $36.56 ii $30.66
5 a i 0 m/s ii 48 m/s
b i t = 8
ii 64 m/s
c dx (m/s)

c 1

c 380 m3

719

Extended response
1 a $0
b C = 25t3 + 25t2 + 800t
c $7750
d $4450
e $1050
t
2 a T = 10 5 sin b No
12
c 15C at 6 pm
d 5C at 6 am
e i 7.5C ii Approx. 13.5C
f 4 pm

3 a (2, e)
b y = (2e 1)x + e + 4e 1
1
c (0, e + 4e )
d (0, 1)

e (2 + 4e 1) sq. units
4 a 1 + loge (x)
b x loge (x) x
c 3 m

d (e e 2) m2 (or approx. 2.58 m2)

e 20(e e 2) m3 (or approx. 51.66 m3)

5 a 5 2 C/cm
b T = 180 40 x
c i 82.02 C
ii 66.86 C

Exam practice 3
Short answer
1 a
x
f(x)
1
3.4
2
2.7
3
1.5
4
0.7
5
0.3
Total

Area
3.4
2.7
1.5
0.7
8.3

dy
= ( x + 1)e x + e x = xe x
dx
c -20e-1(2e-4 - 1)
ex
2 a y dx = x 4 2 sin ( x ) +
2
2x x
3
b y dx =
2
3
2x
3 a The coordinates of P are ( 3 , 3 ) and (
2
4 a =

7
2
2

dx
= 0.12 x
dt
6 a y = 4(x - 1)
1
b 3

b V = r 2 h = 250 h =

720

Answers

y  2r
P 2
y  500
r
1 2 3 4 5 6 7 8 9 10 r

d r 3.4
125

fi r 3.414 cm
iiDefine g(x) = the gradient function on a CAS calculator
and determine sign of gradient close to 3.414 (for
example, use r = 3.4 and r = 3.5). The gradient sequence
in the vicinity of r = 3.414 is negativezeropositive,
which corresponds to a local minimum.
iii 220 cm2
g r 2.710 cm
h 1.47
e r =

Discrete random variables


Exercise 10A Probability revision
1 a
e

1
4

1
36
1
2

1
6
5
12

ii

5
12

iii

1
3

2
3

iii

1
4

iv 1

b
1
4

2 a i

ii

3 a 18

b 83

3
8

1
8

1
4

3
8

5
8

17
40

3
20

1
2

4 a 83

5 0.7
6 0.1
7 a 0.2

b 0.8

8 a

2
5
3
8
1
3

13
40

1
2

c No
11

d Yes

Yes

Column 1 Column 2 Column 3


B

Row 1

0.22

0.10

0.32

Row 2

0.08

0.60

0.68

0.30

0.70

Row 3

500 r
500
= 2 r 2 +
r
r2

5
6

+ 125 + 13 = 1

12 a

250
,
r2

c i 23
e

3 3
, ).
2 2

1
18
7
12

10 No

Multiple choice
1 A
2 E
3 B
4 B
5 C
Extended response
a S = 2r2 + 2rh

y  2Pr2 500
r

e
9 a

so S = 2 r 2 +

y
700
600
500
400
300
200
100

CHAPTER 10

b i Pr(A B) represents an unfit smoker,


Pr(A B) = 0.22.
ii Pr(A B) represents a fit non-smoker,
Pr(A B) = 0.60.
c i 0.10
ii 0.08
iii 0.32
iv 0.70

1 3 a
e
14 a
15 a
16 a

2
3

1
2

Yes
0.7
0.105
0.7

b 0.4
b 0.39
b 0.3

17 a

25
64

9
64

1
3

c 0.5
c 0.495
c 27
35

15
32

2
5

1
15

25 a

25
72

19
66

26 A
28 a

17
32

8
15

1
4

7
15

2
9

22
31

Exercise 10B Discrete


1 a Discrete
c Continuous
e Continuous
g Continuous
2 a
x
0
1
4

Pr(X = x)
b

random variables
b Continuous
d Discrete
f Discrete
h Discrete
1

1
2

1
4

3
4
1
2
1
4

Pr(X = x)

1
13

1
13

7
13

2
5

11 a

5
6

1
8

5
18

5
9

3
8

Pr(X  x)
0.5
0.4
0.3
0.2
0.1

1
8

7
8

Pr(X  x)
0.5
0.4
0.3
0.2
0.1

Pr(X  x)
0.4
0.3
0.2
0.1
2 4 6 8 10 x

Pr(X  x)
0.4
0.3
0.2
0.1

1
18

1
12

7
36

Pr(X = x)

1
5

1
5

1
5

1
5

1
5

3
5

16

25

36

Pr(X = x)

1
6

1
6

1
6

1
6

1
6

1
6

1
2

Pr(X = x)

1
6

1
6

1
6

1
3

1
6

2
5

Pr(X = x)

1
45

1
9

1
5

13
45

17
45

14 a

1
36

29
36

3
160

1
10

9
32

3
5

0.1715

0.4115

0.3292

0.0878

x
Pr(X = x)

b 0.1060
5 a (1, 1), (2, 1), (3, 1), (4, 1), (5, 1), (1, 2), (2, 2), (3, 2),
1
(4, 2), (5, 2), (1, 3), (2, 3), (3, 3), (4, 3), (5, 3), (1, 4),
(2, 4), (3, 4), (4, 4), (5, 4), (1, 5), (2, 5), (3, 5), (4, 5),
(5, 5)
3
5

ii Pr(B) =

2
5

iii Pr(C) = 12
25

iv Pr(D) = 19
25

c i Pr(A | B) = 35

ii Pr(B | C) =

1
2

iii Pr(C | D) = 10
19
16 a

1 2 3 4 x

1
9

5
36

b i Pr(A) =

5 10 15 20 x

1
6

5
36

Pr(X = x)

4
25

12
25

9
25

4
25

+ 12
25 +

9
25

Answers 10a 10B

3
8

1
9

1 1 1
36 18 12

Pr(X = x)

Pr(X = x)

1
13

13 x
0

4 a

12

3 a HHH, HHT, HTH, HTT, THH, THT, TTH, TTT


b x = 0, 1, 2, 3
c
x
0
1
2
3

b 65

Pr(X  x)

d 0.1

3
13

35
36

10 a

3 a 0.1035 (4 d.p.)
3
b 0.3929 (4 d.p.)
c 0.2746 (4 d.p.)

1
11

b 16

9 a

31

Pr(X = x)

6
7

b 0.64

b 0.6

b k = 5 was discarded as it would result in negative


probabilities.
8 a x
2 3 4 5 6 7 8 9 10 11 12

27 B

55
64

29 a 0.3277
0 0.5
3
32

7 a

7
33

8 C 19 D 20 D 21 B 22
1
23 a 0.4
b 0.5
c 0.74
d 0.26
3

g

h 12
e 0.76
f 18
25
5
15
7
i 20
24 a

5 a, d
6 a 0.2

d No

=1

Answers

721

17 a 0.54
e

31
55

b 0.55

c 0.54

f 4

g 4

d 0.24

18 a 2.24
b 2.96
9 $1452
1
20 a i 3.5
b i 9
c i 0.5
Exercise 10D Measures
random distributions
1 2.3, 0.81

8 C
1
19 C
20 E
21 D
22 B
23 D
24 a x = 0, 1, 2, 3, 4
b

Pr(X = x)

1
16

1
4

3
8

1
4

1
16

Exercise 10C Measures of centre of discrete


random distributions
1 7.11
2

1
4 a = 18
, E(X) = 5 13

5 b = 0.15, E(X) = 2.39


6 k = 0.05, E(X) = 13
7 a x
1

1
6

1
6

1
6

1
6

1
6

1
6

Pr(X = x)

b $0.02
b 12.9456

c 29.1276

b 79.1091

c 878.99

b 5.41

c 2.33

15 a

2 3 4 5 6 7 8 9 10 11 12

Pr(X = x)

1 1 1
36 18 12

1
9

5
36

1
6

5
36

1
12

1
9

1
18

1
36

Pr(X = x)

1
16

1
4

3
8

1
4

1
16

x
Pr(X = x)

b 1 45
2 13

Answers

9 13

8
125

36
125

54
125

27
125

c 2
c

5 23

1
615

1.5

0.5

0.3

0.15

0.05

b $1.60
c $0.51
14 0.95

b 2
0 a = 0.15, b = 0.05
1
11 a = 0.1, b = 0.2
12 a $3.75
b No, because although his expected gain is $3.75 per
game, he must pay $5 to play each game. Therefore
his loss per game will be $1.25.
c No, because the expected gain is not equal to the
initial cost of the game.
13 a $0
b No, because even though her expected gain (that is,
her average gain per game in the long run) is $0, she
could still lose $40 in one single game.
c Yes, because E(X) = 0.
14 11
15 17

722

3 a $1.42
4 4.96
5 a 3.2364
d 80.91
6 a 8.7899
d 219.7475
7 10
8 5

Pr(Y = y)

b 7

17 a

55
b 2 64

13 a

b 3 12

16 a

ii 5
ii 8
ii 2
of variability of discrete

2 a 5 83

10 a 8.3
11 14
12 11.36

3 a = 0.09, E(X) = 5.42

9 a

d 18.48

9 2 13 , 1.0556, 1.0274

13
18

8 a

c 7.16

Pr(X = x)

1
35

9
35

5
7

b 4 13
25
16 a

c 1.0443

34
35

Pr(X = x)

3
50

4
25

3
10

12
25

b 4 15
21
( 0.9165)
5
d Pr(2.3669 X 6.0330) = Pr(3 X 6)

= 0.94
17 a 0.2
b 1
c 2.2
d 1.36
e 1.1662
f 1
18 a 0 X 8
b 4 X 16
c 21 X 49
d 11.2 X 32
c

f 12 65 X 22 16

e 8.3 X 11.1
19 a 7
20 a

c 0.94

Pr(X = x)

1
4

3
8

1
4

1
8

b 2.25
1 A
2
22 E
23 C

b 5.83

c 0.97

1
6

CHAPTER 11

Chapter review
Short answer
1 a

1
12

2 a 0.15
3 a

2
5

9
24

1
8

iv

5
8

The binomial distribution

b 0.25
b

4
15

4 a i 0.3
ii 0.5
b 2.7 min c 10
5 a Teacher to check
6 a

5
12

iii 0.5

(0.625)

1
10

Pr(X = x)

1
30

2
15

3
10

8
15

Pr(X = x)

7
36

7
36

7
36

7
36

7
36

1
36

1
b 412

7
12

8 No, it is not a fair game.


9 No
10 a i 4
ii 20
b

1
243

1 a
1
b
12 a
13 7
14 a
15 a
16 a

i 4
i 7
$8.10

ii 4
ii 4

40 cents
10 cents
4
x
Pr(X = x)

b $510

c 2

b $2000
b $50

c 20%
c 10%

1
20

3
20

3
10

1
2

c 3 14
Multiple choice
1 D
2
4 D
5
7 D
8
10 B
11
3 D
14
16 C
17
19 B

Pr(X = x) 0.16807 0.36015 0.3087 0.1323 0.02835 0.00243

4
625

7 a

1
16

8
27

1
16

65
81
3
8

8 a 0.096
b 0.0064 c 0.008
d 0.2
9 a 0.0518 b 0.2592
10 a 0.2627 b 0.0084 c 0.2568 d 0.2568
11 0.0381
12 0.0924
13 a 0.1023 b 0.2001
14 0.2281
15 0.0653
16 a 0.0528 b 0.6676
17 a 0.2734 b 0.2965 c 0.1678
18 C
19 E
20 B
21 D
22 C
23 a i 0.0102
ii 0.9898
b 2
24 18 tickets
25 22
26 a 5 645 726
b 352 858
c $1 446 717.80
27 a 2
b Positively skewed
Pr(X  x)
28 a i
0.2

C
A
B
A
C
B

3
6
9
12
15
18

0.1

E
D
C
A
E
D

0.0

0 1 2 3 4 5 6 7 8 910 x

ii Positively skewed

b i

Pr(X  x)
0.3
0.2

b $2.14
d 70 cents
Loss of 54 cents
Loss of 54 cents
Loss of 54 cents
b

0.1
0.0

iii No
iii No
iii No

0 1 2 3 4 5 6 7 8 910 x

ii Symmetrical or normally distributed

c i

Pr(X  x)
0.3

19
32

0.2

d 2.5625 packages

e Var(X) = 2.3086, SD(X) = 1.5194


f 1
g $126.56
h 10
19

Answers 10c 11a

0.3

Extended response
1 a $14.70
c 61.25 cents
e $2.10
2 a i $720
ii
b i $40
ii
c i $60
ii
d 2.7%
3 a 18
7
8

5 0.0036
6 a 32
81

b 3 13
7 a

Exercise 11A The binomial distribution


1 b, d and f constitute a Bernoulli trial.
2 a 0.2613 b 0.0446 c 0.2461
d 0.0092 e 0.0073 f 0.1969
3 a 5
b 0.3
c

0.1
0.0

0 1 2 3 4 5 6 7 8 910 x

ii Negatively skewed

Answers

723

29 a i Positively skewed
ii Symmetrical or normally distributed
iii Negatively skewed
b Controls skewness
30 a Symmetrical or normally distributed
b The curve would resemble more of a bell shape that is
tall and narrow.
31 a Positively skewed
b Negatively skewed
32 a Symmetrical or normally distributed
b Positively skewed
c Symmetrical or normally distributed
d Negatively skewed
Exercise 11B Problems involving the binomial
distribution for multiple probabilities
1 a 0.1792
b 0.6826
2 a 0.23
b 0.85
c 0.92
d 0.45
3 a 0.4718
b 0.9692
4 a 0.3370 b 0.3438 c 0.0333
31
6
5 a 32

b 163
c 31
6 a

1
9

0.65
Thursday
0.35

Poor

724

Answers

0.30

Best Match

b 0.25
b 3500 (train), 7000 (bus)
b 3933 (Pete), 1967 (Sam)
b 35
b 0.652
b 0.3014

c 0.652

Exercise 11D Expected value, variance and


standard deviation of the binomial distribution
1 a 6
b 1.6
b 1.35

c
b
b
b

0.0178
0.0464
0.0170
0.9967

c 6

d 3 43

b
b
16
19

0.1045
0.0930
A
D

3 a
c
4 a
5 a
6 a
7 a
8 a
9 a
10 a
11 a
12 a

1.26
3.24
5
4.62
12
1.67
7.5
5.4
28
96
1
2

b 2.74
d 4.16
c 1.58
c 1.88
c 2.19
b 0.5155
b 0.5
b 0.4613
b 0.4488
b 24
b 20

13 a

3
4

b
b
d
b

0.1878
0.9216
0.5327
0.2276

b 0.4256
b
b
b
b

14 a 13
15 a

0.0973
0.2741
0.0760
0.9573

1
5

6 C
1
19 E

0.65

Good

0.35

Poor

0.55

Good

0.45

Poor
0.65

0.65

Good

0.35

Poor

0.55

Good

0.45

Poor 0.55 0.35

0.55
0.45

0.45

2 a 0.6525
b 0.6652

Kingfisher

2 a 4.8

Poor

Good

Best Match

0.70

d 37 12

Good

b 0.6115

b 0.777
4 a 0.64
5 0.49
6 0.525
7 a 3500
8 a 3661
9 a 265
10 C
11 a 25
12 a 0.1479

Kingfisher

0.20

c 50

Thursday
0.35

Best Match

0.80

9
65

22 a

Exercise 11C Markov chains and transition


matrices
1 a
Saturday
Friday
0.65

Kingfisher

48
81

7 0.028
8 a 0.0878 b 0.9822
9 a 0.3669
10 a 0.9830
11 a 0.5000
c 0.9645
12 a 0.2553
13 a 0.1751
14 A
15 D
17 D
18 C
20 0.1509
21 a 0.9392
22 a 0.9106
c 0.6767
23 a 0.0173
c 0.8960
24 a 0.5981
c 0.2486
25 a 0.6778
26 a 0.8516
27 a 0.3438
28 a 0.9672

3 a 0.3

0.35
0.65

Good
Poor
Good
Poor
Good
Poor
Good
Poor

b 2.5
b 3.55
b 4.8

b 16
b 27
b 15
17 E
20 D

4
5

b 3.2

c 0.1450
c 0.0001

d 0.3980

18 C
21 D
c 1.79

3 a 0.0020
2
b 0.7939
24 0.2642
25 a 0.2335
b 0.3828
26 400
27 a 0.5000 b 0.6535 c 0.3465
28 a 5000
b 2500
c 50
29 a 6
b The company has an extremely popular product.
30 a 0.5455 b 0.3340 c 0.6
d 0.8795 e 0.2769
31 a 160 students
b SD(X) = 12, 2 = 136, + 2 = 184
c There is a probability of 0.95 that between 136 to
184 students (inclusive) will have a reading level that
is inadequate to cope with high school.
32 a 1260 patients
b SD(X) = 19.44, 2 = 1221.12, + 2 = 1298.88
c There is a probability of 0.95 that between 1222 to
1298 patients (inclusive) will be cured.

Chapter review
Short answer
1 a 165
2 a 0.243

31
b 32

b 0.001

1
c 32
c 0.081

3 a

27
128

54
255

4 a

256
625

1
9

5 a 4C1 p1 (1 p)3
6

e Tellya has 45.4% of the market, Yodacall has 54.6%


of the market.
f Tellya has 41.2% of the market, Yodacall has 58.8%
of the market.
g Tellya has 40% of the market, Yodacall has 60% of the
market. Both companies will be viable, but in the long
term Tellyas market share will drop by half, while
Yodacalls share will increase (in fact, will triple).

b p =

1
4

CHAPTER 12

5
16

7 a 0.14
8 a 0.45
9 0.234
10 a i 50
b i 40

b 0.23
b 0.2

11 a p = 15

b n = 50

12 a 8

b 6.4

Multiple choice
1 C 2 B 3
5 B 6 C 7
9 A
10 D
11
13 B
14 B
15
17 E
18 B
19

B 4
C 8
E
12
B
16
A

Continuous distributions
Exercise 12A Continuous random variables
y
1 a

A pdf

ii 25
ii 8

( 1 , 2)
2

(0, 2)

D
A
D
D

Not a pdf

( 1 , 0)
2

(0, 2)

Not a pdf

( 1, 2)
1
1

(1, 0)

d
( 1, 32 )
1

A pdf
(1, 32 )

3
2

A pdf

P
4

Not a pdf

P , 2)
( 3
2

2 P

P P
3
2

Answers 11B 12a

Extended response
1 a 135
b 0.1
c $60
2 a 1
b 0.0015 c 0.2639 d 10
3 a 1
b 0.2642 c 0.0003
4 a i 0.0018
ii 0.8826
iii 0.2761
b 3
5 a 0.0719 b 0.8432 c 0.3378
6 a 1440
b = 24, - 2 = 1392, + 2 = 1488
c This means that there is a probability of about 0.95
that between 1392 and 1488 of the 2400 people
selected will be cured by the drug.
7 X ~ Bi(80, 15 )
a 0.0164
b 0.9713
c 0.7550
8 a X ~ Bi(8, 0.8)
i 0.1678
ii 0.2936
iii 0.214
b i 0.2951
ii 0.2849
9 a 0.018
b 0.8336
c 0.3600
d 0.36
10 a In the first week of August, 229 customers will visit
Kazs Kitchen and 171 people will visit Als fine
dining.
b In the long run, 229 people will dine at Kazs Kitchen
each week and Al will get 171 customers each week.
11 a Let ti = the number of Tellya customers at the end of
January month i.
Let yi = the number of Yodacall customers at the end
of January month i.
ti + 1 = 0.85ti + 0.1yi and yi + 1 = 0.15ti + 0.9yi
b Tellya has 400 customers or 80% of the market;
Yodacall has 100 customers or 20% of the market.
c Tellya has 350 customers or 70% of the market,
Yodacall has 150 customers or 30% of the market.
d Tellya has 52.6% of the market, Yodacall has 47.4%
of the market.

1
2

P
2

A pdf

1
1
e
0

(e, 1e)
1

Answers

725

(2, 2)

(1, 0) 2

c 1
e 1

3
38

6
53

2
5

2
13

6
103

7
2

4
15

( P
, 0)
2

11 a

b
c


12 a

3
2

1
,axb

10 a f(x) = b a
0, elsewhere

b p = 5

0.5
a
a 0

a=2
i 0.25
ii 0.75
iii 49
y
(6,

b i 18
13 a

1
4

(0, 1 )
8

(4, 0)

b Teacher to check
c i 0.5475 ii 0.1179
iv 0.9583 v 0.5182
y
14 a

iii 0.8118
vi 0.3583

(0, 1)
x

c i 0.3935 ii 0.7788
y
6 a

0
P
(
, 0)
4

iii 0.4378

(1, 1)

c i 43
15 a

27
32

( 10 2 , 0)
3

10

ii

3
4

i 0.1563
ii 0.1040
iii 0.7398
iv 0.8767
0.5371
0.6916

1
2

(0, 1 )
4

ii

( P , 0)
4

2 3
2 3
iii
2
4

1
2

1
2

c i 0.5 ii 23 iii 16 iv 13 v 13
7 a = 1.4 8 b = 3

726

( P , 0)
2

1
4

Exercise 12B Using a probability density


function to find probabilities of continuous random
variables
1 a 0.5625
b 0.543 125
c 0.6696
2 a 0.9630
b 0.2963
c 0.2593
d 0.2693
e 0.2693

9 a 3 7 , or 0.3542

c i

ii

6 19
, or 0.8206
2

2 2

2

iii

3 1

2

iv

(0, 1 )
2

1
2

9 k =

b



4 a
b
5 a

f (Q )

a
2

7 3 11
8 k =

3 a

6 22
, or 0.6548
2

1
18

1
d
e 1

c
10 a

1
2

2 a

Not a pdf

Answers

3
2
2 2
2

( P , 1 )
2

16 a

8 a 1
b

y
(0,1)

a1
a2

a3

ii 0.5

2
3

iii

y
(0,

(P
, 0)
4

c Mean = median = mode = 0


d The distribution is symmetrical.

b i 0.75
17 a

( P , 0) 0
4

1
)
2P

9 a

1
4

b 4 8 2

e 4 12 4 4

12 2

10 a

42

y
P
2

e x ,xR

ii 0.9214
iv 0.6987

b 0.5

d i 0.2398
iii 0.6746

Exercise 12C Measures of central tendency and


spread
1 a Mean 1.75, median 1.75, no mode
b Mean

2
3,

median 2 2, mode 0

b Mean 3.5, median 2 3 4 , mode 4


c Mean 1.7, median 1.746, mode 2

b Variance

11
36 ,

c Variance

11
225 ,

2
3

standard deviation
standard deviation

1
4 a Variance 18
, standard deviation

11 m = 127 n =

11
6
11
15

9
32

a 0.7769
b 0.1733
a i 0.5132
ii 0.2413
b 1111 g
c 0.6321
a i 0.0952
ii 0.1647
b 0.9512
c 105.36 h
a Between 30 and 40 minutes
b i 0.3
ii 0.2
c 35 minutes
d 31 minutes
a i 0.9682
ii 0.9873
b 0.0045
c 0.0032
a 0.1110
b 0.2815
a
y

2
6

d
b 2.3863

d 0.5

d 0.9720

1
100

c 0.6623

d 0.68

Exercise 12E The normal distribution


1 a
b

y
( P , 1 )
2 2

c Mean =

c 0.2796

c 0.1790

10 (20, 0) x

100
9

iii 0.8187

P
2

(P , 0)

8 10 12 14 16

5 10 15 20 25 30 35

c
x

, variance = 0.4674
2
e 0.4142

3 2 1

2 a i 15
b i 1 23

ii 15
ii 1.66

c i and ii

d i and iii

iii 45
iii 5

Answers

Answers 12B 12E

c 0
e 103
a 2
a 0.5
b

c 0.0111
iii 0.1653

(10, 10k)

10k

b Variance 0.0742, standard deviation 0.2725


c Variance 0.4674, standard deviation 0.6837
5 a 103
b 103 loge (2)


6
7

6
7

Exercise 12D Applications to problem solving


1 a 2 hours b = 1.29 hours c 0.4375 d 0.2857
ii 43
iii 0.617
2 a i 18
3
4

5

6


7

8
9

2 a Mean 2.37, median 6, mode 11

b = 0.5
c As the graph is symmetrical, the median is 0.5.
d The mode is equal to the maximum of the graph. In
this case we have two points where the maximum is
the same. Therefore the mode = 0, 1.

c Mean 0.75, median 3 12 , mode 1

3 a Variance 43 , standard deviation

1
2

727

3 a

i
ii

iii

0 10 20 30 40 50 60 70 80 90100110120

b It widens the graph and increases the range.


c It moves the graph to the right.
4 a
b

3 2 1

5 10 15 20 25 30 35

20 30 40 50 60 70 80

5


6

a
b
c
a
b

i
The mean increases as time goes on.
The standard deviation also increases as time goes on.
= 65, = 5
(65, 1 )

4

5

6

a
d
a
d
a

b
e
b
e
b

0.9235
0.7462
0.6554
0.3260
0.75

7 a 0.46
8 0.14
9 0.72

8
31

10 a m =

b m = 1.75

Probability

50 55 60 65 70 75 80 x

7 a Mean: translation of 20 units in the positive


x direction; standard deviation: dilation factor of 12
from the x-axis, dilation factor of 2 from the y-axis
b Mean: translation of 9 units in the positive x direction;
standard deviation: dilation factor of 2 from the
x-axis, dilation factor of 12 from the y-axis
8 a 68%
b 95%
c 99.7%
d 16%
e 2.5%
f 0.15%
9 a 16%
b 2.5%
c 0.15%
10 a 68%
b 95%
c 99.7%
d 16%
e 16%
f 0.15%
11 a 28 X 52
b 16 X 64 c 4 X 76
12 a 25.8 X 28.6
b 24.4 X 30
c 23 X 31.4
d 28.6
e 30
f 31.4
13 a 16 X 17.2
b 15.4 X 17.8
c 14.8 X 18.4
d 16
e 15.4
f 14.8
14 a
1
(102.3,
21.4 2P

M
3S MS
2 MS

M
MS
MS
2 MS
3
38.1 59.5 80.9 102.3 123.7 145.1 166.5

b i 34% ii 47.5% iii 49.85% iv 81.5%


15 a 81.5%
b 2.35%
16 a 2.4 kg X 3.6 kg b 1.8 kg X 4.2 kg
c 1.2 kg X 4.8 kg
17 B
18 D
19 C
20 C
21 B
22 B
23 a 10.59 X 13.41
b 9.17 X 14.83
c 7.76 X 16.24
Exercise 12F The standard normal distribution
1 a 0.8413
b 0.0668
c 0.0401
d 0.9966
e 0.2721
f 0.8644
5
3

2 a 3

3 a 13

b 0.5

Answers

3
2

d
2 a
2
23 a
24 a
b
25 a
b

c 0.0082
f 0.5486
c 0.25

0.0082

0.3364

c 0.9022
c 0.6826
16 E

1.80

2.00

0.6006

0.0548

$1.74
0.9804
b 980
0.1056
b 17
i 0.7309 ii 0.6218 iii 0.0914
0.1679
i 0.1870 ii 0.9665
0.6449

Exercise 12G The inverse cumulative normal


distribution
1 a 1.282
b 0.842
c 0.100
d 0.126
2 a 1
b 0.675
c 0.253
d 0.496
3 a 0.675
b 0.253
c 0.583
d 2.576
4 a 11.166
b 9.493
c 9.336
d 11.683
5 a 35.984
b 36.698
c 30.774
d 33.497
6 a 5.182
b 1.527
c 17.525
7 E 8 A 9 C 10 B 11 A 12 D
13 B
14 a 173.16 cm
b 153.27 cm
15 a 44.58 mm
b 45.42 mm
16 48.65 seconds
17 1.907
18 7.896
19 11.704 grams
20 26.305
21 25.844
22 3 minutes 44.27 seconds
23 a > 1.7 kg
b Between 1.3 kg and 1.7 kg
24 a 64
b 44
Chapter review
Short answer
1 a
y

b Yes it could.

(0, 4)
( 1 , 0)
2

c 1.2
c 1.133

c 0.7623

1 a 0.0863
1
b 0.0115
12 a 0.1587
b 0.3874
d 0.8302
e 0.2391
13 A
14 B
15 E
17 A
18 A
19 E
20 0.8676
21 a 0.6006
b 0.0548
c
Cost ($)
1.40
1.60

5 2P

728

0.3874
0.0317
0.0808
0.0501
0.75

2 a

A = 163

4


5


6


7

(4, 2)

3
16

3 a

1
4

4 a

3
8

c m =

( 132 ) 3

1
3

7
8

c 2

d 2

1
8

8
3

5 a

6

7

c 8
a 0.25
c 5 min 48 s
a
5

c 0.191
e 88minutes

b
1

9 10 11

9 17 25 33 41 49

6 14 22 30 38 46 54

Exam practice 4
Short answer
1 34%
1
2 a 15

4
45

c 1

1
8

3 a 74%

b 37

28

0.75 0.20
4 a T =

0.30 0.60

8 a

(1.42, 1.54)

d 0.41

9 6 3

8 10 12 14

b Initial proportion of coffee purchasers 0 . 9


S=
=
In itial proportion of biscotti purchasers 0 . 4
c 0.755
4

0 . 75 0 . 20
0.9
d

0 . 30 0 . 60
0.4

70 80 90 100110120130

9 Mean: translation of 22 units in the positive x direction;


standard deviation: dilation factor of 13 from the x-axis,
dilation factor of 3 from the y-axis
10 a 68%
b 95%
c 99.7%
11 a 16.5 X 22.7
b 13.4 X 25.8
c 10.3 X 28.9
d 22.7
e 25.8
f 28.9
2
3

0.28
0.37
m = 2
0.16

b 0.5
b 0.28
b 0.40
b m = 1.5
b 0.34

c 0.51

y
1

2.5 2

c 0.32

3
7
11
15
19

Extended response
1 a i 0.4066
b 0.8946
2 a 1.82 cm
c 5%
3 a 163.2 seconds
c 0.7357

ii 0.1653
iii 0.0672
c 6:26 pm
d $1.06
b 5%
d 0.9474
b 0.0096
d 137.5 seconds

4
8
12
16
20

Extended response
a i f(-2) = 0.054
ii f(1) = 0.242

0.5

( 2, 0.054)

Multiple choice
1 A
2 D
5 C
6 A
9 E
10 D
13 D
14 D
17 D
18 C

C
B
A
C
C

Multiple choice
1 E 2 B 3 C
4 B 5 D

C
D
E
A
C

5 1 .5

(1, 0.242)
0.5

1.5

2.5 x

.5

c 0.3401
d Pr(-1 < X < 1) 0.7194 (left rectangular) 0.6410 (right
rectangular)
e 0.683
y
1

(0, 0.3989)

( 1, 0.2421)
( 2, 0.1330)
2.5 2

0.5

5 1 .5

(1, 0.2421)
0.5

1.5

(2, 0.1330)
2

2.5 x

.5

Answers 12F 12g

2 a
1
13 a
14 a
15 a
16 a

0.0062
0.8413
0.1535
2.34 cm
0.1963
26.77 cm
0.0625
3.4
3hours, 31minutes
11.9
b

d 4
b 0.325

a
b
c
a
b
c
a
b
c
a

h 0.6827

Answers

729

Index
absolute value function 925, 21215
graphing 3045
hybrid functions, as 945
addition of ordinates 20610, 221
addition rule of probability 4889, 498, 529
angles
complementary 26970
measurement of 2589
negative 267
antidifferentiation 41623, 474 see also integration
areas
approximation methods 4358, 475
between graph and x-axis 4358
between two curves 4604, 475
bound by a curve and the x-axis 4556
enclosed by functions, approximating 4358
finding without sketch graphs 4568
further 4558
signed 448
average rates 325
Bernoulli trials and sequences 53746
binomial distribution 53748, 581
expected value, variance and standard deviation
of 5737, 582
graphs of 5467, 581
multiple probabilities, problems involving 5526, 581
binomial expansions 1
binomial theorem 15, 47
chain rule 3413, 366
change-of-base rule 138
change, rates of 3968, 408
combinations 4967, 498, 529
complementary angles 26970
completing the square 23
composite functions 10810, 122
graphing 3079
conditional probability 4923, 498, 529
constant rates 324
continuous distributions 5913
continuous random variables 58993, 643
applications to problem solving 61316, 644
normal distribution 61923
probability density functions and 595601
standard normal distribution 62633
cosine 261
graphs 2828, 315
special cases 262
cubic function
gradient function of 327
power form, in 659
cubic graphs 2934, 48
curves, sketching 37684
definite integrals 4426, 474

730

Index

degrees
converting radians to 25960
converting to radians 2601
finding number of degrees in one radian 259
derivatives
ex 344, 366
loge (x) 3479, 366
sin (x), cos (x) and tan (x) 3514, 3667
xn 3389, 366
difference functions 1034, 122
graphing 3034
differentiation 325
chain rule 3413, 366
first principles, using 3346, 366
mixed problems on 3604
product rule 3557, 367
quotient rule 3579, 367
dilation 578, 60, 65, 71, 79, 87, 978, 1735, 1835,
283, 291
discrete random distributions
measures of centre of 51016, 52930
measures of variability 51826, 530
discrete random variables 5016, 529
expectation theorems 51214
discriminant 22
division of polynomials 1114
domain and range of functions 1820
restricted 18, 33
equations
absolute value 925, 121
base e 1513, 167
composite 10810, 122
exponential 1426, 1513, 167
finding, for graphs of exponential and logarithmic
functions 2024
functional 10810, 122
literal 15961, 167
logarithmic 1479
natural (base e) logarithms, with 1545, 167
tangents and normals 3735, 408
trigonometric 27381, 2957, 315
Eulers number 151
events 484, 497
independent 489
mutually exclusive 489
exact values 264
expectation theorems 51214
exponential and logarithmic modelling 1624
graphs, using 21617
exponential equations 1426, 167
base e 1513
exponential functions 220
absolute values, with 21215
ex, integration of 4267
graphs of, with base e 191

finding areas without sketch graphs 4568


functional equations 10810, 122
functions 49
absolute value 925, 121, 21215, 3045
average value of 4668
composite 10810, 122, 3079
continuous 325
cubic 2931, 659, 327
difference 1034, 122
domain and range 1820
exponential 172, 191, 21215, 220, 4267
gradient 3257
graphing 3079
hybrid 945
inverse 2403, 252
limits 3326, 366
logarithmic 21215, 2201
polynomial, characteristics of 37684
power 715, 7883
probability density functions 595601
product 1056, 3067
quadratic 22, 5960, 3267
range of 1820
restricting 2459, 252
smooth 325
square root function in power form 8690
sum 103, 122, 3034
tangent, graphs of 2915
their inverses and 2349, 252
trigonometric 31112, 316, 4278
fundamental theorem of integral calculus 4416

sine and cosine functions, of 2828, 315


stationary points 3778, 408
straight line 1517
sum function 3034
tangent function, of 2915, 31516
transformations 5762
trigonometric 2828, 315
truncus 7883
types of 11117
hybrid functions 945
hyperbola 715
index laws 1326, 167
instantaneous rates 325
integral calculus, fundamental theorem of 4416
integrals, properties of 41721
definite 4426, 474
integration
ex, sin (x) and cos (x) 4268
further applications of 46871
recognition, by 4303
interquartile range 61011
intersection 485, 529
interval notation 18
inverse cumulative normal distribution 63640
inverse functions 24043, 252
inverses 1568, 167
functions and 2349, 252
relations, of 22933
Karnaugh maps and probability tables 4912, 498, 529

gradient
rates of change and 3248, 366
gradient function
cubic functions, of 327
graphing from the graph of a function 325
quadratic functions, of 3267
straight lines, of 326
graphs 221, 3039, 316
absolute value 21215, 221, 3045
addition of ordinates 20610, 221
antiderivative function and original function,
relationship 4213, 474
binomial distribution 5467, 581
composite functions 3079
cosine 2828, 315
cubic 2934, 48
exponential and logarithmic modelling, using 21617
exponential functions with any base 17280
exponential functions with base e 1916
hyperbola 715
inverse, of 22933
linear 1520, 478
logarithmic, to any base 18290
logarithmic, to base e 197200
maximum and minimum problems, solving 38694, 408
polynomial functions, characteristics of 37684
power functions, of 120
product functions 3067
properties of straight line 1517
quadratic 227, 48
quartic 3844, 489

linear approximation 4046


linear graphs 1520, 478
literal equations 15961, 167
logarithm laws 13841, 167
logarithmic equations using any base 1479
logarithmic functions with absolute values 21215, 221
logarithmic graphs to any base 18290, 2201
Markov chains 55970, 581
transition matrices 56570, 581
mathematical model 111
matrices
transformations, with 96101, 1212
transition 56570, 581
using to describe a reflection in the line y=x 22930
maximum and minimum problems
where function known 3869, 408
where function unknown 3904, 408
mean 5737, 582, 605
measures of central tendency and spread 60511, 6434
median 51516, 605
mode 51516, 605
modelling 11117, 122
exponential and logarithmic 1624
trigonometric 299301
modulus functions, graphing 3045 see also absolute value
function
negative angles 267
negative quartics 39

Index

731

normal distribution 61923, 6445


common probabilities associated with 61920
graph 619
inverse cumulative normal distribution 63640, 645
properties 619
standard 62633, 645
symmetry properties 6303
ordinates, addition of 20610, 221
parabola and transformations 5762
Pascals triangle 2, 5, 47
percentiles 601, 638
polynomials 610, 47
division of 1114
evaluating 810
positive quartics 38
power functions
hyperbola 715
truncus 7883
probability 48598
addition rule of 4889, 529
combinations 4967, 498, 529
conditional 4923, 498, 529
independent events 489
Karnaugh maps and probability tables 4912,
498, 529
mutually exclusive events 489
outcomes 484, 497, 529
terminology 4845, 497, 529
tree diagrams 4946, 498, 529
Venn diagrams 485, 497, 529
probability density function 5903, 643
using to find probabilities of continuous random
variables 595601, 643
product functions 1056
graphing 3067
product rule 3557, 367
quadratic function in power form 5960
quadratic graphs 227, 48
quantiles 601, 638
quartic graphs 3844, 489
negative 39
positive 38
quotient rule 3579, 367
radians 25863, 314
converting degrees to 2601
converting to degrees 25960
finding number of degrees in one 259
random variables, discrete 5016
discrete probability distributions 5026
rates of change 3968, 408
rates, related 4013, 409

732

Index

reflection 58, 60, 65, 712, 79, 87, 978, 17880, 18790,
2856, 291
related rates 4013, 409
relations and their inverses 22933
restricting function 2459, 252
sample point 484, 497, 529
sample space 484, 497, 529
sigma notation 445
signed areas 44851
sine 261
graphs 2828, 315
special cases 262
sketching curves 37684
square root function in power form 8690
standard deviation 5213, 5737, 582, 607
interpreting 5236
stationary points 3778, 408
types of 3778
straight line graphs, properties of 1517
sum functions 103, 122
graphing 3034
symmetry
exact values and 26471, 314
tangent 262
graphs 2915, 31516
special cases 262
tangents and normals, equations of 3735, 408
transformations 5762
combination of 59, 61, 66, 72, 88
matrices, with 96101
transition matrices 56570, 581
translation 59, 989, 2867, 292
horizontal 60, 66, 72, 7980, 87, 1767, 1867
vertical 60, 66, 72, 80, 87, 176, 1856
tree diagrams 4946, 498, 529
trigonometric equations 27381, 2957, 315
trigonometric functions, integration of 4278
trigonometric functions with an increasing trend 31112,
316
trigonometric graphs 2828, 315
finding equations of 2957
trigonometric modelling 299301
truncus 7883
turning point form 23
union (probability) 484, 497, 529
unit circle 25863, 314
symmetry properties 2645
variability of discrete random distributions, measures
of 51826, 52930
variance 51821, 5737, 582, 607
Venn diagrams 485, 497, 529

You might also like